You are on page 1of 620

www.crackjee.

xyz
ISBN: 978-81-265-8122-1 (ebk)
Note to the Students
The IIT-JEE is one of the hardest exams to crack for students, for a very simple
reason – concepts cannot be learned by rote, they have to be absorbed, and IIT
believes in strong concepts. Each question in the IIT-JEE entrance exam is meant
to push the analytical ability of the student to its limit. That is why the questions
are called brainteasers!
Students find Mathematics the most difficult part of IIT-JEE. We understand that
it is difficult to get students to love mathematics, but one can get students to love
succeeding at mathematics. In order to accomplish this goal, the book has been
written in clear, concise, and inviting writing style. It can be used as a self-study
text as theory is well supplemented with examples and solved examples. Wher-
ever required, figures have been provided for clear understanding.
If you take full advantage of the unique features and elements of this textbook,
we believe that your experience will be fulfilling and enjoyable. Let’s walk
through some of the special book features that will help you in your efforts to
crack IIT-JEE.
To crack mathematics paper for IIT-JEE the five things to remember are:
1. Understanding the concepts
2. Proper applications of concepts
3. Practice
4. Speed
5. Accuracy

About the Cover Picture


Medieval mathematician and businessman Fibonacci (Leonardo Pisano) posed
the following problem in his treatise Liber Abaci (pub. 1202):
How many pairs of rabbits will be produced in a year, beginning with a single pair,
if in every month each pair bears a new pair which becomes productive from the
second month on?
The solution to this problem leads to the recursive sequence, obeying the simple
rule that to calculate the next term one simply sums the preceding two.The Fibonacci
numbers are recursively defined by Fn+1 = Fn + Fn–1, where F0 = 0, F1 = 1. The first
few are
0, 1, 1, 2, 3, 5, 8, 13, 21, 34, 55, 89, 144, 233, 377, …
The image on the cover is that of a Fibonacci spiral.
A. PEDAGOGY

CHAPTER OPENER
Trigonometric Ratios
and Transformations 1 Each chapter starts with an opening vignette, defini-
tion of the topic, and contents of the chapter that give
you an overview of the chapter to help you see the
big picture.
Y
y = sin x
Contents
1.1 Angles and Their
1
Measures
X X
p p/2 O p/2 p 3p/2 2p 5p/2 3p
1.2 Trigonometric Ratios
Y y = cos x
–1
1.3 Periodicity and
1

Y
Variance
X
p/2 O p/2 p 3p/2 2p 5p/2
X 1.4 Trigonometric Ratios
1 of Compound Angles
Y
1.5 Trigonometric Ratios
Y y = tan x of Multiple and
Trigonometric Ratios and

Submultiple Angles
1.6 Sum and Product
Transformations
X X
p/2 O p/2 p 3p/2

Y
y = cot x Worked-Out Problems
Summary
Exercises
Answers
X X
p/2 0 p/2 p 3p/2 2p 5p/2
Transformations

Y y = sec x Trigonometric Ratios: The


relationships between the
2 angles and the sides of a
1
right triangle are expressed
X
p/2 0 p/2 p 3p/2
X
in terms of trigonometric
1

2
Y y = cosec x
ratios.

X X
O p/2 p 3p/2 2p
1

CLEAR, CONCISE, AND INVITING WRITING


Special attention has been paid to present D E F I N I T I O N 7. 1 Random Experiment A random experiment is an experiment in which
1. The experiment can be repeated any number of times under identical conditions.
an engaging, clear, precise narrative in the 2. All possible outcomes of the experiment are known in advance.
layout that is easy to use and designed to 3. The actual outcome in a particular experiment is not known in advance.

reduce math anxiety students may have. Examples

(1) Rolling of an Unbiased Die This experiment can be performed any number of times
Rolling of an unbiased die is a random experiment in under identical conditions.
which all the possible outcomes are 1, 2, 3, 4, 5 and 6, if (2) Tossing of an Unbiased Coin
we denote the six faces of the die with the numbers 1, 2, Tossing of an unbiased coin is a random experiment in
3, 4, 5 and 6. The faces of the die may also contain dots which there are only two possible outcomes, namely, Head
in numbers 1, 2, 3, 4, 5 and 6. In any case, we identify (H) and Tail (T). In a particular experiment (tossing of the
DEFINITIONS the faces of a die with the numbers 1, 2, 3, 4, 5 and 6.
The actual outcome in a particular experiment (rolling
coin), the outcome is not known in advance.This experiment
can also be performed any number of times under iden-
of the die) is the number that appears on the upper- tical conditions and therefore this is a random experiment.
most face of the die and this is not known in advance.
Every new topic or concept starts with de-
fining the concept for students. Related ex- “Tossing of an unbiased coin till tail appears” is also a random experiment. However, experiments such as
“measuring the acceleration due to gravity using a compound pendulum” is not a random experiment, since the
amples to aid the understanding follow the experiment cannot be repeated under identical conditions and the possible outcomes are not known in advance.
Throughout this chapter, by a coin or die we always mean an unbiased coin (or a fair coin) or unbiased die (or a
fair die) unless otherwise mentioned.
definition.
D E F I N I T I O N 7. 2 Some of the important definitions are as follows:
1. A set of possible outcomes of an experiment is called an event.
2. Two or more events are said to be exhaustive if the performance of the experiment always
results in the occurance of atleast one of them.
3. Two or more events are said to be mutually exclusive if the occurance of one of the events
prevents the occurance of any of the remaining events.
4. Two or more events are said to be equally likely (or equiprobable) if there is no reason to
expect one of them in preference to others.
Example 7.3 EXAMPLES
Suppose that an integer is picked from among 1 to 20 2, 3, 5, 7, 11, 13, 17 and 19 and these are 8 in number.
(both inclusive). What is the probability of picking a Therefore, 8 are favourable to the event of picking a Examples pose a specific problem
prime? prime and hence the probability of picking a prime is
8 2 using concepts already presented
Solution: There are 20 outcomes of the experiment =
20 5
of picking an integer. The primes between 1 and 20 are and then work through the solution.
Example 7.4 These serve to enhance the students'
An outcome that none of the dice shows 3 can be understanding of the subject matter.
none of the dice shows 3 (on the upper most face). expressed as 8-tuple of integers from the 5-element set
{1, 2, 4, 5, 6} and there are 58 such outcomes. Thus, the
Solution: Any outcome of “throwing 8 dice” can be probability that none of the dice shows 3 is
expressed as an 8-tuple of integers from 1 to 6 and hence 8
the total number of possible outcomes is 68. 58 æ 5 ö
=ç ÷
68 è 6 ø

Example 7.5

Suppose that a bag contains 6 red, 5 black and 4 blue Drawing one blue, one black and one red ball simulta-
balls. Find the probability that three balls drawn simul- neously can be expressed as a triple (a1, a2, a3), where
taneously are one blue, one black and one red. 1 £ a1 £ 4, 1 £ a2 £ 5 and 1 £ a3 £ 6. The number of such
tuples is
Solution: The total number of balls is
4 ´ 5 ´ 6 = 120
6 + 5 + 4 = 15
Thus, the probability that 3 balls drawn simultaneously
Out of these 15 balls, 3 balls can be drawn in 15C3 ways. are one blue, one black and one red is
Therefore, 3 balls can be drawn simultaneously in
120 24
=
15× 14 × 13 455 91
= 455 ways
1× 2 × 3

THEOREMS DE F I N I TI O N 1 . 8 Two angles are said to be complementary if their sum is a right


angle. Therefore, for 0 £ q < p/2,
p q and p/2
p - q are complementary.

Relevant theorems are provided along THEOREM 1.6 Let q and y be complementary angles. Then
1. sin q = cos y and cos q = sin y
with proofs to emphasize conceptual un- 2. tan q = cot y and cot q = tan y
3. sec q = cosec y and cosec q = sec y
derstanding rather than rote learning.  = 90° and AOP  = q (Figure 1.12). Since the sum of
PROOF Draw a right-angled triangle OAP with OAP
 =y.
the angles of any triangle is 180° and since q + y = 90°, it follows that OPA
P

q 90°
O

FIGURE 1.12 Theorem 1.6.

QUICK LOOK 4 QUICK LOOK


Unlike the trigonometric functions sine and cosine, the and cot x are not defined at x = np,
p n Î, since sin x = 0
trigonometric functions secant, tangent, cosecant and at these x. The domains of all the trigonometric funct-
cotangent are not defined on the whole real line. The ions are collected in Table 1.1 for convenience and for Some important formulae and con-
functions sec x and tan x are not defined at x = (2n + 1) quick reference.
p n Î, since cos x = 0 at these x. Similarly cosec x
p/2, cepts that do not require exhaustive
explanation, but their mention is im-
portant, are presented in this section.
These are marked with a magnifying
glass.
TRY IT OUT Example 7.8

Let S = {1, 2, 3, 4, 5, 6} be the sample space of a random (2) Completeness axiom:


Within each chapter the stu- experiment. Define P(E) = å P( s) for any E £ S, where
sÎE P(S) = å P( s) = P(1) + P(2) + P(3) + P(4) + P(5) + P(6)
dents would find problems sÎS
P(1) = 0.1
to reinforce and check their = 0.1 + 0.2 + 0.2 + 0.4 + 0.05 + 0.05
P(2) = 0.2 = P(3)
understanding. This would =1
P(4) = 0.4
help build confidence as one (3) Union axiom: For any E1 and E2 ÎÃ(S) with
P(5) = 0.05 = P(6) E1 Ç E2 = f, we have
progresses in the chapter.
P is a probability function. P(E1 È E2) = å P( s) = å P( s) + å P( s) = P(E1) + P(E2)
These are marked with a sÎE1 È E2 sÎE1 sÎE2

Solution: We have
pointed finger. since s ÎE1 È E2 if and only if s ÎE1 or s ÎE2 but not
(1) Positive axiom: both.
Thus P is a probability function.
P ( E ) = å P ( s) > 0
s ÎE

since P(s) > 0 for all s ÎS.

Try it out Let S = {H, T


T} be the sample space of a random experiment x and define
1
P( H ) = = P(T )
2
P(f ) = 0
and P (S ) = 1
Then verify that P is a probability function.

SUMMARY
SUMMARY
7.1 Random experiment: An experiment is called 7.8 Usual probability: The classical probability is also At the end of every
random experiment if the following conditions are called usual probability. chapter, a summary is
satisfied.
(1) The experiment can be repeated any number of 7.9 Sample points and sample space: Any possible presented that organ-
times under similar conditions. outcome of a random experiment is called a sample
izes the key formulae
point and the set of all sample points is called
(2) All possible outcomes of the experiment are
known in advance
the sample space of the random experiment. An and theorems in an
elementary event means a sample point. Generally
(3) The actual outcome in a particular experiment sample space is denoted by S.
easy to use layout. The
cannot be exactly predicted. related topics are indi-
7.10 Finite sample space: A set A is called finite if either
7.2 Sample space and event: The set of all possible A is an empty set or it is bijective with the set {1, cated so that one can
outcomes of a random experiment is called sample 2, 3, …, n} for some positive integer n. If a sample quickly summarize a
space of the experiment and any subset of the sample space is finite, then it is called a finite sample space.
space is called an event. chapter.
7.11 Countably infinite sample space: A set A is called
7.3 Exhaustive events: Two or more events are called countably infinite set if it is bijective with the set
exhaustive events if the performance of the experi- + of all positive integers. If the sample space of a
ment results in the occurrence of at least one of random experiment is countably infinite set, then
these events. the sample space is called countably infinite. For
example, tossing a fair coin till head appears has a
7.4 Mutually exclusive events: Two or more events are countably infinite sample space.
said to be mutually exclusive if the occurrence of one
of the events prevents the occurrence of any one of 7.12 Definition: Here afterwards events mean subsets
the other events. of the sample space. If A and B are two events, then
A È B means at least one of A or B and A Ç B
7.5 Equally likely events: Two or more events are said means both A and B. Impossible event is denoted
to be equally likely (or equiprobable) if there is no by empty set f and a certain event means the entire
reason to expect one of them in preference to the sample space.
others.
7.13 Various events in set theoretical form:
7.6 Probability (classical definition): Suppose in a
(1) Events E1, E2, …, En are said to be mutually
random experiment there are n exhaustive, mutually
exclusive if Ei Ç Ej = f for i ¹ j.
exclusive, equally likely outcomes. If m of them are
favourable to an event E, then the probability P(E) (2) E1, E2, …, En are called exhaustive events if E1 È
of E is defined as E2 È  È En = S (sample space).
B. WORKED-OUT PROBLEMS AND ASSESSMENT – AS PER IIT-J
T EE PATTERN
P
Mere theory is not enough. It is also important to practice and test what has been
proved theoretically. The worked-out problems and exercise at the end of each
chapter are in resonance with the IIT-JEE paper pattern. Keeping the IIT-JEE
pattern in mind, the worked-out problems and exercises have been divided into:
1. Single Correct Choice Type Questions
2. Multiple Correct Choice Type
T Questions
3. Matrix-Match Type Questions
4. Comprehension-Type Questions
5. Assertion–Reasoning Type
T Questions
6. Integer Answer Type Questions

WORKED-OUT PROBLEMS
In-depth solutions are provided to all worked-out problems for students to understand the logic behind and
formula used.

WORKED-OUT PROBLEMS
SINGLE CORRECT
Single Correct Choice Type Questions CHOICE TYPE
1. If (sin q + cosecq
c q)2 + (cos q + secc q)2 = tan2 q + cot2 q + k, Now QUESTIONS
then k equals
1 = cos2 q + sin2 q
(A) 9 (B) 7 (C) 5 (D) 8
= m4 n2 + m2 n4 These are the regular mul-
Solution:
= m2 n2 (m2 + n2 )
LHS = sin q + 2 + cosec q + cos q + 2 + sec q
2 2 2 2 tiple choice questions with
Answer: (D)
= 5 + (1 + cot2 q ) + (1 + tan2 q ) four choices provided. Only
4. If a sec q = 1 - b tan q and a sec q = 5 + b tan q, then
2 2 2 2
= tan2 q + cot2 q + 7 one among the four choices
a2b2 + 4a2 = kb2 where value of k is
Answer: (B) (A) 3 (B) 4 (C) 5 (D) 9
will be the correct answer.
Solution: (1 - b tan q)
q 2 = a2 sec2 q = 5 + b2 tan3 q
2. If sin q + sin q = 1, then cos q + 3cos q + 3cos q +
2 12 10 8
Therefore
cos6 q is equal to
(A) 0 (B) 2 (C) 1 (D) 4 1 - 2b tan q + b2 tan2 q = 5 + b2 tan2 q

Solution: Given that sin q + sin2 q = 1. This implies -2


tan q =
sin q = cos2 q. b

MULTIPLE Multiple Correct Choice Type Questions


CORRECT CHOICE 1. A = -7/25 and A lies between 450° and 540° then

= + 1 - cos2 22

A -4 A -3 sin 22
= =
TYPE QUESTIONS (A) sin
2 5
(B) cos
2 5
2 2

A -3 A -4 2+ 2 2- 2
(C) sin = (D) cos = = + 1- =
2 5 2 5 4 2
Multiple correct choice type
Answers: (A), (B), (C), (D)
questions have four choices Solution: By hypothesis 450° < A < 540°. This implies
225° < A/2 < 270°. So
3. Let cos x + cos y = a, cos 2x + cos 2y
2 = b and cos 3x +
provided, but one or more of cos 3y = c. Which of the following is (are) true?
A 1 - cos A 1 + (7 / 25) 4
the choices provided may be sin = = =
2 2 2 5 b
(A) cos2 x + cos2 y = 1 +
correct. 2
Now A/2 lies in the third quadrant. This means
1
(B) (2a2 - b - 2) = cos x cos y
A -4 4
sin =
2 5 (C) 2a3 + c = 3a(1 + b)
Again (D) abc = 0 for all real x and y

A 1 + cos A 1 - (7 / 25) 3 Solution:


cos = = =
2 2 2 5 cos2 x + cos2 y + 2 cos x cos y = a2 (1.11)
MATRIX-MATCH TYPE QUESTIONS
These questions are the
regular “Match the Follow- Matrix-Match Type Questions
ing” variety. Two columns 1. Match the items of Column I with those of Column II. q qö
2
æ
cos + sin ÷
1 + sin q çè 2 2ø
each containing 4 subdivi- sec q + tan q = =
Column I Column II cos q 2 q 2 q
cos - sin
sions or first column with 2 2
æ sin 2q ö æ cos q ö q
(A) ç is equal to (p) cot
four subdivisions and sec- è 1 + cos 2q ÷ø çè 1 + cos q ÷ø 2 cos
q
+ sin
q
1 + tan
q
= 2 2 = 2
ond column with more sub- æ sin 2q ö æ 1 - cos q ö q q q q
(B) ç equals (q) tan cos - sin 1 - tan
divisions are given and the è 1 - cos 2q ÷ø çè cos q ÷ø 2 2 2 2
Answer: (D) Æ (s)
student should match ele- cot q - 1 1 - sin 2q
(C) = (r)
ments of column I to that cot q + 1 cos 2q 2. Match the items of Column I with those of Column II.

of column II. There can be q


1 + tan Column I Column II
(D) sec q + tan q = (s) 2
one or more matches. q
1 - tan (A) The value of cos2q + cos2(60° + q)
q + (p)
3
2 2
cos2(60° - q
q) is
Solution: 1
(B) cos 20° cos 40° cos 80° (q)
(A) 8

æ sin 2q ö æ cos q ö æ 2 sin q cos2 q ö æ 1 ö 3


çè 1 + cos 2q ÷ø çè 1 + cos q ÷ø = çè 2 cos2 q ÷ø çè 2 cos2(q / 2) ÷ø (C) sin2 q + sin2 (120° + q ) + sin2 (120° - q ) (r)
equals 8
2 sin(q / 2)cos(q / 2)
= 3
2 cos2 (q / 2) (D) sin 20° sin 40° sin 80° is equal to (s)
8
q

COMPREHENSION-TYPE QUESTIONS
Comprehension-type questions consist
Comprehension-Type Questions
1. Passage: sin(A ± B) = sin A cos B ± cos A sin B and sin A + sin 2 A + sin 4 A + sin 5 A
of a small passage, followed by three
cos(A ± B) = cos A cos B ∓ sin A sin B. Based on this (ii) =
cos A + cos 2 A + cos 4 A + cos 5 A multiple choice questions. The ques-
information, answer the following questions.
(i) sin A + 2 sin 3A + sin 5A is equal to (A) cot 3A (B) tan 3A
(C) 2 cot 3A (D) 2 tan 3A
tions are of single correct answer type.
(A) 4 sin 3A cos2A (B) 4 cos 3 A sin2A
(C) 4 sin 3A sin2A (D) 4 cos 3 A cos2A cos 7 A + cos 3 A - cos 5 A - cos A
(iii) =
sin 7 A - sin 3 A - sin 5 A + sin A
2A (B) tan 2A
2 (ii) (cot B + cot C )(cot C + cot A)(cot A + cot B)
(C) 2 cot 2A
2 (D) 2 tan 2A
2 equals
Solution: From the given information, we have (A) sin A sin B sin C
(B) sec A sec B sec C
2 sin A cos B = sin( A + B) + sin( A - B)
(C) cosec A cosec B cosec C
2 cos A sin B = sin( A + B) - sin( A - B) (D) cos A cos B cos C
2 cos A cos B = cos( A + B) + cos( A - B) 1 + cos A - cos B + cos C
(iii) =
2 sin A sin B = cos( A - B) - cos( A + B) 1 + cos A + cos B - cos C
(i) sin A + 2 sin 3 A + sin 5 A = (sin A + sin 5 A) + 2 sin 3 A B C B C
(A) cot tan (B) tan cot
2 2 2 2
= 2 sin 3 A cos 2 A + 2 sin 3 A
A B C A
= 2 sin 3 A(cos 2 A + 1) (C) tan cot (D) tan cot
2 2 2 2
= 4 sin 3 A cos A
2

Solution:
Answer: (A)
(i) 1 - 2 sin B sin C cos A + cos2 A
(ii) Numerator = (sin A + sin 5 A) + (sin 2 A + sin 4 A)
= 1 + cos2 A - [cos( B - C ) - cos( B + C )]cos A
= 2 sin 3 A cos 2 A + 2 sin 3 A cos A
= 1 + cos2 A - cos( B - C )cos A + cos( B + C )cos A
= 2 sin 3 A(cos 2A + cos A)
Denominator = (cos A + cos 5 A) + (cos 2 A + cos 4 A) = 1 + cos2 A + cos( B - C )cos( B + C ) - cos2 A

= 2 cos 3 A cos 2 A + 2 cos 3 Acos A = 1 + cos2 B - sin2 C = cos2 B + cos2 C


Answer: (A)
= 2 cos 3 A(cos 2A + cos A) (ii) (cot B + cot C )(cot C + cot A)(cot A + cot B)
ASSERTION–REASONING TYPE QUESTIONS
These questions check the
Assertion–Reasoning Type Questions
analytical and reasoning In the following set of questions, a Statement I is given This implies that either
skills of the students. Two and a corresponding Statement II is given just below it.
a+b
Mark the correct answer as: sin =0
statements are provided – (A) Both Statements I and II are true and Statement II
2

Statement I and Statement is a correct explanation for Statement I. æa - bö a-b


cosç = - 3 sin
è 2 ÷ø
or
(B) Both Statements I and II are true but Statement II is 2
II. The student is expected not a correct explanation for Statement I.
Therefore
to verify if (a) both state- (C) Statement I is true and Statement II is false.
(D) Statement I is false and Statement II is true. æ a - b ö -1 æ -p ö
a + b = 2 np or tan ç = = tan ç
ments are true and if both è 2 ÷ø 3 è 6 ø÷
1. Statement I: If 0 < q < p
p/2 and cos 2q = 2(cos q - a-b p
are true, verify if statement sin q
q), then tan q is equal to 1. a + b = 2 np or = np -
2 6
I follows from statement Statement II: If 0 < a,
a b<p
p/2 and sin(a + b)
b = 1, then p
a + b = p/2.
p a + b = 2 np or a = b + 2 np -
II; (b) both statements are 3
Solution: Statement II is clear. Now
true and if both are true, Now a + b = 2np
p implies
cos 2q = 2 (cos q - sin q ) Þ cos2 q - sin2 q
verify if statement II is not a = sin(6np
sin 3a p - 3b)
b or sin 3a = sin(6np
p + 3b
b - p)
p
= 2 (cos q - sin q )
the correct reasoning for a = -sin 3b
In any case sin 3a b so that sin 3a + sin 3b
b = 0.
This is true when cos q = sin q (i.e., q = p
p/4) in which case Answer: (A)
statement I; (c), (d) which tan q = 1. Suppose cos q ¹ sin q. Therefore

cos q + sin q = 2 3. Statement I: sin(p /18) is a root of 8 x3 - 6 x + 1 = 0.


of the statements is untrue.
æp ö Statement II: For any real a , sin 3a = 3 sin a - 4 sin3 a .
sin ç + q ÷ = 1
è4 ø Solution: Statement II is a standard formula. Put a = p/18.
p
p p Therefore
+q =
4 2
3a = and hence sin 3a =

INTEGER-TYPE QUESTIONS
The questions in this section are nu-
Integer Answer Type Questions
merical problems for which no choices
1. In D ABC, sin(B + C - A) + sin(C
C + A - B) + sin(A + A B C
= 1 - 2 cos cos sin
B - C) = k sin A sin B sin C where k equals . 2 2 2 are provided. The students are required
Solution: A + B + C = p Þ B + C - A = p - 2A
2 , etc. Therefore p = 1, q = 2.
Therefore to find the exact answers to numerical
Answer: 3
LHS = sin(p - 2 A) + sin(p - 2 B) + sin(p - 2C ) problems and enter the same in OMR
1 1
= sin 2 A + sin 2 B + sin 2C 3. If - = cot kq then the value
tan 3q + tan q cot 3q + cot q sheets. Answers can be one-digit or
= 4 sin A sin B sin C of k is .
Answer: 4 Solution: two-digit numerals.
2. If A + B + C = p
p, then 1 1
-
tan 3q + tan q cot 3q + cot q
A C B A B C
sin2 - sin2 + sin2 = p - q cos cos sin
2 2 2 2 2 2 1 tan q tan 3q
= -
where p + q is . tan 3q + tan q tan 3q + tan q

Solution: 1 - tan 3q tan q 1


= = = cot 4q
tan 3q + tan q tan 4q
A B C
sin2 + sin2 - sin2 Answer: 4
2 2 2
EXERCISES EXERCISES
Single Correct Choice Type Questions
1. The value of sin 36° = å m= 0 Am cos mx, where A0,
6. Suppose that sin3 x sin 3 x =
n
For self-assessment, each chapter has
( 5 + 1) 10 + 2 5 5+1 A1, , Am are constants and An ¹ 0, then the value of n is
(A) (B) (A) 8 (B) 6 (C) 4 (D) 9
adequate number of exercise prob-
8 4
10 + 2 5 ( 5 - 1) 10 + 2 5 7. If a, b, c and d are smallest positive angles in the
lems where the questions have been
(C) (D)
8 8 ascending order such that the sine of each angle is equal
to a positive constant l, then 4 sin(a/ 2) + 3 sin(b/ 2) +
subdivided into the same categories as
2 sinq 1 - cosq + sinq 2 sin(c/ 2) + sin(d/
d 2) is equal to asked in IIT-JEE.
2. If a = , then is equal to
1 + cosq + sin q 1 + sinq (A) 2 1 + l (B) 1+ l
(A) 1 - a (B) 1 + a (C) a (D) 1/a
( ) 1+ l ( ) 3 1+ l

Multiple Correct Choice Type Questions


1. 3. q = (m2 − n2)/(m2 + n2), then
n n m2 - n2 m2 + n2
æ cos A + cos B ö æ sin A + sin B ö (A) tanq = (B) cosecq =
x=ç + 2 mn m2 - n2
è sin A - sin B ø÷ çè cos A - cos B ø÷
m2 + n2 2 mn
(C) sec q = (D) cotq =
Then 2 mn m2 + n2
(A) x = 0 if n is an odd positive integer
(B) x = tann(A − B)/ 2 if n is an even positive integer 4. Which of the following are true?

(C) x = 2 cotn(A − B)/ 2 if n is an even positive integer (A) tan 20° + tan 72° + tan 88° = tan 20° · tan 72° · tan 88°
(D) x = 0 if n is an even positive integer (B) tan 51° + tan 62° + tan 67° = tan 51° · tan 62° · tan 67°
(C) cot 27° + cot 32° + cot 31° = cot 27° · cos 32° · cot 31°
2. Which of the following statements are true? (D) 3 + tan 40° + tan 80° = 3 tan 40° tan 80°
(A) If 2
5
Matrix-Match Type Questions
In each of the following questions, statements are given matches are (A) ® (p) and (s); (B) ® (q),(s) and (t);
in two columns, which have to be matched. The state- (C) ® (r); and (D) ® (r),(t) then the correct darkening
ments in Column I are labeled as (A), (B), (C) and of bubbles will look as follows:
(D), while those in Column III are labeled as (p), (q),
(r), (s) and (t). Any given statement in Column I can p q r s t
have correct matching with one or more statements in A
Column II. The appropriate bubbles corresponding to
B
the answers to these questions have to be darkened as
illustrated in the following example. C
D
Example: If the correct matches are (A) ® (p),(s);
(B) ® (q),(s),(t); (C) ® (r); (D) ®(r),(t); that is if the

Comprehension-Type Questions
1. Passage: a cos q + b sin q = c can be 3. Passage: To eliminate a parameter, we need two
solved when | c | £ a2 + b2 . Based on this answer the equations involving the parameter. For example, if
following questions (i), (ii) and (iii). x = rcos q and y = rsin q, then by squaring and adding,
we have x2 + y2 = r2. This shows that q is eliminated
(i) If 0 £ x £ p and sin x + cos x = 1, then the number from the given equations.
of values of x is
Answer the following questions (i), (ii) and (iii).
(A) 1 (B) 2 (C) 4 (D) 0
(i) If tan q + sin q = a and tan q - sin q = b, then after
(ii) The maximum and minimum values of eliminating q, (a2 - b2)2 is equal to
3 cos q + 4 sin q - 5 are respectively (q
q is real)
(A) 4 ab (B) 4ab (C) 16 ab (D) 16ab
(A) 5, - 5 (B) 4, 3 (C) 0, - 10 (D) 4, - 3
(ii) Eliminating q from the equations x = cot q + tan q
(iii) The maximum and minimum values of
and y = sec q - cos q, we have
5 sin q + 12 cos q + 13 are respectively (q
q is real) 2
( ) ( ) ( ) ( )2/3 ( 2 2/3
)
(A) 13
Assertion–Reasoning Type Questions
In the following set of questions, a Statement I is given 3. Statement I: For all values of q, 2(sin6q + cos6q)
q -
and a corresponding Statement II is given just below it. 3(sin4q + cos4q)
q = -1
Mark the correct answer as:
Statement II: x3 + y3 = ( x + y) ( x2 - xy + y2 )
(A) Both Statements I and II are true and Statement II
is a correct explanation for Statement I q = m, then
4. Statement I: If tan(q/2)
(B) Both Statements I and II are true but Statement II is
1 - 2 sin2 (q / 2) 1 + m
not a correct explanation for Statement I =
1 + sin q 1- m
(C) Statement I is true and Statement II is false
(D) Statement I is false and Statement II is true 2 tan q 1 - tan2q
Statement II: sin 2q = and cos 2q =
1 + tan2q 1 + tan2q
1. Statement I: If x cos a + y sin a = x cos b + y sin b = 2a
and 2 sin a / 2 sin b / 2 = 1, then y2 = 4a(a - x) 5. Statement I:
Statement II: sin 3a 11
equation
q a Integer Answer Type Questions =
sin a 25
The answer to each of the questions in this section is a 6. If 2 tan(A + B) = 3 tan A, then sin(2A
2 + B) = ksin B,
non-negative integer. The appropriate bubbles below where k is equal to .
the respective question numbers have to be darkened.
For example, as shown in the figure, if the correct answer 7. If 3 cos x = 2 cos(x - 2y
2 ), then cot(x - y)cot y is equal
to the question number Y is 246, then the bubbles under to .
Y labeled as 2, 4, 6 are to be darkened.
8. If sin 2° + sin 4° + sin 6° +  + sin 178° = n cot 1°, then
X Y Z W n is equal to .
0 0 0 0
9. Let 0 < a j < p
p/2 for j = 1, 2, 3, , 8 and cot a 1 × cot a 2 
1 1 1 1
cot a 8 = 1. If M is the maximum value of cot a 1 cot a 2
2 2 2  cot a 8, then the value of (32) M is .
3 3 3 3
4 4 4 10. The value of (1 + cot A - cosec A)(1 + tan A + sec A)
is .
5 5 5 5
ANSWERS
The Answer key at the end of each chapter contains answers to all exercise problems.

ANSWERS
Single Correct Choice Type Questions
1. (D) 29. (A)
2. (C) 30. (C)
3. (A) 31. (A)
4. (B) 32. (A)
5. (D) 33. (A)
6. (B) 34. (B)
7. (A) 35. (C)
8. (C) 36. (A)
9. (D) 37. (B)
10. (C) 38. (D)
11. (A) 39. (A)
12. (C) 40. (A)
13. (B) 41. (B)
14. (A) 42. (B)
15. (D) 43. (A)
16. (A) 44. (C)
17. (C) 45. (D)
18. (C) 46. (B)
19. (C) 47. (A)
20. (A) 48. (B)
21. (A) 49. (A)
22. (A) 50. (B)
23. (B) 51. (B)
24. (B) 52. (C)
25. (D) 53. (A)
26. (A) 54. (D)
27. (B) 55. (A)
28. (B) 56. (D)

Multiple Correct Choice Type Questions


1. (A), (C) 9. (B), (C)
2. (A), (B), (C) 10. (A), (C), (D)
3. (A), (B), (C) 11. (A), (B), (C)
4. (A), (B), (C), (D) 12. (B), (C)
5. (A), (B), (C), (D) 13. (A), (D)
6. (A), (B), (C) 14. (A), (C), (D)
7. (A), (B), (D) 15. (A), (B), (C), (D)
8. (A), (D) 16. (A), (B), (C), (D)

Matrix-Match Type Questions


1. (A) ® (p), (s), (B) ® (q), (C) ® (r), (D) ® (t) 4. (A) ® (s), (B) ® (r), (C) ® (q), (D) ® (p)
2. (A) ® (p), (B) ® (p), (C) ® (p), (D) ® (s) 5. (A) ® (t), (B) ® (p), (C) ® (r), (D) ® (q)
3. (A) ® (r), (B) ® (s), (C) ® (q), (D) ® (p) 6. (A) ® (p), (B) ® (p), (C) ® (q), (D) ® (q)

Comprehension-Type Questions
1. (i) (B); (ii) (C); (iii) (C) 3. (i) (D); (ii) (B); (iii) (A)
2. (i) (A); (ii) (B); (iii) (D)
BOOK FEEDBACK FORM

WE WOULD LIKE TO HEAR FROM YOU

Please complete this form. Your feedback concerning "Mathematics for IIT-JEE" will be appreci-
ated. If you do want to complete and submit the form online, you may visit us at www.wileyindia.com/
murti. You can also fill-in the form and send it as an attachment via E-mail or Fax it to +91-11-23275895.
Our e-mail address is acadmktg@wiley.com and mailing address is Wiley India Pvt. Ltd, Attn:
Marketing Department, 4436/7, Ansari Road, Daryaganj, New Delhi- 110002, India.

* Fields marked are compulsory

* Name

* Email Address

Address

Phone Mobile
School/Institute/College
Cut from here

* Name of Course Instructor

* Competitive Exam(s) Studying for

How did you hear about this book? Bookseller Classmate Teacher Advertisement
Other
(please mention)

Would you be interested in further readings by the authors? Yes No

Would you be interested in our other bestsellers from the Wiley MAESTRO series? Yes No

Your Comments

May we use your comments in future promotions? Yes No

Send us the form & get a chance to enter




Wiley India’s Monthly Student raffle! Win exciting Prizes!


Visit www.wileyindiablog.blogspot.com or www.wileyindia.com
to check the list of monthly winners.
Contents
1 T
Trigonometric Ratios and Transformations 1

1.1 Angles and Their Measures ............................................................................................................................ 2


1.2 T
Trigonometric Ratios ....................................................................................................................................... 7
1.3 Periodicity and Variance
V ............................................................................................................................... 19
1.4 T
Trigonometric Ratios of Compound Angles ................................................................................................. 30
1.5 T
Trigonometric Ratios of Multiple and Submultiple Angles ........................................................................... 36
1.6 Sum and Product Transformations ................................................................................................................ 47
Worked-Out Problems .................................................................................................................................. 51
Summary ....................................................................................................................................................... 87
Exercises ....................................................................................................................................................... 89
Answers ........................................................................................................................................................ 97

2 Inverse Trigonometric Functions 99

2.1 Domains and Ranges of Inverse Trigonometric


T Functions .......................................................................... 100
2.2 Properties of Inverse Trigonometric Functions ........................................................................................... 102
Worked-Out Problems ................................................................................................................................ 112
Summary ..................................................................................................................................................... 132
Exercises ..................................................................................................................................................... 135
Answers ...................................................................................................................................................... 139

3 T
Trigonometric Equations 141

3.1 The Least Positive Angles with a Given Trigonometric


T Ratio ..................................................................... 142
3.2 General Solution of Equations of the Form sin x = a .................................................................................. 144
3.3 Solutions of Simple Trigonometric
T Equations ............................................................................................. 148
Worked-Out Problems ................................................................................................................................ 150
Summary ..................................................................................................................................................... 177
Exercises ..................................................................................................................................................... 178
Answers ...................................................................................................................................................... 184

4 Properties of Triangles 185

4.1 Relations Between the Sides and the T Trigonometric Ratios of Angles of a Triangle .................................. 186
4.2 Solution of Triangles
T ................................................................................................................................... 201
4.3 Properties of a Triangle ............................................................................................................................... 209
Worked-Out Problems ................................................................................................................................ 232
Summary ..................................................................................................................................................... 287
Exercises ..................................................................................................................................................... 289
Answers ...................................................................................................................................................... 295
xxii Contents

5 Addition and Scalar Multiplication of Vectors


V 297
5.1 Definition and Classification of Vectors V ...................................................................................................... 298
5.2 Addition of Vectors
V ..................................................................................................................................... 300
5.3 Multiplication of a Vector
V by a Scalar ......................................................................................................... 309
5.4 The Division Formula .................................................................................................................................. 312
5.5 Components of a Vector V ............................................................................................................................. 321
5.6 Vector Equation of a Line and a Plane ........................................................................................................
V 333
Worked-Out Problems ................................................................................................................................ 340
Summary ..................................................................................................................................................... 357
Exercises ..................................................................................................................................................... 362
Answers ...................................................................................................................................................... 364

6 Multiplication of Vectors
V 365

6.1 Scalar or Dot Product ................................................................................................................................. 366


6.2 Vector Equations of a Plane and a Sphere .................................................................................................
V 382
6.3 Vector or Cross Product ..............................................................................................................................
V 387
6.4 Vector Areas ...............................................................................................................................................
V 398
6.5 Scalar Triple
T Product ................................................................................................................................... 404
6.6 Vector Triple Product ..................................................................................................................................
V 415
Worked-Out Problems ................................................................................................................................ 419
Summary ..................................................................................................................................................... 475
Exercises ..................................................................................................................................................... 480
Answers ...................................................................................................................................................... 488

7 Probability 489

7.1 Random Experiments and Events ............................................................................................................... 490


7.2 Classical Definition of Probability ............................................................................................................... 491
7.3 Axiomatic Approach to Probability ............................................................................................................. 494
7.4 Independent and Dependent Events ......................................................................................................... 499
7.5 Random Variables
V and Probability Distributions ........................................................................................ 504
7.6 Theoretical Discrete Distribution ................................................................................................................ 508
Worked-Out Problems ................................................................................................................................ 511
Summary ..................................................................................................................................................... 556
Exercises ..................................................................................................................................................... 559
Answers ...................................................................................................................................................... 567

8 Inequalities 569

8.1 Introduction ................................................................................................................................................ 570


Worked-Out Problems ................................................................................................................................ 571
Exercises ..................................................................................................................................................... 590

Index 593
Trigonometric Ratios
and Transformations 1
Y
y = sin x
Contents
1.1 Angles and Their
1
Measures
X X
p p/2 O p/2 p 3p/2 2p 5p/2 3p
1.2 Trigonometric Ratios
Y y = cos x
–1
1.3 Periodicity and
1

Y
Variance
X
p/2 O p/2 p 3p/2 2p 5p/2
X 1.4 Trigonometric Ratios
1 of Compound Angles
Y
1.5 Trigonometric Ratios
Y y = tan x of Multiple and
Trigonometric Ratios and

Submultiple Angles
1.6 Sum and Product
Transformations
X X
p/2 O p/2 p 3p/2

Y
y = cot x Worked-Out Problems
W
Summary
Exercises
Answers
X X
p/2 0 p/2 p 3p/2 2p 5p/2
Transformations

Y y = sec x Trigonometric Ratios: The


relationships between the
2 angles and the sides of a
1
right triangle are expressed
X
p/2 0 p/2 p 3p/2
X
in terms of trigonometric
1

2
Y y = cosec x
ratios.

X X
O p/2 p 3p/2 2p
1

2
2 Chapter 1 Trigonometric Ratios and Transformations

Trigonometry is the study of the relation between angles and sides of a triangle. This is an essential branch of
mathematics useful for the measurement of areas, distances and heights.

1.1 | Angles and Their Measures


Consider a straight line extending indefinitely in both the directions and let O be a point on this straight line. Then the
point O divides the line into two parts and each of these two parts is called a ray and O is called the vertex of the two
rays. Usually rays are denoted by OA, where O is the vertex and A is any point on the ray (ray is a part of the straight
line). In Figure 1.1, OA and OB are rays. The vertex of a ray is called the origin of the ray or the initial pointt of the ray.

FIGURE 1.1 Rays OA and OB.

DEFINITION 1.1 The figure formed by two rays with the common vertex is called an angle and is denoted by
AOB or AOB, where O is the common vertex, A is a point on one ray and B is a point on
the other ray as shown in Figure 1.2.

FIGURE 1.2 Angle.

A real number is associated with each angle and this number is called the measure of the angle. There are two impor-
tant systems of measurement of an angle: the sexagesimal system (or British system) and the radian system (or circular
measure system).

DEFINITION 1.2 Sexagesimal System An angle AOB  is called a right angle if OB is perpendicular to OA,
geometrically. In sexagesimal system a right angle is divided into 90 equal parts called degrees.
Each degree is divided to 60 equal parts called minutes and each minute is divided into
60 equal parts called seconds. One degree, one minute and one second are denoted by 1°,
1¢ and 1², respectively. Therefore,
1 right angle = 90°
1° = 60 ¢
and 1¢ = 60¢¢
Figure 1.3 illustrates the sexagesimal system.

DEFINITION 1.3 Radian System The angle subtended at the centre of a circle by an arc whose length is equal
to the radius of the circle is called a radian and is denoted by 1c.

90°

FIGURE 1.3 Sexagesimal system.


1.1 Angles and Their Measures 3

1c
O A

FIGURE 1.4 Radian system.

Figure 1.4 shows the radian system. In the following theorem, we prove that this angle is independent of the radius of
the circle, we have considered.

T H E O R E M 1.1 The radian is constant, in the sense that it is independent of the radius of the circle.
PROOF Consider a circle with centre O and radius r. Let A and B be points on the circle such that the
length of arc AB is equal to the radius r of the circle (Figure 1.5). Then by definition,
 = 1 radian = 1c
AOB
Produce AO to meet the circle at C. Then AC is a diameter of the circle and the length of the arc
ABC is equal to half of the circumference of the circle. Therefore,
 = 2 right angles = 180°
AOC
We know that the angles subtended at the centre of a circle are proportional to the lengths of the
arcs which subtend them. Therefore

AOC arc ABC
=

AOB arc AB
Also, we know that the ratio of the circumference and diameter of a circle is always a constant
and is denoted by pp. Therefore, the circumference of any circle is equal to pd
p (= 2pr
p ), where d is
the length of any diameter of the circle. From these, we have
180° (2pr )/ 2
= =p

AOB r
 = 180° /p, which is independent of the radius of the circle.
AOB

1c
C A
O

FIGURE 1.5 Theorem 1.1. ■


4 Chapter 1 Trigonometric Ratios and Transformations

QUICK LOOK 1

1. The value of the constant p mentioned in Theo- æpö


A right angle = ç ÷ radians = 90°
rem 1.1 is approximately 22/7 (= 3.1415…). In fact è 2ø
p is not a rational number; nor it is an alge-
braic number in the sense that p is not a root of Also,
any nonzero polynomial with rational coefficients.
p
However, 22/7 or 3.1415… are only approximate 1° = radians
values of p.
p 180
2. From Theorem 1.1, it follows that These provide us methods to convert the measure-
ment of an angle in sexagesimal system into that in
180°
1c = radian system and vice-versa.
p
3. Usually angles are measured in radians or degrees.
or p radians = 180° = 2 right angles For an angle having degree measure q, we write q °.
When we do not mention the superscript °, the
p c = 180° measure is considered to be in radians.
and therefore

Example 1.1

Express 40°36¢20²² in radian measure. Therefore

Solution: æ 6 1 ö°
40°36 ¢20 ¢¢ = ç 40 + + ÷
è 10 180 ø
20
20 ¢¢ = minutes
60 æ 6 1 öæ p ö
= ç 40 + + ÷ç ÷ raadians
° è 10 180 ø è 180 ø
20 æ 1 ö
= degree = ç ÷
60 ´ 60 è 180 ø 7200 + 108 + 1
= p radians
180 ´ 180
° °
æ 36 ö æ 6ö
36 ¢ = ç ÷ = ç ÷ æ 7309 ö
è 60 ø è 10 ø =ç p
è 32400 ÷ø

Example 1.2

p c in degrees.
Express (5p/6) c °
æ 5p ö æ 5 ö
çè ÷ø = çè × 180÷ø = 150°
6 6
Solution:
The given radian measure in degrees is

T H E O R E M 1.2 Let q be the angle in radians subtended by an arc of length a at the centre of a circle of radius r.
Then
a
q= radians
r
PROOF We have from the definition of a radian,
1 radian = The angle subtended by an arc of length r at the centre of circle
1.1 Angles and Their Measures 5

Therefore
1
radian = The angle subtended by an arc of length 1 at the centre
r
a
radian = The angle subtended by an arc of length a at the centre
r ■

Let us consider a unit circle (a circle of unit radius) having centre at C. Let O be a point on the circle. Let CO be the
initial side of an angle. Since the radius of the circle is one unit, the length of an arc of this circle will be the radian
measure of the angle subtended by it at the centre C. Consider the line AOB, which is the tangent to the circle at O.
We can consider AOB as the real line with O representing the real number zero, the points towards OB representing
positive real numbers and those towards OA representing negative real numbers. Now, if the line OB is roped in
anticlockwise direction along the circle, then every positive real number will correspond to a radian measure and
conversely. Similarly, if the line OA is roped in clockwise direction along the circle, then each negative real number
will correspond to a radian measure and conversely. Thus, the radian measures and real numbers are same. However,
regarding angles, note that two different real numbers may represent the same angle; for example, p p/2 and 5pp/2 repre-
sent the same angle. In fact for any angle q, in radians, q + 2npp represents the angle q only for any integer n.

1 unit

FIGURE 1.6 Circle of unit radius.

T H E O R E M 1.3 For any real number x, there exist unique integer n and a real number q such that
x = 2 np + q and 0 £ q < 2p
PROOF p
Let n be the integral part of x/2p; that is, n is the largest integer which is less than or equal to x/2p.
Then
x
n£ <n+1
2p
Therefore
n(2p ) £ x < n(2p ) + 2p

0 £ x - 2 np < 2p
Now, put q = x - 2np.
p Then
x = 2 np + q and 0 £ q < 2p
To prove the uniqueness of n and q, let n and m be integers and q and f be real numbers such that
2 np + q = 2 mp + f, 0 £ q < 2p and 0 £ f < 2p
6 Chapter 1 Trigonometric Ratios and Transformations

Then
|q - f | = 2 | m - n | p ³ 2p if m ¹ n

and
|q - f | < 2p

Therefore m = n and q - f. ■

Note: The integer n above can be considered as the number of completed revolutions of the circle in Figure 1.6 to reach x.

Example 1.3

Suppose that a clock shows half past 3. Find the angle 12


between the hours hand and the minutes hand in degrees
and radians.

Solution: From the hypothesis, the hours hand is


exactly in middle of 3 and 4 and minutes hand is at 6
9 3
(Figure 1.7). The 12 divisions in the clock totally repre-
sent 360° and hence each division represents 30°.
Therefore, the angle between the hours hand and the 4
minutes hand is 30° + 30° + 30°/2 = 75°, which is equal to
75 ´ p/180
p = 5p/12
p radians. 5
6

FIGURE 1.7 Example 1.3.

Example 1.4

Determine the radius of the circle in which an arc of We are given here that q = 80° and l = 16 and we have
length 16 cm subtends an angle of 80° at the centre of to find r. First we have to convert the measure of q into
the circle. radians. We have
p 4p
Solution: If l is the length of the arc and q is the angle q = 80° = 80 ´ =
subtended at the centre of the circle of radius r, we know, 180 9
from Theorem 1.2, that Therefore
l 4p l 16
q= radians = =
r 9 r r
9 ´ 16 36 æ 7 ö
r= = çè = 36 ´ approx.÷
4p p 22 ø

Example 1.5

Let a triangle be given such that its angles are in arithmetic Since d > 0, a - d < a < a + d, from the hypothesis,
progression and that the ratio of the greatest to the smallest we have
angles, in degrees, is p:60. Find all the angles of the triangle.
a+d p 60 + d 180
= or = =3
Solution: Let the angles be a - d, a and a + d, where a - d 60 60 - d 60
d > 0. The sum of the angles in a triangle is 180°. Therefore Therefore d = 30° and so 30°, 60° and 90° are all the
3a = 180° angles of the given triangle.
a = 60°
1.2 Trigonometric Ratios 7

1.2 | Trigonometric Ratios


p
In this section we will define the trigonometric ratios sine and cosine as real-valued functions in the interval [0, 2p]
first and later extend the domain of these to the entire real line. Using the definitions of these, we define the other
trigonometric functions such as cosecant, secant, tangent and cotangent on appropriate domains.

DEFINITION 1.4 Sine and Cosine Ratios Let q be a real number such that 0 £ q £ 2p.
p Consider a rectangular
coordinate system OXY. Draw a circle with centre at O and radius r > 0 in the coordinate
plane. Choose a point P on this circle such that OP makes an angle of q radians with OX,
measured in the anticlockwise sense (see Figure 1.8).
With reference to the coordinate system OXY, let (x, y) be the coordinates of the point P.
Then, we define the sine ratio of q as
y
sinq =
r
and the cosine ratio of q as
x
cosq =
r

First note that these ratios sin q and cos q are independent of the circle and its radius r. Consider Figure 1.9 in which
two circle C1 and C2 of radius r1 and r2 are given and the points P(x1, y1) and Q(x2, y2) lie on these circles, respectively,
such that O, P and Q are collinear.

Y Y

P(x, y) P(x, y)

y q
q
X X
O x O

Y Y

X X
q O O
q

P(x, y) P(x, y)

FIGURE 1.8
8 Chapter 1 Trigonometric Ratios and Transformations

Q
r2
P
r1 y2
r1 y
1
q
X
O M N

FIGURE 1.9

Let M and N be feet of the perpendiculars drawn from P and Q, respectively, on OX. Then, PM = y1, QN = y2.
The triangles OPM and OQN are similar and hence
PM QN
=
OP OQ
Therefore
y1 y2
=
r1 r2
Thus, sin q is independent of the circle and its radius. Similarly, cos q is also independent of the circle and its radius.

QUICK LOOK 2

1. If P(x, y) is a point on the circle of radius r with 3. If P(x, y) is a point on the unit circle (circle of
centre at the origin O such that OP makes an angle radius 1) with centre at the origin O and OP makes
of q radians with the ray OX in the anticlockwise an angle q radians with the X-axis, then
sense, then we have
x = cos q and y = sin q
x = r cos q and y = r sin q
4. For any 0 £ q £ 2p,
p
2. For any 0 £ q £ 2p,
p we have
sin q = 0 Û y = 0 (since r > 0)
sin2 q + cos2 q = 1 Û P( x, y) lies on the X-axis
and hence Û q = 0 or p radians
sin2 q = 1 - cos2 q
and
and cos2 q = 1 - sin2 q
cos q = 0 Û x = 0
where sin2 q = sin q × sin q and cos2 q = cos q × cos q.
Û P( x, y) lies on the Y-axis
p 3p
Ûq = or
2 2

DEFINITION 1.5 Let x be a real number. Then there exist unique integer n and a real number q such that
x = 2 np + q and 0 £ q < 2p
1.2 Trigonometric Ratios 9

(see Theorem 1.3). Now, we define


sin x = sin q
and cos x = cos q
In other words, we are defining
sin(2np + q ) = sin q
and cos(2np + q)
q = cos q
for any integer n and real number q such that 0 £ q < 2p.
p

QUICK LOOK 3

1. Sine (sin) and cosine (cos) are real-valued func- and


tions defined on the entire real line and both are of
p that is,
period 2p, cos(q + 2p ) = cos q

sin(q + 2p ) = sin q for all real numbers.


2. sin = 0 Û x = npp for some integer n and cos x = 0 Û
x = (2n + 1) p
p/2 for some integer n.

DEFINITION 1.6 Secant and Tangent For any real number x ¹ (2n + 1)p/2,
p n Î, we define

1 sin x
secant x = and tangent x =
cos x cos x

and denote secant x and tangent x simply by sec x and tan x; that is,

1 sin x
sec x = and tan x =
cos x cos x

for any real number x which is not of the form (2n + 1)p/2,
p where n is an integer.

DEFINITION 1.7 Cosecant and Cotangent For any real number x ¹ np,
p n Î, we define

1 cos x
cosecant x = and cotangent x =
sin x sin x

and denote cosecant x and cotangent x simply by cosec x and cot x; that is,

1 cos x
cosec x = and cot x =
sin x sin x

p where n is an integer.
for any real number x which is not of the form np,

QUICK LOOK 4

Unlike the trigonometric functions sine and cosine, the and cot x are not defined at x = np,
p n Î, since sin x = 0
trigonometric functions secant, tangent, cosecant and at these x. The domains of all the trigonometric funct-
cotangent are not defined on the whole real line. The ions are collected in Table 1.1 for convenience and for
functions sec x and tan x are not defined at x = (2n + 1) quick reference.
p n Î, since cos x = 0 at these x. Similarly cosec x
p/2,
10 Chapter 1 Trigonometric Ratios and Transformations

Table 1.1 Domains of all the trigonometric functions


Function Domain
sin x 
cos x 
cosec x  - {np |n Î}
ì p ü
sec x  - í(2 n + 1) | n Î ý
î 2 þ
ì p ü
tan x  - í(2 n + 1) | n Î ý
î 2 þ
cot x  - {np | n Î}

We have discussed earlier about all the values of x for which the values of each of the trigonometric ratios sin x, cos x,
tan x, cosec x and cot x are defined. In the following paragraphs, we discuss the values of x for which these are positive
and those values of x for which they are negative.

QUICK LOOK 5

Let q be a real number such that 0 < q < 2p, p q ¹ p/2,


p 3. If p < q < 3p/2,
p then P lies in the third quadrant of
q ¹ p and q ¹ 3p/2.
p Consider a rectangular coordinate the plane and hence x < 0 and y < 0, so that tan q > 0
system OXY, as in Definition 1.4, with O as origin. Let and cot q > 0 and all others are negative. In this case,
r be a positive real number. Choose a point P in the q is said to be in the third quadrant.
coordinate plane such that OP = r and OP makes an p < q £ 2p,
4. If 3p/2 p then P lies in the fourth quadrant
angle of q radians with OX measured in the anticlock- of the plane and hence x > 0 and y < 0, so that cosq
wise sense. Let (x, y) be the coordinates of the point P. and secq are positive and all the others are negative.
We have the following important observations. Recall In this case, q is said to be in the fourth quadrant.
that both x and y are nonzero since q Ï{0, p/2,p pp, 3p/2}.
p The angles 0, p/2,
p p, 3p
p p/2 and 2p are called the
1. If 0 < q < p
p/2, then P lies in the first quadrant of the quadrant angles.
plane and hence x > 0 and y > 0, so that 5. The signs (positive or negative) of the trigonometric
y x ratios in the four quadrants can be remembered as
sin q = > 0; cosq = > 0 shown below.
r r
y r
tan q = > 0; sec q = > 0 I II III IV
x x
x r All Silver Tea Cups
cot q = > 0; cosec q = > 0
y y
· In quadrant I, all are positive.
tions are positive at q when 0 £ q £ pp/2. In this case · In quadrant II, only sine (and its reciprocal cose-
we say that q lies in the first quadrant. cant) is positive.
2. If p/2
p <q<p p, then P lies in the second quadrant of · In quadrant III, only tangent (and its reciprocal
the plane and hence x < 0 and y > 0, so that sin q > 0 cotangent) are positive.
and cosec q > 0 and all others are negative. In this · In quadrant IV, only cosine (and its reciprocal
case, we say that q lies in the second quadrant. secant) is positive.

Now, we shall prove certain basic identities satisfied by the trigonometric functions. For convenience, we write
f 2(x) for (f(
f x))2, where f is any function.

T H E O R E M 1.4 1. sin2 x + cos2 x = 1 for all x Î


2. 1 + tan2 x = sec2 x for all x Î - {(2n + 1) p/2
p |n Î}
3. 1 + cot x = cosec x for all x Î - {np |n Î}
2 2
1.2 Trigonometric Ratios 11

PROOF Let x Î and x = 2np + q, where n is an integer and q is a real number such that 0 £ q < 2p.
p
Consider a rectangular coordinate system OXY with O as origin. Choose a point P(x, y) in the
coordinate plane such that OP = 1 and OP makes an angle of q radians with OX measured in
anticlockwise sense (Figure 1.10).
Y

P(x, y)

1
y

q
X
O x

FIGURE 1.10 Theorem 1.4.

1. By Definition 1.4,
y
sinq = =y
OP
and
x
cos q = =x
OP
Since x2 + y2 = (OP)2 = 1, it follows that
sin2 q + cos2 q = 1
Now,
sin2 q + cos2 q = sin2 (2 np + q ) + cos2 (2 np + q )
= sin2 q + cos2 q
=1
2. Let x Î - {(2n + 1) p/2
p |n Î}. Then cos x ¹ 0 and hence tan x and sec x are defined
sin2 x
1 + tan2 x = 1 +
cos2 x
cos2 x + sin2 x
=
cos2 x
1
= = sec2 x
cos2 x
3. Let x Î - {np |n Î}. Then sin x ¹ 0 and cosec x and cot x are defined and
cos2 x
1 + cot2 x = 1 +
sin2 x
sin2 x + cos2 x
=
sin2 x
1
= = cosec2 x
sin2 x ■

C O R O L L A R Y 1.1 1. For any x Î - {(2n + 1) p/2


p |n Î},
(sec x - tan x)(sec x + tan x) = sec2 x - tan2 x = 1
that is, sec x - tan x and sec x + tan x are reciprocal to each other.
12 Chapter 1 Trigonometric Ratios and Transformations

2. For any x Î - {np |n Î},


(cosec x - cot x)(cosec x + tan x) = cosec2 x - cot2 x = 1
that is, cosec x - cot x and cosec x + cot x are reciprocal to each other.

C O R O L L A R Y 1.2 1. |sin x| £ 1 and |cos x| £ 1 for all x Î


2. |cosec x| ³ 1 for all x Î - {np |n Î} and |sec x| ³ 1 for all x Î - {(2n + 1) p/2
p |n Î}
PROOF This is an immediate consequence from the identity sin2 x + cos2 x = 1. ■

Try it out Express all the trigonometric ratios of an angle in terms of parts (1) or (2) of Corollary 1.2.

In the following examples, we have expressed all the trigonometric ratios of an angle in terms of sine and in terms
of cotangent.

Example 1.6

Express each trigonometric ratios of an angle q in terms 1


cosec q =
of sin q. sin q

Since sin2 q + cos2 q = 1, we have sin q sin q


Solution: tan q = =
cosq 1 - sin2 q
cos q = 1 - sin2 q
cosq 1 - sin2 q
1 1 cot q = =
sec q = = sin q sinq
cos q 1 - sin2q
All the above are valid for approximate values of q.

Example 1.7

q
Express all trigonometric ratios of an angle in terms of cotq. 1 cot q
cos q = 1 - sin2 q = 1 - =
1 + cot q
2
1 + cot2 q
Solution: From part (3) of Theorem 1.4, we have, for
all approximate q, 1
tan q =
cot q
cosec q = 1 + cot2 q
1 1 1 1 + cot2 q
sin q = = sec q = =
cosec q 1 + cot2 q cos q cot q

Example 1.8

If cosq = 2 2 / 3, find the numerical magnitude of all 1 3


sec q = =
trigonometric ratios of q. cos q 2 2

Solution: Using cos q = 2 2 / 3, we get sin q 1


tan q = =
cos q 2 2
2
æ 2 2ö 1 1
sin q = 1 - cos2 q = 1 - ç ÷ =3 cot q = =2 2
è 3 ø tan q
1
cosec q = =3
sin q
1.2 Trigonometric Ratios 13

Inter-Relationships Among Trigonometric Ratios


In Table 1.2, we display formulae expressing each of the trigonometric ratios in terms of any other ratio.

Table 1.2 Relationship among trigonometric ratios


Function sin q cos q cosec q sec q tan q cot q
1 1 sinq
sin q sin q 1 - sin2 q 1 - sin2 q
sinq 1 - sin q 2
1 - sin q 2
sin q
1 1 1 - cos2 q cosq
cos q 1 - cos2 q cos q
1 - cos q2 cosq
cos q 1 - cos2 q
1 cosec q 1
cosec q cosec2 q - 1 cosec q cosec2 q - 1
cosecq
cosec q cosec q - 1
2
cosec q - 1
2

sec2 q - 1 1 sec q 1
sec q sec q sec2 q - 1
sec q secq sec q - 1
2
sec q - 1
2

tanq 1 1 + tan2 q 1
tan q 1 + tan2 q tan q
1 + tan q 2
1 + tan q 2
tan q
tanq

1 cotq 1 + cot2 q 1
cot q 2 2
1 + cot2 q cot q
1 + cot q 1 + cot q cot q
cotq

Example 1.9

If tan q + sec q = 3/2, evaluate the value of sec q. 4( x + 1)2 = 9(1 - x2 )

Solution: First, express tan q and sec q in terms of sin q. 13 x2 + 8 x - 5 = 0


We have (13 x - 5)( x + 1) = 0
sin q 1 5
tan q = and sec q = x = - 1 or
1 - sin q2
1 - sin q 2
13

Put sin q = x. We are given that If x = -1, then cos q = 0 (since cos2 q + sin2 q = 1) and
hence tan q and sec q are not defined. Therefore
3 x = 5/13; that is, sin q = 5/13.
tan q + sec q =
2
Therefore
x 1 3
+ =
1- x 2
1- x 2 2

Now, let us evaluate the trigonometric ratios of certain angles and prepare a table useful for ready reckening. First
recall the following in degrees and radians.

p p p p 2p 3p 5p
Radians p 2p
6 4 3 2 3 4 6
Degrees 30° 45° 60° 90° 120° 135° 150° 180° 360°
14 Chapter 1 Trigonometric Ratios and Transformations

p 1
T H E O R E M 1.5 1. sin 30° = sin =
6 2
p 1
2. sin 45° = sin =
4 2
p 3
3. sin 60° = sin =
3 2
p
4. sin 90° = sin = 1
2
5. sin 0° = sin 0 = 0
PROOF 1. Consider a rectangular coordinate plane OXY with origin O. Let P(a, b) be a point such that
OP makes an angle of 30° with OX in anticlockwise sense. Draw a perpendicular through
P to OX to meet at M. Then PM = b and OM = a. Produce PM to Q such that PM = MQ.
Now, by the properties of triangles, OPQ is an equilateral triangle and PQ = 2b and hence
OP = 2b = OQ. Therefore
b 1
sin 30° = =
2b 2
This is graphically illustrated in Figure 1.11.
Y
P
2b

30°
X
O 3 M

2b
Q

FIGURE 1.11 Theorem 1.5 part (1).

By similar techniques, we can prove (2) through (5) also. ■

Using the values of sine given in Theorem 1.5 and Table 1.2, we can evaluate the values of other trigonometric ratios
also at these angles.

DEFINITION 1.8 Complementary Angles Two angles are said to be complementary if their sum is a right
angle. Therefore, for 0 £ q < p/2,
p q and p/2
p - q are complementary.

T H E O R E M 1.6 Let q and y be complementary angles. Then


1. sin q = cos y and cos q = sin y
2. tan q = cot y and cot q = tan y
3. sec q = cosec y and cosec q = sec y
PROOF Draw a right-angled triangle OAP with OAP  = 90° and AOP  = q (Figure 1.12). Since the sum of
 =y.
the angles of any triangle is 180° and since q + y = 90°, it follows that OPA
P

q 90°
O

FIGURE 1.12 Theorem 1.6.


1.2 Trigonometric Ratios 15

If O is treated as origin and the line OA as the X-axis, then the x-coordinate of P is OA and the
y-coordinate of P is PA. By the definition of the ratio sin q, we have
PA OA
sin q = and cosq =
OP OP
Also, when the angle y is considered, PA becomes the base and OA becomes the perpendicular
and hence
OA PA
siny = and cosy =
OP OP
And therefore (1) is proved. Similarly (2) and (3) can be proved. ■

C O R O L L A R Y 1.3 For any 0 £ q < p/2,


p
æp ö æp ö
1. sin q = cos ç - q ÷ and cosq = sin ç - q ÷
è2 ø è2 ø
æp ö æp ö
2. tan q = cot ç - q ÷ and cot q = tan ç - q ÷
è2 ø è2 ø
æp ö æp ö
3. sec q = cosec ç - q ÷ and cosec q = sec ç - q ÷
è2 ø è2 ø

Try it out Prove parts (1)-(3) of Corollary 1.3.

Thus, we have that


1. The sine of any angle is the cosine of its complement.
2. The tangentt of any angle is the cotangentt of its complement.
3. The secantt of any angle is the cosecantt of its complement.
For these reasons, sine and cosine, tangent and cotangent, and secant and cosecant can be remembered as complementary.
From these observations, we can construct Table 1.3 containing the trigonometric ratios of certain important
angles. The reader is advised before proceeding any further to table that if he/she can memorize accurately the portion
of the table included between thick lines, then one should be able to reproduce the rest easily.

Table 1.3
Degree 0° 30° 45° 60° 90° 180° 270° 360°
Ratios
Radians p p p p 3p
0 p 2p
6 4 3 2 2
1 1
sin 0 3 1 0 -1 0
2 2 2
3 1 1
cos 1 0 -1 0 1
2 2 2
2
cosec Not defined 2 2 1 Not defined -1 Not defined
3
sec 1 2 2 2 Not defined -1 Not defined 1
3
tan 0 1 1 3 Not defined 0 Not defined 0
3
cot Not defined 3 1 1 0 Not defined 0 Not defined
3
16 Chapter 1 Trigonometric Ratios and Transformations

Next, we consider variations and periodicity of the trigonometrical ratios. In fact, we have defined sin x and cos x for
all real numbers x, sec x and tan x for all x Î - {(2n + 1)p/2,
p n Î}, and cosec x and cot x for all x Î -{np |n Î}.
By the very definition of these functions, it follows that
f (2p + x) = f ( x)
for all trigonometric functions ff. Now, we have the following theorem.

T H E O R E M 1.7 For all real numbers x,


sin(- x) = - sin x and cos(- x) = cos x
PROOF The theorem is evidently true when x = 0 or p p/2 or p or 3p
p/2 or 2p
p. Let x be any real number. Then
we can write x = 2np + q, where 0 £ q < 2p p and n ÎZ.
In view of the above remark, we can assume that 0 £ q < 2p p and q ¹ p/2,
p p p, 3p
p/2. Consider a
rectangular coordinate system OXY with O as origin. Draw a unit circle in the coordinate plane
with the origin as the centre. Choose a point P on the unit circle such that XOP  measured in anti-
clockwise sense is q radians. Figure 1.13 illustrates the cases for each of the four quadrants. Draw
a perpendicular from P to OX to meet the circle at Q.
In each of the four cases, the X-coordinates of P and Q are same and their Y-coordinates are
same in absolute value but with different sign. Now, we have
sin(-q ) = sin(q + 2p )
= Y-coordinate of Q

Y Y

P P

q
q
X X
O O

Q Q

Y Y

Q Q

X X
q O O
q

P P

FIGURE 1.13 Theorem 1.7.


1.2 Trigonometric Ratios 17

= - (Y-coordinate of P)
= - sin q
and cos (-q ) = cos(q + 2p )
= X-coordinate of Q
= X-coordinate of P
= cos q
Next, we have x = 2np + q and 2np £ x < 2(n + 1)p p (recall that n is the integral part of x/2p).
p
Put f = 2(n + 1)p - x. Then 0 < f £ 2p.
p From the above argument, we have
sin(-f ) = - sin f and cos(-f ) = cos f
Therefore
sin(- x) = sin[2(n + 1)p - x]
= sin f
= - sin(-f )
= - sin[ x + 2(- n - 1)p ]
= - sin x
and
cos(- x) = cos[2(n + 1)p - x]
= cos f
= cos(-f )
= cos[ x + 2(- n - 1)p ]
= cos x ■

Argument similar to the one given above can be used to prove the following also.

T H E O R E M 1.8 The following hold for any real number x.


1. sin(p - x) = sin x and cos(p - x) = -cos x
2. sin(p + x) = -sin x and cos(p + x) = -cos x
æp ö æp ö
3. cos ç - x÷ = sin x and sin ç - x÷ = cos x
è2 ø è2 ø
æp ö æp ö
4. cos ç + x÷ = - sin x and sin ç + x÷ = cos x
è2 ø è2 ø

Try it out Prove Theorem 1.8.

DEFINITION 1.9 Supplementary Two angles q and f are said to be supplementary if q + f = p.


p

Examples

p 2p
(1) and are supplementary angles. (2) 45° and 135° are supplementary angles, as sin q = sin f
3 3 and cos q = -cos f.
18 Chapter 1 Trigonometric Ratios and Transformations

The following are consequences of Theorems 1.7 and 1.8. They can be obtained by using the “Principle of
Mathematics Induction”.

C O R O L L A R Y 1.4 For any real number x and for any integer n, the following are true.
1. sin(np - x) = (-1)n+1 sin x and cos(np - x) = (-1)n cos x
2. sin(np + x) = (-1)n sin x and cos(np + x) = (-1)n cos x
æ p ö æ p ö
3. sin ç (2 n + 1) + x÷ = (-1)n cos x and cos ç (2 n + 1) + x÷ = (-1)n + 1 sin x
è 2 ø è 2 ø
æ p ö æ p ö
sin ç (2 n + 1) - x÷ = (-1)n cos x and cos ç (2 n + 1) - x÷ = (-1)n sin x
è 2 ø è 2 ø

Try it out Prove parts (1)-(4) of Corollary 1.4.

Table 1.4 contains values of other trigonometric functions also. Here n stands for an integer and x for any real
number in the domain of corresponding functions.

Table 1.4 Values of other trignometric functions


x sin x cos x tan x cot x cosec x sec x
p-q (-1)n+1 sin q (-1)n cos q -tan q -cot q (-1)n+1 cosec q (-1)n sec q
np + q (-1)n sin q (-1)n cos q tan q cot q (-1)n cosec q (-1)n sec q
p
(2 n + 1) - q (-1)n cos q (-1)n sin q cot q tan q (-1)n sec q (-1)n cosec q
2
p
(2 n + 1) +q (-1)n cos q (-1)n+1 sin q cot q tan q (-1)n sec q (-1)n+1 cosec q
2

DEFINITION 1.10 Coterminal Two angles q and f are said to be coterminall if the difference between them is
p or 360° according as the angles are measured in radians or degrees.
an integral multiple of 2p

Example
The following are pairs of coterminal angles: (2) 40° and 400°
p 5p (3) 50° and -310°
(1) and
2 2

If q and f are coterminal angles, then q = 2np + f for some integer n and hence, for any trigometric function f
f (q ) = f (2 np + f ) = f (f )

Example 1.10

If sin q = -5/13 and q is in the third quadrant, evaluate such that OP makes of q in the anticlockwise sense, with
the value of OX. Since q is given in the third quadrant, x < 0 and y < 0
(Figure 1.14). Now, since sin q = -5/13, we get that y = -5,
5 cot2 q + 12 tan q + 13 cosec q
OP = 13 and hence x = - 12(= ± 132 - y2 ). Therefore
Solution: Consider a rectangular coordinate plane y -5 5
OXY with O as origin. Let P(x, y) be a point in the plane tanq = = =
x -12 12
1.3 Periodicity and Variance 19

x 12 Y
cot q = =
y 5
1 -13
cosec q = =
sin q 5 M
X
q O
Hence
5 cot2 q + 12 tan q + 13 cosec q
2
æ 12 ö æ 5ö æ -13 ö N
= 5 ç ÷ + 12 ç ÷ + 13 ç P(x, y)
è 5ø è 12 ø è 5 ø÷
144 169
= +5- =0 FIGURE 1.14 Example 1.10.
5 5

Example 1.11

Find the value of = - sin 60°× sin 60° + (- sin 30°) sin 30° (by Table 1.4)
sin 600° cos 330° + cos 120° sin 150° 3 3 æ 1 1ö
=- × + ç- × ÷ = -1
2 2 è 2 2ø
Solution:
sin 600° cos 330° + cos 120° sin 150° = sin(3p + 60°)
æ 3p ö æp ö
´ cos ç + 60°÷ + cos ç + 30°÷ sin(p - 30°)
è 2 ø è2 ø

Example 1.12

If sec q + tan q = 1/5 and 0 < q < 2p,


p find the quadrant in Therefore
which q lies and the value of sin q. Using this, find all the
other trigonometric ratios of q. 1 5
cosq = = >0
sec q 13
Solution: Since sec q + tan q and sec q - tan q are recip- 1 1 13 -12
rocal to each other, we have tan q = - sec q = - =
5 5 5 5
sec q - tan q = 5 -12 5 -12
sin q = tan q × cosq = × = <0
By adding this to the given equation, we get 5 13 13

1 13 Since sin q < 0 and cos q > 0, q lies in the fourth quadrant.
2 sec q = + 5 or sec q =
5 5

1.3 | Periodicity and Variance


Let us recall that a function f defined on a subset E of the real number system  is called periodic if there exists a
f x + p) = f(
positive real number p such that f( f x) for all x ÎE
E whenever x + p is also in E. In this case, p is called a period
of ff. Note that there may be several periods of the same function. For example, if p is a period of ff, then np is also a
period of f for any positive integer n.

DEFINITION 1.11 Let f be a function defined on a subset E of . If the set all periods of f has minimum, then
that minimum period is called the period of f.
f

A function may possess a period, but still it may not possess the minimum period. For example, consider a constant
function f defined on any E Í . Then every positive real number is a period of f and each period is a positive real
20 Chapter 1 Trigonometric Ratios and Transformations

number and, evidently, there is no minimum positive real number. That is, a function may be periodic without having
the period (i.e., the minimum period).
We know that each trigonometric function is periodic and 2pp is a period of each of these. In the following, we
prove that each trigonometric function has minimum period.

T H E O R E M 1.9 p
The sine function is periodic and its minimum period is 2p.
PROOF By Definition 1.5 of the sine function, sin(2p + q)
q = sin q for all real numbers q and hence 2p is
period of the sine function. We shall prove that any positive real number less than 2p p is not a
period of sine. Suppose, on the contrary, that 0 < p < 2p
p and p is a period of the sine function.
Put x0 = 2p - p. Then since p is a period
sin( x0 + p) = sin x0
Therefore
sin 2p = sin(2p - p) = sin(- p) = - sin p
Therefore
sin p = 0
and hence p = p (since 0 < p < 2p).
p But p is not a period of the sine function, since

æp ö 1 1 p
sin ç + p ÷ = - ¹ = sin
è4 ø 2 2 4

This implies that there is no p such that 0 < p < 2p


p and p is a period of the sine function. Thus,
2p is the minimum period of the sine function. ■

Similar arguments as in Theorem 1.9 yield the following theorem.

T H E O R E M 1.10 p
1. The sine, cosine, cosecant and secant functions are all periodic with minimum period 2p.
2. The tangent and cotangent functions are periodic with minimum period p. p

Example 1.13

Determine the value of the following æp ö


(2) sec 840° = sec(4p + 120°) = sec 120° = sec ç + 30°÷
(1) tan 780° è2 ø
(2) sec 840° = - cosec 30° = - 2
(3) cosec 420° 2
(3) cosec 420° = cosec(2p + 60°) = cosec 60° =
(4) cot 240° 3

Solution: 1
(4) cot 240° = cot(p + 60°) = cot 60° =
(1) tan 780° = tan(4p + 60°) = tan 60° = 3 3

Next, we discuss the variance of different trigonometric functions; that is, how does the change in value of x affects
each trigonometric ratio at x.
Recall that the domain of each of the sine and cosine is the whole real number system , while that of secant and
tangent is  - {(2n + 1)p/2,
p n Î} and that of cosecant and cotangent is  - {np, p |n Î}. In Table 1.5 we display these
together with the ranges (or images) of these functions.
1.3 Periodicity and Variance 21

Table 1.5 Domains and ranges of trigonometric functions


Domain Range
Sine  [-1, 1]
Cosine  [-1, 1]
ì p ü
Tangent  - í(2 n + 1) | n Î ý
î 2 þ
Cotangent  - {np
p | n Î} 
ì p ü
Secant  - í(2 n + 1) | n Î ý -¥, -1] È [1, +¥)
î 2 þ
Cosecant  - {np
p | n Î} (-¥, -1] È [1, ¥)

DEFINITION 1.12 Increasing and Decreasing Functions Let f be a real-valued function defined on a subset
of  and E Í Dom ff. Then f is said to be increasing on E if
x, y ÎE and x < y Þ f ( x) £ f ( y)
f is said to decreasing on E if
x, y ÎE and x < y Þ f ( x) ³ f ( y)
The following can be easily proved.

Variance and Graph of Sin x


1. The function sin x is increasing on [0, p
p/2]. As x increases from 0 to p
p/2, sin x increases from 0 to 1.
p 3p
2. The function sin x is decreasing on [p/2, p/2]. As x increases from p
p/2 and 3p
p/2 sin x decreases from 1 to -1.
p 2p
3. The function sin x is increasing on [3p/2, p]. As x increases from 3p/2
p to 2pp, sin x increases from -1 to 0.
We can plot the graph of sin x in the coordinate plane by taking angles x in radian measure on X-axis and the values of
y = sin x on Y-axis. The graph of sin x is given in Figure 1.15. The sine curve passes through the origin and the values
of sin x vary between -1 and +1 which are, respectively, the minimum and maximum. This curve is in the shape of a
wave whose wavelength is 2p, p which is nothing but the period of sin x.

-p p 3p 5p
X -p 0 p 2p 3p
2 2 2 2
Y 0 -1 0 1 0 -1 0 1 0

Y y = sin x

X X
p p/2 O p/2 p 3p/2 2p 5p/2 3p

–1

FIGURE 1.15 Graph of sin x.


22 Chapter 1 Trigonometric Ratios and Transformations

Variance and Graph of Cos x


1. The function cos x is decreasing on [0, p
p]. As x increases from 0 to p
p, cos x decreases from 1 to -1.
p 2p
2. The function cos x is increasing on [p, p]. As x increases from p to 2p
p, cos x increases from -1 to 1.
By choosing a suitable scale, plot the points (x, cos x) given in the table below and join these points with a smooth
curve to get the graph of cos x. This cosine curve does not pass through the origin. The maximum and minimum values
of cos x are 1 and -1, respectively. Each real number in [-1, 1] is a value of cos x. The curve also looks like a wave with
wavelength 2p, p which is same as the period of cos x. The graph of cos x is given in Figure 1.16.

-p p 3p 5p
X -p 0 p 2p
2 2 2 2
Y -1 0 1 0 -1 0 1 0

Y y = cos x

X X
p/2 O p/2 p 3p/2 2p 5p/2

FIGURE 1.16 Graph of cos x.

Variance and Graph of Tan x


1. The function tan x is of period p
p, the minimum period being p. p
2. The function tan x is not defined at x = [(2n + 1)/2]p,
p for any integer n.
3. The function tan x is increasing in each of the intervals [0, p p/2) and (p/2,
p p p]. As x increases from 0 to p
p/2, tan x
increases from 0 to +¥, and as x increases from p p/2 to p
p, tan x increases from -¥ to 0.
4. The tan x curve passes through the origin and does not intersect the vertical lines at x = [(2n + 1)/2]p, p for any
integer n rather it looks like touching these lines.
5. The function tan x has no minimum and maximum values.
Figure 1.17 illustrates the graph of tan x.

Y y = tan x

X X
p/2 O p/2 p 3p/2

FIGURE 1.17 Graph of tan x.


1.3 Periodicity and Variance 23

Variance and Graph of Cot x


1. The function cot x is of period p
p, the minimum period being p. p
2. The function cot x is not defined at x = np,
p for any integer n.
3. The function cot x is decreasing on the interval (0, p
p) and on each of the intervals (np,
p (n + 1)p),
p n Î. As x increases
from 0 to p
p, cot x decreases from +¥ to -¥.
4. The cot x curve does not pass through the origin and does not intersect the vertical lines x = np, p for any integer n,
but looks like touching these lines.
5. The function cot x has no minimum and maximum values.
Figure 1.18 illustrates the graph of cot x.

Y y = cot x

X X
p/2 0 p/2 p 3p/2 2p 5p/2

FIGURE 1.18 Graph of cot x.

Variance and Graph of Sec x


p is its minimum period.
1. The function sec x is periodic and 2p
p for any integer n.
2. The function sec x is not defined at np/2
3. The function sec x is increasing on each of the intervals

æ pö æp ö æ æ 1ö ö ææ 1ö ö
çè 0, ÷ø , çè , p ÷ø , çè 2 np çè 2 n + ÷ø p ÷ø and ç ç 2 n + ÷ p ,(2 n + 1)p ÷
èè
2 2 2 2ø ø

for any integer n. Also, sec x is decreasing on each of the intervals

æ æ 3ö ö ææ 3ö ö
çè (2 n + 1)p , çè 2 n + ÷p
2 ø ÷ø
and ç ç 2 n +
èè 2
÷ø p ,(2 n + 2)p ÷ø

for any integer n.


4. The sec x curve does not intersect the X X-axis and any line x = np/2,
p n Î but nearly touches these lines.
5. The function sec x has no minimum and maximum values and for no x, -1 < sec x < 1.
Figure 1.19 illustrates the graph of sec x.
24 Chapter 1 Trigonometric Ratios and Transformations

Y y = sec x

1
X X
p/2 0 p/2 p 3p/2
1

FIGURE 1.19 x.

Variance and Graph of Cosec x


p is its minimum period.
1. The function cosec x is periodic and 2p
p for any integer n.
2. The function cosec x is not defined at np
3. The function cosec x is decreasing on each of the intervals

æ æ 1ö ö ææ 3ö ö
çè np , çè n + ÷p
2 ø ÷ø
and ç ç n +
èè 2
÷ø p ,(n + 2)p ÷ø

ææ 1ö ö æ æ 3ö ö
çè çè n + 2
÷ø p ,(n + 1)p ÷ø and ç (n + 1)p , ç n +
è è ÷p
2 ø ÷ø

for any integer n.


4. The cosec x curve does not intersect the X-axis and the lines x = np
p for any integer n. But it nearly touches these
lines x = np.
p
5. The function cosec x has neither minimum nor maximum values and there is no x such that -1 < cosec x < 1.
Figure 1.20 illustrates the graph of cosec x.

Y y = cosec x

X X
O p/2 p 3p/2 2p
1

FIGURE 1.20 Graph of cosec x.


1.3 Periodicity and Variance 25

Next we consider graphs of functions of the form sin(ax + b), cos(ax + b), a sin bx, etc. First of all note that if p is a
period of a function f( f ax + b) for any nonzero real number a and real number b; for, if
f x), then p/| a | is a period of f(
p > 0 and f(
f(p + x) = f(
f x) for all x Î, then
p
>0
|a|

æ æ p ö ö
and f çaç + x÷ + b÷ = f (± p + ax + b) = f (ax + b)
è è |a| ø ø

for all x Î and hence p/| a | is a period of f(


f ax + b).
Further, if p is a period of f(
f x), then p is also a period of af(
f x) for any nonzero real number a.

Examples

(1) sin(2x + 3) is a periodic function of period 2p /| 2 | = p,


p (3) 7 tan(-3x) is periodic and p /| -3 | = p /3 is its period,
p
since sin x is of period 2p. since tan x is of period p.
p
(2) 3 cos(6x - 9) is a periodic function of period 2p /| 6 | = (4) 6 cot(4 - 5x) is of period p /| -5 | = p /5, since cot x is
p /3,, since cos x is of a period 2p.
p of period p.
p

Graph of 3 sin 2x
The function sin x is a periodic function with period 2p p and hence sin 2x is also a periodic function with period
2p /| 2 | = p.
p Therefore, 3 sin 2x is of period p
p. Also, since -1 £ sin 2x £ 1, hence -3 £ 3 sin 2x £ 3 for all x Î.
To draw the graph of y = 3 sin 2x, consider the graph of y = sin x. On the X X-axis change k as k/2 (e.g., pp/4 for p/2,
p
p/2 for p,
p p etc.) and on the YY-axis change k as 3k (e.g., write 3 in place of 1, -3 in place of -1, 3/2 in place of 1/2, etc.).
The graph of y = 3 sin 2x is as given in Figure 1.21.

y = 3 sin 2x
3

X
p p/2 p/4 0 p/4 p/2 3p/4 p 3p/2

FIGURE 1.21 y = 3 sin 2x.

Graph of cos(x - p/4)


To draw the graph of y = cos(x -p/4),
p draw the graph of y = cos x and on X
X-axis write k + p
p/4 in place of k (e.g., write
p/2 in place of p
p p/4, 3p
p/4 in place of p
p/2. etc.). The graph of cos(x -p/4)
p is shown in Figure 1.22.
26 Chapter 1 Trigonometric Ratios and Transformations

Y
x = p /4

y = cos (x p /4)

X
0 p /4 3p/4 5p /4 9p /4

FIGURE 1.22 x - p/4).


p

Another way to draw the graph of y = cos(x - p p/4) is as follows: Draw the graph of y = cos x and shift it to the right side
through a distance of p
p/4 units. The resultant graph is shown in Figure 1.23.

y = cos x

y = cos (x – p/4)

X
0 p/4 p/2 3p/4 5p/4 3p/2 7p/4

FIGURE 1.23 Alternate graph of y = cos(x -p/4).


p

Graphs of y = cos 2x and y = cos x


p and hence the period of cos 2x is p
Period of cos x is 2p p. Also, -1 £ cos x £ 1 and -1 £ cos 2x £ 1 for all x. The graphs of
y = cos x and y = cos 2x are given in Figure 1.24.

y = cos 2x
1

p/2 p 3p/2 2p
X X
O p/4 3p/4

y = cos x

FIGURE 1.24 Graphs of y = cos x and y = cos 2x.


1.3 Periodicity and Variance 27

Example 1.14

Draw the graph of y = sin(x/2).

Solution: The graph of the given function is as shown in Figure 1.25.

2
y = sin x/2
1
p 2p 3p 4p
X X
2p p O
1

FIGURE 1.25 Solution of Example 1.14.

Example 1.15

Draw the graph of y = 4 cos 2x.

Solution: The graph of the given function is as shown in Figure 1.26.

y = 4 cos 2x

X
45° 0 0° 30° 45° 90° 135° 180° 225°
1

FIGURE 1.26 Solution of Example 1.15.

Example 1.16

Draw the graph of y = 3 sin x.

Solution: The graph of the given function is as shown in Figure 1.27.


28 Chapter 1 Trigonometric Ratios and Transformations

3 y = 3 sin x

X
p p/2 0 p/2 p 3p/2 2p

FIGURE 1.27 Solution of Example 1.16.

Example 1.17

Draw the graph of y = 2 tan x.

Solution: The graph of the given function is as shown in Figure 1.28.

y = 2 tan x

X
p/2 p/4 0 p/4 p/2 p 3p/2

FIGURE 1.28

Example 1.18

Draw the graph of y = sin[x - (p/4)].


p

Solution: The graph of the given function is as shown in Figure 1.29.

y = sin [x (p/4)]

X
5p/4 0 p/4 5p/4 9p/4

FIGURE 1.29 Solution of Example 1.18.


1.3 Periodicity and Variance 29

Example 1.19

Draw the graphs of y = sin x, y = sin 2x.

Solution: The graphs of the given functions are as shown in Figure 1.30.

y = sin x
1

p p/2 p/2 p 3p/2 2p


X
O
p/4

1 y = sin 2x

FIGURE 1.30 Solution of Example 1.19.

Example 1.20

Draw the graphs of y = sin x, y = cos x.

Solution: The graphs of the given functions are as shown in Figure 1.31.

y = sin x
1
p 3p/2 2p
X
O p/2

1 y = cos x

FIGURE 1.31 Solution of Example 1.20.

Variance of all the Trigonometric Ratios in the Interval [0, 2p]


In Table 1.6, we have summarized the behavior of the curves representing the trigonometric ratios of angles in the
four quadrants. This can be used as a ready reference.

Table 1.6 Behavior of the curves representing the trigonometric ratios of


angles in the four quadrants

æp ö æ pö
Second quadrant ç , p ÷ First quadrant ç 0, ÷
è2 ø è 2ø

sine decreases from 1 to 0 sine increases from 0 to 1


cosine decreases from 0 to -1 cosine decreases from 1 to 0
cosecant increases from 1 to +¥ cosecant decreases from +¥ to 1
secant increases from -¥ to -1 secant increases from 1 to +¥
tangent increases from -¥ to 0 tangent increases from 0 to +¥
cotangent decreases from 0 to -¥ cotangent decreases from +¥ to 0
(Continued)
30 Chapter 1 Trigonometric Ratios and Transformations

Table 1.6 Continued

æ 3p ö æ 3p ö
Third quadrant ç p , ÷ Fourth quadrant ç , 2p ÷
è 2ø è 2 ø
sine decreases from 0 to -1 sine increases from -1 to 0
cosine increases from -1 to 0 cosine increases from 0 to 1
cosecant increases from -¥ to -1 cosecant decreases from -1 to -¥
secant decreases from -1 to -¥ secant decreases from +¥ to 1
tangent increases from 0 to +¥ tangent increases from -¥ to 0
cotangent decreases from +¥ to 0 cotangent decreases from 0 to -¥

1.4 | Trigonometric Ratios of Compound Angles


The algebraic sum of two or more angles is called a compound angle. In this section we derive formulae for the
trigonometric ratios of compound angles.

T H E O R E M 1.11 Let x and y be any real numbers. Then


cos( x + y) = cos x × cos y - sin x sin y (1.1)
PROOF Choose integers m and n and real numbers q and f such that 0 £ q < 2p,
p 0 £ f < 2p,
p x = 2mp + q
and y = 2np + f. We shall prove that
cos(q + f ) = cos q × cos f - sin q sin f (1.2)
Since cos 0 = 1 and sin 0 = 0, Eq. (1.1) becomes trivial if q = 0 or f = 0. Suppose that q > 0 and f > 0.
We have to distinguish several cases.
Case I: Suppose that q + f < 2p
Consider a rectangular coordinate system OXY with O as origin. Consider the unit circle in the
coordinate plane with centre at the origin. Let P be the point with coordinates (1, 0). Choose
points Q, R and S on the unit circle such that PÔQ, PÔR and PÔS, measured in the anticlockwise
sense, are q, q + f and 2p - f, respectively. Then the coordinates of Q, R and S are (cos q, sin q), q
(cos(q + f), sin(q + f)) and (cos(2p - f), sin(2p - f)), respectively. Since cos(2p - f) = cos f and
sin(2p - f) = -sin f, the coordinates of S are (cos f, -sin f). We will prove that PR = QS in each of
the following three subcases under case I.
1. Suppose that q + f < p : In the triangle POR are QOS
OR = 1 = OS
OP = 1 = OQ
and PÔR = q + f = QÔS
therefore PR = QS (Figure 1.32).

f
q
O X
f P(1, 0)

FIGURE 1.32 Case when q + f < p.


p
1.4 Trigonometric Ratios of Compound Angles 31

2. See Figure 1.33. Suppose that q + f = p.


p Then
PR = Diameter = 2 = QS

q
R X
O P(1, 0)
f

FIGURE 1.33 Case when q + f = p.


p

3. See Figure 1.34. Suppose that q + f > p.


p Then
2p - (q + f) < p

Q Q
S

O q q
X f X
P(1, 0) O P(1, 0)
f

S
R

FIGURE 1.34 Case when q + f > p.


p

In triangles POR and QOS, we have


PÔR = 2p - (q + f) = QÔS
and OP = 1 = OQ and OR = 1 = OS. Therefore
PR = QS
Thus, in each of the three subcases under case I, we have proved that PR = QS and hence
(PR)2 = (QS)2.
Since P = (1, 0), Q = (cos q, sin q
q), R = (cos(q + f), sin(q + f)) and S = (cos(2p - f), sin(2p - f)) =
(cos f, -sin f), it follows that

(cos(q + f ) - 1)2 + (sin(q + f ) - 1)2 = (cos f - cos q )2 + (-sin f - sin q )2

Therefore

cos2 (q + f ) + sin2 (q + f ) + 1 - 2 cos(q + f ) = cos2 f + cos2 q - 2 cos f cos q


+ sin2 f + sin2 q + 2 sin f sin q
32 Chapter 1 Trigonometric Ratios and Transformations

Since sin2 x + cos2 x = 1 we get


2 - 2 cos(q + f ) = 2 - 2 cos f cos q + 2 sin f sin q

Þ cos(q + f ) = cos q cos f - sin q sin f


Case II: Suppose that q + f = 2p,
p then f = 2p - q. In this case

cos(q + f ) = cos 2 p = 1 = cos q cos (2p - q ) - sin q sin(2p - q )

= cos q cosf - sin q sinf


Case III: Suppose that q + f > 2p.p If q £ p and f £ p p, then q + f £ 2p
p and the required iden-
tity is proved as in case I and case II. Therefore, we can assume that q > p or f > p ; that is,
p < q < 2p or p < f < 2p.
1. Suppose that p < q < 2pp and 0 < f £ p
p. Put y = q - p
p. Then 0 < y £ p and y + f £ 2p.
p Hence
cos(q + f ) = cos(p + y + f )

= - cos(y + f )

= - (cos y cos f - sin y sin f )

= cos(p + y ) cosf - sin(p + y ) sin f

= cos q cos f - sin q sinf

2. Suppose that 0 < q £ 2p


p and p < f £ p
p. The proof in this case is similar to the one in (1) above.
3. Suppose that p < q < 2p
p and p < f < 2p.
p Put a = q -p
p and b = f - p
p. Then 0 < a < p and 0 < b < p.
p
By case I,

cos(q + f ) = cos(p + a + p + b )

= cos(2p + a + b )

= cos(a + b )

= cos a cos b - sin a sin b

= cos(p + a ) cos(p + b ) - sin (p + a ) sin (p + b )

= cos q cos f - sin q sin f


Thus, Eq. (1.2) is proved in all cases. Finally

cos( x + y) = cos(2 mp + q + 2 np + f )

= cos(2(m + n)p + q + f )

= cos(q + f )

= cos q cos f - sin q sin f

= cos(2 mp + q ) cos(2 np + f ) - sin(2 mp + q ) sin(2 np + f )

= cos x cos y - sin x sin y


This completes the proof of the theorem. ■
1.4 Trigonometric Ratios of Compound Angles 33

T H E O R E M 1.12 For any real numbers x and y,


sin(x + y) = sin x cos y + cos x sin y
PROOF Let x, y Î. From Theorem 1.11, we have

æp ö æp ö æp ö
cos ç + x + y÷ = cos ç + x÷ cos y - sin ç + x÷ sin y
è2 ø è2 ø è2 ø

Therefore
-sin( x + y) = - sin x cos y - cos x sin y
sin( x + y) = sin x cos y + cos x sin y ■

C OROLLARY 1.5 For any real numbers x and y,


cos( x - y) = cos x cos y + sin x sin y
and sin( x - y) = sin x cos y - cos x sin y
PROOF This follows from Theorems 1.11 and 1.12 by substituting -y
- for y. ■

T H E O R E M 1.13 If none of x, y and x + y is an odd multiple of p


p/2, then
tan x + tan y
tan( x + y) =
1 - tan x tan y
PROOF Suppose that none of x, y and x + y is an odd multiple of p
p/2, Then cos x, cos y and cos(x + y) are
nonzero. Now,
sin( x + y)
tan( x + y) =
cos( x + y)
sin x cos y + cos x sin y
= (by Theorem 1.11 and Quick Look 3)
cos x cos y - sin x sin y
(sin x cos y / cos x cos y) + (cos x sin y / cos x cos y)
=
(cos x cos y / cos x cos y) - (sin x sin y / cos x cos y)
tan x + tan y
=
1 - tan x tan y ■

T H E O R E M 1.14 Suppose that none of x, y and x + y is an integral multiple of p


p. Then
cot x cot y - 1
cot( x + y) =
cot x + cot y
PROOF Since x, y and x + y are all not of the form np,
p n Î, we have sin x ¹ 0, sin y ¹ 0 and sin(x + y) ¹ 0
and therefore cot x, cot y and cot(x + y) are defined. Now, consider
cot x cot y - 1 (cos x /sin x)(cos y /sin x) - 1
=
cot x + cot y (cos x /sin x) + (co
os y /sin y)
cos x cos y - sin x sin y
=
cos x sin y + cos y sin x
cos( x + y)
= = cot( x + y)
sin( x + y) ■
34 Chapter 1 Trigonometric Ratios and Transformations

Note:
1. The formula for tan(x + y) in terms tan x and tan y given in Theorem 1.13 is valid if and only if tan x and tan y are
defined and tan x tan y ¹ 1, which are equivalent to saying that none of x, y and x + y is an odd multiple of p/2.
p
2. The formula for cot(x + y) in terms of cot x and cot y given in Theorem 1.14 is valid if and only if cot x and cot y are
defined and cot x + cot y ¹ 1, which are equivalent to saying that none of x, y and x + y is an integral multiple of p.
p

C OROLLARY 1.6 1. If none of x, y and x - y is an odd multiple of p


p/2, then

tan x - tan y
tan( x - y) =
1 + tan x tan y
2. If none of x, y and x - y is an integral multiple of p
p, then
cot x cot y + 1
cot( x - y) =
cot y - cot x
PROOF These follow from Theorems 1.13 and 1.14 by substituting -y
- for y. ■

T H E O R E M 1.15 The following hold good for any real numbers x and y.
1. sin( x + y) sin( x - y) = sin x - sin y = cos y - cos x
2 2 2 2

2. cos( x + y) cos( x - y) = cos2 x - sin2 y = cos2 y - sin2 x


PROOF Let x and y be any real numbers.
1. sin( x + y) sin( x - y) = (sin x cos y + cos x sin y) × (sin x cos y - cos x sin y)

= sin2 x cos2 y - cos2 x sin2 y


= sin2 x(1 - sin2 y) - (1 - sin2 x) sin2 y
= sin2 x - sin2 y
= (1 - cos2 x) - (1 - cos2 y)
= cos2 y - cos2 x
2. cos( x + y) cos( x - y) = (cos x cos y - sin x sin y) × (cos x cos y + sin x sin y)

= cos2 x cos2 y - sin2 x sin2 y


= cos2 x(1 - sin2 y) - (1 - cos2 x) sin2 y
= cos2 x - sin2 y
= (1 - sin2 x) - (1 - cos2 y)
= cos2 y - sin2 x ■

T H E O R E M 1.16 1. If none of x, y, x + y and x - y is an odd multiple of p/2,


p

tan2 x - tan2 y
tan( x + y) tan( x - y) =
1 - tan2 x tan2 y

2. If none of x, y, x + y and x - y is an integral multiple of p


p, then

cot2 y cot2 x + 1
cot( x + y) cot( x - y) =
cot2 y - cot2 x
PROOF This is similar to the one given for Theorem 1.15. ■
1.4 Trigonometric Ratios of Compound Angles 35

T H E O R E M 1.17 For any real numbers x, y and z,

sin( x + y + z) = sin x cos y cos z + cos x sin y cos z + cos x cos y sin z - sin x sin y sin z
and
cos( x + y + z) = cos x cos y cos z - cos x sin y sin z - sin x cos y sin z - sin x sin y cos z
PROOF Consider

sin( x + y + z) = sin x cos( y + z) + cos x sin( y + z)


= sin x(cos y cos z - sin y sin z) + cos x(sin y cos z + cos y sin z)
= sin x cos y cos z + cos x sin y cos z + cos x cos y sin z - sin x sin y sin z

The expansion for cos(x + y + z) can be similarly proved. ■

T H E O R E M 1.18 Let none of x, y, z and x + y + z be an odd multiple of p


p/2 and at least one of x + y, y + z and z + x
be not an odd multiple of pp/2. Then

S tan x - p tan x
tan( x + y + z) =
1 - S tan x tan y

where

S tan x = tan x + tan y + tan z

S tan x tan y = tan x tan y + tan y tan z + tan z tan x

and p tan x = tan x tan y tan z


PROOF Suppose that none of x, y, z, x + y and x + y + z is an odd multiple of p
p/2. Then

tan( x + y + z) = tan[( x + y) + z]

tan( x + y) + tan z
=
1 - tan( x + y)tan z

[(tan x + tan y)/(1 - tan x tan y)] + tan z


=
1 - [(tan x + tan y)/(1 - tan x tan y)]tan z

tan x + tan y + tan z - tan x tan y tan z


=
1 - tan x tan y - tan y tan z - tan z tan x

S tan x - p tan x
=
1 - S tan x tan y ■

T H E O R E M 1.19 If none of x, y, z and x + y + z is an integral multiple of p and at least one of x + y, y + z and z + x


is not an integral multiple of p
p, then

S cot x - p cot x
cot( x + y + z) =
1 - S cot x cot y
36 Chapter 1 Trigonometric Ratios and Transformations

Example 1.21

Evaluate sin 15°, cos 15° and tan 15°. tan 15° = tan(60° - 45°)

Solution: tan 60° - tan 45°


=
1 + tan 60° tan 45°
sin 15° = sin(60° - 45°)
3-1
= sin 60° cos 45° - cos 60° sin 45° =
1+ 3
3 1 1 1 3 -1
= × - × = ( 3 - 1)2
2 2 2 2 2 2 =
(1 + 3 )( 3 - 1)
cos 15° = cos(60° - 45°)
4-2 3
= cos 60° cos 45° + sin 60° sin 45° =
2
1 1 3 1 3+1 =2- 3
= × + × =
2 2 2 2 2 2

Example 1.22

cos 42° + cos 78° + cos 162° = 0 = cos 60° cos 18° + sin 60° sin 18°
+ cos 60° cos 18° - sin 60° sin 18° - cos 18°°
Solution:
cos 42° + cos 78° + cos 162° æ 1ö
= 2 ç ÷ cos 18° - cos 18° = 0
è 2ø
= cos(60° - 18°) + cos(60° + 18°) + cos(180° - 18°)

Example 1.23

æp ö æp ö cot(p / 4)cot q - 1 cot(p / 4)cot q + 1


Prove that cot ç + q ÷ cot ç - q ÷ = 1 = ×
è4 ø è4 ø cot(p / 4) + cot q cot q - cot(p / 4)
Solution: cot q - 1 cot q + 1
= × =1
æp ö æp ö cot q + 1 cot q - 1
cot ç + q ÷ cot ç - q ÷
è4 ø è4 ø

1.5 | Trigonometric Ratios of Multiple and Submultiple Angles


In the formulae for cos(x + y) and sin(x + y) if we substitute x for y, we get formulae for cos 2x and sin 2y 2 . If x is an
angle, then 2x, 3x, 4x, ¼ are called multiple angles of x and x/2, x/3, 2x/3, 2x/5, ¼ are called submultiple angles of x. In
this section, we derive formulae for the trigonometric ratios of multiple and submultiple angles.

T H E O R E M 1.20 For any real number x,

sin 2 x = 2 sin x cos x

and cos 2 x = cos2 x - sin2 x = 2 cos2 x - 1 = 1 - 2 sin2 x


PROOF Let x be any real number. Then
sin 2 x = sin( x + x) = sin x cos x + cos x sin x = 2 sin x cos x
1.5 Trigonometric Ratios of Multiple and Submultiple Angles 37

and cos 2x = cos( x + x) = cos x cos x - sin x sin x


= cos2 x - sin2 x
= cos2 x - (1 - cos2 x) = 2 cos2 x - 1
= (1 - sin2 x) - sin2 x = 1 - 2 sin2 x ■

By substituting x/2 for x in the above, we get the following.

C OROLLARY 1.7 For any real number x,


x x
sin x = 2 sin cos
2 2
x x x x
and cos x = cos2 - sin2 = 2 cos2 - 1 = 1 - 2 sin2
2 2 2 2

T H E O R E M 1.21 1. If x and 2x are not odd multiples of p


p/2, then
2 tan x
tan 2 x =
1 - tan2 x
2. If 2x is not an integral multiple of p
p, then
cot2 x - 1
cot 2 x =
2 cot x
or 2 cot 2 x = cot x - tan x
tan x + tan x
PROOF 1. tan(2 x) = tan( x + x) =
1 - tan x × tan x
2 tan x
=
1 - tan2 x

cot x cot x - 1 cot2 x - 1


2. cot(2 x) = cot( x + x) = =
cot x + cot x 2 cot x
Therefore

cot2 x - 1
2 cot 2 x = = cot x - tan x
cot x ■

C OROLLARY 1.8 1. If x and x/2 are not odd multiples of p


p/2, then
2 tan( x / 2)
tan x =
1 - tan2 ( x / 2)
2. If x is not an integral multiple of p
p, then
cot2 ( x / 2) - 1
cot x =
2 cot( x / 2)
x x
or 2 cot x = cot - tan
2 2
38 Chapter 1 Trigonometric Ratios and Transformations

T H E O R E M 1.22 Let x be any real number. Then


sin 3 x = 3 sin x - 4 sin3 x
and cos 3 x = 4 cos3 x - 3 cos x
PROOF We have
sin 3 x = sin(2 x + x)
= sin 2 x × cos x + cos 2 x × sin x
= 2 sin x cos x cos x + (1 - 2 sin2 x) sin x
= 2 sin x(1 - sin2 x) + (1 - 2 sin2 x) sin x
= 3 sin x - 4 sin3 x
cos 3 x = cos(2 x + x)
= cos 2 x cos x - sin 2 x sin x
= (2 cos2 x - 1) cos x - (2 sin x cos x) sin x
= 2 cos3 x - cos x - 2 sin2 x cos x
= 2 cos3 x - cos x - 2(1 - cos2 x) cos x
= 4 cos3 x - 3 cos x ■

T H E O R E M 1.23 1. If 3x is not an odd multiple of p


p/2, then
3 tan x - tan3 x
tan 3 x =
1 - 3 tan2 x
2. If 3x is not an integral multiple of p
p, then

3 cot x - cot3 x
cot 3 x =
1 - 3 cot2 x
PROOF 1. By hypothesis none of x and 3x is an odd multiple of p
p/2. Then
tan 3 x = tan( x + 2 x)
tan x + tan 2 x
=
1 - tan x tan 2 x
tan x + [2 tan x /(1 - tan2 x)]
=
1 - tan x[2 tan x /(1 - tan2 x)]
(1 - tan2 x) tan x + 2 tan x
=
(1 - tan2 x) - 2 tan2 x
3 tan x - tan3 x
=
1 - 3 tan2 x ■

2 is an odd multiple of p
Note that the above formula for tan 3x is valid even if 2x p/2. Similarly, formula (2) for cot 3x can also
2 is an integral multiple of p
be proved. Note that (2) remains valid even if 2x p. If 3x is not integral multiple of p
p, then so is x.

T H E O R E M 1.24 1. For any real number x,


1 - cos 2 x
sin x = ±
2
1.5 Trigonometric Ratios of Multiple and Submultiple Angles 39

1 + cos 2 x
and cos x = ±
2
2. If x is not an odd multiple of p
p/2, then
1 - cos 2 x
tan x = ±
1 + cos 2 x

The signs ± for sin x, cos x and tan x in the above formulae can be determined depending on the
quadrants in which the angle x lies.
PROOF From Theorem 1.20,
1 - cos 2 x = 2 sin2 x
and therefore

1 - cos 2 x
sin x = ±
2

Similarly, the other equalities can also be proved. ■

By substituting x/2 for x in the above theorem, we get the following corollary.

C OROLLARY 1.9 1. For all real numbers x,


x 1 - cos x
sin =±
2 2

x 1 + cos x
and cos =±
2 2
If x/2 is not an odd multiple of p
p/2, then

x 1 - cos x
tan =±
2 1 + cos x

The signs ± in the above can be determined depending on the quadrant in which the angle x/2 lies.

T H E O R E M 1.25 1. If x is not an odd multiple of p


p/2, then
2 tan x
sin 2 x =
1 + tan2 x

1 - tan2 x
and cos 2 x =
1 + tan2 x
2. If neither x nor 2x is an odd multiple of p
p/2, then
2 tan x
tan 2 x =
1 - tan2 x
PROOF 1. We have
sin 2 x = 2 sin x cos x
2(sin x /cos x)
=
1/cos2 x
2 tan x 2 tan x
= =
sec2 x 1 + tan2 x
40 Chapter 1 Trigonometric Ratios and Transformations

and cos 2 x = cos2 x - sin2 x

æ sin2 x ö
= cos2 x × ç 1 -
è cos2 x ÷ø
1
= (1 - tan2 x)
sec2 x
1 - tan2 x
=
1 + tan2 x
2. From (1) it follows that
sin 2 x 2 tan x
tan 2 x = =
cos 2 x 1 - tan2 x ■

x
On substituting for x in the above, we get the following corollary.
2

C OROLLARY 1.10 1. If x/2 is not an odd multiple of p


p/2, then
2 tan(x / 2) 1 - tan2 ( x / 2)
sin x = and cos x =
1 + tan2 ( x / 2) 1 + tan2 ( x / 2)

2. If neither x nor x/2 is an odd multiple of p


p/2, then
2 tan(x / 2)
tan x =
1 - tan2 ( x / 2)

Example 1.24

Determine the values of all the trigonometric ratios at cos 18° = 1 - sin2 18°
18°, 36°, 54° and 72°.
2
æ 5 - 1ö
Solution: Put x = 18°. Then 2x + 3x = 90°. Therefore = 1- ç ÷
è 4 ø
sin 2 x = sin(90° - 3 x) = cos 3 x
16 - (5 + 1 - 2 5 )
2 sin x cos x = 4 cos3 x - 3 cos x =
16
cos x(2 sin x - 4 cos2 x + 3) = 0
10 + 2 5
=
Since x = 18°, cos x ¹ 0. Therefore 4

2 sin x - 4 cos2 x + 3 = 0 1 4
cosec 18°° = =
sin 18° 5 -1
2 sin x - 4(1 - sin2 x) + 3 = 0
1 4
4 sin2 x + 2 sin x - 1 = 0 sec 18° = =
cos 18° 10 + 2 5
Solving this quadratic equation in sin x, we get sin 18° 5 -1
tan 18° = =
-2 ± 4 + 16 ± 5 - 1 cos 18° 10 + 2 5
sin x = =
8 4
cos 18° 10 + 2 5
cot 18° = =
18° > 0, we get that sin 18° 5 -1

5 -1 é 10 + 2 5 ù
sin 18° = cos 36° = 2 cos2 18° - 1 = 2 ê ú-1
4 ë 16 û
1.5 Trigonometric Ratios of Multiple and Submultiple Angles 41

2+2 5 5+1 5+1


= = sin 54° = sin(90° - 36°) = cos 36° =
8 4 4

æ 5 + 1ö
2
10 + 2 5
sin 36° = 1 - cos 36° = 1 - ç
2 sin 72° = cos 18° =
÷ 4
è 4 ø
10 - 2 5
16 - (6 + 2 5 ) 10 - 2 5 cos 54° = sin 36° = , etc.
= = 4
4 4

By using these, we get

5 -1
cos 72° = cos(90° - 18°) = sin 18° =
4

Example 1.25

Determine the value of Therefore


p 3p 5p 7p p 3p 5p 7p
sin4 + sin4 + sin4 + sin4 sin4 + sin4 + sin4 + sin4
8 8 8 8 8 8 8 8
æpö æpö æpö æpö
Solution: We have = sin4 ç ÷ + cos4 ç ÷ + cos4 ç ÷ + sin4 ç ÷
è 8ø è 8ø è 8ø è 8ø
3p æp pö p p
sin = sin ç - ÷ = cos = 2(sin4 x + cos4 x), where x =
8 è 2 8ø 8 8
= 2(sin2 x + cos2 x)2 - 4 sin2 x cos2 x
5p æ 3p ö 3p p
sin = sin ç p - ÷ = sin = cos
8 è 8ø 8 8 = 2 - (2 sin x cos x)2
7p æ pö p = 2 - (sin 2 x)2
sin = sin ç p - ÷ = sin
8 è 8ø 8 2
æ pö
= 2 - ç sin ÷
è 4ø
2
æ 1 ö 1 3
= 2-ç = 2- =
è 2 ÷ø 2 2

p Also, we have -1 £ sin x £ 1 for


Let us recall that the sine and cosine functions are periodic with minimum period 2p.
all x Î and
p 3p
sin =1 and sin = -1
2 2
Therefore, 1 and -1 are the extreme values of sin x, 1 being the maximum value and -1 being the minimum value.
Similarly, 1 and -1 are the extreme values of cos x; note that cos 0 = 1 and cos p = -1.
The trigonometric functions other than cosine and sine are not bounded and hence they do not have extreme
values. In this context, we have the following.

T H E O R E M 1.26 Let a, b and c be any real numbers and


f ( x) = a cos x + b sin x + c

for all x Î. Then f(


f x) has extreme values and c + a2 + b2 and c - a2 + b2 are its maximum and
minimum values, respectively.
42 Chapter 1 Trigonometric Ratios and Transformations

PROOF Put a = r cos q and b = r sin q, so that a2 + b2 = r . Now, consider

f ( x) = a cos x + b sin x + c
æ a bö
= r (cos q cos x + sin q sin x) + c ç where r = a2 + b2 , cos q = , sin q = ÷
è r rø
= r cos ( x - q ) + c

Since -1 £ cos(x - q)
q £ 1, we have

-r £ r cos( x - q ) £ r

Therefore

-r + c £ r cos( x - q ) + c £ r + c

That is,

c - a2 + b2 £ f ( x) £ c + a2 + b2

Also,

f (q ) = c + a2 + b2 and f (p + q ) = c - a2 + b2

Thus c + a2 + b2 and c - a2 + b2 are the extreme values of a cos x + b sin x + c, x Î . ■

Example 1.26

Find the extreme values of This is of the form a cos x + b sin x + c, where a = 13/2,
b = -3 3 / 2 and c = 8. We have
æ pö
5 cos x + 3 cos ç x + ÷ + 8, x Î 
è 3ø 2
æ 13 ö æ -3 3ö
2

a2 + b2 = ç ÷ + ç ÷ =7
Solution: Let è 2ø è 2 ø

æ pö From Theorem 1.26, 8 + 7 (= 15) and 8 - 7 (= 1) are the


f ( x) = 5 cos x + 3 cos ç x + ÷ + 8
è 3ø maximum and minimum values of f( f x), respectively.
æ p pö
= 5 cos x + 3 ç cos x cos - sin x sin ÷ + 8
è 3 3ø

æ 1 3ö
= 5 cos x + 3 ç cos x × - sin x × ÷ + 8
è 2 2 ø

13 3 3
= cos x - sin x + 8
2 2

Example 1.27

Find the period of sin 5x/3. p 3 6p


= 2p × = = 216°
5/ 3 5 5
p the period
Solution: Since the period p of sin x is 2p,
of sin 5x/3 is
1.5 Trigonometric Ratios of Multiple and Submultiple Angles 43

Example 1.28

Find the period of the function sin4 x + cos4 x. 1 æ 1 - cos 4 x ö


= 1- ç ÷ø
2è 2
Solution: We have
3 1
sin4 x + cos4 x = (sin2 x + cos2 x)2 - 2 sin2 x cos2 x = + cos 4 x
4 4
1 Therefore the required period is
= 1 - (sin 2 x)2
2
1 2p p
(period of cos x) = =
4 4 2

Example 1.29

For any real number x, express cos 5x in terms of cos x. = (8 cos5 x - 10 cos3 x + 3 cos x)

Solution: We have - 2 cos x(1 - cos2 x)(3 - 4(1 - cos2 x))


cos 5 x = cos (3 x + 2 x) = (8 cos5 x - 10 cos3 x + 3 cos x)
= cos 3 x cos 2 x - sin 3 x sin 2 x - 2 cos x(5 cos2 x - 4 cos4 x - 1)
= (4 cos3 x - 3 cos x)(2 cos2 x - 1) = 16 cos5 x - 20 cos3 x + 5 cos x
- (3 sin x - 4 sin3 x)(2 sin x cos x)
= (8 cos5 x - 10 cos3 x + 3 cos x)
- 2 co
os x sin2 x(3 - 4 sin2 x)

Recall that, in Corollary 1.9, we have expressed sin x/2 and cos x/2 in terms of cos x. In the following we express these
in terms of sin x.

T H E O R E M 1.27 For any real number x,


x
2 sin = ± 1 + sin x ± 1 - sin x
2
x
and 2 cos = ± 1 + sin x ∓ 1 - sin x
2
PROOF From Theorem 1.20, we have
x x
2 sin cos = sin x (1.3)
2 2
Also, we have
x x
sin2 + cos2 = 1 (1.4)
2 2
Adding Eqs. (1.3) and (1.4), we get
x x x x
sin2 + 2 sin cos + cos2 = 1 + sin x
2 2 2 2
2
æ x xö
çè sin + cos ÷ø = 1 + sin x
2 2
44 Chapter 1 Trigonometric Ratios and Transformations

x x
or sin + cos = ± 1 + sin x (1.5)
2 2
Again subtracting Eq. (1.3) from Eq. (1.4), we get
x x x x
sin2 - 2 sin cos + cos2 = 1 - sin x
2 2 2 2
Therefore
2
æ x xö
çè sin - cos ÷ø = 1 - sin x
2 2
x x
or sin - cos = ± 1 - sin x (1.6)
2 2

Adding Eqs. (1.5) and (1.6), we get


x
2 sin = ± 1 + sin x ± 1 - sin x
2

Subtracting Eq. (1.6) from Eq. (1.5), we get


x
2 cos = ± 1 + sin x ∓ 1 - sin x
2 ■

In the formulae for sin x/2 and cos x/2 in terms of sin x given in the above theorem, there are two ambiguous signs.
In the following two examples, we have demonstrated how to determine the ambiguity in any particular case.

Example 1.30

Find the values of sin 15° and cos 15°. Since sin 15° and cos 15° are both positive and since
cos 15° > sin 15°, we get that sin 15° + cos 15° is positive
Solution: By taking x = 30°, we get from the proof of and sin 15° - cos 15° is negative. Therefore, we have
Theorem 1.27 that
3
sin 15° + cos 15° = ± 1 + sin 30° sin 15° + cos 15° = +
2
1 3 and
= ± 1+ =±
2 2 1
sin 15° - cos 15° = -
and sin 15° - cos 15° = ± 1 - sin 30° 2

1 Therefore

2 3 -1 3+1
sin 15° = and cos15° =
2 2 2 2

Example 1.31

p/6) and deduce the values of


p
p
sin(19p/12) and cos(19p/12).
p
æ 19p ö
sin ç = sin(p + 30°)
Solution: Note that è 6 ÷ø

19p pö 1
æ = -sin 30° = -
= 2p + ç p + ÷ 2
6 è 6ø
1.5 Trigonometric Ratios of Multiple and Submultiple Angles 45

Also, Therefore

19p 19p 19p 19p


= 570° and = 285° (a) sin + cos is negative
6 12 12 12
19p 19p
As in Example 1.30, we get (b) sin - cos is also negative
12 12
19p 19p 19p
sin + cos = ± 1 + sin Hence
12 12 6
19p 19p 1
1 1 sin + cos =-
= ± 1- =± 12 12 2
2 2
and 19p 19p 3
and sin - cos =-
12 12 2
19p 19p 19p
sin - cos = ± 1 - sin Therefore
12 12 6


3 19p 1+ 3
sin =-
2 12 2 2

p
Since 19p/12 > 3p/2,
p 19pp/12 is in the fourth quadrant
and hence its cosine is positive and its sine is negative. 19p 3 -1
and cos =
p
Therefore, sin 19p/12 < 0 and cos 19p/12
p > 0. Also, 12 2 2
19p 19p 19p
- sin = sin > cos
12 12 12

Note: In general, the ambiguity of the signs ± in the formulae for sin x/2 and cos x/2 in the terms sin x can be cleared
as follows.
We have
x x æ 1 x 1 xö
sin + cos = 2 ç sin + cos ÷
2 2 è 2 2 2 2ø
æ x p x pö
= 2 ç sin cos + cos sin ÷
è 2 4 2 4ø
æ p xö
= 2 sin ç + ÷
è 4 2ø

æ p xö
sin ç + ÷ > 0
è 4 2ø

p + (x/2) lies between 2np


if (p/4) p and 2np + p
p. That is if
p x 3p
2 np - < < 2 np +
4 2 4

x x x p p
(a) sin + cos is positive if lies between 2 np - and 2 np +
2 2 2 4 4
(b) It is negative otherwise.
Similarly we can prove that
x x æx pö
sin - cos = 2 sin ç - ÷
2 2 è 2 4ø
46 Chapter 1 Trigonometric Ratios and Transformations

and therefore
(a) sin x/2 - cos x/2 is positive if x/2 - p/4 lies between 2npp and 2np + p (i.e., if x/2 lies between 2np + p/4 and
2np + 5p/4).
(b) It is negative otherwise.
These results are displayed in Figure 1.35 in which OXY is the rectangular coordinate system and OP, OQ, OR and
OS bisect the angles in the first, second, third and fourth quadrants, respectively.

Q P
sin x/2 + cos x/2 > 0

sin x/2 - cos x/2 > 0

sin x/2 + cos x/2 < 0 sin x/2 + cos x/2 > 0
0 X
sin x/2 - cos x/2 > 0 sin x/2 - cos x/2 < 0

sin x/2 + cos x/2 < 0

sin x/2 - cos x/2 < 0

R S

FIGURE 1.35

Example 1.32

2 sin( x / 2) = - 1 + sin x - 1 - sin x , determine the x x


sin + cos = - 1 + sin x
interval in which x/2 lies. 2 2
x x
and sin - cos = - 1 - sin x
Solution: From Theorem 1.27 and from our hypothesis, 2 2
we should have
From Figure 1.35 it follows that x/2 lies between
2np - 3p/4
p and 2np - p
p/4 (between OR and OS).

T H E O R E M 1.28 If neither x nor x/2 is an odd multiple of p


p/2, then

x ± 1 + tan x - 1
2

tan =
2 tan x
PROOF From part (2) of Corollary 1.10, we have
2 tan(x /2)
tan x =
1 - tan2 (x /2)
æ x ö 2 tan(x /2)
1 - tan2 ç ÷ =
è 2ø tan x
1.6 Sum and Product Transformations 47

æ xö 1 æ xö 1 1 1 + tan2 x
tan2 ç ÷ + 2 × tan ç ÷ + =1+ =
è 2ø tan x è 2 ø tan x
2 2
tan x tan2 x
2
æ x 1 ö 1 + tan2 x
çè tan + =
2 tan x ÷ø tan2 x

x 1 ± 1 + tan2 x
tan + =
2 tan x tan x

x ± 1 + tan x - 1
2

tan =
2 tan x ■

The ambiguity of ± in the above formula can only be determined when we know about the magnitude of x. Consider
the following example.

Example 1.33

It is given that sec 15° = 2 - 3 (see Example 1.31). Find Since tan 15° = tan(p + 15°) = tan(195°), we get from the
tan 7(1/2)° and cot 7(1/2)°. above

Solution: Taking x = 15° in Theorem 1.28, we have 195° ±( 6 - 2 ) - 1


tan =
2 2- 3
1° ± 1 + ( 2 - 3 ) - 1
2

tan 7 =
2 2- 3
195° æp 1° ö
± 8-4 3 -1 ±( 6 - 2 ) tan = tan ç + 7 ÷ < 0
= = 2 è2 2ø
2- 3 2- 3
Therefore
Since tan 7(1/2)° is positive, we must have
195° -( 6 - 2 ) - 1
tan =
1° + 8 - 4 3 - 1 2 2- 3
tan 7 =
2 2- 3
= (- 6 + 2 - 1)(2 + 3 )
( 6 - 2) - 1
= = - ( 2 + 3 )( 2 + 1)
2- 3
1° æp 1° ö
= ( 6 - 2 - 1)(2 + 3 ) cot 7 = - tan ç + 7 ÷
2 è2 2ø
= 6- 3+ 2-2 195°
= - tan = ( 2 + 3 )( 2 + 1)
2
= ( 3 - 2 )( 2 - 1)

1.6 | Sum and Product Transformations


In this section, we derive certain formulae through which we can transform the sum or difference of two trigonometric
ratios into products and vice versa. First we take up transformation of sum into product of trigonometric ratios.

T H E O R E M 1.29 The following hold good for any real numbers x and y.
1. sin(x + y) + sin(x - y) = 2 sin x cos y
2. sin(x + y) - sin(x - y) = 2 cos x sin y
3. cos(x + y) + cos(x - y) = 2 cos x cos y
4. cos(x + y) - cos(x - y) = -2 sin x sin y
48 Chapter 1 Trigonometric Ratios and Transformations

PROOF This is a simple consequence of the following identities:


1. sin(x + y) = sin x cos y + cos x sin y
2. sin(x - y) = sin x cos y - cos x sin y
3. cos(x + y) = cos x cos y - sin x sin y
4. cos(x - y) = cos x cos y + sin x sin y ■

Try it out Prove Theorem 1.29.

By taking
a+b a-b
x= and y=
2 2
in the above, we get the following theorem (note that x + y = a and x – y = b).

æ a + bö æ a - bö
T H E O R E M 1.30 1. sin a + sin b = 2 sin ç ÷ cos ç
è 2 ø è 2 ÷ø
æ a + bö æ a - bö
2. sin a - sin b = 2 cos ç sin ç
è 2 ÷ø è 2 ÷ø
æ a + bö æ a - bö
3. cos a + cos b = 2 cos ç cos ç
è 2 ÷ø è 2 ÷ø
æ a + bö æ a - bö
4. cos a - cos b = - 2 sin ç ÷ sin ç
è 2 ø è 2 ÷ø
æ a + bö æ a - bö
5. cos b - cos a = 2 sin ç ÷ sin ç
è 2 ø è 2 ÷ø

Try it out Prove Theorem 1.30.

QUICK LOOK 6

Regarding formulae given in Theorem 1.29 for 3. 2 cos x cos y = cos( x + y) + cos( x - y)
transforming a sum into a product, the following may
= cos(sum) + cos(difference)
be noted and used as tips to remember the formulae.
1. 2 sin x cos y = sin( x + y) + sin( x - y) 4. -2 sin x sin y = cos( x + y) - cos( x - y)

= sin(sum) + sin(difference) = cos(sum) - cos(difference)

2. 2 cos x sin y = sin( x + y) - sin( x - y) Therefore


= sin(sum) - sin(difference) 2 sin x sin y = cos(difference) - cos(sum)

Example 1.34

Find the value of 4(cos 66° + sin 84°). æ æ 66° + 6° ö æ 66° - 6° ö ö


= 4 ç 2 cos ç ÷ø cos çè ÷
è è 2 2 ø ÷ø
Solution:
= 8 cos 36° cos 30°
æ æp öö
4(cos 66° + sin 84°) = 4 ç cos 66° + sin ç - 6°÷ ÷ æ 5 + 1ö æ 3 ö
è è2 øø = 8ç ÷ç ÷ = 3 + 15
= 4(cos 66° + cos 6°) è 4 øè 2 ø

(See Example 1.24 for cos 36°.)


1.6 Sum and Product Transformations 49

Example 1.35

Prove that for any real numbers x, y and z,


cos ( y + z - x) - cos (z + x - y) + cos ( x + y - z) cos ( x + y - z) - cos ( x + y + z)
- cos ( x + y + z) = 4 sin x cos y sin z æ ( x + y + z) + ( x + y - z) ö
= 2 sin ç ÷ø
è 2
Solution: We have
æ ( x + y + z) - ( x + y - z) ö
cos( y + z - x) - cos(z + x - y) sin ç ÷ø
è 2
æ (z + x - y) + ( y + z - x) ö = 2 sin( x + y) sin z
= 2 sin ç ÷ø
è 2
Therefore
æ (z + x - y) - ( y + z - x) ö cos ( y + z - x) - cos (z + x - y) + cos ( x + y + z)
sin ç ÷ø
è 2
= 2 sin z sin ( x - y) + 2 sin ( x + y) sin z
= 2 sin z sin( x - y)
= 2 sin z [sin ( x + y) + sin ( x - y)]
= 2 sin z(2 sin x sin y)
= 4 sin x cos y sin z

Example 1.36

Prove that cos 20° × cos 40° × cos 60° × cos 80° = 1/16. 1 æ -1 ö
= cos 20° ç + cos 40°÷
4 è 2 ø
Solution: Consider
-1 1
= cos 20° + (2 cos 40° cos 20°)
cos 20°× cos 40°× cos 60°× cos 80° 8 8
1 æ 1ö æ 1ö æ -1ö
= (2 cos 80°× cos 40°) × cos 20° ç ÷ çè since cos 60° = ÷ø çè since cos 120° = ÷ø
2 è 2ø 2 2
1 -1 1
= cos 20°(cos (80° + 40°) + cos(80° - 40°)) = cos 20° + (cos 60° + cos 20°)
4 8 8
1 1 1 1 1
= cos 20°(cos 120° + cos 40°) = cos 60° = × =
4 8 8 2 16

Hence proved.

Example 1.37

If A, B, C are angles of a triangle, prove that = - 2 cos C sin( A - B) + 2 sin C cos C


sin 2 A - sin 2 B + sin 2C = 4 cos A sin B cos C = 2 cos C[sin C - sin( A - B)]
= 2 cos C[sin{p - ( A + B)} - sin( A - B)]
Solution: Note that
= 2 cos C (2 cos A sin B)
A + B + C = 180° = p
= 4 cos A sin B cos C
Now
(sin 2 A - sin 2 B) + sin 2C
= 2 cos( A + B) sin( A - B) + 2 sin C cos C
= 2 cos(p - C ) sin( A - B) + 2 sin C cos C
50 Chapter 1 Trigonometric Ratios and Transformations

Example 1.38

If A + B + C = p
p, prove that C A-B C A+ B
= 2 cos cos + 2 cos cos
A B C 2 2 2 2
sin A + sin B + sin C = 4 cos cos cos
2 2 2 Cæ A-B A + Bö
= 2 cos ç cos + cos ÷
2è 2 2 ø
Solution: We have
Cæ A Bö
= 2 cos ç 2 cos cos ÷ø
A+ B A-B 2è 2 2
(sin A + sin B) + sin C = 2 sin cos
2 2 A B C
C C = 4 cos cos cos
+ 2 sin cos 2 2 2
2 2
Hence proved.

Example 1.39

If A + B + C = p
p, then prove that (ii) Consider
(i) tan A + tan B + tan C = tan A tan B tan C A+ B p
= -C
A B B C C A 2 2
(ii) tan tan + tan tan + tan tan = 1
2 2 2 2 2 2
It gives
(iii) When tan A, tan B, tan C are not equal to zero, then
æ A + Bö C
cot A cot B + cot B cot C + cot C cot A = 1 tan ç ÷ = cot
è 2 ø 2
and
tan( A / 2) + tan ( B / 2) 1
=
A B C A B C 1 - tan( A / 2) tan( B / 2) tan(C / 2)
cot cot cot = cot + cot + cot
2 2 2 2 2 2 B C
å tan tan = 1
2 2
Solution:
(i) If tan(A + B) = tan(p - C) = -tan C, then (iii) follows from (i) and (ii).
tan A + tan B
= - tan C
1 - tan A tan B
tan A + tan B + tan C = tan A tan B tan C

QUICK LOOK 7

Values of trigonometrical ratios of some important 1° 1


6. cos 22 = 2+ 2
angles. 2 2
3-1 1°
1. sin 15° = cos 75° = tan 22 = 2 - 1
2 2 2
1
3+1 8. sin 18° = cos 72° = ( 5 - 1)
2. cos 15° = sin 75° = 4
2 2
1
3. tan 15° = cot 75° = 2 - 3 9. cos 18° = sin 72° = 10 + 2 5
4
4. cot 15° = tan 75° = 2 + 3 1
10. sin 36° = cos 54° = 10 - 2 5
1° 1 4
5. sin 22 = 2- 2
2 2 1
11. cos 36° = sin 54° = ( 5 + 1)
4
Worked-Out Problems 51

WORKED-OUT PROBLEMS
Single Correct Choice Type Questions

1. If (sinq + cosecq)
q 2 + (cosq + secq)
q 2 = tan2 q + cot2 q + k, Now
then k equals
1 = cos2 q + sin2 q
(A) 9 (B) 7 (C) 5 (D) 8
= m4 n2 + m2 n4
Solution:
= m2 n2 (m2 + n2 )
LHS = sin q + 2 + cosec q + cos q + 2 + sec q
2 2 2 2

Answer: (D)
= 5 + (1 + cot2 q ) + (1 + tan2 q )
4. If a sec q = 1 - b tan q and a2 sec2 q = 5 + b2 tan2 q, then
= tan2 q + cot2 q + 7
a2b2 + 4a2 = kb2 where value of k is
Answer: (B) (A) 3 (B) 4 (C) 5 (D) 9
Solution: (1 - b tan q)
q 2 = a2 sec2 q = 5 + b2 tan3 q
2. If sin q + sin2 q = 1, then cos12 q + 3cos10 q + 3cos8 q + Therefore
cos6 q is equal to
(A) 0 (B) 2 (C) 1 (D) 4 1 - 2b tan q + b2 tan2 q = 5 + b2 tan2 q

Solution: Given that sin q + sin2 q = 1. This implies -2


tan q =
sin q = cos2 q. b

cos12 q + 3 cos10 q + 3 cos8 q + cos6 q Now

= cos6 q [cos6 q + 3 cos4 q + 3 cos2 q + 1] a2 (1 + tan2 q ) = a2 sec2 q = 5 + b2 tan2 q


æ 4ö æ 4ö
= cos6 q (1 + cos2 q )3 a2 ç 1 + 2 ÷ = 5 + b2 ç 2 ÷
è b ø èb ø
= sin3 (1 + sin q )3 a2 (b2 + 4) = 9b2
= (sin q + sin2 q )3 k=9
=1 Answer: (D)

Answer: (C) 5. If tan2 q = 1 - a2, then the value of secq + tan3 q cosecq is
(A) (2 - a2)1/2 (B) (2 - a2)3/2
3. If cosec q - sin q = m3 and sec q - cos q = n3, then m2n2
(m2 + n2) is equal to (C) (2 + a2)1/2 (D) (2 + a2)3/2
3
(A) 4 (B) 2 2 (C) 2 (D) 1 Solution:
Solution: sec q + tan3 q cosecq = sec q (1 + tan3 q cot q )
1 1 = sec q (1 + tan2 q )
- sin q = m3 and - cosq = n3
sin q cosq = (1 + tan2 q )3 / 2
Therefore = (2 - a2 )3 / 2

cos2 q = m3 sin q and sin2 q = n3 cos q Answer: (B)

Now f n) = sinn q + cosn q, Then


6. For positive integer n, let f(

cos4 q = m6 sin2 q = m6 n3 cos q f (3) - f (5)


=
f (5) - f (7)
Therefore
f (1) f (3) f (2)
cos3 q = m6 n3 and sin3 q = m3 n6 (B) (C) f(2)
f (D)
f (3) f (1) f (3)
52 Chapter 1 Trigonometric Ratios and Transformations

Solution: We have 1
= [sin4 x + cos4 x + 2 sin2 x cos2 x]
12
f (3) - f (5) = (sin3 q + cos3 q ) - (sin5 q + cos5 q )
1
= sin3 q cos2 q + cos3 q sin2 q = (sin2 x + cos2 x)2
12
= sin2 q cos2 q (sin q + cos q ) 1
=
= (sin q cos q ) f (1)
2 2
12

Similarly Answer: (D)


f (5) - f (7) = (sin2 q cos2 q ) f (3) 9. (1 - cos 22°)(1 - cos 23°) equals
Therefore (A) 1 (B) 2 (C) 4 (D) –2
f (3) - f (5) f (1) Solution: It is known that
=
f (5) - f (7) f (3) cot A cot B - 1
cot( A + B) =
Answer: (A) cot A + cot B

f x) = sin x(sin x + sin 3x). Then f(


7. Let f( f x) A = 22°, B = 23°. Therefore
(A) ³ 0 only when x £ 0 cot 22° + cot 23° = cot 22° cot 23° - 1
(B) £ 0 for all real x (1 - cot 22°)(1 - cot 23°) = 2
(C) ³ 0 for all real x
(D) £ 0 only when x £ 0 Answer: (B)

Solution: æ p öæ 3p ö
10. ç 4 cos2 - 3÷ ç 4 cos2 - 3÷ =
è 20 ø è 20 ø
f ( x) = sin2 x + sin x sin 3 x
p p
1 - cos 2 x cos 2 x - cos 4 x (A) sec (B) tan
= + 20 20
2 2 p p
(C) cot (D) cosec
1 20 20
= (1 - cos 4 x)
2
Solution: It is known that
= sin2 2 x ³ 0 for all real x
cos 3 x = 4 cos3 x - 3 cos x
Answer: (C)
Therefore
8. Let cos 3 x
4 cos2 x - 3 =
1 cos x
fn ( x) = (sinn x + cosn x)
n x ¹ (2k + 1)(p /2) where k is an integer. Hence
for n = 1, 2, 3, …. Then f4(x) - f6(x) is equal to
æ 2 p öæ 3p ö cos(3p / 20) cos(9p / 20)
1 1 çè 4 cos - 3÷ ç 4 cos2 - 3÷ = ´
(A) (B) 10 (C) 12 (D) 20 øè 20 ø cos(p / 20) cos(3p / 20)
10 12
cos(9p / 20)
Solution: =
cos(p / 20)
1 1
f4 ( x) - f6 ( x) = (sin4 x + cos4 x) - (sin6 x + cos6 x) cos[(p / 2) - (p / 20)]
4 6 =
cos(p / 20)
1
= [3 sin4 x + 3 cos4 x - 2 sin6 x - 2 cos6 x] p
12 = tan
20
1
= [3 sin4 x + 3 cos4 x - 2 sin4 x(1 - cos2 x) Answer: (B)
12
- 2 cos4 x(1 - sin2 x)]
Worked-Out Problems 53

11. If æp ö æp ö
12. tan a tan ç + a ÷ tan ç - a ÷ is equal to
è3 ø è3 ø
sin4 a cos4 a 1
+ = a
(A) cot 3a (B) cot 2a
a b a+b
(C) tan 3a (D) tan 2a
then
Solution:
sin8 a cos8 a 1
+ =
a3 b3 (a + b)k æp ö æp ö
tan a tan ç + a ÷ tan ç - a ÷
è3 ø è3 ø
where the value of k is
(A) 3 (B) 2 (C) –3 (D) –2 é 3 + tan a ù é 3 - tan a ù
= tan a ê úê ú
Solution: From the hypothesis ë 1 - 3 tan a û ë 1 + 3 tan a û
é 3 - tan2 a ùû
æ sin4 a cos4 a ö = tan a ë
(a + b) ç + = 1 = (sin2 a + cos2 a )2 1 - 3 tan2 a
è a b ÷ø
3 tan a - tan3 a
Therefore =
1 - 3 tan2 a
b 4 a = tan 3a
cos4 a + sin4 a + sin a + cos4 a
a b
Answer: (C)
= sin4 a + 2 sin2 a cos2 a + cos4 a
b 4 a 13. If (1 + tan 1°) (1 + tan 2°)  (1 + tan 45°) = 2n
sin a + cos4 a - 2 sin2 a cos2 a = 0 then n is equal to
a b
2
(A) 22 (B) 23 (C) 21 (D) 24
æ b 2 a ö
ç a sin a - cos2 a ÷ = 0 Solution:
è b ø
(1 + tan k°)[1 + tan(45° - k°)]
Let
é 1 - tan k ° ù
b 4 a = (1 + tan k °) ê1 + ú=2
sin a = cos4 a = l (say) ë 1 + tan k ° û
a b
Then Therefore

2n = [(1 + tan 1°)(1 + tan 44°)][(1 + tan 2°)


1 sin4 a cos4 a l l (a + b)
= + = + =l
a+b a b b a ab (1 + tan 43°)]  [(1 + tan 22°)(1 + tan 23°)] ´ 2

This gives = (2 ´ 2 ´  ´ 22 times) ´ 2 = 223

ab Hence
l=
(a + b)2 n = 223
Hence Answer: (B)
sin8 a cos8 a é a2 b2 ù
3
+ 3
= l2 ê 3 2 + 2 3 ú 14. 2(sin6 q + cos6 q)
q - 3(sin4 q + cos4 q
q) is equal to
a b ëa b ab û
(A) 0 (B) 1 (C) –1 (D) 2
a2 b2 é a + b ù
= Solution:
(a + b)4 êë a2 b2 úû
1 2(sin6 q + cos6 q ) - 3(sin4 q + cos4 q )
=
(a + b)3 = 2[sin2 q + cos2 q ][sin4 q - sin2 q cos2 q + cos4 q ]
Answer: (A) - 3[(sin2 q + cos2 q )2 - 2 sin2 q cos2 q ]
54 Chapter 1 Trigonometric Ratios and Transformations

= 2[(sin2 q + cos2 q )2 - 3 sin2 q cos2 q ] Let

- 3[1 - 2 sin2 q cos2 q ] p 3p 5p


s = sin2 sin2 sin2
14 14 14
= 2(1 - 3 sin2 q cos2 q ) - 3(1 - 2 sin2 q cos2 q )
p 3p 5p
= 2 - 3 = -1 and t = sin sin sin
14 14 14
Answer: (C)
Now
15. The value of the expression
æpö æ p 6p ö 6p
sin ç ÷ = sin ç - ÷ = cos
3 1 è 14 ø è 2 14 ø 14
-
sin 20° cos 20° æ 8p ö æ 8p ö
= cos ç p - ÷ = - cos çè ÷ø
è 14 ø 14
2 sin 20° 2 sin 20° æ 3p ö æ p 4p ö 4p
(A) (B) sin ç ÷ = sin ç - = cos
cos 40° sin 40° è 14 ø è 2 14 ÷ø 14
(C) 2 (D) 4
æ 5p ö æ p 2p ö 2p
sin ç ÷ = sin ç - ÷ = cos
Solution: è 14 ø è 2 14 ø 14

3 1 3 cos 20° - sin 20° Therefore


- =
sin 20° cos 20° sin 20° cos 40°
8p 4p 2p
2[( 3 / 2)cos 20° - 1/ 2 sin 20°] t = - cos × cos cos
= 14 14 14
sin 20° cos 20°
2p 4p 8p
2[sin 60° cos 20° - cos 60° sin 20°] = - cos cos × cos
= 14 14 14
sin 20° cos 20°
-[2 sin(2p / 14)cos(2p / 14)] 4p 8p
2 sin(60° - 20°) = × cos × cos
= 2 sin(2p / 14) 14 14
sin 20° cos 20°
2 sin 40° -1 é 4p 4p ù 8p
= = sin cos ú cos
sin 20° cos 20° 2 sin(2p / 14) êë 14 14 û 14
2(2 sin 20° cos 20°)
= -1 æ 8p ö 8p
sin 20° cos 20° = ç sin ÷ø cos
22 sin(2p / 14) è 14 14
=4
Answer: (D) - sin(16p / 14) - sin(8p / 7) 1
= = =
8 sin(2p / 14) 8 sin(p / 7) 8
16. The value of
Hence
p 3p 5p 7p 9p 11p 13p 2
sin sin sin sin sin sin sin æ 1ö 1
14 14 14 14 14 14 14 s = t2 = ç ÷ =
è 8ø 64
is Answer: (B)
1 1 1 1
(A) (B) (C) (D)
32 64 128 256 17. If sec(q + a)
a + sec(q - a)
a = 2sec q and a is not an
even multiple of p
p, then (1 + cos a
a) is equal to
Solution: Since (A) sec2 q (B) cosec2 q
13p p 11p 3p 9p 5p 7p p (C) sin2 q (D) cos2 q
=p - , =p - , =p - and =
14 14 14 14 14 14 14 2 Solution: Given that sec(q + a)
a + sec(q - a)
a = 2sec q
the given expression can be written as 1 1 2
+ =
p 3p 5p cos(q + a ) cos(q - a ) cos q
sin2 sin2 sin2
14 14 14
Worked-Out Problems 55

Therefore Therefore
cos(q - a ) + cos(q + a ) 2 tan A - tan B = 1 + tan A tan B
=
cos(q + a )cos(q - a ) cos q
tan A - tan B - tan A tan B = 1
(2 cos q cos a )cos q = 2 cos(q + a )cos(q - a )
1 + tan A - tan B - tan A tan B = 2
cos2 q cos a = cos2 q - sin2 a
(1 + tan A)(1 - tan B) = 2
sin2 a = cos2 q (1 - cos a )
Now
1 - cos2 a = cos2 q (1 - cos a )
x = 2 Þ (1 + x)x = 32 = 9
1 + cos a = cos2 q (∵ cos a ¹ 1)
Answer: (B)
Answer: (D)

1 1 p 20. The value of tan 20° tan 80° cot 50° is equal to
18. If tana = and sin b = , where 0 < a , b < , then 1 1
7 10 2 (A) 3 (B) 2 3 (C) (D)
a + 2b is equal to 2 3 3
p p 3p p
(A) (B) (C) (D) Solution:
4 3 4 2
Solution: tan 20° tan 80° cot 50°

tan a + tan 2 b = (tan 20° tan 40° tan 80°)cot 40° cot 50°
tan(a + 2b ) =
1 - tan a tan 2 b é æp ö æp öù
= êtan(q ) tan ç - q ÷ tan ç + q ÷ ú ´ 1
(1/ 7) + tan 2 b ë è3 ø è3 øû
=
1 - (1/ 7)tan 2 b (where q = 20°)
(1// 7) + [2 tan b / 1 - tan2 b ]
= = tan 3q = tan 60° = 3
1 - (1/ 7)[ 2 tan b / 1 - tan2 b ]
Answer: (A)
æ 1 pö
çè∵ sin b = Þb ¹ ÷
4ø 21. If a and b are acute angles and such that
10
1 éæ 1ö æ 1ö ù m 1
+ ç 2 ´ ÷ /ç 1 - ÷ ú tan a = and tan b =
7 êëè 3ø è 9ø û m+1 2m + 1
=
1 éæ 1ö æ 1ö ù then a + b equals
1 - êç 2 ´ ÷ / ç 1 - ÷ ú
7 ëè 3ø è 9ø û p p p 3p
(A) (B) (C) (D)
(1/ 7) + (3 / 4) 6 4 3 4
=
1 - (3 / 4) × (1/ 7) Solution:
4 + 21
= =1 tan a + tan b
28 - 3 tan(a + b ) =
1 - tan a tan b
p
0 < a, b < and tan 2b > 0 Þ 0 < a + 2b
b<p
p. Therefore [m / m + 1] + [1/ 2 m + 1]
2 =
1 - [m /(m + 1)(2 m + 1)]
p
a + 2b =
4 2 m2 + 2 m + 1
=
Answer: (A) (m + 1)(2 m + 1) - m

19. If A - B = p /4 and x = (1 + tan A)(1 - tan B), then 2 m2 + 2 m + 1


=
(1 + x)x is 2 m2 + 3m + 1 - m
(A) 27 (B) 9 (C) 18 (D) 81 2 m2 + 2 m + 1
=
Solution: 2 m2 + 2 m + 1
p =1
tan( A - B) = tan =1
4
56 Chapter 1 Trigonometric Ratios and Transformations

since 0 < a , b < p / 2 Þ 0 < a + b < p and tan(a + b) = 1 sec 8q - 1


24. =
we have sec 4q - 1
p (A) cot 2q tan 8q (B) cot 8q tan 2q
a+b=
4 (C) tan 8q tan 2q (D) cot 8q cot 2q
Answer: (B) Solution:
2 2
sin q
22. If 0 £ q £ p and 81 + 81cos q = 30, then the number sec 8q - 1 æ 1 - cos 8q ö æ cos 4q ö
=
of values of q is sec 4q - 1 çè 1 - cos 4q ÷ø çè cos 8q ÷ø
(A) 4 (B) 3 (C) 2 (D) 8
æ 2 sin2 4q ö æ cos 4q ö

è 2 sin2 2q ÷ø çè cos 8q ÷ø
2
sin q
Solution: Put t = 81 so that
81 sin 4q (2 sin 4q cos 4q )
=
2
q
81cos =
t (2 sin2 2q ) cos 8q
Therefore 2 sin 2q cos 2q (sin 8q )
=
2 sin2 2q cos 8q
81 = cot 2q tan 8q
t+ = 30
t
Answer: (A)
Þ t 2 - 30t + 81 = 0
Þ (t - 3)(t - 27) = 0 25. The value cos 12° + cos 84° + cos 132° + cos 156° is
1 -1 1 -1
(A) (B) (C) (D)
2 2 4 4
2
q 1 3 Solution: Given expression is
81sin = 3 or 33 Þ sin2 q = or
4 4
(cos 12° + cos 132°) + (cos 84° + cos 156°)
p p 5p 2p = (2 cos 72° cos 60°) + (2 cos 120° cos 36°)
0 £q £p Þq = or or or
6 3 6 3
( 5 - 1) 1 æ 1 ö ( 5 + 1)
Answer: (A) =2 ´ + 2ç- ÷
4 2 è 2ø 4
23. Let 0 < q < p /4 and 5 - 1 - ( 5 + 1)
=
4
t1 = (tan q )tan q , t2 = (tan q )cot q
1
=-
t3 = (cot q )tan q , t4 = (cot q )cot q 2
Answer: (B)
then
(A) t1 > t2 > t3 > t4 (B) t4 > t3 > t1 > t2 26. The value of 4(sin 24° + cos 6°) is
(C) t3 > t1 > t2 > t4 (D) t2 > t3 > t1 > t4 (A) 3+ 2 (B) 15 - 3
Solution: Since 0 < q < p
p/4, we can take
15 + 3
(C) (D) 15 + 3
tan q = 1 - x and cot q = 1 + y 2
where x and y are positive and x < 1. Observe that Solution:
0 < tan q < 1 and cot q > 1. Now
4(sin 24° + cos 6°) = 4(sin 24° + sin 84°)
1- x 1+ y
t1 = (1 - x) , t2 = (1 - x) = 4(2 sin 54° cos 30°)
1- x 1+ y
t3 = (1 + y) , t4 = (1 + y) 5+1 3
=8 ´
Clearly 4 2

t4 > t3 > t1 > t2 (∵ 1 - x < 1 + y) = 15 + 3


Answer: (B) Answer: (D)
Worked-Out Problems 57

27. If sin q, cos q and tan q are in GP, then 30. If


cos9 q + cos6 q + 3 cos5 q is equal to
æ p xö æ p xö
(A) 1 (B) 0 (C) 2 (D) –1 f ( x) = sin2 ç + ÷ - sin2 ç - ÷
è 8 2ø è 8 2ø
Solution: By hypothesis
sin2 q then the period of f is
cos2 q = (sin q )tan q = p p
cos q (A) p (B) (C) (D) 2p
2 3
Therefore
Solution:
cos3 q = sin2 q = 1 - cos2 q
cos3 q + cos2 q = 1 sin( A + B)sin( A - B) = sin2 A - sin2 B

(cos3 q + cos2 q )3 = 1 This implies


cos q + 3 cos q + 3 cos q + cos q = 1
9 8 7 6
æpö 1
f ( x) = sin ç ÷ sin x = sin x
cos q + 3 cos q (cos q + cos q ) + cos q = 1
9 5 3 2 6 è 4ø 2

cos9 q + 3 cos5 q + cos6 q = 1 Therefore period of f( p


f x) is 2p.
(∵ cos q + cos q = 1)
3 2
Answer: (D)
Answer: (A)
31. If a = tan 15°, b = cosec 75° and c = 4 sin 18°, then

28. The least period of the function f ( x) = |sin x | + | cos x | (A) c > b > a (B) a > c > b
is (C) b > a > c (D) b > c > a
p p 3p Solution:
(A) (B) (C) (D) 2p
2 4 2
1 4
a = tan 15° = 2 - 3 = =
Solution: 2+ 3 8+4 3
æ pö æ pö æ pö 4
f ç x + ÷ = sin ç x + ÷ + cos ç x + ÷ b = cosec 75° = 6 - 2 =
è 2ø è 2ø è 2ø 6 + 2
= |cos x | + | - sin x |
4( 5 - 1) 4
n x | + |cos x |
= |sin c = 4 sin 18° = = 5 -1=
4 5+1
= f ( x)
Now
Answer: (A)
8+4 3> 6 + 2 > 5+1
29. If 0 < q < p /2 and
Therefore
p p
sin q = sin + sin
12 4 4 4 4
> >
then q equals 5+1 6+ 2 8+4 3
p p 5p p Hence
(A) (B) (C) (D)
6 3 12 8
c>b>a
Solution: Answer: (A)
3-1 1 3+1 5p
sin q = + = = sin 75 = sin 32. If cos(x - y), cos x, cos(x - y) are in HP, then
2 2 2 2 2 12
Now 0 < q < p /2 and sin q = sin 75°. This implies y
sec x cos =
2
5p
q=
12 1
(A) (B) 2 (C) 1 (D) 3
Answer: (C) 2
58 Chapter 1 Trigonometric Ratios and Transformations

Solution: By hypothesis 34. If

2 cos( x - y)cos( x + y) cos x sin x cos y sin y


cos x = + =1= +
cos( x - y) + cos( x + y) cos q sin q cos q sin q
2(cos2 x - sin2 y) Then
=
2 cos x cos y cos x cos y sin x sin y
+ =
This implies cos2 q sin2 q

cos2 x cos y = cos2 x - sin2 y (A) 2 (B) 0 (C) 1 (D) –1


Solution: From the hypothesis
sin2 y = cos2 x(1 - cos y)
sin(x + q)
q = sin(y + q)
q
y y æ yö
4 sin2 cos2 = cos2 x ç 2 sin2 ÷
2 2 è 2ø Therefore
y æ x + y + 2q ö æ x - yö
2 cos2 = cos2 x 2 cos ç ÷ø sin çè ÷ =0
2 è 2 2 ø
y 1
cos2 sec2 x = Now
2 2
æ x - yö
Therefore x - y ¹ 2 np Þ sin ç ¹0
è 2 ÷ø
y 1
cos sec x = Hence
2 2
Answer: (A) x + y + 2q p
= (2 n + 1)
2 2
33. If cos x = cos y, sin x = –sin y, then
x + y = (2n + 1)p - 2q (1.7)
sin(2010) x + sin(2010) y =
Again
(A) 1 (B) 2010 (C) –2010 (D) 0
æ cos x sin x ö æ cos y sin y ö
Solution: çè + ÷ç + ÷ =1
cos q sin q ø è cos q sin q ø
æ x + yö æ x - yö
sin x + sin y = 0 Þ 2 sin ç cos ç =0
è 2 ÷ø è 2 ÷ø
Therefore
æ cos x cos y sin x sin y ö
æ x + yö æ x - yö çè + ÷
cos x - cos y = 0 Þ 2 sin ç sin ç =0 cos2 q sin2 q ø
è 2 ÷ø è 2 ÷ø
sin x cos y + cos x sin y
æ x - yö æ x - yö + =1
Both cos ç and sin ç cannot be simultane- sin q cos q
è 2 ÷ø è 2 ÷ø
ously zero. So this implies cos x cos y sin x sin y sin( x + y)
+ + =1
cos2 q sin2 q sin q cos q
æ x + yö
sin ç =0 cos x cos y sin x sin y sin 2q
è 2 ÷ø + + =1
cos2 q sin2 q sin q cos q
Þ x + y = 2 np
cos x cos y sin x sin y
+ +2=1
where n is an integer. Now cos2 q sin2 q

sin(2010) x + sin(2010) y = sin(2010) x + sin 2010(2np - x)


Answer: (D)
= sin(2010) x - sin(2010) x
=0 f x) = sin(x/3) + cos(x/2) has period
35. The function f(
Answer: (D) equal to
(A) p p
(B) 2p p
(C) 6p (D) 12p
Worked-Out Problems 59

Solution: Period of sin(x/3) is 6pp and period of cos(x/2) Solution: Let P = cos q cos 2q cos 4q cos 2n-1 q.
p Therefore period of f(
is 4p. p
f x) is 12p.
(2n sin q )P = (2 sin q cos q )(2 cos 2q )
Answer: (D)
(2 cos 4q )  (2 cos 2n - 1 q )
36. If A + B + C = p
p, then the minimum value of
= [(sin 2q )(2 cos 2q )](2 cos 4q )
A B C
tan + tan2 + tan2
2
(2 cos 8q )  (2 cos 2n- 1 q )
2 2 2
= [(sin 4q )(2 cos 4q )]
is
(A) 1 (B) 3 (C) 9 (D) 27 2 cos 8q  (2 cos 2n - 1 q )
Solution:   
A B C p A æ B + Cö = (sin 2n - 1 q )(2 cos 2n - 1 q )
+ + = Þ tan = cot ç
2 2 2 2 2 è 2 ÷ø
= sin 2n q
1 - tan( B / 2)tan(C / 2)
=
tan( B / 2) + tan(C / 2)

Therefore sin 2n q
P=
2n sin q
A B
S tan tan = 1 Answer: (B)
2 2
Now, 38. cos 36° cos 72° cos(108°) cos(144°) is equal to
2 2 1 1 1 1
æ A Bö æ B Cö (A) (B) (C) (D)
çè tan - tan ÷ø + çè tan - tan ÷ø 16 32 8 64
2 2 2 2
2
Solution: The given product is
æ C Aö
+ ç tan - tan ÷ ³ 0 cos 36° sin 18°(- sin 18°)(- cos 36°) = sin2 18° cos2 36°
è 2 2ø
2 2
æ A B Cö A B æ 5 - 1ö æ 5 + 1ö
Þ 2 ç tan2 + tan2 + tan2 ÷ - 2 S tan tan ³ 0 =ç ÷ ç ÷
è 2 2 2 ø 2 2 è 4 ø è 4 ø

A B C (5 - 1)2
Þ tan2 + tan2 + tan2 - 1 ³ 0 =
2 2 2 44
A B C 1
Þ tan2 + tan2 + tan2 ³ 1 =
2 2 2 16

and the equality holds when Answer: (A)

A B C 39. If A + B + C = p
p, then
tan = tan = tan
2 2 2
sin( B + C - A) + sin(C + A - B) + sin( A + B - C ) =
so that when A = B = C = p /3, the minimum value of the
(A) 4 cos A cos B cos C (B) 4 cos A sin B sin C
given expression is 1.
(C) 4 sin B cos C cos A (D) 4 sin A sin B sin C
Answer: (A)
Solution: The given sum is
37. cos q cos 2q cos 4q cos 8q  cos 2n-1 q is equal to
sin(p - 2 A) + sin(p - 2 B) + sin(p - 2C )
2n sin q sin 2n q
(A) (B)
sin 2n q 2n sin q = sin 2 A + sin 2 B + sin 2C

2n sin 2n q sin q = 4 sin A sin B sinC


(C) (D)
sin q 2 sin 2n q
n
Answer: (D)
60 Chapter 1 Trigonometric Ratios and Transformations

40. tan a + 2tan 2a + 4tan 4a + 8cot 8a = Solution: Given that


(A) tan 2aa (B) cot 2a cos x a
=
(C) cot a (D) tan 4a cos y b
Solution: Let
cos a - sin a
2 2
a b
cot a - tan a = = 2 cot 2a = = k (suppose)
sin a cos a cos x cos y
Therefore Therefore
cot 2a - tan 2a = 2 cot 4a a+b
=k (1.8)
cot 4a - tan 4a = 2 cot 8a cos x + cos y

cot a - 2 cot 2a = tan a Now

2 cot 2a - 4 cot 4a = 2 tan 2a a sin x b sin y


a tan x + b tan y = +
cos x cos y
4 cot 4a - 8 cot 8a = 4 tan 4a
= k[sin x + sin y]
On adding, we have
é sin x + sin y ù
cot a - 8 cot 8a = tan a + 2 tan 2a + 4 tan 4a = (a + b) ê ú
ë cos x + cos y û
Hence
[from Eq. (1.8)]
cot a = tan a + 2 tan 2a + 4 tan 4a + 8 cot 8a
æ x + yö
= (a + b)tan ç
Answer: (C) è 2 ÷ø

41. If x is any real number, then the value of Answer: (A)


2
tan[(p /4) sin x] lies between
43. If 90° < q < 180° and tan q = -2/3, then
(A) –1 and 1 (B) 0 and ¥
(D) –¥ and ¥ tan(90° + q ) + cos(180° - q )
(C) 0 and 1 =
sin(270° - q ) - cot(-q )
Solution:
p p 2 + 13 2 - 13
0 £ sin2 x £ 1 Þ 0 £ sin2 x £ (A) (B)
4 4 2 - 13 2 + 13

This implies tan[(p /4)sin2 x] lies between 0 and 1. 2 + 39


(C) (D) 2 + 39
Answer: (C) 2 - 39
Solution: 90° < q < 180° implies
42. If
2 -3
cos x a sin q = and cos q =
= 13 13
cos y b
then a tan x + b tan y equals
æ x + yö tan(90° + q ) + cos(180° - q ) - cot q - cos q
(A) (a + b)tan ç =
è 2 ÷ø sin(270° - q ) - cot(-q ) - cos q + cot q

æ x yö -(-3 / 2) - (-3 / 13 )
(B) (a + b) ç tan + tan ÷ =
è 2 2ø -(-3 / 13 ) - (3 / 2)
æ x yö (3 / 2) + (3 / 13 )
(a + b) ç cot + cot ÷ =
è 2 2ø (3 / 13 ) - (3 / 2)
æx+ yö
2 + 13
(a + b)cot ç ÷ø
è 2 =
2 - 13
Answer: (A)
Worked-Out Problems 61

44. For all angles A [¹ (2n + 1)p],


p 46. If

sin 2 A cos A
= tan( A - B) sin2 C
+ =1
(1 + cos 2 A)(1 + cos A) tan A sin2 A
A A then tan A tan B is equal to
(A) sin (B) cos
2 2 (A) tan2 C (B) 2 tan2 C
A (C) cot2 C (D) 2 cot2 C
(C) tan (D) sin A
2
Solution: From the hypothesis
Solution:
sin2 C tan( A - B) tan B(1 + tan2 A)
sin 2 A cos A 2 sin A cos2 A = 1 - =
= sin2 A tan A tan A(1 + tan A tan B)
(1 + cos 2 A)(1 + cos A) (2 cos2 A)[2 cos2 ( A / 2)]
sin( A / 2)cos( A / 2) Therefore
=
cos2 ( A / 2) tan B sin2 A 1
sin2 C = ´ ´
= tan
A tan A cos2 A 1 + tan A tan B
2
tan B tan A
=
Answer: (C) 1 + tan A tan B

45. If xcosa + ysina = xcosb + ysinb = 2a and (2sina/2) ´ This gives


sinb/2 = 1, then
sin2 C + sin2 C tan A tan B = tan B tan A
(A) y2 = 4a(x - a) (B) y2 = 4a(a - x)
(C) x2 = 4a(y - a) (D) x2 = 4a(a - y) sin2 C = (1 - sin2 C )tan A tan B

Solution: Consider the equation tan2 C = tan A tan B


(2a - x cos q )2 = y2 sin2 q Answer: (A)
which can be written as
47. If
(x2 + y2)cos2 q - (4ax)cos q + 4a2 - y2 = 0 (1.9)
cos x - cos a sin2 a cos b
From the hypothesis, cos a and cos b are roots of =
cos x - cos b sin2 b cos a
Eq. (1.9). Therefore
4ax 4a2 - y2
cos a + cos b = and cos a cos b =
x + y2
2
x2 + y2 a b
tan2 tan2 =
Also 2 2
x
a b a b (A) cot2 x (B) cot2
2 sin sin = 1 Þ 4 sin2 sin2 = 1 2
2 2 2 2
2 x
(C) tan2 x (D) tan
Therefore 2
a b Solution: From the hypothesis
1 = 4 sin2 sin2 = (1 - cos a )(1 - cos b )
2 2
(cos x - cos a )sin2 b cos a = (cos x - cos b )sin2 a cos b
= 1 - (cos a + cos b ) + cos a cos b
cos x[sin2 b cos a - sin2 a cos b ]
4ax 4a2 - y2
=1- 2 + 2
x +y 2
x + y2 = sin2 b cos2 a - cos2 b sin2 a
Hence cos x[(1 - cos2 b )cos a - (1 - cos2 a ) cos b ]
-4ax + 4a2 - y2 = 0 = (1 - cos2 b )cos2 a - cos2 b (1 - cos2 a )
y2 = 4a(a - x) cos x[(cos a - cos b ) + cos a cos b (cos a - cos b )]
Answer: (B) = cos2 a - cos2 b
62 Chapter 1 Trigonometric Ratios and Transformations

cos x(1 + cos a cos b ) = cos a + cos b Hence


cos a + cos b q 1 - cos q 1 - [(cos a + cos b ) / (1 + cos a cos b )]
cos x = tan2 = =
1 + cos a cos b 2 1 + cos q 1 + [(cos a + cos b ) / (1 + cos a cos b )]
Therefore (1 - cos a )(1 - cos b )
=
(1 + cos a )(1 + cos b )
x 1 - cos x
tan2 = a b
2 1 + cos x = tan2 tan2
2 2
1 - [(cos a + cos b ) / (1 + cos a cos b )]
= Answer: (C)
1 + [(cos a + cos b ) / (1 + cos a cos b )]
1 + cos a cos b - (cos a + cos b ) 49. If a, b lie between 0 and p
p/4 and cos(a + b)
b = 4/5,
= sin(a - b)
b = 5/13, then value of tan 2a
a is
1 + cos a cos b + cos a + cos b
11 46 56 13
(1 - cos a )(1 - cos b ) (A) (B) (C) (D)
= 12 33 33 11
(1 + cos a )(1 + cos b )
Solution:
[2 sin2 (a / 2)][2 sin2 (b / 2)]
=
[2 cos2 (a / 2)][2 cos2 (b / 2)] tan 2a = tan(a + b + a - b )
a b tan(a + b ) + tan(a - b )
= tan2 tan2 =
2 2 1 - tan(a + b )tan(a - b )
Answer: (D) (3 / 4) + (5 / 12)
=
1 - (3 / 4) ´ (5 / 12)
48. If
56
sin a sin b =
tan q = 33
cos a + cos b
Answer: (C)
then
50. If a + b = p
p/2 and b + g = a,
a then tan a equals
a b (A) 2(tan b + tan g ) (B) tan b + tan g
tan tan2 =
2

2 2
(C) tan b + 2tan g (D) 2tan b + tan g
q
(A) cot2 q (B) cot2 Solution:
2
p
q a+b=
(C) tan2 (D) tan2 q 2
2
æp ö
Solution: We have Þ tan a = tan ç - b ÷
è2 ø
sin2 a sin2 b Þ tan a tan b = 1
sec2q = 1 + tan2 q = 1 +
(cos a + cos b )2
Again,
(cos a + cos b )2 + sin2 a siin2 b
= b +g =a
(cos a + cos b )2
Þg =a - b
cos2 a + cos2 b + 2 cos a cos b + (1 - cos2 a )(1 - cos2 b )
= tan a - tan b tan a - tan b
(cos a + cos b )2 Þ tan g = =
1 + tan a tan b 1+ 1
(1 + cos a cos b )2
= Þ 2 tan g + tan b = tan a
(cos a + cos b )2
Answer: (C)
Therefore
51. Let n be a positive integer such that
cos a + cos b
cos q =
1 + cos a cos b p p n
sin + cos =
2n 2n 2
Worked-Out Problems 63

Then 1é 2p 4p 12p 10p ù


= cos + cos - cos - cos
(A) 6 £ n £ 8 (B) 4 < n £ 8 2 êë 7 7 7 7 úû
(C) 4 £ n £ 8 (D) 4 < n < 8
1é 2p 4p æ 2p ö æ 4p ö ù
Solution: n = 1, 2 cannot satisfy the given equation. = cos + cos - cos ç 2p - ÷ - cos çè 2p - ÷
2 êë 7 7 è 7 ø 7 ø úû
2
p p n æ p p ö n 1é 2p 4p 2p 4p ù
sin + cos = Þ ç sin + cos ÷ = = + cos - cos - cos ú
è 2n ø cos
2n 2n 2 2n 4 2 êë 7 7 7 7 û
p n
Þ 1 + sin = =0
n 4
Answer: (A)
p n-4
Þ sin =
n 4 2p 4p 6p
54. cos + cos + cos =
7 7 7
Therefore
1 -1
n-4 (A) 0 (B) 1 (C) (D)
0< <1 2 2
4
0<n-4<4 Solution: Let

4<n<8 2p 4p 6p
s = cos + cos + cos
Answer: (D) 7 7 7

52. Let 0 < x < p


p/4. Then (sec 2x - tan 2x) equals Therefore

æ pö æp ö æ pö p 2p p 4p
(A) tan ç x - ÷ (B) tan ç - x÷ çè 2 sin ÷ø s = 2 sin cos + 2 sin cos
è 4ø è4 ø 7 7 7 7 7
æ pö æ pö p 6p
(C) tan ç x + ÷ (D) tan2 ç x + ÷ + 2 sin cos
è 4ø è 4ø 7 7
Solution: é 3p p ù é 5p 3p ù
= êsin - sin ú + êsin - sin ú
ë 7 7û ë 7 7 û
1 - sin 2 x (cos x - sin x)2
sec 2 x - tan 2 x = =
cos 2 x cos2 x - sin2 x é 7p 5p ù
+ êsin - sin ú
Now 0 < x < p/4
p Þ cos x - sin x > 0. Therefore ë 7 7 û

cos x - sin x 7p p
æp ö = sin - sin
sec 2 x - tan 2 x = = tan ç - x÷ 7 7
cos x + sin x è4 ø
Answer: (B) p
= 0 - sin
7
53. The value of
This gives
2p 4p 4p 8p 8p 2p
sin sin + sin sin + sin sin -1
7 7 7 7 7 7 s=
2
is Answer: (D)
1 -1
(A) 0 (B) (C) (D) 1
2 2 55. The value of

Solution: The given expression can be written as 2p 4p 8p


sin + sin + sin
1 éæ 2p 6p ö æ 4p 12p ö 7 7 7
ç cos - cos ÷ + ç cos - cos ÷
2 êëè 7 7ø è 7 7 ø is
æ 6p 10p ö ù 5 5 7 7
+ ç cos - cos ÷ (A) (B) (C) (D)
è 7 7 ø úû 2 2 2 2
64 Chapter 1 Trigonometric Ratios and Transformations

Solution: Let 1é 3ù
= 2 sin2 x + cos 2 x + ú
2p 4p 8p 2 êë 2û
s = sin + sin + sin
7 7 7 1æ 3ö 5
= ç 1 + ÷ø =
Therefore 2è 2 4
f x) = 5/4 for all x. Hence
Therefore f(
2p 4p 8p 2p 4p
s = sin
2 2
+ sin2 + sin2 + 2 S sin sin
7 7 7 7 7 æpö æ 5ö
(g f ) ç ÷ = g ç ÷ = 1
2 2p 2 4p 2 8p
è 8ø è 4ø
= sin + sin + sin + 0 (see Q.53)
7 7 7 Answer: (B)
1 - cos(4p / 7) 1 - cos(8p / 7) 1 - cos(16p / 7)
= + + f x) = sin 3x + |cos 6x| is
57. Period of the function f(
2 2 2
3 1é 4p 8p 16p ù p 2p p
= - êcos + cos + cos (A) (B) (C) (D) p
2 2ë 7 7 7 úû 3 3 6
3 1é 4p 8p 2p ù Solution: Period of sin 3x = 2p/
p3
= - cos + cos + cos ú
2 2 êë 7 7 7 û p
Period of |cos 6 x| =
3 1 æ 1ö 6
= - ç- ÷ (see Problem 54)
2 2 è 2ø Therefore
7
= æ 2p ö æ p ö 2p
4 1× ç ÷ = 4 × ç ÷ =
è 3ø è 6ø 3
Therefore
f x) = 2p/3.
Hence period of f( p
7 Answer: (B)
s=
2
f x) = 3sin(px
58. Period of f( p /3) + 4cos(px
p /4) is
Answer: (C)
(A) 6 (B) 12 (C) 16 (D) 24
56. If
Solution:
æ pö px 2p
f ( x) = sin2 x + sin2 ç x + ÷ Period of sin = =6
è 3ø 3 p /3
æ pö æ 5ö px 2p
+ cos x cos ç x + ÷ and g ç ÷ = 1 Period of cos = =8
è 3 ø è 4ø 4 p /4

then Now 4(6) = 3(8) = 24. Therefore period of f(


f x) = 24
Answer: (D)
æpö
(g f ) ç ÷ =
è 8ø f x) = sin(px
59. Period of f( p /5!) + cos(px
p /6!) is
5 (A) 5! (B) 2(5!) (C) 2(6!) (D) 6!
(A) (B) 1 (C) 2 (D) p
4 Solution:
Solution: px 2p
Period of sin = = 2(5!)
5! (p / 5!)
æ pö æ pö
sin2 x + sin2 ç x + ÷ + cos x cos ç x + ÷ px 2p
è 3ø è 3ø Period of cos = = 2(6 !)
6 ! (p / 6 !)
1é æ 2p ö æ pö pù
= ê 2 sin2 x + 1 - cos ç 2 x + ÷ø + cos çè 2 x + ÷ø + cos ú Now 6(2×5!) = 2(6!). Therefore, period of f(
f x) = 2(6!)
2ë è 3 3 3û
Answer: (C)
1é æ 2p ö æ p ö 3ù
= 2 sin2 x - cos ç 2 x + ÷ + cos çè 2 x + ÷ø + ú
2 êë è 3ø 3 2û
Worked-Out Problems 65

n-1
å sin( x / 2r ) + å r = 1 tan( x / 2r ) Solution: The given expression is equivalent to
n
60. If period of f ( x) = r =0
p then k equals
is kp, 1 3
1 cos 20° + cos 40° + - 2[cos 30° + cos 10°]
(A) 1 (B) 2 (C) 2n (D) 2 2
2n
1 3
= 2 cos 30° cos 10° + - - 3 cos 10°
Solution: Periods of sinx, sin(x ( /2)2, …, sin(x
( /2), sin(x ( /2n-1) 2 2
p 2 p,
are 2p, 2
p 2 p,
3
p …, 2 p and those of tan((x/2), tan(x
n
( /22),
æ 3ö
( /2 ), …, tan(x
tan(x 3 n
p 2 p,
( /2 ) are 2p, 2
p 2 p,
3
p …, 2 pn
p. Therefore = 2ç ÷ cos 10° - 1 - 3 cos 10°
Period of f(f x) is 2np and hence k = 2n. è 2 ø
Answer: (C) = -1
Answer: (C)
f x) = cos x + cos(bx) is periodic where
61. If the function f(
b ¹ 0, then b must be cos 8 x - cos 7 x
64. =
(A) an even integer (B) odd integer 1 + 2 cos 5 x
(C) irrational number (D) rational number (A) cos 3x + cos 2x (B) cos 3x - cos 2x
Solution: Period of cos x = 2p (C) sin 3x - sin 2x (D) sin 3x + sin 2x

2p Solution:
Period of cos bx =
|b| cos 8 x - cos 7 x sin 5 x(cos 8 x - cos 7 x)
=
Since f is periodic, there exist positive integers m and n 1 + 2 cos 5x sin 5 x + sin 10 x
such that - sin 5 x[2 sin(15 x / 2)sin( x / 2)]
=
2 sin(15x / 2)cos(5x / 2)
æ 2p ö
m(2p ) = n ç -2 sin(5x / 2)cos(5xx /2)sin( x / 2)
è | b | ÷ø =
cos(5 x / 2)
Therefore = -(cos 2 x - cos 3x)
n = cos 3 x - cos 2 x
|b| =
m
Answer: (B)
which implies b is rational.
65. cos2 x + cos 2(q + x) - 2cos q cos x cos(q + x) is equal to
Answer: (D)
(A) cos2 x (B) sin2 x (C) cos2 q (D) sin2 q
f x) = x2 - 1 and g(x) = sin x + cos x. Then f g is
62. Let f( Solution: The given expression is equivalent to
invertible if x belongs to the interval
cos2 x + cos(q + x)[cos(q + x) - 2 cos q cos x]
é 3p ù é pù
(A) ê0, ú (B) ê0, ú
ë 2 û ë 2û = cos2 x - cos(q + x)cos(q - x)
é -p p ù é -p ù = cos2 x - (cos2 x - sin2 q )
(C) ê , ú (D) ê , 0 ú
ë 4 4û ë 2 û = sin2 q
Solution: Answer: (D)

( f g )( x) = f ( g( x)) = (sin x + cos x)2 - 1 = sin 2 x 66. If A, B and C are positive such that A + B + C = p,
p
sin 2x is invertible, if -p/
p 2 £ 2x £ p/
p 2 that is, -p/4
p £ x £ p/4.
p then the minimum value of
Answer: (C) æ A - Bö æ B -Cö æ C - Aö
cos ç
è 2 ÷ø
cos ç
è 2 ÷ø
cos ç
è 2 ÷ø
+ +
63. cos 20° + cos 40° + cos 60° - 4cos 10°cos 20°cos 30° is æ A + Bö æ B + Cö æ C + Aö
cos ç ÷ cos ç ÷ cos ç
equal to è 2 ø è 2 ø è 2 ÷ø
(A) 0 (B) 1
is
5+1
(C) -1 (D) 3
4 (A) 3 (B) 3/2 (C) 6 (D)
16
66 Chapter 1 Trigonometric Ratios and Transformations

Solution: Solution:
æ A - Bö æ A - Bö æ A + Bö sin 20° + 2 sin 40°
cos ç cos ç sin ç tan 20° + 4 sin 20° =
è 2 ÷ø è 2 ÷ø è 2 ÷ø cos 20°
=
æ A + Bö æ A + Bö æ A + Bö (sin 20° + sin 40°) + sin 40°
cos ç cos ç sin ç
è 2 ÷ø è 2 ÷ø è 2 ÷ø =
cos 20°
sin A + sin B cos 10° + cos 50°
= =
sin( A + B) cos 20°
sin A + sin B 2 cos 30° cos 20°
= =
sin C cos 20°
sin A sin B
= + = 3
sin C sin C
Answer: (D)
Similarly
68. In DPQR, R = p / 2. If tan(P/2) and tan(Q/2) are
æ B - Cö roots of the quadratic equation ax2 + bx + c = 0 then
cos ç
è 2 ÷ø sin B sin C (A) b + c = a (B) c + a = b
= +
æ B + C ö sin A sin A (C) a + b = c (D) b = c
cos ç
è 2 ÷ø
Solution:
æC - Aö p
cos ç
è ÷ R= Þ P + Q = p /2
2 ø sin C sin A 2
and = +
æC + A ö sin B sin B
cos ç ÷ Therefore
è 2 ø
P Q p
Therefore + =
2 2 4
æ A - Bö æ B - Cö æC - Aö p tan( P / 2) + tan(Q / 2)
cos ç cos ç cos ç 1 = tan =
è 2 ÷ø è 2 ÷ø è 2 ø
÷ 4 1 - tan( P / 2)tan(Q / 2)
+ +
æ A + Bö æ B + Cö æC + Aö (- b / a) -b
cos ç cos ç cos ç = =
è 2 ÷ø è 2 ÷ø è 2 ø
÷
1 - (c / a) a - c
æ sin A sin B ö æ sin B siin C ö a - c = -b or a + b = c
=ç + + +
è sin C sin C ÷ø çè sin A sin A ÷ø Answer: (C)

æ sin C sin A ö 3 + cot 76° cot 16°


+ç + 69. =
è sin B sin B ÷ø cot 76° + cot 16°
(A) tan 46° (B) cot 76°
æ sin A sin C ö æ sin B sin C ö
=ç + + +
è sin C sin A ÷ø çè sin C sin B ÷ø
(C) tan 16° (D) cot 44°
Solution:
æ sin A sin B ö
+ç + 3 + cot 76° cot 16° 3 sin 76° sin 16° + cos 76° cos 16°
è sin B sin A ÷ø =
cot 76° + cot 16° sin 16° cos 76° + cos 16° sin 76°
³2+2+2 (∵ AM ³ GM)
2 sin 76° sin 16° + cos 60°
=6 =
sin 92°
Note that equality occurs when A = B = C = p/3.
p os 92° + cos 60°
cos 60° - co
=
Answer: (C) sin 92°
1 - cos 92°
67. tan 20° + 4 sin 20° = = = tan 46°
sin 92°
(A) 3 (B) 2 (C) 2 (D) 3
Answer: (A)
Worked-Out Problems 67

70. In DABC, if cot q = cot A + cot B + cot C, then Solution:

sin( A - q )sin( B - q )sin(C - q ) 4 xy ( x + y)2 - ( x - y)2 æ x - yö


2
= = =1-ç £1
sin3 q ( x + y)2
( x + y)2 è x + y ÷ø

(A) 1 (B) -1 (C) - 2 (D) 2 But sec2 q ³ 1 for all q. Therefore


Solution: We have cotq - cotA = cotB + cotC. Therefore 2
æ x - yö
1=1-ç Û x = y, x ¹ 0
sin( A - q ) sin(B + C ) sin A è x + y ÷ø
= =
sin A sin q sin B sin C sin B sin C Answer: (B)
sin( A - q ) sin A 2
= 73. If tan A = (1 - cos B)/sin B, then tan 3A is equal to
sin q sin B sin C
æ 3B ö
(A) tan ç (B) tan 2B
Similarly è 2 ÷ø

sin(B - q ) sin2 B sin(C - q ) sin2 C (C) tan B (D) -tan 2B


= , =
sin q sin C sin A sin q sin A sin B Solution:
sin( A - q )sin( B - q )sin(C - q ) 1 - cos B B
=1 tan A = = tan
sin3 q sin B 2
Answer: (A) Therefore
71. Which one of the following numbers is rational? 2 tan A 2 tan( B / 2)
tan 2 A = = = tan B
(A) sin 15° (B) cos 15° 1 - tan A 1 - tan2 ( B / 2)
2

(C) sin 15° cos 15° (C) sin 15° cos 75° tan A + tan 2 A tan( B / 2) + tan B 3B
tan 3 A = = = tan
Solution: 1 - tan A tan 2 A 1 - tan( B / 2)tan B 2
Answer: (A)
sin 15° = sin(45° - 30°)
= sin 45° cos 30° - cos 45° sin 30° 74. Consider the following two statements.

3 1 S1: All cyclic quadrilaterals ABCD satisfy


= -
2 2 2 2 A B C D
tan tan tan tan = 1
3 -1 2 2 2 2
= is irrational
2 2 S2: All trapeziums ABCD satisfy
3+1 A B C D
cos 15° = cos(45° - 30°) = is irrational tan tan tan tan = 1
2 2 2 2 2 2
3-1 1
sin 15° cos 15° = = is rational Then
8 4
(A) both S1 and S2 are true
2
æ 3 - 1ö (B) S1 is true but S2 is not true
sin 15° cos 75° = sin2 15° = ç ÷
è 2 2 ø (C) S1 is not true and S2 is true
(D) neither S1 nor S2 is true
4-2 3 2- 3
= = is irrational
8 4 Solution: In a cyclic quadrilateral ABCD, it is known
that
Answer: (C)
A + C = 180° = B + D
72. sec2 q = 4xy/(x + y)2 is true if and only if
Therefore
(A) x + y = 0 (B) x = y, x ¹ 0
(C) x = y (D) x ¹ 0, y ¹ 0 A C B D
tan = cot and tan = cot
2 2 2 2
68 Chapter 1 Trigonometric Ratios and Transformations

A B C D Solution: It is known that the sum of the interior


tan tan tan tan = 1 angles of an n-sided convex polygon is (2n - 4) right
2 2 2 2
angles. Therefore
This implies S1 is true. Let ABCD be a trapezium in
which AB and DC C are parallel sides (see Figure 1.36). cos( Ak + 1 + Ak + 2 +  + An ) = cos[(n - 2)p
Therefore - ( A1 + A2 +  + Ak )]
DCA = CAB (1.10) = ± cos( A1 + A2 +  + Ak )

Now in DADC, Now


CAD + D + DCA = 180° cos( A1 + A2 +  + Ak )
= ±1
Therefore cos( Ak +1 + Ak + 2 +  + An )

( A - CAB) + D + DCA = 180° according as n is even or odd.


Answer: (B)
A + D = 180° [From Eq. (1.10)]
Similarly B + C = 180°. Therefore 76. If cosx = tany, cosy = tanz, cosz = tanx, then sinx =
siny = sinz = 2sinq
q where q is
A D B C 1°
tan = cot , tan = cot (A) 15° (B) 18° (C) 22 (D) 75°
2 2 2 2 2
A B C D
tan tan tan tan = 1 Solution:
2 2 2 2
cos2 x = tan2 y = sec2 y - 1 = cot2 z - 1
Hence S2 is also true. This implies both S1 and S2 are true.
Therefore
D C
cos2 z tan2 x
1 + cos2 x = cot2 z = =
sin z 1 - tan2 x
2

sin2 x sin2 x
= =
cos2 x - sin2 x 1 - 2 sin2 x
(1 + cos2 x)(1 - 2 sin2 x) = sin2 x
(2 - sin2 x)(1 - 2 sin2 x) = sin
n2 x
FIGURE 1.36 Single correct choice type question 74.
Answer: (A) 2 sin4 x - 6 sin2 x + 2 = 0
sin4 x - 3 sin2 x + 1 = 0
75. Let A1, A2, …, An be the interior angles of an n-sided
convex polygon. Then the value of 3± 5
sin2 x =
cos( A1 + A2 +  + Ak ) 2
cos( Ak +1 + Ak + 2 +  + An ) Since (3 + 5 / 2) > 1, it follows that
where 2
3 - 5 æ 5 - 1ö
sin x =
2
=ç ÷
æ k ö 2 è 2 ø
cos ç å Ai ÷ ¹ 0 for k = 1, 2, 3, …, n - 1
è i =1 ø
Hence
(A) is independent of both k and n
5-1 æ 5 - 1ö
(B) is independent of k but depends on n sin x = = 2ç ÷ = 2 sin 18°
2 è 4 ø
(C) is independent of n but depends on k
(D) depends on both k and n Answer: (B)
Worked-Out Problems 69

Multiple Correct Choice Type Questions


1. If cos A = -7/25 and A lies between 450° and 540° then
1° 1°
A -4 A -3 sin 22 = + 1 - cos2 22
(A) sin = (B) cos = 2 2
2 5 2 5
A -3 A -4 2+ 2 2- 2
(C) sin = (D) cos = = + 1- =
2 5 2 5 4 2
Answers: (A), (B), (C), (D)
Solution: By hypothesis 450° < A < 540°. This implies
225° < A/2 < 270°. So
3. Let cos x + cos y = a, cos 2x + cos 2y
2 = b and cos 3x +
A 1 - cos A 1 + (7 / 25) 4 cos 3y = c. Which of the following is (are) true?
sin = = =
2 2 2 5 b
(A) cos2 x + cos2 y = 1 +
2
Now A/2 lies in the third quadrant. This means
1
(B) (2a2 - b - 2) = cos x cos y
A -4 4
sin =
2 5 (C) 2a3 + c = 3a(1 + b)
Again (D) abc = 0 for all real x and y

A 1 + cos A 1 - (7 / 25) 3 Solution:


cos = = =
2 2 2 5 cos2 x + cos2 y + 2 cos x cos y = a2 (1.11)
Again A/2 lies in the third quadrant implies b = cos 2x + cos 2y
2 = 2(cos2 x + cos2 y) - 2
A -3 Therefore
cos =
2 5
b+2 b
Answers: (A), (B) cos2 x + cos2 y = =1+ (1.12)
2 2

2. Which of the following are true? Hence (A) is true. From Eqs. (1.11) and (1.12)

1° 2- 2 1
(A) sin 22 = cos x cos y = (2a2 - b - 2)
2 2 4

1° 2+ 2 Hence (B) is true. Now


(B) cos 22 =
2 2 c = cos 3 x + cos 3 y = 4(cos3 x + cos3 y) - 3(cos x + cos y)
1 °
tan 22 = 2 -1 = 4[(cos x + cos y)(cos2 x + cos2 y - cos x cos y)]
2
1° - 3(cos x + cos y)
(D) cot 22 = 2 +1
2 é æb+ 2 1 öù
= 4 êa ç - (2a2 - b - 2)÷ ú - 3a
Solution: ë è 2 4 øû

1 1° = 2a(b + 2) - 2a3 + ab + 2a - 3a
= cos 45° = 2 cos 22 - 1
2

2 2 = - 2a3 + 3ab + 3a

Therefore Therefore

1° 1 æ 1 ö 1 æ 2 + 2ö 2+2 2a3 + c = 3a(b + 1)


cos2 22 = ç + 1÷ = ç =
2 2è 2 ø 2 è 2 ÷ø 4 Hence (C) is true. abc = 0 for all x, y is not true. Hence
(D) is not true.
1° 2+2 æ 1° ö
= Answers: (A), (B), (C)
cos 22 ç∵ cos 22 > 0÷
2 2 çè 2 ÷ø
70 Chapter 1 Trigonometric Ratios and Transformations

4. Which of the following are true? Solution:


(A) cos 2q cos q - sin 4q
q sin q = cos 3q cos 2q 1
(B) cos 20° + cos 100° + cos 140° = 0 (A) LHS =
(sin 3 A /cos 3 A) - (sin A /cos A)
(C) sin 5q + sin 2q - sin q = sin 2q(2 cos 3q + 1)
1
cos q + sin q -
(D) = tan 2q + sec 2q (cos 3 A /sin 3 A) - (cos A /sin A)
cos q - sin q
cos 3 A cos A
Solution: =
sin 3 A cos A - cos 3 A sin A
(A) cos 2q cos q - sin 4q sin q
sin 3 A sin A
1 1 -
= (cos 3q + cos q ) - (cos 3q - cos 5q ) sin A cos 3 A - cos A sin 3 A
2 2
cos 3 A cos A + sin 3 A sin A
1 =
= (cos 5q + cos q ) sin 3 A cos A - cos 3 A sin A
2
= cos 3q cos 2q cos(3 A - A) cos 2 A
= =
sin(3 A - A)) sin 2 A
Therefore (A) is true.
= cot 2 A
(B) cos 20° + cos 100° + cos 140°
Therefore (A) is true.
= cos 20° + 2 cos 120° cos 20°
cos 3a - sin 3a
= cos 20° - cos 20° (B)
cos a + sin a
=0
(4 cos3a - 3 cos a ) - (3 sin a - 4 sin3a )
Hence (B) is true. =
cos a + sin a
(C) sin 5q + sin 2q - sin q = (sin 5q - sin q ) + sin 2q
4(cos3 a + sin3 a ) - 3(cos a + sin a )
= 2 cos 3q sin 2q + sin 2q =
cos a + sin a
= sin 2q (2 cos 3q + 1)
4[(cos a + sin a )(1 - sin a cos a )] - 3(cos a + sin a )
=
So (C) is true. cos a + sin a
sin 2q + 1 (cos q + sin q )2 = 4 - 4 sin a cos a - 3
(D) tan 2q + sec 2q = =
cos 2q cos2 q - sin2 q
= 1 - 2 sin 2a
cos q + sin q
= Therefore (B) is true.
cos q - sin q
cot q tan q
This implies that (D) is true. (C) +
cot q - cot 3q tan q - tan 3q
Answers: (A), (B), (C), (D)
æ cos q ö æ sin q ö
5. Which of the following are true? çè sin q ÷ø çè cos q ÷ø
1 1 = +
(A) - = cot 2 A æ cos q ö æ cos 3q ö æ sin q ö æ sin 3q ö
tan 3 A - tan A cot 3 A - cot A çè sin q ÷ø - çè sin 3q ÷ø çè cos q ÷ø - çè cos 3q ÷ø
cos a - sin 3 a
(B) = 1 - 2 sin 2a cos q sin 3q sin q cos 3q
cos a + sin a = +
sin 3q cos q - cos 3q sin q sin q cos 3q - cos q sin 3q
cot q tan q
(C) + =1
cot q - cot 3q tan q - tan 3q cos q sin 3q sin q cos 3q
= -
4(cos q + sin q)
6
q = 1 + 3 cos 2q
6 2 sin 2q sin 2q
Worked-Out Problems 71

sin 3q cos q - cos 3q sin q (B) LHS = (3 sin a - sin 3a )cos 3a


=
sin 2q + (3 cos a + cos 3a )sin 3a
sin 2q
= =1 = 3[sin a cos 3a + sin 3a cos a ]
sin 2q
= 3 sin 4a
Therefore (C) is true.
Hence (B) is true.
(D) 4(cos q + sin q )
6 6
(C) 2 sin 75° sin 15° = cos(75° - 15°) - cos(75° + 15°)
= 4[(cos q ) + (sin q ) ]
2 3 2 3
= cos 60° - cos 90°
= 4[cos2 q + sin2 q ][cos4 q - cos2 q sin2 q + sin4 q ] 1
= -0
= 4[(cos q + sin q ) - 3 cos q sin q ]
2 2 2 2 2 2
1
= 4[1 - 3 cos2 q sin2 q ] =
2
é 3 ù
= 4 ê1 - sin2 2q ú
ë 4 û
(D) LHS = 2 sin C cos(B - A) + 2 cos( A + B)sin(-C )
= 4 - 3 sin2 2q
= 2 sin C[cos( B - A) - cos( A + B)]
= 1 + 3(1 - sin2 2q )
= 2 sin C (2 sin A sin B)
= 1 + 3 cos2 2q
= 4 sin A sin B sin C
Hence (D) is true.
Answers: (A), (B), (C), (D) Therefore (D) is true.
Answers: (A), (B), (C), (D)
6. Which of the following are true?
(A) cos3 a cos 3a + sin3 a sin 3a = cos3 2a 7. Which of the following are true?

(B) 4 sin3 a cos 3a + 4 cos3 a sin 3a = 3 sin 4a (A) cos(30° - A) + cos(30° + A) = 3 cos A
(C) 2 sin 75°sin 15° = 1/2 (B) sin(60° + A) - sin(60° - A) = sin A
(D) sin(B + C - A) + sin(C + A - B) + sin(A + B - C) cos 2q - cos 3q q
- sin(A
( + B - C)C = 4sinAsinBsinC (C) = tan
sin 2q + sin 3q 2
Solution:
sin2 5q - sin2 3q = sin 8q sin 2q
(A) LHS = cos3 a (4 cos3 a - 3 cos a ) Solution:
+ sin a (3 sin a - 4 sin a )
3 3
(A) cos(30° - A) + cos(30° + A)
= 4(cos6 a - sin6 a ) - 3(cos4 a - sin4 a ) = 2 cos 30° cos A
= 4(cos2 a - sin2 a )[cos4 a + sin2 a cos2 a + sin4 a ] æ 3ö
= 2ç ÷ cos A = 3 cos A
- 3(cos a - sin a )
2 2
è 2 ø
= cos 2a [4 cos4 a + 4 sin2 a cos2 a + 4 sin4 a - 3] Therefore (A) is true.

= cos 2a [4 cos2 a (1 - sin2 a ) + 4 sin2 a cos2 a (B) sin(60° + A) - sin(60° - A)

+ 4 sin2 a (1 - cos2 a ) - 3] æ 1ö
= 2 cos 60°sin A = 2 ç ÷ sin A = sin A
è 2ø
= cos 2a [4 - 4 sin2 a cos2 a - 3]
Therefore (B) is true.
= cos 2a [1 - sin 2a ] 2
cos 2q - cos 3q 2 sin(5q / 2)sin(q / 2) q
(C) = = tan
= cos 2a 3
sin 2q + sin 3q 2 sin(5q / 2)cos(q / 2) 2
So (A) is true. Therefore (C) is true.
72 Chapter 1 Trigonometric Ratios and Transformations

(D) sin2 A - sin2 B = sin( A + B)sin( A - B) cot q - 1 cos q - sin q


(D) =
cot q + 1 cos q + sin q
sin2 5q - sin2 3q = sin(5q + 3q )sin(5q - 3q )
= sin 8q sin 2q (cosq - sin q )2
=
cos2 q - sin2 q
1 - 2 sin q cosq
Answers: (A), (B), (C), (D) =
cos 2q
8. Which of the following are true? 1 - sin 2q
=
(A) 4(cos 10° + sin 20°) = 3(cos 10° + sin 20°)
3 3
cos 2q
1 1 Therefore (D) is true.
(B) - = cot 4q
tan 3q + tan q cot 3q + cot q Answers: (A), (B), (D)
1 - cos 2q + sin 2q
(C) = cot q 9. Which of the following are true?
1 + cos 2q + sin 2q
cot q - 1 1 - sin 2q (A) If cosq - sinq = 2 sinq,
q then cosq + sinq = 2 cosq
(D) =
cot q + 1 cos 2q px px
(B) Period of the function f ( x) = 2 sin + 3 cos
4 3
Solution: is 24
p
4(cos3 10° + sin3 20°) = (cos 30° + 3 cos 10°) (C) Period of tan(x + 2x + 3x) is
3
+ (3 sin 20° - sin 60°)
(D) tan 855° + tan 45° = -1
æ 3 ö
=ç + 3 cos 10°÷ Solution:
è 2 ø
(A) cos q - sin q = 2 sin q
æ 3ö
+ ç 3 sin 20° - cos q = ( 2 + 1)sin q
è 2 ÷ø
( 2 - 1)cos q = sin q
= 3(cos 10° + sin 20°)
2 cos q = cos q + sin q
Therefore (A) is true.
cos q cos 3q
(B) LHS = æ px ö 4
sin 3q cos q + cos 3q sin q (B) Period of sin ç ÷ = (2p ) = 8
è 4ø p
sin 3q sin q
-
sin q cos 3q + cos q sin 3q æ px ö 3
Period of cos ç ÷ = (2p ) = 6
è 3ø p
cos 3q cos q - sin 3q sin q
=
sin(30 + q ) 3(8) = 4(6) = 24 Þ 24 is a period of f(
f x)
cos 4q This implies (B) is also true.
=
sin 4q p
(C) Period of tan 6 x =
= cot 4q 6
Hence (B) is true. So (C) is not true.
(D) tan 855° + tan 45° = tan(720° + 135°) + 1
1 - cos 2q + sin 2q 2 sin2 q + 2 sin q cos q
(C) = = tan 135° + 1
1 + cos 2q + sin 2q 2 cos2 q + 2 sin q cos q
= -1+ 1
sin q (sin q + cos q )
= =0
cos q (cos q + sin q )
So (D) is not true.
= tan q
Answers: (A), (B)
Hence (C) is not true.
Worked-Out Problems 73

10. A, B and C are angles of triangle such that (cos 1° - cos 1°) + (cos 2° - cos 2°) + 
cos A cos B + sin A sin B sin C = 1 + (cos 89° - cos 89°) + cos 90° = 0
then DABCC is Hence (B) is true.
(A) isosceles triangle (C) sin2 85° = sin2(90° - 5°) = cos2 5°, sin2 80° = cos2 10°, etc.
(B) right-angled triangle The given expression is
(C) isosceles and right angled 1 17
8 + sin2 45° = 8 + =
(D) equilateral triangle 2 2
Solution: Therefore (C) is true.
cos A cos B + sin A sin B sin C = 1
æ p xö æ p xö
(D) sin2 ç + ÷ - sin2 ç - ÷
cos A cos B + sin A sin B è 8 2ø è 8 2ø
- sin A sin B + sin A sin B sin C = 1
æ p x p xö æ p x p xö
= sin ç + + - ÷ sin ç + - + ÷
cos( A - B) + sin A sin B(sin C - 1) = 1 è 8 2 8 2ø è 8 2 8 2ø
sin A sin B(sin C - 1) = 1 - cos( A - B) æ pö
= ç sin ÷ sin x
è 4ø
Now LHS £ 0 and RHS ³ 0. Therefore LHS = RHS = 0.
Hence 1
= sin x
sin C - 1 = 0 and 1 - cos( A - B) = 0 2
sin C = 1 and cos( A - B) = 1
Therefore period = 2p.
p Hence (D) is also true.
C = 90° and A=B
Answers: (A), (B), (C), (D)
Answers: (A), (B), (C)
12. Which of the following are true?
11. Which of the following are true?
(A) If sin A + sin B = 3(cos B - cos A), then sin 3A
(A) Minimum value of 27 tan 2 q + 3 cot2 q is 18
+ sin3B is equal to 0
(B) cos 1° + cos 2° + cos 3° +  + cos 179° = 0
p 3p 5p 7p
17 (B) The value of cos2 + cos2 + cos2 + cos2
(C) sin2 5° + sin2 10° + sin2 15° +  + sin2 85° = 16 16 16 16
2 3
is
æ p xö æ p xö 2
(D) Period of sin2 ç + ÷ - sin2 ç - ÷ is 2p
è 8 2ø è 8 2ø
(C) If A + B + C = 180° and cos A = cos Bcos C, then
Answers: (A), (B), (C), (D) tan A – tan B - tan C = 0
Solution: (D) For all values of q , 3 - cos q + cos((p / 3) + q )
lies in the interval [2, 4]
(A) If a, b are positive, then
Solution:
a+b
³ ab (A) Given sin A + sin B = 3 (cos B - cos A). Now two
2
cases arise
and equality holds if and only if a = b. Therefore Case 1: cos B – cos A = 0
Therefore
27 tan2 q + 3 cot2 q ³ 2 (27 tan2 q )(3 cot2 q )
sin A + sin B = 0
= 2 ´ 9 = 18
sin A = - sin B
Hence the minimum value of 27 tan q + 3 cot q 2 2
sin 3 A = 3 sin A - 4 sin3 A
is 18 and it will occur when 27 tan 2 q = 3 cot2 q. So
(A) is true. = - 3 sin B + 4 sin3 B
(B) It is known that cos(180° - q)
q = -cos q. Therefore = - sin 3B
the given expression is sin 3 A + sin 3B = 0
74 Chapter 1 Trigonometric Ratios and Transformations

Case 2: cos B – cos A ¹ 0 tan A - tan B - tan C = 0


Therefore
Hence (C) is true.
sin A + sin B
= 3 æp ö
cos B - cos A (D) 3 - cos q + cos ç + q ÷
è3 ø
2 sin[( A + B)/ 2]cos[( A - B)/ 2]
= 3 1 3
2 sin[( A + B)/ 2] sin[( A - B)/ 2] = 3 - cos q + cos q - sin q
2 2
æ A - Bö 1 p
tan ç ÷ = = tan 1
è 2 ø 3 6 = 3 - [cos q + 3 sin q ]
2
A-B p
= np + The maximum and minimum values of cosq + 3 sinq
2 6
are ± 1 + 3 = ± 2. Therefore, the given expression
p
A = B + 2 np + lies in [2, 4]. So, (D) is true.
3
Answers: (A), (C), (D)
sin 3 A = sin(3B + 6 np + p )
= sin(p + 3B) 13. Which of the following are true?
(A) If cos a + cos b = 0 = sin a + sin b, then cos 2a +
= - sin 3B cos 2bb is equal to 2 cos(a + b)
sin 3 A + sin 3B = 0 (B) sin 18° and cos2 36° are roots of the quadratic
2

equation 16x2 - 12x + 1 = 0


Therefore (A) is true.
(C) If sin q = l sin(q + 2a),
a then tan(q + a a) equals
p 3p 5p 7p
(B) cos2 + cos2 + cos2 + cos2 æ1 + lö
16 16 16 16 çè 1 - l ÷ø tan a
1 + cos(p / 8) 1 + cos(3p / 8)
= +
2 2 1 + cos A m 2 mn
(D) If = , then tanA = 2
1 + cos(5p / 8) 1 + cos(7p / 8) 1 - cos A n m + n2
+ +
2 2
Solution:
1é p 3p 5p 7p ù
= 2 + êcos + cos + cos + cos ú (A) (cos a + cos b) 2 + (sin a + sin b) 2 = 0 Þ 1 + cos
2ë 8 8 8 8 û
(a - b) = 0
1é p 3p 3p pù Therefore
=2+ ê cos + cos - cos - cos ú
2ë 8 8 8 8û
cos(a - b ) = - 1
=2+0=2
cos 2a + cos 2 b = 2 cos(a + b )cos(a - b )
Hence (B) is not true.
(C) Given A + B + C = 180° and cos A = cos B cos C. Hence (A) is not true.
Now,
5-1 5+1
A + B + C = 180°Þ tan A + tan B + tan C (B) sin18° = and cos 36° =
4 4
= tan A tan B tan C
Therefore sin 2 18° and cos 2 36° are roots of
Also
éæ 5 - 1 ö 2 æ 5 + 1 ö 2 ù 2
æ 5 - 1ö æ 5 + 1ö
2

cos A = cos B cos C Þ - cos( B + C ) = cos B cos C x - êç


2
÷ +ç ÷ úx +ç 4 ÷ ç 4 ÷ = 0
êëè 4 ø è 4 ø úû è ø è ø
Þ - cos B cos C + sin B sin C = cos B cos C
12 x 1
Þ sin B sin C = 2 cos B cos C x2 - + =0
16 16
Þ tan B tan C = 2
16 x2 - 12 x + 1 = 0
Therefore
Hence (B) is true.
tan A + tan B + tan C = tan A tan B tan C = 2 tan A
Worked-Out Problems 75

1 sin(q + 2a ) 1
(C) = [(cos A - cos B)2 + (cos B - cos C )2
l sin q 2
Therefore + (cos C - cos A)2 ] = 0
cos A = cos B = cos C
l + 1 sin(q + 2a ) + sin q
= A= B=C (∵ A + B + C = 180°)
1 - l sin(q + 2a ) - sin q
Answers: (A), (B), (C), (D)
2 sin(q + a )cos a
=
2 cos(q + a )ssin a 15. For a positive integer n, let

= tan(q + a )cot a æ qö
fn (q ) = ç tan ÷ (1 + sec q )(1 + sec 2q )
è 2ø
æ l + 1ö
tan(q + a ) = ç tan a
è 1 - l ÷ø (1 + sec 4q )  (1 + sec 2n q )

Hence (C) is true. Then


æpö æpö
m 2 cos2 ( A / 2) A (A) f2 ç ÷ = 1 (B) f3 ç ÷ = 1
(D) = = cot è 16 ø è 32 ø
2
n sin ( A / 2) 2
æpö æ p ö
(C) f4 ç ÷ = 1 (D) f5 ç =1
è 128 ÷ø
Therefore
è 64 ø
A n 2 tan( A / 2) 2 mn
tan = and tan A = = 2 Solution:
2 m 1 - tan ( A / 2) m - n2
2

So (D) is not true. q sin(q / 2) æ 1 + cos q ö


tan (1 + sec q ) = = tan q
Answers: (B), (C) 2 cos(q / 2) çè cos q ÷ø

Therefore
14. If A + B + C = 180° where A, B, C are positive angles
and fn (q ) = tan q (1 + sec 2q )(1 + sec 4q )  (1 + sec 2n q )
cos A cos B + cos B cos C + cos C cos A = tan 2q (1 + sec 4q )(1 + sec 8q )  (1 + sec 2n q )
= 1 - 2 cos A cos B cos C Finally
then fn (q ) = tan 2n - 1 q (1 + sec 2n q ) = tan(2n q )
(A) A = B (B) B = C
Therefore
(C) C = A (D) A = B = C
Solution: æpö æ pö p
f2 ç ÷ = tan ç 22 ÷ = tan = 1
è 16 ø è 16 ø 4
S cos B cos C
Hence (A) is true.
= 1 - 2 cos A cos B cos C
æpö æ pö p
= 1 - cos A[cos( B + C ) + cos( B - C )] f3 ç ÷ = tan ç 23 × ÷ = tan = 1
è 32 ø è 32 ø 4
= 1 + cos2 A + cos( B + C )cos( B - C )
So (B) is true.
(∵ A + B + C = 180°)
æpö æ pö p
f4 ç ÷ = tan ç 24 ÷ = tan = 1
= 1 + cos A + cos B - sin C
2 2 2
è 64 ø è 64 ø 4

= cos2 A + cos2 B + cos2 C So (C) is true.

Therefore æ p ö æ p ö p
f5 ç = tan ç 25 × ÷ø = tan = 1
è 128 ÷ø è 128 4
cos2 A + cos2 B + cos2 C - cos B cos C - cos C cos A
- cos A cos B = 0 Hence (D) is true.
Answers: (A), (B), (C), (D)
76 Chapter 1 Trigonometric Ratios and Transformations

16. Which of the following are true Therefore


-2 -p p
(A) If 90° < q < 180° and tan q = , then < q < Þ cos q - sin q > 0
3 4 4
tan(90° + q ) + cos(180° + q ) 2 + 13 Hence (C) is true.
=
sin(270° - q ) - cot(-q ) 2 - 13 a b
(D) = = k (say)
For any angle q, cos x cos y
so that
sin 2q cos q q
= tan a+b
(1 + cos 2q )(1 + cos q ) 2 k=
cos x + cos y
-p p
If < q < , then cosq - sinq is always positive Now
4 4
cos x a a tan x + b tan y = (k cos x)tan x + (k cos y)tan y
(D) If = , then a tan x + b tan y is equal to
cos y b = k(sin x + sin y)
æ x + yö
(a + b)tan ç
è 2 ÷ø =
(a + b)(sin x + sin y)
cos x + cos y
Solution: q > 0, cos q < 0,
tan q < 0 and cot q < 0. æ x + yö
= (a + b)tan ç
è 2 ÷ø
tan(90° + q ) + cos(180° + q )
(A) Therefore (D) is true.
sin(270° - q ) - cot(-q )
Answers: (A), (B), (C), (D)
- cot q - cos q
=
- cos q + cot q 17. Which of the following are true?
cot q + cos q (-3 / 2) + (-3 / 13 ) æp ö
= = (A) If x is any real number, then tan ç sin2 x÷ lies
cos q - cot q -(3 / 13 ) + (3 / 2) è4 ø
between 0 and 1
2 + 13 p 3p 5p 7p
= (B) Thevalue of cos4 + cos4 + cos4 + cos4
2 - 13 8 8 8 8
is 2/3
Therefore (A) is true. (C) If a and b are two different solutions lying
sin 2q cos q 2 sin q cos2 q between –p/2p and p p/2 of the equation 2 tanq +
(B) = secq = 2, then the value of tana + tanb
b is 4/3
(1 + cos 2q )(1 + cos q ) (2 cos2 q )(2 cos2 (q / 2)
æqö æ 3q ö æqö
2 sin(q / 2)cos(q / 2) (D) The value of 2 sin ç ÷ cos ç ÷ + 4 sin q sin2 ç ÷
= è 2ø è 2ø è 2ø
2 cos2 (q / 2) equals sin q.
q Solution:
= tan
2
æp ö
(A) 0 £ sin2 x £ 1 for all x Þ 0 £ tan ç sin2 x÷ £ 1
Therefore (B) is true. è4 ø
-p Left equality holds when x is a multiple of p and
(C) < q < 0 Þ cos q > 0 and sinq < 0
4 right equality holds when x is an odd multiple of p/2.
p
Therefore (A) is true.
Þ cos q - sin q > 0
(B) The given expression is equivalent to
p
0 < q < Þ cos q > sin q 2 2
4 p 3p 1 æ pö 1æ 3p ö
2 cos4 + 2 cos4 = ç 1 + cos ÷ + ç 1 + cos ÷
Þ cos q - sin q > 0 8 8 2è 4ø 2 è 4ø
2 2
q = 0 Þ cos q - sin q = 1 1æ 1 ö 1æ 1 ö
= ç 1+ ÷ + ç1 - ÷
2è 2ø 2è 2ø
Worked-Out Problems 77

æ 1ö 3 (B) cos(a - b)
b = -1 Þ a - b = (2k + 1)p
p (where k is an
= ç1 + ÷ = integer)
è 2ø 2
Hence (B) is not true. æa + bö æa - bö
sin a + sin b = 2 sin ç cos ç
è 2 ÷ø è 2 ÷ø
(C) tan a, tan b are roots of the equation
sec2 q = (2 - 2 tan q )2 æa + bö p
= 2 sin ç cos(2k + 1) = 0
è 2 ÷ø 2
3 tan q - 8 tan q + 3 = 0
2

Therefore æa + bö æa - bö
cos a + cos b = 2 cos ç ÷ cos ç
8 è 2 ø è 2 ÷ø
tan a + tan b =
3 æa + bö p
= 2 cos ç cos(2k + 1) = 0
Hence (C) is not true. è 2 ÷ø 2
æqö æ 3q ö æqö The converse is clear. Hence (B) is true.
(D) 2 sin ç ÷ cos ç ÷ + 4 sin q sin2 ç ÷
è 2ø è 2ø è 2ø
(C) 1 - (sin2 A + sin2 B + sin2 C )
= (sin 2q - sin q ) + 2 sin q (1 - cosq )
= (sin 2q - sin q ) + (2 sin q - sin 2q ) é 1 - cos 2 A 1 - cos 2 B 1 - cos 2C ù
=1- ê + + úû
= sin q ë 2 2 2

Hence (D) is true. -1 1


= + [cos 2 A + cos 2 B + cos 2C ]
Answers: (A), (D) 2 2
-1 1
18. Which of the following are true? = + [-1 - 4 cos A cos B cos C ]
2 2
(A) If a, b are positive numbers, then the values of r and = - 1 - 2 cos A cos B cos C
q (0 £ q £ p)
p for which asinx + bcosx = rsin(x ( + q)
q
are r = a + b and tanq = b/a
2 2
/ Hence (C) is not true.
(D) tan a + tan b = b, tan a tan b = c
(B) cos(a - b)
b = -1, if and only if sin a + sin b = 0 =
Now
cos a + cos b
(C) If A + B + C = 180°, then the value of 1 -(sin2 A + tan a + tan b b
tan(a + b ) = =
sin2 B + sin2 C) is 2 sin Asin Bsin C 1 - tan a tan b 1 - c
(D) If tan a and tan b are roots of the equation
1 - tan2 (a + b )
x2 - bx + c = 0, then the value of sin 2(a + bb) is cos 2(a + b ) =
b2/(b2 + c2) 1 + tan2 (a + b )

Solution: 1 - [b2 /(1 - c)2 ]


=
(A) a sin x + b cos x 1 + [b2 /(1 - c)2 ]

é ù (1 - c)2 - b2
= a +b ê
2 a
2
sin x +
b
cos x ú =
(1 - c)2 + b2
êë a + b2
2
a2 + b2 úû

= a2 + b2 [cos q sin x + sin q cos x]


1 - cos 2(a + b )
= a2 + b2 sin( x + q ) sin2 (a + b ) =
2
where
1é (1 - c)2 - b2 ù
= ê 1 - ú
a b 2ë (1 - c)2 + b2 û
cos q = and sin q =
a +b
2 2
a + b2
2

b2
=
(1 - c))2 + b2
r = a2 + b2 and tan q = b / a So (D) is not true.
(A) is true. Answers: (A), (B)
78 Chapter 1 Trigonometric Ratios and Transformations

Matrix-Match Type Questions


1. Match the items of Column I with those of Column II. q qö
2
æ
cos + sin ÷
1 + sin q çè 2 2ø
(D) sec q + tan q = =
Column I Column II cos q q q
cos2 - sin2
2 2
æ sin 2q ö æ cos q ö q
(A) ç ÷ ç ÷ is equal to (p) cot q q q
è 1 + cos 2q øè 1 + cos q ø 2 cos + sin 1 + tan
= 2 2 = 2
æ sin 2q ö æ 1 - cos q ö q q q q
(B) ç equals (q) tan cos - sin 1 - tan
è 1 - cos 2q ÷ø çè cos q ÷ø 2 2 2 2
Answer: (D) Æ (s)
cot q - 1 1 - sin 2q
(C) = (r)
cot q + 1 cos 2q 2. Match the items of Column I with those of Column II.

q
1 + tan Column I Column II
(D) sec q + tan q = (s) 2
q
1 - tan (A) The value of cos2q + cos2(60° + q)
q + (p)
3
2 2
cos2(60° - q
q) is
Solution: 1
(B) cos 20° cos 40° cos 80° (q)
(A) 8

æ sin 2q ö æ cos q ö æ 2 sin q cos2 q ö æ 1 ö 3


çè 1 + cos 2q ÷ø çè 1 + cos q ÷ø = çè 2 cos2 q ÷ø çè 2 cos2(q / 2) ÷ø (C) sin2 q + sin2 (120° + q ) + sin2 (120° - q ) (r)
equals 8
2 sin(q / 2)cos(q / 2)
= 3
2 cos2 (q / 2) (D) sin 20° sin 40° sin 80° is equal to (s)
8
q
= tan
2 Solution:
Answer: (A) Æ (q) (A) Given expression is equivalent to
æ sin 2q ö æ 1 - cos q ö (2 sin q cos q )2 sin2 (q / 2) 1
(B) ç = [3 + cos 2q + cos(120° + 2q ) + cos(120° - 2q )]
è 1 - cos 2q ÷ø çè cos q ÷ø (2 sin2 q )(cos q ) 2

2 sin2 (q / 2) 1é 1 3 1
= 3 + cos 2q - cos 2q - sin 2q - cos 2q
=
sin q 2 êë 2 2 2
2 sin2 (q / 2) 3 ù
= + sin 2q ú
2 sin(q / 2)cos(q / 2) 2 û
æqö 3
= tan ç ÷ =
è 2ø 2
Answer: (B) Æ (q) Answer: (A) Æ (p)
(B) cos 20°cos 40°cos 80°
1 - sin 2q (cos q - sin q )2
(C) =
cos 2q cos2 q - sin2 q = cos q cos(60° - q )cos(60° + q )
cos q - sin q cos 3q
= =
cos q + sin q 4
cot q - 1 cos 60° 1
= = =
cot q + 1 4 8
Answer: (C) Æ (r) where q = 20°.
Answer: (B) Æ (q)
Worked-Out Problems 79

(C) Given expression is (B) A + B + C = 180°


3 1
- [cos 2q + cos(240° + 2q)
q + cos(240° - 2q)]
q Þ tan A + tan B + tan C = tan A tan B tan C
2 2 Þ 6 = 2 tan C

=
3
-0=
3 Þ tan C = 3
2 2 Answer: (B) Æ (s)
Answer: (C) Æ (p) cos 70° + 4 cos 70°sin 70°
(C) cot 70° + 4 cos 70° =
(D) Given that sin 20°sin 40°sin 80° = sin q sin(60° - q)
q sin 70°
sin(60° + q)
q cos 70° + 2 sin 140°
=
sin 3q sin 70°
=
4 cos 70° + 2 sin 40°
=
sin 60° 3 sin 70°
= =
4 8 sin 20° + sin 40° + sin 40°
=
where q = 20°. sin 70°
Answer: (D) Æ (r) 2 cos 10°sin 30° + sin 40°
=
sin 70°
3. Match the items of Column I with those of Column II. cos 10° + cos 50°
=
sin 70°
Column I Column II 2 cos 30° cos 20°
=
(A) tan 9° - tan 27° - tan 63° + tan 81° is (p) 0 sin 70°
equal to
3 cos 20°
(B) If tan A + tan B + tan C = 6 and (q) 3 = = 3
cos 20°
tan A tan B = 2 where A + B +
C = 180°, then tan C value is Answer: (C) Æ (q)
(C) The value of cot 70° + 4 cos 70° is (r) 4 1 - tan q2
(D) cos 2q + sin2 a = + sin2 a
(D) If tan2 q = 2 tan2 a + 1, then the value (s) 3 1 + tan2 q
of cos 2q + sin2a is
1 - (2 tan2 a + 1)
= + sin2 a
1 + (2 tan2 a + 1)
Solution:
- tan2 a
(A) tan 9° - tan 27° - tan 63° + tan 81° = + sin2 a
1 + tan2 a
= (tan 9° + tan 81°) - (tan 27° + tan 63°)
= - sin2 a + sin2 a = 0
= (tan 9° + cot 9°) - (tan 27° + cot 27°)
Answer: (D) Æ (p)
æ sin 9° cos 9° ö æ sin 27° cos 27° ö
=ç + - +
è cos 9° sin 9° ÷ø çè cos 27° sin 27° ÷ø 4. Match the items of Column I with those of Column II.
1 1
= -
sin 9° cos 9° sin 27° cos 27°° Column I Column II
2 2 4
= - (A) If tan q = , then the value of (p) 2
sin 18° sin 54° 5
2[sin 54° - sin 18°] 5 sin q - 3 cos q
= is
sin 18°sin 54° 5 sin q + 2 cos q
(B) The minimum value of cos2 q + sec2 q is (q) 2n
4 cos 36°sin 18°
= (C) The minimum value of 3 sin2q + (r) 2n+1
sin 18°sin 54° 2 cos2q is
= 4 (∵ cos 36° = sin 54°) (D) If sin x + cosec x = 2, then for any (s) 1/6
Answer: (A) Æ (r) positive integer n, sinn x + cosecnx is
equal to
80 Chapter 1 Trigonometric Ratios and Transformations

Solution:
5 sin q - 3 cos q 5 tan q - 3 5(4 / 5) - 3 1
(A) = = = æp ö
5 sin q + 2 cos q 5 tan q + 2 5(4 / 5) + 2 6 tan(2 n - 1)q = tanç - q ÷ = cot q
è2 ø
Answer: (A) Æ (s)
æp ö
tan 2(n - 1)q = tanç - 2q ÷ = cot 2q
(B) cos2 q + sec2 q ³ 2 cos2 q × sec2 q = 2 è2 ø
Therefore minimum value of cos2q + sec2q is 2. æp ö
tan(2 n - 3)q = tanç - 3q ÷ = cot 3q
Answer: (B) Æ (p) è2 ø
(C) 3 sin2 q + 2 cos2 q = 2(sin2 q + cos2 q ) + sin2 q   
= 2 + sin2 q ³ 2 p
tan nq = tan =1
Answer: (C) Æ (p) 4
sin x + cosec x = 2 Þ (sin x - 1)2 @ 0
The given expression is equal to
Þ sin x = 1
(tan q cot q )(tan 2q cot 2q )(tan 3q cot 3q ) 
Therefore (tan(n - 1)q cot(n - 1)q )tan nq
sin x + cosec x = 1 + 1 = 2
n n
æ p ö
= (1)(1)(1) (1) 1 ç∵ tan nq = tan = 1÷
Answer: (D) Æ (p) è 4 ø
=1
5. Match the items of Column I with those of Column II.
Answer: (B) Æ (q)
a - bö æa + bö
Column I Column II sinæç
÷ sinç ÷
a - bö æa + bö è
2 ø è 2 ø
(C) tanæç =
è 2 ÷ø çè ÷
-1 tan
( - B) = 3/5 and tanAtanB = 2, (p)
(A) If cos(A 2 ø cosæ a - b ö cosæ a + b ö
çè ÷ çè ÷
the value of cosAcosB is
3 2 ø 2 ø
(B) If q = p /4n (n is a positive integer), (q) 1 sin2 (a / 2) - sin2 (b / 2)
=
then the value of tan q tan 2q tan 3q cos2 (a / 2) - sin2 (b / 2)
 tan 2(n - 1)q tan(2n - 1)q q is
(1 - cos a ) - (1 - cos b )
1 =
(C) If cos a = 2 cos b, then the value of (r) (1 + cos a ) - (1 - cos b )
5
æa - bö æa + bö - cos a + cos b
tanç tanç =
è 2 ÷ø è 2 ÷ø cos a + cos b
(D) If A + B + C = 180°, sin Acos B = 1/4 (s) 3 - 2 cos b + cos b -1
and 3 tan A = tan B, then cot 2A is = =
2 cos b + cos b 3
equal to
Answer: (C) Æ (p)
Solution: (D) 3 tan A = tan B Þ 3 sin A cos B = sin B cos A
3
(A) cos A cos B + sin A sin B = (1.13) 3
5 Þ = cos A sin B
4
tan A tan B = 2 Þ sin A sin B = 2 cos A cos B
Therefore
Therefore from Eq. (1.13) we have
sin( A + B) = sin A cos B + cos A sin B
3
3 cos A cos B = 1 3
5 = +
4 4
1
cos A cos B = =1
5
This implies
Answer: (A) Æ (r)
p p A + B = 90°
q= Þ 2 nq =
4n 2 C = 90°
Worked-Out Problems 81

3 tan A = tan B = tan(90° - A) = cot A 3 (2 + 3 )


=
cot A = 3
2 2 (2 + 3 )
Answer: (D) Æ (s) 3
=
2
6. Match the items of Column I with those of Column II.
This implies
Column I Column II p 2p
x= or
3 3
p
(A) If 0 < a < p
p/16 and (1 + tan a)
a (p) Answer: (B) Æ (p)
(1 + tan 4a)
a = 2 then a is equal to 3
3 + cot 80°cot 20°
p (C)
(q) cot 80° + cot 20°
(B) If 0 < x < p / 2 and 1 + sin x + sin2x + 20
sin3x +  + ¥ is equal to 4 + 2 3, 3 sin 80° sin 20° + cos 80° cos 20°
5p =
then x equals (r) sin 20°cos 80° + cos 20°sin 80°
18
3 + cot 80°cot 20° n 20°
2 sin 80°sin 20° + cos 80°cos 20° + sin 80°sin
(C) The value of is =
cot 80° + cot 20° 2p sin100°
(s)
equal to tan q, then q can be 3
(cos 60° - cos 100°) + cos 60°
=
p sin 100°
(D) If 2 sec 2q = tan a + cot a, then one (t)
of the values of a + q is equal to 4
1 - cos 100°
=
sin 100°
Solution:
= tan 50°
(A) We have
Therefore
1 + tana + tan 4a + tana tan 4a = 2
5p
Therefore q=
18
tan(a + 4a ) = 1 Answer: (C) Æ (r)
p æ pö 2 1 2
5a = = =
çè∵ 0 < a < ÷ø (D)
cos 2q sin a cos a sin 2a
4 16
p Therefore
a=
20 cos 2q = sin 2a
Answer: (A) Æ (q) p
Hence one of the values of 2q = - 2a
1 2
(B) =4+2 3
1 - sin x
p
So one of the values of a + q is
1 4
sin x = 1 -
4+2 3
Answer: (D) Æ (t)
3+2 3
=
4+2 3

Comprehension-Type Questions
1. Passage: sin(A ± B) = sin A cos B ± cos A sin B and sin A + sin 2 A + sin 4 A + sin 5 A
cos(A ± B) = cos A cos B ∓ sin A sin B. Based on this (ii) =
cos A + cos 2 A + cos 4 A + cos 5 A
information, answer the following questions.
(i) sin A + 2 sin 3A + sin 5A is equal to (A) cot 3A (B) tan 3A
(A) 4 sin 3A cos2A (B) 4 cos 3 A sin2A (C) 2 cot 3A (D) 2 tan 3A
(C) 4 sin 3A sin2A (D) 4 cos 3 A cos2A cos 7 A + cos 3 A - cos 5 A - cos A
(iii) =
sin 7 A - sin 3 A - sin 5 A + sin A
82 Chapter 1 Trigonometric Ratios and Transformations

(A) cot 2A
2 (B) tan 2A
2 (ii) (cot B + cot C )(cot C + cot A)(cot A + cot B)
(C) 2 cot 22A (D) 2 tan 22A equals
Solution: From the given information, we have (A) sin A sin B sin C
(B) sec A sec B sec C
2 sin A cos B = sin( A + B) + sin( A - B)
(C) cosec A cosec B cosec C
2 cos A sin B = sin( A + B) - sin( A - B) (D) cos A cos B cos C
2 cos A cos B = cos( A + B) + cos( A - B) 1 + cos A - cos B + cos C
(iii) =
2 sin A sin B = cos( A - B) - cos( A + B) 1 + cos A + cos B - cos C
(i) sin A + 2 sin 3 A + sin 5 A = (sin A + sin 5 A) + 2 sin 3 A B C B C
(A) cot tan (B) tan cot
2 2 2 2
= 2 sin 3 A cos 2 A + 2 sin 3 A
A B C A
= 2 sin 3 A(cos 2 A + 1) (C) tan cot (D) tan cot
2 2 2 2
= 4 sin 3 A cos2 A
Solution:
Answer: (A)
(i) 1 - 2 sin B sin C cos A + cos2 A
(ii) Numerator = (sin A + sin 5 A) + (sin 2 A + sin 4 A)
= 1 + cos2 A - [cos( B - C ) - cos( B + C )]cos A
= 2 sin 3 A cos 2 A + 2 sin 3 A cos A
= 1 + cos2 A - cos( B - C )cos A + cos( B + C )cos A
= 2 sin 3 A(cos 2A + cos A)
Denominator = (cos A + cos 5 A) + (cos 2 A + cos 4 A) = 1 + cos2 A + cos( B - C )cos( B + C ) - cos2 A

= 2 cos 3 A cos 2 A + 2 cos 3 Acos A = 1 + cos2 B - sin2 C = cos2 B + cos2 C


Answer: (A)
= 2 cos 3 A(cos 2A + cos A) (ii) (cot B + cot C )(cot C + cot A)(cot A + cot B)
Therefore the given quotient is sin( B + C ) sin(C + A) sin( A + B)
= × ×
sin 3 A sin B sin C sin C sin A sin A sin B
= tan 3 A
cos 3 A 1
Answer: (B) = = cosec A cosec B cosec C
sin A sin B sin C
(iii) Numerator = (cos7A
7 - cosA) + (cos3A - cos5A
5 )
Answer: (C)
= - 2 sin 4 A sin 3 A + 2 sin 4 A sin A (iii) Numerator
= - 2 sin 4 A(sin 3 A - sin A) A+ B A- B C
= 1 - 2 sin sin + 2 cos2 - 1
= - 2 sin 4 A(2 cos 2 A sin A) 2 2 2

= -4 sin 4 A cos 2 A sin A


Cé C æ A - Bö ù
= 2 cos êcos - sinç
2ë 2 è 2 ÷ø úû
Denominator = (sin 7 A + sin A) - (sin 3 A + sin 5 A)
Cé A+ B A - Bù
= 2 sin 4 A cos 3 A - 2 sin 4 A cos A = 2 cos êsin - sin
2ë 2 2 úû
= 2 sin 4 A(cos 3 A - cos A)
Cæ A Bö
= 2 cos ç 2 cos sin ÷
= 2 sin 4 A(- 2 sin 2 A sin A) 2è 2 2ø
= - 4 sin 4 A sin 2 A sin A A B C
= 4 cos sin cos
Therefore the given quotient is cot 2A
2 . 2 2 2
Answer: (A) A B C
Similarly, the denominator = 4 cos cos sin
2 2 2
2. Passage: It is given that A, B and C are angles such Therefore the given quotient is
that A + B + C = p.
p Answer the following questions.
(i) 1 - 2 sin B sin C cos A + cos2 A equals B C
tan cot
2 2
(A) cos2 B + cos2 C (B) cos2 B - cos2 C
Answer: (B)
(C) sin2 B + sin2 C (D) sin2 B - sin2 C
Worked-Out Problems 83

Assertion–Reasoning Type Questions


In the following set of questions, a Statement I is given This implies that either
and a corresponding Statement II is given just below it.
a+b
Mark the correct answer as: sin =0
2
(A) Both Statements I and II are true and Statement II
is a correct explanation for Statement I. æa - bö a-b
cosç = - 3 sin
è 2 ÷ø
or
(B) Both Statements I and II are true but Statement II is 2
not a correct explanation for Statement I.
Therefore
(C) Statement I is true and Statement II is false.
(D) Statement I is false and Statement II is true. æ a - b ö -1 æ -p ö
a + b = 2np or tan ç = = tan ç
è 2 ø ÷
3 è 6 ÷ø
1. Statement I: If 0 < q < p
p/2 and cos 2q = 2(cos q - a-b p
sin q
q), then tan q is equal to 1. a + b = 2 np or = np -
2 6
Statement II: If 0 < a,
a b<p
p/2 and sin(a + b)
b = 1, then p
a + b = p/2.
p a + b = 2 np or a = b + 2 np -
3
Solution: Statement II is clear. Now
a + b = 2np
p implies
cos 2q = 2 (cos q - sin q ) Þ cos2 q - sin2 q
sin 3a = sin(6np - 3b) or sin 3a = sin(6np + 3b - p)
p
= 2 (cos q - sin q )
In any case sin 3a = -sin 3b
b so that sin 3a + sin 3b = 0.
This is true when cos q = sin q (i.e., q = p
p/4) in which case Answer: (A)
tan q = 1. Suppose cos q ¹ sin q. Therefore

cos q + sin q = 2 3. Statement I: sin(p /18) is a root of 8 x3 - 6 x + 1 = 0.

æp ö Statement II: For any real a , sin 3a = 3 sin a - 4 sin3 a .


sin ç + q ÷ = 1
è4 ø Solution: Statement II is a standard formula. Put a = p/18.
p
p p Therefore
+q =
4 2 p 1
3a = and hence sin 3a =
p 6 2
q=
4 1
tan q = 1 3 sin a - 4 sin3 a =
2
Answer: (A) 8 sin3 a - 6 sin a + 1 = 0

2. Statement I: If sin a + sin b = 3(cos a - cos b), then That is sin a = sin(p / 18) is a root of 8 x3 - 6 x + 1 = 0.
sin 3a + sin 3b = 0. Answer: (A)
Statement II: sin q = 0 Û q = npp and tan q = tan f Û
4. Statement I:
q = np + f, where n is an integer.
1 1 1
Solution: + + +
cos 0°cos1° cos 1°cos 2° cos 2°cos 3°
sin a + sin b = 3 (cos a - cos b )
1 cos 1°
+ =
If cos a = cos b , then sin a = - sin b . Therefore cos 88°cos 89° sin2 1°
3 sin a - 4 sin3 a = - 3 sin b + 4 sin3 b sin( A - B)
Statement II: = tan A - tan B.
sin 3a = - sin 3b cos A cos B
sin 3a + sin 3b = 0 Solution:

Suppose cos a ¹ cos b. Then sin( A - B) sin A cos B - cos A sin B


=
cos A cos B cos A cos B
a+b æa - bö a+b a -b
2 sin cosç ÷ = - 3 ´ 2 sin sin = tan A - tan B
2 è 2 ø 2 2
Statement II is true.
84 Chapter 1 Trigonometric Ratios and Transformations

Now z[2i sin(nq / 2)][cos(nq / 2) + i sin(nq / 2)]


=
sin 1° sin 1° sin 1° 2i sin(q / 2)(cos(q / 2) + i sin(q / 2))
+ +
cos 0°cos 1° cos 1°cos 2° cos 2°cos 3° sin(nq / 2)(cos q + i sin q)[cos(nq / 2) + i sin(nq / 2)]
=
sin 1° sin(q / 2)[cos(q / 2) + i sin(q / 2)]
++
cos 88°cos 89°
sin(nq / 2) é æ n 1ö æn 1ö ù
= cosç + 1 - ÷ q + i sinç + 1 - ÷ q ú
=
sin(1° - 0°) sin(2° - 1°) sin(3° - 2°)
+ + sin(q / 2) êë è 2 2ø è2 2ø û
cos 0°cos 1° cos 1°cos 2° cos 2°cos 3°
sin(nq / 2) é æ n + 1ö æ n + 1ö ù
sin(89° - 88°) = cosç ÷ q + i sinçè ÷q
++ sin(q / 2) êë è 2 ø 2 ø úû
cos 88°cos 89°
= (tan 1° - tan 0°) + (tan 2° - tan 1°) + (tan 3° - tan 2°) Equating real and imaginary parts both sides, we have

+  + (tan 89° - tan 88°) sin(nq / 2) æ æ n + 1ö ö


Cn = cosç q
ç
sin(q / 2) è è 2 ÷ø ÷ø
= tan 89° - tan 0° = cot 1°

Therefore sin(nq / 2) æ æ n + 1ö ö
and Sn = sinç ÷q
sin(q / 2) çè è 2 ø ÷ø
89
1 cot 1° cos 1°
å cos(k - 1)cos k = sin 1° = sin 1°
k =1
2
Answer: (A)

6. Statement I: (4 cos 9° - 3)(4 cos 27° - 3) = tan 9°.


2 2
Statement I is true.
Answer: (A)
Statement II: If x ¹ (2 n + 1)(p / 2) where n is an integer,
then
5. Statement I: Let n be a positive integer and let
cos 3 x
Sn = sin q + sin 2q + sin 3q +  + sin nq 4 cos2 x - 3 =
cos x
and
Solution: We know that
Cn = cos q + cos 2q + cos 3q +  + cos nq
cos 3 x = 4 cos3 x - 3 cos x
Then Therefore
æ nq ö æ æ n + 1ö ö p cos 3 x
çè sin ÷ø çè sinçè ÷q x ¹ (2 n + 1) Þ = 4 cos2 x - 3
2 2 ø ÷ø 2 cos x
Sn =
sin(q / 2)
Hence Statement II is true. Now
æ nq ö æ æ n + 1ö ö cos 27° cos 81°
çè sin ÷ø çè cosçè ÷q
2 2 ø ÷ø (4 cos2 9° - 3)(4 cos2 27° - 3) = ×
cos 9° cos 27°
and Cn =
sin(q / 2)
cos 81° sin 9°
Statement II: f z = cos q + isin q, then zk = cos kq
q+ = = = tan 9°
cos 9° cos 9°
isin kq, where k is an integer.
Answer: (A)
Solution: Statement II is true (by De’Moivre’s Theorem).
Now, by Statement II, we have
7. Statement I: sin4 x + 4 cos2 x - cos4 x + 4 sin2 x = cos2x
2
z(z - 1)
n
Cn + iSn = z + z2 + z3 +  + zn = Statement II: cos2 x + sin2 x = 1
z-1
Solution: Statement II is obvious.
[cos nq + i sin nq - 1]
=z
cos q + i sin q - 1 sin4 x + 4 cos2 x - cos4 x + 4 sin2 x
[- 2 sin2 (nq / 2) + 2i sin(nq / 2)cos(nq / 2)]
=z = sin4 x + 4(1 - sin2 x) - cos4 x + 4(1 - cos2 x)
-2 sin2 (q / 2) + 2i sin(q / 2)cos(q / 2)
Worked-Out Problems 85

= (2 - sin2 x)2 - (2 - cos2 x)2 = a2 + b2 [cos a sin x ± sin a cos x]

= (2 - sin2 x) - (2 - cos2 x) = a2 + b2 sin( x ± a )


= cos2 x - sin2 x = cos 2 x Statement II is true. Now
Statement I is true. (1 - cot 23°)(1 - cot 22°)
Answer: (A)
(sin 23° - cos 23°)(sin 22° - cos 22°)
=
8. Statement I: (1 - cot 23°)(1 - cot 22°) = 2 sin 23°sin 22°

Statement II: If a and b are non-zero real numbers, 2 sin(23° - 45°) ´ 2 sin(22° - 45°)
=
then asin x ± bcos x = a2 + b2 sin(x ± aa) where a sin 23°sin 22°
satisfies the relations
(- 2 sin 22°)(- 2 sin 23°)
= =2
a b sin 23°sin 22°
cosa = and sina =
a +b
2 2
a + b2
2
Statement I is also true.
Solution: Answer: (A)

é a b ù
a sin x ± b cos x = a2 + b2 ê 2 sin x ± cos x ú
ë a +b a +b
2 2 2
û

Integer Answer Type Questions


1. In DABC, sin(B + C - A) + sin(C + A - B) + sin(A + A B C
B - C) = k sin A sin B sin C where k equals . = 1 - 2 cos cos sin
2 2 2
Solution: A + B + C = p Þ B + C - A = p - 2A
2 , etc. p = 1, q = 2.
Therefore
Answer: 3
LHS = sin(p - 2 A) + sin(p - 2 B) + sin(p - 2C )
1 1
= sin 2 A + sin 2 B + sin 2C 3. If - = cot kq then the value
tan 3q + tan q cot 3q + cot q
= 4 sin A sin B sin C of k is .
Answer: 4 Solution:

2. If A + B + C = p
p, then 1 1
-
tan 3q + tan q cot 3q + cot q
A C B A B C
sin2 - sin2 + sin2 = p - q cos cos sin
2 2 2 2 2 2 1 tan q tan 3q
= -
where p + q is . tan 3q + tan q tan 3q + tan q

Solution: 1 - tan 3q tan q 1


= = = cot 4q
tan 3q + tan q tan 4q
A B C
sin2 + sin2 - sin2 Answer: 4
2 2 2
1 - cos A 1 - cos B 1 - cos C 4. (2 cos A + 1)(2 cos A - 1) - 2 cos 2 A is equal to .
= + -
2 2 2
Solution: (2cosA + 1) (2cosA - 1) - 2cos2A
2 = 4 cos A - 2

1 1 1 - 2(2cos2 A - 1) = 1
= - (cos A + cos B - cos C )
2 2 Answer: 1
1 1æ A B C ö
= - ç 4 cos cos sin - 1÷ 5. Let 0 < a, b < p / 2. If tan a = 1/7 and sin b = 1/ 10, then
2 2è 2 2 2 ø
value of tan(a + 2 b ) is .
86 Chapter 1 Trigonometric Ratios and Transformations

Solution: [(1/ 2)cos 10° - ( 3 / 2)sin 10°]


=2
1 p 3 sin 10° cos 10°
sin b = and 0 < b < Þ cos b =
10 2 10 2 sin(30° - 10°)
=
Therefore sin 10°cos 10°
1 4 sin 20°
tan b = =
3 sin 20°
2 tan b 2(1/ 3) 3 =4
tan 2 b = = =
1 - tan b 1 - (1/ 9) 4
2
Answer: 4
Hence
8. tan q + 2 tan 2q + 4 tan 4q - cot q = -kcot 8q, where the
tan a + tan 2 b (1/ 7) + (3 / 4) 25 value of k is .
tan(a + 2 b ) = = = =1
1 - tan a tan 2 b 1-(3 / 28) 25
Solution:
Answer: 1
1 - tan2 q (1 - tan2 q )
cot q - tan q = =2 = 2 cot 2q
é p 3p 5p 7p ù tan q 2 tan q
6. The value of 2 ê cos4 + cos4 + cos4 + cos4
ë 8 8 8 8 úû
Therefore
is .
cot q - tan q = 2 cot 2q
Solution:
cot q - 2 cot 2q = tan q
7p p 5p 3p
=p - , =p - 2 cot 2q - 4 cot 4q = 2 tan 2q
8 8 8 8
The given expression is equal to 4 cot 4q - 8 cot 8q = 4 tan 4q
Adding we get cot q - 8 cot 8q = tan q + 2 tan 2q + 4 tan 4q
é p 3p ù
4 êcos4 + cos4 ú Therefore
ë 8 8 û
tan q + 2 tan 2q + 4 tan 4q - cot q = - 8 cot 8q
é 1 + cos(p /4) ö 2 æ 1 + cos(3p /4) ö 2 ù
= 4 êæç ÷ø + çè ÷ø ú Answer: 8
ëè 2 2 û
(cos 3 A - cos 5 A)(sin 9 A + sin 3 A)
2 2 9. The value of
æ pö æ pö (sin 7 A + sin A)(cos 5 A - cos 7 A)
= ç 1 + cos ÷ + ç 1 - cos ÷
è 4 ø è 4ø is .
2 2 Solution: Numerator = (2 sin 4 A sin A)(2 sin 6 A cos 3A)
æ 1 ö æ 1 ö
Denominator = (2 sin 4 A cos 3A)(2 sin 6 A sin A)
= ç1 + ÷ + ç1 - ÷
è 2ø è 2ø Therefore quotient = 1.
æ 1ö Answer: 1
= 2ç1 + ÷
è 2ø
q1 + sinq
10. If sinq q2 + sinq
q3 = 3, then cos2 q1 + cos2 q2 + cos2 q3
=3 = .
Answer: 3 Solution: It is known that maximum value of sin q is 1.
Therefore
7. If cosec 10° - 3 sec 10° = k, then k equals .
sin q1 + sin q2 + sin q3 = 3
Solution:
Þ sin q1 = sin q2 = sin q3 = 1
cos 10° - 3 sin 10°
k= cos q1 = cos q2 = cos q3 = 0
sin 10° cos 10°
Answer: 0
Summary 87

SUMMARY

1.1 Domain and ranges of trigonometric functions 1.2 Increasing/decreasing nature of trigonometric func-
(Table 1). tions in different quadrants. Upward  denotes
increasing nature and downward ¯ denotes
Table 1 decreasing nature (Table 2).
Function Domain Range
sin x  [−1, 1]
cos x  [−1, 1]
ì p ü
tan x  - í(2 n + 1) , n Î  ý
î 2 þ
cosec x  - {np,
p n Î}  - (-1, 1)
ì p ü
sec x  - í(2 n + 1) , n Î  ý - (-1, 1)
î 2 þ
cot x  - {np,
p n Î} 

Table 2
Function Quadrant I Quadrant II Quadrant III Quadrant IV

sin x  from 0 to 1 ¯ from 1 to 0 ¯ from 0 to -1  from −1 to 0


cos x ¯ from 1 to 0 ¯ from 0 to −1  from −1 to 0  from 0 to 1
tan x  from 0 to +¥  from −¥ to 0  from 0 to +¥  from −¥ to 0
cosec x ¯ from +¥ to 1  from 1 to +¥  from −¥ to −1 ¯ from −1 to −¥
sec x  from 1 to ¥  from −¥ to −1 ¯ from −1 to −¥ ¯ from +¥ to 1
cot x ¯ from +¥ to 0 ¯ from 0 to −¥ ¯ from +¥ to 0 ¯ from 0 to −¥

1.3 Ratios of some of the standard angles (Table 3).

Table 3
p p p p 2p 3p 5p
Radians 0 p
6 4 3 2 3 4 6
Angle
Degree 0° 30° 45° 60° 90° 120° 135° 150° 180°
Functions
1 1 3 3 1 1
sin q 0
2 2 2 2 2 2
3 1 1 -1 -1 - 3
cos q 1 −1
2 2 2 2 2 2
1 -1
tan q 0 1 3 Not defined - 3 −1 0
3 3
2 2
cosec q Not defined 2 2 1 2 2 Not defined
3 3
2 -2
sec q 1 2 2 Not defined −2 - 2 −1
3 3
1 -1
cot q Not defined 3 1 0 −1 - 3 Not defined
3 3
88 Chapter 1 Trigonometric Ratios and Transformations

(2) cos(2x) = cos x − sin x = 2 cos x – 1 = 1 − 2 sin x,


2 2 2 2
1.4 Period: Let f be a real valued function defined on
a subset E of the real number set . If there exists x x 1 - tan x 2
cos x = cos2 - sin2 , cos 2 x =
positive real number p such that for x and x + p 2 2 1 + tan2 x
belonging to E, f(f x + p) = f( f x), then f is said to be
a periodic function and p is called a period of f. f 1 - cos 2 x 1 + cos2 x
(3) sin x = and cos2 x =
2

Among the periods the least one (if it exists) is called 2 2


the least period of ff. If p is a period of ff, and n is any 2 tan x 2 tan x / 2
(4) tan (2 x) = , tan x = and
positive integer then np is also a period. 1 - tan2 x 1 - tan2 x / 2
1 - cos 2 x
1.5 Least periods of trigonometric functions: tan2 x =
1 + cos 2 x
(1) sin x: 2p (2) cos x: 2p (3) tan x: p
(4) cosec x: 2p (5) sec x: 2p (6) cot x: p (5) sin(3x) = 3 sin x − 4 sin3x
(6) cos(3x) = 4 cos3x − 3 cos x
1.6 Trigonometrical ratios of sum and difference of two
and three angles: 3 tan x - tan3 x
(7) tan 3 x =
(1) sin(x + y) = sin x cos y + cos x sin y, sin(x - y) 1 - 3 tan2 x
= sin x cos y - cos x sin y 1.10 Useful identities:
(2) cos(x + y) = cos xcosy - sinxsiny, cos(x - y) = cosx 1
cosy + sinxsiny (1) sin x sin(60° − x) sin(60° + x) = sin(3x)
4
tan x - tan y 1
( + y) =
(3) tan(x (provided tanxtany ¹ 1) (2) cos x cos(60° − x) cos(60° + x) = cos(3x)
1 + tan x tan y 4
tan x + tan y (3) tan x tan(60° − x) tan(60° + x) = tan(3x).
( − y) =
tan(x (provided tanxtany ¹ - 1)
1 - tan x tan y
1° 1°
1.7 (1) sin(x + y + z) = sin x cos y cos z + cos x sin y cos z + 1.11 Trigonometrical ratios of 7 , 15°, 18°, 22 , 36°,
cos x cos y sin z − sin x sin y sin z. 2 2
54°, 72°, 75° (Table 4):
(2) cos(x + y + z) = cos x cos y cos z − cos x sin y sin z −
sin x cos y sin z − sin x sin y cos z. 1.12 Transformations:
tan x + tan y + tan z - tan x tan y tan z (1) sin(x + y) + sin(x − y) = 2 sin x cos y
( + y + z) =
(3) tan(x
1 - 3 å tan x tan y
æ C + Dö æ C - Dö
sin C + sin D = 2 sinçè ÷ø cosçè ÷
1.8 Useful formulae: 2 2 ø
(1) sin(x + y) sin(x − y) = sin x − sin y
2 2
sin(x + y) − sin(x − y) = 2 cos x sin y
(2) cos(x + y) cos(x − y) = cos x − sin2 y
2

æ C + Dö æ C - Dö
sin C − sin D = 2 cosç
è 2 ÷ø çè 2 ÷ø
sin
1.9 Trigonometrical ratios of multiple and submul-
tiple angles:
(3) cos(x + y) + cos(x − y) = 2 cos x cos y
x x
(1) sin(2x) = 2 sin x cos x, sin x = 2 sin cos and
2 2 æ C + Dö æ C - Dö
cos C + cos D = 2 cos ç
è 2 ÷ø
cos ç
sin 2 x =
2 tan x è 2 ÷ø
1 + tan2 x

Table 4
1° 1°
Function 7 15° 18° 22 36° 54° 72° 75°
2 2

8-2 6 -2 2 3-1 5-1 2- 2 10 - 2 5 5+1 10 + 2 5 3+1


sin q
4 2 2 4 2 4 4 4 2 2

8+2 6 +2 2 3+1 10 + 2 5 2+ 2 5+1 10 - 2 5 5-1 3-1


cos q
4 2 2 4 2 4 4 4 2 2

tan q ( 3 - 2 )( 2 - 1) 2- 3 – 2 -1 – – – 2+ 3
Exercises 89

(4) cos(x + y) − cos(x − y) = −2 sin x sin y 1.14 Sum to n terms of sine and cosine series where the
angles are in AP:
æ C + Dö æ C - Dö
cos C − cos D = -2 sin ç sin ç (1) s i n a + s i n ( a + b ) + s i n ( a + 2 b ) +  +
è 2 ÷ø è 2 ÷ø
sin(a + (n − 1)b) =
1.13 Some of the useful formulae: é 2a + (n - 1)b ù æ nb ö
It is given that A + B + C = 180°(or pp). Then sin ê
ë 2 úû sin çè 2 ÷ø
2 + sin 2B + sin 2C = 4 sin A sin B sin C
(1) sin 2A æ bö
sin ç ÷
2 + cos2B + cos2C = −1 −4 cos AcosB cosC
cos2A è 2ø
A B C
(2) sin A + sin B + sin C = 4 cos cos cos
2 2 2 (2) c o s a + c o s ( a + b ) + c o s ( a + 2 b ) +  +
A B C cos(a + (n − 1)b)
b =
cos A + cos B + cos C = 1 + 4 sin sin sin
2 2 2 é 2a + (n - 1)b ù æ nb ö
(3) tan A + tan B + tan C = tan A tan B tan C and
cos ê
ë 2 úû sin çè 2 ÷ø
A B B C C A æ bö
tan tan + tan tan + tan tan = 1 sin ç ÷
2 2 2 2 2 2 è 2ø

EXERCISES
Single Correct Choice Type Questions
1. The value of sin 36° = å
n
6. Suppose that sin3 x sin 3 x = m=0
Am cos mx, where A0,
( 5 + 1) 10 + 2 5 5+1 A1, , Am are constants and An ¹ 0, then the value of n is
(A) (B) (A) 8 (B) 6 (C) 4 (D) 9
8 4
10 + 2 5 ( 5 - 1) 10 + 2 5 7. If a, b, c and d are smallest positive angles in the
(C) (D)
8 8 ascending order such that the sine of each angle is equal
to a positive constant l, then 4sin(a/2) + 3sin(b/2) +
2 sinq 1 - cosq + sinq 2sin(c/2) + sin(d/
d 2) is equal to
2. If a = , then is equal to
1 + cosq + sin q 1 + sinq (A) 2 1 + l (B) 1+ l
(A) 1 - a (B) 1 + a (C) a (D) 1/a
(C) 1 + l (D) 3 1 + l

sec 8q - 1 8. If cos(a + b)
b = 4/5, sin(a - b)
b = 5/13 and 0 < a, b < p/4
p
3. =
sec 4q - 1 a is equal to
then tan 2a
tan 8q cot 8q 46 56 56 65
(A) (B) (A) (B) (C) (D)
tan 2q cot 2q 33 23 33 33
tan 8q cot 8q
(C) (D) 9. If tan A = (1 - cos B)/sin B, then tan 2A
2
tan 4q cot 4q
B B
(A) cot B (B) tan (C) cot (D) tan B
4. If 0 < q < p / 2 and sin q = 3/5, then tan 2q = 2 2
25 24 25 23
(A) (B) (C) (D) p cos 3p 5p 7p 9p
24 7 7 7 10. cos + + cos + cos + cos =
11 11 11 11 11
5. -sin248° + cos212° = 1
(A) 0 (B) 1 (C) (D) -1
5-1 5-1 2
(A) (B)
8 4
11. If cos 2B = cos(A + C)/cos(A - C), then tan A, tan B,
5+1 5+1 tan C are in
(C) (D)
4 8 (A) GP (B) HP (C) AP (D) AGP
90 Chapter 1 Trigonometric Ratios and Transformations

12. If a is a root of the equation 25 cos2q + 5cos q - 12 = 0 21. If tan a and tan b are roots of the equation x2 + ax +
and p / 2 < a < p
p, then sin 2a = b = 0 (a ¹ 0), then tan(a + b)
b =
8 24 -24 -8 a a
(A) (B) (C) (D) (A) (B)
13 25 25 13 b-1 b+1
b b
13. sin2 A + sin2 ( A - B) - 2 sin A cos B sin( A - B) = (C) (D)
a-1 a+1
(A) cos2 B (B) sin2 B (C) cos2 A (D) sin2 A
22. If n is a positive integer and
14. If tan q = n sin a cos a /(1 - n cos2 a ), then tan(q + a ) =
p p n
1 1 sin + cos =
(A) tana (B) cota 2n 2n 2
1- n 1- n
(C) (1 + n)tan a (D) (1 + n)cot a then
(A) 1 £ n £ 8 (B) 1 £ n £ 4 (C) n = 6 (D) n > 8
15. Let z = cos q + isin q. Then, at q = 2°, the value of
å 23. If cos(q - a ) = a and sin(q - b ) = b, then the value
15
n=1
Re(z2 n - 1 ) is
of cos2 (a - b ) + 2ab sin(a - b ) is
3 3 (A) 4a2b2 (B) a2 + b2
(A) cosec 2° (B) sec 2°
2 4
(C) a2 - b2 (D) a(a + b)
3 3
(C) sec 2° (D) cosec 2° 24. If tan a = 3, then
2 4
2 sin 2a - 3 cos 2a
16. If a and b are solutions of the equation a tan q + =
4 sin 2a + 5 cos 2a
b sec q = c, then tan(a + b) =
9 -9 13 -13
2ca 2ab 2bc a2 + c2 (A) (B) (C) (D)
(A) (B) (C) (D) 4 4 4 4
a2 - c2 a2 - b2 b2 - c2 a2 - c2
1 + sin 2a 1 - tan2 a / 2
17. In a D ABC, if p = sin Asin Bsin C and q = cos Acos B 25. - =
sin a + cos a 1 + tan2 a / 2
cos C, then å tan B tanC =
1+ p 1+ p 1+ q 1+ q (A) -cos a (B) cos a (C) -sin a (D) sin a
(A) (B) (C) (D)
p q q p
26. sin2 70° sin2 50° sin210° =
( > B) such that A + B and
18. If A and B are acute angles (A
(A)
1
(B)
1
(C)
1
(D)
1
A - B satisfy the equation tan2q - 4tan2q + 1 = 0, then 64 32 128 256
p p p p
(A) A = , B = (B) A = , B = 27. If 0 < a, b < p
p/2, sin(a + b)
b = 1 and sin(a
a − b) = 1/2,
3 4 3 6
then tan(a + 2b)
b =
p p p p
(C) A = , B = (D) A = , B = 1 -1
4 6 3 12 (A) 3 (B) - 3 (C) (D)
3 3
19. If tana tan b = a and a + b = p / 6, then tan a and
tan b are roots of the equation 28. If tan a/2
a = m, then
(A) x2 + 3 (1 + a) x - a = 0 1 - 2 sin2 a / 2
=
(B) x - 3 (1 - a) x + a = 0
2
1 + sin a
(C) 3 x2 - (1 - a) x + a 3 = 0 1+ m 1- m m -m
(A) (B) (C) (D)
(D) 3 x2 + (1 + a) x - a 3 = 0 1- m 1+ m 1+ m 1+ m

20. If tana = m/(m - 1) and tanb = 1/(2m - 1), then a - b is 29. If tan a + cot a = a, then tan4 a + cot4 a =
p p p 5p (A) (a2 − 2)2 − 2 (B) (a2 + 2)2 − 2
(A) (B) (C) (D)
4 6 3 12 (C) (a2 − 2)2 + 2 (D) (a2 − 2)2 + 4
Exercises 91

30. If 0 < a < p such that sin a + cos a = 1/5, then tan a/2
a = 39. The value of tan q · tan(q +60°) + tan q tan(q − 60°) +
-1 1 q − 60°) tan(q + 60°) + 3 is
tan(q
(A) (B) (C) 2 (D) -2
3 3 1
(A) 0 (B) 1 (C) 6 (D)
3
31. If cos(a + b)
b = 0, then sin(a + 2b)
b =
(A) sin a (B) cos a (C) sin b (D) cos b 40. The value of sin310° + sin350° − sin370° is
-3 -3 3 2
(A) (B) (C) (D)
sin b n 8 4 5 5
32. If = and | n | < | m |, then
sin (2a + b ) m
1 + (tan b / tan a ) 1 - tan a tan b 41. If
(A) =
m+n m-n A+ B A-B C+D C-D
sin cos + sin cos =2
1 - (tan b / tan a ) 1 - tan a tan b 2 2 2 2
=
m+n m-n then åcos(A/ 2) cos(B/ 2) =
1 + tan a tan b 1 - (tan a / tan b ) (A) 0 (B) 3 (C) 2 (D) 4
=
m+n m-n
1 - tan a tan b 1 + (tan b / tan a ) 42. If 450° < q < 540° and sin q = 336/625, then sin(q /4) =
=
m+n m-n 3 4 7 1
(A) (B) (C) (D)
33. If a, b, g are in AP, then 5 5 8 5 2
sin a - sin g
= 43. If A + B + C = p
p, then
cos g - cos a
(A) cot b (B) tan b sin2 A + sin2 B + sin2 C
=
cos A cos B cos C
(C) tan a cot b (D) tan g cot b
(A) 2 (B) 4 (C) -2 (D) -4
sin 3a × cos a + cos 3a × sin a
3 3
34. = 44. If A + B + C = p
p, then
3
1 1 A B C
(A) sin 4a (B) sin 4a sin + sin + sin
2 4 2 2 2
1 æ p - Aö æ p - Bö æp - Cö
(C) cos 4a (D) 4 cos 4a = 1 + k sinç sinç sinç
4 è 4 ø ÷ è 4 ø ÷ è 4 ÷ø

sin 2a - sin 3a + sin 4a where k is equal to


35. =
cos2a - cos 3a + cos 4a (A) 2 (B)
1
(C) 4 (D)
1
2 4
a
(A) tan 2a (B) cot 2a
(C) tan 3a (D) cot 3a 45. If A + B + C = 2s, then sin(s − A) sin(s − B) +
sin s · sin(s − C) =
36. If cot x, cot y, cot z are in AP, then cot(y − x), cot y,
(A) sin A cos B (B) sin B cos A
cot(y − z) are in
(C) cos A cos B (D) sin A sin B
(A) AP (B) GP (C) HP (D) AGP
46. In DABC
D , if cos 3A + cos 3B + cos 3C = 1, then one
37. If sin q + cos q = 7/ 2 and 0 < q < p
p/6, then tan(q/2)
q =
angle of the triangle must be
1 1 p 2p 4p
(A) (3 - 7 ) (B) ( 7 - 2) (A) (B) (C) p (D)
3 3 3 3 3
(C) 7-2 (D) 7+2
47. If cos q = (a cos a + b)/(a + bcos a
a), then tan2(q /2) =
38. If q = p/(2
p + 1), the cos q cos 2q
q cos 2 q  cos 2 q =
n 2 n
a-b a a+b a
(A) tan2 (B) cos2
1 1 a+b 2 a-b 2
(A) 2n (B) 2n+1 (C) (D)
2n+ 1 2n a-b a a-b a
(C) sin2 (D) cot2
a+b 2 a+b 2
92 Chapter 1 Trigonometric Ratios and Transformations

48. Minimum value of 4 x2 - 4 x |sin q | - cos2q is (A) 0 (B) tan q


-1
(A) -2 (B) -1 (C) 0 (D) (C) - tan q +
1
tan(nq ) (D) tan q +
1
tan(nq )
2 n n
q and 27q are not odd multiples of p
49. If 3q, 9q p/2, then
53. If sin(y + z − x), sin(z + x − y), sin(x + y − z) are in
æ sin q sin 3q sin 9q ö
2ç + + = AP, then tan x, tan y, tan z are in
è cos 3q cos 9q cos 27q ÷ø (A) AP (B) GP (C) HP (D) AGP

q − tan q
(A) tan 27q (B) tan q − tan 27q q − a),
54. If sec(q a sec q, sec(q + a
a) are in AP, then the
q − cotq
(C) cot 27q q − cot 27q
(D) cotq ratio cos q : cos(a/2)
a can be
(A) 2 : 1 (B) 1 : 2 (C) 1 : 2 (D) 2 :1
50. If

cos x cos( x + q ) cos( x + 2q ) cos( x + 3q ) 55. If


= = =
a b c d 2 sin A sin C
tan B =
then (a + c)/(b + d) is equal to sin( A + C )
c b d a then cot A, cot B, cot C are in
(A) (B) (C) (D)
d c a d
(A) AP (B) GP (C) HP (D) AGP
51. If sin a = −3/5 and p < a < 3p
p/2, then cos(a/2)
a =
56. If
1 -1 -1 1
(A) (B) (C) (D) (1 + m)tan a (1 - m)tan a
5 10 5 10 tan x = and tan y =
1 - m tan2 a 1 + m tan2a
52. If an = sin(nq)
q · secnq and bn = cos(nq)
q secnq ¹ 1, then
bn - bn - 1 a æ x - yö æ x + yö
+ n = tan ç cot ç =
an - 1 nbn è 2 ÷ø è 2 ÷ø
(A) 2 m (B) 1 + m2 (C) m2 − 1 (D) m

Multiple Correct Choice Type Questions


1. Let 3. If sin q = (m2 − n2)/(m2 + n2), then
n n m2 - n2 m2 + n2
æ cos A + cos B ö æ sin A + sin B ö (A) tanq = (B) cosecq =
x=ç + 2 mn m2 - n2
è sin A - sin B ÷ø çè cos A - cos B ÷ø
m2 + n2 2 mn
(C) sec q = (D) cotq =
Then 2 mn m2 + n2
(A) x = 0 if n is an odd positive integer
(B) x = tann(A − B)/ 2 if n is an even positive integer 4. Which of the following are true?

(C) x = 2 cotn(A − B)/ 2 if n is an even positive integer (A) tan 20° + tan 72° + tan 88° = tan 20° · tan 72° · tan 88°
(D) x = 0 if n is an even positive integer (B) tan 51° + tan 62° + tan 67° = tan 51° · tan 62° · tan 67°
(C) cot 27° + cot 32° + cot 31° = cot 27° · cos 32° · cot 31°
2. Which of the following statements are true? (D) 3 + tan 40° + tan 80° = 3 tan 40° tan 80°
(A) If cos x + sin x = 2 cos x, then cos x - sin x =
5. Which of the following are true?
2 sin x
(B) If 3 sin q + 5 cos q = 5, then 5 sin q − 3 cos q = ±3 3
(A) cos 10°× cos 30°× cos 50°× cos 70° =
16
3
(C) If A = sin2q + cos4q, then £ A£1 1
4 (B) cos 20°× cos 40°× cos 60°× cos 80° =
(D) If (1 + sin A) (1 + sin B) (1 + sin C) = (1 − sin A) 16
(1 − sin B) (1 − sin C) then each is equal to ±1 (C) tan 20°× tan 40°× tan 80° = 3
(D) cos 55° + cos 65° + cos 175° = 0
Exercises 93

6. Which of the following are true? 12. If 1 + cos(x - y) = 0, then


(A) If tan(a + q)
q = n tan(a - q
q), then sin 2q/sin
q 2a = (A) cos x - cos y = 0 (B) cos x + cos y = 0
(n - 1)/(n + 1) (C) sin x + sin y = 0 (D) cos x + sin y = 1
(B) If sin q = nsin(q + 2a),
a then (1 - n)tan(q + a) a =
(1 + n)tan a 13. If y = sin x × sin(60° - x) sin(60° + x), then
(C) If A + B + C = p and sin [A [ + (C/2)]
C = nsin(C/2),
C (A) Minimum value of y is -1/4
then tan( A / 2) × tan( B / 2) = (n - 1)/(n + 1) (B) Maximum value of y is 1
(D) If A + B + C = p
p, then tan(A/2) + tan(B/2) + (C) Minimum value of y is - 1
C =1
tan(C/2)
(D) y < 1
7. Which of the following are (is) irrational?
14. If (cos2 x + sec2 x) (1 + tan2 2 y) (3 + sin 3z) = 4, then
(A) sin 15° (B) cos15°
(A) x can be a multiple of p
(C) sin 15°× cos 15° (D) sin 15°× cos 75°
(B) x ¹ 2np,
p n Î
8. If p = sin(a - b)sin(
b g - d ), q = sin(b - g)
g sin(a - d ) p
and r = sin(g - a)
a sin(b - d ), then (C) z = (4n - 1) , n Î
6
(A) p + q + r = 0 (B) p + q - r = 0 np
(D) y =
(C) q + r - p = 0 (D) p3 + q3 + r3 - 3pqr
3 =0 2

9. If 0 £ q £ p / 2, x = a cos q + b sin q , y = a sin q - b cosq 15. If A + B + C = p


p, which of the following are true?
and x + 4 xy + y = aa + bb , then
2 2 2 2
(A) sin 2 A + sin 2 B + sin 2C = 4 sin A sin B sin C
p (B) sin 2 A + sin 2 B - sin 2C = 4 cos A cos B sin C
(A) a = - 1, b = 3 (B) q =
4 (C) cos A + cos B + cos C = 1 + 4 sin(A/2)sin(B/2)
p C
sin(C/2)
(C) a = 3, b = - 1 (D) q =
3 (D) cot B cot C + cot C cot A + cot A cot B = 1

å
n
10. If cos3 x × sin 2 x = a sin(mx) is an identity in x, 16. In a triangle ABC, which of the following are true?
m=1 m
then (A) tan A + tan B + tan C = tan A tan B tan C
A B C
(A) a3 = 3 / 8, a2 = 0 (B) n = 6, a1 = 1/ 2 (B) tan2 + tan2 + tan2 ³ 1
2 2 2

å
n
(C) n = 5, a1 = 1/ 4 (D) a = 3/ 4 sin 2 A + sin 2 B + sin 2C A B C
m=1 m (C) = 8 sin sin sin
sin A + sin B + sin C 2 2 2
11. If 7 cos x - 24 sin x = lcos(
l x+q
q) and 0 < q < p/
p 2, then A B C A B C
(D) cot + cot + cot = cot cot cot
(A) l = 25 (B) cos q = 7/25 2 2 2 2 2 2
(C) sin q = 24/25 (D) l = 15

Matrix-Match Type Questions


In each of the following questions, statements are given matches are (A) ® (p) and (s); (B) ® (q),(s) and (t);
in two columns, which have to be matched. The state- (C) ® (r); and (D) ® (r),(t) then the correct darkening
ments in Column I are labeled as (A), (B), (C) and of bubbles will look as follows:
(D), while those in Column III are labeled as (p), (q),
(r), (s) and (t). Any given statement in Column I can p q r s t
have correct matching with one or more statements in A
Column II. The appropriate bubbles corresponding to
B
the answers to these questions have to be darkened as
illustrated in the following example. C
D
Example: If the correct matches are (A) ® (p),(s);
(B) ® (q),(s),(t); (C) ® (r); (D) ®(r),(t); that is if the
94 Chapter 1 Trigonometric Ratios and Transformations

1. Match the items in Column I with those in Column II. 4. Let q = 22(1/2)°. Match the items in Column I with
those in Column II.
Column I Column II
Column I Column II
æ Aö
(A) tan ç 45° + ÷ (p) sec A + tan A
è 2ø (A) sin q (p) 2 +1
1
(q) sin A
2 (q) 2 -1
æ p Aö æ p Aö
(B) sin2 ç + ÷ - sin2 ç - ÷ (B) cos q
è8 2ø è8 2ø æ A - Bö 1
(r) 4 cos2 ç (r) 2+ 2
è 2 ÷ø 2

(C) (cosA + cosB)2 + (sinA + sinB)2 1 + sin A (C) tan q (s)


1
2- 2
(s) 2
1 - sin A
1
æ A - Bö (D) cotq (t) ( 2 + 1)
(D) (cosA - cosB) + (sinA - sinB) (t) 4 sin ç
2 2 2
2
è 2 ÷ø
5. Match the items in Column I with those in Column II.
2.

Column I Column II
Column I Column II
(A) If tan(x - y)/2, tan z, tan(x + y)/2 (p) cos 2q
3 are in GP, then cos x is equal to
(A) cos q + cos (q + 120°) + cos (q - 120°) (p)
2 2 2

2 3 sin q
(q)
(B) If sin x, sin q, cos x are in GP, 2 + cos q
(q) 2
p 3p 5p 7p then 2 cos2[(p /4) + x] is
(B) cos4 + cos4 + cos4 + cos4 x
8 8 8 8 1 equal to (r) 2 cos
(r) 2
16
(C) If sec(q - x), sec q, sec(q + x) are 2
p 3p 5p 7p
(C) sin4 + sin4 + sin4 + sin4 -1 in AP, then cos q is equal to (s) sin x
8 8 8 8 (s) 2
4
(D) If cos q = (2 cos x - 1)/(2 - cos x), (t) cos ycos 2z
p 13p 1 then sin x is equal to
(D) sin × sin (t)
10 10 4
6. Match the items in Column I with those in Column II.
3. Assuming that A + B + C = p
p, match the items in
Column I with those in Column II.
Column I Column II
tan( A - B) + tan B
Column I Column II (A) (p) tan A
1 - tan( A - B)tan B
A B C
(A) cos2A + cos2B + cos2C (p) 4 cos cos cos
2 2 2 tan 4 A - tan 3 A
(B) (q) cot A
1 - tan 3 A × tan 4 A
A B C
(B) cos A + cos B + cos C - 1 (q) 4 sin cos sin (r) tan 2A
2
2 2 2 2 (1 + cot 2A
(C) sin 2A 2 × cot A)
(C) sin A - sin B + sin C (r) 1 - 2cosAcosBcosC (D) cos 2A 2 + cos A × cosec A)
2 (tan 2A (s) cot 2A
2
A B C
(D) sin A + sin B + sin C (s) 4 sin sin sin
2 2 2
Exercises 95

Comprehension-Type Questions
1. Passage: The equation a cos q + b sin q = c can be 3. Passage: To eliminate a parameter, we need two
solved when | c | £ a + b . Based on this answer the
2 2 equations involving the parameter. For example, if
following questions (i), (ii) and (iii). x = rcos q and y = rsin q, then by squaring and adding,
we have x2 + y2 = r2. This shows that q is eliminated
(i) If 0 £ x £ p and sin x + cos x = 1, then the number from the given equations.
of values of x is
Answer the following questions (i), (ii) and (iii).
(A) 1 (B) 2 (C) 4 (D) 0
(i) If tan q + sin q = a and tan q - sin q = b, then after
(ii) The maximum and minimum values of eliminating q, (a2 - b2)2 is equal to
3 cos q + 4 sin q - 5 are respectively (q
q is real)
(A) 4 ab (B) 4ab (C) 16 ab (D) 16ab
(A) 5, - 5 (B) 4, 3 (C) 0, - 10 (D) 4, - 3
(ii) Eliminating q from the equations x = cot q + tan q
(iii) The maximum and minimum values of
and y = sec q - cos q, we have
5 sin q + 12 cos q + 13 are respectively (q
q is real)
(A) xy(x + y) = 1 (B) (x2y)2/3 - (xy2)2/3 = 1
(A) 13, 0 (B) 1, - 1 (C) 26, 0 (D) 26, 13
(C) x2/3 - y2/3 = 1 (D) xy = 1
2. Passage: If a, b and g are roots of the equation (iii) Eliminating q from the equations x/a = cos q +
ax3 + bx2 + cx + d = 0 and a ¹ 0, then a + b + g = -b/a, q and y/b = sin q + sin 2q, which of the following
cos 2q
ab + bg + ga = c/a and abgg = -d/a. It is given that relations between x and y can be obtained?
cos p
p/7, cos 3p/7
p and cos 5p/7
p are roots of 8x3 - 4x2 -
4x + 1 = 0. 2 x æ x2 y2 ö æ x2 y2 ö
(A) = + + - 3÷
Answer questions (i), (ii) and (iii). a çè a2 b2 ÷ø çè a2 b2 ø
(i) The value of sec(p / 7) + sec(3p / 7) + sec(5p / 7) is x2 y2 2 x
(B) + = +3
(A) 4 (B) 2 (C) 8 (D) 6 a2 b2 a
(ii) The value of sin(p / 14) sin(3p / 14) sin(5p / 14) is x2 y2 x
(C) + = +3
7 1 7 1 a2 b2 a
(A) (B) (C) (D)
8 8 4 4 æ x2 y2 ö æ x2 y2 ö æ x yö
(iii) The value of cos(p / 14) cos(3p / 14) cos(5p / 14) is çè a2 + b2 ÷ø çè a2 + b2 - 3÷ø = 2 çè a + b ÷ø
1 1 7 7
(A) (B) (C) (D)
8 4 4 8

Assertion–Reasoning Type Questions


In the following set of questions, a Statement I is given 3. Statement I: For all values of q, 2(sin6q + cos6q)
q -
and a corresponding Statement II is given just below it. 3(sin4q + cos4q)
q = -1
Mark the correct answer as:
Statement II: x3 + y3 = ( x + y) ( x2 - xy + y2 )
(A) Both Statements I and II are true and Statement II
is a correct explanation for Statement I q = m, then
4. Statement I: If tan(q/2)
(B) Both Statements I and II are true but Statement II is
1 - 2 sin2 (q / 2) 1 + m
not a correct explanation for Statement I =
1 + sin q 1- m
(C) Statement I is true and Statement II is false
(D) Statement I is false and Statement II is true 2 tan q 1 - tan2q
Statement II: sin 2q = and cos 2q =
1 + tan q
2
1 + tan2q
1. Statement I: If x cos a + y sin a = x cos b + y sin b = 2a
and 2 sin a / 2 sin b / 2 = 1, then y2 = 4a(a - x) 5. Statement I: If
Statement II: If x1 and x2 are roots of the quadratic sin 3a 11
equation ax2 + bx + c = 0, then x1 + x2 = -b/a and x1x2 = c/a =
sin a 25
2. Statement I: If x = cot q + tan q and y = cosec q - sin q, then tan(a /2) has four values.
then (x2y)2/3 - (xy2)2/3 = 1
2 tan(q / 2)
Statement II: The conclusion of Statement I is that Statement II: sin q =
1 + tan2 (q / 2)
cosec2q - cot2q = 1
96 Chapter 1 Trigonometric Ratios and Transformations

6. Statement I: If 0 < a < p


p/2 and cot a = 3, then the 9. Statement I: If a + b + g = p
p/2, then
a is 1/(3 + 10).
value of tan(a/2)
sin 2a + sin 2 b + sin 2g
a sin a = cot a cot b
Statement II: tan = sin 2a + sin 2 b - sin 2g
2 1 + cos a
Statement II: If A + B + C=p,p then cos 2A
2 + cos 2B +
cos 2C = -1 - 4 cos Acos Bcos C.
7. Statement I: If

3 3 10. Statement I: If A + B + C = p
p, then
tan A = and tan B =
4- 3 4+ 3 A B C
cos + cos + cos
then tan(A - B) = 0.355. 2 2 2
æ p - Aö æ p - Bö æp - Cö
tan A - tan B = 4 cos ç cos ç cos ç
Statement II: tan( A - B) = è 4 ÷ø è 4 ÷ø è 4 ÷ø
1 + tan A tan B
8. Statement I: If a + b + g = p
p/2, then åtan b tan g = 1. Statement II: cos(a + b)cos(
b a - b) = cos2 a - cos2 b
for all a and b.
Statement II: If A + B + C = p
p, then tan A + tan B +
tan C = tan Atan Btan C.

Integer Answer Type Questions


The answer to each of the questions in this section is a 6. If 2 tan(A + B) = 3 tan A, then sin(2A
2 + B) = ksin B,
non-negative integer. The appropriate bubbles below where k is equal to .
the respective question numbers have to be darkened.
For example, as shown in the figure, if the correct answer 7. If 3 cos x = 2 cos(x - 2y
2 ), then cot(x - y)cot y is equal
to the question number Y is 246, then the bubbles under to .
Y labeled as 2, 4, 6 are to be darkened.
8. If sin 2° + sin 4° + sin 6° +  + sin 178° = n cot 1°, then
X Y Z W n is equal to .
0 0 0 0
9. Let 0 < a j < p
p/2 for j = 1, 2, 3, , 8 and cot a 1 × cot a 2 
1 1 1 1
cot a 8 = 1. If M is the maximum value of cot a 1 cot a 2
2 2 2  cot a 8, then the value of (32) M is .
3 3 3 3
4 4 4 10. The value of (1 + cot A - cosec A)(1 + tan A + sec A)
is .
5 5 5 5
6 6 6 11. If sin x + sin y + sin z = 3, then | cos 2x + cos 2y
2 + cos 2z |
7 7 7 7 is equal to .
8 8 8 8
9 9 9 9
12. (sin a + cosec a)
a 2 + (cos a + sec a)
a 2 - (tan2a + cot2a)
a
is equal to .

13. If tan q = 1 - e , then sec q + tan q cosec


o q = (k - e ) ,
2 2 3 2 3/2
1. tan 12°tan 24°tan 48°tan 84° is equal to .
where k is .
2. If sin 18° and cos 36° are roots of the equation ax2 -
2 5x + c = 0, then the value of a + c is . 14. If qj ( j = 1, 2, 3, 4) satisfy the equation 3 tan 3q =
tan(45° + qq), then the value of tan q1 + tan q2 +
3. tan 6°tan 42°tan 66°tan 78° is equal to . tan q3 + tan q4 is .

sin 3a cos 3a 15. For n > 2, the value of


4. + = k cos 2a , where k is equal to .
sin a cos a p 3p 5p p
sin + sin + sin +  + sin(2 n - 1)
n n n n
5. (1 - 2sinq)(1
q + 2sinq)(1
q + 2cos2q)
q = 1 + kcos4q,
q where
k is equal to . is .
Answers 97

ANSWERS
Single Correct Choice Type Questions
1. (D) 29. (A)
2. (C) 30. (C)
3. (A) 31. (A)
4. (B) 32. (A)
5. (D) 33. (A)
6. (B) 34. (B)
7. (A) 35. (C)
8. (C) 36. (A)
9. (D) 37. (B)
10. (C) 38. (D)
11. (A) 39. (A)
12. (C) 40. (A)
13. (B) 41. (B)
14. (A) 42. (B)
15. (D) 43. (A)
16. (A) 44. (C)
17. (C) 45. (D)
18. (C) 46. (B)
19. (C) 47. (A)
20. (A) 48. (B)
21. (A) 49. (A)
22. (A) 50. (B)
23. (B) 51. (B)
24. (B) 52. (C)
25. (D) 53. (A)
26. (A) 54. (D)
27. (B) 55. (A)
28. (B) 56. (D)

Multiple Correct Choice Type Questions


1. (A), (C) 9. (B), (C)
2. (A), (B), (C) 10. (A), (C), (D)
3. (A), (B), (C) 11. (A), (B), (C)
4. (A), (B), (C), (D) 12. (B), (C)
5. (A), (B), (C), (D) 13. (A), (D)
6. (A), (B), (C) 14. (A), (C), (D)
7. (A), (B), (D) 15. (A), (B), (C), (D)
8. (A), (D) 16. (A), (B), (C), (D)

Matrix-Match Type Questions


1. (A) ® (p), (s), (B) ® (q), (C) ® (r), (D) ® (t) 4. (A) ® (s), (B) ® (r), (C) ® (q), (D) ® (p)
2. (A) ® (p), (B) ® (p), (C) ® (p), (D) ® (s) 5. (A) ® (t), (B) ® (p), (C) ® (r), (D) ® (q)
3. (A) ® (r), (B) ® (s), (C) ® (q), (D) ® (p) 6. (A) ® (p), (B) ® (p), (C) ® (q), (D) ® (q)

Comprehension-Type Questions
1. (i) (B); (ii) (C); (iii) (C) 3. (i) (D); (ii) (B); (iii) (A)
2. (i) (A); (ii) (B); (iii) (D)
98 Chapter 1 Trigonometric Ratios and Transformations

Assertion–Reasoning Type Questions


1. (A) 6. (A)
2. (A) 7. (D)
3. (A) 8. (B)
4. (D) 9. (B)
5. (A) 10. (C)

Integer Answer Type Questions


1. 1 9. 2
2. 5 10. 2
3. 1 11. 3
4. 4 12. 7
5. 2 13. 2
6. 5 14. 0
7. 5 15. 0
8. 1
Inverse Trigonometric
Functions 2
Contents
2.1 Domains and
Ranges of Inverse
Inverse Trigonometric Functions

Trigonometric
Functions
Y 2.2 Properties of Inverse
Trigonometric
Functions
p
Worked-Out Problems
W
Cos 1 x Summary
3p/4 Exercises
Answers

p/2

The inverse trigonometric


p/4 functions (or cyclometric
functions) are the inverse
functions of the trigono-
X metric functions, though they
do not meet the official defi-
1 0.5 0.5 1 nition for inverse functions
as their ranges are subsets of
p/4 the domains of the original
functions. Since none of the
Sin 1 x six trigonometric functions
are one-to-one (by failing
p/2
the horizontal line test), they
must be restricted in order
to have inverse functions.
100 Chapter 2 Inverse Trigonometric Functions

Let us recall that a function f is a bijection (injection and surjection) if and only if there is a function g such that f g
and g f are both identity functions. In this case, g is unique and is denoted by f -1. The trigonometric functions are
not bijections, since each of them is of period 2p p and hence not injective. For example, sin 30° = sin(2p + 30°) and
therefore the sine function is not an injection. Neither, it is surjective, since we cannot find a real number x such that
sin x = 2. However, by restricting the natural domain and codomain of the sine function, we can arrive at a bijection.
In this chapter, we discuss various properties of the inverses of certain trigonometric functions by first deter-
mining the appropriate domains and codomains of these functions.

2.1 | Domains and Ranges of Inverse Trigonometric Functions


p and later extended it to all real numbers x by defining
We have first defined the sine function on the interval [0, 2p)
sin x = sin q
where x = 2np + q, 0 £ q < 2p (here n is the integral part x/2p).
p However, the sine function is not an injection on [0, 2p),p
since sin 0 = sin p and sin p/4
p = sin 3p/4.
p Instead of interval [0, 2p p), if we consider the interval [-p/2,
p p p/2], then the sine
function is one-to-one on this interval. Also, the range of sine function is the interval [-1, 1]. Therefore, the function

é -p p ù
f : ê , ú ® [-1, 1]
ë 2 2û
f x) = sin x for all x Î[-p/2,
defined by f( p p p/2], is a bijection and hence there exists an unique function

é -p p ù
g : [-1, 1] ® ê , ú
ë 2 2û
such that f g is the identity on [-1, 1] and g f is the identity on [-p/2,
p p p/2]; that is,
sin g(y) = y
for all -1 £ y £ 1 and
-p p
g(sin x) = x for all £x£
2 2

DEFINITION 2.1 The unique function g : [-1, 1] ® [-p/2,


p p p/2] such that
g(sin x) = x and sin(g(y)) = y
for all -p/2
p £x£p p/2 and -1 £ y £ 1 is called the inverse sine function and is denoted by Sin-1.
-1
Therefore, Sin is a function whose domain is [-1, 1] and codomain is [-p/2, p p p/2] and it satis-
fies the relations
Sin-1 (sin x) = x and sin (Sin-1 y) = y
for all -p/2
p £x£p
p/2 and -1 £ y £ 1.

p p
Note that, in place of [-p/2, p/2] if we take [p
p/2, 3p
p/2] (or [(2n - 1)p/2,
p (2n + 1)p/2]
p for any integer n), then the function

é (2 n - 1)p (2 n + 1)p ù
sin: ê
ë 2
,
2 úû ® [-1, 1]

is a bijection and hence has inverse whose domain is [-1, 1] and range is [(2n - 1)p/2,
p (2n + 1)p/2].
p This can also be
called the inverse of the sine function (restricted to [(2n - 1)p/2,
p (2n + 1)p/2]).
p However, if we consider the sine func-
p p
tion restricted to [-p/2, p/2] then its inverse

-p p ù
Sin-1 : [-1, 1] ® êé , ú
ë 2 2û
will be usually considered the inverse of the sine function. Also, for any -1 £ y £ 1, Sin-1 y in [-p/2,
p p p/2] is called the
principal value of Sin-1 y and is often denoted by arcsin y. The principal value of Sin-1 y will be denoted by Sin-1 y (with
a capital letter S).
2.1 Domains and Ranges of Inverse Trigonometric Functions 101

QUICK LOOK 1

-p p 5. Sin-1 y = 0 Û y = 0
1. £ Sin-1 y £ for all - 1 £ y £ 1
2 2
é -p ö
-p p 6. Sin-1 y Î ê , 0÷ Û y Î[-1, 0)
2. -1 £ sin x £ 1 for all £x£ ë 2 ø
2 2
3. sin (Sin-1 y) = y for all -1 £ y £ 1 æ -p ù
7. Sin-1 y Î ç 0, Û y Î[0, 1)
-p p
è 2 úû
4. Sin-1 (sin x) = x for all £x£
2 2 8. Sin-1 y = x Û sin x = y

The discussion made in Quick Look 1 can be extended to other trigonometric functions also. The restrictions of
the functions cosine, tangent, cosecant, secant and cotangent restricted to [0, p
p], (-p/2,
p p p/2), [-p/2,
p p/2]
p - {0}, [0, p]
p -
p and (0, p
{p/2} p), respectively, are injections and hence can be considered as bijections onto their respective ranges.
Therefore, these functions have inverses which are denoted by Cos-1, Tan-1, Cosec-1, Sec-1 and Cot-1, respectively.
The inverse trigonometric functions are also called inverse circular functions. The domains and ranges of the inverse
trigonometric functions are given in Table 2.1.

Table 2.1 Domains and ranges of the inverse trigonometric functions


Function Domain Range

é -p p ù
Sin-1 [-1, 1] êë 2 , 2 úû

Cos-1 [-1, 1] [0, p]


p
æ -p p ö
Tan-1  çè , ÷
2 2ø
Cot-1  (0, p)
p
é -p ö æ p ù
Cosec-1 (-¥, -1] È [1, ¥) êë 2 , 0÷ø È çè 0, 2 úû

é -p ö æ p ù
Sec-1 (-¥, -1] È [1, ¥) êë0, 2 ÷ø È çè 2 , p úû

The graphs of inverse trigonometric functions are illustrated in Figure 2.1 by taking the domains on the X-axis
and the ranges on the Y-axis.

Y y = Sin 1x Y y = Cos 1x

p p

p/2 p/2

X X
1 0 1 1 0 1

p/2 p/2

p p

FIGURE 2.1 Graphs of inverse trigonometric functions.


102 Chapter 2 Inverse Trigonometric Functions

y = Tan 1x Y Y y = Cot 1x

p/2 p

X
0 p/2

p/2 X
0

y = Sec 1 x Y Y y = Cosec 1 x

p p/2

p/2
X
1 1 0 1

X
0 1 p/2

p/2
p

FIGURE 2.1 Continued.

2.2 | Properties of Inverse Trigonometric Functions


In this section we collect certain important properties of inverse trigonometric functions.

T H E O R E M 2.1 For any -1 £ y £ 1,


1. Sin-1 (- y) = - Sin-1 y
2. Cos-1 (- y) = p - Cos-1 y
PROOF Let -1 £ y £ 1. Then y and -y
- Î[-1, 1].
1. Put Sin-1(-y
- ) = x. Then

é -p p ù
- y = sin x and x Îê , ú
ë 2 2û
2.2 Properties of Inverse Trigonometric Functions 103

Therefore

é -p p ù
y = - sin x = sin(- x) and - x Î ê , ú
ë 2 2û

Sin-1 y = Sin-1 (sin(- x)) = - x = - Sin-1 (- y)


Therefore
Sin-1 (- y) = - Sin-1 y
2. Put Cos-1(-y
- ) = z. Then -y
- = cos z and z Î[0, p
p]. Therefore y = -cos z = cos(p - z) and p - z Î
[0, p
p]. Hence
Cos-1 y = p - z = p - Cos-1 (- y)
or Cos-1 (- y) = p - Cos-1 y ■

The following can be similarly proved.

T H E O R E M 2.2 For any real number y,


Tan-1(-y
- ) = -Tan-1 y
and Cot-1(-y
- ) = p - Cot-1 y

T H E O R E M 2.3 For any y Î - (-1, 1),


Sec-1 (-y
- ) = p - Sec-1 y
and Cosec-1(-y
- ) = -Cosec-1 y

Examples

æ 3ö æ 3ö æ pö
(1) Sin-1 ç - = - Sin-1 ç -1 (4) Cot-1 (-1) = p - Cot-1 (1) = p - Cot-1 ç cot ÷
÷ ÷ = - Sin (sin 60°) = - 60° è 4ø
è 2 ø è 2 ø
p 3p
æ 1 ö
-1 -1 æ 1 ö -1 æ pö =p - =
(2) Cos ç - ÷ø = p - Cos çè ÷ø = p - Cos çè Cos 4 ÷ø 4 4
è 2 2
p 3p
p 3p (5) Sec-1 (- 2 ) = p - Sec-1 ( 2 ) = p - =
=p - = 4 4
4 4
(6) Cosec-1 (- 2 ) = - Cosec-1 ( 2 )
æ pö p
(3) Tan-1 (- 3 ) = - Tan-1 ( 3 ) = - Tan-1 ç tan ÷ = - pö p
è 3 ø 3 æ
= - Cosec-1 ç cosec ÷ = -
è 6ø 6

Example 2.1

Find the values of the following: æ 3p ö


(3) Tan-1 ç tan ÷
æ æ 5p ö ö è 4ø
(1) Sin-1 ç sin ç ÷ ÷
è è 6 øø Solution:
æ 2p ö æ æ 5p ö ö æ æ p öö
Cos-1 ç cos ÷ (1) Sin-1 ç sin ç ÷ ÷ = Sin-1 ç sin ç p - ÷ ÷
è 3ø è è 6 øø è è 6 øø
104 Chapter 2 Inverse Trigonometric Functions

æ pö æ 2p ö 2p 2p
= Sin-1 ç sin ÷ (2) Cos-1 ç cos ÷ = , since Î[0, p ]
è 6ø è 3ø 3 3
p p é -p p ù æ 3p ö æ æ p öö
= , since Î , (3) Tan-1 ç tan ÷ = Tan-1 ç tan ç p - ÷ ÷
6 6 êë 2 2 úû è 4ø è è 4øø
Here, note that æ pö
= Tan-1 ç -tan ÷
è 4ø
5p é -p p ù
Îê , ú pö
6 ë 2 2û æ
= - Tan-1 ç tan ÷
è 4ø
and therefore Sin-1 (sin (5p / 6)) is not equal to 5p/6.
p
-p -p æ -p p ö
= , since Îç , ÷
4 4 è 2 2ø

T H E O R E M 2.4 For any 0 ¹ y Î[-1, 1]


æ 1ö
1. Sin-1 ç ÷ = Cosec-1 y
è yø

æ 1ö
2. Cos-1 ç ÷ = Sec-1 y
è yø

PROOF Put Sin-1(1/y


/ ) = x. Then sin x = 1/y
/ and hence y = 1/sin x = cosec x.
Therefore

æ 1ö
Sin-1 ç ÷ = x = Cosec -1 (cosec x) = Cosec-1 y
è yø

Similarly (2) and the following theorem can be proved. ■

T H E O R E M 2.5 1. For any y > 0,

æ 1ö
Tan-1 ç ÷ = Cot-1 y
è yø

2. For any y < 0,

æ 1ö
Tan-1 ç ÷ = Cot-1 y - p
è yø

T H E O R E M 2.6 p, Sin-1(cos x) = p/2


1. For any 0 £ x £ p p - x.
2. For any -p/2
p £x£p p/2, Cos-1(sin x) = p/2
p - x.
PROOF p. Then -1 £ cos x £ 1. Put Sin-1(cos x) = z. Then
1. Let 0 £ x £ p

-p p æp ö
£z£ and sin z = cos x = sin ç - x÷
2 2 è2 ø

Therefore, z = (p/2)
p - x (since both z and p/2
p - x belong to [-p/2,
p p p/2]). Hence
Sin-1(cos x) = p/2
p -x
Similarly (2) and the following theorem can be proved. ■
2.2 Properties of Inverse Trigonometric Functions 105

T H E O R E M 2.7 p, Tan-1(cot x) = (p/2)


1. For any 0 < x < p p -x
2. For any -p/2
p <x<p p/2, Cot-1(tan x) = (p/2)
p -x
-1
3. For any 0 ¹ x Î[-p/2,
p p p/2], Sec (cosec x) = (p/2)
p -x
4. For any p/2 p], Cosec-1(sec x) = (p/2)
p ¹ x Î[0, p p -x

T H E O R E M 2.8 For any -1 £ y £ 1,


ìï Cos-1 1 - y2 if 0 £ y £ 1
Sin-1 y = í
-1
ïî-Cos 1 - y if - 1 £ y < 0
2

PROOF Let 0 £ y £ 1. Then 0 £ Sin-1 y £ p


p/2. Put Sin-1 y = x. Then
sin x = y and 0 £ x £ p/2
p
Therefore

cos x = 1 - y2 and x = Cos-1 1 - y2

That is

Sin-1 y = Cos-1 1 - y2

p £ Sin-1 y < 0. Put Sin-1 y = -z. Then 0 < z £ p


Next, let -1 £ y < 0. Then -p/2 p/2 and y = sin(-z) =
-sin z. Therefore

sin z = -y
- and cos z = 1 - sin2 z = 1 - y2
This implies
z = cos 1 - y2
So

Sin-1 y = - z = - cos 1 - y2 ■

The following can be proved on the lines of the above proof.

T H E O R E M 2.9 1. If -1 < y < 1,

æ y ö
Sin-1 y = Tan-1ç 2 ÷
è 1- y ø
2. If 0 < y £ 1,
æ 1 - y2 ö
Cos-1 y = Sin-1 ( ) -1
1 - y2 = Tan ç
çè y ÷ø
÷

3. If -1 £ y < 0,
æ 1 - y2 ö
Cos-1 y = p + Tan-1 ç ÷
è y ø

4. If y > 0,
æ y ö æ 1 ö
Tan-1 y = Sin-1 ç ÷ = Cos-1 ç ÷
è 1+ y ø è 1+ y ø
2 2
106 Chapter 2 Inverse Trigonometric Functions

Example 2.2

Find the value of æ æ öö


æ 5ö 1
æ 3ö (2) cos ç Tan-1 ÷ = cos ç Cos-1 ç ÷÷
(1) sin ç Cos-1 ÷ è 12 ø çè çè 1 + (5 / 12)2 ÷ø ÷ø
è 5ø
æ 5ö æ æ 12 ö ö 12
(2) cos ç Tan-1 ÷ = cos ç Cos-1 ç ÷ ÷ =
è 12 ø è è 13 ø ø 13

Solution:

æ 3ö æ æ 3ö ö
2

(1) sin ç Cos-1 ÷ = sin ç Sin -1


1 - çè ÷ø ÷÷
è 5ø çè 5 ø
[by part (2) of Theorem 2.9]

æ 4ö 4
= sin ç Sin-1 ÷ =
è 5ø 5

T H E O R E M 2.10 For any -1 £ y £ 1, Cos-1 y + Sin-1 y = p/2.


p
PROOF If y = 0, then
p p
Cos-1 y + Sin-1 y = Cos-1 0 + Sin-1 0 = +0=
2 2

Let 0 < y £ 1. Then


p p
0 £ Cos-1 y < and 0 < Sin-1 y £
2 2

Put Sin-1 y = x. Then 0 < x £ p/2


p and sin x = y. Therefore, 0 £ p/2
p - x < p/2
p and cos(p/2
p - x) =
sin x = y and hence
p
Cos-1 y = -x
2

æp ö p
Cos-1 y + Sin-1 y = ç - x÷ + x =
è2 ø 2

Next, let -1 £ y < 0. Then


p p
- £ Sin-1 y < 0 and < Cos-1 y £ p
2 2

Put Sin-1 y = z. Then sin z = y and -p/2


p £ z < 0 and hence p/2
p ³ - z > 0 so that p/2
p < (p/2
p - z) £ p.
p
p - z) = sin z = y. Therefore
Now, cos(p/2
p
Cos-1 y = -z
2

æp ö p
Cos-1 y + Sin-1 y = ç - z÷ + z =
è2 ø 2 ■
2.2 Properties of Inverse Trigonometric Functions 107

T H E O R E M 2.11 For any real number y,


p
Tan-1 y + Cot-1 y =
2
PROOF Let y be a real number and x = Tan-1 y. Then tan x = y and -p/2
p <x<p
p/2. Also,
p - x) = tan x = y
cot(p/2
We shall distinguish two cases.
Case I: Suppose that y ³ 0. Then 0 £ x < p
p/2 and therefore 0 < (p/2
p - x) £ p
p/2. Since cot(p/2
p - x) = y,
we get
p
Cot-1 y = -x
2
Therefore
æp ö p
Tan-1 y + Cot-1 y = x + ç - x÷ =
è2 ø 2

Case II: Suppose that y < 0. In such case -p/2


p < x < 0 and therefore p/2
p < (p/2
p - x) < p
p. Again,
p - x) = y, we get that
since cot(p/2
p
Cot-1 y = -x
2
Therefore
æp ö p
Tan-1 y + Cot-1 y = x + ç - x÷ =
è2 ø 2 ■

C O R O L L A R Y 2.1 For any nonzero real number y,


ìp
if y > 0
æ 1 ö ïï 2
Tan-1 y + Tan-1 ç ÷ = í
è y ø ï -p
if y < 0
ïî 2

PROOF If y > 0, then

æ 1ö
Tan-1 y + Tan-1 ç ÷ = Tan-1 y + Cot-1 y [by part (1) of Theorem 2.5]
è yø
p
= (by Theorem 2.10)
2
If y < 0, then

æ 1ö
Tan-1 y + Tan-1 ç ÷ = Tan-1 y + (Cot-1 y - p ) [by part (2) of Theorem 2.5]
è yø

p
= -p (by Theorem 2.10)
2
-p
=
2 ■
108 Chapter 2 Inverse Trigonometric Functions

T H E O R E M 2.12 Let x and y be positive real numbers. Then


ì -1 æ x + y ö
ï Tan ç if xy < 1
ï è 1 - xy ÷ø
ïï æ x+ yö
Tan-1 x + Tan-1 y = í Tan-1 ç + p if xy > 1
ï è 1 - xy ÷ø
ï p
ï if xy = 1
ïî 2

PROOF If xy = 1, then x = 1/y


/ and y > 0 and hence, by Corollary 2.1,
p
Tan-1 x + Tan-1 y =
2
Suppose that xy ¹ 1(note that both x and y are given to be positive). Put Tan-1 x = q and Tan-1 y = f.
Then tan q = x and tan f = y. Since x and y are positive, both q and f belong to (0, p
p/2) and hence
0<q+f<p
Since xy ¹ 1, tan q × tan f ¹ 1 and hence
p
q +f ¹
2
tan q + tan f x+y
and tan(q + f ) = = (2.1)
1 - tan q tan f 1 - xy
Now, suppose that xy < 1. Then, by Eq. (2.1)
tan(q + f) > 0
Since 0 < q + f < p and tan(q + f) > 0, we have 0 < q + f < p/2.
p Therefore, by Eq. (2.1),
æ x+ yö
Tan-1 x + Tan-1 y = q + f = Tan-1 ç
è 1 - xy ÷ø
Next, suppose that xy > 1. Then, by Eq. (2.1),
tan(q + f) < 0
Since 0 < q + f < p and tan(q + f) < 0, we have
p
<q +f <p
2
and hence
-p
<q +f -p < 0
2
Also,
x+ y
tan(q + f - p ) = - tan(p - (q + f )) = tan(q + f ) =
1 - xy
Therefore

æ x+ yö
Tan-1 ç =q +f -p
è 1 - xy ÷ø
æ x+ yö
Tan-1 x + Tan-1 y = q + f = Tan-1 ç +p
è 1 - xy ÷ø ■

The following can be proved on the lines of the above proof.


2.2 Properties of Inverse Trigonometric Functions 109

T H E O R E M 2.13 1. For any negative real numbers x and y,


ì -1 x + y
ï Tan 1 - xy if xy < 1
ï
ï x+y
Tan-1 x + Tan-1 y = í - p + Tan-1 if xy > 1
ï 1 - xy
ï -p
ï if xy = 1
î 2
2. If xy + yz + zx + < 1 and x, y, z have the same sign (that is, either all the three are positive or
all the three are negative), then

é x + y + z - xyz ù
Tan-1 x + Tan-1 y + Tan-1 z = Tan-1 ê ú
ë 1 - ( xy + yz + zx) û

T H E O R E M 2.14 If x and y are positive real numbers, then


æ x-yö
Tan-1 x - Tan-1 y = Tan-1 ç
è 1 + xy ÷ø

PROOF Let x > 0, y > 0, Tan-1 x = q and Tan-1 y = f. Then tan q = x and tan f = y. Since x and y are positive,
both q and f belong to (0, p p/2) and hence
-p p
<q -f <
2 2
Also,
tan q - tan f x-y
tan(q - f ) = =
1 + tan q tan f 1 + xy

æ x-yö
q - f = Tan-1 ç
è 1 + xy ÷ø
Therefore
æ x-yö
Tan-1 x - Tan-1 y = Tan-1 ç
è 1 + xy ÷ø ■

The following can be proved similarly.

T H E O R E M 2.15 Let x and y be real numbers such that xy ¹ -1. Then

ì -1 æ x - y ö
ï Tan ç if xy > - 1
ï è 1 + xy ÷ø
ï æ x-yö
ï
Tan-1 x - Tan-1 y = í p + Tan-1 ç if xy < - 1, x > 0 and y < 0
ï è 1 + xy ÷ø
ï æ ö
ï-p + Tan-1 x - y if xy < - 1, x < 0 and y > 0
ïî ç
è 1 + xy ÷ø
110 Chapter 2 Inverse Trigonometric Functions

T H E O R E M 2.16 Let x and y be non-negative real numbers such that x2 + y2 £ 1. Then

Sin-1 x + Sin-1 y = Sin-1 ( x 1 - y2 + y 1 - x2 )


-1
PROOF 0 = 0, the theorem is trivial if x = 0 or y = 0. Therefore, we can assume that both x and y
are positive and belong to the internal (0, 1). Put Sin-1 x = q and Sin-1 y = f. Then sin q = x, sin f = y
and q and f Î(0, p p/2). Therefore

cosq = 1 - x2 and cosf = 1 - y2

Also, 0 < q + f < p and we have


cos (q + f ) = cos q cosf - sin q sin f

= 1 - x2 1 - y2 - xy

= 1 - x2 y2 - ( x2 + y2 ) - xy ³ 0 x2 + y2 £ 1)

Therefore
q + f Î(0, p
p) and hence 0 < q + f £ p/2
p
Now,
sin (q + f ) = sin q cosf + cosq sin f

= x 1 - y2 + y 1 - x2

therefore

q + f = Sin-1 ( x 1 - y2 + y 1 - x2 )

Sin-1 x + Sin-1 y = Sin-1 ( x 1 - y2 + y 1 - x2 ) ■

The following can be proved similarly.

T H E O R E M 2.17 1. If 0 < x < 1, 0 < y < 1 and x2 + y2 > 1, then

Sin-1 x + Sin-1 y = p - Sin-1 ( x 1 - y2 + y 1 - x2 )


For any x and y Î[0, 1],

Sin-1 x + Sin-1 y = Cos-1[ (1 - x2 )(1 - y2 ) - xy]


3. For any x and y Î[0, 1],

Sin-1 x - Sin-1 y = Sin-1[ x 1 - y2 - y 1 - x2 ]


4. If 0 £ y £ x £ 1,

Sin-1 x - Sin-1 y = Cos-1[ (1 - x2 )(1 - y2 ) + xy]

T H E O R E M 2.18 £ x £ 1 and 0 £ y £ 1.
1. Cos-1 x + Cos-1 y = Cos-1[ xy - (1 - x2 )(1 - y2 )]

2. If x2 + y2 ³ 1, Cos-1 x + Cos-1 y = Sin-1[ x 1 - y2 + y 1 - x2 ]


2.2 Properties of Inverse Trigonometric Functions 111

3. Cos-1 x - Cos-1 y = Sin-1[ y 1 - x2 - x 1 - y2 ]

4. If x £ y, Cos-1 x - Cos-1 y = Cos-1[ xy + (1 - x2 )(1 - y2 )]

Example 2.3

Evaluate the following: é 3 15 8 4 ù


= Sin-1 ê × + × ú
æ 3ö æ 8ö ë 5 17 17 5 û
(1) Sin-1 ç ÷ + Sin-1 ç ÷
è 5ø è 17 ø
æ 77 ö
= Sin-1 ç ÷
æ 3ö æ 12 ö è 85 ø
(2) Cos-1 ç ÷ + Cos-1 ç ÷
è 5ø è 13 ø (2) Using part (1) of Theorem 2.18,
-1 -1
(3) Tan 2 + Tan 3 æ 3ö æ 12 ö
Cos-1 ç ÷ + Cos-1 ç ÷
è 5ø è 13 ø
Solution:
é 3 12 æ 3ö
2
æ 12 ö ù
2
(1) Using Theorem 2.16 we have = Cos-1 ê × - 1-ç ÷ 1-ç ÷ ú
êë 5 13 è 5ø è 13 ø ú
û
Sin-1 x + Sin-1 y = Sin-1[ x 1 - y2 + y 1 - x2 ]
é 36 4 5 ù
if x2 + y2 £ 1. Here = Cos-1 ê - × ú
ë 65 5 13 û
2 2
æ 3ö æ 8ö 9 64 4201 æ 16 ö
çè ÷ø + çè ÷ø = + = <1 = Cos-1 ç ÷
5 17 26 289 7225 è 65 ø
Therefore (3) By Theorem 2.12,
æ 2+3 ö
æ 3ö æ 8ö Tan-1 2 + Tan-1 3 = p + Tan-1 ç
Sin-1 ç ÷ + Sin-1 ç ÷ è 1 - 2.3 ÷ø
è 5ø è 17 ø
= p + Tan-1 ( - 1)
é3 æ 8ö
2
8 æ 3ö ù
2

= Sin-1 ê 1 - ç ÷ + 1-ç ÷ ú = p - Tan-1 (1)


êë 5 è 17 ø 17 è 5ø ú
û
p 3p
=p - =
4 4

Example 2.4

Prove that for any -1 < x < 1, Now, we have

1 æ 2x ö æ 2x ö æ 2 tan q ö
Tan-1 x = Tan-1 ç Tan-1 ç = Tan-1 ç
2 è 1 - x2 ÷ø 2÷
è1- x ø è 1 - tan2 q ÷ø

Solution: Put q = Tan-1 x. Then = Tan-1 (tan 2q )


tan q = x and -p/2
p < q < p/2
p æ -p pö
= 2q çè since < 2q < ÷
2 2ø
Since -1 < x < 1, it follows that
= 2 Tan-1 x
-p p
<q <
4 4 Therefore
-p p æ 2x ö
or < 2q < Tan-1 x =
1
Tan-1 ç
2 2 2 è 1 - x2 ÷ø
112 Chapter 2 Inverse Trigonometric Functions

WORKED-OUT PROBLEMS
Single Correct Choice Type Questions
å å
n n
1. Tan-1 2 + Tan-1 3 = 3. If Cos-1 ( xi ) = 0, then xi is
i =1 i =1
p 3p 5p 2p np
(A) (B) (C) (D) (A) 0 (B) -n (C) n (D)
4 4 4 3 2
Solution: Here x = 2, y = 3 so that xy > 1. From
å
n
Solution: Since 0 £ Cos-1 x £ p and Cos-1 xi = 0
Theorem 2.12 we have i =1
we have
æ x+ yö Cos-1 xi = 0 and xi = 1 for i = 1, 2, 3, …, n
Tan-1 x + Tan-1 y = p + Tan-1 ç
è 1 - xy ÷ø
Therefore
Therefore substituting the values we get n

æ 2+3 ö
åx
i =1
i =n
Tan-1 2 + Tan-1 3 = p + Tan-1 ç
è 1 - 2 × 3 ÷ø Answer: (C)
-1
= p + Tan (-1)
4. The value of
p
=p - æ
4 4 æ 2öö
tan ç Cos-1 + Tan-1 ç ÷ ÷
3p è 5 è 3ø ø
=
4 is
Answer: (B)
6 17 16 18
(A) (B) (C) (D)
1 1 1 1 17 6 7 7
2. Tan-1 + Tan-1 + Tan-1 + Tan-1 is equal to
5 7 3 8 Solution: Put Cos-1(4/5) = a and Tan-1(2/3) = b. There-
p 3p p 2p fore 0 < a, b < p
p/2 and so
(A) (B) (C) (D)
4 4 3 3 4 2
cos a = and tan b =
Solution: 5 3
This gives
æ -1 1 1ö æ 1 1ö
çè Tan + Tan-1 ÷ + ç Tan-1 + Tan-1 ÷
5 7ø è 3 8ø 3 2
tana = and tan b =
4 3
æ 1 1 ö æ 1 1 ö
+ +
ç 5 7 ÷
-1 -1 ç 3 8 ÷ Therefore
= Tan ç + Tan ç (∵ xy < 1)
1 1 ÷ 1 1÷
çè 1 - × ÷ø çè 1 - × ÷ø tan a + tan b
5 7 3 8 n(a + b ) =
1 - tan a tan b
æ 6ö æ 11 ö
= Tan-1 ç ÷ + Tan-1 ç ÷ (3 / 4) + (2 / 3)
è 17 ø è 23 ø =
1 - (3 / 4)×(2 / 3)
æ 6 11 ö
+ 17
ç ÷ =
= Tan ç 17 23 ÷
-1
∵ xy < 1)
(∵ 6
6 11
çè 1 - × ÷ø
17 23 Answer: (B)
æ 325 ö
= Tan-1 ç
è 325 ÷ø æ
5. Tan ç Cos-1
1
- Sin-1
4 ö
è ÷ is
5 2 17 ø
= Tan-1 (1)
29 29 3 3
p (A) (B) (C) (D)
= 3 3 29 29
4
Answer: (A)
Worked-Out Problems 113

Solution: Put We have that


1 4 1 1
Cos-1 =a and Sin-1 =b =
5 2 17 (1 + x) + 1
2
x +1
2

< a, b < p/
p 2 and so Therefore
1 4
cos a = and sin b = (1 + x)2 + 1 = x2 + 1
5 2 17
2x + 1 = 0
From these values we can find out sin a and cos b as
-1
x=
7 1 2
sin a = and cos b =
5 2 17 Answer: (D)
Therefore
8. If 0 < x < 1, then
sin a
tana = =7 1 + x2 [{xcos(Cot-1 x) + sin(Cot-1 x)}2 - 1]1/2 =
cos a
x
sin b (A) (B) x
and tan b = =4 1 + x2
cos b
Now (C) x 1 + x2 (D) 1 + x2

tan a - tan b 7-4 3 Solution: Put Cot-1 x = a. Then


tan(a - b ) = = =
1 - tan a tan b 1 + 7 × 4 29 p
0 < x < 1Þ 0 <a <
Answer: (D) 2
Therefore the given expression equals
6. The number of real values of x satisfying the equation 1/ 2
éæ 2
ù
Tan-1 x( x + 1) + Sin-1 x2 + x + 1 = p / 2 is 2 ê x2 1 ö
1+ x ç + ÷ - 1ú
(A) 0 (B) 1 (C) 2 (D) infinite êè 1 + x2 1 + x2 ø ú
ë û
Solution: Observe that x(x + 1) ³ 0 and 0 < x2 + x + 1 £ 1.
Therefore = 1 + x2 [( 1 + x2 )2 - 1]1/ 2

x( x + 1) + 1 ³ 1 and x2 + x + 1 £ 1 = 1 + x2 x2

This implies = x 1 + x2 (∵ x > 0)

x2 + x + 1 = 1 Answer: (C)

or x = 0, - 1 9. The numerical value of tan(2 Tan-1(1/5) - (p/4))


p is
Answer: (C) 7 17 -17 -7
(A) (B) (C) (D)
17 7 7 17
7. The value of x for which sin(Cott-1(1 + x)) = cos(Tan-1 x) is
Solution: Put Tan-1(1/5) = a. Since 1/5 < 1 we have
1 -1 0 < a < p/4.
/ Therefore
(A) (B) 1 (C) 0 (D)
2 2
p
0 < 2a <
Solution: Let Cot-1(1 + x) = a and Tan-1 x = b. Therefore 2
cot a = 1 + x and tan b = x Hence
This gives æ p ö tan 2a - 1 [2 tana /(1 - tan2 a )] - 1
tan ç 2a - ÷ = =
-p p è 4 ø tan 2a + 1 [2 tana /(1 - tan2 a )] + 1
0<a <p and <b< and sin a = cos b
2 2 2 tan a + tan2 a - 1
=
2 tan a + 1 - tan2 a
114 Chapter 2 Inverse Trigonometric Functions

(2 / 5) + (1/ 25) - 1 12. The number of real values of x satisfying the equation
=
(2 / 5) + 1 - (1/ 25)
sin[2 Cos-1 {cot(2 Tan-1 x)}] = 0
-14 -7
= = is
34 17
(A) 2 (B) 4 (C) 6 (D) 8
Answer: (D)
Solution: Clearly x = 0 cannot be a solution and x = ±1
10. cos(Tan-1(sin(Cot-1 x))) is equal to are solutions. We suppose x ¹ 1 and -1. Put Tan-1 x = a.
Therefore
x2 + 1 x2 - 1
(A) (B)
x2 + 2 x2 + 2 tana = x ¹ 1, - 1

x2 - 1 x2 + 1 Now
(C) (D)
x2 + 2 x2 + 2 1 1 - tan2 a 1 - x2
cot 2a = = =
Solution: -1
x = a so that cot a = x. Now tan 2a 2 tan a 2x
1 1 Now put
sina = =
cosec a x2 + 1 æ 1 - x2 ö
Cos-1 ç =b
Again put è 2 x ÷ø
æ 1 ö
Tan-1 (sin a ) = Tan-1 ç ÷ =b
è x2 + 1 ø 1 - x2
cos b =
so that 2x
1 Therefore
tan b =
x +1
2
LHS = sin 2 b = 2 sin b cos b
Now 2
æ 1 - x2 ö 1 - x2
= 2 1-ç ×
1 1 è 2 x ÷ø 2x
cos b = =
sec b 1 + tan2 b
Now
1 x +1
2
æ 1 - x2 ö
2
= = sin 2 b = 0 Þ 1 - ç = 0 (∵ x ¹ 1, - 1)
1 + [1/( x2 + 1)] x2 + 2 è 2 x ÷ø
Answer: (A)
Þ (1 - x2 )2 = 4 x2
11. If Cos-1 x + Cos-1 y + Cos-1 z = 3p,
p then the value of Þ (1 + 2 x - x2 )(1 - 2 x - x2 ) = 0
xy + yz + zx is
This implies
1
(A) 0 (B) -3 (C) (D) 3
3 x = -1 ± 2 or x =1± 2
-1 -1 -1
Solution: Since 0 £ Cos x, Cos y, Cos z £ p
p, we have which gives
-1 -1 -1
Cos x = p , Cos y = p and Cos z = p x = ± 1, - 1 ± 2 and 1 ± 2
Answer: (C)
x = y = z = -1
13. If Sin-1 x + Sin-1 y + Sin-1 z = 3p/2,
p then the value of
Substituting we get
9
x2010 + y2011 + z2012 +
xy + yz + zx = 3 x2010 + y2011 + z2012
Answer: (D)
is
(A) 0 (B) 3 (C) 6 (D) 9
Worked-Out Problems 115

Solution: We have -p/2 p £ Sin-1 x, Sin-1 y, Sin-1 z £ p/2


p -1 æ 1ö -1 æ 1ö
-1 æ 142 ö
and Sin x + Sin y + Sin-1 z = 3p /2. This implies
-1 -1 15. 3 Tan ç ÷ + Tan ç ÷ + Sin ç is equal to
è 2ø è 5ø è 65 5 ÷ø
p
Sin-1 x = Sin-1 y = Sin-1 z = p 3p
2 (A) (B) p (C) (D) -p
2 4
Therefore x = y = z = 1 and hence
Solution:
9 9
x2010 + y2011 + z2012 + =3+ =6 æ ö
x2010 + y2011 + z2012 3 æ 142 ö 142 -1 æ 142 ö
Sin-1 ç = Tan -1
ç ÷ = Tan ç
Answer: (C)
÷
è 65 5 ø çè (65 5 ) - (142) ÷ø
2 2 è 31 ÷ø

14. If 0 £ p, q < 1 and æ 1ö æ 2/5 ö æ 5ö


2 Tan-1 ç ÷ = Tan-1 ç = Tan-1 ç ÷
è 5ø è 1 - (1/ 25) ÷ø è 12 ø
æ 2p ö æ 1 - q2 ö æ 2x ö
Sin-1 ç - Cos-1 ç = Tan-1 ç

è1+ p ø è 1 + q2 ÷ø è 1 - x2 ÷ø æ 1ö æ (3 / 2) - (1/ 8) ö æ 11ö
3 Tan-1 ç ÷ = Tan-1 ç = Tan-1 ç ÷
then x is equal to
è 2ø è 1 - (3 / 4) ÷ø è 2ø
p-q p+q Therefore the given sum equals
(A) (B)
1 + pq 1 + pq
p-q p+q æ 11ö æ 5ö æ 142 ö
Tan-1 ç ÷ + Tan-1 ç ÷ + Tan-1 ç
(C) (D) è 2ø è 12 ø è 31 ÷ø
p+q p-q
Solution: Put p = tan q so that 0 £ q < p
p/4 (∵ 0 £ p < 1). æ (11/22) + (5 / 12) ö æ 142 ö
= p + Tan-1 ç + Tan-1 ç
Therefore è 1 - (55 / 24) ÷ø è 31 ÷ø

æ 2p ö æ pö æ 142 ö æ 142 ö
Sin-1 ç = Sin-1 (sin 2q ) = 2q çè∵ 0 £ 2q < ÷ø = p - Tan-1 ç + Tan-1 ç
è 1 + p2 ÷ø 2 è 31 ÷ø è 31 ÷ø

= 2 Tan-1 p =p
Answer: (B)

16. If Cos-1 x + Cos-1 y + Cos-1 z = p


p, then the value of
æ 1 - q2 ö
-1
Cos ç = 2 Tan-1 q x2 + y2 + z2 + 2xyz is
è 1 + q2 ÷ø
(A) 1 (B) 0 (C) x + y + z (D) x2 y2 z2
Therefore
Solution: Let Cos-1 x = a, Cos-1 y = b and Cos-1 z = g .
Therefore cos a = x, cos b = y and cos g = z and at least
æ 2p ö -1 æ 1 - q ö
2
Sin-1 ç - Cos -1 -1
çè 1 + q2 ÷ø = 2(Tan p - Tan q) one of x, y and z is non-negative. Now
è 1 + p2 ÷ø
a + b +gg = p Þ a + b = p - g
æ p-q ö
-1
= 2 Tan ç Therefore
è 1 + pq ÷ø
cos(a + b ) = cos(p - g ) = - cos g
æ 2x ö cos a cos b - sin a sin b = - cos g
= Tan-1 ç (Given)
è 1 - x2 ÷ø
This gives
= 2 Tan-1 x
xy - 1 - x2 1 - y2 = -z
This implies
( xy + z)2 = (1 - x2 )(1 - y2 )
p-q
x= x2 y2 + z2 + 2 xyz = 1 - x2 - y2 + x2 y2
1 + pq
Answer: (A) x2 + y2 + z2 + 2 xyz = 1
Answer: (A)
116 Chapter 2 Inverse Trigonometric Functions

17. If a + b = c , c ¹ 0, then the number of values of x


2 2 2
Therefore
such that Sin-1(ax/c) + Sin-1(bx/c) = Sin-1 x is
é ïì x 1 - x2 ïü ù æ p ö
(A) 2 (B) 1 (C) 0 (D) 3 tan êSin-1 í -1
+ ý - Sin x ú = tan çè q + - q ÷ø
Solution: Clearly x = 0 is a solution. Suppose x ¹ 0. Put êë îï 2 2 þï úû 4

ax ax p
Sin-1 =a and Sin-1 =b = tan =1
c c 4
Answer: (B)

ax bx
sin a = and sin b = æ 1ö
c c 19. sin ç 2 Tan-1 ÷ + cos (Tan-1 2 2 ) =
è ø 3
Now 1 3 14 11
-1
(B) (C) (D)
a + b = Sin x Þ sin(a + b ) = x 15 15 15 15
Þ sin a cos b + cos a sin b = x Tan-1(1/3) = a and Tan-1 2 2 = b.
Solution: Put T
Therefore
ax c2 x 2 æ a2 x2 ö bx
Þ 1- 2 + ç 1- 2 ÷ =x 1
c c è c ø c tan a = and tan b = 2 2
3
a b
Þ c2 - b2 x2 + c2 - a2 x2 = 1 This implies
c2 c2
1 3 1
Þ a2 (c2 - b2 x2 ) + b2 (c2 - a2 x2 ) + sin a = , cos a = and cos b =
10 10 3
2ab c2 - b2 x2 × c2 - a2 x2 = c4 Therefore
Therefore LHS = sin 2a + cos b = 2 sin a cos a + cos b

c2 (a2 + b2 ) - 2a2 b2 x2 + 2ab c2 - b2 x2 × c2 - a2 x2 = c4 2 3 1


= ´ +
10 10 3
a2 b2 x2 = ab c2 - b2 x2 × c2 - a2 x2 (∵ a2 + b2 = c2 )
6 1 14
= + =
a b x = (c - b x )(c - a x )
2 2 4 2 2 2 2 2 2
10 3 15
= c4 - (a2 + b2 ) x2 + a2 b2 x4 Answer: (C)

(a2 + b2 ) x2 = c4 20. If
This gives x2 = 1 or x = ±1. æ 1 ö
sin ç Sin-1 + Cos-1 x÷ = 1
Answer: (D) è 5 ø

18. If 0 £ x £ 1/2, then the value of x is


1 1 1 1
é ìï x üï ù (A) (B) (C) (D)
1 - x2 2 3 4 5
tan êSin-1 í + -1
ý - Sin x ú
êë ïî 2 2 ïþ úû Solution: Put Sin-1(1/5) = a and Cos-1 x = b.
Therefore
is
1 1
(A) -1 (B) 1 (C) 3 (D) sin a = and cos b = x
3 5
Solution: Put x = sin q where 0 £ q £ p/
p 6. Now Now

ìï x 1 - x2 üï 1 = sin(a + b )
-1 æ Sin q + Cos q ö
Sin-1 í + ý = Sin ç ÷ø
è 1 = sin a cos b + cos a sin b
îï 2 2 þï 2

æ æ p öö p æ p pö 1 24
= Sin-1 ç sin ç q + ÷ ÷ = q + 1 = ( x) + 1 - x2
è è çè∵ 0 £ q + £ ÷ø 5 5
4øø 4 4 2
Worked-Out Problems 117

5 - x = 24 1 - x2 From these we get


2 1
(5 - x)2 = 24(1 - x2 ) cos a = and sin b =
5 10
25x2 - 10 x + 1 = 0
Therefore
(5 x - 1)2 = 0 1 1
tan a = and tan b =
1 2 3
x=
5 Hence
Answer: (D)
tan a + tan b (1/ 2) + (1/ 3) 5
tan(a + b ) = = = =1
21. If 1 - tan a tan b 1 - (1/ 6) 5
p
Tan-1
1
+ Tan-1
1 2
= Tan-1 2 a + b = Tan-1 (1) =
2x + 1 4x + 1 x 4
Answer: (B)
and x ¹ 0, then x equals
-2 2 23. If cot[Sin-1 ( 13 / 17 )] = sin(Tan-1 a ), then a is
(A) 3 (B) (C) -3 (D)
3 3
4 172 - 132
Solution: (A) (B)
17 17 ´ 13
æ {1/(2 x + 1)} + {1/(4 x + 1)} ö 2
Tan-1 ç ÷ = Tan-1 2 172 - 132 2
è 1 - {1/(2 x + 1)}× {1/(4 x + 1)} ø x (C) (D)
172 + 132 3
æ 6x + 2 ö 2
Tan-1 ç 2 = Tan-1 2 Solution: Put Sin-1 ( 13 / 17 ) = q . Therefore
è 8 x + 6 x ÷ø x
13
Therefore sinq =
17
3x + 1 2
= 2 Also
4 x + 3x x
2

x2 (3 x + 1) = 2(4 x2 + 3 x) cot q = cosec2 q - 1

1
3 x3 - 7 x2 - 6 x = 0 = -1
sin2 q
x(3 x2 - 7 x - 6) = 0
17 2
x( x - 3)(3x + 2) = 0 = -1=
13 13
We take x = 3 because when x = -2/3, LHS will be Again put Tan-1 a = f so that tan f = a. Now
negative whereas Tan-1(2/x
/ 2) is positive.
1 1
Answer: (A) sin f = =
cosec f cot f + 1
2

22. If Sin-1 (1/ 5 ) and Cos-1 (3 / 10 ) are angles in [0, p/2],


p 1 a
= =
then their sum is equal to (1/a ) + 1
2
a2 + 1
p p p 1
(A) (B) (C) (D) Sin-1 Therefore
6 4 3 50
2 a
cotq = sin f Þ =
Solution: Let Sin-1 (1/ 5 ) = a and Cos-1 (3 / 10 ) = b . 13 1 + a2
Then
4 a2
1 3 p Þ =
sin a = , cos = b and 0 < a , b < 13 1 + a 2
5 10 2
13 1 + a 2 1
Þ = = 2 +1
4 a2 a
118 Chapter 2 Inverse Trigonometric Functions

13 1
Þ -1= 2
4 a p
a+b=
9 1 4
Þ = 2
4 a
p /4.
and hence the third angle is 3p
2
Þa = Answer: (C)
3
Answer: (D) 26. Consider the following two statements P and Q.

-1
24. If x, y, z are in GP and Tan x, Tan y, Tan z are in
-1 -1
æ 4p ö 2p
P : Cos-1 ç cos ÷ =
AP, then è 3ø 3
(A) x = y = z or y = ±1
æ 1ö æ 1ö
1 Q : sec2 ç Cot-1 ÷ + cosec2 ç Tan-1 ÷ = 15
(B) z = è 2ø è 3ø
x
(C) x = y = z, but their common value may not Which of the following is true?
necessarily be 0 (A) Both P and Q are true
(D) x = y = z = 0 (B) P is true, but Q is false
Solution: We have (C) P is false, but Q is true
(D) Both P and Q are false
y2 = zx and Tan-1 x + Tan-1 z = 2 Tan-1 y
Solution: We have
æ x+z ö -1 -1 æ 2y ö
y = zx and Tan ç
2
÷ = Tan ç 2 ÷
y ¹ ±1 æ 4p ö p -1
è 1 - xz ø è1- y ø cos ç ÷ = - cos =
è 3ø 3 2
æ x+z ö æ 2y ö
y2 = zx and Tan-1 ç 2 ÷
= Tan-1 ç 2 ÷
è1- y ø è1- y ø
æ -1ö æ 1ö p 2p
Cos-1 ç ÷ = p - Cos-1 ç ÷ = p - =
x + z = 2y
2 è 2ø è 2ø 3 3
This implies x, y, z are in AP. That is x, y, z are in G.P. as This implies P is true. Now,
well as in AP. Therefore
æ 1ö æ 1ö
x=y=z or y = ±1 sec2 ç Cot-1 ÷ + cosec2 ç Tan-1 ÷ = [1 + tan2 (Tan-1 2)]
è 2 ø è 3ø
Answer: (A)
+ [1 + cot2 (Cot-1 3)]
25. If Sin-1 (1/ 5 ) and Sin-1 (1/ 10 ) are two angles of a = 1 + 4 + 1 + 9 = 15
triangle, then the third angle is
Therefore Q is also true.
p p 3p 2p
(A) (B) (C) (D) Answer: (A)
4 2 4 3
Solution: We have 27. The value of x satisfying the equation

1 æ 1ö æ 1 ö -1 æ 1 ö p
Sin-1 = Tan-1 ç ÷ Sin-1 ç ÷ = Tan ç ÷ Tan-1 (2 x) + Tan-1 (3 x) =
5 è2ø è 10 ø è 3ø 4
(by Section 2.2) is
Put Tan-1(1/2) = a and Tan-1(1/3) = b. Therefore
1 -1
(A) -1 (B) (C) 1 (D)
1 1 6 6
tan a = and tan b =
2 3
Solution: Observe that x = ±1 is not a solution. Let
This implies that 0 < a and b < p
p/4. Now Tan-1(2x) = a and Tan-1(3x) = b. Therefore tan a = 2x,
tan b = 3x. Now
tan a + tan b (1/ 2) + (1/ 3) 5
tan(a + b ) = = = =1
1 - tan a tan b 1 - (1/ 2) × (1/ 3) 5
Worked-Out Problems 119

p æ 2x ö æ 2 tan q ö
a+b= Þ tan(a + b ) = 1 Sin-1 ç = Sin-1 ç
4 è 1 + x2 ÷ø è 1 + tan2 q ÷ø
tan a + tan b = Sin-1 (sin 2q ) = 2q
Þ =1
1 - tan a tan b
Again
2 x + 3x
Þ =1 æ 1 - x2 ö æ 1 - tan2 q ö
1 - 6 x2 Cos-1 ç = Cos-1 ç

è1+ x ø è 1 + tan2 q ÷ø
Þ 6 x2 + 5 x - 1 = 0
= Cos-1 (cos 2q )
Þ (6 x - 1)( x + 1) = 0
= 2q
1
Þx= (∵ x ¹ ± 1) Therefore, the given expression equals
6
Answer: (B) æ1 1 ö
tan ç (2q ) + (2q )÷ = tan 2q
è2 2 ø
28. The value of cos(2 Cos-1 x + Sin-1 x) at x = 1/5 where
Cos-1 x and Sin-1 x are principal values is 2 tan q
=
1 - tan2 q
24 -4 4 24
(A) (B) (C) (D) - 2x
5 5 5 5 =
1 - x2
Solution: Answer: (B)
-1 -1
cos(2 Cos x + Sin x)
30.
æ pö
= cos ç Cos-1 x + ÷ æ 5ö æ 12 ö p
è 2ø Sin-1 ç ÷ + Sin-1 ç ÷ =
è xø è xø 2
= -sin(Cos-1 x) = -sin(Sin-1 1 - x2 )
then x is equal to
= - 1- x 2
(A) 17 (B) 15 (C) 13 (D) 19
-1 -1
Therefore at x = 1/5 the required value is Solution: Sin (5/x/ ) and Sin (12/x
/ ) are both defined
when x > 12. Therefore by Theorem 2.7,
1
cos(2 Cos-1 x + Sin-1 x) = - 1 - æ 25 ö æ 12 ö p
25 Cos-1 ç 1 - 2 ÷ + Sin-1 ç ÷ =
è x ø è xø 2
24
=-
æ 12 ö æ 12 ö
5 = Cos-1 ç ÷ + Sin-1 ç ÷
è xø è xø
Answer: (D)
Comparing we get
29. For any 0 £ x £ 1,
12 25
é1 æ 2x ö 1 ù = 1- 2
-1 æ 1 - x ö
2
tan ê Sin-1 ç ÷ + Cos ç 2 ÷ú
x x
è1+ x è 1 + x øû
2
ë2 ø 2
Therefore
is equal to
144 = x2 - 25 or x = 13
1- x 2
2x Answer: (C)
(A) (B)
1 + x2 1 - x2
2x 31. If x, y, z are positive real numbers, then
(C) (D) 0
1 + x2
x( x + y + z) y( x + y + z)
Tan-1 + Tan-1
Solution: Put Tan-1 x = q. T Then tan q = x and -p/2
p <q< yz zx
p/2. Since 0 £ x < 1, we get that 0 £ q < p
p p/4. Hence 0 £ 2q <
p/2. Now
p z( x + y + z)
+ Tan-1 =
xy
120 Chapter 2 Inverse Trigonometric Functions

p p Solution: We have
(A) 0 (B) (C) (D) p
2 4
æ4ö æ 3ö æ 3 ö -1 æ 3 ö
Cos-1 ç ÷ = Tan-1 ç ÷ and Sin-1 ç ÷ = Tan ç ÷
Solution: Put è 5ø è4ø è 34 ø è 5ø
x( x + y + z) (by Theorem 2.9)
a=
yz Therefore
y( x + y + z) æ 4ö æ 3 ö
b= Cos-1 ç ÷ + Sin-1 ç
zx è 5ø è 34 ÷ø
z( x + y + z) æ 3ö æ 3ö
c= = Tan-1 ç ÷ + Tan-1 ç ÷
xy è 4ø è 5ø

Therefore æ (3 / 4) + (3 / 5) ö æ 3 3 ö
= Tan-1 ç çè∵ × < 1÷ø
è 1 - (3 / 4)×(3 / 5) ÷ø 4 5
( xy)
ab = ( x + y + z)
xyz2 æ 27 ö
= Tan-1 ç ÷
è 11 ø
x+ y+z x+y
= =1+ >1 Answer: (B)
z z
Now 33. If

æ a+b ö 1 1 1 p
Tan-1 a + Tan-1 b = p + Tan-1 ç ÷ Cot-1 + Cot-1 + Cot-1 =
è 1 - ab ø x y z 2

(by Theorem 2.11) then xy + yz + zx is equal to


(A) 1 (B) -1
é x+ y+zæ x yö ù
ê çè y + ú (C) x + y + z (D) -(x + y + z)
-1 ê
z x ÷ø ú
= p + Tan ê Solution: Put Cot-1(1/x / ) = a, Cot-1(1/y
/ )=b and
-( x + y) ú
ê ú Cot-1(1/z) = g . Therefore
T
êë z úû
æp ö
cot(a + b ) = cot ç - g ÷ = Tan g
é x + y + z æ x + yö æ z ö ù è2 ø
= p + Tan-1 ê - ç ÷ç ÷ú
êë z è xy ø è x + y ø úû cot a cot b - 1
= tan g
cot b + cot a
é z( x + y + z) ù
= p - Tan-1 ê ú (1/ x) × (1/ y) - 1
xy =z
ë û (1/ y) + (1/ x)

= p - Tan-1c 1 - xy = z( x + y)
So xy + yz + zx = 1
Answer: (A)
Tan-1 a + Tan-1 b + Tan-1 c = p
Answer: (D) 34. If Cos-1(3/5) + Cos-1(12/13) = Cos-1(k), then the value
of k is
æ4ö
-1 -1 æ 3 ö
32. Cos ç ÷ + Sin ç ÷= 16 12 11 19
è 5ø è 34 ø (A) (B) (C) (D)
65 65 65 65
æ 27 ö æ 27 ö Solution: It is known that
(A) Tan-1 ç ÷ (B) Tan-1 ç ÷
è 13 ø è 11 ø
Cos-1 x + Cos-1 y = Cos-1 ( xy - 1 - x2 1 - y2 )
æ 23 ö æ 23 ö
(C) Tan-1 ç ÷ (D) Tan-1 ç ÷ when x + y ³ 0.
è 11 ø è 12 ø
Worked-Out Problems 121

Therefore æ n ö æ n ö
(C) Tan-1 ç (D) Tan-1 ç
æ 3 12 è n + 2 ÷ø è n + 1÷ø
9 144 ö
Cos-1 (k ) = Cos-1 ç × - 1 - × 1-
è 5 13 25 169 ÷ø Solution:
æ 36 20 ö æ 1 ö æ k + 1- k ö
= Cos-1 ç -
è 65 65 ÷ø uk = Tan-1 ç ÷
è 1 + k(k + 1) ø
= Tan-1 ç
è 1 + k(k + 1) ÷ø
æ 16 ö = Tan-1 (k + 1) - Tan-1 k
= Cos-1 ç ÷
è 65 ø
Put k = 1, 2, 3, …, n and add so that the given sum equals
This implies
æ n+ 1-1 ö
16 Tan-1 (n + 1) - Tan-1 (1) = Tan-1 ç
k= è 1 + 1× (n + 1) ÷ø
65
Answer: (A) æ n ö
= Tan-1 ç
è n + 2 ÷ø
-1 æ 1 ö æ 1 ö æ 1 ö
35. Tan ç + Tan-1 ç + Tan-1 ç + Answer: (C)
è 1 + 1× 2 ÷ø è 1 + 2 × 3 ÷ø è 1 + 3 × 4 ÷ø
upto n terms is equal to
æ n - 1ö æ n + 1ö
(A) Tan-1 ç (B) Tan-1 ç
è n + 1÷ø è n + 2 ÷ø

Multiple Correct Answer Type Questions


1. Which of the following are true? æ 1ö æ 1ö
(B) Tan-1 ç ÷ + Tan-1 ç ÷
1 1 2 è 2ø è 3ø
(A) Tan-1 + Tan-1 = Tan-1
7 13 9
æ (1/ 2) + (1/ 3) ö
= Tan-1 ç
æ1ö æ 1ö p
(B) Tan-1 ç ÷ + Tan-1 ç ÷ = è 1 - (1/ 2) × (1/ 3) ÷ø
è2ø è 3ø 4
æ 5ö
3p = Tan-1 ç ÷
-1
(C) Tan (3) + Tan (4) = -1 è 5ø
4
= Tan-1 (1)
æ x+yö
(D) Tan-1 x + Tan-1 y = Tan-1 ç ÷ if xy > 1 p
è 1 - xy ø =
4
Solution: Therefore (B) is true.
(A) We know that (C) Here x > 0, y > 0 and xy > 1 so that
æ x+ yö
Tan-1 x + Tan-1 y = Tan-1 ç if xy < 1 æ x+ yö
è 1 - xy ÷ø Tan-1 x + Tan-1 y = p + Tan-1 ç
è 1 - xy ÷ø
Therefore
Therefore
æ 1ö æ 1ö æ (1/ 7) + (1/ 13) ö
Tan-1 ç ÷ + Tan-1 ç ÷ = Tan-1 ç æ 4+3 ö
è 7ø è 13 ø è 1 - (1/ 7) × (1/ 13) ÷ø Tan-1 3 + Tan-1 4 = p + Tan-1 ç ÷
è 1 - 12 ø
æ 13 + 7 ö
= Tan-1 ç æ -7 ö 3p
è 90 ÷ø = p + Tan-1 ç ÷ ¹
è 11 ø 4
æ 2ö
= Tan-1 ç ÷
è 9ø Hence (C) is not true.
(D) According to Theorem 2.12, (D) is not true.
Therefore (A) is true.
Answers: (A), (B)
122 Chapter 2 Inverse Trigonometric Functions

2. The values of x satisfying the equation 4. Identify the correct statements.


(A) The number of solutions of the equation
æ 2ö -1 æ 2ö -1 æ 4 ö
Tan-1 ç x + ÷ - Tan çè x - ÷ø - Tan çè ÷ø = 0
è xø x x æ x ö æ 1 ö 3p
Tan-1 ç + Tan-1 ç 3 ÷ =
è 1 - x2 ÷ø èx ø 4
(A) 3 (B) - 3 (C) - 2 (D) 2 belonging to the interval (0, 1) is two.
Solution: The given equation can be written as (B) Tan -1(1/2) + Tan -1(1/8) + Tan -1(1/18) +  upto
p - Tan-1[1/(2n + 1)].
n terms is (p/4)
æ x + (2 / x) - x + (2 / x) ö æ 4ö
Tan-1 ç ÷ - Tan-1 ç ÷ = 0 (C) If 0 £ x £ 1 and q = Sin-1 x + Cos-1 x - Tan-1 x, then
è 1 + [ x - (4 / x )] ø
2 2
è xø p £ q £ p/
p/4 p 2.
æ 4x ö æ 4ö (D) The value of Sin-1[sin (4p /3)] + Cos-1[cos(7p / 6)]
Tan-1 ç 4 ÷ - Tan-1 ç ÷ = 0 is p / 2.
è x + x 2
- 4 ø è xø
Solution:
æ [4 x /( x4 + x2 - 4)] - (4 / x) ö
Tan-1 ç =0
è 1 + [16 /( x4 + x2 - 4)] ÷ø æ x öæ 1 ö 1
(A) ç = > 1 because 0 < x < 1
è 1 - x2 ÷ø çè x3 ÷ø (1 - x2 ) x2
4x 4
- =0
x +x -4 x
4 2
Therefore
x2 - ( x4 + x2 - 4) = 0 æ x ö æ 1 ö 3p
Tan-1 ç 2÷
+ Tan-1 ç 3 ÷ =
Therefore è 1 - x ø èx ø 4
x4 = 4 or x=± 2
æ {x /(1 - x2 )} + (1/ x3 ) ö 3p
Answers: (C), (D) Þ p + Tan-1 ç =
è 1 - {1/ x2 (1 - x2 )} ÷ø 4
3. When 0 £ x < 1, then Cos-1 x is equal to æ x4 - x2 + 1 ö -p
Þ Tan-1 ç =
æ 2 ö è x(- x4 + x2 - 1) ÷ø 4
(A) Sin-1 1 - x2 (B) Tan-1 ç 1 - x ÷
è x ø æ -1ö -p
Þ Tan-1 ç ÷ =
1 p è xø 4
(C) Cosec-1 (D) - Sin-1 x
1- x 2 2
Þ x = 1. But 0 < x < 1. Therefore (A) is false.
-1 -1
Solution: Let Cos x = q. Since 0 £ Cos x £ p and (B) Let
x > 0, it follows that 0 < q < p/2.
p
æ 1 ö
(A) sin q = 1 - cos2q = 1 - x2 uk = Tan-1 ç 2 ÷ for k = 1, 2, 3, ….
è 2k ø
Therefore
æ 2 ö
= Tan-1 ç 2 ÷
q = Sin-1 1 - x2 è 4k ø

Hence (A) is true. æ 2 ö


= Tan-1 ç 2 ÷ø
sin q 1- x 2 è 4 k - 1 + 1
(B) tan q = = and hence (B) is true.
cos q x æ 2 ö
= Tan-1 ç
1 1 è 1 + (2k + 1)(2k - 1) ÷ø
(C) cosec q = =
sin q 1 - x2
æ 2k + 1 - (2k - 1) ö
Therefore = Tan-1 ç
è 1 + (2k + 1)(2k - 1) ÷ø
1
q = Cosec-1
1 - x2 æ [1/(2k - 1)]] - [1/(2k + 1)] ö
= Tan-1 ç
è 1 + [1/(2k + 1)][1/(2k - 1)] ÷ø
Hence (C) is also true. By Theorem 2.10 (D) is also
true. æ 1 ö æ 1 ö
= Tan-1 ç ÷ - Tan-1 ç
Answers: (A), (B), (C), (D) è 2 k - 1 ø è 2k + 1÷ø
Worked-Out Problems 123

Therefore x x
1 + sin x = cos + sin
2 2
æ 1 ö
u1 + u2 +  + un = Tan-1 (1) - Tan-1 ç
è 2 n + 1÷ø
Therefore
é 1 - sin x + 1 + sin x ù -1 æ xö
Hence (B) is true. Cot-1 ê ú = Cot ç -cot ÷
êë 1 - sin x - 1 + sin x úû è 2 ø
p
(C) Sin-1 x + Cos-1 x - Tan-1 x = - Tan-1 x
2 æ æ xöö
= Cot-1 ç cot ç p - ÷ ÷
è è 2øø
p
q= - Tan-1 x x
2 =p -
2
Now
p (B) Let Cos-1 x = q so that cos x = q where -1 £ x £ 1 and
0 £ x £ 1 Þ 0 £ Tan-1 x £
4 0£q£p p. Now
-p
Þ £ - Tan-1 x £ 0 q
4 2 sin2 = 1 - cos q = 1 - x
2
p p p
Þ £ - Tan-1 x £ Therefore
4 2 2
Therefore q 1- x æ pö
sin = çè∵ 0 £ q £ ÷ø
p p 2 2 2
£q £
4 2 q 1- x
= Sin-1
Hence (C) is true. 2 2
æ 4p ö æ æ 7p ö ö 1- x
(D) Sin-1 ç sin ÷ + Cos-1 ç cos ç ÷ ÷ q = 2 Sin-1
è 3ø è è 6 øø 2

æ - 3ö æ
-1 - 3
ö p Hence (B) is true.
= Sin-1 ç ÷ + Cos ç 2 ÷ = 2
è 2 ø è ø (C) When x > 0, then we know that Tan-1(1/x
/ ) = Cot-1 x.
Therefore
Hence (D) is also true.
1 p
Answers: (B), (C), (D) Tan-1 x + Tan-1 = Tan-1 x + Cot-1 x =
x 2
5. Which of the following are true? So (C) is true.
p (D) Put x = tan q where –p/ 2 < q < p/ 2. Therefore
(A) If 0 < x < , then
2
é 1 + x2 - 1 ù æ S ec q - 1ö
é 1 - sin x + 1 + sin x ù x Tan-1 ê ú = Tan-1 ç
Cot-1 ê ú=p - êë x úû è T an q ÷ø
êë 1 - sin x - 1 + sin x úû 2
æ 1 - Cos q ö
1- x = Tan-1 ç
-1
(B) Cos ( x) = 2 Sin -1
è Sin q ÷ø
2
æ qö
1 p = Tan-1 ç Tan ÷
Tan-1 x + Tan-1 = x>0 è 2ø
x 2
q æ -p q p ö
æ ö 1 = çè∵ < < ÷
(D) Tan-1 ç 1 + x - 1÷ = Tan-1 ( x)
2
2 4 2 4ø
è x ø 2
1
Solution: = Tan-1 x
2
x x æ x xö Hence (D) is also true.
(A) 1 - sin x = cos - sin çè∵ cos > sin ÷ø
2 2 2 2 Answers: (A), (B), (C), (D)
124 Chapter 2 Inverse Trigonometric Functions

Matrix-Match Type Questions


1. Match the items of Column I with those of Column II. æ 1 1ö 1
= 2 ç Tan-1 + Tan-1 ÷ + Tan-1
è 5 8ø 7
Column I Column II æ (1/ 5) + (1/ 8) ö 1
= 2 Tan-1 ç ÷ + Tan-1
p è 1 - (1/ 40) ø 7
(A) If Sin-1 x + Sin-1 y = 2p/ 3, then the (p)
value of Cos-1 x + Cos-1 y is 4 æ 1ö 1
= 2 Tan-1 ç ÷ + Tan-1
3p è 3ø 7
(B) Tan-1(sec x + tan x) at x = p
p/3 is (q)
10 æ (2 / 3) ö 1
equal to = Tan-1 ç + Tan-1
p è 1 - (1/ 9) ÷ø 7
(C) For -1 £ x £ 1, if Sin-1 x = p
p/5, then (r)
Cos-1 x equals 3 æ 3ö æ 1ö
= Tan-1 ç ÷ + Tan-1 ç ÷
è 4ø è 7ø
5p
(D) 2 Cot-1 5 + Cot-1 7 + 2 Cot-1 8 equals (s) æ (3 / 4) + (1/ 7) ö
12 = Tan-1 ç
è 1 - (3 / 28) ÷ø
Solution: æ 25 ö p
= Tan-1 ç ÷ =
2p è 25 ø 4
(A) Sin-1 x + Sin-1 y =
3
Answer: (D) Æ (p)
Suppose Cos-1 x + Cos-1 y = a
a. Adding we get
p p 2p 2. Match the items of Column I with those of Column II.
+ = +a
2 2 3
Therefore Column I Column II

p p
a= (A) In DABC
D , if A = 90°, then Tan-1 (p)
3 4
T -1[c/(a + b)] =
[b/(c + a)] + Tan
Answer: (A) Æ (r) p
(B) The sides of a triangle are 5, 12 (q)
æ 1 + sin x ö and 13. Then Sin-1(12/13) + 2
(B) Tan-1 (sec x + tan x) = Tan-1 ç
è cos x ÷ø Cos-1(5/13) =

æ 1 - cos{(p / 2) + x} ö æ x - 1ö æ x + 1ö p
(C) Sin-1 ç ÷ + Sec-1 ç = (r)
= Tan-1 ç è x + 1ø è x - 1 ÷ø
è sin{(p / 2) + x} ÷ø 6
æ 3x + 2 ö æ 2x + 3ö
æ æ p xöö p x (D) Cos-1 ç + Cosec-1 ç = (s) p
= Tan-1 ç tan ç + ÷ ÷ = + è 2 x + 3 ÷ø è 3 x + 2 ÷ø
è è 4 2øø 4 2

Therefore at x = p/3,
p Solution:
p p 5p (A) It is given that a2 = b2 + c2. Therefore
Tan-1 (sec x + tan x) = + =
4 6 12
æ b ö æ c ö
Tan-1 ç ÷ + Tan-1 ç
Answer: (B) Æ (s) è c + aø è a + b ÷ø
(C) We have æ {b /(c + a)} + {c /(b + a)} ö
= Tan-1 ç
-1 -1
Sin x + Cos x =
p è 1 - {bc /(c + a)(a + b)} ÷ø
2
æ b2 + c2 + ab + ca ö
p p 3p = Tan-1 ç
è a2 + ab + ca ÷ø
-1
Cos x = - =
2 5 10
Answer: (C) Æ (q) = Tan-1 (1) (∵ b2 + c2 = a2 )
(D) 2 Cot-1 5 + Cot-1 7 + 2 Cot-1 8 p
=
1 4
= 2(Cot-1 5 + Cot-1 8) + Tan-1
7 Answer: (A) Æ (p)
Worked-Out Problems 125

(B) The given triangle is a right-angled triangle with hypot- = 1 - Cos2 q


enuse 13. Hence the other two angles are Sin-1(12/13)
and Cos-1(5/13) whose sum must be equal to p/2 / . 9
= 1-
Answer: (B) Æ (q) 25
(C) We have 4
=
æ 1ö 5
Sec-1 x = Cos-1 ç ÷
è xø Answer: (A) Æ (s)

Therefore æp 3ö æp pö
(B) sin ç - Sin-1 = sin ç - ÷
è2 2 ÷ø è 2 3ø
æ x - 1ö æ x + 1ö
Sin-1 ç ÷ + Sec-1 ç
è x + 1ø è x - 1 ÷ø p
= sin
6
æ x - 1ö æ x - 1ö
= Sin-1 ç + Cos-1 ç
è x + 1÷ø è x + 1÷ø =
1
2
p
= Answer: (B) Æ (p)
2
æ 3ö æ p 3ö
Answer: (C) Æ (q) cos ç Tan-1 ÷ø = cos q çè where 0 < q < and tan q = ÷ø
è 4 2 4
(D) We have

æ 1ö 1
Cosec-1 ( x) = Sin-1 ç ÷ =
è xø 1 + Tan2q
Therefore 1
=
9
æ 3x + 2 ö æ 2x + 3ö 1+
Cos-1 ç + Cosec-1 ç 16
è 2 x + 3 ÷ø è 3 x + 2 ÷ø
4
æ 3x + 2 ö æ 3x + 2 ö =
-1
= Cos ç ÷ + Sin-1 ç 5
è 2x + 3ø è 2 x + 3 ÷ø
Answer: (C) Æ (s)
p
= æp æ -1ö ö æ p æ -p ö ö
2 (D) cos ç - Sin-1 ç ÷ ÷ = cos ç - ç
è2 è 2 øø è 2 è 6 ÷ø ÷ø
Answer: (D) Æ (q)
æ 2p ö
3. Match the items of Column I with the items of Column II. = cos ç ÷
è 3ø
-1
Column I Column II =
2
1 Answer: (D) Æ (r)
(A) Value of sin[Cos-1(3/5)] is (p)
2
æ ö 4.
-3
(B) sinç p - Sin-1 3 ÷ = (q)
è2 2 ø 5
Column I Column II
æ 3ö -1
(C) cos ç Tan-1 ÷ = (r)
è 4ø 2 (A) If 1/ 2 £ x £ 1, then (p) p - Sin-1(3x - 4x3)
2 Sin-1 x is equal to
æp æ -1ö ö 4
(D) cos ç - Sin-1 ç ÷ ÷ = (s) (B) If 1/2 < x £ 1, then (q) p - Sin-1 (2 x 1 - x2 )
è2 è 2 øø 5 -1
3 Sin x is equal to
(C) If - 1 £ x £ - 1/ 2 , (r) -p - Sin-1(3x - 4x3)
Solution: then 2 Sin-1 x equals
æ 3ö æ 3 ö (D) -1 < x £ -1/2, then (s) -p - Sin-1 (2 x 1 - x2 )
(A) sin ç Cos-1 ÷ = sin q çè where cosq = , 0 < q < p ÷ø 3 Sin-1 x is
è 5ø 5
126 Chapter 2 Inverse Trigonometric Functions

Solution: = - 2 sin q cosq = - 2 x 1 - x2


-1
(A) It is given that 1/ 2 £ x £ 1. Put Sin x = q so that
p £q£p
p/4 p/2 and sin q = x. Therefore Now
p -p p + 2q = - Sin-1 (2 x 1 - x2 )
£ 2q £ p Þ - p £ - 2q £
2 2
p 2q = - p - Sin-1 (2 x 1 - x2 )
Þ 0 £ p - 2q £
2 Therefore

Also 2 Sin-1 x = - p - Sin-1 (2 x 1 - x2 )


sin(p - 2q) 2 1 - x2 (∵ 0 £ 2x
q = sin2q = 2x 2 1 - x2 £ 1) Answer: (C) Æ (s)
Therefore It is given that -1 £ x £ -1/2. Put Sin-1 x = q.
Therefore x = sin q and
p - 2q = Sin-1 (2 x 1 - x2 ) -p -p
£q <
-1 -1
2 Sin x = p - Sin (2 x 1 - x ) 2 2 6
-3p -p
Answer: (A) Æ (q) £ 3q <
2 2
(B) It is given that 1/2 < x £ 1. Put Sin-1 x = q so that p/6
p <
q<p p/2 and sin q = x. We know that p p
- £ p + 3q <
2 2
sin 3q = 3 sin q - 4 sin3 q = 3x - 4x3
Now
Therefore
sin 3q = 3 sin q - 4 sin3 q = 3 x - 4 x3
p p p 3p
< q £ Þ < 3q £ sin(p + 3q ) = - sin 3q = - (3 x - 4 x3 )
6 2 2 2
-3p -p
Þ £ - 3q <
2 2 p + 3q = - Sin-1 (3 x - 4 x3 )
-p p 3q = - p - Sin-1 (3 x - 4 x3 )
Þ £ p - 3q <
2 2
3 Sin-1 x = - p - Sin-1 (3 x - 4 x3 )
This implies Answer: (D) Æ (r)
sin(p - 3q ) = sin 3q = 3 x - 4 x3
5. II
p - 3q = Sin-1 (3 x - 4 x3 ) their corresponding solutions are given. Match them.
3q = p - Sin-1 (3 x - 4 x3 )
Column I Column II
Hence
3 Sin-1 x = p - Sin-1(3x - 4x3) (A) 2 Tan-1(2x + 1) = Cos-1 x (p) 3
-1 -1
Answer: (B) Æ (p) (B) 3(Tan x) - 4p(Tan
p 2
x) + p = 0 2
(q) 1
-1 -1 -1
(C) It is given that -1 £ x £ - 1/ 2 . Put Sin-1 x = q so that (C) Sin x - Cos x = Sin (3x - 2) (r) 0
-p/2
p £ q £ -p/4 p and sin q = x. Therefore æ 1ö æ 1ö p 1
(D) Tan-1 ç x + ÷ + Tan-1 ç x - ÷= (s)
-p è 2ø è 2ø 4 2
-p £ 2q £
2
Solution:
This implies
(A) Cos-1 x is defined when -1 £ x £ 1. Put Tan-1(2x
2 +
p 1) = q so that tan q = 2x
2 + 1. Therefore
0 £ p + 2q £
2
Cos-1 x = 2q
and so
1 - tan2 q 1 - (2 x + 1)2
sin(p + 2q ) = - sin 2q Þ x = cos 2q = =
1 + tan2 q 1 + (2 x + 1)2
Worked-Out Problems 127

Þ x + x(2 x + 1)2 = 1 - (2 x + 1)2 Therefore

Þ x + 4 x3 + 4 x2 + x = 1 - 4 x2 - 4 x - 1 p
2 Sin-1 x - = Sin-1 (3 x - 2)
2
Þ 4 x3 + 8 x2 + 6 x = 0
æ pö
Þ x(2 x2 + 4 x + 3) = 0 3 x - 2 = sin ç 2q - ÷ (where q = Sin-1 x)
è 2ø
The equation 2x2 + 4x + 3 = 0 has no real solutions. = - cos 2q
Therefore x = 0 satisfies the original equation.
= - (1 - 2 sin2 q )
Answer: (A) Æ (r)
-1
(B) Put Tan x = y. Then the equation transforms to = 2 x2 - 1
3 y2 - 4py + p 2 = 0 Therefore
(3 y - p )( y - p ) = 0 2 x2 - 3 x + 1 = 0
Therefore (2 x - 1)( x - 1) = 0
p Hence x = 1/2, 1 and these values satisfy the original
y= or y=p
3 equation.
This implies Answer: (C) Æ (q), (s)
(D) The given equation can be transformed to
p
Tan-1 x = or Tan-1 x = p
3 æ x + (1/ 2) + x - (1/ 2) ö p
Tan-1 ç =
Since -p/2 -1
p < Tan x < p
p/2, it follows that è 1 - [ x2 - (1/ 4)] ÷ø 4
p 2x
Tan-1 x = =1
3 (5 / 4) - x2
Therefore 4 x2 + 8 x - 5 = 0
p (2 x - 1)(2 x + 5) = 0
x = tan = 3
3
Therefore
Answer: (B) Æ (p)
(C) The given equation can be written as 1 -5
x= or x=
2 2
æp ö
Sin-1 x - ç - Sin-1 x÷ = Sin-1 (3 x - 2) x = 1/2 satisfies the equation and x = -5/2 does not.
è2 ø
Answer: (D) Æ (s)

Comprehension-Type Questions
f x) +
1. Passage: To solve an equation of the form af( (ii) The number of solutions of the equation
bg(x) = ch(x), where f(
f x), g(x) and h(x) are some inverse
3p
trigonometric functions reduce the equation to an alge- Tan-1 (2 x) + Tan-1 (3 x) =
braic equation by calculating a certain trigonometric 4
function of both sides and isolate the extraneous roots
by verification. Answer the following three questions.
(A) 0 (B) 1 (C) 2 (D) 4
(i) If Sin-1 6 x + Sin-1 (6 3 x) = - p / 2, then the value -1 -1
2 ) = 3 Sin x has solutions whose number is
(iii) Sin (2x
of x is
(A) 0 (B) 1 (C) 2 (D) 3
1 -1 1 -1
(A) (B) (C) (D)
12 12 13 13
128 Chapter 2 Inverse Trigonometric Functions

Solution: æ x-yö
Tan-1 x - Tan-1 y = Tan-1 ç + mp
(i) Observe that the equation has meaning if -1/6 3 £ è 1 + xy ÷ø
x £ 1/6 3 . This gives where
-p
Sin-1 6 x = - Sin-1 (6 3 x) ì0 if xy > - 1
2 ï
m = í1 if x > 0, y < 0 and xy < - 1
æp ö ï-1 if x < 0, y > 0 and xy < - 1
Þ 6 x = - sin ç + Sin-1 (6 3 x)÷ î
è2 ø
= - cos(Sin-1 (6 3 x)) Answer the following questions:

= - 1 - 108 x2 æ 4ö
(i) Tan-1 (5) + Tan-1 (3) - Cot-1 ç ÷
è 7ø
Þ 36 x2 = 1 - 108 x2
p 3p p -p
(A) (B) (C) (D)
2 4 4 2

144 x2 = 1 or x=±
1 æ 1ö æ 1ö æ 1 ö
(ii) 3 Tan-1 ç ÷ + Tan-1 ç ÷ + Tan-1 ç equals
12 è 4ø è 20 ø è 1985 ÷ø
So x = -1/12 is a solution and x = 1/12 is not a solu- p 2p p p
(A) (B) (C) (D)
tion because x = 1/12 makes LHS positive and RHS 2 3 3 4
negative.
(iii) The number of solutions of the equation
Answer: (B)
(ii) Put Tan-1(2x) = a, Tan-1(3x) = b. Therefore tan a = æ 8ö
Tan-1 ( x + 1) + Tan-1 ( x - 1) = Tan-1 ç ÷
2x and tan b = 3x. Now è 31ø

3p when - 2 < x < 2 is


-1 = tan = tan (a + b )
4 (A) 1 (B) 2 (C) 3 (D) 0
tan a + tan b 5x
= = Solution:
1 - tan a tan b 1 - 6 x2
(i) x = 5, y = 3 and xy = 15 > 1. Therefore
Therefore
æ 5+3ö
6 x2 - 5 x - 1 = 0 Tan-1 (5) + Tan-1 (3) = p + Tan-1 ç
è 1 - 15 ÷ø
(6 x + 1)( x - 1) = 0
æ 4ö
= p - Tan-1 ç ÷
The solution is x = 1, -1/6. We can verify that è 7ø
x = -1/6 is not a solution. Therefore x = 1 is the only
solution and it satisfies the given equation. æ 4ö
Tan-1 (5) + Tan-1 (3) - Cot-1 ç ÷
è 7ø
Answer: (B)
(iii) Put Sin-1 x = q so that -1 £ x £ 1 and sin q = x. æ 4 4ö
= p - ç Tan-1 + Cot-1 ÷
Therefore è 7 7ø
2 x = sin 3q = 3 sin q - 4 sin3 q = 3 x - 4 x3 p p
=p - =
Þ 4x - x = 0
3 2 2
Answer: (A)
1
Þ x = 0, ±
2 æ 1ö æ 1ö æ 1 ö
(ii) 3 Tan-1 ç ÷ + Tan-1 ç ÷ + Tan-1 ç
è 4ø è 20 ø è 1985 ÷ø
Answer: (D)
æ (3 / 4) - (1/ 64) ö 1
= Tan-1 ç + Tan-1
-1 æ x+ yö
2. Passage: Tan x + Tan y = Tan ç-1 -1
+ lp
p where è 1 - (3 / 16) ÷ø 20
è 1 - xy ÷ø
æ 1 ö æ 1 1 ö
+ Tan-1 ç çè∵ 4 <
ì0 if xy < 1 è 1985 ÷ø ÷

ï
l = í-1 if xy > 1 and x < 0
ï1 if xy > 1 and x > 0
î
Worked-Out Problems 129

æ 47 ö 1 æ 1 ö (A) (x2 + 1)(y2 - 1) = 0 (B) (x2 - 1)(y2 + 1) = 0


= Tan-1 ç ÷ + Tan-1 + Tan-1 ç
è 52 ø 20 è 1985 ÷ø (C) (x2 - 1)(y2 - 1) = 0 (D) x2 + y2 = 2
æ 1 ö (iii) If a = 2, b = 2, then (x, y) satisfies the equation
(47 / 52) +
ç 20 ÷ + Tan-1 æ 1 ö (A) (x2 - 1)(4y2 - 1) = 0
= Tan-1 ç çè ÷
1 - (47 / 1040) ÷ 1985 ø (B) (4x2 - 1)(y2 - 1) = 0
ç ÷
è ø
(C) (4x2 - 1)(y2 - 2) = 0
æ 992 ö æ 1 ö (D) (x2 - 2)(4y2 - 1) = 0
= Tan-1 ç + Tan-1 ç
è 993 ÷ø è 1985 ÷ø
Solution:
æ 1 - (1/ 1985) ö æ 1 ö (i) When a = 1 and b = 0, the given equation reduces to
= Tan-1 ç + Tan-1 ç
÷
è 1 + (1/ 1985) ø è 1985 ÷ø
æ pö
Sin-1 x + Cos-1 y = 0 ç∵ Cos-1 (0) = ÷
æ 1 ö æ 1 ö è 2ø
= Tan-1 (1) - Tan-1 ç ÷ + Tan-1 ç
è 1985 ø è 1985 ÷ø
Therefore
p
=
4 Sin-1 x = Cos-1 y = - Sin-1 1 - y2
Answer: (D)
This gives
(iii) - 2 < x < 2 Þ x - 1 < 1. Therefore
2

x = - 1 - y2 or x2 + y2 = 1
-1 -1 æ 8ö -1
Tan ( x + 1) + Tan ( x - 1) = Tan ç ÷
è 31ø Answer: (A)
(ii) When a = 1, b = 1, the equation is
æ x + 1 + x - 1ö æ 8ö
Þ Tan-1 ç = Tan-1 ç ÷
è 1 - ( x2 - 1) ÷ø è 31ø Sin-1 x + Cos-1 y + Cos-1 xy =
p
2
æ 2x ö æ 8ö
Þ Tan-1 ç = Tan-1 ç ÷
è 2 - x2 ÷ø è 31ø
2x 8 p
Þ = Sin-1 x + Cos-1 y = - Cos-1 ( xy) = Sin-1 xy
2 - x2 31 2
Þ 8 x2 + 62 x - 16 = 0 xy = sin(Sin-1 x + Cos-1 y)

Þ 4 x2 + 31x - 8 = 0 = xy - 1 - x2 1 - y2
Þ 4 x2 + 32 x - x - 8 = 0 ( x2 - 1)( y2 - 1) = 0
Þ ( x + 8)(4 x - 1) = 0 Answer: (C)
(iii) When a = 2, b = 2, the equation becomes
Therefore x = 1/4 [since -8 Ï(- 2 , 2 )] and one
can check that x = 1/4 satisfies the initial equation. p
Sin-1 (2 x) + Cos-1 y + Cos-1 (2 xy) =
Answer: (A) 2
Therefore
3. Passage: Consider the equation
p
p Sin-1 2 x + Cos-1 y = - Cos-1 (2 xy) = Sin-1 (2 xy)
-1 -1 -1 2
Sin (ax) + Cos ( y) + Cos (bxy) =
2
2 xy = sin(Sin-1 2 x + Cos-1 y)
Answer the following questions.
(i) If a = 1 and b = 0, then (x, y) satisfies the equation = 2 xy - 1 - 4 x2 1 - y2
(A) x2 + y2 = 1 (B) x2 + y2 = 2 (1 - 4 x2 )(1 - y2 ) = 0
1 That is
(C) x2 + y2 = (D) y = x
2
(4 x2 - 1)( y2 - 1) = 0
(ii) If a = 1, b = 1, then (x,
( y) is a solution of the equation
Answer: (B)
130 Chapter 2 Inverse Trigonometric Functions

Assertion–Reasoning Type Questions


In the following set of questions, a Statement I is given Solution: Put Cos-1 x = a and Cos-1 y = b so that 0 £ a <
and a corresponding Statement II is given just below it. p/2 and 0 £ b < p
p p/2 and hence 0 £ a + b < p
p. Also
Mark the correct answer as:
cos(a + b ) = cos a cos b - sin a sin b
(A) Both Statements I and II are true and Statement II
is a correct explanation for Statement I = xy - 1 - x2 1 - y2
(B) Both Statements I and II are true but Statement II is
not a correct explanation for Statement I Þ a + b = Cos-1 ( xy - 1 - x2 1 - y2 )
(C) Statement I is true and Statement II is false Therefore Statement II is true. Now
(D) Statement I is false and Statement II is true
æ 3ö æ 12 ö æ 63 ö
Cos-1 ç ÷ + Cos-1 ç ÷ + Cos-1 ç ÷
æ æ 4ö æ 2 ö ö 17 è 5ø è 13 ø è 65 ø
1. Statement I: Tan ç Cos-1 ç ÷ + Tan-1 ç ÷ ÷ =
è è 5ø è 3ø ø 6
æ 3 12 9 144 ö æ 63 ö
= Cos-1 ç × - 1- × 1- ÷ + Cos-1 ç ÷
æ 1- x ö 2 è 5 13 25 169 ø è 65 ø
Statement II: If -1 £ x £ 1, then Cos-1 x = Tan-1ç
è x ÷ø æ 36 4 5 ö æ 63 ö
= Cos-1 ç - × ÷ + Cos-1 ç ÷
è 65 5 13 ø è 65 ø
Solution: Statement II is false, because
æ 16 ö æ 63 ö
æ 1 ö 2p = Cos-1 ç ÷ + Cos-1 ç ÷
-1
Cos ç - ÷ = è 65 ø è 65 ø
è 2ø 3
162 æ 63 ö
whereas = Sin-1 1 - + Cos-1 ç ÷
652 è 65 ø
æ 1 - (1/ 4) ö -p æ 63 ö æ 63 ö
Tan-1 ç -1
÷ = Tan (- 3 ) = = Sin-1 ç ÷ + Cos-1 ç ÷
è ( -1 / 2 ) ø 3 è 65 ø è 65 ø
p
But, =
2
æ 4 2ö
tan ç Cos-1 + Tan-1 ÷ Answer: (A)
è 5 3ø
æ 3 2ö -1 æ 4ö-1 æ 5ö
-1 æ 16 ö p
= tan ç Tan-1 + Tan-1 ÷ 3. Statement I: Sin ç ÷ + Sin ç ÷ + Sin ç ÷ =
è 4 3ø è 5ø è 13 ø è 65 ø 4
= tan(a + b ) Statement II: If x > 0, y > 0 and x2 + y2 < 1, then Sin-1 x +
where Sin-1 y = Sin-1 ( x 1 - y2 + y 1 - x2 )

3 2 Solution: Put Sin-1 x = a and Sin-1 y = b. Therefore


a = Tan-1 and b = Tan-1
4 3 p p
0<a < , 0<b <
2 2

tan a + tan b (3 / 4) + (2 / 3) 17
tan(a + b ) = = =
1 - tan a tan b 1 - (3 / 4) × (2 / 3) 6 p
x2 + y2 < 1 Þ 0 < a + b <
Answer: (C) 2
sin(a + b ) = sin a cos b + cos a sin b
æ 3ö æ 12 ö -1 æ 63 ö p
2. Statement I: Cos-1 ç ÷ + Cos-1 ç ÷ + Cos çè ÷ø = = x 1 - y2 + y 1 - x2
è ø 5 è 13 ø 65 2
p
Statement II: If 0 < x, y £ 1, then Cos-1 x + Cos-1 y = 0<a + b < Þa + b
2
Cos-1 ( xy - 1 - x2 × 1 - y2 )
= Sin-1 ( x 1 - y2 + y 1 - x2 )
Therefore Statement II is true.
Worked-Out Problems 131

Now Now x > y > z > 0 Þ x - y > 0, y - z > 0 and z - x < 0.


Therefore
æ 4ö æ 5ö æ 16 ö
Sin-1 ç ÷ + Sin-1 ç ÷ + Sin-1 ç ÷
è 5ø è 13 ø è 65 ø æ xy + 1ö æ yz + 1ö æ zx + 1ö
Cot-1 ç + Cot-1 ç + Cot-1 ç
æ4 è x - y ÷ø è y - z ÷ø è z - x ÷ø
æ 4ö ö
2 2
æ 5ö 5 æ 16 ö
= Sin-1 ç × 1 - ç ÷ + 1 - ç ÷ ÷ + Sin-1 ç ÷
çè 5 è 13 ø 13 è 5 ø ÷ø è 65 ø æ x-yö æ y-zö æ z- x ö
= Tan-1 ç ÷ + Tan-1 ç ÷ + p + Tan-1 ç
è xy + 1ø è yz + 1ø è zx + 1÷ø
æ 4 12 5 3 ö æ 16 ö
= Sin-1 ç × + × ÷ + Sin-1 ç ÷ = (Tan-1 x - Tan-1 y) + (Tan-1 y - Tan-1z) +
è 5 13 13 5 ø è 65 ø
æ 63 ö æ 16 ö (Tan-1z - Tan-1 x) + p
= Sin-1 ç ÷ + Sin-1 ç ÷
è 65 ø è 65 ø =p
æ 16 ö æ 16 ö
= Cos-1 ç ÷ + Sin-1 ç ÷ Therefore Statement I is also true and Statement II is a
è 65 ø è 65 ø correct explanation of Statement I.
p Answer: (A)
=
2
Hence Statement I is false. 5. Statement I: For x > 14, if Sin-1(14/x
/ ) + Sin-1(2 15/x
/ )
=p
p/2, then x = 16
Answer: (D)
Statement II: If 0 < x £ 1, then Cos-1 x = Sin-1 1 - x2
4. Statement I: If x > y > z > 0, then
Solution: Statement II is obviously true.
æ xy + 1ö æ yz + 1ö æ zx + 1ö æ 2 15 ö
Cot-1 ç + Cot-1 ç + Cot-1 ç =p æ 14 ö p
è x-yø ÷ è y-zø ÷ è z - x ÷ø Sin-1 ç ÷ = - Sin-1 ç
è xø 2 ÷
è x ø
-1
æ 1 ö ìïCot x, if x > 0 æ 2 15 ö
Statement II: Tan-1 ç ÷ = í = Cos-1 ç ÷
è x ø ï-p + Cot-1 x, if x < 0 è x ø
î
Solution: Let Cot-1 x = q so that 0 < q < p and Cot q = x. 60
= Sin-1 1 -
Therefore x2
1 Therefore
Tan q =
x
p 14 x2 - 60
ì =
æ 1ö ïïq , if 0 < q < 2 x x
Tan-1 ç ÷ =í
è xø ï-p + q if p < q < p Þ x2 - 60 = 196
ïî 2 Þ x2 = 256
ì p
-1 -1
ïïCot x, if 0 < Cot x < 2 Þ x = 16
=í Therefore Statements I and II are true and Statement II
ï-p + Cot-1 x, if p < Cot-1 x < p is a correct explanation of Statement I.
ïî 2
Answer: (A)
ìïCot-1 x, if x > 0

-1
îï-p + Cot x, if x < 0

Integer Answer Type Questions


1. If cos(2 Sin-1 x) = 1/9, then the value of 3|x| is . Now,
-1
Solution: Put Sin x = q so that sin q = x. 1
= cos 2q = 1 - 2 Sin2 q = 1 - 2 x2
9
132 Chapter 2 Inverse Trigonometric Functions

Therefore 4. If x > 1, then the value of

1 8 æ1 æ 2x ö ö
2 x2 = 1 - = -sin ç Tan-1 ç - Tan-1 x÷
9 9 2÷
è2 è1- x ø ø
4
x2 =
9 is .
2 Solution:
x=±
3
æ1 æ 2x ö ö
or 3 | x| = 2 - sinç Tan-1 ç 2÷
- Tan-1 x÷
è2 è1- x ø ø
Answer: 2
æ1 ö
2. 1 + cos[Cos-1 (- 3 / 2) + (p / 6)] is equal to
= - sin ç (-p + 2 Tan-1 x) - Tan-1 x÷
. è2 ø
Solution: æ -p ö
= - sin ç =1
æ è 2 ÷ø
æ - 3ö p ö æ 5p p ö
1 + cos ç Cos-1 ç ÷ + ÷ = 1 + cos ç + ÷
è è 2 ø 6ø è 6 6ø Answer: 1

= 1 + cos (p ) = 0 5. 2p + Tan-1(tan 5) is .
Answer: 0 Solution: 5 does not lie between -p /2 and p /2. But

3. Number of values of x satisfying the equation Sec-1 x + æ 44 ö


5 - 2p = 5 - ç ÷ = 5 - (6.28) = -(1.28)
Sin-1 x = p
p/2 is . è 7ø
Solution: Sec-1 x is defined for |x| ³ 1 whereas Sin-1 x is lies between -p /2 and p /2. Also
defined for |x| £ 1. Therefore Sec-1 x + Sin-1 x is defined
when |x| = 1 in which case their sum is p/2.
p Tan-1(tan 5) = Tan-1(tan(5 - 2p))
p = 5 - 2p
Answer: 2 Therefore
2p + Tan-1(tan 5) = 5
Answer: 5

SUMMARY
Inverse Trigonometric Functions -p p
and g(sin x) = x for all £ x £ . This function
-1
2.1 Definition of Sin x or arcsin x: The function f : 2 2
é p pù
é p pù g : [-1, 1] ® ê - , ú
êë - 2 , 2 úû ® [-1, 1] defined by f(
f x) = sin x is a bijection ë 2 2û
and hence it has unique inverse function g :[-1, 1] ® function and is noted by Sin-1 x or arcsin x.
é p pù
(f(y)) = y for all -1 £ y £ 1
êë - 2 , 2 úû such that sin(f

QUICK LOOK

p p
1. For -1 £ x £ 1, and - £ y £ , Sin-1 x = y Û sin y = x. 4. Sin-1 x = 0 Û x = 0.
2 2
-1 é p ù é pù
2. Domain of Sin x is the closed interval [-1, 1] and 5. Sin-1 x Î ê - , 0 ú Û y Î [-1, 0] and Sin-1 x Î ê0, ú Û
é p pù ë 2 û ë 2û
range is the closed interval ê - , ú. x Î [0, 1].
ë 2 2û
3. sin(Sin-1 x) = x for all x Î [-1, 1] and Sin-1(sin y) = y
é p pù
for all y Î ê - , ú.
ë 2 2û
Summary 133

2.2 Table showing the domains and ranges of inverse 2.4 Useful Formulae:
trigonometric functions
ìp
if x > 0
-1 æ 1ö ï 2
-1
(1) Tan x + Tan ç ÷ = í
Function Domain Range è xø ï p
- if x < 0
î 2
é p pù
Sin-1 x [-1, 1] êë - 2 , 2 úû (2) (a) If x and y are positive real numbers, then

Cos-1 x [-1, 1] [0, p ] ì -1 æ x + y ö


ïTan ç if xy < 1
ï è 1 - xy ÷ø
é p pù
Tan-1 x  êë - 2 , 2 úû ï
ï æ x+ yö
Tan-1 x + Tan-1 y = íp + Tan-1 ç if xy > 1
Cot-1 x  [0, p ] ï è 1 - xy ÷ø
ï
Cosec-1 x (-¥, -1] È [1, ¥)
é p ö æ pù ïp if xy = 1
êë - 2 , 0÷ø È çè 0, 2 úû ï2
î
é pö æp ù
Sec-1 x (-¥, -1] È [1, ¥) êë0, 2 ÷ø È çè 2 , p úû
(b) If x and y are negative real numbers, then

ì -1 æ x + y ö
ïTan ç if xy < 1
2.3 Fundamental Identities: ï è 1 - xy ÷ø
p ï
(1) Sin-1 x + Cos-1 x = for all -1 £ x £ 1 ï æ x+ yö
Tan-1 x + Tan-1 y = í-p + Tan-1 ç if xy > 1
è 1 - xy ø÷
2
ï
p ï
(2) Tan-1 x + Cot-1 x = for all real x
2 ï- p if xy = 1
ï 2
p î
(3) Cosec-1 x + Sec-1 x = for all x Î(-¥, -1] È [1, ¥)
2
2.5 (a) If x and y are positive reals, then
(4) sin(Cos-1 x) = 1 - x2 and cos(S in-1 x) = 1 - x2
æ x-yö
x 1 Tan-1 x - Tan-1 y = Tan-1 ç
sin(Tan-1 x) = and cos(Tan-1 x) = è 1 + xy ÷ø
1 + x2 1 + x2
(b) Let x, y be reals and xy ¹ -1. Then
-1 x
(6) tan(Sin x) = for - 1 < x < 1 Tan-1 x - Tan-1 y
1 - x2
ì -1 æ x - y ö
1 - x2 ïTan ç if xy > - 1
è 1 + xy ÷ø
-1
(7) tan(Cos x) = for 0 ¹ x Î[-1, 1]
x ï
ï
1 ï æ x-yö
(8) tan(Cot-1 x) = for x ¹ 0 = íp + Tan-1 ç if x > 0, y < 0 and xy < - 1
x ï è 1 + xy ÷ø
1 ï
(9) sin(Cot-1 x) = ï-p + Tan-1 æ x - y ö
1 + x2 ï çè 1 + xy ÷ø if x < 0, y > 0 and xy < - 1
î
x
(10) cos(Cot-1 x) =
1 + x2 2.6 If x ³ 0, y ³ 0 and x2 + y2 £ 1, then

(11) Sin-1 x = Cos-1 ( 1 - x2 ) for 0 £ x £ 1 Sin-1 x + Sin-1 y = Sin-1( x 1 - y2 + y 1 - x2 )


x 1 1
(12) Sin-1 = Cos-1 = Cot-1 = Tan-1 x 2.7 -1 £ x, y £ 1, xy < 0 and x2 + y2 > 1, then
1+ x 2
1+ x 2 x
for all x > 0 Sin-1 x + Sin-1 y = Sin-1( x 1 - y2 + y 1 - x2 )
(13) Sin-1(-x
- ) = -Sin-1 x and Cos-1(-x
- ) = p - Cos-1 x
for -1 £ x £ 1 2.8 -1 £ x, y < 0 and x2 + y2 > 1, then
(14) Tan-1(-x
- ) = -Tan-1 x
Sin-1 x + Sin-1 y = - p - Sin-1( x 1 - y2 + y 1 - x2 )
134 Chapter 2 Inverse Trigonometric Functions

2.9 If -1 £ x, y £ 1 and x2 + y2 £ 1, then ìïCos-1 (2 x2 - 1), if 0 £ x £ 1


2.19 2 Cos-1 x = í
Sin-1 x - Sin-1 y = Sin-1( x 1 - y2 - y 1 - x2 ) -1
îï2p - Cos (2 x - 1), if - 1 £ x £ 0
2

2.10 < x, y £ 1 and x2 + y2 > 1, then


ì -1 1
ïCos (4 x - 3 x), if 2 £ x £ 1
3
-1 -1 -1
Sin x - Sin y = Sin ( x 1 - y - y 1 - x )
2 2
ï
ï 1 1
2.20 3 Cos x = í 2p - Cos-1 (4 x3 - 3 x), if - < x £
-1
2.11 < x £ 1, -1 £ y < 0 and x2 + y2 > 1, then ï 2 2
ï 1
Sin-1 x - Sin-1 y = p - Sin-1( x 1 - y2 - y 1 - x2 ) -1
ï2p + Cos (4 x - 3 x), if - 1 £ x £ -
3

î 2
2.12 If -1 £ x < 0, 0 < y £ 1 and x2 + y2 > 1, then
ì -1 2x
-1 -1 -1
Sin x - Sin y = -p - Sin ( x 1 - y - y 1 - x )
2 2 ïTan 1 - x2 , if - 1 < x < 1
ï
ï 2x
ï -1
2.13 If -1 £ x, y £ 1 and x + y ³ 0, then 2.21 2 Tan-1 x = íp + Tan , if x > 1
1 + x2
ï
Cos-1 x + Cos-1 y = Cos-1( xy - 1 - x2 1 - y2 ) ï æ ö
ï-p + Tan-1 2 x 2 , if x < - 1
ïî ç
è 1 - x ÷ø
2.14 -1 £ x, y £ 1 and x + y < 0, then

Cos-1 x + Cos-1 y = 2p - Cos-1( xy - 1 - x2 1 - y2 ) ì -1 2 x


ïSin 1 + x2 , if - 1 £ x £ 1
ï
2.15 If -1 £ x, y £ 1 and x £ y, then ï
ï -1 æ 2x ö
2.22 2 Tan-1 x = íp - Sin ç , if x > 1
Cos-1 x - Cos-1 y = Cos-1( xy + 1 - x2 1 - y2 ) è 1 + x2 ÷ø
ï
ï
2.16 -1 £ y £ 0, 0 < x £ 1, then ï-p - Sin-1 æ 2 x ö , if x < - 1
ïî çè 1 + x2 ÷ø
Cos-1 x - Cos-1 y = - Cos-1 ( xy - 1 - x2 1 - y2 )
ì -1 æ 1 - x ö
2

ì -1 ïCos ç , if x ³ 0
è 1 + x2 ÷ø
1 1
ïSin (2 x 1 - x ), if - 2 £ x £ 2
2
-1 ï
2.23 2 Tan x = í
ï
ï -1 æ 1 - x ö
2
ï - çè 1 + x2 ÷ø , if x £ 0
1 ï Cos
2.17 2 Sin x = íp - Sin-1(2 x 1 - x2 ), if
-1
£x£1
2 î
ï
ï 1
ï-p - Sin-1(2 x 1 - x2 ), if - 1 £ x £ - ì -1 æ 3 x - x ö -1
3
1
î 2 ïTan ç 2 ÷
, if £x£
ï è 1 - 3 x ø 3 3
ï
ì -1 1 1 ï -1 æ 3 x - x ö
3
1
ïSin (3 x - 4 x ), if - 2 £ x £ 2
3 -1
2.24 3 Tan x = íp + Tan ç 2 ÷
, if x ³
ï ï è 1 - 3 x ø 3
ï 1 ï
2.18 3 Sin x = íp - Sin-1 (3 x - 4 x3 ), if < x £ 1
-1
ï-p + Tan-1 æ 3 x - x ö , if x £ - 1
3

ï 2 ï çè 1 - 3 x2 ÷ø
ï î 3
-1 1 1
ï-p - Sin (3 x - 4 x ), if - £ x < -
3

î 2 2
Exercises 135

EXERCISES
Single Correct Choice Type Questions
1. Sin-1[sin(33p /7)] is equal to -1
11. (1/7) + Tan-1(1/3) is equal to
33p p -p 2p p æ 1ö p
(A) (B) (C) (D) (A) + Tan-1 ç ÷ (B)
7 7 7 7 4 è 3ø 4
p æ 1ö æ 2ö
2. Tan-1[tan(2p /3)] is equal to (C) - Tan-1 ç ÷ (D) Tan-1 ç ÷
4 è 3ø è 3ø
2p p -p -2p
(A) (B) (C) (D)
3 3 3 3 12. If Tan-1 x = 2 Tan-1(2/3), then x is equal to
7 9 11 12
3. If p £ Tan-1[(1 - x)/(1 + x)] £ q for all 0 £ x £ 1, then (A) (B) (C) (D)
5 5 5 5
p -p
(A) p = 0, q = (B) p = ,q=0
4 4 13. If Sin-1 x + Sin-1 y + Sin-1 z = 3p / 2, then the value of
p p -p sin[(p / 2)(x + y + z)] is
(C) p = , q = (D) p = ,q=0
4 2 2 1
(A) 0 (B) -1 (C) 1 (D)
2
4. The value of Cos-1(cos10) is
(A) 4p - 10 (B) 2p - 10 (C) 2p + 10 (D) 10 -1 -1
p and
14. Suppose that Cos x + Cos y + Cos z = 3p
-1

a = sin[p(
p x + y + z)] + icos[p(
p x + y + z)], then Arg a is
5. Tan-1(1/2) + Tan-1(1/3) is equal to
p 3p 3p -p
p p p p (A) (B) (C) (D)
(A) (B) (C) (D) 2 4 2 2
2 4 3 6
15. Sin-1(sin 5) is equal to
-1 -1 -1
6. 2 Tan (1/5) + Sec (5 2 /7) + 2 Tan (1/8) =
(A) 5 (B) 5 - 2p
p (C) 2p - 5 (D) p - 5
p p -p -p
(A) (B) (C) (D)
2 4 2 4
å
n
16. If uk = Cosec-1 (k 2 + 1)(k 2 + 2k + 2), then k =1 k
u =
-1 -1
7. Tan (1/4) + Tan (2/9) = p p
(A) Tan-1 (n + 1) - (B) Tan-1 (n + 1) +
p p p 1 æ 3ö 4 4
(A) (B) (C) (D) Cos-1 ç ÷
3 4 6 2 è 5ø p p
(C) Cot-1 (n + 1) - (D) Cot-1 (n + 1) +
4 4
8. If x = ( 2 - 3 )/ 2, y = 12 / 4 and z = 2,, then the
value of Sin-1{cot(Sin-1 x + Cos-1 y + Sec-1 z)} is 17.
p p p
(A) 0 (B) (C) (D) (Sin-1 x)3 + (Cos-1 x)3
4 3 2 =7
(Tan-1 x + Cot-1 x)3
9. Tan-1(1/21) + Tan-1(1/13) + Tan-1(-1/8) = then the value (s) of x is (are)
p p p
(A) (B) (C) (D) 0 1 1
3 4 2 (A) -1 (B) 1 (C) (D) -
2 2

æ 1 ö
å
n
10. Tan-1 ç 2 ÷ is equal to
18. The value of x satisfying the equation
k =1 è1+ k + k ø
æ x + 1ö æ x - 1ö
p p Tan-1 ç + Tan-1 ç = p + Tan-1 (-7)
(A) Tan-1 (n + 1) + (B) Tan-1 (n + 1) - è x - 1 ÷ø è x ÷ø
4 4
-1 p -1 æ 1 ö p is
(C) Tan (n + 2) + (D) Tan ç +
4 è n + 1÷ø 4 -1 1
(A) -2 (B) (C) 2 (D)
2 2
136 Chapter 2 Inverse Trigonometric Functions

19. Domain of the function 22. If 0 < x < 1, then

p æ 1 - x2 ö æ 1 - x2 ö
f ( x) = Sin-1 (2 x) + Tan-1 ç ÷ + Cos-1 ç =
6 è 2x ø è 1 + x2 ÷ø
is -p p
(A) 0 (B) (C) (D) p
é -1 1 ù é -1 1 ù é -1 1 ù é -1 1 ù 2 2
(A) ê , ú (B) ê , ú (C) ê , ú (D) ê , ú
ë 4 2û ë 2 4û ë 2 2û ë 4 4û
23. If
20.
æ 1 + xö æ 1 - x2 ö
æ ö a = 2 Tan-1 ç b = Sin-1 ç
Sin-1 ç x -
x2 x3 x4
+ - +  + ¥÷ è 1 - x ÷ø è 1 + x2 ÷ø
è 2 4 8 ø
where 0 < x < 1, then a + b is equal to
æ x 4 x6 x8 ö p
+ Cos-1 ç x2 - + - +  + ¥÷ = p
è 2 4 8 ø 2 p
(A) 2p (B) p (C) 0 (D)
2
for 0 < | x| < 2 , then x is equal to
1 -1 æ k - k - 1ö
å
n
(A) (B) 1 (C) (D) -1 24. k =1
Sin-1 ç ÷=
2 2 è k(k + 1) ø
p p
21. The equation 2 Tan-1 x = Sin-1[4x/(1 + 4x2)] is valid if (A) Tan-1 n + (B) Tan-1 n + 1 +
x belongs to the interval 4 4
p
æ1 ö æ -1ö (C) Tan-1 n- (D) Tan-1 n
(A) ç , ¥÷ (B) ç -¥, ÷ 4
è2 ø è 2ø

é -1 1 ù
(C) ê , ú (D) [-1, 1]
ë 2 2û

Multiple Correct Choice Type Questions


1. Which of the following are integer pair solutions of f x) = Sin-1 x and g(x) = Cos-1 x, then
5. If f(
-1 -1 -1
/ ) = Tan (3)?
the equation Tan x + Tan (1/y 1
f x) > g(x) if
(A) f( <x£1
(A) (1, 2) (B) (4, -13) (C) (5, -8) (D) (2, 7) 2

2. Let y = (p /2) + Cos-1(cos x). Then the values of y such


1
f x) £ g(x) if - 1 £ x £
(B) f(
that y = | tan x |(0 £ x £ 2p)
p are 2
p p 5p 3p (C) g(x) ³ f(
1
£x£1
(A) x + (B) x + (C) - x + (D) - x + f x) if
4 2 2 2 2
1
3. If x and y are real numbers such that Tan-1 y = 4Tan
T -1 x, (D) g(x) £ f(
f x) if - 1 £ x £
2
then
4 x - 4 x3 4 x + x3 6. Which of the following statements are true?
(A) y = (B) y =
1 - 6 x2 + x4 1 - 6 x2 + x4 (A) x = 1 is a solution of the equation Cos-1(1 - x) +
1° 4 x - x3 Sin-1 x = p
(C) x ¹ Tan 22 (D) y =
2 1 + 6 x2 - x4 (B) If Cos-1 x + Cos-1 y = p
p/3, then x2 - xy + y2 = 3/4
(C) Tan-1(4/3) = Tan-1(1/7) + (p/4)
p
4. If Sin-1 x + Sin-1 y + Sin-1 z = 3p /2, then
x æ a - xö
(A) x + y + z = xyz (D) For 0 < x < a, 2 Tan-1 = Cos-1 ç
(B) x + y + z = 3xyz
a è a + x ÷ø
+ +z
(C) 1 + w x+y = 2, where w ¹ 1 is a cube root of unity
(D) w x + w 2y
2
+ w 3z = 0, where w is a nonreal root of
x -1=0
3
Exercises 137

Matrix-Match Type Questions


In each of the following questions, statements are given 3. Match the items in Column I with those in Column II
in two columns, which have to be matched. The state-
ments in Column I are labeled as (A), (B), (C) and
Column I Column II
(D), while those in Column III are labeled as (p), (q),
(r), (s) and (t). Any given statement in Column I can æp 3- 5
æ 1ö ö
have correct matching with one or more statements in (A) sin ç - Sin-1 ç ÷ ÷ = (p)
è3 è 2øø 2
Column II. The appropriate bubbles corresponding to
-24
the answers to these questions have to be darkened as æ æ 3ö p ö (q)
illustrated in the following example. (B) cos ç Cos-1 ç ÷+ ÷= 7
è è 2 ø 6ø 1
Example: If the correct matches are (A) ® (p), (s); (B) ® (r)
æ1 5ö 2
(q), (s), (t); (C) ® (r); (D) ® (r), (t); that is if the matches (C) tan ç Cos-1 =
2 ÷ø
are (A) ® (p) and (s); (B) ® (q), (s) and (t); (C) ® (r); and 24
è2 (s)
(D) ® (r), (t) then the correct darkening of bubbles will 7
æ 4ö 1
look as follows: (D) - tanç 2 Sin-1 ÷= (t) -
è 5ø 2
p q r s t
A
4. Match the items in Column I with those in Column II.
B
C
Column I Column II
D
- 3 ± 17
(A) If Cot-1 x + Cot-1 2x = 3p/4,
p then (p)
1. Match the items in Column I with those in Column II. x= 4
(q) 0
Column I Column II (B) If Tan-1 x = Cot-1 x, then x = (r) 1
p (C) If Cos-1 x - Sin-1 x = Cos-1 ( 3 x), 1
(A) Sin-1(1/2) + Cos-1(-1/2) is equal to (p) (s)
6 then x = 2
5p 1
(q) (D) If Tan-1 x + Tan-1(1 - x) = (t) -
(B) Tan-1 3 + Cot-1 (- 3 ) is equal to 6 2
p 2 Tan-1 x - x2 , then x =
(r)
3
(C) Sin-1(-1/2) + Cos-1(1/2) is equal to 7p 5. Match the items in Column I with those in Column II.
(s)
6
2p
(D) Cos-1[cos(7p/6)]
p + Sin-1[sin(2p/3)]
p = (t) Column I Column II
3
(A) If f1(x) = Cos-1 x + Sin-1 x + é 3p 5p ù
(p) ê ,
2. Match the items in Column I with those in Column II
Cot-1 x, then f1(x) belongs to ë 4 4 úû
(B) If f2(x) = Tan-1 x + Cot-1 x +
Cosec-1 x, then f2(x) belongs (q) [0, p ] - {p / 2}
Column I Column II
to
p é p 3p ù
(A) If Sin-1 x - Cos-1 x = , then x = (C) Range of the function f3(x) = (r) ê ,
ë 2 2 úû
(p) 6
6 Cot-1 x + Tan-1 x + Cos-1 x
5
(B) If Tan-1 x + Tan-1 3 = Tan-1 8, then x = (q) 3 lies in the interval
é pù
(s) ê0, ú
1 (D) The range of f4(x) = ë 2û
(C) If Tan(Sec-1 x) = Sin(Tan-1 2) and (r) Cot-1 x + Tan-1 x + Sin-1 x
x > 0, then x is equal to 5
lies in (t) [0, p]
p
3
(s)
2
(D) If Tan-1(4/x/ ) + Tan-1(9/x
/ ) = p/2,
p 1
then x = (t)
3
138 Chapter 2 Inverse Trigonometric Functions

Comprehension-Type Questions
1. Passage: If x > 0 and y > 0 (i) Sum to n terms of the series
x x x
æ x-yö Tan-1 + Tan-1 + Tan-1 +
Tan-1 x - Tan-1 y = Tan-1 ç 1 + 2x 2
1 + 6x 2
1 + 12 x2
è 1 + xy ÷ø
is
Answer the following questions.
(A) Tan-1(n + 1) - Tan-1 x
(i) Tan-1 2 - Tan-1 x = p
p/4 implies that the value of x is
(B) Tan-1nx - Tan-1 x
1 1
(A) 2 (B) (C) 2 (D) æ 1 ö
2 3 (C) Tan-1 ç - Tan-1 x
è n + 1÷ø
(ii) If x and y are positive, then
Tan-1(n(n + 1)x) - Tan-1 x
æ xö æ y - xö
Tan-1 ç ÷ + Tan-1 ç (ii) Sum of the first 2010 terms of the series
è yø è y + x ÷ø
x x
is equal to Tan-1 x + Tan-1 + Tan-1
1 + 1× 2 x2 1 + 2 × 3 x2
p -p p -p
(A) (B) (C) (D) x
4 4 3 3 + Tan-1 +
(iii) If 1 + 3 × 4 x2
is
æ 1 ö -1 æ 1 ö -1 æ 1 ö
3 Tan-1 ç ÷ - Tan çè x ÷ø = Tan çè 3 ÷ø (A) Tan-1(2011x) - Tan-1 x
è 2 + 3ø
(B) Tan-1(2010x) - Tan-1 x
then x is equal to (C) Tan-1(2010x)
1 (D) Tan-1(2009x)
(A) 2 (B) (C) 1 (D) -1
2 (iii) If the sum of the first n terms of the series

If x > 0 and y > 0 4 6 8


2. Passage: Tan-1 + Tan-1 + Tan-1 +
13 73 241
æ x-yö
Tan-1 x - Tan-1 y = Tan-1 ç is Tan-1((n + 1)(n + 2)) - Tan-1 x, then x is equal to
è 1 + xy ÷ø
(A) 1 (B) -2 (C) -1 ( D) 2
Answer the following questions.

Assertion–Reasoning Type Questions


In the following set of questions, a Statement I is given 1 - tan2q
and a corresponding Statement II is given just below it. Statement II: cos 2q =
1 + tan2q
Mark the correct answer as: 2 tan q
(A) Both Statements I and II are true and Statement II sin 2q =
1 + tan2q
is a correct explanation for Statement I
(B) Both Statements I and II are true but Statement II is 2. Statement I: If b = Cot-1 cos a - Tan-1 cos a , then
not a correct explanation for Statement I sin b = Tan2(a/2)
a
(C) Statement I is true and Statement II is false Statement II: Cot-1 x = Tan-1 (1/x
/ ) for any x > 0
(D) Statement I is false and Statement II is true
3. Statement I: If Tan-1 x + Tan-1 y + Tan-1 z = p/2
p and
-1 -1
1. Statement I: Ifa = Tan (1/7) and b = Tan (1/3),then x + y + z = 3, then x = y = z
cos 2a = sin 4b.
æ x+ yö
Statement II: Tan-1 x + Tan-1 y = Tan-1 ç ,
if xy < 1 è 1 - xy ÷ø
Answers 139

æ 2x ö æ 3ö æ 8ö æ 77 ö
4. Statement I: Tan-1 x + Sin-1 ç = p for x > 1 5. Statement I: Sin-1 ç ÷ + Sin-1 ç ÷ = Sin-1 ç ÷
è 1 + x2 ÷ø è 5ø è 17 ø è 85 ø

p £ Sin-1 x £ p/2
Statement II: For -1 £ x £ 1, -p/2 p Statement II: For 0 £ x £ 1, Sin-1 x = Cos-1 1 - x2 and
Cos-1 x = Sin-1 1 - x2

Integer Answer Type Questions


The answer to each of the questions in this section is a 3. If 2 Tan-1(1/2) + Cos-1(4/5) = p/
p K, then k is equal to
non-negative integer. The appropriate bubbles below .
the respective question numbers have to be darkened.
-1 -1 -1
For example, as shown in the figure, if the correct answer 4. If 2 Tan (1/5) + Tan (1/4) = Tan k, then the inte-
to the question number Y is 246, then the bubbles under gral part of k is .
Y labeled as 2, 4, 6 are to be darkened.
5. Cot-1(cot4) = k - p
p, where k is equal to .
X Y Z W
0 0 0 0 6. If Sin-1(4/5) + Sin-1(3/5) = 2 Tan-1 k, then k is equal to
1 1 1 1
.
2 2 2
15 é æ -1 æ 1 ö ö -1 ù
3 3 3 3 7. êsin çè 2 Tan çè ÷ø ÷ø + cos(Tan (2 2 ))ú = .
2 ë 3 û
4 4 4
5 5 5 5
1 é æ 4ö æ 2ö ù
6 6 6 8. + Tan êCos-1 ç ÷ + Tan-1 ç ÷ ú = .
6 ë è 5ø è 3ø û
7 7 7 7
8 8 8 8
9. If x satisfies the equation Tan-1(x + 3) - Tan-1(x - 3) =
9 9 9 9 Sin-1(3/5), then |x| is .

1. 4 Tan-1(1/5) - Tan-1(1/239) = p/
p K, where k is equal to 10. If Sin-1 x + Sin-1 y + Sin-1 z = p
p, then x4 + y4 + z4 +
2 2 2 2 2 2 2 2 2
. 4 y z = k(x y + y z + z x ), where k is equal to
4x .

2. Sin-1 (1/ 2) + Cos-1 (1/ 2 ) = kp / 12, where k is equal to


.

ANSWERS
Single Correct Choice Type Questions
1. (D) 13. (B)
2. (C) 14. (D)
3. (A) 15. (B)
4. (A) 16. (A)
5. (B) 17. (A)
6. (B) 18. (C)
7. (D) 19. (A)
8. (A) 20. (B)
9. (D) 21. (D)
10. (B) 22. (C)
11. (C) 23. (B)
12. (D) 24. (D)
140 Chapter 2 Inverse Trigonometric Functions

Multiple Correct Choice Type Questions


1. (A), (B), (C), (D) 4. (B), (C), (D)
2. (B), (C) 5. (A), (B)
3. (A), (C) 6. (A), (B), (C), (D)

Matrix-Match Type Questions


1. (A) ® (q), (B) ® (s), (C) ® (p), (D) ® (s) 4. (A) ® (p), (B) ® (r), (C) ® (q), (s), (t), (D) ® (s)
2. (A) ® (s), (B) ® (r), (C) ® (q), (D) ® (p) 5. (A) ® (p), (B) ® (q), (t), (C) ® (r), (D) ® (t)
3. (A) ® (r), (B) ® (r), (C) ® (p), (D) ® (q)

Comprehension-Type Questions
1. (i) (D); (ii) (A); (iii) (A) 2. (i) (A); (ii) (C); (iii) (D)

Assertion–Reasoning Type Questions


1. (A) 4. (A)
2. (A) 5. (A)
3. (A)

Integer Answer Type Questions


1. 4 6. 1
2. 5 7. 7
3. 2 8. 3
4. 0 9. 4
5. 4 10. 2
Trigonometric
Equations 3
Contents
3.1 Least Positive
Angles with a Given
Trigonometric Ratio
3.2 General Solution of
Equations of the Form
sin x = a
3.3 Solutions of Simple
Trigonometric
Equations
Trigonometric Equations

Worked-Out Problems
W
Summary
Exercises
Answers

A trigonometric equation
is one that involves one or
more of the six functions
sine, cosine, tangent, cotan-
gent, secant, and cosecant.
142 Chapter 3 Trigonometric Equations

Consider a polynomial equation of the form anxn + an-1xn-1 +  + a1x + a0 = 0, where a0, a1, …, an are real numbers.
If a is a real number such that

an an + an-1an-1 +  + a1a + a0 = 0

then a is called a roott or a solution of the equation anxn + an-1xn-1 +  + a1x + a0 = 0. If atleast one ai is nonzero, then
the equation has at most n roots. Note that if all the ai’s are zero, then every real number is a root of this equation. If
we consider a quadratic equation, then we know from Chapter 4 of Vol. 1 that the equation has at most two roots. In
trigonometry, we come across equations such as
sin q = 1/ 2
a cos q + b sin q = 1
a cos2 q + b cosq + c = 0, etc
where a, b, c are given real numbers. Equations of this type are called trigonometric equations and an angle q satisfying
a trigonometric equation is called a roott or solution of this equation.
Unlike polynomial equations, any trigonometric equation has infinite number of solutions, if at all there is a solu-
tion. The trivial reason is that each trigonometric function is periodic and 2pp is a period. Therefore, if q is a solution of
a given trigonometric equation, then q + 2np p is also a solution of the same equation for every integer n. In this chapter
we discuss trigonometric equations and their solutions.

DEFINITION 3.1 p are called principal


The solutions of a trigonometric equation lying in the interval [0, 2p)
solutions.

Example

p/6 and 5p
p p/6 are the principal solutions of the equation sin x = 1/2.

3.1 | The Least Positive Angles with a Given Trigonometric Ratio


For any trigonometric function f and for any real number a, we explore the possibility of the existence of a real
number x such that f(f x) = a. If f is a sine function or a cosine function and a is a real number such that |a| > 1, then
clearly there is no real number x for which f( f x) = a, since the range of sine or cosine function is the interval [-1, 1]
and hence any real number outside this interval is neither sin x nor cos x for any x Î. However, if |a| £ 1, we have
infinite number of real numbers x for which sin x = a. Among these, there must be least positive x (recall that each
trigonometric function is having a minimum period). In the following, we construct such least positive real numbers,
represented by angles, between 0 and 2p. p

3.1.1 Construction of Least Positive Angle whose Sine is Equal to a, | a | £ 1


Let a be a given real number with |a| £ 1. If a = 0, then clearly p is the least positive real number x satisfying the
equation sin x = 0. If a = 1 or a = -1, then p
p/2 or 3p
p/2 are, respectively, the least positive reals x for which sin x = 1 or
sin x = -1. Therefore, we can assume that 0 < |a| < 1.
Consider a rectangular coordinate system OXY. Let A be a point on OY such that OA = a (if a is negative, the
point A has to be chosen on YO produced, as shown in Figure 3.1).
Draw a circle of unit radius with O as the centre. Draw a line parallel to the X-axis through A meeting the circle
at P. Draw line PB perpendicular to the X-axis through P meeting the X-axis at B. Let q be the angle BOP. Then
BP = a and OP = 1 and hence
BP a
sin q = sin BOP = = =a
OP 1
Therefore q is the required angle.
3.1 The Least Positive Angles with a Given Trigonometric Ratio 143

Y Y

X
O

A P

a 1
A
q
X
O B

FIGURE 3.1

3.1.2 Construction of Least Positive Angle whose Cosine is Equal to a, | a | £ 1


a be a real number with |a| £ 1. Now pp/2 is the least positive real x such that cos x = 0. We can suppose that a ¹ 0.
Along the X-axis, let A be a point such that OA = a. (If a is negative, then A has to be chosen on the left part of the
X-axis). Draw a circle of unit radius with the origin O as the centre. Draw a perpendicular to X-axis at A meeting the
circle at P and P¢, with P on the semicircle above the x-axis (Figure 3.2). Then q = AOP is the required angle, since
OA a
cosq = cos AOP = = =a
OP 1

q
X
O a A

FIGURE 3.2

3.1.3 Construction of Least Positive Angle whose Tangent is a Given Real Number a
Let a be a nonzero real number. Let A be a point on the X-axis with OA as unit length. Draw a perpendicular to
X-axis at A and choose a point P on the perpendicular such that AP = a. If a is negative, then we have to consider P¢
in the lower part of the plane (Figure 3.3). Then q = AOP is the required angle, since
AP a
tan q = tan AOP = = =a
OA 1
144 Chapter 3 Trigonometric Equations

q
X
O 1 A

FIGURE 3.3

3.2 | General Solution of Equations of the Form sin x = a


As we have discussed earlier, for any real number a with |a| > 1, there is no real number x satisfying sin x = a or
cos x = a. However, if |a| £ 1, there are infinite number of solutions of the equation sin x = a or the equation cos x = a.
In the following, general forms of solutions of such type of equations are discussed.

3.2.1 General Solution of the Equation sin x = 0


For x Î[0, 2p),
p we know that
sin x = 0 Û x = 0 or p
Now, for any x Î, we have sin x = sin(x - 2kp),
p where k = [x/2p],
p the integral part of x/2p.
p Also, x - 2kp Î[0, 2p).
p
Therefore
sin x = 0
Û sin( x - 2kp ) = 0
Û x - 2kp = 0 or p
Û x = 2kp or (2k + 1)p
Since k is an integer, it follows that sin x = 0 Û x = np
p for any integer n. Therefore, the general solution of the equation
sin x = 0 is x = np,
p n Î, or the solution set of the equation sin x = 0 is {np |n Î}.

3.2.2 General Solution of Equations cos x = 0 and tan x = 0


For any real number x, we have

æ pö
cos x = 0 Û sin ç x - ÷ = 0
è 2ø
p
Ûx- = np for some integer n
2
2n + 1
Ûx= p or some integer n
fo
2
Therefore, the solution set of the equation cos x = 0 is
ìæ 2 n + 1ö ü
íçè ÷ø p n Î  ý
î 2 þ
3.2 General Solution of Equations of the Form sin x = a 145

Also
tan x = 0 Û sin x = 0 Û x = np , n Î
Note that sin x = 0 implies that cos x ¹ 0 and hence tan x is defined. On the other hand, tan x is defined; also tan x = 0
implies that sin x = 0 and cos x ¹ 0. Thus, for any real number x, tan x = 0 Û x = np
p for some integer n.

3.2.3 General Solution of the Equation sin x = a


We can assume that |a| £ 1; for, otherwise sin x = a has no solution. Then -1 £ a £ 1. Let q be a solution of sin x = a;
that is, sin q = a. Now, for any x,
sin x = a Û sin x = sin q

Û sin x - sin q = 0

æ x +qö æ x -qö
Û 2 cos ç sin ç =0
è 2 ÷ø è 2 ÷ø

x +q x -q
Û cos = 0 or sin =0
2 2

x +q p
Case 1: = (2 n + 1) (by Section 3.2.2)
2 2
x -q
2: = np (by Section 3.2.1)
2
For Case 1:

x = (2 n + 1)p - q , n Î 

Û x = (2 n + 1)p + (-1)2 n+1 q , n Î 

For Case 2:
x = 2 np + q , n Î 

Û x = 2 np + (-1)2 n q , n Î 

x = mp + (-1)m q , m Î 

Thus the general solution of the equation sin x = a is mp + (-1)m q, m Î, where q is any real number such that sin q = a;
for example, using Section 3.1.1 we can take the least positive angle q such that sin q = a or we can take Sin-1 a for q.
Note: For any real number a such that |a| ³ 1 (i.e., a Î(-¥, -1] È [1, ¥)), general solution of the equation cosec x = a
is mp + (-1)m q, where q is such that cosec q = a.

Example 3.1

Find the general solution of the equation Solution: The smallest positive angle whose sine is
3 / 2 is 60° or p
p/3. Therefore, the general solution of the
sin x =
3 equation sin x = 3 / 2 is mp + (-1)m p/3,
p m Î.
2
146 Chapter 3 Trigonometric Equations

3.2.4 General Solution of the Equation cos x = a


If |a| > 1, then the equation cos x = a has no solution. Suppose that |a| £ 1 and q is a solution of cos x = a; that is,
cos q = a. Now, for any real number x,
cos x = a Û cos x = cos q

Û cos x - cosq = 0

æ x +qö æ x -qö
Û - 2 sin ç sin ç =0
è 2 ÷ø è 2 ÷ø

æ x +qö æ x -qö
Û sin ç = 0 or sin ç =0
è 2 ÷ø è 2 ÷ø

x +q x -q
Û = np , n Î  or = np , n Î 
2 2
Û x = 2 np ± q, n Î 

Thus, if q is any angle such that cos q = a, then the solution set of cos x = a is {2np ± q|n Î}. We can take q = Cos-1 a.
Note: If |a| ³ 1, the general solution of the equation sec x = a is 2np ± q, n Î, where q is any solution of sec x = a.

Example

The general solution of the equation cos x = -1/2 is 2np ± (2p/3


p ), n Î since 120°(=2p/3)
p is a solution of cos x = -1/2.

Example 3.2

Find the general solution of the equation sin2 x = 1/4. Also, since sin(-p/6)p = -1/2, the general solution of
sin x = -1/2 is np + (-1)n (-p/6).
p Thus the general solution
Solution: The given equation can be written as of sin2 x = 1/4 is
sin x = ±1/2. Since sin p/6
p = 1/2, the general solution of
p
sin x = 1/2 is np + (-1)n(p/6),
p n Î. np ± (-1)n , n Î
6

3.2.5 General Solution of the Equation tan x = a


Let a Î be fixed and q be any solution of tan x = a; that is, q is a real number such that tan q = a. Then, for any real
number x,
tan x = a Û tan x = tan q

sin x sin q
Û =
cos x cos q

Û sin x cosq - cos x sin q = 0

Û sin( x - q ) = 0
Û x - q = np , n Î 

Û x = np + q , n Î 

T -1 a.
x = a is np + q, n Î, where q is a solution of tan x = a; here also, we can take q = Tan
Note: For any a Î, the general solution of the equation cot x = a is np + q, n Î, where q is a solution of cot x = a.
3.2 General Solution of Equations of the Form sin x = a 147

Example

The general solution of the equation tan x = 1/ 3 is np + (p/6),


p n Î, since tan p /6 = 1/ 3; that is, p
p/6 is a solution of
tan x = 1/ 3.

3.2.6 General Solution of the Equation sin2 x = sin2 q


For any given real number q
q, let us find the general solution of the equation sin2 x = sin2 q
q. Note that, for any real number x,
1 - cos 2 x 1 - cos 2q
sin2 x = sin2 q Û =
2 2
Û cos 2 x = cos 2q
Û 2 x = 2 np ± 2q , n Î 
Û x = np ± q , n Î 
If x is a solution sin x = sin q, then -x
2 2
- is also a solution of the same equation. Therefore, the general solution of the
equation sin2 x = sin2 q is np ± q, n Î.
Note: The general solution of cos2 x = cos2 q and tan2 x = tan2 q are np ± q, n Î

Example

The general solution of the equation tan2 x = 1/3 is np ± p/6,


p n Î; for, tan(p /6) = 1/ 3 and hence tan2(p/6)
p = 1/3.

Example 3.3

Find the general solution of the equation sec2 x = 4/3. p


Û cos2 x = cos2 (30°) = cos2
6
Solution: We have p
Û x = np ± , n Î
2 6
4 3 æ 3ö
sec2 x = Û cos2 x = = ç
3 4 è 2 ÷ø

Example 3.4

Find the general solution satisfying both the equations between 0 and 2p p that satisfy the equation tan x = 1/ 3
sin x = -1/2 and tan x = 1/ 3 . are x = 30° = p
p/6 and x = 210° = 7p/6.
p Therefore, the only
angle between 0 and 2p p that satisfies both the equations
Solution: Consider the equation sin x = -1/2. The only sin x = -1/2 and tan x = 1/ 3 is 210° = 7p/6.
p Hence, the
angles between 0 and 2p p that satisfy sin x = -1/2 are general solution of these two equations is 2np + (7p/6),
p
x = 330° = 11p/6
p and x = 210° = 7p/6.
p Also, the only angles n Î.

Example 3.5

Find the general solution of the combined equations æ -p ö


cot ç =- 3
cot x = - 3 and cosec x = -2. è 6 ÷ø

Solution: T æ -p ö
The general solution of the combined equations cosec ç = -2
è 6 ÷ø
cot x = - 3 and cosecx = -2 is 2np - (p/
p 6), n Î because
148 Chapter 3 Trigonometric Equations

3.3 | Solutions of Simple Trigonometric Equations


DEFINITION 3.2 Equations involving trigonometric ratios of an unknown angle are called trigonometric
equations.

An equation is not completely solved unless we obtain a general expression for all angles which satisfy it. In this
section, we solve certain elementary types of trigonometric equations.

3.3.1 General Solution of an Equation of the Form a sin x + b cos x = c


a, b and c be any given real numbers such that a ¹ 0 or b ¹ 0. Consider the equation
a sin x + b cos x = c

Put r = a2 + b2 . Then r > 0 (since a ¹ 0 or b ¹ 0). Suppose that |c/r| £ 1. Choose a real number q such that
a
sinq =
r
b
and cosq =
r
Then the given equation can be written as
r sin q sin x + r cosq cos x = c
that is
c
cos( x - q ) =
r
This can be solved for x, since |c/r| £ 1.
We can choose a Î[0, p p] such that cos a = c/r. Then x = 2np ± a + q, n Î is the general solution of the given
equation a sin x + b cos x = c.
Alternate Form
The alternate form of the general solution of the above equation can be obtained as follows: Let r = a2 + b2 and
suppose that |c/r| £ 1. Then we can choose q Î such that
cos q = a / r and sin q = b / r
Then the given equation can be written as
c
cos q sin x + sin q cos x =
r
that is
c
sin( x + q ) =
r
This can be solved for x. Since |c/r| £ 1, there exists a Î[-p/2,
p p p/2] such that sin a = c/rr this a is precisely Sin-1(c/r).
Then x = np + (-1) a - q, n Î is the general solution of the given equation.
n

Example 3.6

Solve sin x + 3 cos x = 2 . Therefore we have

Solution: The given equation can be written in the r = 12 + ( 3 )2 = 2


form a sin x + b cos x = c. Now r is given by
Dividing both sides of the given equation by 2, we get
r = a +b
2 2
1 3 1
sin x + cos x =
2 2 2
3.3 Solutions of Simple Trigonometric Equations 149

Since sin p/6


p = 1/2 and cos p / 6 = 3 / 2, the equation can Alternate Solution
be written as As mentioned in the note “Alternate Form”, the
given equation can be solved as follows also. Since
p p 1
sin sin x + cos cos x = sin p / 3 = 3 / 2 and cos p / 3 = 1/ 2, the given equation can
6 6 2 be written as
that is p p 1
cos sin x + sin cos x =
æ pö 1 p 3 3 2
cos ç x - ÷ = = cos
è 6ø 2 4 that is
Therefore æ pö 1 p
sin ç x + ÷ = = sin
p p è 3 ø 2 4
x- = 2 np ± , n Î  (by Section 3.2.4)
6 4 Therefore
p p p p
x = 2 np + ± , n Î  x+ = np + (-1)n , n Î  (by Section 3.2.3)
6 4 3 4
This gives p p
x = np + (-1)n - , n Î
4 3
5p
x = 2 np + , n Î
12 It can be seen that both the solution sets obtained above
are equal. That is,
p
or x = 2 np - , n Î
12 ì p p ü ì p p ü
í2 np + ± n Î  ý = ínp + (-1)n - n Îý
î 6 4 þ î 4 3 þ

Example 3.7

Solve 2 sin2 x - cos 2x = 0. p æ p 1ö


Û x = np ± çè since sin = ÷ø
6 6 2
Solution: Since cos 2x = 1 - 2 sin2 x, we have
Therefore np ± p/6,
p n Î is the general solution of the
2 sin2 x - cos 2 x = 0 given equation.
Û 4 sin2 x = 1
1
Û sin x = ±
2

Example 3.8

Solve sin 5x = cos 2x. p


Û 5 x + (-1)n 2 x = np + (-1)n , n Î
2
Solution: We have
æ 2 n + (-1)n ö p
Ûx=ç , n Î
sin 5 x = cos 2 x è 5 + 2(-1)n ÷ø 2
æp ö
Û sin 5 x = sin ç - 2 x÷ Therefore
è2 ø
æ 2 n + (-1)n ö p
æp ö çè 5 + 2(-1)n ÷ø 2 , n Î 
Û 5 x = np + (-1) ç - 2 x÷ , n Î 
n
è2 ø
is the general solution of the equation sin 5x = cos 2x.
150 Chapter 3 Trigonometric Equations

WORKED-OUT PROBLEMS
Single Correct Choice Type Questions
1. The general solution of the equation 11 sin x + 2 cos2 x = 2p p
Ûx+ y= and x - y = 4 np ± , n Î
7 is 3 3
(A) 2np ± (p/3)
p p p
Û x = 2 np + and y= - 2 np , n Î  or vice- versa
(B) 2np + (-1)n(p/6),
p n Î 6 2
(C) (2n + 1)p + (-1)n(p/6),
p n Î Answer: (B)
(D) np + (-1) (p/6),
n
p n Î
3. If 0 < x < p
p/2 and sin(x + 28°) = cos(3x - 78°), then the
Solution:
possible value(s) of x is (are)
11 sin x + 2 cos2 x = 7 (A) 8° or 35° (B) 9° or 36°
Û 2(1 - sin2 x) + 11 sin x = 7 (C) 18° or 45° (D) 28° or 55°
( + 28°) = cos(3x - 78°). Then
Solution: Suppose that sin(x
Û 2 sin2 x - 11 sin x + 5 = 0
Û (sin x - 5)(2 sin x - 1) = 0 æp ö
sin( x + 28°) = sin ç - (3 x - 78°)÷ = sin(168° - 3 x)
è2 ø
Û sin x = 5 or sin x = 1/ 2
1 Therefore
Û sin x = (since sin x = 5 is impo
ossible)
2 x + 28° = 2 np + 168° - 3 x, n Î 
Û x = np + (-1) (p / 6), n Î 
n
or x = p - (168° - 3 x) + 2 np , n Î 
Answer: (D)
Hence
2. The values of x and y, when x + y = 2p/3
p and cos x + 4 x = 2 np + 140°, n Î 
cos y = 3 / 2, are respectively
p p or 2 x = 16° - 2 np, n Î 
(A) 2 np + and - 2 np , n Î 
3 3 This gives
p p
(B) 2 np + and - 2 np , n Î 
6 2 x = 35° + n(90°)
p p
(C) np + and - np , n Î  or x = 8° - n(180°), n Î 
6 2
p p Now 0 < x < 90° Þ x = 35° or 8°.
(D) np + and - 2 np , n Î 
3 3 Answer: (A)
Solution:
4. Suppose that cos 3x cos 2x cos x = 1/4. If 0 < x < p/4,
p
2p 3 then the value of x is
x+ y= and cos x + cos y =
3 2 p p p p
(A) (B) (C) (D)
2p x+y x-y 3 5 6 8 7
Ûx+ y= and 2 cos × cos =
3 2 2 2 2 cosx = 1/4.Therefore
Solution: It is given that cos3xcos2x
2p p x-y 3 4 cos x cos 2 x cos 3 x = 1
Ûx+ y= and 2 cos cos =
3 3 2 2
(2 cos x cos 3 x) 2 cos 2 x = 1
2p 1 x-y 3
Ûx+ y= and 2 × cos =
3 2 2 2 (cos 4 x + cos 2 x) 2 cos 2 x = 1
2p x-y p 2 cos2 2 x + 2 cos 2 x cos 4 x - 1 = 0
Ûx+ y= and cos = cos
3 2 6
cos 4 x + 2 cos 2 x cos 4 x = 0
2p x-y p
Ûx+ y= and = 2 np ± , n Î 
3 2 6 cos 4 x(1 + 2 cos 2 x) = 0
Worked-Out Problems 151

This gives Solution:


cos 4x = 0 tan x + cot x = 2 cosec x

or 1 + 2 cos 2 x = 0 sin x cos x 2


Þ + =
cos x sin x sin x
Therefore
Þ sin2 x + cos2 x = 2 cos x
(2 n + 1)p
4x = , n Î 1 p
2 Þ cos x = = cos
2 3
-1
or cos 2 x = p
2 Þ x = 2 np ± , n Î
3
Hence
Hence p
p/3, 2p - (p/3)
p (= 5p/3)
p are the principal solutions.
æ 2 n + 1ö Answer: (C)
x=ç p , n Î
è 8 ÷ø
7. General solution of the equation tan x + tan(p/3
p + x) +
2p p + x) = 3 is
tan(2p/3
or 2 x = 2 np ± , n Î
3 p p
(A) np + , n Î  (B) np + , n Î
2p 3 4
or x = np ± , n Î np p np p
6 (C) + , n Î (D) + , n Î
3 12 3 4
If 0 < x < p
p/4, then x = p/8.
p
Solution:
Answer: (C)
æp ö æ 2p ö
5. General solution of the equation cos 5x = -1/2 is
tan x + tan ç + x÷ + tan ç + x÷ = 3
è3 ø è 3 ø
2p tan(p / 3) + tan x tan(2p / 3) + tan x
(A) np + (-1)n , n Î Þ tan x + + =3
3 1 - tan(p / 3)tan x 1 - tan(2p / 3)tan x
2p
(B) 2 np ± , n Î 3 + tan x - 3 + tan x
3 Þ tan x + + =3
1 - 3 tan x 1 + 3 tan x
2 np p
(C) ± , n Î
5 15 8 tan x
Þ tan x + =3
2 np 2p 1 - 3 tan2 x
(D) ± , n Î
5 15 Þ tan x - 3 tan3 x + 8 tan x = 3(1 - 3 tan2 x)
Solution: 3(3 tan x - tan3 x)
Þ =3
-1 1 - 3 tan2 x
cos 5 x =
2 p
Þ tan 3 x = 1 = tan
2p 4
Û cos 5 x = cos p
3 Þ 3 x = np +
, n Î
4
2p
Û 5 x = 2 np ± , n Î np p
3 Þx= + , n Î
3 12
2np 2p
Ûx= ± , n Î Answer: (C)
5 15
Answer: (D) 8. x + sin 3x +
sin 5x = 0 is
6. A principal solution of the equation tan x + cot x = 2 np p
2 cosec x is (A) , n Î (B) (3n ± 1) , n Î 
3 3
7p 3p 5p 2p 4 np np
(A) (B) (C) (D) (C) , n Î (D) , n Î
8 4 3 3 3 3
152 Chapter 3 Trigonometric Equations

Solution: Þ tan2 q - tan4 q + 1 + tan2 q = 1 - tan2 q


sin x + sin 3 x + sin 5 x = 0 Þ tan4 q = 3 tan2 q
Þ (sin 5 x + sin x) + sin 3 x = 0
Þ tan2 q (tan2 - 3) = 0
Þ 2 sin 3 x cos 2 x + sin 3 x = 0
This implies that either
Þ (sin 3 x)(2 cos 2 x + 1) = 0
tan2 q = 0
This implies that either
or tan2 q = 3
sin 3x = 0
which further gives
or 2 cos 2x + 1 = 0
Therefore tanq = 0

3x = np, n Î  æ pö
or tan q = ± 3 = tan ç ± ÷
è 3ø
-1 2p
or cos 2 x = = cos
2 3 Hence

From these we get q = np , n Î 


np p
x= , n Î or q = np ± , n Î
3 3

æ 2p ö Note that for tan q to be defined, we must have q ¹


or 2 x = 2 np ± ç ÷ , n Î  (2n + 1)p/2
p and for sec 2q q to be defined, we must have
è 3ø
2 q ¹ (2n + 1)p/2
p or q ¹ (2n + 1)p/4.
p Therefore, the general
values of q satisfying the given equation are
np p
x= , n Î np , np ± , n Î
3 3
p Answer: (B)
or x = (3n ± 1) , n Î 
3
10. General value of x for which the sets P = {sin x, sin 2x,
Hence sin 3x} and Q = {cos x, cos 2x, cos 3x} are equal is
np p np p np
x= , n Î  (since 3n ± 1 Î ) (A) + , n Î (B) + , n Î
3 4 2 8 2
p np p np
Answer: (D) (C) + , n Î (D) + , n Î
12 2 15 2
9. The general values of q satisfying the equation tan2 q + Solution: Since P = Q, the sum of elements in P and
sec 2q = 1 are sum of the elements in Q are equal. Therefore
p
(A) (2 n + 1)p , np ± , n Î sin x + sin 2 x + sin 3 x = cos x + cos 2 x + cos 3 x
3
p (sin x + sin 3 x) + sin 2 x = (cos x + cos 3 x) + cos 2 x
(B) np , np ± , n Î
3
p 2 sin 2 x cos x + sin 2 x = 2 cos 2 x cos x + cos 2 x
(C) np ± , n Î
3 sin 2 x(2 cos x + 1) = cos 2 x(2 cos x + 1)
p
(D) np ± , n Î  If cosx = -1/2, then x = 2np ± (2p/3)
p in which case P ¹ Q.
4
Hence sin2x 2 = cos2x
2 which gives that tan2x 2 = 1 and
Solution: 2 = np + (p/4)
hence 2x p or x = (np/2)
p + (p/8)
p and these values
tan2 q + sec 2q = 1 of x satisfy the condition that P = Q (check this).
1 + tan2 q Answer: (B)
Þ tan2 q + =1
1 - tan2 q
Worked-Out Problems 153

11. If the angles A and B of DABC


C satisfy the relations kp p p
3 sin A + 4 cos B = 6 and 4 sin B + 3 cos A = 1, then the (A) + , k Î (B) (2k + 1) , k Î
3 12 12
third angle C is
kp p kp p
p 2p p 5p (C) + , k Î (D) + , k Î
(A) (B) (C) (D) 12 3 6 3
3 3 6 6
Solution: The given equation can be written as
Solution: Squaring and adding the two given equa-
tan x + tan 2 x
tions we get =1
1 - tan x tan 2 x
24 sin( A + B) + 25 = 37

Therefore tan 3x = 1
sin( A + B) = 1/ 2 Therefore
A + B = 30° or 150° p
3 x = kp + , k Î
4
This gives that either C = 30° or 150°. If C = 150°, then kp p
3 11 1 x= + , k Î
A < 30° so that 6 = 3sin A + 4cos B < + 4 = =5 3 12
2 2 2
which is impossible. Therefore Answer: (A)
C = 30° 14. The number of solutions in the interval [0, p
p] of the
Answer: (C) equation sin3 x cos 3x + sin 3x cos3 x = 0 is
(A) 7 (B) 6 (C) 5 (D) 4
12. General solution of the equation sin6 x = 1 + cos4 3x is
p Kp Solution: The given equation can be written as
(A) (2 K + 1) , K Î  (B) , K Î
6 3 æ 3 sin x - sin 3 x ö (3 cos x + cos 3 x)
çè ÷ø cos 3 x + sin 3 x =0
p Kp 4 4
(C) (2 K + 1) , K Î  (D) , K Î
2 2
which is equivalent to
Solution: sin x cos 3 x + sin 3 x cos x = 0
sin6 x £ 1 and 1 + cos4 3 x ³ 1 Therefore
Þ sin6 x = 1 and cos4 3 x = 0 sin 4x = 0
np
Þ sin2 x = 1 and cos 3x = 0 or x= , n Î
4
Therefore Answer: (C)
cos x = 0 and cos 3x = 0
15. The general solution of the equation 2 sin x cos x +
which gives 5 cos2 x = 4 is
x = (2 K + 1)p / 2 and 3 x = (2 K + 1)p / 2 æ ö æ ö
(A) np + Tan-1 ç 5 + 1÷ , np - Tan-1 ç 5 - 1÷ , n Î 
è 4 ø è 4 ø
Hence
np æ 3 + 1ö np æ 3 - 1ö
x = (2 K + 1)p / 2 and x = (2 K + 1)p / 6 + Tan-1 ç ÷ , - Tan-1 ç , n Î
2 è 2 2 ø 2 è 2 2 ÷ø
All solutions of the form x = (2K + 1)p/2
p are included in
æ 6 + 2ö
the solution set (C) (2 n + 1)p + Tan-1 ç ,
è 2 2 ÷ø
ì p ü
í(2 K + 1) K Î  ý æ 6 - 2ö
î þ (2 n + 1)p - Tan-1 ç , n Î
è 2 2 ÷ø
6

which is the solution set. p æ ö


Answer: (A) (D) (2 n + 1) + Tan-1 ç 5 + 1÷ ,
2 è 4 ø
p æ ö
13. General solution of the equation tanx + tan2x
2 + (2 n + 1) + Tan-1 ç 5 - 1÷
2 = 1 is
tanxtan2x 2 è 4 ø
154 Chapter 3 Trigonometric Equations

Solution: np p np p
(C) x = + ,x= +
15 20 20 100
2 sin x cos x + 5 cos2 x = 4(sin2 x + cos2 x)
2 np p 2np p
(D) x = + ,x= -
4 sin2 x - 2 sin x cos x - cos2 x = 0 15 20 25 50
4 tan2 x - 2 tan x - 1 = 0 Solution: Squaring both sides of the given equation we get
(since cos x = 0 is not a solution of the original equation.) (sin 10 x + cos 10 x)2 = 2 sin2 15 x
Therefore
Therefore
2 ± 20 1 ± 5
tan x = = 1 + sin 20 x = 1 - cos 30 x
8 4
æ ö æ ö æp ö
cos 30 x = - sin 20 x = cos ç + 20 x÷
x = np + Tan-1 ç 5 + 1÷ , np - Tan-1 ç 5 - 1÷ , n Î  è2 ø
è 4 ø è 4 ø
æp ö
Answer: (A) 30 x = (2 np ) ± ç + 20 x÷
è2 ø

16. The number of solutions of the equation 1 + sin 2 x = p p


10 x = 2np + , 50 x = 2np -
p 3p/2]
2 cos 3 x belonging to the interval [p, p is 2 2
(A) 5 (B) 4 (C) 3 (D) 2 np p np p
x= + ,x= -
Solution: Squaring both sides of the equation we get 5 20 25 100
Answer: (A)
1 + sin 2 x = 2 cos2 3 x = 1 + cos 6 x
Therefore 18. General solution of the equation sin2 x - 5 sin x cos x +
6 cos2 x = 0 is
æp ö (A) (2 n + 1)p + Tan-1 2, 2 np + Tan-1 (1/ 2)
cos 6 x = sin 2 x = cos ç - 2 x÷
è2 ø
np + Tan-1 2, np + Tan-1 3
æp ö (C) np + Tan-1 (1/ 2), np + Tan-1 (1/ 3)
6 x = (2 np ) ± ç - 2 x÷
è2 ø (D) 2 np + Tan-1 2 , 2 np + Tan-1 3
np p np p Solution: Clearly cos x = 0 is not a solution of the equa-
x= + ,x= - , n Î
4 16 2 8 tion. Therefore by dividing the given equation with cos2 x,
we get
17p é 3p ù
(i) n = 4 Þ x = Î p,
16 êë 2 úû tan2 x - 5 tan x + 6 = 0

When x = 17p/16,
p the left-hand side of the original Solving this we get that tan x = 2 and tan x = 3 are solu-
equation is positive while the right-hand side is tions. Therefore the general solution is
negative. Hence x = 17p/16
p is not a solution. x = np + Tan-1 2
21p é 3p ù
(ii) n = 5 Þ x = Î p, and x = np + Tan-1 3
16 êë 2 úû
Answer: (B)
3p p 11p é 3p ù
(iii) n = 3 Þ x = - = Î êp ,
2 8 8 ë 2 úû 19. The equation
Answer: (D) æ æ x2 + 1 ö
2 xö
çè 2 cos ÷ø sin x = çè 2 ÷ø , x £ p / 9
2

2 x
17. General solution of the equation sin 10 x + cos 10 x =
2 sin 15 x is has
np p np p (A) no solution
(A) x = + ,x= -
10 20 25 100 (B) one solution
np p np p
(B) x = + ,x= - (C) more than one real solution
20 10 50 50
(D) cannot be said
Worked-Out Problems 155

Solution: We have Therefore

æ xö x2 + 1 cos7 x + cos4 x - 2 cos2 x = 0


2 cos2 ç ÷ sin2 x £ 2 and ³2
è 2ø x2 cos2 x(cos5 x + cos2 x - 2) = 0
and x ¹ 0, x £ p
p/9. Therefore
Now cos2 x = 0 which means x = (2n + 1)p/2p or cos5 x +
æ xö x +1
2
cos x - 2 = 0 which implies that x = 2np.
2
p Therefore the
2 cos2 ç ÷ sin2 x = 2 and =2 p pp) are -p/2,
p 0, pp/2 corresponding
è 2ø x2 roots which lie in (-p,
to n = -1, 0.
which gives Answer: (B)
(1 + cos x)(1 - cos x) = 2 and
2
x = ±1
23. The values of q Î (0, 2p)
p for which 2 sin2 q - 5 sin q +
cos x + cos x - cos x + 1 = 0 and
3 2
x = ±1 2 > 0 are
æ p ö æ 5p ö æ p 5p ö
(A) ç 0, ÷ È ç , 2p ÷ (B) ç , ÷
solution. è 6ø è 6 ø è8 6 ø
Answer: (A) æ p ö æ p 5p ö æ 41p ö
(C) ç 0, ÷ È ç , ÷ (D) ç , p÷
è 8ø è 6 6 ø è 48 ø
20. The number of solutions of the equation sin(ex) =
5x + 5-x
-
is Solution: We have
(A) 0 (B) 1 (C) 2 (D) infinitely many 2 sin2 q - 5 sin q + 2 > 0
Solution: We have Û (2 sin q - 1)(sin q - 2) > 0
-
-x
sin(e ) £ 1
x
and 5 +5 ³2
x
Û 2 sin q - 1 < 0 (∵ sin q < 2)
Therefore the equation has no solution. 1
Answer: (A) Û sinq <
2

21. The general value of q satisfying the equation


p 5p
Û 0<q < or < q < 2p
2 sin q - 3 sin q - 2 = 0 is
2 6 6
p p Answer: (A)
(A) np + (-1)n (B) np + (-1)n
6 2
24. Number of roots of the equation sin 2x + 2 sin x -
5p 7p
(C) np + (-1)n (D) np + (-1)n cos x - 1 = 0 in the range 0 £ x £ 2p
p is
6 6
(A) 1 (B) 2 (C) 3 (D) 4
Solution: The given equation can be written as
Solution: The given equation can be written as
(2 sin q + 1)(sin q - 2) = 0
2 sin x cos x + 2 sin x - cos x - 1 = 0
Therefore
(2 sin x - 1)(cos x + 1) = 0
2 sin q + 1 = 0
-1 æ 7p ö
sin q = = sin ç ÷
2 è 6 ø sin x = 1/ 2 or cos x = - 1
7p which implies
q = np + (-1)n
6
p 5p
Answer: (D) x= , ,p
6 6
22. The number of roots of the equation cos7 x + sin4 x = 1 p
are the roots of the equation in the range [0, 2p].
p p
in the open interval (-p, p) is Answer: (C)
(A) 4 (B) 3 (C) 2 (D) 1
25. In DABC, angle A is greater than angle B. If
Solution: The given equation is the measures of the angles satisfies the equation
3 sin x - 4 sin3 x - k = 0, 0 < k < 1, then the measure of
cos x + (1 - cos x) = 1
7 2 2
the angle C is
156 Chapter 3 Trigonometric Equations

p 5p p 2p Solution: We simplify the left-hand side of the given


(A) (B) (C) (D) equation:
3 6 2 3
Solution: By hypothesis | 3(sin4 x + cos4 x) - 2(sin6 x + cos6 x) + y |2

sin 3 A = k = sin 3B = | 3[1 - 2 sin2 x cos2 x] - 2[1 - 3 sin2 x cos2 x] + y |2


= | 1 + y |2
Therefore
3 A + 3B (3 A - 3B) This is equal to the right-hand side (=9) when y = 2 or -4.
2 cos sin =0 Answer: (A)
2 2
3 p
( A + B) = or A = B 28. The number of values of x in the range -p/2
p < x < p/2
p
2 2
and satisfying the equation sin3 x + sin2 x + sin x -
But A > B, which implies sin x sin 2x - sin2x - 2 cos x = 0 is
p (A) 0 (B) 1 (C) 2 (D) 3
A+ B=
3 Solution: The given equation can be written as
p
Þp -C = sin x(1 + sin x + sin2 x) - (1 + sin x + sin2 x) 2 cos x = 0
3
2p (1 + sin x + sin2 x)(sin x - 2 cos x) = 0
ÞC =
3
Therefore
Answer: (D)
sin x - 2 cos x = 0 (∵ 1 + sin x + sin2 x > 0 for all real x)
26. x - 3 sin 2x +
tan x = 2
sin 3x = cos x - 3 cos 2x + cos 3x is
p np p as cos x = 0 is not a root of the initial equation. Therefore
(A) np + (B) +
8 2 8 x = Tan-1 2 is the only solution in (-p/2,
p p/2).
p
np p 2 Answer: (B)
(C) (-1)n + (D) 2 np + Cos-1
2 8 3
29. If 2 sec 2a = tan b + cot b, then a + b may be
Solution: We have p p p
(A) (B) (C) (D) 0
(sin x + sin 3x) - 3 sin 2 x = (cos x + cos 3x) - 3 cos 2 x 2 4 3
2 sin 2 x cos x - 3 sin 2 x = 2 cos 2 x cos x - 3 cos 2 x Solution:
sin 2 x(2 cos x - 3) = cos 2 x(2 cos x - 3)
2 sec 2a = tan b + cot b
This gives sin b cos b
= +
sin 2 x = cos 2 x (∵ 2 cos x ¹ 3) cos b sin b
p sin2 b + cos2 b
2 x = np + =
4 sin b cos b
That is 1
=
sin b cos b
np p
x= + = 2 cosec 2 b
2 8
Answer: (B) Therefore

27. The equation |3sin x - 2sin x + y - 2cos x + 3cos x| =


4 6 6 4 2 sec 2a = cosec 2 b
9 is true Now 2a = 2b = p
p/4 is possible. Therefore
(A) for any value of x and y = 2 or y = -4 p
(B) only for x = p
p/4 or p and y = -2 or 4 a+b=
4
(C) only for x = p
p/2 or p and y = 2 or -4
is a possible value.
(D) only for x = 0 or p
p/2 and y = ±2
Answer: (B)
Worked-Out Problems 157

30. General value of q satisfying the equation sin 6q = p


a+b=
sin 4q - sin 2q
q is 2
np p Answer: (B)
(A) , n Î (B) np ± , n Î
4 6
np p np 32. The set of values of q which satisfy the equation
(C) or np ± , n Î  (D) , n Î
4 6 2 cos 2q = sin q + cos q is
(A) q = np + (p/2),
p n Î
Solution: The given equation can be written as sin 6q +
sin 2q = sin 4q. Therefore (B) q = 2npp or q = np/4
p where n Î
(C) q = 0
2 sin 4q cos 2q = sin 4q
(D) q = 2 np or 2 np - (np / 2) or q = np - (p / 4), n Î 
sin 4q (2 cos 2q - 1) = 0
Solution: The given equation, cos 2q = sin q + cos q, can
Now either be written as
4q = np (cosq + sin q )(cosq - sin q - 1) = 0
p Therefore
or 2q = 2 np ±
3 cos q + sin q = 0
Therefore or cos q - sin q = 1
np Hence
q=
4
tan q = - 1
p
or q = np ± æ pö 1
6 or cos ç q + ÷ =
è 4 ø 2
Answer: (C)
This implies that
31.
with regard to the solutions of the equation tan2 x - p
q = np -
5 tan x + 1 = 0 ? 4
(A) No solution exists in the range 0 £ x £ p/2.
p p p
or q + = 2 np ±
(B) There exist two solutions a and b in the interval 4 4
[0, p
p/2] such that a + b = p/2.
p Hence
(C) Two solutions a, b exist in [0, pp/2] such that p
a + b = p/4.
p q = np -
4
Two solutions a, b exist in [0, p
(D) T p/2] such that
a - b = p/4.
p or q = 2np
Solution: From the given equation we have p
or q = 2 np -
2
5±1 Answer: (D)
tan x =
2
6 + 8sin2 3x = 0 is
33. General solution of the equation sin2 6x
Let
np np
(A) , n Î (B) , n Î
5+1 5-1 3 2
tan a = and tan b =
2 2 np np
(C) , n Î (D) , n Î
4 6
Clearly a, b Î[0, p
p/2] and also
Solution: We have
5-1 2 æp ö
tan b = = = cot a = tan ç - a ÷ sin2 6 x + 8 sin2 3 x = 0
2 5+1 è2 ø
4 sin2 3 x(cos2 3 x + 2) = 0
Therefore
Therefore
p
b= -a sin2 3 x = 0
2
158 Chapter 3 Trigonometric Equations

and hence sin 3x = 0. This gives Therefore


3 x = np , n Î  1
sinq = ±
np 2
x= , n Î
3 which implies
Answer: (A) p 5p 7p 11p
q= , , ,
6 6 6 6
34. The equation sin(cos x) = cos(sin x) has
(A) only one real solution The second equation is
(B) infinitely many solutions 2 sin2 q + 3 sin q - 2 = 0
(C) no real solution (2 sin q - 1)(sin q + 2) = 0
(D) two real solutions
which implies
Solution: We have
1
cos[(p / 2) - cos x] = cos(sin x) sin q =
2
Therefore p 5p
or q= ,
6 6
(p / 2) - cos x = 2 np ± sin x, n Î 
Therefore p
p/6 and 5p
p/6 satisfy both equations.
cos x + sin x = (p / 2) - 2 np = cos x - sin x
Answer: (C)
Hence sin x = 0 and cos x = (p/2)
p - 2np p which is impos-
sible, because sin x = 0 implies cos x = ±1.
2
37. The number of solutions of the equation 16sin x +
2
Answer: (C) 16cos x = 10 in the range 0 £ x £ 2p
p is
(A) 4 (B) 6 (C) 8 (D) 10
35. The value of x satisfying the equation 2 cos2(x3 + x) =
Solution: We have
2x + 2-x
-
is
2 2
(A) 0 (B) -1 (C) x = 2np
p (D) x = np 16sin x + 16cos x = 10
Solution: The given equation can be written as 2 16
16sin x + 2
x
= 10
1 + cos(2 x + 2 x) = 2 + 2
3 x -x 16sin
2

Left-hand side is £ 2 and right-hand side is ³ 2. Therefore t = 16sin x


we get

cos(2 x3 + 2 x) = 1 t 2 - 10t + 16 = 0

and 2x + 2- x = 2 (t - 2)(t - 8) = 0

Now Þ t = 2, 8

2x + 2- x = 2 Û 22 x = 1 Case 1: When t = 2 we have


2
Ûx=0 16sin x = 2
2
x = 0 Þ cos(2 x3 + 2 x) = 1 24 sin x = 1

x = 0 is the only solution of the equation. Þ 4 sin2 x = 1


Answer: (A) 1
Þ sin x = ±
2
36. The number of solutions of the pair of equations p 5p 7p 11p
2 sin2 q - cos 2q = 0 and 2 cos2 q - 3 sin q = 0 in the Þx= , , ,
6 6 6 6
p is
interval [0, 2p]
(A) 0 (B) 1 (C) 2 (D) 4 Case 2: When t = 8 = 23 we have
2
Solution: The first equation is 16sin x = 23
2
-1 + 4 sin2 q = 0 24 sin x
= 23
Worked-Out Problems 159

Þ 4 sin2 x = 3 Since -p/4


p £x£p
p/4, sin x + 2 cos x ¹ 0. Therefore

3 sin x - cos x = 0 or tan x = 1


Þ sin x = ±
2 Therefore x = p
p/4 is the only solution.
p 2p 4p 5p Answer: (D)
Þx= , , ,
2 3 3 3
40. The number of values of q in the range 0 £ q £ 360°
p
Therefore there are total eight solutions in [0, 2p]. satisfying the equation cos4 2q + 2 sin2 2q = 17(sin q +
Answer: (C) cos q)
q 8 is
(A) 4 (B) 6 (C) 2 (D) 8
38. General solution of the equation 4 cot 2q + tan2 q =
Solution: Given equation is
cot2 q is
p p (1 - sin2 2q )2 + 2 sin2 2q = 17(1 + sin 2q )4
(A) np ± , n Î (B) np ± , n Î
4 3
p p Put x = sin 2q. Therefore
(C) 2 np ± , n Î  (D) 2 np ± , n Î 
3 6 (1 - x2 )2 + 2 x2 = 17(1 + x)4
Solution: The given equation can be written as
x4 + 1 = 17(1 + x)4 = 17(1 + 2 x + x2 )2 (3.1)
2(1 - tan q )
2
1
+ tan2 q = Clearly x = sin 2q = 0 is not a solution of the given equa-
tan q tan2 q tion. Therefore dividing both sides of Eq. (3.1) by x2 we get
2(1 - tan2 q ) + tan3 q 1 2
= 1 æ 1 ö
tan q tan2 q x2 + = 17 ç x + + 2÷
x2 è x ø
Therefore
x + (1/x
/ ) = y, we have
2(1 - tan q )tan q + tan q - 1 = 0
2 4

y2 - 2 = 17( y + 2)2
(1 - tan q )[2 tan q - (1 + tan q )] = 0
2 2

16 y2 + 68 y + 70 = 0
(1 - tan q )(1 - tan q ) = 0
2 2

8 y2 + 34 y + 35 = 0
(1 + tan q )(1 - tan q )(1 - tan q ) = 0
2
(4 y + 7)(2 y + 5) = 0
(1 - tan q )3 (1 + tan q ) = 0

This gives
-7 -5
y= or
tan q = ± 1 4 2
p p Case 1: If y = -7/4, then 4x2 + 7x + 4 = 0 has no real roots.
q = np + , q = np -
4 4 Case 2: If y = -5/2, then
p
q = np ± 2 x2 + 5 x + 2 = 0
4
Answer: (A) -1
x= , -2
2
39. The number of distinct real roots of the equation Now sin 2q = x = -2 is not possible. Therefore
sin x cos x cos x sin 2q = x = - 1/ 2
cos x sin x cos x = 0
Þ q = (np / 2) + (-1)n (7p / 12)
cos x cos x sin x
Now
in the interval -p/4
p £x£p
p/4 is
(A) 0 (B) 2 (C) 3 (D) 1 n = 0 Þ q = 7p / 12 = 105°

Solution: Given equation is n = 2 Þ q = p + (7p / 12) = 285°

(sin x + 2 cos x)(sin x - cos x) = 0


160 Chapter 3 Trigonometric Equations

n = 3 Þ q = (3p / 2) - (7p / 12) = 165° Therefore


n = 5 Þ q = (5p / 2) - (7p / 12) = 345° 8 mp 2p
x= + and x = 2kp
5 5
Therefore there are total four roots.
Answer: (A) which gives

æ 8p ö 2p
41. If x is not a multiple of p and satisfies the equation 2kp = ç ÷ m +
è 5ø 5
sin 5x cos 3x = sin 6x cos 2x, then x is
p np p np so that
(A) + , n Î (B) + , n Î
6 3 3 6 4m + 1
p p k=
(C) + np , n Î  (D) np + , n Î  5
6 3
Since k is an integer, we can take m = 5n + 1. Therefore
Solution: From the given equation we have
x = 2p + 8p n = 2p(4 n + 1), n Î 
1 1 Answer: (B)
(sin 8 x + sin 2 x) = (sin 8 x + sin 4 x)
2 2
43. General solution of the equation sin4 2x + cos4 2x =
Therefore
sin 2x cos 2x is
sin 2 x = sin 4 x p p
(A) (2 n + 1) , n Î  (B) (4 n + 1) , n Î 
8 8
2 cos 3 x sin x = 0
p p
Now (C) (2 n + 1) , n Î  (D) (4 n + 1) , n Î 
6 3
x ¹ np , n Î  Þ cos 3 x = 0 Solution: We have
p
Þ 3 x = (2 n + 1) (sin2 2 x + cos2 2 x)2 - 2 sin2 2 x cos2 2 x = sin 2 x cos 2 x
2
np p 2 sin2 2 x cos2 2 x + sin 2 x cos 2 x - 1 = 0
Þx= +
3 6 Put sin 2x cos 2x = t. Therefore
Answer: (A)
2t 2 + t - 1 = 0

42. General solution of the equation (2t - 1)(t + 1) = 0


1
æ x ö æ x ö t= , -1
çè cos - 2 sin x÷ø sin x + çè 1 + sin - 2 cos x÷ø cos x = 0 2
4 4
Case 1: When t = -1 we have
is
sin 2 x cos 2 x = - 1
(A) (4n + 1)p,
p n Î (B) (4n + 1)2p,
p n Î
(C) (2n + 1)2p,
p n Î (D) (2n + 1)p,
p n Î Þ sin 4 x = - 2

Solution: Given equation can be written as which is impossible. Therefore t = -1 is to be rejected.


x x Case 2: When t = 1/2 we have
sin x cos + cos x sin - 2(sin2 x + cos2 x) + cos x = 0
4 4 sin 4 x = 1
5x Þ 4 x = 2 np + (p / 2)
sin + cos x - 2 = 0
4
np p p
Þx= + = (4 n + 1)
Since maximum values of sin 5x/4 and cos x are 1, it 2 8 8
follows that sin 5x/4 = 1 and cos x = 1. Therefore
Answer: (B)
5x p
= 2 mp + , m Î  44. The number of values of x in the range 0 £ x £ 2p
4 2
satisfying the equation tan x + tan[(p/4)
p + x] = 2 is
x = 2kp,
p k Î (A) 2 (B) 4 (C) 6 (D) 1
Worked-Out Problems 161

Solution: We have Solution: The equation can be written as


tan x 3(sin2 q + cos2 q )(sin q + cosq ) - 2(sin3 q + cos3 q ) = 8
tan x + 1 + =2
1 - tan x
sin3 q + cos3 q + 3 sin2 q cos q + 3 sin q cos2 q = 8
tan2 x - 4 tan x + 1 = 0
Therefore
tan x = 2 ± 3 (sin q + cosq )3 = 8 = 23
x = 15°, 75°, 195°, 255°
Therefore sin q + cos q = 2. Hence sin q = 1 and cos q = 1,
Answer: (B) which is impossible and so the initial equation has no
real roots.
45. Number of solutions of the equation Answer: (A)
sin3 x cos x + sin2 x cos2 x + cos3 x sin x = 1
48. If real numbers x and y satisfy the equations 4 tan x =
p is
in the interval [0, 2p] tan y and 5 sin x cos y = 1, then
(A) 0 (B) 2 (C) 4 (D) 8 (A) x + y = (2 n + (-1)n )p / 2 and x - y =
Solution: The given equation can be written as kp + (-1)k Sin-1 (-3 / 5)
x + y = (2 n + 1)p / 2 and x - y = kp + Sin-1 (3 / 5)
sin x cos x(sin2 x + cos2 x) + sin2 x cos2 x = 1
(C) x + y = (n + 1)p / 2, x - y = np / 2
Substituting t = sin x cos x in this equation, we get (D) x + y = 2 np, x - y = (k + 1) Sin-1 (3 / 5)

t2 + t + 1 = 0 Solution:

The equation t2 + t + 1 = 0 has no real roots. tan x 1


=
Answer: (A) tan y 4
sin x cos y 1
Þ =
46. The number of solutions of the equation 3 sin2 x - cos x sin y 4
7 sin x + 2 = 0 in [0, 5p]
p is
4 æ 1ö
(A) 0 (B) 5 (C) 6 (D) 10 Þ cos x sin y = ç∵ sin x cos y = ÷
5 è 5ø
Solution:
x cos y = 1/5 and cos x sin y = 4/5,
3 sin2 x - 6 sin x - sin x + 2 = 0 we have
3 sin x(sin x - 2) - 1(sin x - 2) = 0 sin( x + y) = 1 and sin( x - y) = - 3 / 5
(3 sin x - 1)(sin x - 2) = 0 Therefore
This gives sin x = 1/3 (since sin x ¹ 2). Therefore p
x + y = np + (-1)n
2
x = np + (-1)n Sin-1(1/3) for n = 0, 1, 2, 3, 4, 5.
(-3)
Answer: (C) and x - y = kp + (-1)k Sin-1
5
47. For the following equation Answer: (A)

3(sin q + cosq ) - 2(sin3 q + cos3 q ) = 8 49. The number of solutions of the equation

exactly one of the following statements is true. Which 1 + 3 tan2 q = (1 + 3 )tan q


one is it?
(A) The equation has no real solutions. p is
in [0, 3p/2]
(B) The equation has exactly one solution. (A) 2 (B) 3 (C) 1 (D) 4
(C) The equation has exactly two solutions in the Solution: The given equation is
interval [0, p].
p
1 + 3 tan2 q - (1 + 3 )tan q = 0
(D) The equation has infinitely many solutions.
( 3 tan q - 1)(tan q - 1) = 0
162 Chapter 3 Trigonometric Equations

Therefore ( x + 1)2 £ 0
1 Þ x = -1
tan q = 1,
3 Hence the given equation is
Hence tan2 ( y - 1) + cot2 ( y - 1) = 2
p 5p p 7p
q= , , , Substituting tan(y - 1) = a, we get
4 4 6 6
1
p
belong to [0, 3p/2]. a2 + - 2 = 0 where
a2
Answer: (D)
which gives
50. If tan2(x + y) + cot2(x + y) = 1 - 2x - x2, then
p (a2 - 1)2 = 0 Þ a2 = 1
(A) x = 1, y = np ± -1
4 Þa = ±1
p
(B) x = - 1, y = np ± + 1 Þ tan( y - 1) = ± 1
4
np p Þ y - 1 = np ± p / 4
(C) x = - 1, y = ± -1
2 4
np p æ pö
(D) x = 1, y = ± -1 Þ y = ç np ± ÷ + 1
2 4 è 4ø
where n Î. Therefore
Solution: Left-hand side is greater than or equal to 2. æ pö
This implies x = - 1 and y = ç np ± ÷ + 1
è 4ø
1 - 2 x - x2 ³ 2 Answer: (B)

Multiple Correct Choice Type Questions


1. If x and y are solutions of the equations x + y = - 90°, x - y = - 30°Þ x = - 60°, y = - 30°
Answers: (A), (B), (C), (D)
3 3
sin x sin y = and cos x cos y =
4 4
2. Some of the common roots of the equations cos 2x +
x, y (in degrees) are sin 2x = cot x and 2 cos2 x + cos2 2x = 1 are
(A) x = 60°, y = 30° (B) x = 30°, y = 60° p 3p -p p
(A) (B) (C) (D)
(C) x = -30°, y = -60° (D) x = -60°, y = -30° 4 4 4 2
Solution: Adding and subtracting the equations, we have Solution: We have
cos(x + y) = 0 cos 2 x + sin 2 x = cot x

cos( x - y) = 3 / 2 cos x
and Þ cos 2 x + 2 sin x cos x - =0
sin x
Therefore æ 1 - 2 si n 2 x ö
Þ cos 2 x - cos x ç =0
x + y = (2n + 1)p/2
p è sin x ÷ø
and x - y = 2np ± (p/6)
p Þ cos 2 x(1 - cot x) = 0 (3.2)

Now The second equation is

x + y = 90°, x - y = 30°Þ x = 60°, y = 30° 2 cos2 x + cos2 2 x = 1


x + y = 90°, x - y = - 30°Þ x = 30°, y = 60° Þ cos2 2 x + cos 2 x = 0
x + y = - 90°, x - y = 30°Þ x = - 30°, y = - 60° Þ cos 2 x(cos 2 x + 1) = 0 (3.3)
Worked-Out Problems 163

From Eqs. (3.2) and (3.3), the common roots are given x = np ± (p/2)
p implies cos x = 0 so that we can reject
by the equation cos 2x = 0. Therefore this value. Therefore x = np ± (p/3)
p satisfy the equation
p f x) = g(x). But
f(
2 x = (2 n + 1)
2 h(x) = 2x+1 - 8 > 0 Û x > 2
p Therefore all (A), (B), (C) and (D ) are true.
or x = (2 n + 1)
4 Answers: (A), (B), (C), (D)
Answers: (A), (B), (C)
4. The values of q Î(0, p
p/2) and satisfying the equation

3. Let 1 + sin2 q cos2 q 4 sin 4q


sin2 q 1 + cos2 q 4 sin 4q = 0
1
f ( x) = sin2 q cos2 q 1 + 4 sin 4q
tan x - 1
2

g( x) = 1 + cos 2 x are
7p 5p 11p p
and h(x) = 2x+1 - 8 (A) (B) (C) (D)
24 24 24 24
Then
Solution: Adding column 2 to column 1 we get
æpö æpö æpö
(A) f ç ÷ = g ç ÷ and h ç ÷ < 0
è 3ø è 3ø è 3ø 2 cos2 q 4 sin 4q
2 1 + cos2 q 4 sin 4q = 0
æ -p ö æ -p ö æ -p ö
(B) f ç = gç and h ç <0
è 3 ÷ø è 3 ÷ø è 3 ÷ø 1 cos2 q 1 + 4 sin 4q

æpö æpö æpö Now R2 - R1, R3 - R1 implies


(C) f ç ÷ ¹ g ç ÷ and h ç ÷ < 0
è 6ø è 6ø è 2ø
2 cos2 q 4 sin 4q
æ 4p ö æ 4p ö æ 4p ö 0 1 0 =0
(D) f ç ÷ = g ç ÷ and h ç ÷ > 0
è 3ø è 3ø è 3ø -1 0 1

Solution: Consider the equation f( f x) = g(x). First, we Þ 1(2 + 4 sin 4q ) = 0


have that cos x ¹ 0. Domain of f(
f x) is the set of all real x -1 æ -p ö
for which tan x ¹ ±1. Þ sin 4q = = sin ç
2 è 6 ÷ø
f ( x) = g( x)
æ -p ö
Þ 4q = np + (-1)n ç , n Î
1 è 6 ÷ø
Þ = 1 + cos 2 x
tan2 x - 1 Therefore

- cos2 x np p
Þ = 1 + cos 2 x q= + (-1)n + 1
cos 2 x 4 24
is the general value of q. Therefore
-(1 + cos 2 x)
Þ = 1 + cos 2 x 7p
2 cos 2 x q= (when n = 1)
24
æ 1 ö
Þ (1 + cos 2 x) ç 1 + =0 11p
è 2 cos 2 x ÷ø and q= (when n = 2)
24
-1 Answers: (A), (C)
Þ cos 2 x = - 1 or cos 2 x =
2
5. Solution of the equation sinx + cosx - 2 2 sinx cosx =
2p 0 is
Þ 2 x = 2 np ± p , 2 x = 2 np ±
3 p
(A) x = np + (-1)n , n Î
p p 4
Þ x = np ± , x = np ± p
2 3 (B) 2 np ± , n Î
6
164 Chapter 3 Trigonometric Equations

p Dividing by cos2 x (clearly cos x ¹ 0), we have


(C) np - , n Î
4
tan3 x + tan2 x - 3 tan x - 3 = 0
p p
(D) np - + (-1)n + 1 , n Î  (using sec2 x = 1 + tan2 x). Clearly tan x = -1 is a root and
4 6
hence
Solution: Put sin x + cos x = y. Therefore
(tan x + 1)(tan2 x - 3) = 0
2 sin x cos x = y2 - 1 Therefore
The given equation is
tan x = - 1, tan x = ± 3
y - 2 ( y2 - 1) = 0
From this we get
Þ 2y - y - 2 = 0
2
p
x = np - , n Î
4
Þ ( y - 2 )( 2 y + 1) = 0
p
and x = np ± , n Î
3
-1 Answers: (B), (C), (D)
y = 2,
2
7. Solution of the equation 1 + sin x + cos 3x = cos x +
Case 1: When y = 2 we have cos 2x + sin 2x is
(A) x = np,
p n Î
sin x + cos x = 2 (B) x = 2np ± (p/3),
p n Î
Therefore (C) x = 2np ± (p/8)
p
(D) x = np + (-1)n+1(p/6),
p n Î
æ pö
sin ç x + ÷ = 1
è 4ø Solution: Given equation is
p 1 + sin x + cos 3x = cos x + cos 2x + sin 2x
Þ x = np + (-1)n , n Î
4 Solving this we get
Case 2: When y = -1/ 2 we have 1 + sin x - sin 2 x = cos x - cos 3x + cos 2 x

-1 1 + sin x - 2 sin x cos x = 2 sin 2 x sin x + 1 - 2 sin2 x


sin x + cos x =
2 sin x(1 - 2 cos x) = 4 sin2 x cos x - 2 sin2 x
Therefore sin x(1 - 2 cos x) = - 2 sin2 x(1 - 2 cos x)
æ p ö -1 sin x(1 - 2 cos x)(1 + 2 sin x) = 0
sin ç x + ÷ =
è 4ø 2
Therefore
p p
Þ x = np - + (-1)n+1 , n Î  sin x = 0
4 6
Answers: (A), (D) or cos x = 1/ 2
or sin x = -1/ 2
6. If x is a solutions of the equation sin2 x(tan x + 1) =
3 sin x(cos x - sin x) + 3, then x is Hence
p p
(A) np + , n Î  (B) np - , n Î  x = np, n Î 
2 4
p
p p or x = 2 np ± , nÎ
(C) np + , n Î  (D) np - , n Î  3
3 3
p
Solution: The given equation can be written as or x = np + (-1)n + 1 , n Î 
6
sin2 x tan x + sin2x - 3 sin x cos x + 3 sin2 x - 3 = 0 Answers: (A), (B), (D)
Worked-Out Problems 165

8. Let a be a solution of tan x + sin 2x = sec x. Then p


q ¹ np + , n Î
p 4
(A) a ¹ 2 np + , n Î 
2 p p
and q = np - = (4n - 1) , n Î 
æ 3 - 1ö 4 4
(B) a = np + (-1)n Sin-1 ç ÷ , n Î
è 2 ø If tan x ¹ ±1, then the given equation reduces to
p
(C) a = 2 np + , n Î 1 - tan2 x = 1 + tan2 x
2
Therefore
æ 3 - 1ö
-1
(D) a = np + (-1) Sin ç
n
÷ , n Î
è 2 2 ø 2 tan2 x = 0 Þ x = np
Hence
Solution: The given equation can be written as
q = np , n Î 
sin x + 2 sin x cos2 x = 1
Answers: (B), (C)
sin x + 2 sin x(1 - sin2 x) = 1
10. If cos(x + 1) sin 2(x + 1) = cos 3(x + 1) sin 4(x + 1), then
2 sin3 x - 3 sin x + 1 = 0
np np
(sin x - 1)(2 sin2 x + 2 sin x - 1) = 0 (A) x + 1 = , nÎ (B) x + 1 = , nÎ
2 3
Now np np
(C) x + 1 = , nÎ (D) x + 1 = (2 n + 1)
6 10
sin x = 1 Þ cos x = 0
Solution: The given equation can be transformed to
so that tan x and sec x are not defined. Therefore
p sin 3( x + 1) + sin( x + 1) = sin 7( x + 1) + sin( x + 1)
x ¹ (2 np ) +
2 sin 3( x + 1) = sin 7( x + 1)

and hence sin 7( x + 1) - sin 3( x + 1) = 0


2 cos 5( x + 1)sin 2( x + 1) = 0
-1 ± 3
sin x =
2 This gives
Since p
5( x + 1) = (2 n + 1) , n Î 
2
-1 - 3
< -1 or 2( x + 1) = np , n Î 
2
Answers: (A), (D)
we have
11. If
3 -1
sin x =
2 2 tan( x / 2)
= y2 - 4 y + 5
1 + tan2 ( x / 2)
æ 3 - 1ö
or x = Sin-1 ç ÷
è 2 ø then (x, y) can be
Answers: (A), (B) æp ö æ 5p ö
(A) ç , 2÷ (B) ç , 2÷
è2 ø è 2 ø
9. q be a solution of the equation (1 - tan x)
(1 + sin 2x) = 1 + tan x. Then æ p ö æ np ö
(C) ç (4 n + 1) , 2÷ , n Î  (D) ç , - 2÷ , n Î
p p è 2 ø è 2 ø
(A) q = (4 n + 1) (B) q = (4 n - 1)
4 4 Solution: Given equation is
p
(C) q = np p (D) q = np + , n Î  sin x = y2 - 4y + 5
8
Solution: Clearly tanx = 1 is not a solution while tanx = -1 y2 - 4y + 5 = (y - 2)2 + 1 ³ 1
is a solution. Therefore
166 Chapter 3 Trigonometric Equations

(equals to 1 if and only if y = 2). Therefore the minimum then x is equal to


value of right-hand side is 1, whereas the maximum value p p
of sin x is 1. (A) 2 np ± , n Î  (B) 2 np ±
4 6
p p p
sin x = 1 Û x = (4 n + 1) (C) np ± , n Î  (D) np ±
2 2 3

and in such a case y = 2. Solution: Given equation is


Answers: (A), (B), (C) 1
cos2 2 x = cos2 x -
2
12. If
Þ 2 cos2 2 x = cos 2 x
1 1 1
+ + =0 Þ cos 2 x(2 cos 2 x - 1) = 0
cos x cos 2 x cos 2 x cos 3 x cos 3 x cos 4 x

x are 1: cos 2x = 0. Therefore


p -p cos x = ±1/ 2
(A) (B)
3 3
p np But cos x ¹ -1/ 2 . Therefore cos x = +1/ 2 . So,
(C) np ± , n Î (D) , n Î
3 3 p
x = 2 np ± , nÎ
Solution: Suppose sin x ¹ 0. Then 4

sin x sin(2 x - x) Case 2: 2 cos 2x – 1 = 0. Therefore


T1 = =
sin x cos x cos 2 x sin x cos x cos 2 x cos 2 x = 1/ 2
1
= [tan 2 x - tan
n x] Þ cos x = ± 3 / 2
sin x
Þ x = 2 np ± p / 6
1
T2 = [tan 3 x - tan 2 x] Answers: (A), (B)
sin x
1 14. q in [0, 2p
p] satisfying the equation
T3 = [tan 4 x - tan x]
sin x
(sin 2q + 3 cos 2q )2 = 5 + cos(2q - p / 6)
Therefore are
T1 + T2 + T3 = 0 7p 3p
(A) q = (B) q =
12 4
Þ tan 4 x - tan x = 0
5p 19p
Þ sin 3 x = 0 (C) (D)
12 12
Þ 3 x = np , n Î  and n ¹ 3k, k Î  Solution: Given equation is
np 2
Þx= , n is an integer ¹ 3k, k Î  æ1 3 ö æ pö
3 4 ç sin 2q + cos 2q ÷ = 5 + cos ç 2q - ÷
è2 2 ø è 6ø
p
Þ x = kp ± , k Î 2
3 é p p ù æ pö
4 êsin sin 2q + cos cos 2q ú = 5 + cos ç 2q - ÷
Answers: (A), (B), (C) ë 6 6 û è 6ø

13. If
Therefore

cos2 2 x æ pö æ pö
= cos x -
1 4 cos2 ç 2q - ÷ = 5 + cos ç 2q - ÷
è 6ø è 6ø
cos x + (1/ 2 ) 2
Worked-Out Problems 167

Substituting 2q - (p/6)
p = a we get But

4 cos2 a - cos a - 5 = 0 sin 2 x ¹ 2


Therefore
(4 cos a - 5)(1 + cos a ) = 0
p p
Now, one solution is cos a = -1. Then x- = 2 np ±
4 4
p p
2q - = a = 2 np ± p x = 2 np +
6 2
7p or x = 2np,
p n Î
2q = 2 np +
6 Answers: (A), (C)
5p
or q = 2 np - f x) = tan x + cos 2x - 1. Then f(
16. Let f(
2
f x) = 0 has
6
p
(A) four solutions in (0, 5p)
Therefore
(B) two solutions in (0, p)
p
7p p
(C) two solutions in (p, 3p/2)
q = np +
12 (D) one solution in (0, p/2)
p
5p Solution:
or q = np -
12
f ( x) = 0
Hence
Þ tan2 x - 2 sin2 x = 0
7p 19p
q= (n = 0) and q = (n = 1) Þ sin2 x(1 - 2 cos2 x) = 0
12 12
Answers: (A), (D) Þ sin x = 0, cos x = ± 1/ 2
p 3p
15. If sin3 x + cos3 x + sin x cos x = 1, then x is equal to Þ x = np or x = 2 np ± , 2 np ±
4 4
(A) 2np, n Î (B) np, n Î Answers: (A), (B), (C), (D)
p p
(C) 2 np + , n Î  (D) 2 np - , n Î 
2 2 17. If 2(cos x + cos 2x) + (1 + 2 cos x) sin 2x = 2 sin x, then
Solution: Given equation can be written as p p
some of the values of x in the interval [-p, p] are
p p -p -p
(A) (B) (C) (D)
(sin x + cos x)(sin2 x - sin x cos x + cos2 x) 3 4 3 2
- (1 - sin x cos x) = 0 Solution: Given that
(sin x + cos x)(1 - sin x cos x) - (1 - sin x cos x) = 0 2 cos x + 2 cos 2 x + sin 2 x + 2 cos x sin 2 x - 2 sin x = 0
(sin x + cos x - 1)(1 - sin x cos x) = 0 2 cos x + 2 cos 2 x + 2 sin x cos x + 4 sin x cos2 x - 2 sin x = 0

Therefore 2 cos x + 2 cos 2 x + 2 sin x cos x + 4 sin x(1 - sin2 x)

sin x + cos x = 1 - 2 sin x = 0

or 1 - sin x cos x = 0 2 cos x + 2 cos 2 x + 2 sin x cos x + 2 sin x - 4 sin3 x = 0

This gives 2 cos x + 2 cos 2 x + 2 sin x cos x + 2 sin x


- (3 sin x - sin 3 x) = 0
æ pö 1
cos ç x - ÷ =
è 4ø 2 2 cos x + 2 cos 2 x + 2 sin x cos x + sin 3 x - sin x = 0

or sin 2x = 2 2 cos x + 2 cos 2 x + 2 sin x cos x + 2 cos 2 x sin x = 0


168 Chapter 3 Trigonometric Equations

cos x(1 + sin x) + cos 2 x(1 + sin x) = 0 provided


(1 + sin x)(cos x + cos 2 x) = 0 a-4
-1 £ £1
æ 3x xö 2
(1 + sin x)2 ç cos cos ÷ = 0
è 2 2ø Therefore equation possess solution if 2 £ a £ 6.
Therefore Case 1: When a = 5, then sin x = 1/2 so that

p p
1 + sin x = 0 Þ x = np + (-1)n + 1 , n Î x = np + (-1)n
2 6
x p is the general solution.
cos = 0 Þ x = 2(2 n + 1) = (2 n + 1)p , n Î 
2 2 Case 2: If a = 3, then sin x = -1/2 and hence x = -p/6
p is a
3x p solution.
cos = 0 Þ x = (2 n + 1) , n Î 
2 3 Answers: (B), (C), (D)

The values p
p/3 and -p/3
p correspond to n = 0, -1. When
19. If a ¹ np, n Î , then the equation sin 3a = 4 sin a
n = 0 in
sin(x + a)
a sin (x - a ) has
n+1 p
x = np + (-1) (A) a solution in the first quadrant
2
(B) no solution in the second quadrant
then x = -p/2.
p (C) a solution in the third quadrant
Answers: (A), (C), (D) (D) no solution in the fourth quadrant

18. If the equation cos 2x + a sin x = 2a - 7 has solutions,


Solution: a ¹ np Þ sin a ¹ 0. Therefore
then sin 3a
= 4(sin2 x - sin2 a )
(A) 0 < a < 2 sin a
(B) 2 £ a £ 6
p 3 - 4 sin2 a = 4 sin2 x - 4 sin2 a
(C) x = np + (-1)n , n Î , if a = 5
6
3
(D) x = -p/6
p is a solution, if a = 3 sin x = ±
2
Solution: The given equation is equivalent to
Therefore, x = 60°, 120°, 240° and 300° are solutions of
(1 - 2 sin2 x) + a sin x = 2a - 7 the equation in each of the quadrants.
2 sin2 x - a sin x - 8 + 2 a = 0 Answers: (A), (C)

Therefore 20. The equation cos x + sin x = 1 has


7 4

a ± a2 - 8(2a - 8) (A) exactly two solutions in 0 £ x £ p/2


p
sin x = (B) exactly one solution in p/2
p <x£p
4
(C) exactly one solution in p < x £ 3p/2
p
a ± (a - 8)2
= (D) p < x < 2p
no solution in 3p/2
4
Solution: Clearly x = 2np, n Î is a solution of
a + (a - 8) a - ( a - 8)
= or the given equation. Also x = (2k + 1)p/2,
p k Î is also a
4 4 solution. Further, the equation can be written as
a-4
= or 2 cos3 x(1 - sin2 x)2 = (1 - sin2 x)(1 + sin2 x)
2
But sin x ¹ 2. Therefore (1 - sin2 x)(cos5 x - 1 - sin2 x) = 0

a-4 sin x = ± 1 (∵ cos5 x ¹ 1 + sin2 x)


sin x =
2 Answers: (A), (C), (D)
Worked-Out Problems 169

Matrix-Match Type Questions


1. Some trigonometric equations are given in Column I (C) cos 3 x + sin 2 x - sin 4 x = 0
and some of their solutions are in Column II. Match
them. Þ cos 3 x - 2 cos 3 x sin x = 0
Þ cos 3 x(1 - 2 sin x) = 0
Column I Column II Þ cos 3 x = 0 or sin x = 1/ 2
p
(A) sin x + cos x = 2 (p) Case 1: cos 3x = 0. Then
6
p
9p 3 x = (2 n + 1) , n Î 
3 1 (q) 2
(B) cos x + sin x = 1 4
2 2 p
p Þ x = (2 n + 1)
(r) 6
4
(C) cos 3x + sin 2x - sin 4x = 0 n = 1 Þ x = p/2.
p
13p
(s)
6 Case 2: sin x = 1/2. Then
p p
(D) sin 5x cos 3x = sin 6x cos 2x (t) x = np + (-1)n
2 6
Answer: (C) Æ (p), (s), (t)
Solution:
(D) sin 5 x cos 3 x = sin 6 x cos 2 x
1 1
(A) sin x + cos x = 1 Þ sin 8 x + sin 2 x = sin 8 x + sin 4 x
2 2
Þ sin 4 x - sin 2 x = 0
æ pö p
sin ç x + ÷ = 1 = sin
è 4ø 2 Þ 2 cos 3x sin x = 0

Therefore Therefore
p p x = np,
p x = kp/3
p + p/6
p
x+ = np + (-1)n , n Î 
4 2 Answer: (D) Æ (p), (s), (t)
Now 2. Match the trigonometric equations of Column I with
p p p some of their solutions in Column II.
n=0Þx= - =
2 4 4
p 9p Column I Column II
n = 2 Þ x = 2p + =
4 4 p
(A) sin x + cos x = 1 + sin x cos x (p)
Answer: (A) Æ (q), (r) 2
p
3 1 æ pö x x (q)
1= cos x + sin x = sin ç x + ÷ (B) 2 sin cos2 x - 2 sin sin2 x = cos 2 x 6
2 2 è 3ø 2 2
p 5p
(r) ,
(C) sin2 x + sin2 2x = 1 4 3
p p
x + = np + (-1)n , n Î  (s) 0
3 2
1 p
Now (D) sin4 x - cos4 x = (t) ±
2 3
p p p
n=0Þx= - =
2 3 6 Solution:
p 13p (A) sin x + cos x = 1 + sin x cos x
n = 2 Þ x = 2p + = Put sin x + cos x = y. Therefore
6 6
Answer: (B) Æ (p), (s) y2 - 1
y = 1+
2
170 Chapter 3 Trigonometric Equations

y2 - 2 y + 1 = 0
( y - 1)2 = 0 p
x = (2 n + 1)
6
y=1
Now n = 0, 1 Þ x = p
p/6 and x = p/2.
p
Hence
Answer: (C) Æ (p), (q)
sin x + cos x = 1 (D) We have
æ pö 1
cos ç x - ÷ = sin4 x - cos4 x = 1/ 2
è 4 ø 2
1
p æ pö Þ sin2 x - cos2 x =
x = + ç 2 np ± ÷ 2
4 è 4ø 1
Þ cos 2 x = -
For n = 0, we have x = p
p/2, 0. 2
Answer: (A) Æ (p), (s) 2p
Þ 2 x = 2 np ±
(B) We have 3
p
x Þ x = np ±
2 sin cos 2 x = cos 2 x 3
2
Therefore n = 0 Þ x = ±p/3.
p
æ x ö
çè 2 sin - 1÷ø cos 2 x = 0 Answer: (D) Æ (t)
2
x 1 3. Column I consists of trigonometric equations and
sin = or cos 2 x = 0
2 2 Column II consists of their solutions. Match them.

Case 1: We have sin(x/2) = 1/2. This implies


Column I Column II
p
x = 2 np + (-1) n
3p
3 (A) cos 3x + sin 5x = 0 (p)
4
For n = 1, x = 5p/3.
p
(q) p
Case 2: We have cos 2x = 0. This implies (B) sin x cos 5x = sin 9x cos 3x
p
p (r)
x = (2 n + 1) 3
4
(C) cos 2x - cos 8x + cos 6x = 1 p
(s)
For n = 0, x = p/4.
p 8
Answer: (B) Æ (r) 3p
(D) 4 sin x sin 2x sin 3x = sin 4x (t)
(C) sin x + sin 2x = 1. Therefore
2 2
16
1 - cos 2 x 1 - cos 4 x
+ =1 Solution:
2 2
(A) We have
cos 4 x + cos 2 x = 0
cos 3 x + sin 5 x = 0
2 cos 3x cos x = 0
p p æp ö
x = (2n + 1) , x = (2 n + 1) cos 3 x + cos ç - 5 x÷ = 0
è2 ø
6 2
æ 3 x + (p / 2) - 5 x ö æ 3 x - (p / 2) + 5 x ö
2 cos ç
è ÷ø cos çè ÷ø = 0
2 2
ì p ü
x = í(2 n + 1) , n Î  ý æp ö æ pö
î 2 þ cos ç - x÷ cos ç 4 x - ÷ = 0
è4 ø è 4ø
is a subset of
Therefore
ì p ü p p
x = í(2 n + 1) , n Î  ý x- = (2 n + 1)
î 6 þ 4 2
Worked-Out Problems 171

p p 4. Match the entries of Column I with the entries of


or 4x - = (2 n + 1) , n Î 
4 2 Column II.
which gives
Column I Column II
3p np 3p
x = np + or x= + , n Î
4 4 16 (A) If tan x + sin x + tan x - sin x = (p) 0
3tan x then sinx value is 3
When (q)
(B) If 1 + |cos x| + cos x + |cos x| +  + ¥
2 3 2
n = 0 Þ x = 3p / 4 or 3p / 16
is 2 (0 < x < p
p/2) then cos x is equal to - 3
Answer: (A) Æ (p), (t) (r)
(B) The given equation is sinxcos5x = sin9x
9 cos3x. Solving æp ö æp ö 2
(C) If tan ç + x÷ + tan ç - x÷ = 2,
it we get è4 ø è4 ø 1
(s)
then tan x is equal to 2
sin 6 x - sin 4 x = sin 12 x + sin 6 x
-1
sin 12 x + sin 4 x = 0 (D) If 7 cos2 q + 3 sin2 q = 4, then sin q is (t)
2
2 sin 8 x cos 4 x = 0
Now two cases arise. Solution:
Case 1: sin 8x = 0. Then x = np/8.
p (A) Squaring both sides, we have
Case 2: cos 4x = 0. Then x = (2n + 1)p/8.
p tan x = 2 tan2 x - sin2 x
Also
4 sin4 x
n = 6 Þ x = 3p / 4 tan2 x = 4(tan2 x - sin2 x) =
cos2 x
n=8Þ x=p sin2 x(1 - 4 sin2 x) = 0
Answer: (B) Æ (p), (q)
sin x = 0 or ± 1/ 2
(C) The given equation is cos 2x - cos 8x + cos 6x = 1. So
Answer: (A) Æ (p), (s), (t)
2 sin 5 x sin 3 x = 1 - cos 6 x = 2 sin2 3 x p
(B) 0 < x < Þ |cosx| ¹ 1. Therefore the given sum equals
2
sin 3 x(sin 5 x - sin 3 x) = 0
1
sin 3 x(-2 cos 4 x sin x) = 0 =2
1 - | cos x |
Three cases arise. 1 - | cos x | = 1/ 2
Case 1: sin x = 0. Then x = np,
p n Î. | cos x | = 1/ 2
Case 2: sin 3x = 0. Then x = np/3,
p n Î. cos x = ± 1/ 2
Case 3: cos 4x = 0. Then x = (2n + 1)p/8,
p n Î. Answer: (B) Æ (s), (t)
Therefore x = p, p p/3,
p p p/8 are solutions. (C) We have
Answer: (C) Æ (q), (r), (s)
æp ö æp ö
(D) We have tan ç + x÷ + tan ç - x÷ = 2
è4 ø è4 ø
4 sin x sin 2 x sin 3 x = sin 4 x = 2 sin 2 x cos 2 x
Therefore
sin 2 x(2 sin x sin 3 x - cos 2 x) = 0
sin 2 x(cos 2 x - cos 4 x - cos 2 x) = 0 æp ö æp ö
tan ç + x÷ + cot ç + x÷ = 2
è4 ø è4 ø
sin 2 x cos 4 x = 0
Therefore The left-hand side is ³ 2. Therefore

np p æp ö
x= or x = (2 n + 1) tan ç + x ÷ = 1
2 8 è 4 ø
Hence x = p or x = p
p/8 are solutions. tan x = 0
Answer: (D) Æ (q), (s) Answer: (C) Æ (p)
172 Chapter 3 Trigonometric Equations

(D) We have Therefore

7(1 - sin2 q ) + 3 sin2 q = 4 -4 sin2 q = - 3


3
sin2 q =
4
Answer: (D) Æ (q), (r)

Comprehension-Type Questions
1. Passage: answer Case 2: cos 2x = 0. Therefore
questions (i), (ii) and (iii).
p
(i) If sin 5x sin 4x + cos 6x cos 3x = 0, then x is equal to 2 x = (2 n + 1)
2
ì p np ü ìp ü
(A) í + , n Î  ý È í + kp , k Î  ý From both cases we get
î4 2 þ î2 þ
ì p ü ì np p
(B) ínp + , n Î  ý È í
î 3 þ î 3
ü
+ , n Îý
4 þ
x= { np p
2 4 } { p
+ , n Î  È kp + , k Î 
2 }
p Answer: (A)
ì ü ì np p ü
(C) íkp + , k Î  ý È í + , n Îý (ii) We have sin 2x sin 6x = cos x cos 3x. Therefore
î 4 þ î 2 6 þ
ì p ü cos 4 x - cos 8 x cos 4 x + cos 2 x
(D) í2 np ± , n Î  ý =
î 8 þ 2 2
(ii) If sin 2x sin 6x = cos x cos 3x, then x can be cos 8 x + cos 2 x = 0
p p p p 2 cos 5 x cos 3 x = 0
(A) , (B) ,
8 4 12 10
p p p p Two cases arise.
(C) , (D) , Case 1: cos 3x = 0. Therefore
10 6 12 4
(iii) General solution of the equation cos 2x - cos 8x + p
x = (2 n + 1)
cos 6x = 1 is 6
ì pü ì pü
(A) ínp + ý È í(2n + 1) ý , n Î  Case 2: cos 5x = 0. Therefore
î 4þ î 3þ
p
ì p ü ì np ü x = (2k + 1)
(B) í(2 n + 1) ý È í ý , n Î  10
î 2þ î 8 þ
From both cases we get
ì np ü ì pü
(C) í ý È í(2 n + 1) ý , n Î 
î þ î
3 4þ ì p ü ì p ü
x = í(2 n + 1) , n Î  ý È í(2k + 1) , k Î  ý
ì p ü ì np ü î 6 þ î 10 þ
(D) í(2 n + 1) ý È í ý , n Î 
î 8þ î 3 þ If n = 0 and k = 0, then x = p/6,
p p p/10, respectively.
Solution: Answer: (C)
(i) We have (iii) We have cos 2x - cos 8x + cos 6x = 1. Therefore

sin 5 x sin 4 x + cos 6 x cos 3 x = 0 2 sin 5 x sin 3 x = 1 - cos 6 x = 2 sin2 3 x


cos x - cos 9 x cos 9 x + cos 3 x sin 3 x(sin 5 x - sin 3 x) = 0
+ =0
2 2 sin 3 x(2 cos 4 x sin x) = 0
cos x + cos 3x = 0
2 cos 2 x cos x = 0 This gives either sin x = 0 or sin 3x = 0 or cos 4x = 0.
Case 1: sin x = 0 implies sin 3x = 0. Therefore
Two cases arise.
Case 1: cos x = 0. Therefore np
x=
3
p
x = (2k + 1) , k Î 
2
Worked-Out Problems 173

p (t - 1)(t + 3) = 0
Case 2: cos 4x = 0. This implies x = (2 K + 1)
8
t = 1, - 3
Hence
Case 1: t = 1. Therefore
ì p ü ì np ü
x = í(2k + 1) , k Î  ý È í , n Î  ý sin x + cos x = 1
î 8 þ î 3 þ
æ pö 1 p
Answer: (D) sin ç x + ÷ = = sin
è 4 ø 2 4
2. Passage: R(sin x + p p
cos x, sin x cos x) = 0, where R is a rational function of x+ = kp + (-1)k
4 4
sin x + cos x and sin xcos x then substitution t = sin x +
cos x can be used, so that sin x cos x = (tt2 - 1)/2. Answer If k is even, then x = 2mp,
p m Î.
the following questions (i), (ii) and (iii). 2. If k is odd, then so that x = 2np + (p/2).
p
(i) If sinx + cosx + sinxcosx = 1, then the general value Therefore the solution set is
off x is
ì p ü
ì p ü {2 np , n Î } È í2kp + , k Î  ý
(A) í(2 n + 1) , n Î  ý È {kp , k Î } î 2 þ
î 2 þ
ì p ü Case 2: t = -3. That is sin x + cos x = -3. This equation
(B) {2kp , k Î } È í2np + , n Î  ý has no solution, because 3 > 2 . Recall that a sin x +
î 2 þ
b cos x = c is solvable only if
ì kp ü ì p ü
(C) í , k Î  ý È ínp + , n Î  ý |c|
î 2 þ î 4 þ £1
a + b2
2

ì p ü ì p ü
(D) í(2k + 1) , k Î  ý È í2 np + , n Î  ý Answer: (B)
î 2 þ î 4 þ
(ii) Which of the following values off x satisfy the equa- (ii) We have
tion sin x + cos x - 2 2 sin x cos x = 0. sin x + cos x = 2 2 sin x cos x = 0
p -p -p -p
(A) , (B) ,
2 4 2 4 Substituting sin x + cos x = t we get
p -5p p p t - 2 (t 2 - 1) = 0
(C) , (D) ,
4 12 6 3
(iii) The set of all solutions of the equation sinx + cosx - 2t 2 - t - 2 = 0
2sinxcosx = 1 is ( 2t + 1)(t - 2 ) = 0
(A)
t = 2 , - 1/ 2
ì 3p ü ì p np ü
íkp + , k Î  ý È í(4 n - 1) + (-1) (n Î )ý
î 4 þ î 4 4 þ Case 1: t = 2 . Therefore
ì p ü ì p ü sin x + cos x = 2
(B) íkp + , k Î  ý È ínp + , n Î  ý
î 4 þ î 6 þ
1 1
p p sin x + cos x = 1
ì ü ì ü
(C) í(2k + 1) , k Î  ý È ínp + , n Î  ý 2 2
î 3 þ î 3 þ
æ pö p
p sin ç x + ÷ = 1 = sin
ì ü è 4ø 2
(D) {2kp , k Î } È ínp + , n Î  ý
î 6 þ
p p
x+ = np + (-1)n , n Î
Solution: 4 2
(i) We have sinx + cosx + sinxcosx = 1. Put sinx + cosx = t. when n = 0, then x = p/4.
p
Therefore
Case 2: t = - 1/ 2 . This implies
t2 - 1
t+ =1 1 1 -1
2 sin x + cos x =
2 2 2
t 2 + 2t - 3 = 0
174 Chapter 3 Trigonometric Equations

Therefore p
x+ = np , n Î 
4
æ p ö -1 æ -p ö
sin ç x + ÷ = = sin ç
è 4ø 2 è 6 ÷ø x = np -
p
= (n - 1)p +
3p
4 4
p æ -p ö 3p
x+ = kp + (-1)k ç
4 è 6 ÷ø x = kp +
4
When k = 0, then x = -5p/12.
p Therefore x = p/4,
p -5p/12
p Case 2: t = 1. Therefore
are solutions.
Answer: (C) sin x + cos x = 1
(iii) We have æ pö 1 p
sin ç x + ÷ = = sin
è 4ø 2 4
sin x + cos x - 2 sin x cos x = 1
p p
Substituting sin x + cos x = t we get x+ = np + (-1)n , n Î 
4 4
t - t2 = 0 p p p p
x = np - + (-1)n = (4 n - 1) + (-1)n
t = 0, 1 4 4 4 4
Case 1: t = 0. Therefore Therefore the solution set is
sin x + cos x = 0 3p p p
ì ü ì ü
í kp + , k Î  ý È í(4 n - 1) + (-1)n , n Î  ý
æ pö î 4 þ î 4 4 þ
sin ç x + ÷ = 0
è 4ø Answer: (A)

Assertion–Reasoning Type Questions


In the following set of questions, a Statement I is given 2 is not an odd multiple of p
2. Statement I: If 2x p/2 and
and a corresponding Statement II is given just below it. 4 - tan2x
(tan4x 2 )/(1 + tan4x 2 ) = 1, then x = (kp/2)
4 tan2x p +
Mark the correct answer as: p
(p/8), k Î.
(A) Both Statements I and II are true and Statement II Statement II: General solution of the equation tan q =
is a correct explanation for Statement I tan a is q = np + a,
a n Î.
(B) Both Statements I and II are true but Statement II is
Solution: Statement II is true (see Section 3.2.5). Now
not a correct explanation for Statement I
(C) Statement I is true and Statement II is false tan 4 x - tan 2 x
=1
(D) Statement I is false and Statement II is true 1 + tan 4 x tan 2 x
p
1. Statement I: The solution set of cos
100
x - sin100 x = 1 Þ tan 2 x = 1 = tan
4
p n Î}.
is {np,
p
Statement II: -1 £ cos x £ 1 for all real values of x. Þ 2 x = kp +
4
Solution: We have p p
Þx=k +
cos100 x - sin100 x = 1 2 8
Answer: (A)
Þ cos100 x = 1 + sin100 x ³ 1
Þ cos100 x = 1 3. Statement I: The equation sin x = ex has a solution for
x ³ 0.
Þ sin x = 0
Statement II: -1 £ sin x £ 1 for all real x.
Þ x = np , n Î
Solution: Since ex ³ 1 for all x ³ 0 and | sin x | £ 1, the equa-
Answer: (A)
tion sin x = ex has no solution (see the following figure).
Worked-Out Problems 175

Y Statement II is false because cos x + sec x < 0 for infinitely


y = ex many values of x.
Answer: (C)

y = sin x 5. Statement I: General solution of the equation 5tan4 x -


1
(p /2,1) sec4 x - 29 = 0 is x = np ± (p/3),
p n Î.

0 p /2 2
X The equation a sin x + b cos x = c has
Statement II: T
solutions if | c | £ a2 + b2 .
Solution: According to Section 3.3.1, Statement II is true.
Now

5 tan4 x - sec4 x - 29 = 5 tan4 x - (1 + tan2 )2 - 29


Therefore Statement II is true, but Statement I is false.
Answer: (D) 4 tan4 x - 2 tan2 x - 30 = 0
Note: Since e ® 0 as x ® -¥, the graph of y = sin x inter-
x
2 tan4 x - tan2 x - 15 = 0
sects the graph of y = ex at infinitely many points for x < 0
and hence sin x = ex has infinitely many solutions when (2 tan2 x + 5)(tan2 x - 3) = 0
x < 0.
tan2 x = 3
4. Statement I: T
The general solution of the equation æ pö
cos x + sec x = 2 when x is not an odd multiple of p/2
p tan x = ± 3 = tan ç ± ÷
è 3ø
is x = 2np,
p n Î.
p
Statement II: cos x + sec x ³ 2 for all real values of x. x = np ±
3
Solution: Since the maximum value of cos x is 1 and
Statement I is also true, but Statement II is not a correct
| sec x| ³ 1, cos x + sec x = 2 if cos x = 1 and sec x = 1.
explanation of Statement I.
Therefore
Answer: (B)
x = 2np,
p n Î
Hence Statement I is true.

Integer Answer Type Questions


1. The number of solutions of the equation sin 3x + cos 3x = Solution: We have
0 in the interval [0, p
p] is . Therefore
Solution: As cos 3x = 0 is not a solution of the initial cos x = 2 sin2 ( x / 2) = 1 - cos x
equation, the given equation is equivalent to the equa-
tion tan 3x = -1 = tan(-p/4).
p Therefore Þ 2 cos x = 1

-p Þ cos x = 1/ 2
3x = + np , n Î 
4 Þ x = 2 np ± (p / 3), n Î 
-p np Therefore k = 3.
x= + , n Î
12 3 Answer: 3
When n = 1, 2, 3, respectively, then x = p
p/4, 7p
p/12 and
p
11p/12. 3. The general solution of the equation
Answer: 3
tan 2 x sin x + 3 (sin x - 3 tan 2 x) = 3 3
2. If the general solution of the equation cos x = is x = (3n - 1)p/
p k, n Î where k is equal to .
2 sin2(x/2) is given by x = 2np ± (p/
p k), n Î, then the
value of k is . Solution: The given equation can be written as
tan 2 x(sin x - 3) + 3 (sin x - 3) = 0
176 Chapter 3 Trigonometric Equations

As sin x ¹ 3, When n = 0, 1, 2, 3 and 4, then x = 0, p/2,


p p p, 3p
p/2 and 2p.
p
Answer: 5
æ -p ö
tan 2 x = - 3 = tan ç
è 3 ÷ø 7. For 0 < x, y < p
p, the number of ordered pairs (x, y)
satisfying the equations sin x + cos y = 0 and sin2 x +
Therefore
cos2 y = 1/2 is .
p p
2 x = np - = (3n - 1) Solution: We have
3 3
sin x + cos y = 0 (3.4)
p
x = (3n - 1)
6 1
sin2 x + cos2 y = (3.5)
2
k = 6.
0<x<p and 0<y<p (3.6)
Answer: 6
Substituting cos y = -sin x in Eq. (3.5) we get
4. The equation 3 sin x + 4 cos x = c is solvable if | c | value
is less than or equal to . 1
2 sin2 x =
2
Solution: It is known that (see Section 3.3.1) a sin x +
b cos x = c is solvable if | c | £ a2 + b2 . Therefore Therefore
1
sin x = ±
| c | £ 32 + 42 = 5 2
Answer: 5 p
x = np + (-1)n , n Î
6
5. If
æ -p ö
x = np + (-1)n ç , n Î
1 æp ö è 6 ÷ø
tan x - 1 +
3
- 3 cot ç - x÷ = 3
cos2 x è2 ø
p 5p
x= , (∵ 0 < x < p )
then the number of values of tan x is . 6 6
Solution: Given equation can be written as Now sin x = ±1/2 Þ cos y = ∓1/2. So

tan3 x - 1 + sec2 x - 3 tan x - 3 = 0 1 -1


sin x = Þ cos y =
2 2
tan3 x + tan2 x - 3 tan x - 3 = 0
2p
Þ y = 2 np ±
(tan x + 1)(tan x - 3) = 0
2
3
tan x = - 1, ± 3 2p
Þy= (∵ 0 < y < p )
Answer: 3 3
Between 0 to p
p, sin x cannot be -1/2 so that cos y ¹ 1/2.
p and
6. The number values off x belonging to interval [0, 2p] Therefore the ordered pairs (x, y) are
satisfying the equation sin 4x + cos 4x = cos4x
4 is .
æ p 2p ö æ 5p 2p ö
Solution: Given equation can be written as çè , ÷ and ç ,
6 3ø è 6 3 ÷ø
(1 - 2 sin2 x cos2 x) - cos 4 x = 0 Answer: 2
1 2
1- sin 2 x - cos 4 x = 0 8. The number of roots of the equation (1 - cos 2x 2 =
2 )sin 2x
2 p p
3 sin2 x which lie in the interval [-p, p/3] is .
1 (1 - cos 4 x)
1- - cos 4 x = 0 Solution: We have
2 2
4 - 1 + cos 4 x - 4 cos 4 x = 0 3
(1 - cos 2 x)sin 2 x = 3 sin2 x = (1 - cos 2 x)
2
3 cos 4 x = 3
Therefore
cos 4 x = 1
æ 3ö
np (1 - cos 2 x) ç sin 2 x - =0
x=
2
, n Î è 2 ÷ø
Summary 177

Two cases arise. is the principal solution. Hence


Case 1: cos 2x = 1. Then 2x = 2np or x = np.
p This implies 1
x=-p p, 0 when n = - 1, 0. k=
2
Case 2: sin 2 x = 3 / 2 . Then so that 2k = 1.
p Answer: 1
2 x = kp + (-1) ; n, k Î 
k

3
\ p 4p/9)
10. The number of values x lying in the interval (p/9, p
satisfying the equation 1 + cos10x 6 = 2cos2 8x
0 cos6x 8 +
kp p 2
sin 8x
8 is .
or x= + (-1)k
2 6 Solution: We have
When k = -2, -1, 0, 1, respectively, the n 1 + cos 10 x cos 6 x = 2 cos2 8 x + sin2 8 x
5p -2p p p
x=- , , , = cos2 8 x + 1
6 3 6 3
Therefore
-5p -2p p p
x = - p, , , 0, , 1 + cos 16 x
6 3 6 3 cos 10 x cos 6 x = cos2 8 x =
Answer: 6 2
2 cos 10 x cos 6 x = 1 + cos 16 x
9. x = Tan-1(-k) is a solution of the equation cos2 x - cos 16 x + cos 4 x = 1 + cos 16 x
2 cos x = 4 sin x - sin 2x, then the value of 2k is .
cos 4 x = 1
Solution: The given equation can be written as
4 x = 2 np , n Î 
cos x(cos x - 2) = 2 sin x(2 - cos x)
x = np / 2, n Î 
Therefore
Therefore for any value of n,
(cos x - 2)(cos x + 2 sin x) = 0
Now cos x ¹ 2 implies cos x + 2 sin x = 0. Clearly cos x = 0 æ p 4p ö
x Ïç ,
is not a solution of the initial equation. Hence tan x = -1/2. è 9 9 ÷ø
Therefore
Therefore the number of values off x Î(p/9,
p 4p p/9) is zero.
æ -1ö
-1
x = Tan ç ÷ Answer: 0
è 2ø

SUMMARY
Trigonometric Equations where
3.1 Principal Solution: Solutions of a trigometric equa- a b
p is called prin-
tion belonging to the interval [0, 2p] sin q = and cos q =
a +b
2 2
a + b2
2
cipal solution of the equation.
a Î [0, p]
p such that
3.2 (a) General solution of the eqution sin x = K where
-1 £ K £ 1 is x = np + (-1) a where sin a = K and
n c
cos a = , n Î
n Î. a + b2
2

(b) General solution of the equation cos x = K (-1 £


K £ 1) is x = 2np ± a where cos a = K and n Î. or x = np + (-1)na - q where
(c) General solution of the equation tan x = a where a
‘a’ is any real number is x = np + q where tan q = a. cos q =
a + b2
2

3.3 If a, b and c are real numbers, then the equation


b
a sin x + b cos x = c is solvable only when | c | £ a2 + b2 sin q =
c + b2
2
and the general solution is x = (2np ± a)a +q
178 Chapter 3 Trigonometric Equations

c p p
and sin a = , n Î , - £a £ 3.5 If a trigonometric equation is a rational function
a +b
2 2 2 2
of sin mx, cos nx, tan px, etc. and equals zero, where
Note: Writing m, n, p, etc. are positive integers, then the formulae
2 tan ( x / 2)
2 tan x / 2 sin x =
sin x = 1 + tan2 ( x / 2)
1 + tan2 x / 2
1 - tan2 ( x / 2)
1 - tan2 x / 2 cos x =
and cos x = 1 + tan2 ( x / 2)
1 + tan2 x / 2
2 tan( x / 2)
we can solve the equations. tan x =
1 - tan2 ( x / 2)
3.4 Useful substitution: In some trigonometric equa-
tion, the substitution sin x + cos x = t will be useful.
Example: The equation
Example: Solve sin x + cos x = 1 + sin x cos x.
(cos x - sin x) (2 tan x + sec x) + 2 = 0

can be solved by substituting tan(x/2) = t.

EXERCISES
Single Correct Choice Type Questions
1. The number of roots of the equation 5. General solution of the equation

cot 2 x cot x sin3 ( x / 2) - cos3 ( x / 2) 1


+ +2=0 = cos x
cot x cot 2 x 2 + sin x 3
p is
in the interval [0, 2p]
(A) 1 (B) 2 (C) 3 (D) 4 p p
(A) x = (4 n + 1) , n Î  (B) x = (2 n + 1) , n Î 
2 2
2. General solution of the equation sin 6x + 2 = 2 cos 4x is np
np p (C) x = (2 n + 1)p , n Î  (D) x = , n Î
(A) , n Î (B) np + , n Î 4
6 3
np p 6. General solution of the equation tan x sin x - cos x =
(C) , n Î (D) 2 np + , n Î  (1/2) sec x is
2 3
p p
(A) (2 n ± 1) , n Î  (B) (3n ± 1) , n Î 
3. The number of solutions of the equation tan x × tan 3x 2 2
3 3
× tan 4x = tan2 x - tan2 3x + tan 4x in the interval [0, p
p] is p p
(C) (4 n + 1) , n Î  (D) (2 n ± 1) , n Î 
(A) 0 (B) 2 (C) 3 (D) 4 6 6

4. General solution of the equation 7. If cos2 3x + sin2[(p/2)


p + 5x] = cos2(7x) + cos2(9x), then
12x
2 is equal to
(sin 19°)cot x + cot 3 x = sin2(2p - x) - cos(p - x)
p n Î
(A) 4np, p n Î
(B) 3np,
æp ö p n Î p n Î
´ sinç + x÷ (C) 2np, (D) np,
è2 ø
8. The number of solutions of the equation 1 - 2 sin x -
is
2 cos x + cot x = 0 (0 < x < 2p)
p is
np
(A) x = , n Î (A) 4 (B) 6 (C) 2 (D) 1
4
np 9. If cos 3x + sin[2x - (7p/6)]
p = -2, then x is of the form
(B) x = , n Î  and n ¹ 4k, k Î 
4 p p
np (A) (4 m + 1) , m Î  (B) (6 m + 1) , m Î 
(C) x = , n Î 3 3
2 p p
np (C) (3m + 1) , m Î  (D) (2 m + 1) , m Î 
(D) x = , n Î  and n ¹ 2k, k Î  3 3
2
Exercises 179

10. General solution of 1 + sin x + cos x = (3/2) sin 2x is


3 3
16. The number of solutions of the equation
ì p ü
(A) {(2 n + 1)p , n Î } È í(4k - 1) , k Î  ý æ ö æ ö
î 2 þ cosç 2 + 1 x÷ × cosç 2 - 1 x÷ = 1
è 2 ø è 2 ø
ì p ü
(B) {np , n Î } È í(2k - 1) , k Î  ý p p
in the interval [-p, p] is
î 2 þ
(A) 0 (B) 1 (C) 2 (D) 4
(C) {(4 n + 1)p , n Î } È {2kp , k Î }
17. The number of solutions of the equation
(D) {(3n + 1)p , n Î } È {kp , k Î }
æp ö æp ö
tan ç + x÷ = tan ç - x÷
11. If ( 3 / 2)sin x = cos x + cos2 x, then x is equal to è4 ø è4 ø
p
p or (2 n ± 1) , n Î 
(A) np in the interval (-¥, ¥) is
3
p (A) 0 (B) 2 (C) finite (D) infinite
(B) (2 n ± 1)p or (6 n ± 1) , n Î 
3
18. If the pair (x, y) satisfies both the equations sin2 x +
np p sin2 y = 1/2 and x - y = 4p/3,
p then
(C) or (2 n - 1) , n Î
2 3 7p -p
p (A) x = np + ,y= + np , n Î 
(D) 2npp or np + , n Î  6 6
3
5p p
(B) x = np + , y = np - , n Î 
12. The general value off x satisfying the equation 6 3
sin 2x + 5 sin x + 5cos x + 1 = 0 is 7p p
(C) x = np - , y = np + , n Î
p p 6 6
(A) (3n + 1) , n Î  (B) (2 n + 1) , n Î 
4 4 5p p
p p (D) x = np - , y = np + , n Î 
(C) (4 n - 1) , n Î  (D) (6 n + 1) , n Î  6 3
4 4
19. (x, y) satisfies the system of equations tanx + tany = 4
13. The number of solutions of the equation 2 sin x = 3
and cotx + coty = 5, then
p 5p
cos x in the interval (-p, p) is (A) x = np + Tan-1 [(10 ± 4 5 )/ 5], y = mp + Tan-1 [(10 ∓
(A) 4 (B) 6 (C) 5 (D) 2 4 5 )/ 5], n and m Î  (either both have upper
signs or both have lower signs)
14. General solution of the equation sin x + cos x = 2
(sin3x + cos3x) belongs to (B) x = (np/2) p + Tan-1
p + Tan-1 [2 ± (0.8) 5 ], y = (mp/2)
[2 ∓ (0.8) 5 ], n, m Î 
ì p ü ì p ü
(A) í(2 n - 1) , n Î  ý È ínp + (-1)n , n Î  ý
î 2 þ î 4 þ 2 p + Tan-1 (±4 / 5 ), y = 2m
(C) x = 2n 2 p + Tan-1 (∓4 / 5 ),
n, m Î

ì np æ 1 ö ü ì p ü p + Tan-1 (2 ± 5 ), y = [(2m + 1)
(D) x = [(2n + 1)p/2]
í + (-1)n sin-1ç ÷ , n Î  ý È í(2 n + 1) , n Î  ý p + Tan-1 (2 ∓ 5 ), n, m Î
p/2]
î 2 è 2 2 ø þ î 2 þ
ì æ 1ö ü ì p ü p p
20. The number of solutions in the interval [-p, p] of the
(C) í2 np ± cos-1 ç ÷ , n Î  ý È í(2 n + 1) , n Î  ý equation 4 sin 3x + sin 5x - 2 sin x cos 2x = 0 is
î è 3 ø þ î 2 þ
(A) 8 (B) 7 (C) 6 (D) 4
(D)

ì -p
í + np , n Î  È
ý í
ì
ü ï np (-1)
+
n
sin -1
2- 2
,
(n Î 
ü
ï
ý
) x + x2)] = - 3 is
p )/(1 +
21. The number of values off x such that tan[(2px

î 4 þ ï 2 2 2 ïþ
î (A) 8 (B) 7 (C) 5 (D) 6

15. If a > 0, the number of solutions of the equation sin x = 22. The general solution of the equation cos 2x + sin 2x =
(| x |/2) + a in the interval (-2, 0) is cos x + sin x belongs to
(A) infinite (B) 4 (C) 2 (D) 0 ì p ü
(A) {2 np , n Î } È í(4m + 1) , m Î  ý
î 6 þ
ì p ü
(B) {np , n Î } È í(2 m + 1) , m Î  ý
î 6 þ
180 Chapter 3 Trigonometric Equations

ì p ü 27. The smallest positive value off x (in degrees) for


(C) {np , n Î } È í(4 m + 1) , m Î  ý
î 3 þ which tan(x + 100°) = tan(x + 50°)tan x tan(x - 50°) is
ì np ü ì p ü (A) 30° (B) 45° (C) 55° (D) 60°
í , n Î  ý È í(2 m + 1) , m Î  ý
î 2 þ î 3 þ
28. The smallest positive value of p for which the equa-
( sin x) = sin(p
tion cos(p ( cos x) has a solution is
23. The number of solutions of the equation 2 sin x +
cot x = 0 in the interval (-p,
p 2p
p) is p p p p
(A) (B) (C) (D)
(A) 6 (B) 4 (C) 3 (D) 2 2 3 3 2 2 2

24. The number of solutions of the equation 2(sin3 x + 29. The number of integral values of k for which the
cos x) - 3(sin x + cos x) = 8 in the interval (-p,
3
p pp) is equation 7 cos x + 5 sin x = 2k + 1 has a solution is
(A) infinite (B) 4 (C) 2 (D) 0 (A) 4 (B) 8 (C) 10 (D) 12

25. The equation sin6 x + cos6 x = a has a solution if and 30. The number of solutions of the equation sin x +
only if sin 5x = sin 3x in the interval [0, p
p] is
1 1 (A) 6 (B) 5 (C) 4 (D) 3
(A) 1 £ a £ 2 (B) £a£
6 3
31. The number of solutions of the equation tan2 x =
1 1 1
(C) £ a £ 1 (D) £ a £ cos 2x - 1 in the interval [-p,
p pp] is
4 8 4
(A) 6 (B) 5 (C) 4 (D) 3
26. The number of solutions of the equation cos x + 7

sin4 x = 1 in the interval (-p,


p pp) is
(A) 2 (B) 3 (C) 4 (D) 5

Multiple Correct Choice Type Questions


1. Solution of the equation 1 + sin4 2x = cos2 6x is (C) the number of pairs (a, b) is 2
p n Î
(A) np/3, p n Î
(B) np/2, (D) the number of pairs (a, b
b) is 4
p n Î
(C) np, (D) (2n + 1)p/4,
p n Î
6. For the equation sin2 x + (1/4) sin2 3x = sin x sin2 3x
2. If 7 cos2 x + cos x sin x - 3 = 0, then x is (A) x = 13p/6
p is a solution
-1
(A) np + Tan (4/3), n Î (B) x = 0, p are the solutions
(B) np + (3p/4),
p n Î p p
(C) the number of solutions in the interval [-p, p] is 5
p + Tan-1(4/3), n Î
(C) (np/2) (D) x = p
p/2 is not a solution
p + (3p/4),
(D) (np/2) p n Î
7. For the equation tan x × tan 4x = 1
-sin2 x
3. If 2 + 1 = 3.2
cos 2 x
, then x may be equal to (A) the number of solutions in [0, p
p] is 5
p
(A) np (B) 2np (B) x = 3p/4
p is a solution
p
(C) np/2 (D) (2n ± 1)p/2(
p n Î) (C) x = 11p/4
p is not a solution
(D) x = (2n + 1)p/10
p is the general solution
4. If z = (3 + 2i sin q)/(1
q - 2i sin q
q), then
(A) z is pure real, if q = np,
p n Î p - x]cos[(p/3)
8. For the equation cosxcos[(p/3) p + x] = 1/4,
(B) z is pure real, if q = 2np,
p n Î (A) the number of solutions in [-p,p pp] is 3
(C) z is pure imaginary, iff q = np + (p/3),
p n Î p is 10
(B) the number of solutions in [0, 6p]
(D) z is pure imaginary, iff q = np - (p/3),
p n Î p is 30p
(C) the sum of all the solutions in [0, 6p]
(D) the general solution is x = 2np/3,
p n Î
5. If a and b are smallest positive values off x and y satis-
fying the equation 2(sinx + siny) - 2cos(x - y) = 3, then
f x) = sin4 x + 4(1 - sin2 x) -
9. Let f( cos4 x + 4(1 - cos2 x).
(A) sin[(a + b)/2]
b =1
Then
(B) cos[(a - b)/2]
b = 1/2
Exercises 181

f x) = 0 is np,
(A) general solution of f( p n Î (A) A + B = 30° (B) C = 150°
p p]
(B) the number of values off x belonging to [-p, p (C) A + B = 150° (D) C = 30°
f x) = 0 is 4
satisfying f(
(C) general solution of f(f x) = 0 is 2np,
p n Î 13. Consider the following two statements:

f x) = 0 belonging
(D) the sum of all the solutions of f( I. If 0 < x < p
p/2 and logsin
cos x + logsin x = 2, then x = p/4.
x cos x
p
p 2p
to [-2p, p] is 0 II. If a and b are any two angles such that sin a = sin b
and cos a = cos b, then a - b = 2np,
p n Î.
10. The region S consists of all the points (x, y) such that
x2 + y2 £ 100 and sin(x + y) ³ 0. Then which of the following are correct?
(A) S is cut by the lines of the form x + y = kp p for (A) I is true (B) II is false
some k Î (C) I is false (D) II is true
(B) if (x, y) belongs to the region S and lies between
14. Consider the following statements
two lines of the form x + y = kp, p then either
sin(x + y) > 0 or sin(x + y) < 0 P: The equation cos x + cos 3x = cos 2x has two solu-
(C) the area of the region S is 50p p square units tions in the interval (0, p/2).
p
p square units
(D) the area of the region S is 100p Q: If 1 + sinx×sin2x
2 = cosxcos2x2 , then x = 2np/3,
p n Î.
which of the following are correct?
11. Consider the equation (A) P is true
3-1 3+1 (B) Q is true
+ =4 2 (C) P is true, but Q is false
sin x cos x
(D) P is false, but Q is true
For this equation
(A) x = p/12,
p n Î, is a solution 15. Consider the equation 2 2 |cos x | = cosec x.
(B) x = 11p/36,
p n Î, is a solution p
(A) This has four solutions in [0, 2p]
(C) the number of solutions in (0, pp/2) is 2 p p]
(B) This has two solutions in [p/2, p
(D) 2x = np + (-1) (x + p/12)
n
p (C) Solutions of this equation belonging to [0, p]
p
form an AP in some order
12. In triangle ABC, the angles A and B satisfy the rela-
p 3p/2)
(D) This equation has no solutions in (p, p
tions 3sinA + 4cosB = 6 and 3cosA + 4sinB = 1. Then

Matrix-Match Type Questions


In each of the following questions, statements are given 1. Match the items in Column I with those in Column II.
in two columns, which have to be matched. The state-
ments in Column I are labeled as (A), (B), (C) and
Column I Column II
(D), while those in Column III are labeled as (p), (q),
(r), (s) and (t). Any given statement in Column I can -p 3p
have correct matching with one or more statements in (A) If cos 3x - cos 2x = sin 3x, (p) ,
then some of the values of 4 4
Column II. The appropriate bubbles corresponding to
the answers to these questions have to be darkened as x can be (q) -p,
p pp, 2p
illustrated in the following example. (B) If sin 3x - 4 sin x cos 2x = 0,
p 5p
then x can be (r) ,
Example: If the correct matches are (A) ® (p), (s); 4 4
(C) If sec2 x - tan2 x +
(B) ® (q), (s),(t); (C) ® (r); (D) ® (r), (t); that is if the
p + x) = cos 2x sec2 x,
cot(p/2 p n Î
(s) 2np,
matches are (A) ® (p) and (s); (B) ® (q), (s) and (t);
then x can be
(C) ® (r); and (D) ® (r), (t), then the correct darkening p
of bubbles will look as follows: (D) If tan x + sin(p + x) = (t) (4 n - 1) , n Î 
2 sin2 x/2, then x is 4
p q r s t
A 2. Some trigonometric equations are given in Column I
B and some or whole of their solutions are given in
C Column II. Match these.
D
182 Chapter 3 Trigonometric Equations

4. Certain trigonometric equations are given in Column I


Column I Column II
and their solution sets are given in Column II. Match
p these.
(A) If 0 £ x, y £ 2p
p and a and b (p)
are solutions of the equation 3
sin x + sin y = 2, then a + b is 5p Column I Column II
(q)
(B) If 4 cos2 x + 4 cos x - 3 = 0, then the 3 (A) 1 + sin4 x = cos5 x (p) Empty set
p is
value off x in the interval [0, 2p] 2p (B) sin4 x = 1 + cos6 x p n Î
(q) 2np,
(r) p
(C) The solution of the equation 3 (C) sin2 2x = 1 + cos4 x (r) 2 np + , n Î 
2 sin2 x + (4 - 3 )sin x - 2 3 = 0 in p 6
(s) (D) sin4 x = 2 sin2 x + 1 (s) (2n + 1)p/2,
p n Î
the interval [0, p
p] is 2
(D) Solutions of the equation (t) p
3 - tan x = tan(p / 3 - x) are 5. Match the items in Column I with those in Column II.

3. Certain trigonometric equations are given in Column I Column I Column II


and some of their solutions are given in Column II. (A) The number of solutions of the (p) 0
Match these. equation tan x + sec x = 2 cos x in the
p is
interval [0, 2p]
Column I Column II (B) The number of solution of the (q) 3
2 np equation
(A) sin2 x = sin 3x + (p) , n Î
cos x(cos x - 1) 3 tan 3 x - tan 2 x
=1 (r) 4
p 1 + tan 3 x tan 2 x
(q) np , (4k - 1)
1 + tan x 4 p is
in [0, 2p]
(B) = (sin x + cos x)2 p
1 - tan x (r) (2 n + 1) (C) The number of values off x in (s) 2
6 [0, p
p] satisfying the equation
tan 3x + cos 6x = 1 np p sin x + sin 3x + sin 5x = 0 is
(s) , (4k + 1)
3 12 (D) The number of values off x in (t) 6
p p satisfying the equation
[0, 3p/2]
(D) cos 4x + 2 cos2 x = 1 (t) (4k + 1) sin x(2 cos x + 1) = 0 is
4

Comprehension-Type Questions
1. Passage: Using trigonometric transformations, answer 8 -
(i) General solution set of the equation sin8x
the following questions. 6 = 3 (sin6x
cos6x 6 + cos 8x
8 ) is
(i) The number of solutions of the equation sin 2x ì p ü ì p ü
× sin 6x = cos x cos 3x in the interval [0, p
p/2] is (A) ínp + : n Î  ý È í2kp + : k Î  ý
î 2 þ î 6 þ
(A) 4 (B) 6 (C) 2 (D) 8
ì p p ü ì p ü
(ii) The number of solutions of the equation tan x + ín + : n Î  ý È í(4k + 1) : k Î  ý
tan(x + p/3)
p + tan(x + 2p/3)p = 3 in the interval [-p,
p î 7 12 þ î 4 þ
p] is
p ì p ü ì p ü
(C) ínp + : n Î  ý È íkp + : k Î  ý
(A) 2 (B) 3 (C) 4 (D) 6 î 6 þ î 2 þ
(iii) The number of solutions of the equation sin x + ì p ü ì p ü
sin 7x = sin 4x in the interval [0, p
p/2] is (D) í(2 n + 1) : n Î  ý È í(2k + 1) : k Î  ý
î 3 þ î 7 þ
(A) 2 (B) 3 (C) 4 (D) 6
(ii) The number of solutions of the equation cos 5x +
2. Passage: Equations of the form a cos x + b sin x = c can
sin 5x = 0 in the interval [0, p
p] is
be solved if | c | £ a2 + b2 , by substituting a = r cos q (A) 4 (B) 5 (C) 6 (D) 8
and b = r sin q or a = r sin q and b = r cos q. Based on (iii) The number of solutions of the equation sin x +
this, answer the following questions. cos x = ( 3 + 1)/2 in [0, p
p/2] is
(A) 4 (B) 3 (C) 2 (D) 1
Exercises 183

Assertion–Reasoning Type Questions


In the following set of questions, a Statement I is given 3. Statement I: cos x = 1/2 Þ x = 2np ± (p/3),
p n Î.
and a corresponding Statement II is given just below it.
Statement II: sin x = 1/2 Þ x = kp + (-1)k(p/6),
p k Î.
Mark the correct answer as:
(A) Both Statements I and II are true and Statement II 4. Statement I: tan x = 1 Û x = np + (p/4),
p n Î.
is a correct explanation for Statement I
tan 3 x - tan 2 x
(B) Both Statements I and II are true but Statement II is Statement II: = tan x for all x Î.
1 + tan 3 x tan 2 x
not a correct explanation for Statement I
(C) Statement I is true and Statement II is false
5. Statement I: If y > 0 and 0 £ x £ p
p/2 are such that
(D) Statement I is false and Statement II is true sin x + cos x = y + 1/ y , then x = p
p/4 and y = 1.
1. Statement I: cos x + sin x = 2 has solutions. Statement II: |sin x + cos x | £ 2 and y + (1/ y) ³ 2.
Statement II: a cos x + b sin x = c has solutions if | c | £
6. Statement I: For 0 £ x, y £ 2p,
p sin x + sin y = 2 Þ x +
a2 + b2 .
y = p.
p

2. Statement I: sin x + sin 2x + sin 3x = cos x + cos 2x + Statement II: | sin q | £ 1 for all q.
cos 3x has solutions.
Statement II: The sets {sin x, sin 2x, sin 3x} and {cos x,
cos 2x, cos 3x} are equal for all real values of x.

Integer Answer Type Questions


The answer to each of the questions in this section is a p satis-
2. The number of values off x in the interval (0, 5p)
non-negative integer. The appropriate bubbles below fying the equation 3 sin2 x - 7 sin x + 2 = 0 is .
the respective question numbers have to be darkened.
For example, as shown in the figure, if the correct answer 3. The number of solutions of the equation sin4 x - 2sin2 x -
to the question number Y is 246, then the bubbles under 1 = 0 in the intercal [0, 2p]
p is .
Y labeled as 2, 4, 6 are to be darkened.
f(x) = 2sinx[cosx + cos3x + cos5x + cos7x
4. Let f( 7 + cos9x
9 ].
X Y Z W Then the number of values off x satisfying the equation
0 0 0 0 f(x) = 1 in the interval [0, p
f( p] is .
1 1 1 1
5. The number of values off x satisfying the equation
2 2 2
2[cos2(x/2)]sin2 x = x2 + (1/x
/ 2), x £ p
p/9 is .
3 3 3 3
4 4 4 6. The number of solutions of the equation e
sin x
- e-sin x =
5 5 5 5 4 is .
6 6 6
7. The number of values off x satisfying the equation
7 7 7 7 2
2cos 2 x + 1 = 3.2- sin x
p is
in the interval [0, 2p] .
8 8 8 8
9 9 9 9 8. The number of solutions of the equation sin x +
2 sin 2x = 3 + sin 3x, 0 < x < p is .
1. If cos 2x + a sin x = 2a - 7 has real solutions, then the
9. Let 0 < x, y < p
p. Then the number of ordered pairs
sum of the minimum and maximum values of a is 2
x - 3 sec2 y
. (x, y) satisfying the system of equations 5cosec =1
and 22 cosec x + 3|sec y| = 64 is .
184 Chapter 3 Trigonometric Equations

ANSWERS
Single Correct Choice Type Questions
1. (D) 17. (D)
2. (C) 18. (A)
3. (D) 19. (A)
4. (B) 20. (B)
5. (A) 21. (C)
6. (B) 22. (A)
7. (D) 23. (C)
8. (A) 24. (D)
9. (B) 25. (C)
10. (A) 26. (B)
11. (B) 27. (A)
12. (C) 28. (D)
13. (B) 29. (B)
14. (D) 30. (A)
15. (D) 31. (D)
16. (B)

Multiple Correct Choice Type Questions


1. (B), (C) 9. (B), (D)
2. (A), (B) 10. (A), (B), (C)
3. (A), (B), (D) 11. (A), (B), (C), (D)
4. (A), (B), (C), (D) 12. (C), (D)
5. (A), (B), (C) 13. (A), (D)
6. (A), (B), (C), (D) 14. (A), (B)
7. (A), (C), (D) 15. (A), (C)
8. (A), (B), (C), (D)

Matrix-Match Type Questions


1. (A) ® (p), (s), (t), (B) ® (q), (s), (C) ® (q), (r), (s), (D) ® (r), (s)
2. (A) ® (t), (B) ® (p), (q), (C) ® (p), (r), (D) ® (p), (t)
3. (A) ® (p), (B) ® (q), (C) ® (s), (D) ® (r)
4. (A) ® (q), (B) ® (s), (C) ® (p), (D) ® (p)
5. (A) ® (q), (B) ® (p), (C) ® (q), (D) ® (r)

Comprehension-Type Questions
1. (i) (A); (ii) (D); (iii) (C) 2. (i) (B); (ii) (B); (iii) (C)

Assertion–Reasoning Type Questions


1. (D) 4. (C)
2. (C) 5. (A)
3. (B) 6. (A)

Integer Answer Type Questions


1. 8 6. 0
2. 6 7. 5
3. 0 8. 0
4. 5 9. 4
5. 0
Properties
p
Triangles
of
4
Contents
4.1 Relations Between
the Sides and
Trigonometric
Ratios of Angles of a
Triangle
4.2 Solution of Triangles
4.3 Properties of
Triangles

Worked-Out Problems
W
Summary
Exercises
Properties of Triangles

Answers

Triangles are one of the


most fundamental geometric
shapes and have a variety
of often studied properties
relating to their Interior
angles (angles on the inside)
and relationship between
measurement of the sides
and angles in a triangle.
186 Chapter 4 Properties of Triangles

The three sides and the three angles of a triangle are called the elements of the triangle. One important aspect of
trigonometry is to find all the elements of triangle when some of its sides and angles are known. The process of finding
all the unknown elements of a triangle with the help of known elements is called solving a triangle and the values of
the unknown elements are called solution of the triangle. It is known that when all the three angles are known, then
the triangle is not uniquely determined. For example, each angle in an equilateral triangle is p p/3 and an equilateral
triangle is determined by its side. In this chapter, we will be proving that a triangle is completely determined (i.e., all
its elements are known) if any three of the six elements of the triangle, with at least one side, are known. Further, we
discuss various methods and formulae for solving a triangle.

4.1 | Relations Between the Sides and the Trigonometric Ratios


of Angles of a Triangle
We derive certain important formulae relating the sides of a triangle and the trigonometric ratios of the triangle
which are useful in solving a triangle. First, let us fix certain notations. A triangle is usually denoted by DABCC or
simply ABC C if A, B and C are vertices of the triangle. The letters A, B and C also denote angles BAC, ABC and
BCA, respectively. The lengths of the sides opposite to A, B and C are denoted by a, b and c, respectively, as shown
in Figure 4.1.

B
c

FIGURE 4.1

Since the secondary school level, we are familiar with the following geometrical properties of the sides and angles of
a triangle ABC.
1. A + B + C = p = 180°. That is the sum of the three angles is p radians or 180°.
2. a < b + c; b < c + a and c < a + b. That is, the sum of any two sides is greater than the third side.
3. a, b and c are positive real numbers.

T H E O R E M 4.1 In any triangle ABC, the sines of the angles are proportional to opposite sides; that is
(S I N E
sin A sin B sin C
FORMULA) = = (4.1)
a b c

PROOF Consider angle C. We shall prove the theorem using the following three cases:
Case I: Suppose ABCC is a right-angled triangle (see Figure 4.2); say

p
C= = 90°
2
Then
b a
sin B = , sin C = 1 and sin A =
c c
4.1 Relations Between the Sides and the Trigonometric Ratios of Angles of a Triangle 187

c b

FIGURE 4.2 Case I.

Therefore
sin A 1 sin B sin C
= = =
a c b c
Case II: Suppose that each of the angles A, B, and C is acute. Then draw a perpendicular to BC
through A to meet BCC at D (see Figure 4.3). Now
AD AD
sin B = =
AB c
and hence
c sin B = AD
Also,
AD AD
sinC = =
AC b
and hence
b sin C = AD = c sin B
Therefore
sin C sin B
=
c b
Similarly
sin A sin B
=
a b
Therefore
sin A sin B sin C
= =
a b c

FIGURE 4.3 Case II.


188 Chapter 4 Properties of Triangles

Case III: Suppose that one of the angles is obtuse; say C is obtuse. Then A and B are acute angles
(Figure 4.4).

A
A

D c
b b

FIGURE 4.4 Case III.

Draw perpendicular from C touching AB at D and from A touching extended BC at E. Then


CD CD CD CD
sin A = = and sin B = =
CA b CB a
Therefore
b sin A = CD = a sin B
and hence
sin A sin B
=
a b
Also,
AE AE
sin B = =
AB c
AE AE
and sin C = sin(p - C ) = sin ECA = =
AC b
Therefore
b sin C = AE = c sin B
and hence
sin C sin B
=
c b
Thus
sin A sin B sin C
= =
a b c ■

T H E O R E M 4.2 In any triangle ABC,


(C O S I N E
b2 + c2 - a2
FORMULA) 1. cos A = or a2 = b2 + c2 - 2bc cos A
2bc
a2 + c2 - b2
cos B = or b2 = a2 + c2 - 2ac cos B
2ac
a2 + b2 - c2
cos C = or c2 = a2 + b2 - 2ab cos C
2ab
4.1 Relations Between the Sides and the Trigonometric Ratios of Angles of a Triangle 189

PROOF Consider angle A. If A is a right angle (Figure 4.5), then cos A = 0. Clearly a2 + b2 = c2 implies

a2 = b2 + c2 - 2 bc cos A
Therefore, we can suppose that A ¹ 90°. Now draw a perpendicular to CA through B to meet CA
at D (extend CA if necessary). Two cases arise:
Case I: If A is acute (as in Figure 4.6), then
AD AD
cos A = = or AD = c cos A
BA c
BD BD
and sin A = = or BD = c sin A
BA c
and therefore
CD = AC - AD = b - c cos A
Case II: If A is obtuse (as in Figure 4.7), then
BD BD
sin A = sin(p - A) = sin DAB = =
AB c
or BD = c sin A

a
c

FIGURE 4.5

FIGURE 4.6 Case I.

FIGURE 4.7 Case II.


190 Chapter 4 Properties of Triangles

and
AD AD
cos A = - cos(p - A) = - cos DAB = - =-
AB c
or AD = - c cos A
Hence
CD = CA + AD = b - c cos A
From both Cases I and II, we have
CD = b - c cos A and BD = c sin A
From the triangle BCD, we have

a2 = BC 2

= CD2 + BD2

= (b - c cos A)2 + (c sin A)2

= b2 + c2 cos2 A - 2bc cos A + c2 sin


n2 A

= b2 + c2 (cos2 A + sin2 A) - 2bc cos A

= b2 + c2 - 2bc cos A
This prove the formula (1). Similarly, formulae (2) and (3) also can be proved. ■

Try it out Show that


a + c2 - b2
2
1. cos B = or b2 = a2 + c2 - 2ac cos B
2ac
a2 + b2 - c2
2. cos C = or c2 = a2 + b2 - 2ab cos C
2ab

Example 4.1

If ABCC is a triangle, such that a = 36, b = 15 and c = 39, B


then verify the sine formula.

a c
Solution: We have

b2 + c2 - a2 (15)2 + (39)2 - (36)2 450 5


cos A = = = =
2bc 2 ´ 15 ´ 39 1170 13
FIGURE 4.8 Example 4.1.
c2 + a2 - b2 (39)2 + (36)2 - (15)2 2592 12
cos B = = = =
2ca 2 ´ 39 ´ 36 2808 13 CA b 15 5
cos A = = = =
BA c 39 13
a2 + b2 - c2 (36)2 + (15)2 - (39)2
cos C = = =0 CB a 36 12
2ab 2 ´ 36 ´ 15 cos B = = = =
AB c 39 13
That is, C is a right angle and ABC C is a right-angled CB a 36 12
triangle (Figure 4.8). Note that, in this case sin A = = = =
AB c 39 13
b 5
sin B = =
c 13
4.1 Relations Between the Sides and the Trigonometric Ratios of Angles of a Triangle 191

Therefore which implies


sin A 12 1 1 sin A sin B sin C
= = = = =
a 13 36 39 a b c
sin B 5 1 1 Hence the sine formula [Eq. (4.1)] is verified.
= = =
b 13 15 39
sin C 1
=
c 39

Example 4.2

Let ABC C be a triangle such that a = 2, b = 3 and c = 4. 2


æ 7ö 15
Verify the sine formula. sin A = 1 - cos2 A = 1 - ç ÷ =
è 8ø 8
Solution: From the given data we get 2
æ 11 ö 3 15
sin B = 1 - cos2 B = 1 - ç ÷ =
b2 + c2 - a2 è 16 ø 16
cos A =
2bc 2
æ -3 ö 3 15
3 +4 -2
2 2
72
sin C = 1 - cos2 C = 1 - ç ÷ =
= = è 12 ø 12
2´3´4 8
Therefore
c2 + a2 - b2
cos B =
2ca sin A 15 1 15
= × =
4 +2 -3
2 2 2
11 a 8 2 16
= =
2´4´2 16 sin B 3 15 1 15
= × =
a +b - c
2 2 2 b 16 3 16
cosC =
2ab sin C 3 15 1 15
= × =
2 +3 -4
2 2
-3
2
c 12 4 16
= =
2´2´3 12 Therefore the sine formula [Eq. (4.1)] is verified.
Note that A and B are acute angles and C is an obtuse
angle, since cos C < 0. Now

T H E O R E M 4.3 In any triangle ABC,


(P R O J E C T I O N 1. a = b cos C + c cos B
FORMULAE)
2. b = c cos A + a cos C
3. c = a cos B + b cos A
PROOF We shall prove formula (1) in each of the following three cases.
Case I: Suppose that C = 90°. See Figure 4.9.

c b

FIGURE 4.9 Case I.


192 Chapter 4 Properties of Triangles

Then,
BC a
cos B = =
AB c
and hence a = cos B. This can be written as
a = b cos C + c cos B
since cos C = 0.
Case II: Suppose that C is an acute angle. Draw a perpendicular to BC
C through A to meet BC
at D (Figure 4.10). Then
BD BD
cos B = =
AB c
and therefore BD = c cos B. Also,
DC DC
cosC = =
AC b
and hence DC = b cos C.
Therefore
a = BC = BD + DC = c cos B + b cos C

c b

FIGURE 4.10 Case II.

Case III: Suppose that C is an obtuse angle. Here again, draw a perpendicular through A to
extended BCC meeting it at D (Figure 4.11). Then
BD BD
cos B = =
BA c
Therefore
BD = c cos B
Also,
CD CD
cos ACD = =
AC b
Now,
cos C = cos(p - ACD)
= - cos ACD
CD
=-
b
4.1 Relations Between the Sides and the Trigonometric Ratios of Angles of a Triangle 193

Therefore CD = -b cos C. Hence


a = BC
= BD - CD
= c cos B - (- b cos C )
= c cos B + b cos C

c b

FIGURE 4.11 Case III.

Thus, in each of the above cases, a = c cos B + b cos C.


Similarly we can prove formulae (2) and (3). ■

Try it out Prove the following:


1. b = c cos A + a cos C
2. c = a cos B + b cos A

Remark: The above formulae can be proved by using the cosine formulae given in Theorem 4.2. Assuming the cosine
formulae in Theorem 4.2, we get that
a2 + b2 - c2 c2 + a2 - b2 2a2
b cos C + c cos B = + = =a
2a 2a 2a

Similarly the other formulae in Theorem 4.3 can be proved. Conversely, by assuming the formulae in Theorem 4.3, we get
a2 + b2 - c2 = a(b cos C + c cos B) + b(c cos A + a cos C ) - c(a cos B + b cos A)
= 2ab co
osC
and similarly the other cosine formulae can be proved.
In the following, we derive formulae which give sine of half the angles of a triangle in terms of the sides. If
s denotes half of the perimeter of the triangle then
a+b+c
s= or 2 s = a + b + c
2
We will use the above-mentioned formulae for s in the rest of the chapter.

T H E O R E M 4.4 In any triangle ABC


A ( s - b)( s - c)
1. sin =
2 bc
B ( s - c)( s - a)
2. sin =
2 ca
C ( s - a)( s - b)
3. sin =
2 ab
194 Chapter 4 Properties of Triangles

PROOF We have
A
cos A = 1 - 2 sin2
2
and hence
A
2 sin2 = 1 - cos A
2

b2 + c2 - a2
=1- (by the cosine formula)
2bc

2bc - (b2 + c2 - a2 )
=
2bc

a2 - (b2 + c2 - 2bc)
=
2bc

a2 - (b - c)2
=
2bc
[a + (b - c)][a - (b - c)]]
=
2bc
(a + b - c)(a + c - b)
=
2bc
2( s - c)× 2( s - b)
= (since 2 s = a + b + c)
2bc
Therefore

A ( s - b)( s - c) A ( s - b)( s - c)
sin2 = or sin =
2 bc 2 bc

Similarly,

B ( s - c)( s - a) C ( s - a)( s - b)
sin = and sin =
2 ca 2 ab ■

Note: The sine of any angle of a triangle is always positive (since sin q > 0 for all 0 < q < p
p) and hence, in the above
formulae, we have to take positive square roots only.

T H E O R E M 4.5 In any triangle ABC,


A s( s - a)
1. cos =
2 bc
B s( s - b)
2. cos =
2 ca
C s( s - c)
3. cos =
2 ab
4.1 Relations Between the Sides and the Trigonometric Ratios of Angles of a Triangle 195

PROOF Since
A
cos A = 2 cos2 -1
2
we have
A
2 cos2 = 1 + cos A
2
b2 + c2 - a2
=1+
2bc
2bc + b2 + c2 - a2
=
2bc
(b + c)2 - a2
=
2bc
(b + c - a)(b + c + a)
=
2bc
2( s - a)× 2 s
= (since a + b + c = 2 s)
2bc
Therefore

A s( s - a) A s( s - a)
cos2 = or cos =
2 bc 2 bc
Similarly,

B s( s - b) C s( s - c)
cos = and cos =
2 ca 2 ab ■

T H E O R E M 4.6 In any triangle ABC,


A ( s - b)( s - c)
1. tan =
2 s( s - a)

B ( s - c)( s - a)
2. tan =
2 s( s - b)

C ( s - a)( s - b)
3. tan =
2 s( s - c)

PROOF We know that


A A A
tan = sin ¸ cos
2 2 2
( s - b)( s - c) s( s - a)
= ¸
bc bc
( s - b)( s - c)
=
s( s - a)

C
(Using Theorems 4.4 and 4.5.) Similarly, the formulae for tan(B/2) and tan(C/2) can be
derived. ■
196 Chapter 4 Properties of Triangles

Try it out Show that


B ( s - c)( s - a)
1. tan =
2 s( s - b)

C ( s - a)( s - b)
2. tan =
2 s( s - c)

Example 4.3

In a triangle ABC, suppose a = 6, b = 8 and c = 12. Find C ( s - a)( s - b) 7´5 35


the following: sin = = =
2 ab 6´8 48
A B C A B C
sin ; sin ; sin ; cos ; cos ; cos A s( s - a) 13 ´ 7 91
2 2 2 2 2 2 cos = = =
2 bc 8 ´ 12 96
and hence
B s( s - b) 13 ´ 5 65
A B C cos = = =
tan ; tan ; tan 2 ca 12 ´ 6 72
2 2 2
C s( s - c) 13 ´ 1 13
Solution: Given a = 6, b = 8 and c = 12, we have cos = = =
2 ab 6´8 48
a + b + c 6 + 8 + 12
s= = = 13 Also
2 2

Therefore A A A 5
tan = sin ¸ cos =
2 2 2 91
s - a = 7, s - b = 5 and s - c = 1

Now B B B 7
tan = sin ¸ cos =
2 2 2 65
A ( s - b)( s - c) 5´1 5
sin = = = C C C 35
2 bc 8 ´ 12 96 tan = sin ¸ cos =
2 2 2 13
B ( s - c)( s - a) 1´ 7 7
sin = = =
2 ca 12 ´ 6 72

T H E O R E M 4.7 In any triangle ABC,


2
sin A = s( s - a)( s - b)( s - c)
bc
2
sin B = s( s - a)( s - b)(s - c)
ca
2
sin C = s( s - a)( s - b)( s - c)
ab
PROOF Since
A A
sin A = 2 sin cos
2 2
4.1 Relations Between the Sides and the Trigonometric Ratios of Angles of a Triangle 197

we have

( s - b)( s - c) s(s - a)
sin A = 2
bc bc
2
= s( s - a)( s - b)( s - c)
bc
Similarly the formulae for sin B and sin C can be derived. ■

Note: From the above formulae for sin A, we have


sin A 2
= s( s - a)( s - b)( s - c)
a abc
The right-hand side of the above is symmetric in a, b and c and, therefore,

sin A sin B sin C æ 2 ö


= = çè = s( s - a)( s - b)( s - c) ÷
a b c abc ø

which is precisely the sine formulae proved in the beginning of this chapter.

C O R O L L A R Y 4.1 In any triangle ABC,

4 s( s - a)( s - b)( s - c)
tan A =
b2 + c2 - a2
4 s( s - a)( s - b)( s - c)
tan B =
c2 + a2 - b2
4 s( s - a)( s - b)( s - c)
tan C =
a2 + b2 - c2

Example 4.4

Suppose that in a triangle ABC, a = 8, b = 9 and c = 13. 2 35


Then find tan A, tan B and tan C. sinC = ´ 6 35 =
8´9 6
Solution: Given a = 8, b = 9 and c = 13 we get Hence
8 + 9 + 13 sin A 4 35 35 sin B sin C
s= = 15 = = = =
2 a 39 ´ 8 78 b c
s - a = 7, s - b = 6 and s - c = 2
Now
s( s - a)( s - b)( s - c) = 15 ´ 7 ´ 6 ´ 2 = 6 35
4 s( s - a)( s - b)( s - c) 4 ´ 6 35
tan A = =
Therefore b +c -a
2 2 2
9 + 132 - 82
2

2 4 ´ 6 35
sin A = s( s - a)( s - b)( s - c) = =
24 35 4 35
=
bc 92 + 132 - 82 186 31
2 4 35
= ´ 6 35 = 4 ´ 6 35 24 35 6 35 3 35
9 ´ 13 39 tan B = = = =
132 + 82 - 92 152 38 19
2 3 35
sin B = ´ 6 35 = 4 ´ 6 35
13 ´ 8 26 tan C = = - 35
8 + 92 - 132
2
198 Chapter 4 Properties of Triangles

T H E O R E M 4.8 In any triangle ABC,


B-C b-c A
tan = cot
2 b+c 2
C-A c-a B
tan = cot
2 c+a 2
A-B a-b C
tan = cot
2 a+b 2
PROOF From the sine formulae, we have
sin B sin C
= = K (say)
b c
Now consider

b - c (1/K )(sin B - sin C )


=
b + c (1/K )(sin B + sin C )
sin B - sin C
=
sin B + sin C
æ B + Cö æ B -Cö
2 cos ç
è 2 ÷ø
sin ç
è 2 ÷ø
=
æ B + Cö æ B -Cö
2 sin ç ÷ cos ç
è 2 ø è 2 ÷ø

æ B + Cö æ B -Cö
= cot ç ÷ tan ç
è 2 ø è 2 ÷ø

æ Aö æ B -Cö
= cot ç 90° - ÷ tan ç nce A + B + C = p )
(sin
è 2ø è 2 ÷ø
A æ B - Cö
= tan tan ç
2 è 2 ÷ø
Therefore

æ B - C ö æ b - cö A
tan ç = cot
è 2 ÷ø çè b + c ÷ø 2

Similarly, the other formulae can be proved. ■

Note: The formulae given above are called tangent rules or Napier analogy.

T H E O R E M 4.9 In any triangle ABC,

æ A - Bö a + b C
cos ç =
è 2 ÷ø
sin
c 2
æ B -Cö b + c A
cos ç =
è 2 ÷ø
sin
a 2
æ C - Aö c + a B
cos ç =
è 2 ÷ø
sin
b 2
4.1 Relations Between the Sides and the Trigonometric Ratios of Angles of a Triangle 199

PROOF Consider
a + b sin A + sin B æ sin A sin B sin C ö
= çè since = = ÷
c sin C a b c ø

æ A + Bö æ A - Bö
2 sin ç
è 2 ø ÷ cos ç
è 2 ÷ø
=
C C
2 sin × cos
2 2
æ Cö æ A - Bö
sin ç 90° - ÷ cos ç
è 2ø è 2 ÷ø
=
C C
sin × cos
2 2
C æ A - Bö æ A - Bö
cos × cos ç ÷ø cos çè ÷
2 è 2 2 ø
= =
C C C
sin × cos si n
2 2 2
Therefore

æ A - Bö a + b C
cos ç = × sin
è 2 ÷ø c 2

æ C - Aö c + a B
cos ç = × sin
è 2 ÷ø b 2

æ B -Cö b + c A
and cos ç = × sin
è 2 ÷ø 2 2 ■

The following can be proved similarly.

T H E O R E M 4.10 In any triangle ABC,

æ A - Bö a - b C
sin ç = × cos
è 2 ÷ø c 2

æ B -Cö b - c A
sin ç ÷ = × cos
è 2 ø a 2

æ C - Aö c - a B
= × cos
è 2 ÷ø
sin ç
b 2
PROOF ■

Note: The formulae given in the above two theorems are called Mollweide rules.

T H E O R E M 4.11 In any triangle ABC,


(b2 - c2 )cot A + (c2 - a2 )cot B + (a2 - b2 )cot C = 0
200 Chapter 4 Properties of Triangles

PROOF We have by sine formula that


sin A sin B sin C
= = = K , say
a b c
Now,

cos A cos A b2 + c2 - a2
cot A = = =
sin A aK 2abcK

c2 + a2 - b2 a2 + b2 - c2
cot B = and cot C =
2abcK 2abcK
Therefore

(b2 - c2 )cot A + (c2 - a2 )cot B + (a2 - b2 )cot C

1
= [(b2 - c2 )(b2 + c2 - a2 ) + (c2 - a2 )(c2 + a2 - b2 ) + (a2 - b2 )(a2 + b2 - c2 )]
2abcK
1
= [b4 - c4 - a2 (b2 - c2 ) + c4 - a4 - b2 (c2 - a2 ) + a4 - b4 - c2 (a2 - b2 )]
2abcK
=0 ■

T H E O R E M 4.12 In any triangle ABC, if a, b, c are in Arithmetical Progression, then so are cot(A/2), cot(B/2),
C
cot(C/2).
PROOF Suppose that a, b, c are in Arithmetical Progression. Then a + c = 2b and hence
( s - a) + ( s - c) = 2( s - b)
Now,

A C B s( s - a) s( s - c) s( s - b)
cot + cot - 2 cot = + -2
2 2 2 ( s - b)( s - c) ( s - a)( s - b) ( s - c)( s - a)

s
= [( s - a) + ( s - c) - 2( s - b)]
( s - a)( s - b)( s - c)

=0

C are in Arithmetical Progression.


Therefore cot(A/2), cot(B/2), cot(C/2) ■

T H E O R E M 4.13 In any triangle ABC,

æ A Bö C
(a + b + c) ç tan + tan ÷ = 2c cot
è 2 2ø 2
æ B Cö A
(a + b + c) ç tan + tan ÷ = 2aa cot
è 2 2ø 2
æ C Aö B
(a + b + c) ç tan + tan ÷ = 2b cot
è 2 2ø 2
4.2 Solution of Triangles 201

PROOF From Theorem 4.6, we have

æ A Bö æ ( s - b)( s - c) ( s - c)( s - a) ö
(a + b + c) ç tan + tan ÷ = 2 s ç +
è 2 2ø è s( s - a) s( s - b) ÷ø

s-cæ s-b s - aö
= 2s +
s çè s - a s - b ÷ø

æ ( s - b) + ( s - a) ö
= 2 s( s - c) ç ÷
è ( s - a)( s - b) ø

s( s - c)
=2 × c (since a + b + c = 2 s)
( s - a)( s - b)
A
= 2c cot
2 ■

4.2 | Solution of Triangles


The three sides and the three angles of a triangle are known to be the elements of the triangle. When any three
elements with at least one side are given, then the triangle is completely known, in the sense that its other elements
can be found. When all the three angles are given, only the ratios of lengths of the sides can be found and therefore
only the shape of the triangle can be found and not the size of the triangle. The process of calculating the values of all
the elements of the triangle is termed as solving the triangle and the set of values of the elements is called the solution
of the triangle. In the following, we discuss the solutions of triangles in several cases.

4.2.1 Solutions of Right-Angled Triangles


Let ABCC be a right-angled triangle with C = 90° (Figure 4.12). Then c is the hypotenuse and A and B are acute angles
such that A + B = 90°. We need two more elements to solve the triangle ABC, in which one is necessarily a side.

c b

FIGURE 4.12

Case 1: Suppose that the hypotenuse and one side are given
Let b be the given side and c be the given hypotenuse. Since
b
sin B = < 1 (∵ b < c)
c
we have

b æ pö
B = Sin-1 Î ç 0, ÷
c è 2ø
202 Chapter 4 Properties of Triangles

and A = 180° - (B + C) = 90° - B

Also, a = c cos B. Thus, we can calculate all the three sides and the three angles.
Case 2: Suppose that the two sides a and b are given
In this case

c = a2 + b2

Since
b
tan B =
a
so
b
B = Tan-1
a
Therefore B is known and
p
A = p - (B + C ) = -B
2
Thus all the elements of the triangle are known. The hypotenuse c can also be calculated using the formula
b
sin B =
c
Case 3: Suppose that the angle B and the hypotenuse c are given
Then A = 90° – B and a and b can be obtained from
a b
cos B = and sin B =
c c
Case 4: Suppose that an angle B and one of the sides a are given
Then A = 90° – B and b is obtained from tan B = b/a. Also c can be found from the relation cos B = a/c.

Example 4.5

The length of the perpendicular from one vertex A of a AD 3


triangle to the opposite side is 3 cm and the lengths of the sin B = =
AB 4
sides for which A is the common point are 4 cm and 5 cm.
Solve the triangle. AD 3
and sinC = =
AC 5
A
Therefore
3
B = Sin-1
4 5
4
3
3
C = Sin-1
5
and A = p - (B + C)
FIGURE 4.13 Example 4.5. Also

Solution: Let ABC C be the triangle and AD be the perpen- a = BD + DC = 4 cos B + 5 cos C
dicular from A to BC. We are given that AD = 3 cm, AB = Thus, all the three angles and three sides are found.
4 cm and AC = 5 cm (Figure 4.13). Then
4.2 Solution of Triangles 203

Example 4.6

Solve the right-angled triangle whose hypotenuse is four Also, we have


times as long as the perpendicular drawn to it from the
opposite vertex. a2 + b2 = (4 x)2 = 4ab (∵ 4 x2 = ab)
Now
Solution: Let ABC C be the given right angled triangle
with C = 90°. Let CD be the perpendicular from C to AB. æ aö
2
æ aö
Then AB is the hypotenuse. Let CD = x. Then AB = 4x. çè ÷ø + 1 = 4 çè ÷ø
b b
See Figure 4.14.
2
æ aö æ aö
çè ÷ø - 4 çè ÷ø + 1 = 0
A b b

D
a 4 ± 16 - 4
4x b = =2± 3
b 2
x
Therefore
a
tan A = =2± 3
b
FIGURE 4.14
Hence A = 75° or 15° and B = 15° or 75°.
From the triangle ABC, we have Without loss of generality, we can suppose that A = 75°
and B = 15° and we are given that C = 90°. Therefore
b
sin B = a
4x = tan A = 2 + 3 Þ a = b(2 + 3 )
b
From the triangle BCD, we have
x and c = a2 + b2 = b 8 + 4 3 = 2b 2 + 3
sin B =
a Therefore knowing the value of b we can determine the
Therefore values of a and c. Hence, there are infinitely many trian-
gles with angles 90°, 75° and 15°, if the altitude from the
b x right-angled vertex onto the hypotenuse is one-fourth of
= or 4 x2 = ab
4x a the hypotenuse.

Note: In the above example, the triangle can be determined provided one of the sides is given.

4.2.2 Solution of a Triangle when Three Sides are Given


C be the triangle and let the sides a, b and c be given. Then the perimeter a + b + c = 2s and hence s is known.
Let ABC
Then using the formulae

A ( s - b)( s - c) B ( s - c)( s - a)
tan = and tan =
2 s( s - a) 2 s( s - b)

the angles A and B can be found and the third angle C = p -(A + B). The angles A and B can also be found using the
cosine formulae

b2 + c2 - a2 c2 + a2 - b2
cos A = and cos B =
2bc 2ca
204 Chapter 4 Properties of Triangles

Example 4.7

C if a = 2, b = 6 and c = 3 - 1.
Solve the triangle ABC Also

Solution: We have c2 + a2 - b2 ( 3 - 1)2 + 22 - ( 6 )2


cos B = =
2ca 2( 3 - 1) ´ 2
b2 + c2 - a2 ( 6 )2 + ( 3 - 1)2 - (2)2
cos A = = 3 + 1 - 2 3 + 4 - 6 2((1 - 3 ) -1
2bc 2 ´ 6 ´ ( 3 - 1) = = =
4( 3 - 1) 4( 3 - 1) 2
6 + 3+ 1- 2 3 - 4
=
2 2 3 ( 3 - 1) Therefore B = 120°.
Now
2 3 ( 3 - 1) 1
= = C = p - ( A + B) = 180° - (45° + 120°) = 15°
2 2 3 ( 3 - 1) 2
Thus A, B, C are 45°, 120° and 15°, respectively.
Therefore A = 45°.

Example 4.8

C if a = 2, b = 6 and c = 3 + 1.
Solve the triangle ABC Also

Solution: Consider c2 + a2 - b2 ( 3 + 1)2 + 22 - ( 6 )2


cos B = =
2ca 2( 3 + 1)2
b2 + c2 - a2 ( 6 )2 + ( 3 + 1)2 - 22
cos A = =
2bc 2 6 ( 3 + 1) 3 + 1 + 2 3 + 4 - 6 2(1 + 3 ) 1
= = =
4( 3 + 1) 4( 3 + 1) 2
6 + 3+ 1+ 2 3 - 4 1
= =
2 2 (3 + 3 ) 2 Therefore B = 60°. Now

Therefore A = 45°. C = 180° - (A + B) = 180° - (45° + 60°) = 75°

4.2.3 Solution of a Triangle when Two Sides and the Angle Between Them are Given
Let ABC
C be the triangle. Suppose that the sides b and c and the angle A are given. From the formula

b2 + c2 - a2
cos A =
2bc

we can find that

a = b2 + c2 - 2bc cos A

Therefore, all the three sides are known and hence we can solve the triangle as in Section 4.2.2. We can also use the
formulae

æ B - Cö b - c A
tan ç ÷ = cot (by Theorem 4.8)
è 2 ø b+c 2

to get (B - C)/2 and hence B - C. Also B + C = 180° - A. From these two equations we can find B and C.
The third side can be found from

b sin A
a=
sin B
4.2 Solution of Triangles 205

Example 4.9

C if a =
Solve the triangle ABC 3, b = 1 and C = 30°. Now,

Solution: From the formula b2 + c2 - a2


cos A =
2bc
a2 + b2 - c2
cosC = 12 + 12 - ( 3 )2 -1
2ab = =
2 ´1´1 2
we get that
Therefore A = 120°. Also
c = a + b - 2ab cos C
2 2
B = 180° - ( A + C ) = 180° - (120° + 30°) = 30°
= ( 3 )2 + 12 - 2 3 cos 30°

3
= 3 + 1 - 2 3× =1
2

4.2.4 Solution of a Triangle when Two Sides and an Angle Opposite to


One of Them are Given
ABC
C be a triangle in which the sides a and b and the angle A are given. From the formula

b2 + c2 - a2
cos A =
2bc
we get that

a2 = b2 + c2 - 2bc cos A
Therefore

c2 - 2bc cos A + b2 cos2 A = a2 - b2 + b2 cos2 A


= a2 - b2 sin2 A

which gives

(c - b cos A)2 = a2 - b2 sin2 A

c - b cos A = ± a2 - b2 sin2 A

c = b cos A ± a2 - b2 sin2 A

This is an equation to find the value of c when a, b and A are given. Here we have to distinguish the following four cases:
Case 1: If a < b sin A, then a2 - b2 sin2 A < 0 and hence a2 - b2 sin2 A is not a real number. Therefore the above equa-
tion does not determine the value of c.
Case 2: If a = b sin A, then there is only one value for c, namely b cos A, and in this case the triangle is a right-angled
triangle, B being the right angle since
sin B sin A 1
= =
b a b
and hence sin B = 1.
Case 3: If a > b sin A, then there are two values for c, that is,

c = b cos A + a2 - b2 sin2 A
206 Chapter 4 Properties of Triangles

or c = b cos A - a2 - b2 sin2 A

However, c must be positive and hence the above two distinct values of c are admissible only when

b cos A - a2 - b2 sin2 A > 0

That is

a2 - b2 sin2 A < b cos A

a2 - b2 sin2 A < b2 cos2 A


a2 < b2 sin2 A + b2 cos2 A = b2
which implies a < b. Thus, when a > b sin A, there is unique triangle if a ³ b and there are two triangles if b sin A < a < b.
Case 4: If A is an obtuse angle, then cos A < 0 and hence

b cos A - a2 - b2 sin2 A < 0

and therefore the value of c must be b cos A + a2 - b2 sin2 A . Moreover this too must be positive, that is,

a2 - b2 sin2 A > - b cos A

a2 - b2 sin2 A > b2 cos2 A

a2 > b2 sin2 A + b2 cos2 A = b2


which implies a > b. Thus, when A is obtuse, there is no triangle if a £ b and there is only one triangle if a > b.

Example 4.10

Solve the triangle ABC, given that a = 100, c = 100 2 b = c cos A ± a2 - c2 sin2 A
and A = 30°.
2
3 æ 1ö
Solution: As in Section 4.2.4, we have = 100 2 ´ ± (100)2 - (100 2 )2 ç ÷
2 è 2ø
b = c cos A ± a2 - c2 sin2 A
3
= 100 ± 50000
In this triangle we have 2

æ 1ö
2 3
c sin A = (100 2 ) ç ÷ = 5000 (∵ sin 30° = 1/ 2)
2 2 2 = 100 ± 50 2
è 2ø 2

Therefore = 50 2 ( 3 ± 1)

a2 = (100)2 > c2 sin2 A Now

c2 + a2 - b2
and a2 - c2 sin2 A is a real number. Here cos B =
2ca
1 100
c sin A = 100 2 = = (100 2 )2 + (100)2 - [50 2 ( 3 ± 1)]2
2 2 =
2 ´ 100 2 ´ 100
Therefore
1± 3
c sin A < a < c =
2 2
From the discussion made in Case 3 above, there are two
triangles with the given elements. First, we have
4.2 Solution of Triangles 207

Now two cases arise: Thus the two solutions of the triangle ABC
C are:
1+ 3 (1) a1 = 100, b1 = 50 2( 3 - 1), c1 = 100 2, A1 = 30°, B1 =
(1) cos B = Þ B = 15° and C = 135°
2 2 15° and C1 = 135°
1- 3 (2) a2 = 100, b2 = 50 2( 3 + 1), c2 = 100 2, A2 = 30°, B2 =
(2) cos B = Þ B = 105° and C = 45°
2 2 105° and C2 = 45°

Example 4.11

Is a triangle solvable if a = 5, b = 7 and sin A = 3/4? Therefore


b 7 3 21
Solution: From the sine formula, we have sin B = sin A = × = >1
a 5 4 20
sin A sin B
= This is impossible, since sin B < 1 for all real numbers B.
a b
Therefore, there is no triangle with the given values for
a, b and sin A.

4.2.5 Solution of a Triangle when One Side and Two Angles are Given
Let ABCC be a triangle in which side a and angles B and C are given. Then A = 180° - (B + C). Therefore, all the angles
are known. Also, from the sine formulae, we have
sin B sin C
b= a and c = a
sin A sin A

4.2.6 Solution of a Triangle when All the Angles are Given


There is no unique solution in this case. We can only say that the ratios of the sides are determined from the sine
formulae. That is,

a sin A b sin B c sin C


= , = and =
b sin B c sin C a sin A

Example 4.12

In a triangle ABC, cos A = 17/22 and cos C = 1/14. 195 1 17 195


Find the ratio a : b : c. = × + ×
22 14 22 14
Solution: We have 195 ´ 18 195 ´ 9
= =
22 ´ 14 11 ´ 14
17 172 195
cos A = Þ sin A = 1 - 2 = Therefore
22 22 22
a sin A 195 11 ´ 14 7
1 1 195 = = ´ =
and cos C = Þ sin C = 1 - 2 = b sin B 22 195 ´ 9 9
14 14 14
b sin B 195 ´ 9 14 9
Also, = = ´ =
c sin C 11 ´ 14 1955 11
sin B = sin[p - ( A + C )]
Thus a : b : c = 7 : 9 :11.
= sin( A + C )
= sin A cos C + cos A sin C
208 Chapter 4 Properties of Triangles

Example 4.13


It is given that the base angles of a triangle are 22 and where d = BD. Also, from the triangle BDC,
2
1° 1° d
112 . Prove that the base of the triangle is twice the tan 22 = tan BCD =
2 2 h
height of the triangle.
1° But
Solution: Let ABC be the triangle such that A = 22
2

æ 1° ö
and B = 112 (Figure 4.15). Then 1° ç 2 tan 22 ÷
2 tan 22 = 2 - 1 ç 1 = tan 45° = 2 ÷
2 ç 1° ÷
° çè 1 - tan2 22 ÷
æ 1 1ö 2 ø
C = 180° - ç 22 + 112 ÷ = 45°
è 2 2ø
Therefore
Also
h d
1° 1° = = 2 -1 (4.2)
DBC = 180° - 112 = 67 c+d h
2 2
Þ d = h( 2 - 1)
1° 1°
BCD = 90° - 67 = 22 Substituting the value of d in Eq. (4.2) we get
2 2
where CD is the perpendicular to AB through C. h
= 2 -1
c + h( 2 - 1)
C
h = c( 2 - 1) + h( 2 - 1)2
45° 22 21°
b c( 2 - 1) = h[1 - ( 2 - 1)2 ]
a h
1
c= [1 - (2 + 1 - 2 2 )]h
112 21° 67 21° 2 -1
22 21°
1
= × (- 2 + 2 2 ) h
FIGURE 4.15 Example 4.13. 2 -1
2( 2 - 1)h
From the triangle ADC, = = 2h
2 -1
1° h
tan 22 = Thus the base AB = c = 2 (height).
2 c+d

Example 4.14

If in a triangle ABC, it is given that the angles are in the 5+1


ratio 1: 2 : 7, then what is the ratio of the greatest side to and sin 54° = cos 36° =
4
the smallest side?
Therefore
Solution: Since A + B + C = 180° and A : B : C = 1: 2 : 7,
we get that A = 18°, B = 36° and C = 126°. Therefore a is c sin 54° 5+1
= =
the smallest side and c is the greatest side. Now, a sin 18° 5 -1
c sin C sin 126° sin 54° 5 + 1 : 5 - 1.
= = =
a sin A sin 18° sin 18°

5 -1
sin 18° =
4
4.3 Properties of a Triangle 209

4.3 | Properties of a Triangle


In this section we discuss various properties of triangles, mainly related to the circumcircle, incircle and excircle, using
the formulae we have derived in the previous sections.

T H E O R E M 4.14 The area of a triangle is given by


(A R E A O F A
TRIANGLE) 1
Area DABC = bc sin A
2
1
= ca sin B
2
1
= ab sin c
2

= s( s - a)( s - b)( s - c)

E A F

FIGURE 4.16 Theorem 4.14.

PROOF Let ABC C be a triangle. Draw a perpendicular to BC


C through A to meet BC
C at D. Draw a parallel
to BC
C (say EF)
F passing through A and to EF F draw perpendicular BE
E and CFF (Figure 4.16). Let D
be the area of the triangle ABC. Then, we have
1
D= area of the rectangle BCFE
2
1
= BC ×CF
2
1
= a × AD
2
1
= a × ( AB × sin B)
2
1
= ac sin B
2
Similarly
1 1
D= bc sin A = ab sin C
2 2
210 Chapter 4 Properties of Triangles

Also, from Theorem 4.7 we have


2
sin A = s( s - a)( s - b)( s - c)
bc
Therefore
1
D= bc sin A = s( s - a)( s - b)( s - c)
2 ■

Example 4.15

ABC, a = 18, b = 24 and c = 30. Find the area Therefore the area of ABC
C is
of the triangle.
s( s - a)( s - b)( s - c) = 36 ´ 18 ´ 12 ´ 6
Solution: We have
= 216 sq. units
2s = a + b + c = 18 + 24 + 30 = 72
Therefore s = 36. Now
s – a = 18, s - b = 12, s - c = 6

Example 4.16

ABC, B = 45°, C = 60° and a = 2( 3 + 1). The area of triangle ABC


C is
Find the area of the triangle.
1 1
ab sin C = 2( 3 + 1) ´ 4 ´ sin 60°
Solution: Since A + B + C = 180°, we get A = 75°. From 2 2

a b 3
= = 4( 3 + 1) ´
sin A sin B 2

we have = 2 3 ( 3 + 1)

a sin B 2( 3 + 1)sin 45° = 6 + 2 3 sq. units


b= =
sin A sin 75°

2( 3 + 1)(1/ 2 ) æ 3 + 1ö
= ç∵ sin 75° = ÷
( 3 + 1)/ 2 2 è 2 2 ø
=4

DEFINITION 4.1 The circle passing through the three vertices of a triangle is called the circumcircle and its
centre and radius are called circumcentre and the circumradius, respectively.

We know that the perpendicular bisectors of the sides of a triangle are concurrent and the point of concurrence is
equidistant from the vertices and hence it is the circumcentre.

T H E O R E M 4.15 Let R be the circumradius of a triangle ABC. Then a = 2R sin A, b = 2R sin B and c = 2R sin C.
That is
a b c
2R = = =
sin A sin B sin C
4.3 Properties of a Triangle 211

PROOF Let S be the circumcentre of the triangle ABC. Consider the angle A. We shall distinguish three cases.

B C
R S R

FIGURE 4.17 Case I.

Case I: Suppose that A = 90°. Then S is the mid-point of BC


C and BC
C is a diameter of the circum-
circle (Figure 4.17). Now,
a = BC = 2R = 2R × sin 90°
Therefore
a
= 2R
sin A

D R

S R

B C

FIGURE 4.18 Case II.

Case II: Suppose that A < 90°. Let CD be a diameter of the circumcircle, so that S is its mid-point
and CS = SD = R. Join BD. Then DBC = 90°, since it is an angle in the semicircle (Figure 4.18).
Also, BDC = BAC (since they are angles in the same segment made by BC). Now,

BC a
sin A = sin BDC = =
CD 2 R
and hence a = 2R sin A.
Case III: Suppose that A > 90°. Draw diameter CD of the circumcircle through C. Then CS =
SD = R. Join B and D. Then DBC = 90°, since it is an angle made by a semicircle (Figure 4.19).
In the cyclic quadrilateral BACD, we have
BDC + BAC = 180°

Therefore in the right-angled BDC,


BC a
sin BDC = =
CD 2 R
212 Chapter 4 Properties of Triangles

B C

S
R
D

FIGURE 4.19 Case III.

Therefore
a
sin A = sin(180° - A) = sin BDC =
2R
Hence
a = 2R sin A
Similarly we can prove that b = 2R sin B and c = 2R sin C. ■

C O R O L L A R Y 4.2 Let D be the area of a triangle ABC


C and R be its circumradius. Then
abc abc
R= =
4 D 4 s( s - a)( s - b)( s - c)

PROOF
2 2D
sin A = s( s - a)( s - b)( s - c) =
bc bc
Therefore
a 2D abc
= or R =
2 R bc 4D ■

DEFINITION 4.2 Incircle, Incentre, Inradius The circle that touches the three sides of a triangle internally is
called the incircle or inscribed circle of the triangle. Its centre and radius are called the incentre
and inradius, respectively, and are denoted by I and r, respectively.

It is known from Geometry that the internal bisectors of the angles of a triangle are concurrent and that the point of
concurrence is equidistant from the sides and hence there is a circle touching all the three sides internally. Therefore,
the point of concurrence of the internal bisectors of the angles is precisely the incentre I and the distance from I to any
side is the inradius r. In the following, we express the inradius in terms of the sides of the triangle.

T H E O R E M 4.16 Let D be the area of a triangle ABC. Then the inradius r is equal to D /s; that is,

D 2 s( s - a)( s - b)( s - c)
r= =
s a+b+c
4.3 Properties of a Triangle 213

PROOF Draw internal bisectors of the angles B and C to meet at I. Then I is the incentre. Join A and I.
Then AII becomes the bisector of the angle A. Draw perpendiculars ID, IE E and IFF to the sides
BC, CA and AB, respectively (Figure 4.20). Then, since the sides are tangents to the incircle with
centre I, we have
ID = IE = IF = r, the inradius
Then
1 1
Area of triangle IBC = BC × ID = ar
2 2
Similarly,
1
Area of triangle ICA = br
2
1
and triangle Area of triangle IAB = cr
2
Hence

Area of the triangle ABC = Sum of the areas of the triangles IBC , ICA and IAB
1 1 1
= ar + br + cr
2 2 2
a+b+c
= ×r
2
= sr

Thus

D 2 s( s - a)( s - b)( s - c)
D = sr or r= =
s a+b+c

since D = s( s - a)( s - b)( s - c) and 2s = a + b + c.

E
F

B
D

FIGURE 4.20 Theorem 4.16. ■


214 Chapter 4 Properties of Triangles

T H E O R E M 4.17 The inradius r of a triangle ABC


C can be expressed as
A B C
r = ( s - a)tan = ( s - b)tan = ( s - c)tan
2 2 2
PROOF Note that in Figure 4.20 we have IBD = IBF (since BII is the bisector of the angle B) and
IDB = 90° = IFD. Therefore the two triangles IDB and IFB are equal in all respects and, in
particular,
BD = BF and hence 2BD = BD + BF
Similarly
AE = AF and hence 2AE
2 = AE + AF
CE = CD and hence 2CE = CE + CD
By adding these three equations, we get
2 BD + 2 AE + 2CE = BD + BF + AE + AF + CE + CD
= ( BD + CD) + ( BF + AF ) + ( AE + CE)
= BC + AB + AC
= a + b + c = 2s
Therefore
2 BD + 2( AE + CE) = 2 s
2 BD + 2 AC = 2 s
BD = s - AC = s - b = BF

CE = s - c = CD and AF = s - a = AE
Therefore
B B
r = ID = BD tan IBD = BD tan = (s - b)tan
2 2
C C
r = IE = CE tan ICE = CE tan = (s - c) tan
2 2
A A
r = IF = AF tan IAF = AF tan = ( s - a)tan
2 2
Thus
A B C
r = ( s - a)tan = ( s - b)tan = ( s - c)tan
2 2 2 ■

T H E O R E M 4.18 Let ABC


C be a triangle and r its inradius. Then

B C C A A B
sin sin sin sin sin sin
r=a 2 2 =b 2 2 =c 2 2
A B C
cos cos cos
2 2 2
PROOF
a = BD + CD
= ID cot IBD + ID cot ICD
4.3 Properties of a Triangle 215

B C
= r cot + r cot
2 2
æ cos( B / 2) ö æ cos(C/ 2) ö
= rç + rç
è sin( B / 2) ÷ø è sin(C/ 2) ÷ø

Therefore

B C æ C B C Bö
a sin sin = r ç sin cos + cos sin ÷
2 2 è 2 2 2 2ø
æ B Cö
= r sin ç + ÷
è 2 2ø

æ Aö æ A B C ö
= r sin ç 90° - ÷ çè since + + = 90°÷ø
è 2ø 2 2 2
A
= r cos
2
Hence
sin(B / 2)sin(C/ 2)
r=a
cos( A / 2)

Similarly, the other formulae can be proved. ■

C O R O L L A R Y 4.3 If R is the circumradius and r is the inradius of a triangle ABC, then


A B C
r = 4 R sin sin sin
2 2 2
PROOF Since
A A
a = 2 R sin A = 4 R sin cos
2 2
we have (by Theorem 4.18)

æ A A ö æ sin(B / 2)sin(C/ 2) ö
r = ç 4 R sin cos ÷ ç ÷ø
è 2 2øè cos( A / 2)
A B C
= 4 R sin sin sin
2 2 2 ■

Example 4.17

Find the circumradius and inradius of a triangle whose From Corollary 4.2, the circumradius R is given by
sides are 18 cm, 24 cm and 30 cm.
abc 18 ´ 24 ´ 30
R= = = 15 cm
Solution: Let ABC C be the triangle. Let a = 18 cm, 4D 4 ´ 216
b = 24 cm and c = 30 cm. Then r is given by
a + b + c 18 + 24 + 30 D 216
s= = = 36 r= = = 6 cm
2 2 s 36
The area of the triangle is

D = s( s - a)( s - b)( s - c) = 216 sq. cm


216 Chapter 4 Properties of Triangles

DEFINITION 4.3 Excircles, Excentres, Exradius Let ABC C be a triangle. The circle that touches BC C and the
extended sides AB and AC C (as shown Figure 4.21) is called the excircle or escribed circle oppo-
site to the vertex A. Its centre and radius are called the excentre or escentre and exradius or
esradius opposite to A, respectively. These are denoted by I1 and r1, respectively. Similarly, we
can define the escentre and esradius opposite to B and C which are denoted by I2, r2 and I3, r3,
respectively.

The internal bisector of the angle A and the bisectors of the external angles B and C are known to be concurrent and
the point of concurrence is equidistant from the three sides. This point of concurrence is precisely the excentre I1 and
the distance from I1 to any side is the esradius r1. In the following we derive formulae to find the exradius r1, r2 and r3
in terms of the sides of the triangle.

r1 r1

I1

FIGURE 4.21

T H E O R E M 4.19 Let ABCC be a triangle with area D and r1, r2 and r3 be the radii of the escribed circles opposite to
A, B and C respectively. Then
D D D
r1 = , r2 = and r3 =
s-a s-b s-c
PROOF Produce AB and AC C to L and M. Bisect the angles CBL and BCM M by the lines BII1 and CII1. These
meet at I1 and AII1 will be the bisector of the angle A. Draw perpendiculars I1D1, I1E1 and I1F1 to
the sides BC, AC C and AB, respectively.
The two triangles I1D1B and I1F1B are equal in all respects (since BII1 is the bisector of the
angle D1 BF1 and BF1 I1 = 90° = BD1 I1 ). Therefore I1F1 = I1D1. Similarly I1E1 = I1D1. Therefore,
I1D1 = I1E1 = I1F1. Hence I1 is the centre of the escribed circle opposite to A and I1D1 = r1. Now, the
area of the quadrilateral ABII1C is equal to the sum of the areas of the triangles ABII1 and ACII1
and is also equal to the areas of the triangles ABC C and BII1C. Therefore
DABI1 + D ACI1 = DABC + DBI1C
4.3 Properties of a Triangle 217

C
D1

F1 E1

I1

L M

FIGURE 4.22 Theorem 4.19.

where DABII1 denotes the area of the triangle ABII1, DACII1, DABC
C and DBII1C denote the areas of
the triangles ACII1, ABC
C and BII1C, respectively. Therefore
1 1 1
× I1F1 × AB + I1E1 ×CA = D + I1D1 × BC
2 2 2
where D = DABC = the area of ABC. Hence
1 1 1
r1c + r1b = D + r1a
2 2 2
r1 (c + b - a) æa+b+c ö
D= = r1 ç - a÷ = r1 ( s - a)
2 è 2 ø

D
r1 =
s-a
Similarly we can prove that
D D
r2 = and r3 =
s-b s-c ■
218 Chapter 4 Properties of Triangles

In the following, the exradii r1, r2 and r3 are expressed in terms of s and the tangents of the half angles of the triangle.

T H E O R E M 4.20 Let r1, r2 and r3 be the radii of the escribed circles opposite to A, B and C, respectively, for a
triangle ABC. Then
A B C
r1 = s tan , r2 = s tan and r3 = s tan
2 2 2
PROOF Consider Figure 4.22. Since AE1 and AF F1 are tangents to the circle from the point A, we get that
AE1 = AF
F1 and, for a similar reason, we have
BF1 = BD1 and CE1 = CD1

Now,
2 AE1 = AE1 + AF1
= AC + CE1 + AB + BF1
= AC + CD1 + AB + BD1
= AB + AC + CD1 + BD1
= AB + AC + BC
= a + b + c = 2s
Therefore
AE1 = s = AF1

From the right-angled triangle AII1E1, we have

A I1 E1 r1
tan = =
2 AE1 s

Thus
A
r1 = s ×tan
2
Similarly
B C
r2 = s tan and r3 = s tan
2 2 ■

C O R O L L A R Y 4.4 Let r1, r2 and r3 be the radii of the escribed circles opposite to A, B and C, respectively, for a
triangle ABC. Then
B
r1 = ( s - c)cot
2
C
r2 = ( s - a)cot
2
A
and r3 = ( s - b)cot
2
PROOF In Theorem 4.20 we have proved that s = AF
F1 and hence
s - c = AF1 - AB = BF1
4.3 Properties of a Triangle 219

Now, in the right-angled triangle BF


F1I1, we have

B æ Bö I F r
cot = tan ç 90 - ÷ = 1 1 = 1
2 è 2 ø BF1 s - c

Therefore
B
r1 = ( s - c)cot
2
Similarly
C A
r2 = ( s - a)cot and r3 = ( s - b)cot
2 2 ■

Example 4.18

In a triangle ABC, a = 13, b = 14 and c = 15. Find the area and the exradii are
and hence the exradii.
D 84
r1 = = = 10.5
Solution: We have s-a 8

2s = a + b + c = 13 + 14 + 15 = 42 D 84
r2 = = = 12
s-b 7
Hence
D 84
s = 21, s - a = 8, s - b = 7 and s - c = 6 r3 = = = 14
s-c 6
Therefore the area is

D = s( s - a)( s - b)( s - c)

= 21 ´ 8 ´ 7 ´ 6 = 84 sq. units

QUICK LOOK 1

In any triangle ABC,

D D D D D4
rr1r2 r3 = × × × = 2 = D2
s s-a s-b s-c D

Another formula for the esradius r1 is obtained in the following in terms of the side a and the angles B and C.

T H E O R E M 4.21 In any triangle ABC,


a cos( B / 2) × cos(C/ 2)
r1 =
cos( A / 2)
b cos(C/ 2)× cos( A / 2)
r2 =
cos( B / 2)
c cos( A / 2)× cos( B / 2)
and r3 =
cos(C/ 2)
220 Chapter 4 Properties of Triangles

PROOF Again consider Figure 4.22. Since I1C bisects the angle BCE1 , we have
1 C
I1CD1 = (180° - C ) = 90° -
2 2
Similarly
B
I1 BD1 = 90° -
2
Now,
a = BC
= BD1 + D1C
= I1D1 × cot I1 BD1 + I1D1 × cot I1CD1

é æ Bö æ Cöù
= r1 êcot ç 90° - ÷ + cot ç 90° - ÷ ú
ë è 2 ø è 2øû
æ B Cö
= r1 ç tan + tan ÷
è 2 2ø
æ sin(B / 2) sin(C / 2) ö
= r1 ç +
è cos( B / 2) cos(C / 2) ÷ø

B C æ B C B Cö
a cos cos = r1 ç sin cos + cos sin ÷
2 2 è 2 2 2 2ø
é æ B Cöù
= r1 êsin ç + ÷ ú
ë è 2 2øû
æ Aö
= r1 sin ç 90° - ÷
è 2ø
A
= r1 cos
2
Hence
a cos( B / 2) × cos(C/ 2)
r1 =
cos( A / 2)

Similarly,
b cos(C / 2) × cos( A / 2) c cos( A / 2) × cos( B / 2)
r2 = and r3 =
cos( B / 2) cos(C/ 2) ■

C O R O L L A R Y 4.5 In any triangle ABC,


A B C
r1 = 4 R sin cos cos
2 2 2
B C A
r2 = 4 R sin cos cos
2 2 2
C A B
and r3 = 4R sin cos cos
2 2 2
4.3 Properties of a Triangle 221

PROOF This follows from Theorem 4.21 and from the facts that
A A
a = 2 R sin A = 4 R sin cos
2 2
B B
b = 2R sin B = 4 R sin cos
2 2
C C
and c = 2 R sin C = 4R sin cos
2 2 ■

Example 4.19

In any triangle ABC, prove that 1 2


= ( s + s2 + a2 - 2as + s2 + b2 - 2bs + s2 + c2 - 2cs)
D2
1 1 1 1 a2 + b2 + c2
+ + + = 1
r2 r12 r22 r32 D2 = [4 s2 - 2 s(a + b + c) + a2 + b2 + c2 ]
D2
Solution: 1
= (4 s2 - 2 s × 2 s + a2 + b2 + c2 )
1 1 1 1 s2 ( s - a)2 ( s - b)2 ( s - c)2 D2
+ 2 + 2 + 2 = 2 + + +
r 2
r1 r2 r3 D D2 D2 D2 a2 + b2 + c2
=
D2

Example 4.20

In any triangle ABC, prove that s æ 2 A B Cö


= çè 4 R sin + 4 R sin2 + 4 R sin2 ÷
r1 r r 1 1 abc 2 2 2ø
+ 2 + 3 = -
bc ca ab r 2 R 2 Rs
= (1 - cos A + 1 - cos B + 1 - cos C )
Solution: abc
s æ abc ö
r1 r r 1 = [3 - (cos A + cos B + cos C )] ç∵ R = ÷
+ 2 + 3 = (ar1 + br2 + cr3 ) 2D è 4D ø
bc ca ab abc
1 æ A B Cö 1 é æ A B Cöù æ Dö
= = 3 - ç 1 + 4 sin sin sin ÷ ú ç∵ r = ÷
ç a × s tan + b × s tan + c × s tan ÷ø ê
2r ë è ø
2 û è sø
abc è 2 2 2 2 2

s æ A B Cö 1 é 4 R sin( A / 2)sin( B / 2)sin(C/ 2) ù


= ç 2 R sin A tan + 2 R sin B tan + 2 R sin C tan ÷ø = ê 2- úû
abc è 2 2 2 2r ë R
s æ A A A B B B 1æ rö 1 1
= ç 4 R sin cos tan + 4 R sin cos tan = ç 2 - ÷ø = -
abc è 2 2 2 2 2 2 2r è R r 2R
C C Cö
+ 4 R sin cos taan ÷
2 2 2ø

Example 4.21

In a triangle ABC, if r1 = 8, r2 = 12 and r3 = 24, then find D D


the lengths of the sides. = r2 = 12 Þ s - b =
s-b 12

Solution: Let D be the area. Then D D


= r3 = 24 Þ s - c =
s-c 24
D D
= r1 = 8 Þ s - a =
s-a 8
222 Chapter 4 Properties of Triangles

Therefore Therefore
3s - (a + b + c) = ( s - a) + ( s - b) + ( s - c) D = 96 sq. units

æ1 1 1ö D Now
= Dç + + =
è 8 12 24 ÷ø 4 D 96
s= = = 24
This gives r 4
D 96
D a=s- = 24 - = 12
3s - 2 s = r1 8
4
D D 96
s= b=s- = 24 - = 16
4 r2 12
D D 96
r= =4 c=s- = 24 - = 20
s r3 24

Since
D2 = rr1r2 r3 (see Quick Look 1)
= 4 ´ 8 ´ 12 ´ 24
= (12 ´ 8)2

DEFINITION 4.4 Orthocentre In any triangle, the perpendiculars drawn from the vertices to the respective
opposite sides are known to be concurrentt and the point of concurrence is called the ortho-
centre of the triangle. Also, the triangle formed by the feet of these perpendiculars on the
sides is called the pedal triangle of the triangle.

Let ABC C be a triangle and AK, BL and CM M be perpendiculars drawn from A, B and C to BC, CA and AB, respec-
tively. Then the perpendiculars AK, BL and CM M are concurrent at a point P. The point P is called the orthocentre and
the triangle KLM M is called the pedal triangle of the triangle ABC
C (Figure 4.23).

L
M P

FIGURE 4.23 Orthocenter and pedal triangle.

T H E O R E M 4.22 In any triangle ABC, the distances of the orthocentre P from the vertices A, B and C are given by
AP = 2R cos A, BP = 2R cos B and CP = 2R cos C
where R is the circumradius of the triangle ABC. Also the distances of P from the sides BC, CA
and AB are given by
PK = 2R cos B cos C
PL = 2R cos C cos A
and PM = 2R cos A cos B
4.3 Properties of a Triangle 223

PROOF First observe that KAC = 90° - C and hence BPK = APL = C and PBK = 90° - C . Now, in the
triangle APL,
AL
cos KAC =
AP
and hence
AL AB cos A C
AP = = = cos A = 2 R cos A
cos KAC cos(90° - C ) sin C

BP = 2R cos B and CP = 2R cos C


Also
PK = KB tan PBK
= KB tan(90° - C )
= AB cos B × cot C
cos C
= AB cos B ×
sin C
c
= cos B cos C
sin C
= 2 R cos B cos C
Similarly
PL = 2R cos C cos A and PM = 2R cos A cos B ■

In the following, we derive formulae for the angles and sides of the pedal triangle of a triangle ABC
C in terms of the
angles and sides of ABC.

T H E O R E M 4.23 Let ABC


C be a triangle and K, L and M be the feet of perpendiculars from A, B and C on the sides
BC, CA and AB, respectively; that is, KLM M is the pedal triangle of the triangle ABC
C (Figure 4.24).
The angles and the sides of the pedal triangle are given by
MKL = 180° - 2 A, KLM = 180° - 2 B, LMK = 180° - 2C
LM = a cos A, MK = b cos B and KL = c cos C
PROOF Since CKP = 90° = CLP, the points K, C, L and P are concyclic and therefore
PKL = PCL = 90° - A

(since CMA = 90° in the triangle AMC).

L
M P

FIGURE 4.24 Theorem 4.23.


224 Chapter 4 Properties of Triangles

Similarly M, B, K and P are concyclic and hence


MKP = MBP = 90° - A

Now,
MKL = MKP + PKL = 90° - A + 90° - A = 180° - 2 A

Similarly
KLM = 180° - 2 B and LMK = 180° - 2C

Also, from the triangle MLA, we have


LM AL
=
sin A sin LMA
AB cos A
=
cos LMP
c cos A
=
cos LAP
c cos A
=
cos(90° - C )
c cos A
=
sin C
Therefore

c sin A cos A æ c ö
LM = =ç sin A × cos A = a cos A
sin C è sin C ÷ø

Similarly, MK = b cos B and KL = a cos C. ■

Note: In any triangle ABC, the angles of the pedal triangle are the supplements of twice the angles of the triangle.
The sides of the pedal triangle are a cos A, b cos B and c cos C.

DEFINITION 4.5 Excentric Triangle Let ABC C be a triangle and I1, I2 and I3 be centres of the escribed circles
which are opposite to A, B and C, respectively. Then the triangle I1I2I3 is called the excentric
triangle of ABC.

T H E O R E M 4.24 Let ABC C be a triangle, I its incentre and I1I2I3 be the excentric triangle of ABC. Then ABC
C is the
pedal triangle of I1I2I3 and I is the orthocentre of I1I2I3 (Figure 4.25).
PROOF Let I1, I2 and I3 be the centres of the escribed circles which are opposite to A, B and C, respec-
tively. I is the centre of the incircle of ABC. Then IC
C bisects the angle ACB and I1C bisects the
angle BCM. Therefore
ICI1 = ICB + I1CB

1 1
= ACB + BCM
2 2
1
= ( ACB + BCM )
2
1
= ´ 180° = 90°
2
4.3 Properties of a Triangle 225

A
I3 I2

B C

FIGURE 4.25 Theorem 4.24.

Similarly ICI2 = 90°. Therefore

I1CI2 = ICI1 + ICI2 = 90° + 90° = 180°

and hence I1CII2 is a straight line to which IC


C is a perpendicular. Also, since IA and I1A both bisect
the angle BAC, the three points I1, I and A are collinear; that is, I1IA is a straight line. Similarly,
I2IB and I3ICC are straight lines. Thus I1I2I3 is a triangle such that A, B and C are the feet of the
perpendiculars drawn from I1, I2 and I3 to I2I3, I3 I1 and I1I2, respectively, and further I is the point
of concurrence of these perpendiculars. Thus, I is the orthocenter of the triangle I1I2I3 and ABC
is the pedal triangle of I1I2I3. ■

DEFINITION 4.6 Medians and Centroid Let ABC C be a triangle and D, E and F be the mid-points of BC, CA
and AB, respectively. The lines AD, BE E and CF F are called the medians of the triangle ABC.
These three medians are concurrent and the point of concurrence is called the centroid of the
triangle and is usually denoted by G (Figure 4.26).

F E

FIGURE 4.26 Medians and centroid.

The concurrency of the medians is proved in Geometry at secondary school level. Also, it is known that
2 2 2
AG = AD, BG = BE and CG = CF
3 3 3
G is the point of trisection of each of the medians. In the following, we derive formulae
to express the lengths of the medians in terms of the elements of the given triangle.

T H E O R E M 4.25 Let ABC C be a triangle and D, E and F be the mid-points of BC, CA and AB, respectively. Then
the lengths of the medians are given by
1 1 2
AD = 2b2 + 2c2 - a2 = b + c2 + 2bc cos A
2 2
1 1 2
BE = 2c2 + 2a2 - b2 = c + a2 + 2ca cos B
2 2
1 1 2
and CF = 2a2 + 2b2 - c2 = a + b2 + 2ab cos C
2 2
226 Chapter 4 Properties of Triangles

PROOF From Theorem 4.2, we have

AD2 = AC 2 + CD2 - 2 AC ×CD cos C


2
æ aö
= b2 + ç ÷ - AC × BC cos C
è 2ø
a2
= b2 + - ab cosC
C
4
Therefore

a2
2 AD2 = 2b2 + - 2ab cos C
2
a2
= b2 - + (b2 + a2 - 2ab cos C )
2
a2
= b2 - + c2 (∵ c2 = b2 + a2 - 2ab cos C )
2

b2 a2 c2 1
AD2 = - + = (2b2 + 2c2 - a2 )
2 4 2 4
1 1 2
or AD = 2b2 + 2c2 - a2 = b + c2 + b2 + c2 - a2
2 2
1 2
= b + c2 + 2bc cos A
2

b2 + c2 - a2 = 2bc cos A).


Similarly,
1 1 2
BE = 2c2 + 2a2 - b2 = c + a2 + 2ca cos B
2 2
1 1 2
CF = 2a2 + 2b2 - c2 = a + b2 + 2ab cos C
2 2 ■

T H E O R E M 4.26 Let AD, BE


E and CF
F be the medians of a triangle ABC. Then

q f

E
F
G

a
C
D

FIGURE 4.27 Theorem 4.26.


4.3 Properties of a Triangle 227

a sin B
sin BAD =
2b + 2c2 - a2
2

a sin C
sin CAD =
2b + 2c2 - a2
2

2b sin C
sin ADC =
2b2 + 2c2 - a2

PROOF Put q = BAD, f = CAD and a = ADC . See Figure 4.27. Then

sin f DC a
= =
sin C AD 2 AD

a sin C a sin C
sin f = =
2 AD 2b + 2c2 - a2
2

Similarly
a sin B
sin q =
2b + 2c2 - a2
2

Also
sin a AC b
= =
sin C AD AD

2 b sin C
sin a =
2b2 + 2c2 - a2 ■

T H E O R E M 4.27
it in the ratio 1: 2.

F
P L
G
O

FIGURE 4.28

PROOF Let ABC C be a triangle and O and P be its circumcentre and orthocentre, respectively. Draw
OD and PK K perpendicular to BC. Let AD and OP P meet at G. The triangles OGD and PGA are
similar. See Figure 4.28. From Theorem 4.15, we have OD = R cos A. Also, from Theorem 4.22,
AP = 2R cos A.
228 Chapter 4 Properties of Triangles

Since the triangles OGD and PGA are similar, we have


AG AP
= =2
GD OD
Therefore, the point G is the centroid of the triangle. Also
OG OD 1
= =
GP AP 2
Therefore, the centroid lies on the line joining the circumcentre to the orthocentre and divides it
in the ratio 1: 2. ■

DEFINITION 4.7 Nine-point Circle It is known that the feet of the perpendiculars, the mid-points of the sides
and the mid-points of the line joining the vertices to the orthocentre are all concyclic. The circle
passing through these nine points is called the nine-point
- circle.

Also, it is known that the centre of the nine-point circle lies on the line joining the circumcentre O and the orthocentre
P and it bijects OP. Therefore, the circumcentre, the centroid, the centre of the nine-point circle and the orthocentre
all lie on a straight line.

T H E O R E M 4.28 Let ABC C be a triangle and R its circumradius. Then the distance between the circumcentre O and
the orthocentre P is given by

OP = R 1 - 8 cos A cos B cos C

PROOF Let F be the foot of the perpendicular drawn from O to AB. Then
FAO = 90° - FOA = 90° - C

and from Figure 4.28


LAP = 90° - C

Therefore
PAO = A - FAO - LAP
= A - 2(90° - C )
= A + 2C - 180°
= A + 2C - ( A + B + C ) = C - B

Also, OA = R and, by Theorem 4.22, we have


PA = 2R cos A
Therefore

OP 2 = OA2 + PA2 - 2OA × PA cos PAO


= R2 + 4 R2 cos2 A - 4 R2 cos A cos(C - B)
= R2 + 4 R2 cos A[cos A - cos(C - B)]
= R2 + 4 R2 cos A[cos{180 - ( B + C )} - cos(C - B)]
= R2 + 4 R2 cos A[- cos(C + B) - cos(C - B)]
= R2 - 4 R2 cos A[cos(C + B) + cos(C - B)]
4.3 Properties of a Triangle 229

= R2 - 4 R2 cos A × 2 cos B cos C


= R2 (1 - 8 cos A cos B cos C )

Therefore

OP = R 1 - 8 cos A cos B cos C ■

T H E O R E M 4.29 Let ABC C be a triangle and R its circumradius. Then the distance between the circumcentre O and
the incentre I is given by

A B C
OI = R 1 - 8 sin sin sin = R(R - 2r )
2 2 2

where r is the inradius.

E
F I
O

FIGURE 4.29 Theorem 4.29.

PROOF As above, let OF


F be perpendicular to AB and IE
E be perpendicular to AC (Figure 4.29). As in
Theorem 4.27, we have OAF = 90° - C . Therefore
IAO = FAI - OAF
A
= - (90° - C )
2
A A+ B+C
= +C -
2 2
C-B
=
2
Also
IE r B C
IA = = = 4 R sin sin
sin( A / 2) sin( A / 2) 2 2
Therefore

OI 2 = OA2 + IA2 - 2OA × IA cos IAO


2
æ B Cö æ B Cö C-B
= R2 + ç 4 R sin sin ÷ - 2 R ç 4 R sin siin ÷ cos
è 2 2ø è 2 2ø 2
B 2C B C C-B
= R2 + 16 R2 sin2 sin - 8 R2 sin sin cos
2 2 2 2 2
230 Chapter 4 Properties of Triangles

é B C B Cæ B C B Cöù
= R2 ê1 + 16 sin2 sin2 - 8 sin sin ç cos cos + sin sin ÷ ú
ë 2 2 2 2 è 2 2 2 2øû
é B Cæ B C B Cöù
= R2 ê1 - 8 sin sin ç cos cos - sin sin ÷ ú
ë 2 2 è 2 2 2 2øû
é B C B+Cù
= R2 ê1 - 8 sin sin cos
ë 2 2 2 úû
é B C Aù
= R2 ê1 - 8 sin sin sin ú
ë 2 2 2û

A B C
OI = R 1 - 8 sin sin sin
2 2 2
Also
A B C
OI 2 = R2 - 2 R × 4 R sin sin sin = R2 - 2Rr (by Corollary 4.3)
2 2 2
Hence

OI = R(R - 2r ) ■

QUICK LOOK 2

An argument similar to the above can be used to derive A B C


OI2 = R 1 + 8 cos sin cos = R(R + 2r2 )
formulae for the distances between the circumcentre O 2 2 2
and the excentres I1, I2 and I3. These formulas are as follows.
f
A B C
and OI3 = R 1 + 8 cos R + 2r3 )
cos sin = R(R
A B C 2 2 2
OI1 = R 1 + 8 sin cos cos = R(R + 2r1 )
2 2 2 where R, r1, r2 and r3 are, respectively, the circumradius
and the exradii.

T H E O R E M 4.30 Let ABC


C be a triangle and AD the bisector of the angle A meeting BC
C at D. Then
2bc A
AD = cos
b+c 2
A
and ADC = +B
2

b
c

t
s C
D

FIGURE 4.30 Theorem 4.30.


4.3 Properties of a Triangle 231

PROOF C such that BD = s and DC = t (Figure 4.30). Then, we know from


Suppose that D divides BC
Geometry that
s AB c
= =
t AC b
Therefore
s t s+t a
= = =
c b c+b c+b
Hence
ac ab
s= and t =
b+c b+c
Also, since the sum of the areas of the triangles ABD and ACD is equal to the area of the triangle
ABC, we have
1 A 1 A 1
AB × AD sin + AC × AD sin = AB × AC sin A
2 2 2 2 2
Therefore

A
AD sin × (c + b) = cb sin A
2
bc sin A 2bc A
AD = × = cos
b + c sin( A / 2) b + c 2

Further,
A
ADC = ABC + BAD = B +
2
Similarly, if BE
E and CFF are the bisectors of the angles B and C respectively, then we have the
following (Figure 4.31):
2ca B B
BE = cos and BEA = +C
c+a 2 2

2ab C C
CF = cos and CFB = +A
a+b 2 2

F E

FIGURE 4.31 ■
232 Chapter 4 Properties of Triangles

WORKED-OUT PROBLEMS
Single Correct Choice Type Questions
1. In a triangle ABC, Solution: We have
(a + b)cos C + (b + c)cos A + (c + a)cos B = A C 5 2 1
tan × tan = × =
a+b+c a+b+c 2 2 6 5 3
(A) (B)
4 2
a+b+c
(C) (D) a + b + c ( s - b)( s - c) ( s - a)( s - b) 1
3 × =
s( s - a) s( s - c) 3
Solution:
s-b 1
(a + b)cos C + (b + c)cos A + (c + a)cos B =
s 3
= (a cos C + c cos A) + (b cos C + c cos B)
This gives
+ (b cos A + a cos B)
3s - 3b = s
=b+a+c
Þ 2s = 3b
=a+b+c
Therefore
Answer: (D)
a + b + c = 3b
2. In a triangle ABC, if (a + b + c)(b + c - a) = 3bc, then Þ a + c = 2b
the angle A is equal to Answer: (C)
(A) 30° (B) 60° (C) 45° (D) 120°
4. In a triangle ABC, if a = 6, b = 5 and c = 9, then angle A
Solution: is equal to
(a + b + c)(b + c - a) = 3bc æ 7ö æ 5ö
(A) Cos-1 ç ÷ (B) Cos-1 ç ÷
Þ 2 s(2 s - 2a) = 3bc è 9ø è 9ø

s( s - a) 3 æ 7ö æ 4ö
Þ = Sin-1 ç ÷ (D) Cos-1 ç ÷
bc 4 è 9ø è 9ø
A 3 Solution: From the cosine formula, we have
Þ cos2 =
2 4
b2 + c2 - a2
A 3 cos A =
Þ cos = ± 2bc
2 2
52 + 92 - 62 7
A 3 æ A ö = =
Þ cos = çè∵ < 90°÷ 2´5´9 9
2 2 2 ø
A
Þ = 30°
2 æ 7ö
A = Cos-1 ç ÷
Þ A = 60° è 9ø
Answer: (B) Answer: (A)

3. In a triangle ABC, if 5. In a triangle ABC, if a = 13, b = 14 and c = 15, then


circumdiameter of the triangle is
A 5 C 2
tan = and tan = 65 65 64 56
2 6 2 5 (A) (B) (C) (D)
3 4 5 4
a, b and c are
Solution: We have
(A) in GP (B) in HP
(C) in AP (D) equal 2s = a + b + c = 13 + 14 + 15 = 42
Worked-Out Problems 233

Therefore Therefore
s = 21, s - a = 8, s - b = 7 and s - c = 6 a : b : c = sin A :sin B :sin C
By Corollary 4.2, = sin 60° :sin 30° :sin 90°
abc 3 1
R= = : :1
4 s( s - a)( s - b)( s - c) 2 2
13 ´ 14 ´ 15 = 3 : 1: 2
=
4 21 ´ 8 ´ 7 ´ 6 Answer: (D)
13 ´ 14 ´ 15
= 8. In a triangle ABC, if
4 ´ 84
1 1 3
Therefore the circumdiameter is + =
b+c c+a a+b+c
65 then C =
2R =
4
(A) 30° (B) 60° (C) 45° (D) 75°
Answer: (B)
Solution: From the given equation, we have
6. In a triangle ABC, (c + a) + (b + c) 3
=
C B (b + c)(c + a) a+b+c
b cos2 + c cos2 =
2 2
a+b+c
(A) (B) a + b + c (a + b + 2c)(a + b + c) = 3(b + c)(c + a)
3
a+b+c a+b+c a2 + ab + ac + ab + b2 + bc + 2ca
(C) (D)
2 4
+ 2bc + 2c2 = 3(bc + ab + c2 + ca)
Solution: By Theorem 4.5
a2 + b2 - c2 = ab
C B æ s( s - c) ö æ s( s - b) ö
b cos2 + c cos2 = b ç ÷ + cç Hence
2 2 è ab ø è ca ÷ø
a2 + b2 - c2 ab 1
s( s - c) s( s - b) cosC = = =
= + 2ab 2ab 2
a a
C = 60°
s(2 s - b - c)
= Answer: (B)
a
sa a+b+c 9. In a triangle ABC,
= =s=
a 2
æ C Aö
Answer: (C) 2 ç a sin2 + c sin2 ÷ =
è 2 2ø
7. In DABC, if A = 60° and B = 30°, then a : b : c = a+b+c (B) b + c - a
(A) 2 : 1 : 3 (B) 1 : 3 : 2 (C) a + b - c (D) a + c - b

(C) 3 :2: 2 (D) 3 : 1: 2 Solution: We have

Solution: Since A + B + C = 180°, we have æ C Aö


2 ç a sin2 + c sin2 ÷ = a(1 - cos C ) + c(1 - cos A)
è 2 2ø
60° + 30° + C = 180°
= a - (a cos C + c cos A) + c
and hence C = 90°. From the sine formula, we have
=a+c-b
a b c
= = Answer: (D)
sin A sin B sin C
234 Chapter 4 Properties of Triangles

10. In a triangle ABC, a sin(B - C) + b sin(C


C - A) + æ A - Bö
sin ç
c sin(A - B) = è 2 ÷ø
=
(A) s (B) 1 (C) 0 (D) 2s C
cos
Solution: We know that 2
Therefore
a b c
= = = K , say æ A - Bö C
sin A cos B cos C c sin ç = (a - b)cos
è 2 ÷ø 2
Then
Answer: (B)
a sin(B - C ) + b sin(C - A) + c sin( A - B)
= K sin A sin( B - C ) + K sin B sin(C - A) 12. If A, B and C are the angles of a triangle, then

+ K sin C sin( A - B) sin(B + C )


=
sin(B - C )
= K[sin(B + C )sin(B - C ) + sin(C + A)sin(C - A)
+ sin( A + B) sin( A - B)] b+c b2 + c2
(A) (B)
b-c b2 - c2
= K[(sin2 B - sin2 C ) + (sin2 C - sin2 A)
b2 - c2 a2
(C) (D)
+ (sin A - sin B)]
2 2
a2 b - c2
2

=K´0=0 Solution:
K is the diameter of the circumcircle of DABC
D . sin( B + C ) sin2 ( B + C )
=
Answer: (C) sin( B - C ) sin( B - C )sin( B + C )

11. In a triangle ABC, sin2 (p - A)


=
sin2 B - sin2 C
æ A - Bö
c sin ç =
è 2 ÷ø =
4 R2 sin2 A
4 R2 (sin2 B - sin2 C )
C
(A) (a - b) cos C (B) (a - b)cos
2 a2
=
a-b a-b C b - c2
2

(C) × cos C (D) × cos


2 2 2 Answer: (D)
Solution: Let
C is a triangle with area D, then cot A + cot B +
13. If ABC
a b c cot C =
=K= =
sin A sin B sin C a2 + b2 + c2 a2 + b2 + c2
(A) (B)
Then 2D 4D

a - b K sin A - K sin B a+b+c a2 + b2 + c2


= (C) (D)
c K sin C 4D D

sin A - sin B Solution:


=
sin C cot A + cot B + cot C
æ A + Bö æ A - Bö cos A cos B cos C
2 cos ç = + +
è 2 ÷ø
sin ç
è 2 ÷ø sin A sin B sin C
=
C C
2 sin cos b2 + c2 - a2 c2 + a2 - b2 a2 + b2 - c2
2 2 = + +
2bc sin A 2ca sin B 2ab sin C
æ Cö æ A - Bö
cos ç 90° - ÷ sin ç
è 2ø è 2 ÷ø (by cosine formula)
=
C C
sin cos
2 2
Worked-Out Problems 235

1 2 2 sin A cos B - cos A sin B


= (b + c - a2 + c2 + a2 - b2 + a2 + b2 - c2 )
4D cos A cos B sin C - sin B
=
1 sin A cos B + cos A sin B sin C
(since D = bc sin) cos A cos B
2
1 2 sin( A - B) sin( A + B) - sin B
= (a + b2 + c2 ) =
4D sin( A + B) sin( A + B)
Answer: (B) [∵ C = p - ( A + B)]

14. In a triangle ABC, sin( A - B) = sin( A + B) - sin B


sin( A - B) - sin( A + B) = - sin B
æ A Bö
(a + b + c) ç tan + tan ÷ =
è 2 2ø sin( A + B) - sin( A - B) = sin B
C A 2 cos A sin B = sin B
(A) 2c cot (B) 2c tan
2 2
1
C C cos A = = cos 60°
(C) (a + b + c)cot (D) c cot 2
2 2
Therefore A = 60°.
Solution:
Answer: (C)
æ A Bö æ D D ö
(a + b + c) ç tan + tan ÷ = 2 s ç +
è 2 2ø è s( s - a) s( s - b) ÷ø 16. In a triangle ABC, if a2, b2, c2 are in AP, then cot A,
cot B, cot C are
æ 1 1 ö (A) in GP (B) in AP (C) in HP (D) equal
= 2D ç +
è s - a s - b ÷ø 2 2 2
Solution: Since a , b , c are in AP, we have
æ s-b+ s-a ö
= 2D ç b2 - a2 = c2 - b2
è ( s - a)( s - b) ÷ø
Therefore
2s - a - b
= 2D ×
( s - a)( s - b) sin2 B - sin2 A = sin2 C - sin2 B
c sin(B + A)sin( B - A) = sin(C + B)sin(C - B)
= 2D ×
( s - a)( s - b) sin C sin( B - A) = sin A sin(C - B)
D sin( A - B) sin(B - C )
= 2c × =
( s - a)( s - b) sin A sin C
C sin A cos B - cos A sin B sin B cos C - cos B sin C
= 2c cot =
2 sin A sin B sin C sin B
Answer: (A)
cot B - cot A = cot C - cot B
15. In a triangle ABC, if Therefore cot A, cot B, cot C are in AP.
tan A - tan B c - b Answer: (B)
=
tan A + tan B c
17. In DABC, a2 sin 2B + b2 sin 2A
2 is equal to
then angle A is equal to
(A) 2D (B) 3D (C) 4D (D) 6D
(A) 30° (B) 45° (C) 60° (D) 75°
Solution:
Solution: The given equation can be written as
a2 sin 2 B + b2 sin 2 A = 2a2 sin B cos B + 2b2 sin A cos A
sin A sin B
- = 2a(b sin A)cos B + 2b(a sin B)cos A
cos A cos B sin C - sin B æ sin B sin C ö
= çè∵ = ÷
sin A sin B sin C b c ø = 2ab sin( A + B)
+
cos A cos B
236 Chapter 4 Properties of Triangles

= 2ab sin C 20. In DABC


D , a3 sin(B - C) + b3 sin(C - A) + c3 sin(A
( - B)
equals
= 4D
(A) 0 (B) abc (C) 2abc (D) 3abc
Answer: (C)
Solution:
18. In DABC, if 8R = a + b + c , then the triangle is
2 2 2 2

a3 sin( B - C ) = a2 (2 R sin A)sin( B - C )


(A) isosceles (B) right angled
(C) equilateral (D) scalene = 2 Ra2 sin(B + C )sin( B - C )

Solution: = 2 Ra2 (sin2 B - sin2 C )

8 R2 = 4 R2 (sin2 A + sin2 B + sin2 C ) æ b2 c2 ö


= 2 Ra2 ç 2 -
è 4R 4 R2 ÷ø
1 - cos 2 A 1 - cos 2 B 1 - cos 2C
2= + +
2 2 2 1 2 2
= a (b - c 2 )
-1 = co
os 2 A + cos 2 B + cos 2C 2R

= - 1 - 4 cos A cos B cos C Therefore

Hence åa 3
sin(B - C )
cos A cos B cos C = 0 1 2 2
= [a (b - c2 ) + b2 (c2 - a2 ) + c2 (a2 - b2 )]
So one of the angles of the triangle is a right angle. 2R
Answer: (B) 1
= (0 ) = 0
2R
19. In DABC
D , a3 cos(B - C) + b3 cos(C - A) + c3 cos(A
( - B) Answer: (A)
is equal to
(A) abc (B) 2abc (C) 0 (D) 3abc 21. In DABC,

Solution: C C
(a - b)2 cos2 + (a + b)2 sin2 =
2 2
a3 cos( B - C ) = a2 (2 R sin A)cos( B - C )
(A) a2 + b2 + c2 (B) abc(a + b + c)
= 2 Ra2 sin( B + C )cos( B - C )
(C) (abc)D (D) c2
= Ra2 (sin 2 B + sin 2C ) Solution: We have
= 2 Ra (sin B cos B + sin C cos C )
2
C C
(a - b)2 cos2 + (a + b)2 sin2
æ b c ö 2 2
= 2 Ra2 ç cos B + cos C ÷
è 2R 2R ø (1 + cos C ) (1 - cos C )
= (a - b)2 + (a + b)2
= a (b cos B + c cos C )
2 2 2
1 1
Similarly = [(a - b)2 + (a + b)2 ] + [(a - b)2 - (a + b)2 ] cos C
2 2
b3 cos(C - A) = b2 (a cos A + c cos C ) 1
= a2 + b2 + (-4ab cos C )
2
and c3 cos( A - B) = c2 (a cos A + b cos B)
= a2 + b2 - 2ab cos C
Therefore
= c2
åa 3
cos( B - C ) Answer: (D)
= ab(a cos B + b cos A) + bc(b cos C + c cos B)
22. In DABC, if the angles A, B and C are in the ratio
+ ca(c cos A + a cos C ) 1:2:3, then the sides a, b, c are in the ratio
= ab(c) + bc(a) + ca(b) (A) 1: 2 : 3 (B) 1 : 3 : 2
= 3abc (C) 1 : 2 : 3 (D) 3 : 1: 2
Answer: (D)
Worked-Out Problems 237

Solution: Let A = q, B = 2q
q and C = 3q. Therefore (A) a, b, c (B) b, a, c or c, a, b
180° = 6q (C) b, c, a or a, c, b (D) c, b, a

Þ q = 30° Solution:

This implies A = 30°, B = 60°, C = 90°. Therefore A


cos B + cos C = 4 sin2
2
a : b : c = 2 R sin A : 2 R sin B : 2 R sin C
æ B + Cö æ B - Cö A
2 cos ç cos ç = 4 sin2
= sin 30° :sin 60° :sin 90° è 2 ÷ø è 2 ÷ø 2

=
1 3 æ Aö æ B - Cö A
: :1 sin ç ÷ cos ç = 2 sin2
2 2 è 2ø è 2 ÷ø 2
= 1: 3 : 2 æ B - Cö A
cos ç = 2 sin
Answer: (B) è 2 ÷ø 2

23. In DABC, if A = 30° and the area D is ( )


3 / 4 a2 , then æ Aö æ B - Cö
2 cos ç ÷ cos ç
è 2ø è 2 ø ÷
A
= 4 sin cos
2
A
2
the difference of the angles B and C is
(A) 75° (B) 60° (C) 90° (D) 45° æ B + Cö æ B - Cö
2 sin ç ÷ cos ç = 2 sin A
Solution: è 2 ø è 2 ÷ø
sin B + sin C = 2 sin A
3 2 1 1 sin B sin C
a = D = ab sin C = a2 b + c = 2a
4 2 2 sin A
Answer: (B)
Therefore
25. In DABC, if AD is the internal bisector of the angle A
3
sin B sin C = (∵ 2 sin A = 1) meeting the side BC
C in D, then the length of AD is
4
æ 2bc ö A
Also (A) ç ÷ cos (B) 2b2 + 2c2 - a2
è b + cø 2
3 abc abc
= cos A (C) (D)
2 b+c a+b+c
= - cos( B + C ) A
= - cos B cos C + sin B sin C
A A
3 2 2
= - cos B cos C + b
4
Therefore c

3 C
cos B cos C = -
4 b

Now D

cos( B - C ) = cos B cos C + sin B sin C c

3 3
=- + =0
4 4 FIGURE 4.32 Single correct choice type question 25.

which gives | B - C | = 90°. Solution: See Figure 4.32. It is known that BD : DC =


Answer: (C) AB : AC = c : b. Therefore

24. In DABC, if cos B + cos C = 2(1 - cos A), then the ac ab


BD = and DC =
order in which the sides a, b, c are in AP is b+c b+c
238 Chapter 4 Properties of Triangles

Also Therefore
1 A AD[c sin( A / 2)] BG = 6 sin 60° = 3 3 and CG = 6 sin 30° = 3
Area of D ABD = ( AB)( AD)sin =
2 2 2
Also
AD[b sin
n( A / 2)]
Area of D ACD = Area of DABC = 3 (Area of DBGC)
2
æ1 ö
Now = 3 ç BG × CG÷
è2 ø
1 3
bc sin A = Area of D ABC = D ABD + D ACD = ´3 3´3
2 2
( AD)sin( A / 2) 27
= (b + c ) = 3
2 2
Answer: (C)

æ A Aö A 27. In DABC, if b = 3, c = 5 and cos(B - C) = 7/25, then


bc ç 2 sin cos ÷ = ( AD)(b + c)sin
è 2 2ø 2 the value of “a” is
æ 2bc ö A (A) 10 (B) 15
AD = ç ÷ cos
è b + cø 2 (C) 6 (D) 5
Answer: (A) Solution:
26. In DABC, BC = 6 units. BE
E and CF
F are medians 7 1 - tan2 [( B - C )/ 2]
such that CBE = 30° and BCF = 60°. Then the area = cos( B - C ) =
25 1 + tan2 [( B - C )/ 2]
of DABCC is
æ B - Cö æ B -Cö
7 + 7 tan2 ç = 25 - 25 tan2 ç
è 2 ÷ø è 2 ÷ø
9 3
(A) 8 3 (B)
2
æ B -Cö
32 tan2 ç = 18
è 2 ÷ø
27 3
(C) (D) 9 3
2
Therefore
A
æ B - C ö -3
tan ç =
è 2 ÷ø 4
E
b < c. Now by Napier’s rule
-3 B-C
F G = tan
C 4 2
60°
æ b - cö A
=ç ÷ cot
6 è b + cø 2
30°
æ 3 - 5ö A
=ç ÷ cot
è 3 + 5ø 2
FIGURE 4.33

Solution: See Figure 4.33. It is known that the medians A 1


tan =
intersect at the centroid G of DABC. By hypothesis 2 3
BGC = 90°. Also, it is known that the area of DABCC is
three times of the area of DBGC. In DBGC, Again

BG CG BC 1 - tan2 ( A / 2) 1 - (1/ 9) 4
= = =6 cos A = = =
sin 60° sin 30° sin 90° 1 + tan2 ( A / 2) 1 + (1/ 9) 5
Worked-Out Problems 239

Therefore Therefore

a2 = b2 + c2 - 2bc cos A (cosine formula) l+5 x


= (4.4)
22 - l 8
4
= 9 + 25 - 2(3)(5)
5 From Eqs. (4.3) and (4.4),
= 34 - 24 5(30 - l ) 8(l + 5)
=x=
= 10 l 22 - l

Hence, a = 10 . Therefore
Answer: (A) l 2 + 100 l - 50 ´ 22 = 0

28. E and F are interior points on the sides AC


C and AB, (l + 110)(l - 10) = 0
respectively, of DABC. The lines BE E and CFF inter-
This implies l = 10 and x = 10. Hence area of DBPC
sect in P. If the area of DBPF F is 5, the area of the
is 10.
quadrilateral AEPF F is 22 and the area of DCPE E is 8,
then the area of DBPC C is equal to Answer: (D)
(A) 20 (B) 15 (C) 12 (D) 10 29. Two circles with radii R and r (R > r) touch each
Solution: Let the area of DAPF F be l so that the area other internally at A. ABC C is an equilateral triangle
of DAPEE is 22 - l. Suppose the area of DBPCC is x. See with B on one circle and C on the other. Then the
Figure 4.34. length of the side of the equilateral triangle ABC C is
2 Rr 3 Rr 3
A (A) (B)
R - Rr + r
2 2
R - Rr + r2
2

Rr 3 2 Rr 3
E (C) (D)
22 l R + Rr + r2
2
R + Rr + r2
2
l

F P 8
Q
5 C
x

FIGURE 4.34 Single correct choice type question 28.

P
(A) We have
B
CP Area of DBPC x
= =
PF Area of DBPF 5
Again, C

CP Area of D APC 30 - l
= = D
PF Area of D APF l
Therefore
x 30 - l
= (4.3) FIGURE 4.35 Single correct choice type question 29.
5 l
(B) In a similar way, Solution: Let the diameter AP P of the inner circle meet
the outer circle in Q. Let x be the length of the side of
BP l + 5 BP x the equilateral DABC. Suppose the side AC C meet the
= and =
PE 22 - l PE 8 inner circle in D. Join the point C to Q and the point D
to P (Figure 4.35).
240 Chapter 4 Properties of Triangles

Therefore sin B sin C = sin(A - B)(sin A - sin B)


ACQ = ADP = 90° sin B sin( A + B) = sin A sin( A - B)
- sin B sin( A - B)
Hence D ACQ and D ADP
P are similar triangles. From
DBDA, sin B[sin( A + B) + sin( A - B)] = sin A sin( A - B)
BD sin A(sin 2 B) = sin A sin( A - B)
= 2r
sin 60° sin 2 B = sin( A - B)
Therefore This implies
BD = 3r A - B = 2 B or A - B = p - 2B

Also, from the similarity of triangles Þ A = 3B or A+ B=p

AD AP r But A + B cannot be p
p. Therefore A = 3B which means
= = a = 3b.
AC AQ R
Answer: (A)
Therefore
rx 31. In DABC , ABC = 45°. D is a point on the segment
AD = (∵ AC = x) BC such that 2BD = DC
C and DAB = 15°. Then
R
ACB is
Using cosine rule for DABD, (A) 75° (B) 60° (C) 90° (D) 105°
3r = ( BD) = ( AD) + ( AB) - 2( AD)( AB)cos 60°
2 2 2 2
Solution: Let DAC = q so that
r 2 x2 æ rx ö æ 1ö A = q + 15°
= + x2 - 2 ç ÷ ( x) ç ÷
R2 è Rø è 2ø
Note that
x2
= 2 (R2 - Rr + r 2 )
R ADC = 45° + 15° = 60°
3Rr See Figure 4.36.
x=
R - Rr + r
2 2

A
Answer: (B)

30. The sides a, b and c of a triangle satisfy the equality q


bc2 = (a + b)(a - b)2. If a and b are the angles oppo-
15°
site to a and b, respectively, then C
(A) a = 3b b (B) a = 2b
60°
(C) b = 2a
a (D) b = 3a
45°
Solution: Let a = BC, b = CA and c = AB. So a = A, D
b = B. Now by hypothesis

bc2 = (a + b)(a - b)2 FIGURE 4.36

Therefore From DCAD,


sin B sin C = (sin A + sin B)(sin A - sin B)
2 2
CD AC
= (4.5)
= (sin2 A - sin2 B)(sin A - sin B) sin q sin 60°

= sin( A + B)sin( A - B)(sin A - sin B) Also from DABC,

= sin C sin( A - B)(sin A - sin B) BC AC


= (4.6)
sin(q + 15°) sin 45°
Worked-Out Problems 241

Therefore from Eqs. (4.5) and (4.6) Also COD = BOD = 60°. From DDOC,
(CD)sin(q + 15°) sin 45° DC
= tan 60° =
( BC )sin q sin 60° OC
CD sin q sin 45°
= Therefore by Eq. (4.7) we have
BC sin(q + 15°°)sin 60°
2 sin q sin 45° DC = r 3 = ( AC ) 3
=
3 sin(q + 15°)sin 60° Answer: (C)

But 33. In an isosceles triangle one angle is 120° and the


2 radius of its inscribed circle is 3. Then, the area of
2 æ sin 45° ö
= the triangle is
3 çè sin 60° ÷ø
(A) 12 + 7 3 (B) 7 + 12 3
Therefore
(C) 12 - 7 3 (D) 4p
2
æ sin 45° ö sin q sin 45° Solution: See Figure
F 4.38. In DABC
D , let A = 120°, AB =
çè sin 60° ÷ø = sin(q + 15°) × sin 60° AC. Then

Clearly q = 45° is a solution of the above equation. There- 1 1 3 3 2


D= bc sin 120° = b2 = ×b
fore A = 60°, B = 45° and C = 75°. 2 2 2 4
Answer: (A)
A
32. AB is a diameter of a circle with centre at O and
radius r. D is a point on the tangent at B and DC c 120° b
is the other tangent to the circle at C such that
COA = 60°. Then
30° 30°
(A) (DC ) 3 = r = AC
FIGURE 4.38 Single correct choice type question 33.
(B) r 2 = DC = ( AC ) 3

(C) DC = r 3 = ( AC ) 3 Again

(D) (DC ) 2 = r = AC a b
= Þ a = 3b
sin 120° sin 30°
Solution: See Figure 4.37.
Now
( AC )2 = 2r 2 - 2r 2 cos 60° = r2 (4.7)
2 s = a + b + c = a + 2b
B
D = b( 3 + 2)

Therefore

3 2 b(2 + 3 )
b = D = rs = 3 ×
4 2
O
r Hence b = 2(2 + 3 ). So
r 60°
C 3 2 3
D= b = × 4 (2 + 3 )2
4 4
= 3 (7 + 4 3 ) = 12 + 7 3

FIGURE 4.37
Answer: (A)
242 Chapter 4 Properties of Triangles

34. In DABC, AB = 3, BC = 5, AC = 7 and ABC = 120°. Alternate Solution:


The angle bisectors AD and CE
E of A and C meet
in P. Then the length AP
P is A ( s - b)( s - c)
sin =
2 bc
(A) 6 (B) 6 (C) 7 (D) 7
[(15 / 2) - 7][(15 / 2) - 3]
Solution: Note that P is the incentre of DABC. Draw =
PL perpendicular to AB so that PL is the inradius r of 7´3
DABCC (Figure 4.39). 3
=
B
2 7
5
D
L

E AP = The distance between the vertex A


3
P C and the incentre P
A
7 = r cosec
2

FIGURE 4.39 Single correct choice type question 34. 3 2 7


= × [By Eq. (4.8)]
2 3
Now
= 7
1 Answer: (D)
rs = D = ( AB)(BC )sin 120°
2
35. In DABC
D , C = 90° and AB = 60. The medians AD and
1 3 15 3
= (3)(5) = E intersect at G (centroid). If q is the acute angle
BE
2 2 4
between the medians AD and BE E whose tangent
Therefore value is 1/3, then the area of DABC
D C is
(A) 200 (B) 300 (C) 400 (D) 500
æ 15 ö 15 3
rç ÷ = Solution: See Figure 4.40. We have
è 2ø 4
1
3 BGD = AGE = q and tan q =
r= (4.8) 3
2
By hypothesis
From DBPL,
( BC )2 + ( AC )2 = ( AB)2
PL r 3
tan 60° = = =
BL BL 2 BL a2 + b2 = 602 (4.9)
Therefore Consider the quadrilateral DGEC
C in which

3 1 1 DGE + GDC + 90° + CEG = 360°


BL = ´ =
2 3 2
A
1 5
AL = AB - BL = 3 - =
2 2
Hence
60
25 3
AP 2 = AL2 + PL2 = + =7 E
4 4 G q
Þ AP = 7
q

FIGURE 4.40 Single correct choice type question 35.


Worked-Out Problems 243

Therefore 37. AD, BE F are medians of DABC. Then (AD)2 +


E and CF
(BE)2 + (CF)
F 2 = K(a2 + b2 + c2) where the value of K is
(180° - q ) + D + 90° + E = 360°
3 4 2 3
(A) (B) (C) (D)
where D = GDC and E = CEG. This gives 4 3 3 2
D + E = 90° + q (4.10) A

Therefore
- 3 = - cot q = tan(90° + q ) = tan(D + E) [from Eq. (4.10)]
tan D + tan E
= C
1 - tan D tan E D

( AC / DC ) + ( BC / EC ) FIGURE 4.42 Single correct choice type question 37.


=
1 - ( AC / DC ) × ( BC / EC )
Solution: D is the mid-point of the side BC (Figure 4.42).
(2b / a) + (2a / b)
= Using cosine rule for DABD
D , we get
1- 4
( AD)2 = ( AB)2 + ( BD)2 - 2( AB)( BD)cos B
Therefore
æ aö æ c + a - b ö
2 2 2
a2
æ ab ö = c2 + - 2(c) ç ÷ ç ÷ø
9 ç ÷ = a2 + b2 = 3600 4 è 2ø è 2ca
è 2ø
9 D = 3600 a2 c2 + a2 - b2
= c2 + -
4 2
D = 400
2c2 + 2b2 - a2
Answer: (C) =
4
36. In DABC
C if the line joining the incentre and centroid Therefore
is parallel to the side BC, then the sides a, b and c
(in certain order) form a/an 1
( AD)2 + ( BE)2 + (CF )2 = (4a2 + 4b2 + 4c2 - (a2 + b2 + c2 ))
(A) AP (B) GP (C) HP (D) AGP 4
Solution: In DABC, AE E is the bisector of A, AD is 3
= (a2 + b2 + c2 )
the median, I is the incentre and G is the centroid 4
(Figure 4.41). IG is parallel to BC. This implies Hence K = 3/4.
AI AG 2 Answer: (A)
= =
IE GD 1
38. The sides of a triangle are in AP. Its area is 3/5 times
b+c 2
= [∵ I divides AE is the ratio (b + c): a] the area of an equilateral triangle whose perimeter is
a 1 so as the given triangle. Then the ratio of the sides is
b + c = 2a (A) 5 : 3 : 7 (B) 3 : 5 : 7 (C) 7 : 4 : 3 (D) 3 : 4 : 7
Hence c, a, b are in AP. Solution: Let the sides be x - d, x and x + d. Therefore
the perimeter
A
3x
2 s = 3 x or s =
2
I G Then
B
E
D D = s( s - x + d)( s - x)( s - x - d)

x æ x2 ö
FIGURE 4.41 Single correct choice type question 36. = 3ç - d2 ÷
2 è 4 ø
Answer: (A)
x
= 3( x2 - 4d2 )
4
244 Chapter 4 Properties of Triangles

But the area of the equilateral triangle with the same semi- Now
perimeter 3x/2 is ( 3 / 4) x2. Therefore p q
sin a = and sin b =
b a
3æ 3 2ö
D= x ÷
5 çè 4
Therefore
ø
1 2
x2 9 3 cos a = b - p2
´ 3( x2 - 4d2 ) = ´ ´ x4 b
16 25 16
1 2
9 2 cos b = a - q2
x2 - 4 d 2 = ×x a
25
sin C = sin[p - (a + b )]
16 2
x = 4d2 = sin a cos b + cos a sin b
25
5 1
x= d = [ p a2 - q2 + q b2 - p2 ]
2 ab

Hence the sides of the triangle are


5 3d a2 b2 sin2 C = [ p a2 - q2 + q b2 - q2 ]2 (4.11)
x-d= d-d=
2 2
5d Now
x=
2 a2 p2 + b2 q2 - 2abpq cos C
5d 7
and x+d= +d= d = a2 p2 + b2 q2 + 2abpq cos(a + b )
2 2
= a2 p2 + b2 q2 + 2abpq[cos a cos b - sin a sin b ]
Therefore the ratio of the sides is 3 : 5 : 7.
Answer: (B) é1 pq ù
= a2 p2 + b2 q2 + 2abpq ê (a2 - q2 )(b2 - p2 ) -
ë ab ab úû
39. If p and q are the lengths of the perpendiculars
drawn from the vertices A and B of DABC C onto any = a2 p2 + b2 q2 + 2 pq[ (a2 - q2 )(b2 - p2 ) - pq]
line passing through the vertex C, then the value of
a2p2 + b2q2 - 2abpq cos C is = a2 p2 + b2 q2 - 2 p2 q2 + 2 pq (a2 - q2 )(b2 - p2 )
(A) a2b2 sin2 C (B) a2b2 cos2 C = p2 (a2 - q2 ) + q2 (b2 - p2 ) + 2 pq (a2 - q2 )(b2 - p2 )
(C) a2b2 tan2 C (D) a2b2 cosec2 C
= ( p a2 - q2 + q b2 - p2 )2 (4.12)
Solution: Let AM M and BN
N be perpendiculars drawn from
f
A and B, respectively, onto a line through the vertex C From Eqs. (4.11) and (4.12) it follows that
(Figure 4.43). Then
a2 p2 + b2 q2 - 2abpq cos C = a2 b2 sin2 C
p = AM , q = BN
Answer: (A)
Let ACM = a and BCN = b . Then
40. In an acute-angled DABC
D , the circle described on the
C = p - (a + b ) altitude AD as diameter cuts the sides AB and AC,
respectively, in P and Q. Then the length of the chord
A p PQ is equal to
c b a M D D D D
(A) (B) (C) (D)
C 2R 2r R r
B a b
Solution: See Figure 4.44. From DAPQ, we have
q
N
PQ
= AD
FIGURE 4.43 Single correct choice type question 39.
sin A
Worked-Out Problems 245

Therefore p
A1OA2 =
2n
PQ = ( AD)sin A = (b sin C )sin A
Let the radius of the circle circumscribing the polygon
æ c ö be r. Therefore
= bç sin A
è 2 R ÷ø
æ p ö
æ1 ö 1 ( A1 A2 )2 = r2 + r2 - 2r2 cos ç ÷
= ç bc sin A÷ è 2n ø
è2 øR
p
D = 4r2 sin2
= n
R
Similarly
A
2p
( A1 A3 )2 = 4r 2 sin2
n
3p
c b
and ( A1 A4 )2 = 4r 2 sin2
n
By hypothesis
1 1 1
P a Q = +
sin(p /n) sin(2p /n) sin(3p /n)

Therefore

2p 3p pæ 2p 3p ö
sin sin = sin ç sin + sin ÷
FIGURE 4.44 Single correct choice type question 40. n n nè n nø
Answer: (C) p 3p 2p 3p
2 cos sin = sin + si n
n n n n
41. A1, A2, …, An are the vertices of a regular polygon of
n sides such that 4p 2p 2p 3p
sin + sin = sin + sin
1 1 1 n n n n
= +
A1 A2 A1 A3 A1 A4 4p 3p
sin = sin
n n
Then, the value of n is
4p 3p
(A) 5 (B) 6 (C) 7 (D) 9 sin - sin =0
n n
Solution: See Figure 4.45. Let the centre of the polygon
be “O”. Therefore 7p p
2 cos sin =0
2n 2n
Now
p 7p
sin ¹ 0 Þ cos =0
2n 2n
A4
r 7p p
O Þ =
2n 2
r
r p Þn=7
2n A3
r Answer: (C)

A1 42. In DABC, angle A is twice angle B. Then a2 - b2 is


equal to
(A) 2D (B) bc (C) ca (D) ab
FIGURE 4.45 Single correct choice type question 41.
246 Chapter 4 Properties of Triangles

Solution: 44. A circle is inscribed in an equilateral triangle of side


“a” units. Then, the area of any square inscribed in
a2 - b2 = 4 R2 (sin2 A - sin2 B) the circle is
= 4 R2 sin( A + B)sin( A - B) a2 a2 a2 a2
(A) (B) (C) (D)
= 4 R2 sin C sin B (∵ A = 2 B) 4 6 8 12

= (2 R sin B)(2 R sin C ) A


= bc
Answer: (B)
F G E
43. In DABC, let

B-C A C-A B
x = tan tan , y = tan tan ,
2 2 2 2
A-B C FIGURE 4.46 Single correct choice type question 44.
z = tan tan
2 2
Solution: Let DABC C be equilateral with side lengths
Then x + y + z = K(xyz), where the value of K is “a”. Let AD, BE, CFF be the medians and hence alti-
(A) 2 (B) -2 (C) 1 (D) -1 tudes and G the centroid (orthocentre) (Figure 4.46).
Therefore
Solution: By Napier’s rule
a 3 æ AD ö
B - C æ b - cö A AD = çè∵ sin 60° = ÷
tan =ç cot 2 a ø
2 è b + c ÷
ø 2

AD a
b-c c-a a-b r = Radius of the circle = GD = =
x= , y= , z= 3 2 3
b+c a+c a+b
If x is the side of the square, then
Now
a
b-c x + 1 -b x 2 = 2r =
x= Þ = 3
b+c x-1 c
Therefore
Similarly
a2
y + 1 -c z + 1 -a Area of the square = x2 =
= and = 6
y-1 a z-1 b
Answer: (B)
Therefore
45. In any DABC, if cos A + cos B + cos C = 3/2, then the
æ x + 1ö æ y + 1ö æ z + 1ö triangle is
çè x - 1 ÷ø çè y - 1÷ø çè z - 1 ÷ø = - 1
(A) right angled
( x + 1)( y + 1)(z + 1) = -( x - 1)( y - 1)(z - 1) (B) right-angled isosceles
(C) isosceles but not equilateral
On expanding the products both sides and simplifying
we have (D) equilateral

2 xyz = - 2( x + y + z) Solution:

x + y + z = - xyz A+ B A-B C 3
2 cos cos + 1 - 2 sin2 =
2 2 2 2
Therefore K = -1.
C ( A - B) 1 C
Answer: (D) 2 sin cos = + 2 sin2
2 2 2 2
Worked-Out Problems 247

A-B 1 C and equality holds if c = a. Hence


cos = + sin
2 4 sin(C/ 2) 2
a2 + b2 + c2 ³ 4 3D
2
æ C 1 ö and equality holds when a = b = c. This means that the
= ç sin - + 1³1
è 2 2 sin(C / 2) ÷ø minimum value of a2 + b2 + c2 is 4 3D and this minimum
is attained when DABC C is equilateral. Therefore among
But all triangles, equilateral triangles are those such that the
minimum value of a2 + b2 + c2 is 4 3D.
A-B
cos £1 Answer: (C)
2
Therefore 47. In an equilateral triangle, 3 coins of radii 1 unit each
are kept so that they touch each other and also touch
A-B C 1
cos = 1 and sin = the sides of the triangle. Area of the triangle is
2 2 2
(A) 4 + 2 3 (B) 6 + 4 3
This implies A = B = C = 60°.
7 3 7 3
Answer: (D) (C) 12 + (D) 3 +
4 4
46. In DABC
D , the minimum value of the sum of the squares
A
of its sides is
(A) 3 3D (B) 2 3D
(C) 4 3D (D) 3D P

where D is the area of DABC.


Q R
Solution: By cosine law

a2 = b2 + c2 - 2bc cos A
a2 + b2 + c2 = 2(b2 + c2 ) - 2bc cos A FIGURE 4.47 Single correct choice type question 47.

We know that Solution: Let Q, R be the centres of the circles touching


the side BC
C at M and N, respectively (Figure 4.47). In
1 DQBM, QBM = 30°. Therefore
D= bc sin A
2
Therefore BM = (QM )cot 30° = 3

4 D 3 = (2bc) 3 sin A MN = QR = 2 and NC = 3


Therefore the side BC
C is equal to
Hence
BC = BM + MN + NC = 2 + 2 3
a2 + b2 + c2 - 4 3D
Hence
= 2(b2 + c2 ) - 2bc(cos A + 3 sin A)
æ1 ö 3
3 Area of D ABC = ( BC )2
= 2(b2 + c2 ) - 4bc ç cos A + sin A÷ 4
è2 2 ø
3
æp ö = ( 2 + 2 3)2
= 2(b + c ) - 4bc cos ç - A÷ ³ 2(b2 + c2 ) - 4bc
2 2
4
è3 ø
3
= 2(b - c)2 ³ 0 = ( 4 + 12 + 8 3)
4
Equality holds if b = c. Similarly =6+4 3

a2 + b2 + c2 - 4 3D ³ 2(c - a)2 Answer: (B)

48. In DABC, let C = p/ 2. If r is the inradius and R is the


circumradius of the triangle, then 2(r + R) is
248 Chapter 4 Properties of Triangles

(A) a + b - c (B) b + c (C) c + a (D) a + b Hence


Solution: See Figure 4.48. We have C = 90°, so p
C=
c c 2
= 2R Þ R =
sin C 2 So A : B : C = 1: 2 : 3.
Also Answer: (C)

C 50. Let A0A1A2A3A4A5 be a regular polygon of six sides


r = ( s - c)tan =s-c
2 inscribed in a circle of unit radius. Then the product
of the line segments A0A1, A0A2 and A0A4 is
Therefore
3 3 3
2(r + R) = 2r + 2 R (A) (B) 3 (C) 3 3 (D)
4 2
= 2( s - c) + c
A4
= 2s - c
= (a + b + c ) - c
=a+b A3
1
B
1

a c O 1
A2

1 30° 30°
90°
1

FIGURE 4.48 Single correct choice type question 48.


Answer: (D)
A0 A1
49. The sides of a triangle are in the ratio 1: 3 : 2. Then
the angles opposite to these sides are in the ratio FIGURE 4.49 Single correct choice type question 50.
(A) 1: 3 : 5 (B) 2 : 3 : 4 (C) 1: 2 : 3 (D) 3 :1: 2
Solution: See Figure 4.49. Let O be the centre of the
Solution: In DABC, let BC = a, CA = b, AB = c. hexagon. Therefore
By hypothesis,
A0OA1 = 60°, A0OA2 = 120°, A0OA4 = 120°
a : b : c = 1: 3 : 2
By cosine formula
Let a = K, b = 3K , c = 2K. Therefore
( A0 A1 )2 = 12 + 12 - 2(1)(1)cos 60°
b +c -a
2 2
3+4-1
2
3
cos A = = = = 2(1 - cos 60°)
2bc 2 3 (2) 2
= 4 sin2 30° = 1
p
Þ A=
6 ( A0 A2 )2 = 1 + 1 - 2 cos 120°
Also = 2(1 - cos 120°)
c2 + a2 - b2 4 + 1 - 3 1 æ 1ö
cos B = = = = 2ç1 + ÷ =3
2ca 2(2) 2 è 2ø
p ( A0 A4 )2 = 1 + 1 - 2 cos 120°
ÞB=
3
=3
Worked-Out Problems 249

Therefore a2 + b2 + c2 = 2(a2 + b2 )

( A0 A1 )( A0 A2 )( A0 A4 ) = 1× 3 × 3 = 3 a2 + b2 = c2
Answer: (B) C = 90°
Answer: (D)
51. If the angles of a triangle are in the ratio 4 :1:1, then
the ratio of the largest side to the perimeter of the 53. In DABC, if the median AD is the Geometrical
triangle is Mean between the sides AB and AC, then 1 + cos A
(A) 3 :(2 + 3 ) (B) 1: 6 is equal to
1
(C) 1 : 2 + 3 (D) 2 : 3 (A) cos B - cos C (B) 2 cos B - cos C
2
Solution: Let the angles be A = 4q,
q B = q,
q C = q.
q 1
(C) cos B - cos C (D) 2 cos B - cos C
Therefore 2
6q = 180° Þ q = 30° Solution: By hypothesis,
Hence the angles are 120°, 30°, 30°. Let A = 120°, B = 30°, ( AD)2 = AB × AC
C = 30°. Therefore side BC
C is the largest side. Hence
( AD)2 = bc (4.13)
a b c
= = = 2R But we know that
sin 120° sin 30° sin 30°
1
which gives ( AD)2 = (2b2 + 2c2 - a2 ) (see Theorem 4.25) (4.14)
4
a = (2 R)sin 120° = R 3
Therefore from Eqs. (4.13) and (4.14) we get

and b=c=R 2b2 + 2c2 - a2 = 4bc

Now a2 = 2(b - c)2

2 s = Perimeter = a + b + c = R( 3 + 2) a= 2 b-c

So sin A = 2 sin B - sin C

a : 2 s = 3R : R(2 + 3 ) = 3 :(2 + 3 ) A A B+C B-C


2 sin cos = 2 2 cos sin
2 2 2 2
Answer: (A)
A B-C
cos = 2 sin
52. In DABC, if 2 2
cos A cos B cos C a b A A B-C
+ + = + 2 cos2 = 2 2 cos sin
a b c bc ca 2 2 2
then angle C in degrees is B+C B-C
= 2 2 sin sin
(A) 60° (B) 45° (C) 30° (D) 90° 2 2
Solution: = 2 cos B - cos C
cos A cos B cos C bc cos A + ca cos B + ab cos C Therefore
+ + =
a b c abc
1 + cos A = 2 cos B - cos C
a2 + b2
= (given)
abc Answer: (B)

Therefore 54. Let ABC be a triangle and p = sin A sin B sin C, q =


cos A cos B cos C. Then tan A, tan B and tan C are the
æb +c -a ö æc +a -b ö æa +b -c ö
2 2 2 2 2 2 2 2 2

÷ø + çè ÷ø + çè ÷ø = a + b
2 2
çè roots of the equation
2 2 2
250 Chapter 4 Properties of Triangles

(A) qx3 - px2 + (1 + q)x - p = 0 Solution: See Figure 4.50. Let E be the mid-point of
(B) qx + px - (1 + q)x + p = 0
3 2 DCC so that the distance AE
E is d. Let R be the circumra-
dius of the DABC. Therefore b = AC = 2R. From DABC,
(C) qx3 - px2 + (1 + q)x + p = 0
(D) qx3 - px2 - (1 + q)x - p = 0 cos b =
AB AB
=
AC 2 R
Solution:
p Therefore
tan A + tan B + tan C = tan A tan B tan C =
q AB = (2 R)cos b (4.15)
Now From DABD,
- cos( A + B + C ) + cos A cos B cos C AB 2 R cos b
å tan B tan C = cos A cos B cos C
cos a =
AD
=
AD
1+ q Hence
=
q
2 R cos b
AD = (4.16)
Therefore tan A, tan B, tan C are roots of the equation cos a
px2 (1 + q) p Also
x3 - + x- =0
q q q
BD
sina =
qx3 - px2 + (1 + q) x - p = 0 AD
Answer: (A) and hence

55. In DABC, B = 90°. A line passing through the vertex 2 R cos b sin a
BD = (4.17)
A meets the side BC C at a point D inside the circum- cos a
circle of the triangle ABC. If DAB = a , CAB = b
Now,
and the distance between A and the mid-point of
DCC is d, then the area of the circumcircle is 1
DE = DC
pd cos a 2 2
2
(A)
cos a + cos b + 2 cos a cos b cos(b - a )
2 2
1
= ( BC - BD)
2
2pd2 cos2 a
(B)
cos a + cos b + 2 cos a cos b cos(b - a )
2 2 1æ 2 R cos b sin a ö
= 2 R sin b -
2 çè cos a ÷ø
pd2 cos2 a
(C)
cos2 a + cos2 b + 2 cos a cos b R sin(b - a )
=
2pd2 cos2 a cosa
(D)
cos a + cos2 b + 2 cos a cos b
2
In DACD, AE
E being the median,

B 4( AE)2 = 2( AC )2 + 2( AD)2 - (DC )2

90°
= 2( AC )2 + 2( AD)2 - 4(DE)2
D
( AC )2 + ( AD)2 = 2( AE)2 + 2(DE)2
a E
b d
Using Eqs. (4.15)–(4.18) we have

4 R2 cos2 b 2 R2 sin2 (b - a )
4 R2 + = 2d2 +
cos a
2
cos2 a
é 2 cos2 b sin2 (b - a ) ù
R2 ê + - ú=d
2
2
ë cos2
a cos 2
a û
R2 [2 cos2 b + 2 cos2 a - sin2 (b - a )] = d2 cos2 a
FIGURE 4.50
Worked-Out Problems 251

R2 [2 cos2 b + 2 cos2 a - sin2 b cos2 a - sin2 a cos2 b 57. In DABC, if a + b = (2011)c , then
2 2 2

+ 2 sin a sin b cos a cos b ] = d2 cos2 a cot C


=
cot A + cot B
R2 [cos2 b + cos2 a + cos2 b (1 - sin2 a ) + cos2 a (1 - sin2 b )
(A) 2010 (B) 1006 (C) 1005 (D) 1005.5
+ 2 sin a sin b cos a cos b ] = d2 cos2 a
Solution:
R2 [cos2 b + cos2 a + 2 cos2 a cos2 b
cot C cos C sin A sin B
+ 2 sin a sin b cos a cos b ] = d2 cos2 a =
cot A + cot B sin2 C
R2 [cos2 a + cos2 b + 2 cos a cos b cos(b - a )] = d2 cos2 a [(a2 + b2 - c2 )/ 2ab](ab / 4 R2 )
=
Therefore area of the circle is c2 / 4 R2

pd2 cos2 a a2 + b2 - c2
pR2 = =
cos a + cos b + 2 cos a cos b cos(b - a )
2 2 2 c2

Answer: (A) (2011)c2 - c2


=
2 c2
56. DABC, if the angles A, B, C satisfy A : B : C = 1: 2 : 4, (2010)c2
then (a2 - b2)(b2 - c2)(c2 - a2) is equal to = = 1005
2 c2
(A) a2 + b2 + c2 (B) -a2b2c2 Answer: (C)
1
(C) (a2 + b2 + c2 ) 2 2 2
(D) 2a b c
2 58. In DABC, if the perimeter is 6 times the Arithmetic
Mean (AM) of sin A, sin B and sin C and the side BC
Solution: is of length 1 unit, then the angle A (in degrees) is
p (A) 60° (B) 45° (C) 75° (D) 30°
A + 2A + 4A = p Þ A =
7
Solution: Let a = BC, b = CA and c = AB. By hypothesis,
Therefore
6
a+b+c= (sin A + sin B + sin C )
2p 4p 3
B= ,C =
7 7
Therefore
Now,
2 R(sin A + sin B + sin C ) = 2(sin A + sin B + sin C )
(a2 - b2 )(b2 - c2 )(c2 - a2 ) ÞR=1
= 64R (sin A - sin B)(sin B - sin C )(sin C - sin A)
6 2 2 2 2 2 2
Now,
= 64 R [sin( A + B)sin( A - B)][sin(B + C )sin(B - C )]
6
1 = BC = a = 2 R sin A = 2 sin A
[sin(C + A) sin(C - A)] Therefore
é 3p æ -p ö ù é 6p æ - 2p ö ù 1
= 64 R6 êsin sin ç sin sin ç
ë 7 è 7 ÷ø úû êë 7 è 7 ÷ø úû sin A =
2
Þ A = 30°

é 5p æ - 3p ö ù Answer: (D)
êsin 7 sin çè 7 ÷ø ú
ë û
59. If x > 1, then the largest angle of the triangle whose
p 2p 4p sides are of lengths x2 + x + 1, 2x + 1 and x2 - 1 is
= - 64 R sin sin2
6 2
sin2
7 7 7 (A) 150° (B) 120° (C) 110° (D) 135°
æ pöæ 2p ö æ 4p ö
= - ç 4 R2 sin2 ÷ ç 4 R2 sin2 ÷ ç 4 R2 sin2 ÷ Solution: Let a = x2 + x + 1, b = 2x + 1 and c = x2 - 1.
è 7ø è 7 øè 7 ø Clearly, a is greater than b and c because x > 1 and also
= - a2 b2 c2 sum of any two of a, b, c is greater than the third. Hence
they form a triangle and “a” is the greatest side. If a is
Answer: (B)
the angle opposite to the side a, then
252 Chapter 4 Properties of Triangles

b2 + c2 - a2 Therefore
cos a =
2bc 2 D 420
AD = =
(2 x + 1)2 + ( x2 - 1)2 - ( x2 + x + 1)2 BC 17
=
2(2 x + 1)( x2 - 1) 2 D 420
BE = =
CA 25
-(2 x3 + x2 - 2 x - 1)
= 2D 420
2(2 x3 + x2 - 2 x - 1) CF = =
AB 28
-1
= So AD = 420/17 is the greatest altitude.
2
Note: The reader can see that, in a triangle with unequal
Hence a = 120°. sides, the largest altitude is the altitude drawn from the
Answer: (B) vertex onto the shortest side.
Answer: (A)
60. In DABC, if
62. In DABC, the sides a = BC, c = AB and the angle A
C A 3b
a cos 2
+ c cos2 = are given. If b1 and b2 are two values of the side
2 2 2 b = AC
C such that b2 = 2b1, then the value of sin A is
then 9a2 - c2 9c2 - a2
(A) (B)
(A) b + c = 2a (B) a + b = 2c 2 2c 2 2a
(C) ac = b 2
(D) a + c = 2b
9a - c
2 2
9a2 + c2
Solution: It is given that (C) (D)
2 2a 2 2c
C A 3b Solution: From cosine rule,
a cos2 + c cos2 =
2 2 2
b2 - 2bc cos A + (c2 - a2 ) = 0
Therefore
for which b1 and b2 are roots. Therefore
a(1 + cos C ) + c(1 + cos A) = 3b b1 + b2 = 2c cos A Þ 3b1 = 2c cos A (4.19)
a + c + (a cos C + c cos A) = 3b
b1b2 = c2 - a2 Þ 2b12 = c2 - a2 (4.20)
(a + c) + b = 3b
(using b2 = 2b1). Therefore using Eqs. (4.19) and (4.20)
a + c = 2b we get
Answer: (D) 2
æ 2c cos A ö
÷ø = c - a
2 2

61. In DABC, the sides are of lengths 17, 25, 28 units. è 3
Then, the length of the greatest altitude is 8c2 cos2 A = 9(c2 - a2 )
420 8c2 (1 - sin2 A) = 9(c2 - a2 )
(A) (B) 26 (C) 26.4 (D) 27.5
17
9a2 - c2
Solution: Let the triangle be ABCC where BC = 17, sin2 A =
8 c2
CA = 25 and AB = 28. Suppose AD, BE
E and CF
F are the
altitudes. Then 9a2 - c2
sin A =
2 2c
D2 = s( s - a)( s - b)( s - c)
Answer: (A)
= 35 ´ 18 ´ 10 ´ 7
63. BC, CA and AB of DABC C subtend angles
= 52 ´ 62 ´ 72 a, b and g at the centre of its circumcircle. Keeping
= (210)2 the circle fixed and changing the triangle ABC, then
the minimum value of
So D = 210. Now
1é æ pö æ pö æ pöù
cosç a + ÷ + cosç b + ÷ + cosç g + ÷ ú
1 1 1
D = AD× BC = BE ×CA = CF × AB 3 êë è 2ø è 2ø è 2øû
2 2 2
is
Worked-Out Problems 253

-1 1 - 3 -3 A
(A) (B) (C) (D)
2 2 2 2 2
Solution: See Figure 4.51. Note that a + b + g = 2p.
p Let
a
1é æ pö æ pö æ pöù
Z = êcos ç a + ÷ + cos ç b + ÷ + cos ç g + ÷ ú
3ë è 2ø è 2ø è 2øû
1
= - (sin a + sin b + sin g ) × × ×
3
1æ a b gö FIGURE 4.52 Single correct choice type question 64.
= - ç 4 sin sin sin ÷
3è 2 2 2ø
where a is the length of the side BC. It is clear that DABE
1
= - (4 sin A sin B sin C)) (∵ a = 2 A, b = 2 B, g = 2C ) and DACE E are similar triangles. Therefore
3
h2 = BE × EC = ( BP + PE)(QC - QE)
In DABC, the maximum value of sin A sin B sin C is
3 3 / 8 which will occur when A = B = C = 60°. Therefore = BP × QC - BP × QE + PE ×QC - PE ×QE
= BP × QC + BP( PE - QE) - PE × QE (∵ BP = QC )
1æ 3 3ö 3
Z³- ç =-
3 è 2 ÷ø 2 So

æ n - 1ö a æ n - 1ö a
Therefore the minimum value of Z is - 3 / 2. h2 + PE ×QE = ç + BP × PQ
è 2 ÷ø n çè 2 ÷ø n
A (n - 1)2 a2 æ n - 1ö a a
= +ç ×
4n2 è 2 ÷ø n n
(n2 - 1)a2
= (4.21)
g b 4 n2
O
a Let PAE = q1 and QAE = q2 so that q1 - q2 = a .
B C Therefore

tan a = tan(q1 - q2 )
FIGURE 4.51 Single correct choice type question 63.
tan q1 - tan q2
Answer: (C) =
1 + tan q1 tan q2
64. In DABC, A = 90° and h is length of the altitude ( PE / h) - (QE / h)
drawn from the vertex A onto the opposite side BC. =
1 + ( PE// h) × (QE / h)
The side BCC is divided into n equal parts (n is odd)
such that the segment containing the mid-point D h PQ
=
C subtends angle a at A. Then tan a is equal to
of BC h2 + PE × QE
4 nh 4 nh h(a / n)
(A) (B) = [from Eq. (4.21)]]
a(n + 1)
2
a(n2 - 1) [(n2 - 1)a2 / 4n2 ]
2 nh 2 nh
(C) (D) (4 nh)
a(n + 1)
2
a(n2 - 1) =
(n2 - 1)a
Solution: See Figure 4.52. Let h = AE
E and PQ be the Answer: (B)
segment containing the mid-point D of the side BC
C and
PAQ = a. Since each of BP P and QC
C is divided into 65. A is the obtuse angle of DABC. If the median and
(n - 1)/2 equal parts, we have the altitude from the vertex A divide the angle A
into three equal parts, then
æ n - 1ö a
BP = QC = ç
è 2 ÷ø n æ 3 Aö æ Aö
çè sin ÷ø çè cos ÷ø =
3 3
254 Chapter 4 Properties of Triangles

3a3 3a3 C
(A) (B)
16b2 c 64b2 c
9a3 3a3 z
(C) (D)
128b2 c 128b2 c
D
A 2y

z
y
A A
A
3 3
3
90° 90°

FIGURE 4.54 Single correct choice type question 66.


90°

Let BD = DC = z. Then
FIGURE 4.53 Single correct choice type question 65.
tan A = tan(p - CAM )
Solution: See Figure 4.53. In DADC,
= - tan CAM
A
ADE = 90° - = ACD -2y
3 =
x
Therefore = - 2 tan B
AD = AC = b Therefore
a /2 a tan A + 2 tan B = 0
DE = EC = =
2 4 Answer: (C)
Now
67. In DABC
D , if the median through A is perpendicular to
A A 1 A 2A the side AB, then the value of cos A cos B is equal to
sin cos = sin2 sin
3

3 3 2 3 3 2(b2 - a2 ) 2(c2 - a2 )
2 (A) (B)
1 æ EC ö æ BE ö 3ab 3ac
= ç ÷ ç ÷
2 è AC ø è AB ø 2(a2 - b2 ) 2(c2 + a2 )
(C) (D)
1 æ a2 ö æ 3a ö 3ab 3ab
= ç ÷
2 çè 16b2 ÷ø è 4c ø
C
3a3
=
128b2 c
Answer: (D)
a
D
66. In DABC, if the median through A is perpendicular
to the side AB, then tan A + 2 tan B is equal to 2y
b
(A) tan C (B) -tan C
(C) 0 (D) 2 tan C y

Solution: Draw CM M perpendicular to BA produced.


Since AD is parallel to MC
C and D is the mid-point of BC, 90° 90°
it follows that MA = AB = x(say). Now suppose AD = y
so that MC = 2y
2 . See Figure 4.54.
FIGURE 4.55 Single correct choice type question 67.
Worked-Out Problems 255

Solution: See Figure 4.55. Draw CM M perpendicular to By hypothesis,


BA produced. Since AD is parallel to MC C and D is the
mid-point of BC, it follows that MA = AB = c (say). Now abc
= AD = c sin B
suppose AD = y so that MC = 2y 2 . b - c2
2

From DABD,
Therefore
AB c 2c
cos B = = = sin A sin B
= sin B
BD a / 2 a sin2 B - sin2 C
Also from DACM, sin( B + C )
=1
AM c sin(B + C )sin( B - C )
cos(p - A) = = (∵ MA = AB = c)
AC b B - C = 90° orr B = 90° + C = 113°
Answer: (B)

2 c2 69. In DABC, if
cos A co s B = - (4.22)
ab
A+ B
a tan A + b tan B = (a + b)tan
Now 2
a2 then the triangle is
( AD)2 + ( AB)2 = ( BD)2 Þ y2 + c2 =
4 (A) isosceles
and (B) isosceles and right angled
(C) right angled
(MC )2 + (MA)2 = ( AC )2 Þ 4 y2 + c2 = b2
(D) equilateral
Therefore Solution: From the hypothesis
a - 4c = 4 y = b - c
2 2 2 2 2
æ A + Bö æ A+ B ö
a ç tan A - tan ÷ = b çè tan - tan B÷
a -b
2 2 è 2 ø 2 ø
or c2 = (4.23)
3 Therefore
From Eqs. (4.22) and (4.23), we get
é æ A + Bö æ A + Bö ù
a êsin A cos ç ÷ø - cos A sin çè ÷
2(b - a )
2 2
ë è 2 2 ø úû
cos A cos B =
3ab æ A + Bö
cos A cos ç
Answer: (A) è 2 ÷ø

é æ A + Bö æ A + Bö ù
68. DABC, AD is the altitude from A onto the side b êsin ç
è ÷ø cos B - cos çè ÷ø sin B ú
= ë û
2 2
BC. If b > c, C = 23° and AD = abc/(b2 - c2), then B
is equal to æ A + Bö
cos ç cos B
è 2 ÷ø
(A) 83° (B) 113° (C) 123° (D) 75°
æ A - Bö æ A -B ö
a sin ç b sin ç
è 2 ÷ø è 2 ÷ø
Solution: Draw AD perpendicular to CB produced
(Figure 4.56). =
cos A cos B
A æ A - Bö
sin ç (a cos B - b cos A) = 0
è 2 ÷ø

æ A - Bö
b sin ç (sin A cos B - cos A sin B) = 0
c è 2 ÷ø
90°
æ A - Bö æ A - Bö
23° 2 sin2 ç =0
è 2 ÷ø
cos ç
è 2 ÷ø
FIGURE 4.56
256 Chapter 4 Properties of Triangles

Now Now it is given that


A-B 9 = tan A tan B + tan C (tan A + tan B)
cos ¹0
2
(tan A + tan B)2
( A - B) = tan A tan B +
tan A tan B - 1
[By Eq. (4.24)]
Þ sin =0
2
So
Þ A=B
DABC
C is isosceles. 9(tan A tan B - 1) = tan A tan B(tan A tan B - 1)
Answer: (A) + (tan A + tan B)2

70. The sides of a triangle are three consecutive natural


(tan A + tan B)2 - 10 tan A tan B + tan2 A tan2 B + 9 = 0
numbers and its largest angle is twice its smallest (tan A - tan B)2 + (tan A tan B - 3)2 = 0
one. Then, the perimeter of the triangle is
(A) 18 (B) 21 (C) 15 (D) 24 This gives

Solution: Let a, b, c be the sides and a = n, b = n + 1, tan A = tan B and tan A tan B = 3
c = n + 2 and C = 2 A. Now sin C = sin 2A
2 = 2 sin A cos A, So
therefore
tan2 A = 3
b2 + c2 - a2
c = (2a)
2bc and so
bc2 = a(b2 + c2 - a2 ) tan B = tan A = 3
(n + 1)(n + 2) = n[(n + 1) + (n + 2) - n ]
2 2 2 2
Therefore A = B = 60° and hence C = 60°. This implies
= n[n + 6 n + 5]
2 DABC C is equilateral.
In an equilateral triangle, orthocentre and circum-
= n(n + 1)(n + 5) centre are same. Therefore, if O is the circumcentre, then
(n + 2)2 = n(n + 5) OA + OB + OC = 3R
Therefore Answer: (D)

n2 + 4 n + 4 = n2 + 5n 72. Triangle ABC


C is equilateral with side length a and
P is a point on the side BA produced such that A lies
which gives n = 4 and the perimeter = 4 + 5 + 6 = 15. between P and B. Let r1 be the radius of the circle
Answer: (C) inscribed in DPAC and r2 be the radius of the escribed
circle of DPBC opposite to the vertex P. Then the
71. In DABC, if value of r1 + r2 is
tan A tan B + tan B tan C + tan A tan C = 9 3a 3a
(A) (B)
then the sum of the distances of the vertices from the 2 2
orthocentre is 3 3a
(C) (D) 2 3a
(A) 3 3R (B) 2 3R (C) 3R (D) 3R 2
Solution: From the hypothesis, we can see that at least Solution: See Figure 4.57. Let APC = a and ACP = b.
one of tan A tan B, tan B tan C and tan C tan A is not equal Therefore
to 1. Suppose tan A tan B ¹ 1. Also we know that AC BC
=
tan A + tan B + tan C = tan A tan B tan C sin a sin a
Therefore which implies that the circumradius is same for DPAC
and DPBC.
tan A + tan B
tan C = (4.24)
tan A tan B - 1
Worked-Out Problems 257

and a is a positive integer. Therefore (25/4) sin A must be


T2¢
an integer. Hence
C
A
T1¢ b r2 O2

O1 r1 r2
r1 O R
a 60° 60° C
R 2A

FIGURE 4.57 Single correct choice type question 72.

Let R¢ be the circumradius of DPAC


C and DPBC. Therefore
FIGURE 4.58 Single correct choice type question 73.
a b a b 3
r1 = 4 R¢sin sin sin 60° = 4 R ¢ sin sin × (4.25) 4 8 12 16 20 24
2 2 2 2 2 sin A = , , , , or
25 25 25 25 25 25
a æ 60° + b ö 3
r2 = 4 R ¢ sin cos ç (4.26)
2 è 2 ÷ø 2 Also, since a, b, c are integers cos A, cos B, cos C are
rational numbers where
From Eqs. (4.25) and (4.26),
b2 + c2 - a2
cos A =
aé b æ 3 b 1 böù 2bc
r1 + r2 = 2 R ¢ sin ê 3 sin + 3 ç cos - sin ÷ ú
2 êë 2 è 2 2 2 2 ø úû Therefore sin A, sin B and sin C belong to the two
element set
æ aö æ b 3 b 1 bö
= ç 2 R ¢ sin ÷ 3 ç sin + cos - sin ÷
è 2 ø è 2 2 2 2 2ø ì 20 24 ü
í , ý
î 25 25 þ
aæ1 b 3 bö
= 2 R ¢ 3 sin ç sin + cos ÷ Therefore the possible triangles whose sides are natural
2è2 2 2 2ø
numbers are the triples (5, 5, 5); (5, 5, 6); (5, 6, 6) and
a æ bö (6, 6, 6). Among these triangles, the triangle with sides 5,
= 2R ¢ 3 sin sin ç 60° + ÷
2 è 2ø 5 and 6 satisfies the condition 2R is equal to 6 41 .
Hence the number of such triangles is one.
a æ aö
= 2R ¢ 3 sin sin ç 60° + 30° - ÷ (∵ b = 60° - a ) Answer: (C)
2 è 2ø
a a 74. Two sides of a triangle are 2 and 3 and the included
= 2R ¢ 3 sin cos angle is 30°. Then, the inradius of the triangle is
2 2
3 -1 3+1
= R ¢ 3 sin a (A) (B)
4 2
æ a ö 3
= R¢ 3 ç = a 3 -1 3+1
è 2R ¢ ÷ø 2 (C) (D)
2 4
Answer: (A)
Solution: In DABC, let b = 2, c = 3 and A = 30°. The
73. The number of triangles with circumradius R is third side a is given by
equal to 25/8 and whose sides are natural numbers is
a2 = b2 + c2 - 2bc cos A
(A) 4 (B) 2
(C) 1 (D) infinite æ 3ö
= 4 + 3 - 2(2)( 3 ) ç ÷
è 2 ø
Solution: In DABC, a = BC, b = CA and c = AB are
natural numbers and R = 25/8 (Figure 4.58). Now =7-6=1
æ 25 ö Therefore a = 1 and so
a = 2 R sin A = ç ÷ sin A
è 4ø
1 3+ 3
s = (a + b + c ) =
2 2
258 Chapter 4 Properties of Triangles

Hence C C
= c tan cot
2 2
3+1
s-a= =c
2
Now Hence x ¹ p.
Answer: (B)
A æ 3 + 1ö
r = ( s - a)tan = ç tan 15°
2 è 2 ÷ø 76. In DABC,

1 1 1
3+1 3-1 3-1 + + =
= ´ = bc ca ab
2 3+1 2
1 2
Answer: (C) (A) (B)
Rr Rr
75. In DABC, let r1, r2 and r3 be the exradii opposite to 1 1
(C) (D)
the vertices A, B and C, respectively. Let 2Rr 2(R + r )
x = a(rr1 + r2 r3 ) Solution:
y = b(rr2 + r3r1 ) 1 1 1 a+b+c
+ + =
z = c(rr3 + r1r2 ) bc ca ab abc
2s
C =
and p = (r3 - r )cot abc
2
2(D / r )
Then =
abc
(A) x = y = z = p (B) x = y = z
æ 4D ö 1

(C) x = y = z = 2p
2 (D) y = z = 2p
2 è abc ÷ø 2r
Solution: 1
=
x = a(rr1 + r2 r3 ) 2 Rr
Answer: (C)
æ D2 D2 ö
= aç + ÷ø
è s( s - a ) ( s - b)( s - c ) 77. In DABC,
æ ( s - b)( s - c) + s( s - a) ö r1 r2 r3
= aD 2 ç ÷ø + + =
è D2 ( s - b)( s - c) ( s - c)( s - a) ( s - a)( s - b)
= a[2 s2 - s(a + b + c) + bc] (A)
4
(B)
1
(C)
2
(D)
3
r r r r
= abc
Solution:
Similarly y = abc, z = abc. Therefore x = y = z. Now
r1 D s
C = =
p = (r3 - r )cot ( s - b)( s - c) ( s - a)( s - b)( s - c) D
2
Similarly
æ 1 1ö C
= Dç - cot
è s - c s ÷ø 2 r2 s
= =
r3
( s - c)( s - a) D ( s - a)( s - b)
Dc C
= cot
s( s - a) 2
r1 3s 3
=c
( s - b)( s - c)
× cot
C å (s - b)(s - c) = =
D r
s( s - a) 2
Answer: (D)
Worked-Out Problems 259

78. In DABC, if 8R2 = a2 + b2 + c2, then sin2 A + sin2 B + Similarly


sin2 C =
ADF = FDB = b (suppose)
(A) 2 (B) 3 (C) 4 (D) 1
Solution: See Figure 4.59. Therefore

8 R2 = a2 + b2 + c2 2 EDF = 2(a + b ) = 180°


EDF = 90°
= 4 R2 (sin2 A + sin2 B + sin2 C )
Answer: (A)
Therefore
81. In DABC, if t1, t2 and t3 are the lengths of the tangents
sin2 A + sin2 B + sin2 C = 2
drawn from the excentres I1, I2 and I3 of DABC
C to the
Answer: (A) circumcircle of the triangle, then
1 1 1 1 1 1
79. In DABC, + + is equal to + + =
r1 r2 r3 t12 t22 t32
1 2 D D 1 2 2r 1
(A) (B) (C) (D) (A) (B) (C) (D)
r r r 2r Rr Rr R 2Rr
Solution: Solution: Let I1T1 be the tangent from the excentre I1 to
1 1 1 s-a s-b s-c the circumcircle and O be the circumcentre (Figure 4.60).
+ + = + + It is known that (see Quick Look 2 and Theorem 4.29)
r1 r2 r3 D D D
1 OI12 = R2 + 2 Rr1
= ( 3 s - 2 s)
D and OT12 = R2
s
=
D A

1
= O
r C
R T1
Answer: (A) B

t1
80. In DABC, A = 120°. Let D, E and F be the points of
intersection of the internal bisectors with the oppo-
site sides. Then DDEF F is
(A) right angled FIGURE 4.60 Single correct choice type question 81.
(B) isosceles
(C) isosceles and right angled From the right-angled DOT
T1I1
(D) equilateral
OI12 = R2 + t12
R2 + 2 Rr1 = R2 + t12

60° t12 = 2 Rr1


A
60° E
Similarly
F C
a t22 = 2 Rr2 and t32 = 2 Rr3
b
D
Therefore

1 1 1 1 æ 1 1 1ö
FIGURE 4.59 + 2 + 2 = + +
t1 t2 t3 2 R çè r1 r2 r3 ÷ø
2

Solution: For the DABD, AC C is the external bisector


1 æ 1ö
of BAD and BE E is the internal bisector of DBA. = ç ÷
Therefore E is the excentre to DABD. Hence 2R è r ø

CDE = ADE = a (suppose) Answer: (D)


260 Chapter 4 Properties of Triangles

82. In DABC, if (a - b)(s - c) = (b - c)(s - a), then


(A) 2r2 = r1 + r3 (B) r22 = r1r3 r a
=
2r1r2 2r3 r1 q b
(C) r3 = (D) r2 =
r1 + r2 r3 + r1 Therefore
Solution:
r2 (b2 + c2 ) = a2 ( p2 + q2 )
(a - b)( s - c) = (b - c)( s - a)
r 2 = p2 + q2
b-a c-b
Þ = Note: In general, the inradii of two similar triangles are
s-a s-c
proportional to their sides.
b-a c-b Answer: (C)
Þ =
( s - a)( s - b) ( s - b)( s - c)
84. In DABC, r1r2 + r2r3 + r3r1 is equal to
D D D D
Þ - = -
s-b s-a s-c s-b r2 R2
(A) s2 (B) D 2 (C) (D)
D2 D2
Þ r2 - r1 = r3 - r2
Solution:
Þ 2r2 = r1 + r3
Answer: (A) r1r2 + r2 r3 + r3 r1
é 1 1 1 ù
83. In DABC, A = 90° and AD is drawn perpendicular = D2 ê + + ú
to the side BC. If p and q are the radii of the inscribed ë ( s - a)( s - b) ( s - b)( s - c) ( s - c)( s - a) û
circles of DABD and DACD, respectively, and r is = s[( s - c) + ( s - a) + ( s - b)]
the inradius of DABC, then r is equal to
= s(3s - 2 s)
1 2
(A) 2 p2 + q2 (B) p + q2
2 = s2
1 2 Answer: (A)
(C) p2 + q2 (D) ( p + q2 )
2
85. Let I be the incentre and r the inradius of DABC.
A Let D, E, F be the feet of the perpendiculars drawn
b from I onto the sides BC, CA and AB, respectively. If
r1, r2 and r3 are the radii of the circles inscribed in the
q C quadrilaterals AFIE, BDIF F and CEID, respectively,
c
then
p
D r1 r r K
a + 2 + 3 =
r - r1 r - r2 r - r3 r(r - r1 )(r - r2 )(r - r3 )

FIGURE 4.61 Single correct choice type question 83.


where K is equal to
(A) s2 (B) D 2 (C) D 2s2 (D) D 2r2
Solution: See Figure 4.61. We have a2 = b2 + c2. By hypoth-
esis, DABD and DACD are both similar as well as similar Solution: Let J1 be the centre of the circle inscribed
to DABC. Let R and R¢ be the circumradii of DABC C and in the quadrilateral AFIE E touching the side IE E in M1
DABD, respectively. Therefore and the side AE E in N1. Clearly J1 lies on the segment
AII (Figure 4.62). Clearly J1M1ENN1 is a square, because
A B C J1M1 = J1N1 = r1. Also
r = 4 R sin sin sin
2 2 2
A
æ 1 öæ 1 öæ 1 ö M1 J1 I = N1 J1 A = 90° -
and p = 4 R ¢ ç sin B÷ ç sin BAD÷ ç sin BDA÷ 2
è 2 øè 2 øè 2 ø
Therefore from DIM1J1,
But the angles of DABC
C and DABD are equal in some
order. Therefore æ A ö IM1 IE - M1 E r - r1
tan ç 90° - ÷ = = =
è 2 ø J1 M1 J1 M1 r1
r 4R 2(a /sin A) a
= = =
p 4R ¢ 2(c /sin D) c
Worked-Out Problems 261

A Solution:
N1
æ 1 1ö æ 1 1ö æ 1 1ö
J1 E
çè r - r ÷ø çè r - r ÷ø çè r - r ÷ø
1 2 3
F M1
I C æ s s - a ö æ s s - bö æ s s - c ö
=ç - ÷ç - ÷ç - ÷
èD D øèD D øèD D ø
D

abc 4 RD 4 R
= = 3 = 2
D3 D D
Answer: (D)

87. In a triangle, if r1 = 12, r2 = 18 and r3 = 36, then r is


FIGURE 4.62
equal to
Therefore (A) 6 (B) 9 (C) 4 (D) 3

A r - r1 Solution:
cot =
2 r1 1 1 1 1
= + +
r r1 r2 r3
Similarly
B r - r2 C r - r3 1 1 1
cot = and cot = = + +
2 r2 2 r3 12 18 36
6 1
In any DABC = =
36 6
A B C A B C
cot + cot + cot = cot cot cot Therefore r = 6.
2 2 2 2 2 2
Answer: (A)
Therefore
r - r1 r - r2 r - r3 r1r2 r3 88. In DABC,
+ + =
r1 r2 r3 (r - r1 )(r - r2 )(r - r3 ) æ A + Bö
(r1 + r2 )cot ç =
è 2 ÷ø
rr1r2 r3
=
r(r - r1 )(r - r2 )(r - r3 ) (A) a (B) b (C) c (D) a + b + c
Solution:
D2
=
r(r - r1 )(r - r2 )(r - r3 ) æ A + Bö æ D D ö C
(r1 + r2 )cot ç = + tan
è 2 ÷ø çè s - a s - b ÷ø 2
Hence K is equal to D2.
Answer: (B) Dc æ Cö
= çè tan ÷ø
( s - a)( s - b) 2
86. If r is the inradius and r1, r2 and r3 are exradii of any
triangle, then s( s - c) C
=c × tan
æ 1 1ö æ 1 1ö æ 1 1ö ( s - a)( s - b) 2
çè r - r ÷ø çè r - r ÷ø çè r - r ÷ø =
1 2 3
æ Cö C
= c × ç cot ÷ tan
4R 4R 2
4R2
4R è 2ø 2
(A) (B) (C) (D)
D D D2 D2
=c
Answer: (C)
262 Chapter 4 Properties of Triangles

89. In DABC, if the median AD makes angle q with the A


E
side BC, then sin q is equal to
2b sin C b sin C
(A) (B)
2b + 2c - a
2 2 2
2b + 2c2 - a2
2
C

2b sin B b sin B F
(C) (D)
2b + 2c - a
2 2 2
2b + 2c2 - a2
2

Solution: From DADC


C (Figure 4.63),
AC AD B
=
sin q sin C

A
FIGURE 4.64

Therefore
1 1
EDF = ( B + C ) = 90° - A
q C 2 2
Similarly
D
1
DEF = 90° - B
2
FIGURE 4.63
1
and DFE = 90° - C
Therefore 2
AC sin C Using sine rule for DDEF
F we get
sin q =
AD EF FD
=
b sin C sin[90° - ( A / 2)] sin[90° - ( B / 2)]
= (see Theorem 4.25)
(1/ 2) 2b2 + 2c2 - a2 DE
= = 2R
sin[90° - (C/ 2)]
2b sin C
=
2b2 + 2c2 - a2 Therefore

Answer: (A) A B C
EF = 2 R cos , FD = 2 R cos , DE = 2 R cos
2 2 2
90. The internal bisectors of the angles of DABC
C meet
the circumcircle in D, E and FF. If D¢ and D are the So
areas of DDEF F and DABC, respectively, then the 1
ratio D¢ : D is equal to D= bc sin A
2
(A) 2R :r (B) R :r
= 2 R2 sin A sin B sin C
(C) (R - r) :(R + r) (D) R :2r
A B C A B C
Solution: Since angles in the same segment are equal = 16 R2 sin sin sin cos cos coss
2 2 2 2 2 2
(Figure 4.64), we have
1 æ Aö
1 D¢ = (DE)(DF )sin ç 90° - ÷
ADE = ABE = B 2 è 2ø
2
æ Cö æ Bö æ Aö
1 = 2 R2 sin ç 90° - ÷ sin ç 90° - ÷ sin ç 90° - ÷
and ADF = ACF = C è 2 ø è 2 ø è 2ø
2
A B C
= 2 R2 cos cos cos
2 2 2
Worked-Out Problems 263

Therefore Solution: Given that r1 = r2 + r3 + r. Therefore


D¢ 1 r1 - r = r2 + r3
=
D 8 sin( A / 2)sin(B / 2)sin(C/ 2) A B C A B C
4 R sin cos cos - 4 R sin sin sin
R 2 2 2 2 2 2
=
2[4 R sin( A / 2)sin(B / 2)sin(C / 2)] A B C A B C
= 4 R cos sin cos + 4 R cos cos sin
R 2 2 2 2 2 2
= A B+C A B+C
2r sin cos = cos sin
2 2 2 2
Answer: (D)
A A
sin2 = cos2
91. In DABC, if r1 = 8, r2 = 12, r3 = 24, and BC = a, then 2 2
the value of a is
This implies
(A) 12 (B) 6 (C) 4 (D) 8
A
Solution: In DABC, it is easy to verify that = 45°
2
(r1 - r )(r2 + r3 ) = a2 A = 90°
Answer: (C)
1 1 1 1
and + + =
r1 r2 r3 r 93. In any DABC, b(r1 - r) + a(r2 - r) is equal to
Therefore abc a+b+c cr3 c+r
(A) (B) (C) (D)
1 1 1 1 r3 r3 r r3
= + +
r r1 r2 r3 Solution:
1 1 1 æ 1 1ö æ 1 1ö
= + + b(r1 - r ) + c(r2 - r ) = bD ç - ÷ + aD ç - ÷
8 12 24
è s - a sø è s - b sø
1
= abD abD
4 = +
s( s - a) s( s - b)
Þr=4
abD æ 1 1 ö
= +
s è s - a s - b ÷ø
ç
a2 = (r1 - r )(r2 + r3 )
abD æ c ö
=
= (8 - 4)(12 + 24) s è ( s - a)( s - b) ÷ø
ç
= 4 ´ 36 (abc)D( s - c)
=
Þ a = 12 s( s - a)( s - b)( s - c)
Answer: (A) (abc)( s - c)
=
D
92. In DABC, if r1 = r2 + r3 + r, then angle A must be
abc æ D ö
(A) 120° (B) 60° (C) 90° (D) 45° = çè∵ r3 = s - c ÷ø
r3
Answer: (A)

Multiple Correct Choice Type Questions


1. In DABC, (C) (b + c)cos A + (c + a)cos B + (a + b)cos C =
1
æ A - B + Cö (a + b + c )
÷ø = c + a - b
2 2 2
(A) (2ac)sin ç 2
è 2
(D) If A = 60°, then
(B) a(b cos C - c cos B) = b2 - c2
æ a bö æ c aö
çè 1 + + ÷ø çè 1 + - ÷ø = 3
c c b b
264 Chapter 4 Properties of Triangles

Solution: Therefore
æ A - B + Cö a + c 2 = 2b
(A) 2ac sinç
è ÷ø = 2ac sin(90° - B)
2
(A) is true.
= 2ac cos B
(B) a, b, c are in HP implies
= c2 + a2 - b2
1 1 1
Therefore (A) is true. , , are in AP
a b c
(B) a(b cos C - c cos B) = ab cos C - ac cos B
Therefore
a2 + b2 - c2 a2 + c2 - b2
= - 1 1 1 1
2 2 - = -
b a c b
= b2 - c2
a-b b-c
Hence (B) is true. =
ab bc
(C) (b + c)cos A + (c + a)cos B + (a + b)cos C
sin A - sin B sin B - sin C
= (b cos A + a cos B) + (a cos C + c cos A) =
sin B sin A sin C sin B
+ (b cos C + c cos B)
A+ B A-B B+C B-C
= c + b + a = 2s 2 cos sin 2 cos sin
2 2 = 2 2
So (C) is not true. A A C C
2 sin cos 2 sin cos
æ a bö æ c aö 2 2 2 2
(D) ç 1 + + ÷ ç 1 + - ÷
è c c ø è b bø C A-B A B-C
sin sin sin sin
(c + a + b)(b + c - a) 2 2 = 2 2
= A A C C
bc sin cos sin cos
2 2 2 2
(b + c)2 - a2
= C C A-B A A B-C
bc sin2 cos sin = sin2 cos sin
2 2 2 2 2 2
(b2 + c2 + 2bc) - (b2 + c2 - 2bc cos 60°)
= Cé A+ B A- Bù Aé B+ C B- C ù
bc sin2 sin × sin = sin2 êsin sin
2 êë 2 2 úû 2ë 2 2 úû
3bc æ 1ö
= çè∵ cos 60° = ÷
bc 2ø Cé 2A Bù Aé B Cù
sin2 ê sin - sin2 ú = sin2 êsin2 - siin2 ú
2ë 2 2û 2ë 2 2û
=3
Bé 2C Aù C A
Therefore (D) is true. sin2 ê sin + sin2 ú = 2 sin2 sin2
2ë 2 2û 2 2
Answers: (A), (B), (D)
2 1 1
2. In any DABC
C which of the following are true? 2
= 2
+ 2
sin ( B / 2) sin ( A / 2) sin (C/ 2)
(A) If A = 45°, B = 75°, then a + c 2 = 2b.
(B) If the sides a, b and c are in HP, then sin2(A/2), Therefore sin2(A/2), sin2(B/2) and sin2(C/2)
C are in
sin2(B/2) and sin2(C/2)
C are in AP. HP. Hence (B) is not true.
(C) We have
é A Bù C
(C) (a + b + c) êtan + tan ú = 2c cot
ë 2 2û 2 é A Bù
(a + b + c) êtan + tan ú
A B C ë 2 2û
(D) bc cos2 + ca cos2 + ab cos2 = D2
2 2 2
é A Bù
Solution: ê sin 2 sin ú
2
= (a + b + c ) ê +
(A) A = 45°, B = 75° Þ C = 60°. Now A Bú
ê cos cos ú
ë 2 2û
a c
b = a cos C + c cos A = +
2 2
Worked-Out Problems 265

A+ B
(a + b + c)sin
= 2
A B b2 = c2 + a2 - 2ca cos B
cos cos
2 2 æ 1ö
= c2 + a2 - 2ca ç ÷
C è 2ø
2 R(sin A + sin B + sin C )cos
= 2 = c2 + a2 - ca
A B
cos cos
2 2 Hence (A) is true.
æ A B Cö C (B) We have
2 R ç 4 cos cos cos ÷ cos
è 2 2 2ø 2 sin( B - C ) sin( B + C )sin( B - C )
= =
A B sin( B + C ) sin2 ( B + C )
cos cos
2 2
sin2 B - sin2 C
C =
= 8 R cos 2
sin2 A
2
b2 - c2
æ c ö C =
= 4ç ÷ cos2 a2
è sin C ø 2
Hence (B) is true.
æ ö
ç c ÷ C (C) Consider
= 4ç cos2
C C÷ 2 b2 - c2 4 R2 (sin2 B - sin2 C )
çè 2 sin cos ÷ø =
2 2
cos B + cos C cos B + cos C
C
= 2c cot 4 R2 [cos2 C - cos2 B]
2 =
coos B + cos C
Therefore (C) is true.
= 4 R2 (cos C - cos B)
A B C
(D) bc cos 2
+ ca cos2 + ab cos2
2 2 2 Therefore
s( s - a) s( s - b) s( s - c) b2 - c2
= (bc) + (ca) + (ab) å cos B + cos C = 4R [cos C - cos B + cos A - cos C
2
bc ca ab
= s[3s - a - b - c] = s2 + cos B - cos A]

Hence (D) is not true. = 4 R2 (0) = 0


Answers: (A), (C) So (C) is true.
(D) We have
3. In DABC, which of the following are true?
(A) If the angles are in AP, then b2 = c2 + a2 - ca 1 + cos( A - B)cos C 1 - cos( A - B)cos( A + B)
=
1 + cos( A - C )cos B 1 - cos( A - C )cos( A + C )
sin(B - C ) b2 - c2
(B) = 1 - (cos2 A - sin2 B)
sin(B + C ) a2 =
1 - (cos2 A - sin2 C )
b2 - c2
(C) å cos B + cos C = 0 sin2 A + sin2 B
=
1 + cos( A - B)cos C a2 + b2 sin2 A + sin2 C
(D) =
1 + cos( A - C )cos B a2 + c2 (a2 / 4 R2 ) + (b2 / 4 R2 )
=
(a2 / 4 R2 ) + (c2 / 4 R2 )
Solution:
(A) A, B, C are in AP implies a2 + b2
=
a2 + c2
3B = 180° (∵ A + C = 2 B)
Hence (D) is true.
B = 60°
Answers: (A), (B), (C), (D)
266 Chapter 4 Properties of Triangles

4. In DABC, the altitudes AD, BE


E and CF
F meet the 1
= (2 R sin 2C )(2 R sin 2 B)sin 2 A
circumcircle in L, M and N, respectively. Then 2
(A) the angles of the triangle LMNN are supplements = 2 R2 (2 sin C cos C )(2 sin B cos B)(2 sin Accos A)
of twice the angles of DABC
(B) the angles of the DLMN N are complements of the = 8(2 R2 sin A sin B sin C )(cos A cos B cos C )
angles of DABC = 8 D cos A cos B cos C (∵ 2 R2 sin A sin B sin C = D)
(C) the area of DLMN N is 4D cos A cos B cos C
Hence (D) is true.
(D) the area of DLMN N is 8D cos A cos B cos C
Answers: (A), (D)
Solution: See Figure 4.65. We have
5. Quadrilateral ABCD is inscribed in a circle with AD
ALM = ABM (angles in the same segment)
as diameter. If AD = 4, AB = BC = 1, then
= 90° - A
(A) AC = 15 / 2
and ALN = ACN = 90° - A (B) CD = 7/2
(C) area of DACD = 7 15 / 8
MLN = (90° - A) + (90° - A) = 180° - 2 A (D) perimeter of the quadrilateral ABCD = 19/2

Similarly C

NML = 180° - 2 B 1
90° B
and MNL = 180° - 2C x 1

Therefore (A) is true. a


D

A
FIGURE 4.66 Multiple correct choice type question 5.

Solution: See Figure 4.66.


(A) Let ADC = a and AC = x. Using cosine formula for
N
DABC,
F
M x2 = 1 + 12 - 2(1)(1)cos(180° - a )
E
= 2 + 2 cos a
C
CD
B D =2+2
4
1
=2+ 16 - x2
FIGURE 4.65 2
Therefore
Using sine rule for DLMN
N we get
MN LN ML 16 - x2
= = = 2R ( x2 - 2)2 =
sin(180° - 2 A) sin(180° - 2 B) sin(180° - 2C ) 4
4 x4 - 15x2 = 0
Therefore
15
MN = 2 R sin 2 A, LN = 2 R sin 2 B, LM = 2R sin 2C x2 =
4
Hence 15
x=
1 2
Area of DLMN = (LM )(LN )sin(180° - 2 A)
2 Hence (A) is true.
(See Theorem 4.14)
Worked-Out Problems 267

(B) Now 1 1 1
= ( BC )( PD) + ( PE )(CA) + ( AB)( PF )
2 2 2
CD = 16 - x2
1
= (20)( PD + PE + PF )
15 2
= 16 -
4 E + PF )
= 10( PD + PE
49 7 Therefore
= =
2 2
3 20 ´ 20
Therefore (B) is true. PD + PE + PF = ´ = 10 3
4 10
(C) The area of DACD is given by Answers: (B), (C)
1 1 7 15 7 15
CD × AC = ´ ´ = 7. D ABC , C = 90°, BC = 3, and AC = 4. D is a point on
2 2 2 2 8 the side AB (in between A and B) such that ACD = 60°.
Hence(C) is true. Then
(D) The perimeter of the quadrilateral ABCD is 8 3 20
(A) CD = (B) AD =
4+ 3 4+ 3
7 19
1+ 1+ +4= 15 8
2 2 (C) BD = (D) CD =
3+ 4 3 4+ 3
Hence (D) is true.
Answers: (A), (B), (C), (D) B

6. Let ABC
C be an equilateral triangle with side length D
20 units. Let P be a point inside the triangle ABC. If
PD, PEE and PF F are drawn perpendiculars to the sides 3
5
BC, CA and AB, respectively, then 30°
(A) value of PD + PE + PF F depends on the position 60°
of P
(B) value of PD + PE + PF F is independent of the
FIGURE 4.68 Multiple correct choice type question 7.
position of P
(C) PD + PE + PF = 10 3 Solution: See Figure 4.68. By Pythagoras theorem,
PA + PB + PC AB = 5. From DACD, using sine rule we get
(D) PD + PE + PF =
10 AD CD
=
sin 60° sin A
A
(CD)sin 60°
AD =
sin A
F E
P 5 3
= (CD)
3 2
Similarly from DBCD,
FIGURE 4.67 Multiple correct choice type question 6. 5 1
BD = (CD)
4 2
Solution: See Figure 4.67. We have
Therefore
3
´ 202 = Area of D ABC 5 = AB
4
= (Area of D PBC ) + (Area of DPCA) = AD + BD

+ (Area of D PAB)
268 Chapter 4 Properties of Triangles

æ 1 1ö Solution: See Figure 4.69. By sine rule


= 5(CD) ç + ÷
è 2 3 8ø
8 12 b
= =
(4 + 3 ) sin 45° sin C sin B
= 5(CD)
8 3
Therefore
Hence
12 sin 45°
sin C =
8 3 8
CD =
4+ 3 12 1
= ´
Therefore (A) is true. Now 8 2

5 3 2 3
AD = (CD) =
3 2 4

5æ 8 3 ö 3 3
= =
3 çè 4 + 3 ÷ø 2 2
This implies that C is 60° or 120°. Therefore (A) is true.
20
= Now
4+ 3
C = 60° Þ B = 180° - (45° + 60°) = 75°
Hence (B) is true. Now
C = 120° Þ B = 180° - (45° + 120°) = 15°
5 1
BD = (CD)
4 2 Hence (B) is true.
Finally depending on the two values of angle B we get
5æ 8 3 ö 1
=
4 çè 4 + 3 ÷ø 2 8
b= ´ sin B
sin 45°
5 æ 24 ö 1
=
4 çè 3 + 4 3 ÷ø 2 æ 3 + 1ö
= 2 ´ 8ç or
æ 3 - 1ö
2 ´ 8ç
÷ ÷
è 2 2 ø è 2 2 ø
15
=
3+4 3 = 2 ( 3 + 1) or 2 ( 3 - 1)
Hence (C) is true. = 6+ 2 or 6- 2
Answers: (A), (B), (C)
Therefore (D) is true.
Answers: (A), (B), (D)
8. In D ABC , A = 45°, BC = 8 , AB = 12 . Then
(A) C = 60° or 120° 9. In DABC, if

(B) B = 75° or 15° C A 3b


a cos2 + c cos2 =
2 2 2
(C) AC = 6 + 2 or 6 - 2
then
(D) AC = 6 + 2 or 6 - 2
(A) a, b, c are in HP
C (B) a, b, c are in AP
A B C
(C) cot , cot , cot are in AP
2 2 2
A B C
b 8=a (D) cot , cot , cot are in HP
2 2 2
Solution: Given that
45°
A B C A 3b
a cos2 + c cos2 =
2 2 2
FIGURE 4.69 Multiple correct choice type question 8.
Worked-Out Problems 269

Therefore æ A - Bö C
cos ç ÷ = 2 sin
è 2 ø 2
a(1 + cos C ) c(1 + cos A) 3b
+ =
2 2 2 æ A + Bö æ A - Bö æ Cö æ Cö
2 sin ç ÷ cos ç ÷ = ç 2 sin ÷ ç 2 cos ÷
a + c + (a cos C + c cos A) = 3b è 2 ø è 2 ø è 2øè 2ø
a + c + b = 3b sin A + sin B = 2 sin C

a + c = 2b Therefore a + b = 2c. Hence (A) is true. Also a, c, b are in


AP. Since the sides a, c and b are in AP, from Problem 9
Therefore a, b, c are in AP. Hence (B) is true. Now C
above it follows that cot(A/2), cot(C/2), cot(B/2) are also
C
in AP. Hence tan(A/2), tan(C/2), tan(B/2) are in HP.
A C s( s - a) s( s - c)
cot + cot = + This implies that (D) is true.
2 2 ( s - b)( s - c) ( s - a)( s - b)
Answers: (A) and (D)
s [ s - a + s - c]
= 11. In DABC, which of the following are true?
( s - a)( s - b)( s - c)
(A) If a2, b2, c2 are in AP, then cot A, cot B, cot C are
s (2 s - 2b) in AP.
= (∵ a + c = 2b)
( s - a)( s - b)( s - c) (B) If sin2 A + sin2 B + sin2 C = 2, then the triangle is
right angled.
s( s - b)
=2 A C
( s - c)( s - a) (C) If a, b, c are in AP, then cot cot is equal to 3.
2 2
B (D) b2 sin 2C + c2 sin 2B = (bc)sin 2A
= 2 cot 2
2
Solution:
Therefore cot(A C are in AP. Therefore
( /2), cot(B/2), cot(C/2) T (A) Now a2, b2, c2 are in AP. Therefore
(C) is true.
Answers: (B) and (C) b2 - a2 = c2 - b2
sin2 B - sin2 A = sin2 C - sin2 B
10. In DABC, if
sin(B + A)sin( B - A) = sin(C + B) sin(C - B)
C
cos A + cos B = 4 sin2 sin C sin( B - A) = sin A sin(C - B)
2
sin C (sin B cos A - cos B sin A) = sin A(sin C cos B
then
(A) a + b = 2c - sin B cos C )
(B) a, b, c are in HP Dividing both sides with sin A sin B sin C we get
A B C
(C) tan , tan , tan are in AP cos A cos B cos B cos C
2 2 2 - = -
sin A sin B sin B sin C
A C B
(D) tan , tan , tan are in HP cot A - cot B = cot B - cot C
2 2 2
Solution: Given that Therefore cot A, cot B, cot C are in AP. Therefore
(A) is true.
C
cos A + cos B = 4 sin2 (B) Given that sin2 A + sin2 B + sin2 C = 2. Therefore
2
1 - cos 2 A 1 - cos 2 B 1 - cos 2C
Therefore + + =2
2 2 2
æ A + Bö æ A - Bö C cos 2 A + cos 2 B + cos 2C = - 1
2 cos ç cos ç = 4 sin2
è 2 ÷ø è 2 ÷ø 2 -1 - 4 cos A cos B cos C = - 1
C æ A - Bö 2C cos A cos B cos C = 0
2 sin cos ç
è ÷ø = 4 sin
2 2 2
Hence one of the angles must be a right angle and
so (B) is true.
270 Chapter 4 Properties of Triangles

(C) By hypothesis a + c = 2b. So and sin(a - b)


b =0
Hence
A C s( s - a) s( s - c)
cot cot = ´ 2p pö
2 2 ( s - b)( s - c) ( s - a)( s - b) æ
a=b and a+b= çè∵ 0 < a , b < ÷ø
3 2
s
=
s-b which implies
2s p
= a=b=
2 s - 2b 3
a+b+c So
=
a + b + c - 2b p
q=
3b 3
= =3
3b - 2b
Hence (B) is true.
Hence (C) is true. (C) and (D) are also true because the triangle is equilateral.
(D) We have Answers: (B), (C), (D)

b2 sin 2C + c2 sin 2 B = 2b2 sin C cos C + 2c2 sin B cos B 13. In DABC, the internal bisector AD of the angle A is
= 2b(c sin B)cos C + 2c(b sin C )cos B equal to
= 2bc(sin B cos C + cos B sin C ) æ 2bc ö A æ b + cö C
(A) ç cos (B) ç cos
= 2bc sin((B + C ) è b + c ÷ø 2 è 2bc ÷ø 2
= 2bc sin A b sin C b sin C
(C) (D)
Hence (D) is not true. sin[ B + ( A / 2)] sin[ A + ( B / 2)]
Answers: (A), (B), (C) Solution: See Figure 4.70.

12. Let 0 < a, b < p /2 satisfying the relation


1 A
Area of DABD = AB × AD × sin
2 2
8 cos a cos b cos(a + b)
b = -1
1 A
be two angles of a triangle. Let q be the third angle Area of DACD = AC × AD × sin
2 2
of the triangle. Then
7p A
(A) q =
12
p A
(B) q = A 2
3 2
(C) the circumradius R = a / 3 where “a” is the side C
opposite to the angle q q
(D) the value of cos a + cos b + cos q = 3/2 D
Solution: It is given that
8 cos a cos b cos(a + b)
b = -1
FIGURE 4.70 Multiple correct choice type question 13.
Therefore
Therefore
4[cos(a + b ) + cos(a - b )]cos(a + b ) = - 1
D = Area of DABC = Area of DABD + Area of DACD
4 cos2 (a + b ) + 4 cos(a - b )cos(a + b ) + 1 = 0
1 A
[2 cos(a + b ) + cos(a - b )]2 + 1 - cos2 (a - b ) = 0 = AD( AB + AC )sin
2 2
[2 cos(a + b ) + cos(a - b )]2 = - sin2 (a - b ) 1 A
= AD(c + b)sin
This gives 2 2

2 cos(a + b)
b + cos(a - b)
b =0
Worked-Out Problems 271

This gives 1 1
= (m)(a) = m(2 R)
2D 2 2
AD =
(b + c)sin( A / 2) Therefore
2[(1/ 2)bc sin A] bc
= m=
(b + c)sin( A / 2) 2R
R2
=
bc[2 sin( A / 2) cos( A / 2)] = (∵ bc = AD2 = R2 )
(b + c)sin( A / 2) 2R
R
æ 2bc ö A =
=ç ÷ cos 2
è b + cø 2
Therefore (A) is true. From DADM,
So (A) is true.
Again, from DACD AM m 1
sin( ADM ) = = =
AD R 2
AD AC
= So ADM = 30°. Hence (B) is true. Also
sin C sin(180° - q )
where q = ADB. Hence ADB = 180° - 30° = 150°

b sin C æ A ö and hence


AD = ç∵ B+ + q = 180°÷
sin[ B + ( A / 2)] è 2 ø
DAB = B = 15° (∵ AD = DB)
Therefore (C) is true. This implies
Answers: (A), (C)
C = 75°
14. In DABC, A = 90°, D is the mid-point of BC
C and m Also B = 15°, C = 75° imply that (C) is true.
is the length of the altitude AM
M drawn from A onto
Answers: (A), (B), (C)
BC. It is given that the length of the median AD is
geometric mean between the adjacent sides AB and
15. In DABC, let r be the inradius; and r1, r2 and r3 are the
AC. Let R be the circumradius of DABC. Then
three exradii; s is the semiperimeter of the triangle.
R Then which of the following are true?
(A) m =
2 A
(B) ADM = 30° (A) rr1 = r2 r3 tan2
2
(C) the acute angles of DABC
C are 15° and 75° A
(B) (rr1 )tan = Area of DABC
(D) C = 60° and B = 30° 2
(C) r1r2 + r2 r3 + r3 r1 = s2
C
M C
(D) (r3 - r )cot =c
75° 90° 2
D
b m
Solution: We know that
15°
D D D D
r= , r1 = , r2 = and r3 =
FIGURE 4.71 Multiple correct choice type question 14. s s-a s-b s-c
Also
Solution: See Figure 4.71. Since A = 90°, BC
C is the
circumdiameter of DABC
C and hence AD = DC = DB = R. C s( s - c)
Now cot =
2 ( s - a)( s - b)
1 ( s - b)( s - c)
bc = Area of DABC tan
A
=
2 2 s( s - a)
272 Chapter 4 Properties of Triangles

(A) We have (A) EF = a cos A


D D (B) EDF = 180° - 2 A
´
rr1 s s-a ( s - b)( s - c) A (C) Area of DDEF = 2Dcos A cos B cos C where D is
= = = tan2 the area of DABC
r2 r3 D D s( s - a) 2
´
s-b s-c (D) The radius of the incircle of DDEF
F is 2R cos A
cos B cos C
Therefore (A) is true.
(B) We have A

D D s( s - a)( s - b)( s - c)
rr1 = ´ = = ( s - b)( s - c)
s s-a s( s - a)
Therefore F E
H
A s( s - a)
(rr1 )cot = ( s - b)( s - c)
2 ( s - b)( s - c)

= s( s - a)( s - b)( s - c)
FIGURE 4.72 Multiple correct choice type question 16.
=D
Hence Solution: Let H be the orthocentre of DABC
D (Figure 4.72).
Then
A
(rr1 )tan ¹D HDC = HEC = 90°
2
(B) is not true. which implies HDCE
E is a cyclic quadrilateral. Therefore
(C) We have HDE = HCE = 90° - A

D2 D2 Similarly
r1r2 + r2 r3 + r3r1 = +
( s - a)( s - b) ( s - b)( s - c)
HDF = HBF = 90° - A
D2
+ Therefore
( s - c)( s - a)
EDF = 180° - 2 A
D2 [ s - c + s - a + s - b]
=
( s - a)( s - b)( s - c) Hence (B) is true. Similarly
D s(3s - 2 s)
2
Ds 2 2
DEF = 180° - 2 B and EFD = 180° - 2C
= = 2 = s2
s( s - a)( s - b)( s - c) D
Now, from DAFE,
Hence (C) is true.
EF AE
(D) We have = (4.27)
sin A sin( AFE)
C æ D Dö s( s - c)
(r3 - r )cot =ç - ÷ Now
2 è s - c s ø ( s - a)( s - b)
AFE = 90° - EFH = 90° - HAE = C (4.28)
D (c ) D(c)
= = =c
s( s - a)( s - b)( s - c) D Again, from DABE, ABE = 90° - A. Therefore

So (D) is true. AE AE
cos A = sin(90° - A) = =
Answers: (A), (C), (D) AB c
AE = c cos A
16. In DABC, D, E and F are the feet of the altitudes
drawn from the vertices A, B and C, respectively, From Eqs. (4.27), (4.28) and (4.29) we get
onto the opposite sides. For DDEF,
F which of the EF c cos A
following are true? = = 2 R cos A
sin A sin C
Worked-Out Problems 273

Hence A
(C) B1C1 = a cosec
EF = 2 R sin A cos A = a cos A 2
(D) as n ® ¥, DAnBnCn tends to become equilateral
and so (A) is true. Now
Solution: See Figure 4.73. Clearly C1, A, B1 are collinear
1 and DABC C is the pedal triangle of DA1B1C1. Hence the
Area of DDEF = (DE)(DF )sin D
2 incentre I is the orthocentre of DA1B1C1. Therefore (B)
is true.
1
= (c cos C )(b cos B)sin(180° - 2 A)
2 C1 B1
A
1
= (b
bc)(cos B cos C )(2 sin A cos A)
2
= (bc sin A)(cos A cos B cos C )
I
= 2 D cos A cos B cos C
B C
Hence (C) is true. Also
Area of DDEF
Inradius of DDEF =
(1/ 2)(DE + EF + FD) FIGURE 4.73 Multiple correct choice type question 17.
4 D cos A cos B cos C
= Since DABC C is the pedal triangle of DA1B1C1, from
a cos A + b cos B + c cos C
Problem 16 (part B) we get A = 180° - 2A
2 1, and therefore
4 D cos A cos B cos C A
= A1 = 90° -
R(sin 2 A + sin 2 B + sin 2C ) 2
4 D cos A cos B cos C Hence (A) is true. Also, we can write
=
R(4 sin A sin B sin C )
p æ 1ö æ pö
A1 = + ç- ÷ ç A - ÷
4 D cos A cos B cos C 3 è 2 ø è 3ø
=
æ a b c ö A1
R ç 4× × ×
è 2 R 2 R 2 R ø÷ A2 = 90° -
2
8 R2 (D)cos A cos B cos C
2
p æ - 1ö æ pö
= = + ç ÷ çA- ÷
abc 3 è 2 ø è 3ø
8 R2 (D)cos A cos B cos C By induction,
=
4 RD n
p æ - 1ö æ pö
An = +ç ÷ çA- ÷
= 2 R cos A cos B co
osC 3 è 2ø è 3ø
Hence (D) is also true. Applying limit we get
Answers: (A), (B), (C), (D)
p æ æ 1ö
n
ö
lim An = ç∵ lim ç - ÷ = 0÷
n®¥ è 2 ø
17. For DABC, let A1B1C1 be the triangle formed by the n®¥ 3 è ø
excentres of DABC. Let DA2B2C2 be formed by the
excentres of DA1B1C1 and so on. Finally, let DAnBnCn Therefore (D) is true. Again from Problem 16 (part A),
be the nth derived triangle, that is, it is formed by the BC = ( B1C1 )cos A1
excentres of DAn-1Bn-1Cn-1. Then
æ Aö
A = ( B1C1 )cos ç 90° - ÷
(A) A1 = 90° - è 2ø
2
(B) the incentre of DABC A
C is the orthocentre of = ( B1C1 )sin
DA1B1C1 2
274 Chapter 4 Properties of Triangles

Therefore Therefore

B1C1 = a cosec
A D æ D ö æ D ö
= 2ç ÷ = 3ç
2 s-a è s - bø è s - c ÷ø
Hence (C) is true. 1 2 3 1+ 2 + 3 6
= = = =
Answers: (A), (B), (C), (D) s-a s-b s-c 3s - 2 s s

18. Which of the following statements are true? Equation first with the last one we get
(A) In DABC, if a : b : c = 4 : 5 : 6, then R : r is 16 : 7. 5
s = 6( s - a) Þ a = s
(B) If r1 = 2r2 = 3r3, then a:b is equal to 5 : 4. 6
(C) In DABC, if r1 : r2 : r3 = 4 : 5 : 6, then A = C .
Equation second with the last one we get
(D) DABC C is uniquely determined, if side a, sin A
and circumradius R are known. 4
2 s = 6( s - b) Þ b = s
6
Solution:
(A) Let a = 4K, b = 5K
K and c = 6K. Therefore Therefore a : b = 5 : 4. Hence (B) is true.
(C) We have r1 = 4k, r2 = 5k, r3 = 6k. Therefore
2s = a + b + c = 15K
r2 - r1 = r3 - r2
so that
15K We have from Corollary 4.5 that
s=
2 A B C
r1 = 4 R sin cos cos
Hence 2 2 2
A B C
D2 = s( s - a)( s - b)( s - c) r2 = 4 R cos sin cos
2 2 2
15K æ 15K ö æ 15K ö æ 15K ö A B C
= çè - 4K ÷ ç - 5K ÷ ç - 6K ÷ r3 = 4 R cos cos sin
2 2 ø è 2 ø è 2 ø 2 2 2
K4 Therefore
= (15 ´ 7 ´ 5 ´ 3)
16 r2 - r1 = r3 - r2
2
15 7 K C æ A - Bö A æ B - Cö
D= Þ 4 R cos sin ç
è ÷ø = 4 R cos sin çè ÷
4 2 2 2 2 ø

If A = C, then from above it follows that


D 15 7 K 2 2
K 7 A- B B-C
r= = ´ = =
s 4 15K 2 2 2
abc 4 × 5 × 6 K 3 8 K so that DABC
C becomes equilateral. Hence (C) is
R= = =
4 D 15 7 K 2 7 not true.
(D) We have that a/sin A = 2R. But, the sides b and c
Therefore cannot be determined with this hypothesis. Therefore
(D) is not true.
8 7
R:r = (K ): (K ) = 16 : 7 Answers: (A), (B)
7 2

which implies that (A) is true. 19. For DABC, which of the following are true?
(B) We have (A) If the sides a, b, c and D are rational, then a,
C are rational.
tan(B/2), tan(C/2)
D D D
r1 = , r2 = , r3 = (B) If a, tan(B/2), tan(C/2)
C are rational, then a,
s-a s-b s-c sin A, sin B and sin C are rational.
Worked-Out Problems 275

(C) If a, sin A, sin B and sin C are rational, then a, b, Note: In DABC, the following statements are equivalent.
c and D are rational. (i) a, b, c and D are rational.
(D) If a, b, c and D are rational, then r, r1, r2 and r3 B C
are rational. (ii) a, tan , tan are rational.
2 2
Solution: (iii) a, sin A, sin B and sin C are rational.
(A) Suppose a, b, c and D are rational. Now Actually in this problem, we proved that (i) Þ (ii) Þ
a+b+c (iii) Þ (i).
s= Answers: (A), (B), (C), (D)
2
and so, s is rational. Now 20. In DABC, if the angles are in AP and 3a2 = 2b2, then
(A) A = 45°
B ( s - b)( s - c) ( s - b)( s - c)
tan = = (B) C = 75°
2 s( s - a) D
(C) C = 45°
C/2) is rational. Therefore
C
(D) A = 60°
B C
a, tan , tan are rational Solution: We have
2 2
A + B + C = 180°
and (A) is true.
(B) Suppose a, tan(B/2), tan(C/2)
C are rational. Therefore and A + C = 2B

2 tan( B / 2) 2 tan(C/ 2) Solving the two equations we get


sin B = , sin C =
1 + tan ( B / 2)
2
1 + tan2 (C/ 2) 3B = 180° Þ B = 60°

are rational. Now Therefore

A æ B + Cö b a
tan = tan ç 90° - ÷ =
2 è 2 ø sin 60° sin A

æ B + Cö 2b a
= cot ç Þ =
è 2 ÷ø 3 sin A
1 - tan( B / 2)tan(C/ 2) (2 sin A)b
= Þa=
tan( B / 2) + tan(C/ 2) 3

tan(A/2) is also rational. Hence Now

2 tan( A / 2) 3a2 = 2b2 Þ (4 sin2 A)b2 = 2b2


sin A =
1 + tan2 ( A / 2)
Therefore
is rational. So, a, sin A, sin B, sin C are rational and
1
hence (B) is true. sin2 A =
2
(C) Suppose a, sinA, sinB and sinC C are rational. Therefore
2R = a/sinA is rational. Similarly b = 2RsinB, c = Þ sin A =
1
2RsinC C are rational and hence D is also rational. This 2
implies (C) is true.
Þ A = 45° (∵ B = 60° Þ A ¹ 135°)
(D) Suppose a, b, c and D are rational. Then s and D are
rational and hence Now, A = 45°, B = 60° implies C =75°.
D D D D Answers: (A) and (B)
r= , r1 = , r2 = , r3 = are also rational
s s-a s-b s-c
Therefore (D) is also true.
276 Chapter 4 Properties of Triangles

Matrix-Match Type Questions


1. Match the items of Column I to those of Column II A

90°
Column I Column II
(A) In DABC, if 2a2 + 4b2 + c2 = 4ab + 2ac (p) 1
then the value of 8cosB is
(B) In DABC, if cos A cos B + sin A sin B FIGURE 4.74 Matrix-match type question 1.
(q) 0
sin C = 1, then the value of
Therefore
cos(A - B) equals
(C) Let D be the mid-point of the side (r) 2/3 2b a2 + b2 - c2
BCC of DABC
D . If DAC = 90°, then =
a 2ab
a2 - c2 = mb2, where the value of m is
(s) 3 4b2 = a2 + b2 - c2
(D) In DA
D BC, if cos2 A + cos2 B + cos2 C = 1,
then the value ofcosAcosBcosC C is 3b2 = a2 - c2
equal to (t) 7 Answer: (C) Æ (s)
(D) We have
Solution: Consider Figure 4.74.
(A) We have cos2 A + cos2 B + cos2 C = 1

2a2 + 4b2 + c2 = 4ab + 2ac 1 + cos 2 A 1 + cos 2 B 1 + cos 2C


+ + =1
2 2 2
(a2 + 4b2 - 4ab) + (a2 + c2 - 2ac) = 0
cos 2 A + cos 2 B + cos 2C = - 1
(a - 2b)2 + (a - c)2 = 0
- 1 - 4 cos A cos B cos C = - 1
Therefore (∵ A + B + C = 180°)
a = 2b and a=c cos A cos B cos C = 0

Now Answer: (D) Æ (q)

8(c2 + a2 - b2 ) 2. Match the items of Column I to those of Column II.


8 cos B =
2ca
8(4b2 + 4b2 - b2 ) 8 ´ 7b2 Column I Column II
= = =7
2(2b)(2b) 8b2 (A) In DABC, if a = 3b and (p) 1
Answer: (A) Æ (t) A - B = 90°, then tan B equals
(B) We have (B) AD is a diameter of the (q) 1/3
circumcircle of DABC, intersecting
cos A cos B + sin A sin B sin C = 1 the side BC
C internally. Then
(cos A cos B + sin A sin B)
æ Area of DBDC ö
+ (sin A sin B sin C - sin A sin B) = 1 çè R2 sin A cos B cos C ÷ø =
[sin A sin B(sin C - 1)] = 1 - cos( A - B)
(C) In DABC, the altitudes AD, BE (r) 1/2
Now LHS £ 0 and RHS ³ 0 Þ LHS = RHS = 0. Hence and CF
F meet the circumcircle in P,
Q and R respectively. Then
cos( A - B) = 1 and sin C = 1
Answer: (B) Æ (p) BC CA AB
+ +
(C) From DACD, DP EQ FR

AC 2b = k(tan A + tan B + tan C )


cosC = =
DC a where the value of k is
Also from DABC, (D) In DABC, if sin2 A + sin2 B + (s) 2
sin2 C = 2 and if q is one of A, B and
a2 + b2 - c2 C, then the value of sin q is equal to
cosC =
2ab
Worked-Out Problems 277

Solution: From DACD, we have


(A) From Napier’s rule (Theorem 4.8) DC DC
cot B = =
A-B a-b æCö AC b
tan = cot ç ÷
2 a+b è 2ø
Therefore
æ 3b - b ö C DC = b cot B
1=ç ÷ cot
è 3b + b ø 2
Similarly BD = c cot C . Therefore
C
cot =2 (4.30)
2 1
Area of DBDC = ( BD)(DC )sin D
2
Also
1
A + B + C = 180° Þ (90° + B) + B + C = 180° = (c cot C )(b cot B)sin( B + C )
2
C 1
Þ B = 45° - = (bc)co
ot C cot B sin A
2 2
Therefore 1
= (2 R sin B)(2 R sin C )cot B cot C sin A
2
æ Cö
tan B = tan ç 45° - ÷ = 2 R2 cos B cos C sin A
è 2ø
1 - tan(C/ 2) Therefore
= [from Eq. (4.30)]
1 + tan(C/ 2) Area of DBDC
=2
1 - (1/ 2) R2 sin A cos B cos C
=
1 + (1/ 2) Answer: (B) Æ (s)
1 (C) Join BP
P and CP. Now
=
3 PBC = PAC = 90° - C
Answer: (A) Æ (q) DPB = C and DPC = B
(B) We have that AD is a diameter (Figure 4.75). This
implies that Therefore

ABD = ACD = 90° BPC = B + C = 180° - A

Also ADC = B and ADB = C . Therefore From DDPC,

BDC = B + C DC
tan B =
DP
DC = (DP )tan B
A
A
R
C
E

R
F C

90°
B
D

C P
B
C – c
B 90°

FIGURE 4.75 Matrix-match type question 2 part (B).


FIGURE 4.76
278 Chapter 4 Properties of Triangles

Again from DPBC, Therefore


BD = (DP )tan C B-C
= 45° or B - C = 90°
a = BD + DC = (DP )(tan B + tan C ) 2

BC a Also
= = tan B + tan C
DP DP B + C = 180° - A = 180° - 30° = 150°
Similarly Therefore B = 120°, C = 30°. Now
AC A=CÞa=c=1
= tan C + tan A
EQ
a 1
and 2R = = =2ÞR=1
AB sin A sin 30°
and = tan A + tan B
FR Answer: (A) Æ (s), (B) Æ (q), (C) Æ (p), (D) Æ (p)
On adding we get
4. The sides of a triangle are 13, 14 and 15. In Column I
BC AC AB the inradius and the three exradii of the triangle are
+ + = 2(tan A + tan B + tan C ) given and their values are given in Column II. Match
DP EQ FR
these.
Therefore k = 2.
Answer: (C) Æ (s) Column I Column II
(D) We have
(A) r (p) 14
sin B + sin B + sin C = 2
2 2 2
(q) 21/2
(B) r1
1 - cos 2 A 1 - cos 2 B 1 - cos 2C (r) 4
+ + =2 (C) r2
2 2 2 (s) 12
cos 2 A + cos 2 B + cos 2C = - 1 (D) r3 (t) 19/2
- 1 - 4 cos A cos B cos C = - 1
Solution: Let a = 13, b = 14, c = 15. We know that
cos A cos B cos C = 0 2s = a + b + c. Therefore
Therefore q = 90° and sin q = 1. 2s = 13 + 14 + 15 = 42
Answer: (D) Æ (p) Þ s = 21

3. In DABC, it is given that b = 3 , c = 1 and A = 30°. From these values we have s - a = 8, s - b = 7, s - c = 6,


Match the items of Column I with those in Column II. Now

D2 = s( s - a)( s - b)( s - c) = 21 ´ 8 ´ 7 ´ 6
Column I Column II
D=7´4´3
(A) B is (p) 1
Therefore
(B) C is equal to (q) 30°
D 7´4´3
(C) Side ‘a’ equals (r) 2 r= = =4
s 21
(D) Circumradius R equals (s) 120°
D 7 ´ 4 ´ 3 21
r1 = = =
Solution: We have by Napier’s rule that s-a 8 2

B - C æ b - cö A D 7´4´3
tan =ç cot r2 = = = 12
2 ÷
è b + cø 2 s-b 7
D 7´4´3
æ 3 - 1ö r3 = = = 14
=ç ÷ cot 15° s-c 6
è 3 + 1ø
Answer: (A) Æ (r), (B) Æ (q), (C) Æ (s), (D) Æ (p)
=1
Worked-Out Problems 279

5. For DABC, match the items of Column I with those (B) We have
of Column II.
1 1 1 a+b+c
+ + =
bc ca ab abc
Column I Column II
2s 2s 1
= = =
(A) rr1r2r3 is equal to (p)
abc abc 4 RD 2 Rr
2 R2 Answer: (B) Æ (r)
1 1 1 (C) We have
(B) + + equals (q) D2
bc ca ab
æ a b c ö
1 4 R sin A sin B sin C = (4 R) ç × ×
(C) 4 R sin A sin B sin C is equal to (r) è 2 R 2 R 2 R ÷ø
2Rr
A abc
(D) r2 r12 cot2 equals (s) D =
2 2 R2
Answer: (C) Æ (p)
Solution: (D) We have
(A) We have
A D2 D2 s( s - a)
r2 r12 cot2 = 2× ×
D D D D 2 s ( s - a) ( s - b)( s - c)
2
rr1r2 r3 = × × ×
s s-a s-b s-c
D4
= = D2
D4 D2
= = D2
D2 Answer: (D) Æ (q)
Answer: (A) Æ (q)

Comprehension-Type Questions
1. Passage: In DABC, O, I, H and G respectively denote 1 2
circumcentre, incentre, orthocentre and centroid. It is (C) R2 - (a + b2 + c2 )
9
known that
1
(D) (3R2 - a2 - b2 - c2 )
æ A B Cö 9
(OI ) = R ç 1 - 8 sin sin sin ÷
2 2
è 2 2 2ø
Solution:
(OH )2 = R2 (1 - 8 cos A cos B cos C ) (i) We have

Answer the following questions. æ A B Cö


(OI )2 = R2 ç 1 - 8 sin sin sin ÷
I 2 is equal to
(i) (OI) è 2 2 2ø
(A) R2 + 2Rr æ A B Cö
= R2 - 2 R ç 4 R sin sin sin ÷
(B) R2 - 2Rr è 2 2 2ø
(C) R2 - Rr = R2 - 2Rr
(D) R2 + Rr Answer: (B)
2
I equals
(ii) (OI) (ii) Again
(A) -R2(2 cos A + 2 cosB + 2 cos C - 3) A B C
OI 2 = R2 - 8 R2 sin sin sin
(B) R2(2 cos A + 2 cos B + 2 cos C - 3) 2 2 2
2 + cos 2B + cos 2C)
(C) R2(cos 2A æ A B Cö
= R2 - 2 R2 ç 4 sin sin sin ÷
(D) -R (cos A + cos B + cos C - 2)
2
è 2 2 2ø
(iii) (OG)2 is equal to = R2 - 2 R2(cos A + cos B + cos C - 1)
(A) R - (a + b + c )
2 2 2 2
= R2 - R2(2 cos A + 2 cos B + 2 cos C - 3 + 1)
1
(B) R2 - (a2 + b2 + c2 ) = - R2 (2 cos A + 2 cos B + 2 cos C - 3)
3
Answer: (A)
280 Chapter 4 Properties of Triangles

(iii) By Theorem 4.27 I3 A I2

1
OG : GH = 1: 2 Þ OG = (OH )
3 I

Therefore B C

1
(OG)2 = (OH )2
9
1 2
= (R - 8 R2 cos A cos B cos C ) (Theorem 4.28)
9 FIGURE 4.77 Comprehension type question 2.
R2
= [1 + 2(cos 2 A + cos 2 B + cos 2C + 1)]
9 Also, we have proved that (Problem 17 of Multiple Correct
Choice Type Questions)
R2
= [3 + 2(1 - 2 sin2 A + 1 - 2 sin2 B + 1 - 2 sin2 C )]
9 A
I1 = 90° -
2
R2
= (9 - 4 sin2 A - 4 sin2 B - 4 sin2 C ) B
9 I2 = 90° -
2
1
= R2 - (4 R2 sin2 A + 4 R2 sin2 B + 4 R2 sin2 C ) C
9 and I3 = 90° -
2
1
= R2 - (a2 + b2 + c2 ) (i) We have
9
Answer: (C) A
I I1 = AI1 - AI = cosec (r1 - r )
2
2. Passage: I1, I2, I3 are the three excentres of DABC
opposite to the vertices A, B, C, respectively, and I is Aæ D Dö
= cosec ç - ÷
its incentre. Answer the following three questions. 2 ès-a sø
(i) I I1 is equal to A æ Da ö
= cosec
A A 2 çè s( s - a) ÷ø
(A) a sec (B) a cosec
2 2
A ( s - b)( s - c)
A A = a cosec
(C) a cos (D) 2a sin 2 s( s - a)
2 2
(ii) The length I2I3 is equal to A A
= a cosec tan
2 2
A A
(A) a sec (B) a cosec A
2 2 = a sec
2
A A
(C) 2a cosec (D) 2a sec Answer: (A)
2 2
(iii) Area of DII1I2I3 is (ii) Also from Problem 16 (of Multiple Correct Choice
Type Questions) we have
abc abc
(A) (B) a = BC = ( I2 I3 )cos I1
r Rr
abc abc æ Aö
(C) (D) = I2 I3 cos ç 90° - ÷
2 Rr 2r è 2ø
Solution: See Figure 4.77. It is easy to see that Therefore
A A
AI = r cosec I2 I3 = a cosec
2 2
A Answer: (B)
AI1 = r1 cosec
2
Worked-Out Problems 281

(iii) Now Solution:


(i) We have
C
I1 I2 = c cosec a sec A + b sec B + c sec C
2
B tan A + tan B + tan C
I1 I3 = b cosec
2 2 R tan A + 2 R tan B + 2 R tan C
=
A tan A + tan B + tan C
and I1 = 90° -
2 = 2R
imply that Answer: (B)
(ii) By hypothesis a + c = 2b. Also
Area of DI1 I2 I3 =
1
2
( )
( I1 I3 )( I1 I2 )sin I1
A C ( s - b)( s - c) ( s - a)( s - b)
tan + tan = +
1æ Bö æ Cö æ Aö 2 2 s( s - a) s( s - c)
= çè b cosec ÷ø çè c cosec ÷ø sin çè 90° - ÷ø
2 2 2 2 s - b ( s - c + s - a)
=
1 B C A s( s - a)( s - c)
= (bc) cosec cosec cos
2 2 2 2
s - b (3b - 2b)
æ A Aö = (∵ a + c = 2b)
2 sin cos s( s - a)( s - c)
1 B Cç 2 2÷
= (bc)co
osec cosec ç
2 2 2 A ÷
çè 2 sin ÷ø b s( s - b)
2 =
s ( s - a)( s - c)
æ ö
1 B C ç sin A ÷ b B
= (bc) cosec cosec ç = cot
2 2 2 2 sin A ÷ s 2
çè ÷
2ø 2b B
= cot (∵ 2 s = a + b + c = 3b)
æ ö 3b 2
1 æ a ö ç 1 ÷ 2 B
= (bc) ç ÷ ×ç ÷ = cot
2 è 2R ø A B C 3 2
çè 2 sin sin sin ÷ø
2 2 2
Answer: (D)
abc æ A B Cö (iii) We have
= çè∵ r = 4 R sin sin sin ÷ø
2r 2 2 2
lbc = (b + c)2 - a2 = (b2 + c2 - a2) + 2bc
Answer: (D)
Therefore
3. Passage: Using sine and cosine rules and any standard
formulae for a triangle, answer the following questions. bc(l - 2) = b2 + c2 - a2
a sec A + b sec B + c sec C l - 2 b2 + c2 - a2
(i) is equal to = = cos A
tan A + tan B + tan C 2 2bc
(A) R (B) 2R (C) r (D) 2r Hence
(ii) If the sides a, b, c are in AP, then the value of
l-2
-1 £ £1
A C B 2
tan + tan = k cot
2 2 2
But
where k is equal to
(a + b + c)(b + c - a)
3 3 4 2 l= >0 (∵ b + c > a)
(A) (B) (C) (D) bc
2 4 3 3
(iii) If (a + b + c) (b + c - a) = lbc, then Therefore
(A) 0 < l < 4 (B) 0 < l £ 3 0 < l < 4 (∵ l = 4 Þ cos A = 1)
(C) 0 < l £ 2 (D) 0 < l £ 1 Answer: (A)
282 Chapter 4 Properties of Triangles

Assertion–Reasoning Type Questions


In the following set of questions, a Statement I is given ³ 9 3 sin A sin B sin C
and a corresponding Statement II is given just below it.
Mark the correct answer as: 3 cosec A cosec B cosec C = 9
(A) Both Statements I and II are true and Statement II
Therefore equality implies sinA = sinB = sinC. Statement I
is a correct explanation for Statement I
is true.
(B) Both Statements I and II are true but Statement II is
Answer: (A)
not a correct explanation for Statement I
(C) Statement I is true and Statement II is false 2. Statement I: In DABC, if
(D) Statement I is false and Statement II is true
æ B - Cö A
x = tan ç tan
1. Statement I: In DABC, if è 2 ÷ø 2
æ C - Aö B
a cos A + b cos B + c cos C a + b + c y = tan ç ÷ tan
= è 2 ø 2
a sin B + b sin C + c sin A 9R
æ A - Bö C
then DABC
C is equilateral. z = tan ç ÷ tan
è 2 ø 2
Statement II: If x, y, z are positive, then
then x + y + z = -xyz
- .
x+ y+z 3
³ xyz Statement II: In DABC,
3
where the equality holds if and only if x = y = z. B - C æ b - cö A
tan =ç ÷ cot
Solution: Let x1, y1 and z1 be positive real numbers. Now
2 è b + cø 2

x13 + y13 + z13 - 3 x1 y1z1 C - A æ c - aö B


tan =ç cot
2 è c + a ÷ø 2
= ( x1 + y1 + z1 )( x12 + y12 + z12 - x1 y1 - y1z1 - z1 x1 )
A - B æ a - bö C
x1 + y1 + z1 and tan =ç cot
= [( x1 - y1 )2 + ( y1 - z1 )2 + (z1 - x1 )2 ] ³ 0 2 è a + b ÷ø 2
2
Solution: We have
x1 = y1 = z1. Now put
x = x1 , y = y , z = z1 . Therefore
3
1
3 3
æ b - cö A æ sin B - sin C ö A
çè b + c ÷ø cot 2 = çè sin B + sin C ÷ø cot 2
x + y + z ³ 3(xyz) 1/3

and equality occurs if and only if x = y = z. Hence æ B + Cö æ B - Cö æ Aö


2 cos ç ÷ sin ç
Statement II is true. Now è 2 ø è 2 ÷ø ç cos 2 ÷
=
æ B + Cö æ B - C ö ç sin A ÷
a cos A + b cos B + c cos C a + b + c 2 sin ç ÷ cos ç ç ÷
= è 2 ø è 2 ÷ø è 2ø
a sin B + b sin C + c sin A 9R
æ B - Cö
sin 2 A + sin 2 B + sin 2C
C 2 sin ç
è 2 ÷ø
= (sin A + sin B + sin C ) =
2å (sin A sin B) 9 æ B - Cö
cos ç
4 sin A sin B sin C 2 è 2 ÷ø
= (sin A + sin B + sin C )
2å (sin A sin B) 9 æ B - Cö
= tan ç
è 2 ÷ø
9 sin A sin B sin C = (sin A + sin B + sin C )

(å (sin A sin B))


B-C A b-c
9 = (sin A + sin B + sin C ) x = tan tan =
2 2 b+c
(cosec A + cosec B + cosec C )
Therefore
x(b + c) = b - c
Worked-Out Problems 283

( x + 1)c = (1 - x)b Let R be the circumradius of DABC. Therefore


1+ x b MN NL LM
= = = = 2R
1- x c sin(180° - 2 A) sin(180° - 2 B) sin(180° - 2C )
Similarly
1+ y c 1+ z a LM = 2 R sin 2C
= and =
1- y a 1- z b LN = 2 R sin 2 B
Hence and MN = 2 R sin 2 A

æ 1 + xö æ 1 + y ö æ 1 + zö Now,
çè 1 - x ÷ø × çè 1 - × =1
y ÷ø çè 1 - z ÷ø 1
Area of DLMN = (LN )(LM )sin(180° - 2 A)
(1 + x)(1 + y)(1 + z) = (1 - x)(1 - y)(1 - z) 2
1
Simplifying both sides, we will have = (2 R sin 2 B)(2 R sin 2C )sin 2 A
2
2(x + y + z) = -2(xyz) = 2 R2 (2 sin B cos B)(2 sin C cos C )(2 sin A cos A)
Answer: (A) = 16 R2 (sin A sin B sin C )((cos A cos B cos C )
3. Statement I: In DABC, the altitudes AD, BE
E and æ a b ö
= 16 R2 ç × sin C ÷ (cos A cos B cos C )
CFF meet the circumcircle in L, M and N, respectively, è 2R 2R ø
and D is the area of DABC. Then, the area of DLMN
æ1 ö
is 8D cos A cos B cos C. = 8 ç ab sin C ÷ (cos A cos B cos C )
è2 ø
Statement II: DABC
C and DLMN
N have the same
= 8 D cos A cos B cos C
circumcircle.
Answer: (A)
Solution: See Figure 4.78. Statement II is true from the
hypothesis. A B C 1
4. Statement I: In DABC, sin sin sin £ .
ALM = ABM = 90° - A 2 2 2 8
ALN = ACN = 90° - A Statement II: For any real q, 0 £ cos2 q £ 1.

Therefore Solution: Statement II is true because for any real q,


-1 £ cos q £ 1. Now
MLN = 180° - 2 A
A B C
2 sin sin sin
Similarly, 2 2 2
NML = 180° - 2 B and MNL = 180° - 2C é æ A - Bö æ A + Bö ù æ A + Bö
= êcos ç ÷ø - cos çè ÷ø ú cos çè ÷
ë è 2 2 û 2 ø
A æ A + Bö æ A - Bö æ A + Bö
= - cos2 ç + cos ç cos ç
è 2 ÷ø è 2 ÷ø è 2 ÷ø
2
N M é æ A + Bö 1 æ A - Bö ù 1 2 æ A - Bö
= - êcos ç ÷ø - cos çè ÷ø ú + cos çè ÷
F 90° E ë è 2 2 2 û 4 2 ø
90°
1 æ A - Bö 1
£ cos2 ç £
è 2 ÷ø 4
90°
4
90°
Therefore
B C
D
A B C 1
sin sin sin £
2 2 2 4
Answer: (A)
FIGURE 4.78 Assertion–reasoning type question 3.
284 Chapter 4 Properties of Triangles

a2 + b2 + c2 ( AD)2 = ( AB)2 + ( BD)2 - 2( AB)( BD)cos B


5. Statement I: In DABC
D , cotA + cot B + cot C = .
4D a2 æ a ö (c + a - b )
2 2 2
= c2 + - 2(c) ç ÷
Statement II: In any DABC, 4 è 2ø 2ca

b2 + c2 - a2 a2 c2 + a2 - b2
cos A = = c2 + -
2bc 4 2
c2 + a2 - b2 2b2 + 2c2 - a2
cos B = =
2ca 4
a2 + b2 - c2 4( AD)2 = 2b2 + 2c2 - c2
cos C =
2ab
Thus Statement II true. Now
and
2
1 1 1 BG = ( BE)
bc sin A = ca sin B = ab sin C = D 3
2 2 2 (4.31)
4 1
Þ ( BG) = ( BE)2 = (2c2 + 2a2 - b2 )
2
Solution: Statement II is standard formulae. Now 9 9
cos A cos B cos C Similarly
cot A + cot B + cot C = + +
sin A sin B sin C
1
(CG)2 = (2a2 + 2b2 - c2 ) (4.32)
b2 + c2 - a2 c2 + a2 - b2 a2 + b2 - c2 9
= + +
2bc sin A 2ca sin B 2ab sin C
In DBGC,
b2 + c2 - a2 c2 + a2 - b2 a2 + b2 - c2 cos a
= + +
4D 4D 4D cot a =
sin a
a2 + b2 + c2
= ( BG)2 + (CG)2 - ( BC )2
4D =
2( BG)(CG)sin a
Answer: (A)
b2 + c2 - 5a2
6. Statement I: Let G be the centroid of DABC
C and = [Eqs. (4.31) and (4.32)]
36(Arrea of DBGC )
BGC = a , CGA = b and AGB = g . Then
b2 + c2 - 5a2
cot A + cot B + cot C + cot a + cot b + cot g = 0 = [∵ D = 3(Area of DBGC )]
36(D / 3)
Statement II: If AD, BE
E and CF
F are the medians of b2 + c2 - 5a2
DABC, then =
12 D
2 AD = 2b2 + 2c2 - a2 Similarly

2 BE = 2c2 + 2a2 - b2 c2 + a2 - 5b2


cot b =
12 D
2CF = 2a2 + 2b2 - c2
a2 + b2 - 5c2
and cot g =
A
12 D
Therefore
E
-3(a2 + b2 + c2 )
F G cot a + cot b + cot g =
a
12 D
C
(a2 + b2 + c2 )
D =-
4D
= - (cot A + cot B + cot C )
FIGURE 4.79 Assertion–reasoning type question 6.
(See Problem 5 of Assertion–Reasoning Type Questions.)
Solution: From DABD (Figure 4.79),
Answer: (A)
Worked-Out Problems 285

Integer Answer Type Questions


1. In a triangle, the lengths of two sides are 4 and 6. where equality occurs if C = 90° so that b has only
The angle opposite to the side of length 6 is 30°. The one value. Therefore we can suppose that sin C ¹ 1.
number of such triangles is . Therefore the equation

A b2 - 2bc cos A + c2 - a2 = 0
has two distinct roots say b1 and b2. Therefore
c 6 b1 + b2 = 2c cos A

and b1b2 = c2 - a2
30°

Since b2 = 2b1 (given) we have


FIGURE 4.80
3b1 = 2c cos A
Solution: See Figure 4.80. From DABC,
and 2b12 = c2 - a2
BC AC
= Therefore
sin A sin B
2
Substituting the values we get æ2 ö
c2 - a2 = 2b12 = 2 ç c cos A÷
è3 ø
4 6
= 9c2 - 9a2 = 8c2 cos2 A = 8c2 (1 - sin2 A)
sin A sin 30°
Therefore 9a2 = c2 (1 + 8 sin2 A)

4 sin 30° 4 1 2 3a = c 1 + 8 sin2 A


sin A = = ´ =
6 6 2 3 l = 8.
2 2 Answer: 8
Þ A = Sin-1 or p - Sin-1
3 3
3. In D ABC, suppose the sides a = BC, b = AC
C and the
Hence the number of triangles is 2. angle A are known. If B1, C1 and B2, C2 are the other
Answer: 2 values of the angles B and C, then
sin C1 sin C2
2. In D ABC, it is given that the values of sides BC = a, + = k cos A
c = AB and the angle A. Then, the third side AC = b sin B1 sin B2
has two values b1 and b2 such that b2 = 2b1. Then where k is equal to .
3a = c 1 + l sin2 A where the value of l is .
Solution: Since
Solution: By cosine rule
b2 - 2bc cos A + c2 - a2 = 0 (4.33)
a = b + c - 2bc cos A
2 2 2
we have
Therefore
c2 - 2c(b cos A) + b2 - a2 = 0
b - 2bc cos A + c - a = 0
2 2 2
Let c1 and c2 be values of c satisfying the above quadratic.
is a quadratic equation in b whose discriminant is Therefore

4c2 cos2 A - 4(c2 - a2 ) = 4[c2 (1 - sin2 A) - c2 + a2 ] c1 + c2 = 2b cos A

= 4(a2 - c2 sin2 A) c1c2 = b2 - a2

= 4(a2 - a2 sin2 C ) But also, if b1 and b2 are the two values of b satisfying
Eq. (4.33), then we have two angles B1 and B2 corre-
(∵ c sin A = a sin C )
sponding to b1 and b2. Therefore
= 4a2 (1 - sin2 C ) ³ 0
286 Chapter 4 Properties of Triangles

sin C1 sin C2 sin C1 sin C2 6. In D ABC, the value of


+ = +
sin B1 sin B2 sin B1 sin(180° - B1 ) A B C r
cos2 + cos2 + cos2 = m +
sin C1 sin C2 2 2 2 2R
= +
sin B1 sin B1
where m is equal to .
sin C1 + sin C2
= Solution: We have
sin B1
A B C
k(c1 + c2 ) cos2 + cos2 + cos2
= (where k is a constaant) 2 2 2
kb
1 + cos A 1 + cos B 1 + cos C
c +c 2b cos A = + +
= 1 2 = = 2 cos A 2 2 2
b b
3 1
Therefore k = 2. = + (cos A + cos B + cos C )
2 2
Answer: 2 3 1æ A B Cö
= + ç 1 + 4 sin sin sin ÷
2 2 è 2 2 2ø
4. In D ABC, if the sides a, b, c(in this order) are in AP.
C is equal to
Then cot(A/2) cot(C/2) . 3 1 A B C
= + + 2 sin sin sin
Solution: By hypothesis a + c = 2b. Now 2 2 2 2 2
A B C
2 s = a + b + c = 3b 4 R sin sin sin
=2+ 2 2 2
3 2R
Þs= b
2 r
=2+
Also 2R
Comparing with the given equation we get m = 2.
A C s( s - a) s( s - c)
cot cot = ´ Answer: 2
2 2 ( s - b)( s - c) ( s - a)( s - b)
s 2s 3b 7. In D ABC, if r, r1, r2 and r3 are the inradius and exradii,
= = = =3 respectively, then r1 + r2 + r3 - r = nR where n is equal
s - b 2 s - 2b 3b - 2b
to .
Answer: 3
Solution: According to Corollary 4.3 and Corollary 4.5
5. The sides of a triangle are x, y and x2 + xy + y2 . If q A B C
is the greatest angle of the triangle, then the value of r = 4 R sin sin sin
2 2 2
4 sin2 q is .
A B C
r1 = 4 R sin cos cos
Solution: We have 2 2 2
A B C
x2 + xy + y2 > x, y r2 = 4 R cos sin cos
2 2 2
which implies the greatest angle q is the angle opposite A B C
r3 = 4 R cos cos sin
to the side x + xy + y . Therefore by cosine rule
2 2
2 2 2
where R is the circumradius. Now,
x2 + xy + y2 = x2 + y2 - 2 xy cosq
Cæ A B A Bö
Therefore r1 + r2 = 4 R cos ç sin cos + cos sin ÷ø
2è 2 2 2 2
1
cosq = - Þ q = 120° C æ A + Bö
2 = 4 R cos sin ç
2 è 2 ÷ø
Hence 4 sin2 q = 3.
C
Answer: 3 = 4 R cos2
2
Summary 287

Cæ A B A Bö 2D2
r3 - r = 4 R sin ç cos cos - sin sin ÷ø =
2è 2 2 2 2 D[3s - s(b + c + c + a + a + b) + bc + ca + ab]
2

C æ A + Bö 2D
= 4 R sin cos ç =
è 2 ÷ø
(4.34)
2 bc + ca + ab - s2
C Now
= 4 R sin2
2
a b c a( s - a) b( s - b) c( s - c)
Therefore + + = + +
r1 r2 r3 D D D
æ C Cö s(a + b + c) - (a2 + b2 + c2 )
r1 + r2 + r3 - r = 4 R ç cos2 + sin2 ÷ = 4 R =
è 2 2ø D
Comparing with the given equation we get n = 4. 2 s2 - (a + b + c)2 + 2(ab + bc + ca)
=
Answer: 4 D
2 s2 - 4 s2 + 2(ab + bc + ca)
=
a + b+ c æ a b cö D
8. In any D ABC, + + is equal to______.
r1 + r2 + r3 çè r1 r2 r3 ÷ø 2[(ab + bc + ca) - s2 ]
= (4.35)
D
Solution: We have
Therefore, from Eqs. (4.34) and (4.35)
a+b+c 2s
=
r1 + r2 + r3 é 1 1 1 ù a + b+ cæ a b cö
Dê + + ú + + =4
ës - a s - b s - cû r1 + r2 + r3 çè r1 r2 r3 ÷ø
2 s( s - a)( s - b)( s - c) Answer: 4
=
D[(ss - b)( s - c) + ( s - c)( s - a) + ( s - a)( s - b)]

SUMMARY
Properties of Triangles 4.3 Projection formula:
f a = b cos C + c cos B, b = c cos A +
a cos C and c = a cos B + b cos A.
Notation: Throughout this summary, the following nota-
tion will be used for DABC.
4.4 Area:
(i) The lengths of the sides BC, CA and AB will be 1 1 1
denoted by a, b, c, respectively and (i) bc sin A = ca sin B = ab sin C = D
2 2 2
a+b+c (ii) D = s( s - a)( s - b)( s - c)
s=
2
abc
A, B and C represent the vertices as well as the (iii) D =
4R
angles as per the context.
(ii) O denotes the circumcentre, I the incentre I1, I2, I3 (iv) D = 2R2 sin A sin B sin C
excentres opposite to the vertices A, B and C,
respectively, G the centroid and H the orthocentre. 4.5 Half the angles:
(iii) R is the circumradius and r the inradius and r1, r2, r3 A ( s - b)( s - c)
the exradii of the excircles opposite to the vertices (i) sin =
2 bc
A, B and C, respectively.
(iv) D denotes the area of the triangles. B ( s - c)( s - a)
sin =
2 ca
a b c
4.1 Sine rule: = = = 2R C ( s - a)( s - b)
sin A sin B sin C sin =
2 ab
4.2 Cosine formula: a2 = b2 + c2 - 2bc cos A, b2 = c2 + a2 -
2ca cos B and c2 = a2 + b2 - 2ab cos C.
288 Chapter 4 Properties of Triangles

A s( s - a) 4.11 Inradius:
(ii) cos = D
2 bc (i) r =
s
B s( s - b)
cos = A
2 ca (ii) r = ( s - a)tan
2
C s( s - c) B
cos = r = ( s - b)tan
2 ab 2
A ( s - b)( s - c) C
(iii) tan = r = ( s - c)tan
2 s( s - a) 2
A B C
B ( s - c)( s - a) (iii) r = 4R sin sin sin
tan = 2 2 2
2 s( s - b)

C ( s - a)( s - b) 4.12 Exradii:


tan = D D D
2 s( s - c) (i) r1 = , r2 = , r3 =
s-a s-b s-c
4.6 Napiere’s rule: A B C
(ii) r1 = s tan , r2 = s tan , r3 = s tan
2 2 2
æ B - C ö æ b - cö A
tan ç = cot A B C
è 2 ÷ø çè b + c ÷ø 2 (iii) r1 = 4 R sin cos cos
2 2 2
æ C - Aö æ c - a ö B A B C
tan ç = cot and r2 = 4 R cos
è 2 ÷ø çè c + a ÷ø
sin cos
2 2 2 2
A B C
æ A - Bö æ a - bö C r3 = 4 R cos cos sin
tan ç = cot
è 2 ÷ø çè a + b ÷ø 2
2 2 2
(iv) r1 + r2 + r3 - r = 4R
4.7 Lengths of the medians: Let ma, mb and mc be the 1 1 1 1
lengths of the medians through the vertices A, B (v) + + =
r1 r2 r3 r
and C of DABC. Then

4 ma2 = 2b2 + 2c2 - a2 4.13 If p1, p2 and p3 are the altitudes drawn from the
vertices A, B and C on to the opposite sides, then
4 mb2 = 2c2 + 2a2 - b2
2D 2D 2D
(i) p1 = , p2 = , p3 =
4 mc2 = 2a2 + 2b2 - c2 a b c
1 1 1 1
4.8 In DABC, the (ii) + + =
p1 p2 p3 r
angle A is obtuse if and only if

mb2 + mc2 4.14 Pedal triangle: The triangle whose vertices are the
ma2 < feet of the altitudes drawn from the vertices A, B
5
and C on to the opposite sides is called the pedal
4.9 Lengths of internal angle bisectors: In DABC, the triangle of DABC.
length of the internal bisector of the angle A is
4.15 The distances of the orthocentre from the vertices
æ 2bc ö A bca2 and d from the sides: In DABC
D , D, E and F are the feet of
çè b + c ÷ø cos = bc - the altitudes from the vertices on to the sides BC, CA
2 (b + c)2
and AB, respectively, and H is the orthocentre. Then
4.10 The area of the triangle formed by the feet of the (i) AH = 2R cos A, BH = 2R cos B, CH = 2R cos C.
internal angle bisector of DABC
C is (ii) HD = 2R cos B cos C, HE = 2R cos C cos A, HF =
æ ö 2R cos A cos B.
2abc


çè (a + b)(b + c)(c + a) ÷ø D ( For proof see Chapter 6) (iii) H

divides AD in the ratio cos A : cos B

C
cos
BE in the ratio cos B : cos C cos A and CF in
the ratio cos C : cos A cos B.
Exercises 289

4.16 The points of intersection of the altitudes with the 4.20 In DABC
circumcircle are reflections of the orthocentre with A
respect to the corresponding sides. (i) AI = r cosec
2
4.17 The angles and sides of the pedal triangle: DEF
F is B
BI = r cosec
the pedal triangle of DABC. Then 2
(i) D = 180° - 2 A, E = 180° - 2 B, F = 180° - 2C . C
CI = r cosec
(ii) EF = a cos A, DF = b cos B, DE = c cos C. 2
R A
(iii) (a) The circumradius of DDEF F is . (ii) AI1 = r1 cosec
2 2
(b) The area of DDEF F is 2D cos A cos B cos C. B
BI2 = r2 cosec
(c) The inradius of the pedal triangle DEF F is 2
2R cos A cos B cos C. C
CI3 = r3 cosec
4.18 Excentric triangle: The triangle formed by the
2
three excentres I1, I2 and I3 of DABC
C is called H = R (1 - 8 cos A cos B cos C)
4.21 (i) OH
2 2

excentric triangle of DABC.


æ A B Cö
(ii) OI 2 = R2 ç 1 - 8 sin sin sin ÷ = R2 - 2Rr
4.19 Angles and sides of D ABC: è 2 2 2ø
(i) Note that I1A, I2B and I3C are altitudes of æ A B Cö
OI12 = R2 ç 1 + 8 sin cos cos ÷ = R2 + 2 Rr1
DII1I2I3 and hence DABC is the pedal triangle of è 2 2 2ø
DII1I2I3 and I is the orthocentre of DII1I2I3.
A B C QUICK LOOK
(ii) I1 = 90° - , I2 = 90° - and I3 = 90° - .
2 2 2
In DABC, R ³ 2rr and the equality holds if DABC
C is
A B C equilateral.
(iii) I2 I3 = 4 R cos , I3 I1 = 4 R cos , I1 I2 = 4 R cos .
2 2 2
abc
(iv) Area of DII1I2I3 is 2Rs = .
2r

EXERCISES
Single Correct Choice Type Questions
1. In DABC, a = 6, B = 45° and C = 30°. AD is drawn æ a2 - c2 ö
2
æ a2 + c2 ö
2

(A) ç (B) ç
è 2ac ÷ø è 2ac ÷ø
perpendicular to the side BC. A circle is described on
AD as diameter cuts the sides AB and AC C in P and
Q, respectively. Then the length of the segment PQ is æ a2 - c2 ö
2
æ a2 + c2 ö
2

(C) ç (D) ç
3 3 3 3 3 è ac ÷ø è ac ÷ø
(A) (B) (C) (D) 2 3
2 4 2
4. In DABC, the median AD is perpendicular to the side
2. In DABC, the value of AC
C at A. Then
A B C
(b + c)tan + (c + a)tan + (a + b)tan æ c2 - a2 ö
cos A cos C = k ç
è ca ÷ø
2 2 2
is
where the value of k is
(A) 2(R + r) (B) 4(R + r)
3 3 4 2
(C) 3(R + r) (D) 6(R + r) (A) (B) (C) (D)
2 4 3 3
3. In DABC, a = BC, b = CA, c = AB. If a2, b2, c2 are in
AP (in the given order), then 5. In DABC, if BC = 5, CA = 4 and cos(A - B) = 31/32,
then the area of the triangle is
sin 3B
= (A) 5 7 (B)
15
3 (C)
15
7 (D)
15
7
sin B 4 3 4
290 Chapter 4 Properties of Triangles

6. In DABC, if a = 6, b = 3 and cos(A - B) = 4/5, then the 12. In a circle of 1 unit radius, AB is a chord whose
third side c is equal to length is also 1. If C is any point on the major arc
of AB, then the maximum value of (AC)2 + (BC)2 is
(A) 3 5 (B) 6 (C) 2 5 (D) 6 5
(A) 2 + 3 (B) 2(2 + 3 )
7. In DABC, if cos A = l cos B, then
(C) 2( 3 + 1) (D) 4( 3 + 1)
æ A + Bö (Hint: BCA = 30° and let BAC = q.)
tan2 ç =
è 2 ÷ø
13. In DABC, if a = 2, b = 3 and sec2 A = 8/5, then one of
æ a + bö æ 1 + l ö æ a + bö æ 1 - l ö the values of the third side c is
(A) ç (B) ç
è a - b ÷ø çè 1 - l ÷ø è a - b ÷ø çè 1 + l ÷ø
(A) 10 (B) 6 (C) 5 (D) 6
æ a - bö æ 1 + l ö æ a - bö æ 1 - l ö
(C) ç (D) ç 14. In DABC, if p1, p2 and p3 are the lengths of the
è a + b ÷ø çè 1 - l ÷ø è a + b ÷ø çè 1 + l ÷ø
altitudes, then 2D2 is equal to
8. In DABC, it is given that (A) Rp1 p2 p3 (B) R2p1 p2 p3
3 2
3 (C) 3R2p1 p2 p3 (D) R p1 p2 p3
cos A + cos B + cos C = 2
2
where R is the circumradius.
If one side of the triangle is of length 4 units, the area
of its pedal triangle is 15. In DABC, if cot A + cot B + cot C = 3, then the sides
are in the ratio
3 3
(A) (B) (C) 2 3 (D) 3 (A) 1: 2 :3 (B) 1: 1: 2
4 2
(Hint: See Theorem 4.23.) (C) 1: 2 : 1 (D) 1:1:1

9. In DABC, a = 5, b = 4, c = 3 and its centroid and 16. In DABC, if the distances of the vertices from the
circumcentre are, respectively, G and O. Then the incentre are x, y and z and r is its inradius, then r2 is
magnitude of OG is equal to
5 6 2 3 x+ y+z 1 1 1
(A) (B) (C) (D) (A) (B) + +
6 5 3 2 xyz x y z
æ 1 2 2 ö xyz x2 y2 z2
çè Hint: OG = R - (a + b + c )÷ø
2 2 2
(C) (D)
9 4R x+ y+z

10. In DABC, if 17. In DABC, if the altitude, the angle bisector and the
median through A, divide angle A into four equal
cos A cos B + sin A sin B sin C = 1
parts, then angle A is equal to
then (A) 120° (B) 90° (C) 115° (D) 75°
sin A + sin B + sin C =
2 2 2
18. In DABC, r1 + r2 + r3 = r + kR where k is
3 3
(A) 2 2 (B) (C) 2 (D) (A) 2 (B) 4 (C) 6 (D) 8
2 2
19. If a, b and g are the distances of the vertices from its
11. In DABC, if orthocentre, then
a cos A + b cos B + c cos C a + b + c a b c
= + + =
a sin B + b sin C + c sin A 9R a b g
and I1, I2 and I3 are the excentres of DABC, then the a+b+c abc
(A) (B)
area of DII1I2I3 is abg a +b +g
(A) 3R2 (B) 3 3R2 abc a+b+c
(C) (D)
æ ö abg a +b +g
(C) 3R2 (D) ç 3 3 ÷ R2
è 2 ø
20. In DABC, if p1, p2 and p3 are the lengths of the alti-
(Hint: Area of DII1I2I3 is 2Rs.) tudes, then D( p1-2 + p2-2 + p3-2 ) =
Exercises 291

(A) cos A + cos B + cos C mb2 + mc2 mb2 + mc2


(C) ma2 < (D) ma2 <
(B) sin A + sin B + sin C 4 5
(C) tan A + tan B + tan C (Hint: Use 4 ma2 = 2b2 + 2c2 - a2 , etc.)
(D) cot A + cot B + cot C
23. In DABC, if b2 + c2 = (2011)a2, then
21. If Q is a point on the altitude AD and is lying inside the
cot A
triangle such that CBQ = B / 3, then the length AQ is =
cot B + cot C
B C 2011 2009
(A) 2c sin (B) 2c sin (A) (B) 1005 (C) 1006 (D)
3 3 2 2
B B
(C) 2a sin (D) 2b sin
3 3 24. In DABC
D , if a2 + b2 + c2 = ac + bc 3 , then the triangle is
(A) right angled (B) isosceles
22. In DABC, Let ma, mb and mc be the lengths of the
medians through the vertices A, B and C, respec- (C) equilateral (D) obtuse angled
tively. Then angle A is obtuse if and only if
25. In DABC
D , if (rr2 - r1)(rr3 - r1) = 2rr2r3, then the triangle is
m2 + mc2 m2 + mc2
(A) m < b2
a (B) m < b
2
a
(A) equilateral (B) isosceles
2 3
(C) right angled (D) obtuse angled

Multiple Correct Choice Type Questions


1. In DABC, if a2 + b2 + c2 = bc + (ac) 3 , then A B B C C A
(C) tan tan + tan tan + tan tan = 1
(A) A = 60° (B) B = 90° 2 2 2 2 2 2
A B C
2ab (D) tan2 + tan2 + tan2 ³ 1
(C) C = 90° (D) inradius is 2 2 2
c(3 + 3 )
5. If the lengths of the tangents drawn from the vertices
2. In DABC, if A, B and C of DABC
C to its incircle are in AP, then
(a + b) = ab + c
2 2
4c - 3b
(A) a, b, c are in AP (B) cos A =
2c
and sin A + cos A = 3 / 2
(C) r1, r2, r3 are in AP (D) r1, r2, r3 are in HP
then
6. In DABC, D, E and F are the feet of the altitudes
(A) C = 120° (B) A = 15° drawn from the vertices A, B and C onto the opposite
(C) B = 45° (D) a : b : c = ( 3 - 1): 2 : 6 sides. Let R1, R2 and R3 be the circumradii of DAEF,
F
DBDF F and DDCE, respectively. Then
3. In any DABC
D , which of the following statements are true? (A) 2R1 = a cot A
A B C 1 (B) 2R2 = b cot B
(A) sin sin sin £
2 2 2 8
(C) 2R3 = c cot C
A B C 3
(B) sin2 + sin2 + sin2 ³ (D) The ratio of the perimeter of DABC
C to the peri-
2 2 2 4
meter of DDEF F is r : R
A B C 9
(C) cos2 + cos2 + cos2 £ (Hint: See Theorems 4.22 and 4.23.)
2 2 2 4
A B C 3 3 7. In DABC, which of the following statements are true?
(D) cos cos cos £
2 2 2 8 A B C
(A) IA × IB × IC = (abc)tan tan tan
2 2 2
4. Let ABC
C be a triangle. Then
A B C
A a (B) AI1 × BI2 × CI3 = (r1r2 r3 )cosec cosec cosec
(A) sin £ 2 2 2
2 b+c
(C) I I1 × I I2 × I I3 = 16rR2
(B) If sin A + sin B + sin C £ 1, then min{B + C, C + A,
A + B} < 30° (D) (I I1)2 + (II2I3)2 = 16R2
292 Chapter 4 Properties of Triangles

8. In any DABC 10. In DABC, let


C C
(A) (r1 + r2 )tan =c (B) (r3 - r )cot =c a b c
2 2 cos a = , cos b = and cos g =
b+c c+a a+b
C
(C) (r3 - r )tan = c (D) r2r3 + r3r1 + r1r2 = D2 where each of a, b and g is positive and less than p.
p
2
Then
9. Two sides of a triangle are 12 and 12 2 and the angle a b g
opposite to the shorter side is 30°. Then (A) tan2 + tan2 + tan2 = 1
2 2 2
(A) the number of such triangles is two a b g
(B) the angles of the triangles are 30°, 45°, 105° or (B) tan tan tan = tan A tan B tan C
2 2 2
30°, 135°, 15°
a b g A B C
3+1 3-1 (C) tan tan tan = tan tan tan
(C) area of the triangle is or 2 2 2 2 2 2
2 2 2 2 (D) tan a tan b tan g = tan A tan B tan C
(D) areas are in the ratio ( 3 + 1):( 3 - 1)

Matrix-Match Type Questions


In each of the following questions, statements are given 2. For any DABC, match the items of Column I with
in two columns, which have to be matched. The state- those of Column II.
ments in Column I are labeled as (A), (B), (C) and
(D), while those in Column III are labeled as (p), (q), Column I Column II
(r), (s) and (t). Any given statement in Column I can
have correct matching with one or more statements in A B C
Column II. The appropriate bubbles corresponding to (A) rr1r2r3 is equal to (p) s3 tan tan tan
2 2 2
the answers to these questions have to be darkened as rr1 A
illustrated in the following example. (B) equals (q) tan2
r2 r3 2
Example: If the correct matches are (A) ® (p), (s); (C) r1r2r3 is equal to (r) D 2

(B) ® (q), (s), (t); (C) ® (r); (D) ® (r), (t); that is if the
r1r2 r3 A
matches are (A) ® (p) and (s); (B) ® (q), (s) and (t); (D) ( s - a)( s - b)( s - c) = (s) tan3
(C) ® (r); and (D) ® (r), (t) then the correct darkening D 2
of bubbles will look as follows:
3. Match the items of Column I with those of Column II.
p q r s t
A
Column I Column II
B
C (A) In DABC, if (p) 3
D
7
cos A + cos B + cos C =
4
1. In D ABC , A is least and its value is 45°. If tan A, tan B 4
(q)
and tan C are in AP and the area of the triangle is then the value of R/rr is 3
27 square units, then match the items of Column I (B) In DABC, if AC = 2AB
2 , then
with those of Column II.
A B-C (r) 2 3
cot cot =
Column I Column II 2 2
(C) In a right-angled triangle, the
(A) The side a is of length (p) 6 2 hypotenuse is four times the length of 2
(q) 3 2 the altitude drawn from the opposite (s) 3
(B) The side b is equal to vertex and 2qq is the difference of the
3 10
(r) other two acute angles. Then tan2 2q =
(C) The side c equals 2
(D) In DABC, if A = 30°, b = 2, c = 3 (t) 3
(s) 3 5 and r is the inradius, then the value
(D) Circumradius of DABC
C is (t) 9 of 2r + 1 is
Exercises 293

4. In DABC, it is given that a = 5, b = 3 and c = 7. Match 5. In DABC, match the items of Column I with those of
the items of Column I with those of Column II. Column II.

Column I Column II Column I Column II


(A) Area of DABC
C is (p) 5 æ A + Bö æ A - Bö (p) b2
(A) cot ç
è 2 ÷ø
tan ç
è 2 ÷ø
is
(q)
3 a-b
(B) The circumradius is equal to (q)
2 1 - tan( A / 2)tan( B / 2) a+b
7 3 (B) is equal to c
(C) The inradius equals (r) 1 + tan( A / 2)tan( B / 2) (r)
3 a+b
(C) If the angles A, B, C (in this order) a2 + b2
15 3
(D) The value of 7 cos B + 3 cos C is (s) are in AP, then a2 + c2 - ac is equal to (s) a2 - c2
4
1 + cos( A - B)cos C a2 + b2
(D) equals (t) 2
1 + cos( A - C )cos B a + c2

Comprehension-Type Questions
1. Passage: D1, D2 and D3 are the areas of the triangles cut (iii) Minimum value of
off by the tangents drawn to the incircle of DABC C and
drawn parallel to the sides BC, CA and AB, respec- a2 + b2 b2 + c2 c2 + a2
+ +
tively. Answer the following three questions. a+b b+c c+a
D1 D2 D3
(i) = = = l , where l equals is
( s - a)2 ( s - b)2 ( s - c)2
(A) 4s (B) 3s (C) 2s (D) s
D D 2
D D2
(A) (B) (C) (D) 3. Passage: For any DABC, answer the following three
s2 s2 s s
questions.
(ii) If R1 is the circumradius of the triangle whose
area is D1, then R1 is equal to (i) If the sides a, b, c are in GP (in the given order),
then the three numbers
B C B C
(A) R cot cot (B) R tan tan æ tan B + tan C ö 2 æ tan C + tan A ö
2 2 2 2 (b2 - c2 ) ç , (c - a2 ) ç ,
÷
è tan B - tan C ø è tan C - tan A ÷ø
R R
(C) (D)
3 r æ tan A + tan B ö
(a2 - b2 ) ç
(iii) If R1, R2 and R3 are the circumradii of the trian- è tan A - tan B ÷ø
gles with areas D1, D2 and D3 then R1 + R2 + R3 is
are in
2R 3R 4R
(A) (B) R (C) (D) (A) AP (B) HP
3 2 3
(C) GP (D) increasing order
2. Passage: In DABC, let a, b, c be the sides, s the semi-
perimeter and D the area. Answer the following three (ii) If cos A + 2 cos B + cos C = 2, then
questions. 1 1 1
(A) , , are in HP (B) a, b, c are in GP
(i) Minimum value of a2 + b2 + c2 is a b c
(A) 3 2 D (B) 4 3D (C) 3 3D (D) 2 3D 1 1 1
(C) a, b, c are in HP (D) , , are in GP
a b c
1 1 1
(ii) Minimum value of + + is (iii) If sin A, sin B and sin C are in AP, then the alti-
a2 b2 c2
tudes of the triangle are in
s 2s 3s
(A) (B) (C) (D) 2Ds (A) AP (B) GP (C) HP (D) AGP
abc abc abc
294 Chapter 4 Properties of Triangles

4. Passage: In DABC, the incircle touches the sides BC, (C) right-angled isosceles
CA and AB at the points D, E and F F, respectively. If (D) The sides of DABC
C are in AP in some order
the radius of the incircle is 4 and the lengths of BD,
(ii) Area of DABCC is
CEE and AFF are consecutive integers, then answer the
following three questions. (A) 2 21 (B) 2 17 (C) 3 11 (D) 84
(i) DABC
C is (iii) In some order sin A, sin B and sin C are in
(A) equilateral (A) AP (B) GP (C) HP (D) AGP
(B) right-angled

Assertion–Reasoning Type Questions


In the following set of questions, a Statement I is given Statement II: In any DABC,
and a corresponding Statement II is given just below it.
3
Mark the correct answer as: cos A + cos B + cos C £
2
(A) Both Statements I and II are true and Statement II
is a correct explanation for Statement I where the equality holds if and only if the triangle is
(B) Both Statements I and II are true but Statement II is equilateral.
not a correct explanation for Statement I
(C) Statement I is true and Statement II is false 3. Statement I: In DABC , C = 90° and the bisector of the
angle C meets the side AB internally in D. If p = AD
(D) Statement I is false and Statement II is true and q = BD, then
1. Statement I: In DABC, the inradius r is less than p 1 - tan[( A - B)/ 2]
a2 + b2 + c2 =
q 1 + tan[( A - B)/ 2]
3(a + b + c)
Statement II: n a triangle, internal bisector of an angle
1 1 divides opposite side in the ratio of the other two sides.
Statement II: Area of DABC
D = bc sin A = ca sin B =
1 2 2
ab sin C
2 4. Statement I: The sides a, b, c of DABC
C are, respec-
tively, 18, 24 and 30. Then its inradius is 9.
2. Statement I: In an acute-angled DABC, if Statement II: The inradius r = D /s, where D is the
area and s is the semiperimeter of the triangle.
8(cos A + cos B + cos C - 3 cos A cos B cos C )
3 3 3

+ 36(cos A cos B + cos B cos C + cos C cos A) = 27 5. Statement I: In DABC, ‘I’ and ‘O’ denote the
incentre and circumcentre, respectively. If BIO = 90°,
DABC
C is equilateral. then a + c = 2b.
Statement II: In DABC, (IO)2 = R2 - Rr.

Integer Answer Type Questions


The answer to each of the questions in this section is a X Y Z W
non-negative integer. The appropriate bubbles below 0 0 0 0
the respective question numbers have to be darkened. 1 1 1 1
For example, as shown in the figure, if the correct answer
2 2 2
to the question number Y is 246, then the bubbles under
Y labeled as 2, 4, 6 are to be darkened. 3 3 3 3
4 4 4
5 5 5 5
6 6 6
7 7 7 7
8 8 8 8
9 9 9 9
Answers 295

1. In DABC, if a = 18, b = 24 and c = 30, then the value 7. If the line joining the incentre and the circumcentre
of cosec C is . of triangle ABCC is parallel to the side BC, then the
value of cos B + cos C is .
2. In DABC, if a = 25, b = 52 and c = 63, then the value of
5 tan(A/2) + 2 tan(B/2) is equal to . 8. In DABC, the value of

3. In DABC, if a = 6, b = 3 and cos(A - B) = 4/5, then its


a cos A + b cos B + c cos C r
=
area is . a+b+c kR
where k is equal to .
( I2 I3 )2
4. In D ABC , is equal to .
R(r2 + r3 ) 9. In DABC, the minimum value of

a b c
5. The circumradius of DABC
C is times the cir- + +
cumradius of its pedal triangle. s-a s-b s-c
.
6. In DABC, the distances of the circumcentre from
the sides BC, CA and AB are, respectively, p1, p2 and 10. If H is the orthocentre of DABC
C whose circumradius
p3. Then abc/p
/ 1 p2 p3 is / 1) + (b/p
times (a/p / 2) + is 4, then the circumradius of DBHC
C is .
(c/p
/ 3).

ANSWERS
Single Correct Choice Type Questions
1. (C) 14. (A)
2. (B) 15. (D)
3. (A) 16. (C)
4. (D) 17. (B)
5. (D) 18. (B)
6. (A) 19. (C)
7. (B) 20. (D)
8. (D) 21. (A)
9. (A) 22. (D)
10. (C) 23. (B)
11. (B) 24. (A)
12. (B) 25. (C)
13. (A)

Multiple Correct Choice Type Questions


1. (A), (C), (D) 6. (A), (B), (C)
2. (A), (B), (C), (D) 7. (A), (B), (C), (D)
3. (A), (B), (C), (D) 8. (A), (B), (D)
4. (A), (B), (C), (D) 9. (A), (B), (D)
5. (A), (B), (D) 10. (A), (C)

Matrix-Match Type Questions


1. (A) ® (s), (B) ® (p), (C) ® (t), (D) ® (r) 4. (A) ® (s), (B) ® (r), (C) ® (q), (D) ® (p)
2. (A) ® (r), (B) ® (q), (C) ® (p), (D) ® (r) 5. (A) ® (q), (B) ® (r), (C) ® (p), (D) ® (t)
3. (A) ® (q), (B) ® (t), (C) ® (t), (D) ® (p)
296 Chapter 4 Properties of Triangles

Comprehension-Type Questions
1. (i) (A); (ii) (B); (iii) (B) 3. (i) (C); (ii) (A); (iii) (C)
2. (i) (B); (ii) (B); (iii) (C) 4. (i) (D); (ii) (D); (iii) (A)

Assertion–Reasoning Type Questions


1. (A) 4. (D)
2. (A) 5. (C)
3. (A)

Integer Answer Type Questions


1. 1 6. 4
2. 2 7. 1
3. 9 8. 1
4. 4 9. 6
5. 2 10. 4
Addition and Scalar
Multiplication of Vectors 5
Contents
5.1 Definition and
Classification of
Vectors
5.2 Addition of Vectors
5.3 Multiplication of a
Vector by a Scalar
5.4 The Division Formula
5.5 Components of a
Vector
5.6 Vector Equation of a
Multiplication of Vectors

Line and a Plane


V V+W
Worked-Out Problems
W
W Summary
Exercises
Addition and Scalar

Answers

V V + 2·W
In the 19th century, the
famous mathematical physi-
2·W cist R. Hamilton (1805–1865)
introduced the notion of a
vector and his studies on
vectors brought out their
importance. In recent years,
the algebra of vectors has
become an indispensable
tool in the study of Applied
Mathematics, Physics and
Engineering.
298 Chapter 5 Addition and Scalar Multiplication of Vectors

In this chapter, we define the concept of vectors, their addition and two types of multiplication. Also we derive
various properties of addition and multiplication of vectors and obtain the vector equations of straight lines and
planes.

5.1 | Definition and Classification of Vectors


Physical quantities such as length, area, volume, mass, temperature, density, etc. possess only magnitude and are not
related to any definite direction in space. Such quantities are known as scalar quantities or simply scalars. On the
other hand physical quantities such as displacement, velocity, acceleration, force momentum, couple, electric current,
magnetic and gravitational intensities, etc. possess magnitude and are associated with direction in space. Such quanti-
ties are known as vector quantities or simply vectors. In this section we provide concrete mathematical definitions for
these intuitive ideas of scalars and vectors.
The three-dimensional space is simply called the space and the points in the space are associated with ordered
triads of real numbers, with respect to a given rectangular coordinate system. We first introduce the concept of a
directed line segment.

DEFINITION 5.1

A and B be two distinct points in the space. Then the ordered pair (A, B) is denoted by
Let
AB and is called

the directed line segment


t with initial point A and terminal point B. The


straight line AB is called the supportt of AB.

A B
Line segment

A B

A B

FIGURE 5.1 Directed line segment.



A directed line segment
AB

is represented by

line segment AB with an arrow mark pointing towards B as shown


the
in Figure 5.1. Note that AB is different from BA, though their supports are same.


DEFINITION 5.2

AB be a directed line
Let

segment. The distance between A and B is called the magnitude of


AB and is denoted by | AB | or simply AB.

A B

C D

(a)

A B

D C

(b)

FIGURE 5.2 Directed line segments in (a) same direction; (b) opposite direction.

Note that the line segment and the length of that line segment are denoted in the same

way,
namely,

AB. There is
no ambiguity and one should understand as per the context. Two directed line segments AB and CD are said to have
5.1 Definition and Classification of Vectors 299


same direction

if their supports AB and

CD are

parallel and both B and D lie in the same half-plane

determined


by the line AC [see Figure 5.2(a)]. If AB and

CD are parallel

but do not have the same direction, then AB and CD


are said to have opposite direction. Note that AB and BA have opposite direction [see Figure 5.2(b)].

DEFINITION 5.3 Equivalence Let DLS be the set of all directed line segments in the space. Two directed line
segments are said to be equivalentt if they have the same magnitude and the same direction.

The relation “being equivalent” is an equivalence relation on DLS (i.e., a binary relation which is reflexive on DLS,
symmetric and transitive) and hence partitions DLS into pair-wise disjoint classes (which are called equivalence
classes) satisfying the following properties
1. Any two directed line segments in the same equivalence class are equivalent (i.e., they have same magnitude as
well as direction).
2. Every directed line segment belongs to exactly one equivalence class.
3. Directed line segments belonging to different equivalence classes are not equivalent.

DEFINITION 5.4 Vector Each equivalence class discussed above is called a vectorr and is usually denoted by

a, b, c , etc.


If AB is a directed line

in the equivalence class a, then we simply write AB = a. Note that AB = CD if and


segment

= CD and AB and CD have the same direction.


only if
AB


If AB belongs to the vector a,
then

we say that AB represents the


vector

in this case the magnitude of a


and

is defined as the magnitude AB of AB and is denoted by | a | . Note that AB and CD represent the same vector if and



only if they have same direction

as well as magnitude.
Two vectors a and b are said to have same direction if any AB

representing a and any CD representing b have same direction.


DEFINITION 5.5 Zero Vector For any two points

P and Q in the space, clearly PP and QQ are equivalent


and hence the collection of PP where P is a point in the space is a vector (equivalence
class
of directed line segments), which is called the zero vectorr and is denoted by 0.


DEFINITION 5.6 Support For any non-zero vector a, a supportt of a is defined to be the support of any AB

representing a.

The zero vector has neither support nor direction; however, it has magnitude, namely 0, that is, | 0 | = 0. Note that
1. Any two supports of a non-zero vector are parallel in the space.
2. Any vector is uniquely determined by its direction and magnitude but not its support.

DEFINITION 5.7 Unit Vector A vector of magnitude one unit is called a unit vector. If a is a non-zero vector

and e is a unit vector having the same direction as a, then e is called a unit vector in the direc-

tion of a (Figure 5.3).

FIGURE 5.3 Unit vector.

DEFINITION 5.8 Collinear, Like and Unlike Vectors Two or more vectors are said to be collinearr if they have
parallel or same supports. Two collinear vectors are said to be like or unlike vectors according
as they have the same direction or opposite direction.

In Figure 5.4(a) a and b are like vectors and in Figure 5.4(b) a and b are unlike vector as they are collinear and have
opposite direction.
300 Chapter 5 Addition and Scalar Multiplication of Vectors

(a)

FIGURE 5.4 (a) Like vectors; (b) Unlike vectors.

DEFINITION 5.9 Coplanar and Non-coplanar Vectors A set of vectors is said to be coplanarr if their supports
are in the same plane or parallel to the same plane. Vectors which are not coplanar are called
non-coplanar.


Note that any two vectors are always coplanar. Further, three vectors a = OA, b = OB and c = OC are coplanar if and
only if the points O, A, B and C are all in the same plane.

5.2 | Addition of Vectors


In this section we define the notion of addition of two vectors and derive various properties of vectors involving addi-
tion. Before going for the definition of addition, we shall first have the following theorem.


T H E O R E M 5.1 Let a and b be two vectors and O, A, B, O1, A1 and B1 be points such that


OA = a = O1 A1 and AB = b = A1 B1


Then OB = O1 B1


PROOF Since OA = O1 A1 and AB = A1 B1 , we have

OA = O1 A1 and AB = A1 B1

O
A B1

O1
A1

FIGURE 5.5 Theorem 5.1.


5.2 Addition of Vectors 301

Also, the lines OA and O1A1 are parallel and so are the lines AB and A1B1. Therefore OO1 A1A
and AA1 B1B are parallelograms (Figure 5.5) and

OO1 = AA1 = BB1

Thus OO1B1B is a parallelogram, so that OB = O1B1 and hence



OB = O1 B1 ■

Note: a and b are collinear if and only if O, A, B and O1, A1, B1 are collinear, and, in this case, O, A, B, O1, A1 and
B1 lie on the same line.


DEFINITION 5.10 Resultant

Vector Let a and b be vectors and O, A and

B
be points such that OA = a and

AB = b. Then the sum a + b is defined to be the vector OB. a + b is also called the resultant

vectorr of a and b.

DEFINITION 5.11 Triangle Law for Addition The equation



OA + AB = OB
is known as the triangle law for addition (Figure 5.6).


DEFINITION 5.12 Parallelogram Law for Addition If a = OA and b = OC are co-initial vectors, then
complete

the
triangle

in Figure 5.6 to form the parallelogram OABC


C (Figure 5.7) to get that
AB = b and OB = a + b and, for this reason, the equation


OA + OC = OB
is known as the parallelogram law for addition.

Note: In view of the Theorem 5.1, the sum a + b does not depend on the initial points or final points of a or b.

b
a+
b

O A

FIGURE 5.6 Triangle law for addition.

C a B

b
a+
b b

O A

FIGURE 5.7 Parallelogram law for addition.


302 Chapter 5 Addition and Scalar Multiplication of Vectors


T H E O R E M 5.2 For any vectors a and b,
(C O M M U T A T I V E
PROPERTY FOR a+b=b+a
ADDITION)


PROOF Choose points A, B and C such that AB = a and BC = b (Figure 5.8).

D a C

b b
a+b

A B

FIGURE 5.8 Theorem 5.2.

Then, by Definition 5.11, we have



a + b = AB + BC = AC (5.1)
Now, draw parallels to AB and BC
C through C and A, respectively, and let them intersect at D.
Now, ABCD is a parallelogram. Also,


a = AB = DC


and b = BC = AD
Then, by the triangle law for addition, we have


b + a = AD + DC = AC (5.2)

From Eqs. (5.1) and (5.2), it follows that a + b = b + a. ■


The addition of vectors is defined to be a binary operation, in the sense that given two vectors a and b, we have

defined the vector a + b and proved in Theorem 5.2 that a + b = b + a. This property is called the commutativity of
addition.

If a, b and c are three given vectors, the expression a + b + c has no meaning right g now, since + is defined
only

between two vectors. However, (a + b) + c is meaningful, since it is the sum of a + b and c. Similarly a + (b + c ) is

since it is the sum of a and b + c . In the following, we prove that both the expressions (a + b) + c and
meaningful,

a + (b + c ) convey the same meaning and they represent the same vector. This property is called the associativity of
addition.


T H E O R E M 5.3 For any vectors a, b and c,
(A S S O C I A T I V E
PROPERTY FOR (a + b ) + c = a + (b + c )
ADDITION)


PROOF A, B, C and D such that AB = a, BC = b and CD = c . Then, by the triangle law
(Figure 5.9),
5.2 Addition of Vectors 303

D c C

b
b
a+

A B

FIGURE 5.9 Theorem 5.3.


(a + b) + c = ( AB + BC ) + CD


= AC + CD


= AD


= AB + BD


= AB + ( BC + CD)

= a + (b + c ) ■


Note: In view of the above result, we simply write a + b + c for (a + b) + c or a + (b + c ).


T H E O R E M 5.4 1. There is a unique vector 0 such that a + 0 = a for all vectors a.

2. For each vector a, there is a unique vector b such that a + b = 0.


PROOF Recall that for any points A and B, the vectors AA and BB are equal and are denoted by 0.


1. For any vector a = AB, we have


a + 0 = AB + BB = AB = a

Also, if z is any vector such that a + z = a for all vectors a, then

z=z+0=0+z=0

Thus 0 is the unique vector such that a + o = a for all vectors a.


2. Let a = AB be any given vector. Put b = BA. Then


a + b = AB + BA = AA = 0

If c is any vector such that a + c = 0, then

c=c+0

= c + (a + b )

= (c + a ) + b

=0+b=b


Thus b (= BA) is the unique vector such that a + b = 0. ■
304 Chapter 5 Addition and Scalar Multiplication of Vectors


DEFINITION 5.13 The unique vector 0 such that a + 0 = a for all vectors a is called the additive identity or zero
vector.

DEFINITION 5.14 For any vector a, the unique vector b such that a + b = 0 is called the additive inverse of a

and is denoted by - a.


Note that, if a = AB, then - a = BA.


T H E O R E M 5.5 The following hold for any vector a, b and c.

1. a + b = 0 Û b = - a 4. -(- a ) = a

2. a + b = a + c Û b = c 5. -(a + b) = (- a ) + (- b)

3. a + b = a Û b = 0 6. a + b = c Þ a = c + (- b)

PROOF 1. We have a + b = 0 which implies

b=0+b

= [a + (- a )] + b

= - a + (a + b )

= -a + 0 = -a
The other implication is trivial.

2. We have a + b = a + c which implies

- a + ( a + b ) = - a + (a + c )

(- a + a ) + b = (- a + a ) + c

0+b=0+c

b=c
3. We have

a+b=a

Þa+b=a+0

Þ b = 0 [by (2)]

Since - a + a = 0, it follows from (1) that a = -(- a ).
5. Since

a + b + (- a ) + (- b) = [a + (- a )] + [b + (- b)]

=0+0=0

it follows again from (1) that (- a ) + (- b) = -(a + b).

6. If a + b = c , then

c + (- b) = a + b + (- b) = a + 0 = a ■

Note: For any vectors a and b, for simplicity we write a - b for a + (- b).
5.2 Addition of Vectors 305


DEFINITION 5.15 Let “O” be a fixed point. If A is any point in the space, than the vector OA is called the posi-
tion vectorr of the point A with reference to the origin O.


T H E O R E M 5.6 Let a and b be

the
position
vectors of the points A and B respectively with reference to the
origin O. Then AB = b - a.

b a
b

O A

FIGURE 5.10 Theorem 5.6.


PROOF We are given that OA = a and OB = b. By the triangle law (Figure 5.10), we have OA + AB = b.
Therefore, by part (6) of Theorem 5.5, we have


AB = b - OA = b - a ■


Note: For any three points A, B, C , AB + BC = AC and hence BC = AC - AB


T H E O R E M 5.7 Let a and b be any vectors. Then

|a + b| £ |a | + |b|

Also, a and b are like vectors if and only if

|a + b| = |a | + |b|


PROOF Choose points A, B and C such that a = AB and b = BC . We shall distinguish three cases.

Case I: Suppose that a and b are not collinear. Then A, B and C are not collinear and, in the
triangle ABC,
AC < AB + BC
(i.e., any side of a triangle is less than the sum of the other two sides, Figure 5.11).

a+b b

A B

FIGURE 5.11 Case I.


306 Chapter 5 Addition and Scalar Multiplication of Vectors

Therefore, we have


| a + b | = | AC | = AC < AB + BC = | AB| + | BC | = | a | + | b |

Case II: Suppose that a and b are collinear and have opposite directions (Figure 5.12). Then B
does not lie between A and C. B is either to the left of A or to the right of C. Then


| a + b | = | AB + BC |


= | AC |
= AC

< AB + BC = | a | + | b |

B a A C

FIGURE 5.12 II.



Case III: Suppose that a and b are collinear and have the same direction (Figure 5.13), that is, a
and b are like vectors. Then B lies between A and C. Then


| a + b | = | AB + BC |


= | AC |
= AC
= AB + BC

= |a | + |b|

Therefore, if a and b are not like vectors [Case (I) or (II)], then

|a + b| < |a | + |b|

and, if a and b are like vectors [Case (III)], then

|a + b| = |a | + |b|

FIGURE 5.13 Case III. ■



Note: The inequality | a + b | £ | a | + | b | is called the triangular inequality.


C O R O L L A R Y 5.1 For any vectors a and b,

|a | - |b| £ |a - b|

PROOF By the triangular inequality, we have



|a | = |a - b + b| £ |a - b| + |b|
5.2 Addition of Vectors 307


and hence | a | - | b | £ | a - b |. Also,

|b| = |a + b - a | £ |a | + |b - a |

and hence

| b | - | a | £ | b - a | = | -(a - b)| = | a - b |

Thus | a | - | b | = ± (| a | - | b |) £ | a - b | ■

DEFINITION 5.16 For any vector a and for any integer n, we define the vector na by

ì0 if n = 0
ïï
na = í(n - 1)a + a if n > 0
ï
ïî-(- n)a if n < 0

QUICK LOOK 1

In particular, (-3)a = -(3a ) = - a - a - a

1a = a (-2)a = - a - a

2a = a + a (-1)a = - a

3a = a + a + a


C O R O L L A R Y 5.2 For any integer n and for any vector a,

| na | = | n || a |

PROOF This can be proved from the equalities



|a + a | = |a | + |a |

and | -a | = |a |
and by using induction on n. ■

Example


Let ABCD be a parallelogram (Figure 5.14). Then (3) AC + BD = AB + BC + BC + CD


(1) AB = DC = AB + 2 BC - DC = 2 BC (since DC = AB)


(2) AD = BC


(4) AC - BD = AB + BC - ( BC + CD)


D C = AB + DC = 2AB

FIGURE 5.14
308 Chapter 5 Addition and Scalar Multiplication of Vectors

Example


Let a + 2b, 2a - b, a and 3a + b be the position vectors DA = OA - OD
of the points A, B, C and D, respectively, relative to a
point O. Then = (a + 2b) - (3a + b) = - 2a + b

AB = OB - OA
OA = a + 2b

= (2a - b) - (a + 2b) = a - 3b
OB = 2a - b

BC = OC - OB
OC = a

= a - (2a - b) = - a + b
OD = 3a + b
Therefore


AC = OC - OA (see Theorem 5.6)

= a - (a + 2 b ) = - 2 b

Example 5.1

Let ABCC be a triangle in which P, Q and R are the mid- Solution: Since P, Q and R are the mid-points of AB,
points of AB, BC C and CA, respectively (Figure 5.15). BCC and CA we have
Then, for any point O, prove that

OA + OB + OC = OP + PA + OQ + QB + OR + RC
OA + OB + OC = OP + OQ + OR


= OP + OQ + OR + PA + RC + QB
C


= OP + OQ + OR + ( BA + AC + CB)
2


R Q = OP + OQ + OR


(∵ BA + AC + CB = 0)

A B

FIGURE 5.15 5.1.

Example 5.2

Let S be the circumcentre and O the orthocentre of a Solution: Let D be the mid-point of BC. Then
triangle ABC (Figure 5.16). Then prove that

DB + DC = - DC + DC = 0
(1) OA + OB + OC = 2OS

Also, we know that (Theorem 4.22, Chapter 4)


(2) SA + SB + SC = SO
AO = 2Rcos A
A
and from DBDS
SD
cos A = cos BSD =
O R
S Hence


C 2SD = AO
D

FIGURE 5.16 5.2.


5.3 Multiplication of a Vector by a Scalar 309

(1) Now, (2) Again



OA + OB + OC = OA + (OD + DB) + (OD + DC ) SA + SB + SC = SA + SD + DB + SD + DC


= OA + 2OD + DB + DC = SA + 2SD + DB + DC


= 2 DS + 2OD (∵ DB + DC = 0) = SA + AO + 0


= 2(OD + DS) = SO


= 2OS (See the Note under “Single Correct Choice Type
Question 3” in Worked-Out Problems.)

5.3 | Multiplication of a Vector by a Scalar


Multiplication of a vector by a positive integer is only a repeated addition. In Definition 5.16, we have defined the

product na for any integer n and a vector a. In this section we extend multiplication of vectors by integers to that by
any real number.

DEFINITION 5.17 Let r be any real number and a any vector.

1. r a is defined to be 0 if r = 0 or a = 0.

2. If r > 0 and a ¹ 0, then r a is defined to be the vector whose magnitude is r | a | and direc-

tion is same as that of a.

3. If r < 0 and a ¹ 0, then r a is defined as (-r )(-a ), which is the vector with the magnitude

(-r ) | a | and the direction same as -a, or equivalently, the direction is opposite to that of a.

Note that Definition 5.17 coincides with Definition 5.16 when r is an integer. The following is an immediate conse-
quence of Definition 5.17.


T H E O R E M 5.8 For any real number r and for any vector a,

(-r )a = -(ra ) = r(-a )

The following is a generalization of the above.


T H E O R E M 5.9 For any real numbers r and s and for any vector a,

r( sa ) = (rs)a = s(ra )

PROOF The result is trivial if r = 0 or s = 0 or a = 0. Suppose that r ¹ 0, s ¹ 0 and a ¹ 0. We have

| r( sa )| = | r || sa | = | r || s || a | = | rs || a | = |(rs)a |

Therefore, r( s a ) and (rs)a have same magnitude. If rs > 0, then (r > 0 and s > 0) or (r < 0 and

s < 0) and hence both of r( sa ) and (rs)a have the same direction as that of a. If rs < 0, then (r > 0

and s < 0 or r < 0 and s > 0) both of r( sa ) and (rs)a have the same direction which is opposite to

that of a.

Therefore, in all cases, r( sa ) and (rs)a have same direction. Thus r( sa ) = (rs)a. Since rs = sr,
we have

r( sa ) = (rs)a = ( sr )a = s(ra ) ■

Note: By taking s = -1 in Theorem 5.9, we get Theorem 5.8.


310 Chapter 5 Addition and Scalar Multiplication of Vectors


T H E O R E M 5.10 For any real numbers r and s and for any vector a,

(r + s)a = ra + sa

PROOF Without loss of generality, we can assume that r ¹ 0, s ¹ 0 and a ¹ 0. We shall distinguish the
following four cases.

Case I: Suppose that r > 0 and s > 0. Then r + s > 0 and hence the directions of ra, sa, ra + sa and

(r + s)a are all equal to the direction of a. That is, (r + s)a and ra + sa have the same direction.
Also, regarding their magnitudes, we have

|(r + s)a | = | r + s || a | = (r + s) | a | (since r + s > 0)

= r |a | + s|a |

= | r || a | + | s || a | (since r > 0 and s > 0)

= | ra | + | sa |

= | ra + sa | (by Theorem 5.7)

Thus (r + s)a and ra + sa have same direction and magnitude and hence they are equal.
Case II: Suppose that r < 0 and s < 0. Then -r > 0 and - s > 0. Now we have

(r + s)a = [-(-r - s)]a

= -[(- r - s)a ] (by Theorem 5.8)

= -[(- r )a + (- s)a ] [by Case (I) above)

= -[- (ra ) - ( sa )] (again by Theorem 5.8)

= ra + sa [by part (5) of Theorem 5.5)
Case III: Suppose that r > 0 and s < 0.
(A) If r + s > 0, then

ra = [(r + s) + (- s)]a

= (r + s)a + (- s)a [by Case (I) above]

= (r + s)a - sa (by Theorem 5.8)
and hence

(r + s)a = ra + sa
(B) If r + s > 0, then

sa = [(r + s) + (- r )]a

= (r + s)a + (- r )a [by Case (II) above]

= (r + s)a - ra (by Theorem 5.8)
and hence

(r + s)a = ra + sa
(C) If r + s = 0, then s = -rr and

ra + sa = ra + (- r )a

= ra - ra

= 0 = (r + s)a
Case IV: Suppose that r < 0 and s > 0. By interchanging r and s and using Case (III), we get that

(r + s)a = ( s + r )a = sa + ra = ra + sa ■
5.3 Multiplication of a Vector by a Scalar 311


T H E O R E M 5.11 For any vectors a and b and for any real number r,

r(a + b) = ra + rb

PROOF The result is trivial if r = 0 or a = 0 or b = 0. Therefore we can suppose that r ¹ 0, a ¹ 0 and b ¹ 0.


Case I: Suppose that r > 0. Choose points O, A and B such that OA = a and AB = b (Figure 5.17).
Then


a + b = OA + AB = OB

B1

b
rb

O ra A1 a A

B1

rb
b

ra
O a A A1

FIGURE 5.17 Theorem 5.11.



Choose a point A1 on the line OA such that OA1 = ra and then choose a point B1 that A1 B1 = rb. Then

OAB
the triangles 1 B1 are similar and B1 lies on the line OB. Since r > 0 and OB1 = r × OB, we
OA

and
get that OB1 = r ×OB = rb. Now


r(a + b) = r ×OB


= OB1


= OA1 + A1 B1

= ra + rb
Case II: Suppose that r < 0. Then - r > 0 and

r(a + b) = [-(- r )](a + b)

= -(- r )(a + b) (by Theorem 5.8)

= -[(- r )a + (- r )b] [by Case (I)]

= -(- ra - rb)

= ra + rb ■

Note: For any vector a and non-zero real number r, we shall write a / r for (1/ r )a.
312 Chapter 5 Addition and Scalar Multiplication of Vectors


DEFINITION 5.18 A vector a is said to be unit vectorr if its magnitude is the unit 1; that is | a | = 1.

If a is any non-zero vector, then

a 1
= |a | = 1
|a | |a |

and hence a /| a | is a unit vector for any a ¹ 0. a /| a | is called
the unit vector in the direction
of a.

For any non-zero vectors a and b, note that a and b are parallel if and only if b = ra, for some non-zero real
number r and, in this case,

|b|
r=±
|a |

depending on whether a and b are like vectors or opposite vectors. In other words, two non-zero vectors a and b are

parallel if and only if a a + bb = 0 for some non-zero real numbers a and b.

5.4 | The Division Formula



Given two points A and B whose position vectors with respect to a point O are a and b, we derive a formula for the

position vector of any point on the line joining A and B, in terms of the vectors a and b and the ratio in which the
point divides the line segment AB.


T H E O R E M 5.12 Let a and b be the position vectors

of two points A and B, respectively, with respect to a point O.


Then a point C lies on the line AB and divides the line segment AB in the ratio r : s if and only if
the position vector of C with respect to O is

rb + sa
r+s
that is

r ×OB + sOA
OC =
r+s


PROOF Let C be a point on the line AB, dividing the line segment AB in the ratio r : s.

b C

O A

FIGURE 5.18 I.
5.4 The Division Formula 313

Case I: Suppose that C divides AB internally (Figure 5.18). Then r ³ 0, s ³ 0 and r + s > 0.
We have, from Theorem 5.6,


AC = OC - OA


and CB = OB - OC
Since C divides AB in the ratio r : s, we have
AC :CB = r : s
that is
sAC = rCB
Therefore


sAC = rCB


s(OC - OA) = r(OB - OC )


rOB + sOA = sOC + rOC = ( s + r )OC

rOB + sOA rb + sa
OC = =
r+s r+s
Case II: Suppose that C divides AB externally (Figure 5.19) in the ratio r : s. Then


AC | r |
rs < 0 and

=
CB | s |

Therefore
| s | AC = | r | CB


sAC = rCB


Since AC ¹ CB, | r | ¹ | s | and hence r + s ¹ 0. Now, from sAC = rCB, we have


s(OC - OA) = r(OB - OC )


sOC + rOC = rOB + sOA

O A

FIGURE 5.19 Case II.


314 Chapter 5 Addition and Scalar Multiplication of Vectors


(r + s)OC = rOB + sOA

rOB + sOA rb + sa
OC = =
r+s r+s
Conversely, suppose that C is a point such that

rb + sa
OC =
r+s
Then


(r + s)OC = rb + sa = rOB + sOA

Therefore


r(OC - OB) = s(OA - OC )

CA r
rCB = sCA and =
CB s


Therefore CA and CB are parallel

vectors with a common initial point C. Hence A, B, C are


collinear. Thus, C lies on the line AB and C divides AB in the ratio r : s. ■


C O R O L L A R Y 5.3 Let A and B be points whose position vectors with respect to a point O are a and b. If C is the
mid-point of AB, then

a + b
OC =
2

C O R O L L A R Y 5.4 The position vectors of the two points dividing a line segment AB into three equal parts are


2OA + OB OA + 2OB
and
3 3
where O is the point of reference. These two points are called the trisecting points of AB.


T H E O R E M 5.13 Let A, B, and C be three distinct points whose position vectors with respect to a point O are a, b

and c, respectively. Then A, B and C are collinear if and only if there exist real numbers r, s and
t such that atleast one of them is non-zero,

r + s + t = 0 and ra + sb + tc = 0
PROOF Suppose that r, s and t are real numbers, not all zero, such that

r + s + t = 0 and ra + sb + tc = 0
Suppose that t ¹ 0. Then

tc = - ra - sb and t = - r - s
Therefore

- ra - sb ra + sb
OC = c = =
-r - s r+s


and hence, by Theorem 5.12, C lies on the line AB. Thus A, B and C are collinear.
5.4 The Division Formula 315

Conversely, suppose that A, B and C are collinear (Figure 5.20). Without loss of generality, we
can assume that B lies in between A and C.
Put AB = t and BC = r. Since A, B and C are distinct, t and r are positive real numbers and B
divides AC
C internally in the ratio t : r. Hence, by Theorem 5.12, we have

tc + ra
b = OB =
t+r

Now, put s = -t - r. Then, clearly r + s + t = 0 and ra + sb + tc = 0.

FIGURE 5.20 Theorem 5.13. ■


C O R O L L A R Y 5.5 Let O and A be distinct points. Then a point P lies on the line OA if and only if


OP = r ×OA
for some real number r.

Recall that the line joining a vertex of a triangle and the mid-point of the side opposite to it is called a median of the
triangle.

T H E O R E M 5.14 The medians of a triangle are concurrent and their point of concurrence divides each median in
the ratio 2 :1.
PROOF Let ABCC be a triangle and D, E and F be the mid-points of the sides BC, CA and AB, respectively.

Fix an arbitrary point O and let a, b and c be the position vectors of A, B and C, respectively,
with respect to O (Figure 5.21). Then, we have

OB + OC b + c
OD = =
2 2

OC + OA c + a
OE = =
2 2

OA + OB a + b
and OF = =
2 2
G be the point that divides AD internally in the ratio 2 :1. Then

2OD + OA b + c + a a + b + c
OG = = =
2+1 3 3

F E

FIGURE 5.21 Theorem 5.14.


316 Chapter 5 Addition and Scalar Multiplication of Vectors

Also

a + b + c
OG =
3

2[(c + a)/ 2] + b
=
2+1


2OE + OB
=
2+1
By Theorem 5.12, the point G lies on the median BE E and divides it in the ratio 2 :1. Again,

a + b + c
OG =
3

2[(a + b)/ 2] + c
=
2+1


2OF + OC
=
2+1
and hence G lies on the median CF F and divides it in the ratio 2 :1. Therefore, G is a common point
of all the three medians. Thus, the medians are concurrent and G is their point of concurrence.
Also, G divides each of the medians in the ratio 2 :1. ■

T H E O R E M 5.15 A point P lies on the plane determined by three non-collinear points O, A and B if and only if
there exists real numbers r and s such that


OP = rOA + sOB

and, in this case, the real numbers r and s are unique satisfying this equation.
PROOF Let O, A and B be three non-collinear points and P be the plane determined by them.

Suppose that
P is a point on P. Then OA, OB and OP P are coplanar. If P lies on the line OA, then there is a
real number r such that


OP = rOA = rOA + sOB


where s = 0. Similarly, if P lies on the line OB, then there is a real number s such that


OP = sOB = r × OA + sOB


where

r = 0. Suppose that P lies on

neither
OA

nor OB. Through P draw parallels to the lies OA


and OB. These parallels intersect OB and OA, say at C and D, respectively (since O, A and B are
not collinear). Then ODPC C is a parallelogram (Figure 5.22). Therefore, there exist real numbers
r and s such that


OD = rOA


and OC = sOB
Now, we have


OP = OD + DP = OD + OC = rOA + sOB
5.4 The Division Formula 317

C
P

FIGURE 5.22 Parallelogram ODPC.


Conversely,

that OP = rOA + s ×OB


suppose

real numbers r and s. If r = 0 or s = 0, then


for some
OP = sOB or rOA and hence P lies on

OB or OA

, so that P lies

on the P. Suppose

plane

that
r ¹ 0 and s ¹ 0. Choose points C on OB and D on OA such that OC = sOB and OD = rOA. Since
O, A and B are non-collinear, it follows that O, D and C are not collinear. Therefore we can form
a parallelogram ODQC C (Figure 5.23). Here, Q lies on P and


OQ = OD + DQ


= OD + OC


= rOA + sOB


= OP
and hence P = Q. Thus P lies on the plane P determined by O, A and B. The uniqueness of r and
s can be proved as follows. Suppose that r¢ and s¢ are real numbers such that


rOA + sOB = OP = r ¢× OA + s ¢× OB
Then


(r - r ¢)OA + ( s - s ¢)OB = O

C Q

FIGURE 5.23 Parallelogram ODQC.


318 Chapter 5 Addition and Scalar Multiplication of Vectors

If r ¹ r¢, then

s - s ¢


OA = ×OB
r - r¢
5.5, O, A and B are collinear, which is a contradiction to the hypothesis.
Therefore r - r¢ = 0 and hence r = r¢. Similarly, s = s¢. ■

T H E O R E M 5.16 A quadrilateral is a parallelogram if and only if its diagonals bisect each other.

PROOF Let ABCD a quadrilateral. Let b, c and d be the position vectors of B, C and D with respect to
A respectively; that is,


AB = b, AC = c and AD = d
Let P be the point of intersection of the diagonals ACC and BD (Figure 5.24). Suppose that the
diagonals bisect each other. Then,


AB + AD

AC
= AP =
2 2
That is

b+d c
=
2 2
Therefore

b = c - d and d = c - b


AB = AC - AD = DC and AD = AC - AB = BC
That is, the opposite sides of the quadrilateral are parallel and equal and hence ABCD is a
parallelogram.
Conversely, suppose that ABCD is a parallelogram. Then


b = AB = DC and d = AD = BC
If Q is the mid-point of BD, then

AB + AD AB + ( AC + CD)
AQ = =
2 2


AB + AC - DC AC


= = (∵ AB = DC )
2 2
Therefore Q is the mid-point of AC
C also. Thus BD and AC
C bisect each other.

D C

d
P

c
A B

FIGURE 5.24 Theorem 5.16. ■


5.4 The Division Formula 319


DEFINITION 5.19 Linear Combination Let a, a1 , a2 , …, an be any vectors. a is said to be a linear combination

of a1 , a2 , …, an if

a = r1a1 + r2 a2 +  + rn an

for some real numbers r1, r2, …, rn.

DEFINITION 5.20 Linearly Dependent and Independent Vectors Non-zero vectors a1 , a2 , … , an are said to
be linearly dependentt if one of them is a linear combination of the others. If they are not

linearly dependent, then we say that a1 , a2 , …, an are linearly independent.

The following is a very important tool in solving certain dependency problems.


T H E O R E M 5.17 Let a1 , a2 , …, an be non-zero vectors. Then a1 , a2 , …, an are linearly independent if and only if, for
any real numbers r1, r2, …, rn,

r1a1 + r2 a2 +  + rn an = 0 Þ r1 = r2 =  = rn = 0

PROOF a1 , a2 , …, an are linearly dependent. Then one of

a1 , a2 , …, an is a linear combination of the others. Without loss of generality, we can assume that

a1 is a linear combination of a2 , …, an . Therefore, there exist real numbers r2, r3, …, rn such that

a1 = r2 a2 + r3a3 +  + rn an

Put r1 = -1. Then



r1a1 + r2 a2 +  + rn an = 0 and r1 ¹ 0

and hence the required condition is not satisfied. Conversely, suppose that the required condition
is not satisfied. That is, there exist real numbers r1, r2, …, rn such that

r1a1 + r2 a2 +  + rn an = 0

and not all of r1, r2, …, rn are zero. Without loss of generality, we can assume that r1 ¹ 0. Then,
from the above equation, we get that

æ -r ö æ -r ö æ -r ö
a1 = ç 2 ÷ a2 + ç 3 ÷ a3 +  + ç n ÷ an
è r1 ø è r1 ø è r1 ø

and hence a1 is a linear combination of a2 , a3 , …, an . Thus a1 , a2 , …, an are linearly dependent. ■


T H E O R E M 5.18 Two non-zero vectors a and b are collinear if and only if a, b are linearly dependent.


PROOF that a = OA and b
=

OB
Let O, A and B be points such

a and b are non-zero, O ¹ A and


. Since

O ¹ B. Suppose

that a and b are collinear. Then OA and OB are collinear and hence B lies on
the line OA. Then, by Corollary 5.5, we get that


OB = r ×OA or b = ra

for some real number r and hence a, b are linearly dependent. Conversely, suppose that a, b are
linearly dependent. Then, we can assume, without loss of generality, that

a = sb

some real number s. Since OA = s OB, again by Corollary


for
5.5 we get that A lies on the line
OB and hence O, A and B are collinear and therefore a and b are collinear. ■
320 Chapter 5 Addition and Scalar Multiplication of Vectors


C O R O L L A R Y 5.6 The following are equivalent to each other for any non-zero vectors a and b.

1. a and b are non-collinear.

2. a and b are linearly independent.

3. For any real numbers r and s, ra + sb = 0 Þ r = 0 = s.

Example 5.3

Let ABCD be a parallelogram and E the mid point of


s × (1/ 2)b + d sb + 2d
AB. Prove that the point of intersection of DE E and AC = =
s+1 2( s + 1)
divide then in the same ratio and find the ratio.

Therefore
Solution:

Let AB = b and AD = d. Then DC = b and


BC = d. Now, b + d sb + 2d

=
r + 1 2( s + 1)
AC = AB + BC = b + d
Since b and d are not collinear, by Theorem 5.15, we get

1
and AE = AB = b s 1 2
2 2 = =
2( s + 1) r + 1 2( s + 1)
Let P be the point of intersection of DE E and AC
from which we get that r = 2 = s.
(Figure 5.25). Suppose that P divides AC C and DE
E in the
ratios r :1 and s :1,, respectively. Then
D b C

r × AA + 1× AC
AP =
r+1 r
s
d d
r × 0 + 1× (b + d ) b + d
= = P
r+1 r+1 1
1
Also

A E B

s × AE + 1× AD
AP = FIGURE 5.25 Example 5.3.
s+1


DEFINITION 5.21 Coplanar Vectors a1 ,
a

2 , …, an are said to be coplanar



if all of them lie in a single plane. In
other words, when ai = OAi , 1 £ i £ n, then a1 , a2 , …, an are said to coplanar if all the points O,
A1, A2, …, An lie on a single plane.

The following is a direct consequence of Theorem 5.15.


T H E O R E M 5.19 The following are equivalent to each other for any non-zero vectors a, b and c:

1. a, b, c are not coplanar.

2. a, b, c are linearly independent.
3. For any real numbers, r, s and t,

ra + sb + tc = 0 Þ r = s = t = 0
5.5 Components of a Vector 321

Example 5.4

Let a, b and c be non-coplanar vectors. Let A, B and C be and
points whose
position

vectors
with respect to the origin


O are a + 2b + 3c , - 2a + 3b + 5c and 7a - c , respectively. AC = OC - OA
Then prove that A, B and C are collinear.
= (7a - c ) - (a + 2b + 3c )

Solution: We are given that = 6a - 2b - 4c


OA = a + 2b + 3c = - 2(-3a + b + 2c )


OB = - 2a + 3b + 5c Therefore


and OC = 7a - c AC = 2 AB
Then


By Corollary 5.6, AC and AB are collinear and hence

A, B and C are collinear.


AB = OB - OA

= (- 2a + 3b + 5c ) - (a + 2b + 3c )

= - 3a + b + 2c

Example 5.5

Let a , b and c be non-coplanar vectors. Prove that Since a, b and c are non-coplanar, we have, from

a + 2b - c , 4a + b + 3c and 2a - 3b + 2c are non-coplanar. Theorem 5.19, that

Solution: It is enough if we prove that these are line- r + 4 s + 2t = 0


arly independent. Let r, s and t be any real numbers such 2r + s - 3t = 0
that
- r + 3s + 2t = 0

r(a + 2b - c ) + s(4a + b + 3c ) + t(2a - 3b + 2c ) = 0 Solving these, we get that r = 0 = s = t. Therefore the given
Then vectors are non-coplanar.


(r + 4 s + 2t )a + (2r + s - 3t )b + (- r + 3s + 2t )c = 0

5.5 | Components of a Vector



Given any three non-coplanar
vectors a, b and c, we shall prove in this section that any vector can be expressed as a

linear combination of a, b and c uniquely which facilitates us in representing any vector by a triad of real numbers.

In particular, when the given vectors a, b and c are pairwise perpendicular, we have several advantages of the repre-
sentation of vectors by triads of real numbers.


T H E O R E M 5.20 Let a, b and c be any non-coplanar vectors. For any vector x, there exists a unique triad (r, s, t)
of real numbers such that

x = ra + sb + tc

PROOF Let x be a given vector. Choose points O, A, B, C and P such that


OA = a, OB = b, OC = c and OP = x

If x = 0, then we can take r = s = t = 0 and these are unique since a, b and c are linearly indepen-

dent (Theorem 5.19). Therefore, we can suppose that x ¹ 0.
322 Chapter 5 Addition and Scalar Multiplication of Vectors


Since OA, OB and OC

are not coplanar, BOC , COA and AOB are three different planes.
If P lies on the plane AOB, then by Theorem 5.15, there exist real numbers r and s such that


OP = rOA + sOB
Therefore

x = ra + sb + 0 × c


Therefore we may assume that

P is not in the plane AOB. Draw a line through P parallel to OC


and it meet the plane AOB at Q (Figure

let

5.26). Then there exists a real number t such that



QP = tOC = tc . Since Q lies on the plane AOB, there exists real numbers r and s such that


OQ = rOA + sOB (by Theorem 5.15)
Now,


OP = OQ + QP = rOA + sOB + tOC
Therefore

x = ra + sb + tc

Since a, b and c are not coplanar, they are linearly independent and hence

ra + sb + tc = r1a + s1b + t1c

Þ (r - r1 )a + ( s - s1 )b + (t - t1 )c = 0
Þ r - r1 = s - s1 = t - t1 = 0

This implies r = r1, s = s1 and t = t1. Thus, there exist unique real numbers r, s and t such that

x = ra + sb + tc

O
b B
a Q
A

FIGURE 5.26 Theorem 5.20. ■

C O R O L L A R Y 5.7 Any four or more distinct vectors are linearly dependent.



PROOF Let A be any set of vectors having atleast four vectors. Choose distinct vectors a, b, c in A. If

a, b, c are linearly dependent, then so are all the vectors in A. Suppose that a, b, c are linearly

independent. Since there are atleast four vectors in A, we can choose x in A other than a, b, c .

Then, by Theorem 5.20, x is a linear combination of a, b and c and therefore the vectors in A are
linearly dependent. ■
5.5 Components of a Vector 323


Next, we shall define

the concept

of angle
b are two vectors, we can choose points
between two vectors. If a and

O, A and B such that OA = a and OB = b. Then the angle between a and b is defined to be the smaller angle AOB

and is denoted by (a, b). However, we have to prove that the angle between a and b does not depend on the choice
of O, A and B. This is proved in the following.

T H E O R E M 5.21 Let O, A, B, O1, A1 and B1 be points such that



OA = O1 A1 and OB = O1 B1

Let AOB and A1O1 B1 denote the smaller angles between OA and OB and between O1A1 and
O1B1, respectively. Then

AOB = A1O1 B1

b b a

O A

B1

b b a

FIGURE 5.27 Theorem 5.21.



PROOF Let a = OA = O1 A1 and b = OB = O1 B1 . We shall distinguish the following two cases. First notice
that both AOB and A1O1 B1 are in between 0 and p. p

Case I: Suppose that
a and b are parallel. Then O, A and B are collinear and O1, A1 and B1 are also
collinear. If a and b are like vectors, then A and B lie on the same side of O and hence AOB = 0°

and similarly A1O1 B1 = 0°. If a and b are opposite vectors, then A and B lie on different
f sides of O


on the line AOB and hence AOB = p and similarly A1O1 B1 = p.

Case II: Suppose that a and b are not parallel. Then O, A and B are not collinear and O 1, A 1


and B1 are also not collinear, as shown in Figure 5.27. Since OA = a = O1 A1 and OB = b = O1 B1 ,
we have
OA = O1A1 and OB = O1B1


Also, AB = b - a = A1 B1 , which implies AB = A1 B1 . Therefore, the triangles OAB and O1A1B1 are
congruent and hence AOB = A1O1 B1 . ■
324 Chapter 5 Addition and Scalar Multiplication of Vectors


DEFINITION 5.22 Let a and b be any vectors and O, A and B be points such that OA = a and OB = b. Then

the measure
of the angle AOB
between o and p is defined as the angle between a
which lies
and b and is denoted by (a, b). a and b are said to be perpendicular vectors if
p
(a , b ) =
2

By Theorem 5.21, (a, b) is independent
of the choice of the points O, A and B. Also, 0 £ (a, b) £ p . In Figure 5.28,

a and b are given such that (a, b) = 0 or acute or obtuse or p. p

O a A b B O a A
(a, b) = 0 0 < (a, b) < p
2

O a A B b O a A
p < (a, b) < p (a, b) = p

FIGURE 5.28

QUICK LOOK 2


1. (a, b) = (b, a ) = (- a, - b) = (- b, - a ) 4. (- ra, sb) = p - (a, b) = (ra, - sb) for all r, s > 0

2. (a, - b) = p - (a, b) = (- a, b) 5. (a, b) = 0 Û a and b are like vectors

3. (ra, sb) = (a, b) for all positive real numbers r and s 6. (a, b) = p Û a and b are opposite vectors


DEFINITION 5.23 If a , b and c are non-coplanar vectors, then the triad (a, b, c ) is called a basis vector triad. If

(a, b, c ) is a basis vector triad and x is any vector, then the unique real numbers r, s and t (as
in Theorem 5.20) satisfying

x = ra + sb + tc

are, respectively,
called a -component, b -componentt and c -componentt of x with respect to

the triad (a, b, c ).

Let (a, b, c ) be a basis vector triad and O, A, B, C be points in the space such that


OA = a, OB = b and OC = c

Since a, b and c are linearly independent, the points O, A and B are not collinear (see Theorem 5.18) and hence the

angle (a, b) < p . Also, in view of Theorem 5.15, c does not lie in the plane AOB. The basis vector triad (a, b, c ) is said
5.5 Components of a Vector 325

to be a right-handed system if, when observed from C, the angle of rotation from OA to OB in anticlockwise direction

does not exceed 180°. (a, b, c ) is said to be a left-handed system if it is not a right-handed system.

DEFINITION 5.24 A right-handed system (i , j , k ) of vectors is said to be orthogonall if every pair of these are
perpendicular, that is,
p
( i , j ) = ( j , k ) = (k , i ) =
2

Let (i , j , k ) be a right-handed
orthogonal triad of unit vectors and O be a point in the space. Let OX, X OY Y and OZ
be directed lines along i , j and k, respectively. Then OX, X OY Y and OZ Z are pairwise perpendicular and non-coplanar
lines. The directed lines OX, X OY Y and OZZ are said to be the positive X X-axis, the positive Y
Y-axis and the positive Z-axis,
respectively. From Figure 5.29, it can be observed that if a right-handed screw with axis along the Z-axis is rotated
through an angle 90° from OX X to OY,
Y then it advances in the direction of positive Z-axis.

O
j Y

FIGURE 5.29 Right-handed system of orthogonal directed lines.


The triad (OX, X OY, Y OZ) is called a right-handed system of orthogonal directed lines. i , j and k are unit vectors
along positive X, X Y and Z axis, respectively. Hereafter, unless otherwise stated, (i , j , k ) always denotes a right-
handed triad
orthogonal unit vectors.
of
Let (i , j , k ) be a right-handed triad of orthogonal unit vectors. Choose a point O in the three-dimensional space
and let A, B, C, A¢, B¢ and C¢ be points in the space such that


OA = -OA¢ = i


OB = -OB¢ = j


and OC = -OC = k

AA

¢, BB¢ and CC¢ are pairwise perpendicular. Usually (i , j , k ) is called an orthonormal


basis and

the
lines AA¢, BB¢, CC are called the coordinate axes relative to (

i , j , k ). O

is called the origin and AA ¢ , BB ¢ and CC¢



are,


respectively,

called the x-axis, the y-axis and the z-axis. OA , OB and OC are called the positive directions and OA ¢, OB¢
and OC¢ are called the negative directions of the coordinate axes.


T H E O R E M 5.22 Let (i , j , k ) be an orthonormal basis of vectors. For any vector a, there exists a unique ordered
triad (a1, a2, a3) of real numbers such that

a = a1i + a2 j + a3k

and a (a1, a2, a3) is a one-to-one correspondence between the set V of vectors and the set 3 of
ordered triads of real numbers.

PROOF This follows from the fact that i , j , k are non-coplanar and hence linearly independent and from
Theorem 5.20. ■
326 Chapter 5 Addition and Scalar Multiplication of Vectors


DEFINITION 5.25 Let a be any vector and (i , j , k ) be an orthonormal basis of vectors. If a = a1i + a2 j + a3k ,

by writing a = (a1 , a2 , a3 ). a1, a2 and a3 are, respectively,
then we denote this called the
x-component (or i -component or the first component),
y-component (or j-component or

the second component) and z-component (or k-component or the third component) of a.


T H E O R E M 5.23 Let a = (a1 , a2 , a3 ) and b = (b1 , b2 , b3 ) with respect to an orthonormal basis (i , j , k ) of vectors and r
be any real number. Then the following hold:

1. a = 0 Û a1 = a2 = a3 = 0

2. a + b = (a1 + b1 , a2 + b2 , a3 + b3 )

3. - a = (- a1 , - a2 , - a3 )

4. ra = (ra1 , ra2 , ra3 )
PROOF These are consequences of the results that

a = a1i + a2 j + a3k

and b = b1i + b2 j + b3k
and Theorems 5.8–5.11. ■


T H E O R E M 5.24 Let a = (a1 , a2 , a3 ) with respect to an orthonormal basis (i , j , k ) of vectors. Then

| a | = a12 + a22 + a32


PROOF Let (OX , OY , OZ )

be an

orthogonal

right-handed system of directed


lines and i , j and k be the
unit vectors along OX , OY and OZ, respectively. Then (i , j , k ) is an orthonormal right-handed
system of unit vectors. Let P be a point in the space such that


OP = a = a1i + a2 j + a3k

Let Q be the foot of the perpendicular from P to the plane ZOX,X as shown in Figure 5.30. Draw
perpendiculars from Q to the X
X-axis and Z-axis to meet at A and C, respectively. Then OCQA is
a rectangle. We have


a = OP = OQ + QP


= OC + CQ + QP


= OC + OA + QP

j
i A
O X
k
C

FIGURE 5.30 Theorem 5.24.


5.5 Components of a Vector 327

Since QPP is perpendicular


to the plane ZOX,X QPP is parallel to the Y


Y-axis. Let B be a point on the
Y-axis such that OB = QP. Now,
Y


a = OC + OA + QP


= OC + OA + OB


= OA + OB + OC
Therefore


a1i + a2 j + a3k = OA + OB + OC


Since OA, OB and OC are linearly independent, we get that


OA = a1 i , OB = a2 j and OC = a3 k

Hence


|OA | = a1 , |OB | = a2 and |OC | = a3

Now


| a |2 = |OP |2 = OP 2 = OQ2 + QP 2 (since OQP is a right angle)
= OC 2 + CQ2 + QP 2 (since OCQ is a right angle)
= OC 2 + OA2 + OB2
= a32 + a12 + a22
Thus

| a | = a12 + a22 + a32 ■


C O R O L L A R Y 5.8 a = (a1 , a2 , a3 ) and b = (b1 , b2 , b3 ) be the position vectors of two points P and Q with respect

to an orthonormal basis (i , j , k ). Then the distance between P and Q is given by

PQ = (a1 - b1 )2 + (a2 - b2 )2 + (a3 - b3 )2

PROOF Let O be the origin and



a = OP = a1i + a2 j + a3k


and b = OQ = b1i + b2 j + b3k

Then


PQ = OQ - OP

= (b1 - a1 )i + (b2 - a2 ) j + (b3 - a3 )k

Therefore


PQ = | PQ| = (b1 - a1 )2 + (b2 - a2 )2 + (b3 - a3 )2

= (a1 - b1 )2 + (a2 - b2 )2 + (a3 - b3 )2 ■



DEFINITION 5.26 Let (i , j , k ) be a right-handed orthonormal
vector basis and a any vector. Let a, b and

g be the angles made by a with i , j and k, respectively. That is a = (a, i ), b = (a, j ) and

g = (a, k ). Then cos a,
cos b, cos g taken in that order are called the direction cosines of a
with respect to (i , j , k ) and these are denoted by l, m, n, respectively.
328 Chapter 5 Addition and Scalar Multiplication of Vectors


T H E O R E M 5.25 Let 0 ¹ a = (a1 , a2 , a3 ) with respect to a right-handed orthonormal vector basis (i , j , k ) and l, m, n

be direction cosines of a. Then
a a a
l = 1 , m = 2 and n = 3
|a| |a| |a|
and l 2 + m2 + n2 = 1

PROOF Let O be the origin and OX, X OY Y and OZ Z be the directed lines along i , j and k, respectively
(Figure 5.31). Let P be a point such that


OP = a = a1i + a2 j + a3k

Let a,
a b, g be the angles made by a with i , j , k , respectively. Then cos a = l, cos b = m and cos g = n.
From Figure 5.31, we have
a1
a1 = OP cos a = rl and hence l =
r
a2
a2 = OP cos b = rm and hence m =
r
a3
a3 = OP cos g = rn and hence n =
r


r = OP = |OP | = | a | . Now,

a = a1i + a2 j + a3k

= rli + rmj + rnk
Therefore

r2 = | a |2 = (rl )2 + (rm)2 + (rn)2 = r2 (l 2 + m2 + n2 )

Hence

l 2 + m2 + n2 = 1

since a ¹ 0 and hence r ¹ 0.

P = (a1, a2, a3)

a
k

O Y
j

FIGURE 5.31 Theorem 5.25. ■


5.5 Components of a Vector 329


DEFINITION 5.27 Let (l, m, n) be direction cosines of a vector a. Then any triad (r, s, t) of real numbers are
said to be direction ratios if (r, s, t) = c(l, m, n) for some position real number c.

Note that, with respect to a given right-handed orthonormal unit vector triad (i , j , k ), any vector has exactly one
triad of direction cosines, whereas it has infinitely many triads of direction ratios, one triad for each positive real
number c.

QUICK LOOK 3


Let (i , j , k ) be a right-handed orthonormal system of 2. The direction ratios of PQ are
vectors

and O the origin.


Let P and Q be points such that (b1 - a1, b2 - a2, b3 - a3)


OP = (a1 , a2 , a3 ) and OQ = (b1 , b2 , b3 ).


1. The direction cosines of PQ are 3. The direction cosines of i , j and k are (1, 0, 0),
(0, 1, 0) and (0, 0, 1), respectively.
æ b1 - a1 b2 - a2 b3 - a3 ö
çè PQ , PQ , PQ ÷ø

Note: Let us recall that for a 2 ´ 2 matrix

æ a bö
çè c d÷ø

the determinant is defined as

æ a bö
det ç = ad - bc
è c d÷ø

and that, for a 3 ´ 3 matrix, the determinant is defined a

æ a11 a12 a13 ö


det ç a21 a22 a23 ÷ = a11 (a22 a33 - a23a32 ) - a12 (a21a33 - a23a31 ) + a13 (a21a32 - a22 a31 )
ç ÷
è a31 a32 a33 ø

In the following we obtain a necessary and sufficient condition for three vectors to be linearly independent in
terms of the determinant of the matrix of their coordinates with respect to any orthonormal vector triad.


T H E O R E M 5.26 Let (i , j , k ) be a right-handed orthonormal vector triad and a = (a1 , a2 , a3 ), b = (b1 , b2 , b3 ) and

c = (c1 , c2 , c3 ) be vectors. Then a, b, c are linearly dependent if and only if

æ a1 a2 a3 ö
det ç b1 b2 b3 ÷ = 0
ç ÷
è c1 c2 c3 ø

PROOF

a = a1i + a2 j + a3k

b = b1i + b2 j + b3k

c = c1i + c2 j + c3k
330 Chapter 5 Addition and Scalar Multiplication of Vectors


Suppose that a, b, c are linearly dependent. Then, there exist real numbers r, s, t, not all zero,
such that

ra + sb + tc = 0
that is

r(a1 i + a2 j + a3k ) + s(b1i + b2 j + b3k ) + t(c1i + c2 j + c3k ) = 0


(ra1 + sb1 + tc1 )i + (ra2 + sb2 + tc2 ) j + (ra3 + sb3 + tc3 )k = 0

Since i , j , k are not coplanar and hence linearly independent, it follows that
ra1 + sb1 + tc1 = 0 (5.3)

ra2 + sb2 + tc2 = 0 (5.4)

ra3 + sb3 + tc3 = 0 (5.5)

Since one of r, s and t must be non-zero, we can assume, without loss of generality, that r ¹ 0. By
multiplying Eqs. (5.3)–(5.5) by
b2 c3 - b3 c2
b3 c1 - b1c3
and b1c2 - b2 c1

respectively and by adding all the resultants we get that


det A = 0
where

æ a1 b1 c1 ö
A = ç a2 b2 c2 ÷
ç ÷
è a3 b3 c3 ø

Since r ¹ 0, det A = 0. Now,

æ a1 a2 a3 ö
det ç b1 b2 b3 ÷ = det AT = det A = 0
ç ÷
è c1 c2 c3 ø

Conversely, suppose that the determinant is zero. Therefore, the homogeneous system of equa-
tions (Vol. 1, Chapter 8)
a1 x + b1 y + c1z = 0
a2 x + b2 y + c2 z = 0
a3 x + b3 y + c3z = 0

has a nonzero solution. Therefore



(a1 x + b1 y + c1z)i + (a2 x + b2 y + c2 z) j + (a3 x + b3 y + c3z)k = 0

That is, xa + yb + zc = 0 where at least one of x, y, z is not zero. Hence a, b, c are linearly
dependent. ■
5.5 Components of a Vector 331

Example 5.6

Let = (3 + 2 + 4)i + (2 - 3 - 1) j + (-4 + 1 + 2)k

a = 3i + 2 j - 4 k = 9i - 2 j - k

b = 2i - 3 j + k The magnitude is given by

c = 4 i - j + 2k | a + b + c | = 92 + (- 2)2 + (- 1)2 = 86

Then find the sum a + b + c, its magnitude and direction
The direction cosines of a + b + c are
cosines.
æ 9 - 2 -1 ö
Solution: çè , , ÷
86 86 86 ø

a + b + c = ( 3i + 2 j - 4 k ) + ( 2 i - 3 j + k ) + ( 4 i - j + 2 k )

Example 5.7

A and
Let B be points
whose position vectors are Then
2 i + 3 j - k and i - 2 j + 2k , respectively. Find the position
vector of the point C that divides AB in the ratio 3:2

2(2 i + 3 j - k ) + 3(i - 2 j + 2k )
OC =
internally. 2+3

Solution: We are given that (4 + 3)i + (6 - 6) j + (-2 + 6)k
=

5
OA = 2 i + 3 j - k 7 4

= i + k
and OB = i - 2 j + 2k 5 5

Example 5.8

A boat is moving in a river. The velocity
of the boat rela- Let us assume that the velocity of Earth is 0.
tive to water is represented
by 3 i + 4 j and
that of water The velocity of water relative to Earth = i - 3 j .
relative to Earth is i - 3 j , where i and j are unit vectors The velocity of the boat relative to water = 3i + 4 j .
along North and East, respectively. Find the velocity and Therefore
direction of the boat relative to the Earth.
3i + 4 j = Velocity of boat - Velocity of water
Solution: If two particles P and Q are moving with veloc-
i - 3 j = Velocity of water - Velocity of Earth
ities a and b, respectively, then the relative velocity of P

with
respect to Q is a - b and that of Q with respect to P Adding these two, we get
is b - a. See Figure 5.32.
4 i + j = Velocity of boat - Velocity of Earth
East
= Velocity of boat relative to Earth
P
Therefore, the speed of the boat is

4i + j | 4 i + j | = 42 + 12 = 17
j 1 and its direction of motion makes an angle

æ 1ö
q = Tan-1 ç ÷
è 4ø
North
O 4 due East–North.
FIGURE 5.32 Example 5.8.
332 Chapter 5 Addition and Scalar Multiplication of Vectors

Example 5.9

Prove
that the points
with position
p
vectors 2 i + j + k ,

P, Q, R are collinear if and only if


6 i - j + 2k and 14 i - 5 j + 4k are collinear. OP, OQ, OR are linearly dependent. Now, consider

Solution: Let O be the origin and P, Q and R be points æ 2 1 1ö


such that det ç 6 - 1 2÷
ç ÷

è 14 - 5 4ø
OP = 2 i + j + k

= 2[(- 1)4 - 2(- 5)] - 1(6 ´ 4 - 2 ´ 14) + 1[6(- 5) - (- 1)14]


OQ = 6 i - j + 2k

= 2 ´ 6 - (- 4) + (- 16) = 0
and OR = 14 i - 5 j + 4k
Therefore by Theorem 5.26, P, Q and R are collinear.

Example 5.10

P, Q, R and S be
Let points
with position
vectors
and
3i - 2 j - k , 2 i + 3 j - 4k , - i + j + 2k and 4 i + 5 j + rk .


Find the value of r so that P, Q, R and S are coplanar. PS = OS - OP

= (4 i + 5 j + rk ) - (3i - 2 j - k )
Solution: Let O be the origin. We are given that

= i + 7 j + (r + 1)k
OP = 3i - 2 j - k


Now P, Q, R, S are coplanar if and only if PQ, PR, PS
OQ = 2 i + 3 j - 4k

are coplanar. Equivalently,


OR = - i + j + 2k

æ -1 5 - 3 ö
OS = 4 i + 5 j + rk det ç - 4 3 3 ÷ =0
ç ÷
Now, è 1 7 r + 1ø


PQ = OQ - OP

= ( 2 i + 3 j - 4 k ) - ( 3i - 2 j - k ) -1[3(r + 1) - 3 ´ 7] - 5[-4(r + 1) - 3 ´ 1] - 3(- 4 ´ 7 - 3)
= - 3r + 18 + 20r + 35 + 93
= - i + 5 j - 3k

= 17r + 146
PR = OR - OP
Therefore P, Q, R, S are coplanar if and only if 17r +
= ( - i + j + 2 k ) - ( 3i - 2 j - k ) 146 = 0. That is

= - 4 i + 3 j + 3k -146
r=
17

Example 5.11

A(1, 1, 1), B(1, 2, 3) and C(2, -1, 1) Therefore


are the vertices
of an isosceles

triangle

and find the direc-


tion cosines of AB, BC and CA. AB = OB - OA = j + 2k


BC = OC - OB = i - 3 j - 2k
Solution: We are given that

and CA = OA - OC = - i + 2 j
OA = i + j + k

Now,
OB = i + 2 j + 3k

AB = | AB| = 02 + 12 + 22 = 5
OC = 2 i - j + k
5.6 Vector Equation of a Line and a Plane 333

æ 1
BC = | BC | = 12 + (- 3)2 + (- 2)2 = 14 -3 -2 ö
Direction cosines of BC = ç , ,

è 14 14 14 ÷ø
CA = |CA| = (- 1)2 + 22 + 02 = 5

æ - 1 2 ö
Direction cosines of CA = ç , ,0
AB = CA, the triangle ABC
C is an isosceles triangle. è 5 5 ÷ø

æ 1 2 ö
Direction cosines of AB = ç 0, , ÷
è 5 5ø

5.6 | Vector Equation of a Line and a Plane


We shall derive vector equation of a straight line and a plane in different forms using the results on collinearity and
coplanarity proved earlier.


T H E O R E M 5.27 Let A be a point with position vector a and let b be a given vector. Then the vector equation of
the straight line parallel to b and passing through A is

x = a + rb, r Î 


PROOF Let O be the origin. Then, we are given that OA = a. Let

L be

line parallel to b and


the straight

passing through A. Let P be arbitrary point on L and OP = x. Since AP is parallel to b, we get
that


AP = rb
for some real number r. Now,


x = OP = OA + AP = a + rb
Conversely, let P be a point such that


OP = x = a + rb
where r is a real number. Then


AP = OP - OA = (a + rb) - a = rb


and hence AP is parallel to b, and so AP P is parallel to L. Since A lies on L, P also lies on L. Thus
P lies on L if and only if


OP = a + rb, r Î  ■


C O R O L L A R Y 5.9 The vector equation of the straight line parallel to a given vector b and passing through the
origin is

x = rb, r Î 

T H E O R E M 5.28 Let (a1, a2, a3) be the coordinates of a point A with reference to a right-handed rectangular
(C A R T E S I A N Cartesian coordinate system OXYZ Z and let (r1, r2, r3) be a triad of non-zero real numbers. Then
FORM OF A the Cartesian equation of the straight line passing through A and with direction ratios (r1, r2, r3) is
LINE)
x - a1 y - a2 z - a3
= = = t, t Î 
r1 r2 r3
334 Chapter 5 Addition and Scalar Multiplication of Vectors


PROOF Let i , j , k be the unit vectors along the positive X, X Y,
Y Z axes, respectively. Let L be the
straight
through A and with direction ratios (r1, r2, r3). The position vector of A is
line passing
a1i + a2 j + a3k = a, say. L is parallel to the vector

b = r1i + r2 j + r3k

Let P(x, y, z) be an arbitrary point in the space. Then



P lies on L Û OP = a + tb for some real number t

Û xi + yj + zk = (a1 + tr1 )i + (a2 + tr2 ) j + (a3 + tr3 )k
Û x = a1 + tr1 , y = a2 + tr2 , z = a3 + tr3 , t Î

x - a1 y - a2 z - a3
Û = = = t, t Î 
r1 r2 r3 ■


T H E O R E M 5.29 Let A and B be two distinct points with position vectors a and b, respectively. The vector equa-
tion of the straight line passing through A and B is

x = (1 - r )a + rb, r Î 


PROOF Let O be the origin. We are given that OA = a and OB = b and


AB = OB - OA = b - a ¹ 0


Let L be the straight line passing through A and B. Then L is parallel to the vector AB and
passing through A. By Theorem 5.27, the vector equation of L is


x = a + r AB

= a + r (b - a )

that is

x = (1 - r )a + rb, r Î  ■

C O R O L L A R Y 5.10 Let A(a1, a2, a3) and B(b1, b2, b3) be two distinct points in the space. Then the Cartesian equation
of the straight line passing through A and B is
x - a1 y - a2 z - a3
= = = t, t Î 
b1 - a1 b2 - a2 b3 - a3

PROOF OXYZ Z be a right-handed rectangular Cartesian coordinate system and i , j and k be the
unit vectors along the positive X,
X Y and Z axes, respectively. Let L be the straight line passing
through A and B. Let P (x, y, z) be an arbitrary point in the space. Then


OA = a1i + a2 j + a3k = a, say


OB = b1i + b2 j + b3k = b, say


and OP = xi + yj + zk
Now


P lies on L Û xi + yj + zk = OP

= (1 - r )a + rb , r Î  (by Theorem 5.29)

Û ( x - a1 )i + ( y - a2 ) j + (z - a3 )k = r[(b1 - a1 )i + (b2 - a2 ) j + (b3 - a3 )k]
5.6 Vector Equation of a Line and a Plane 335

Û x - a1 = r(b1 - a1 ), y - a2 = r(b2 - a2 ), z - a3 = r(b3 - a3 )


x - a1 y - a2 z - a3
Û = = = r, r Î 
b1 - a1 b2 - a2 b3 - a3 ■

Example 5.12

Find the vector equation and the Cartesian


equation of
the
Let A and B be the points such that
straight line passing through 2 i + j + 3k and - 4 i + 3 j - k.


OA = 2 i + j + 3k
Solution: From Theorem 5.29, the vector


equation
and OB = - 4 i + 3 j - k
of line passing through a = 2 i + j + 3k and
the straight
b = - 4 i + 3 j - k is The Cartesian coordinates of A and B are (2, 1, 3) and (-4,
3, -1), respectively. From Corollary 5.10,, the Cartesian
x = (1 - r )a + rb, r Î  equation of the line joining A and B is
That is x-2 y-1 z-3
= = = r, r Î 
-4 - 2 3 - 1 -1 - 3
x = (2 - 2r - 4r )i + (1 - r + 3r ) j + (3 - 3r - r )k
x-2 y-1 z-3
x = (2 - 6r )i + (1 + 2r ) j + (3 - 4r )k , r Î  = =
-6 2 -4
x-2 z-3
= y-1=
3 -2

Example 5.13

Does the straight line passing through the points x = (1 - s)(a - 2b + 3c ) + s(a - 6b + 6c )
and
2a + 3b - c and 3a + 4b - 2c intersect
the straight
line

passing through the points a - 2b + 3c and a - 6b + 6c . = a + (- 2 - 4 s)b + (3 + 3s)c , r Î  (5.7)

If so, find the point of intersection. (Here a, b, c are non-
coplanar vectors.) For L1 and L2 to have a common point, we have to
find r and s such that the corresponding coefficients in
Solution: Let L1 and L2 be the first and second Eqs. (5.6) and (5.7) of L1 and L2 are equal, that is
straight lines given. Then the equations of L1 and L2 are, 2 + r = 1,
respectively,
3 + r = - 2 - 4s
x = (1 - r )(2a + 3b - c ) + r(3a + 4b - 2c )
and - 1 - r = 3 + 3s

= (2 + r )a + (3 + r )b - (1 + r )c , r Î (5.6)
These give r = -1 and s = -1. Also, the point of intersec-

tion is a + 2b.

Example 5.14

Using vector methods prove that respectively. Let A and B be points on OX


X and OY,
Y respec-
x y tively, and
+ =1


a b OA = ai and OB = bj
is the equation of a straight line in intercept form (where Then
the coordinate axes may be perpendicular or oblique).


OA OB

i = and j=
Solution:
OX , OY , OZ be any non-coplanar
Let

vectors.

a b
Let i and j be the unit vectors along OX and OY ,
336 Chapter 5 Addition and Scalar Multiplication of Vectors


Let P(x, y) be an arbitrary

point on the straight line AB. Since OA and OB are not collinear and hence linearly
Choose a point Q on OA such that QP P is parallel to OB independent, we get that
(Figure 5.33). Then
x y

= 1 - r and =r
OQ = xi = OA a b
a
From these two, we get that


and QP = yj = OB x y
b + =1
a b


OP = OQ + QP = OA + OB B
a b


Since P is a point on the line AB,

P (x, y)
OP = (1 - r )OA + rOB

for some r Î. Thus


x


OA + OB = (1 - r )OA + rOB
a b

FIGURE 5.33 Example 5.14.


T H E O R E M 5.30 Let A be a point in the space with position vector a with respect to the origin O. Let b and c be
non-collinear
vectors. Then the vector equation of the plane through the point A and parallel

to b and c is

x = a + rb + sc , r and s Î 


PROOF We are given that OA = a. Choose

points
B C such that AB = b and AC = c . The points A,
and

B and C are not collinear, since AB and AC are not collinear (Figure 5.34). Therefore, there
exists a unique plane passing through A, B and C. Let that plane be P. Then P is the plane

passing through the point A and parallel to the vectors b and c.

Let P be any point with position vector x with respect to the origin O; that is,


OP = x

P
c

A B
b
x
a

FIGURE 5.34 Theorem 5.30.


5.6 Vector Equation of a Line and a Plane 337

Since P lies on the plane P, then by Theorem 5.15, there exist real numbers r and s such that


AP = r AB + sAC = rb + sc
and hence


x = OP = OA + AP = a + rb + sc


Conversely, if P is a point and r and s are real number such that OP = a + rb + sc , then


AP = OP - OA = OP - a = rb + sc
and therefore, again

by
Theorem
5.15, P lies on the plane determined by A, B and C. Thus P lies

on P if and only if OP = a + rb + sc for some real numbers r and s. ■


C O R O L L A R Y 5.11 The vector equation of the plane passing through origin and parallel to the vectors b and c is

x = rb + sc , r, s Î 


T H E O R E M 5.31 Let A, B and C be three non-collinear points with position vectors a, b and c, respectively, rela-
tive to the origin O. Then the vector equation of the plane passing through A, B and C is

x = (1 - r - s)a + rb + sc , r and s Î 


PROOF We are given that OA = a
,

= b
and
OB

OC = c . Let P be the plane passing through A, B and


C. Then P is parallel to AB and AC and is passing through A (Figure 5.35). Therefore, by
Theorem 5.30, the vector equation of the plane P is


x = a + r AB + sAC , r, s Î 


= a + r(OB - OA) + s(OC - OA)

= a + r(b - a ) + s(c - a )

= (1 - r - s)a + rb + sc , r and s Î 

A
B
c x
b

FIGURE 5.35 Theorem 5.31. ■


338 Chapter 5 Addition and Scalar Multiplication of Vectors


C O R O L L A R Y 5.12 Let A, B and C be non-collinear points with position vectors a, b and c, respectively, relative

to the origin O. Then the vector equation of the plane passing through A and B parallel to c is

x = (1 - r )a + rb + sc , r and s Î 


PROOF We have OA = a, OB = b and OC = c . Also,


AB = OB - OA = b - a

Let P be the plane passing

through
A and B and parallel to c. Then P is the plane passing

through A and parallel to AB and c. By the Theorem 5.30 the vector equation of P is


x = a + r AB + sc , r, s Î 

= a + r(b - a ) + sc

that is

x = (1 - r )a + rb + sc , r and s Î  ■

Example 5.15

Find the vector equation of the plane passing through By Theorem 5.31, the vector equation of the plane
the points (1, -2, 5), (0, -5, -1) and (-3, 5, 0). passing through A, B and C is

Solution: Let A = (1, -2, 5), B = (0, -5, -1) and C = (-3, x = (1 - r - s)a + rb + sc , r, s Î 
5, 0). Let the position vectors of A, B and C relative to
= (1 - r - s)(i - 2 j + 5k ) + r(- 5 j - k ) + s(- 3i + 5 j )
the origin be a, b and c, respectively. Then

that is
OA = a = i - 2 j + 5k

x = (1 - r - 4 s)i + (- 2 - 3r + 7 s) j + (5 - 6r - 5s)k
OB = b = - 5 j - k


and OC = c = - 3i + 5 j


T H E O R E M 5.32 Four points A, B, C and D with position vectors a, b, c and d, respectively, are coplanar if and
only if there exist real numbers r, s, t and u, not all zero, such that
r+s+t+u=0

and ra + sb + tc + ud = 0
PROOF r, s, t and u, not all zero, such that
r+s+t+u=0

and ra + sb + tc + ud = 0
Without loss of generality, we can assume that r ¹ 0. Then
s + t + u = -r ¹ 0

and -( s + t + u)a + sb + tc + ud = 0


s(b - a) + t(c - a ) + u(d - a ) = 0

and hence


sAB + t AC + uAD = 0
5.6 Vector Equation of a Line and a Plane 339


and one of s, t, u is not zero (since r + s + t + u = 0 and r ¹ 0). Therefore, AB, AC and AD are
coplanar vectors and hence the points A, B, C and D are coplanar.


Conversely,

suppose that the points A, B, C and D are coplanar. Then the vectors AB, AC
and AD are coplanar and hence there exist real numbers a and b such that


AD = a AB + b AC
that is

d - a = a (b - a ) + b (c - a )

Therefore

(1 - a - b )a + ab + bc + (- 1)d = 0

By taking r = 1 - a - b, s = a, t = b and u = -1, we get that


r+s+t+u=0

and ra + sb + tc + ud = 0 ■


T H E O R E M 5.33 Equation of the angle bisectors of the lines whose equations are r = a + tb and r = a + sc is

é ±b c ù
r =a + lê + ú
ë |b| |c | û
where t, s and l are scalar parameters.

PROOF The
given lines are intersecting in the point A with position vector a and parallel to the vectors

b and c respectively. Let P be a point with position vector r on the bisector of angle LOM M (see

Figure 5.36). Through P draw a line parallel to c meeting the line AL in Q. Therefore
QAP = APQ Þ AQ = PQ

Hence

æ bö

æ cö
AQ = l ç ÷ and QP = l ç ÷
è | b |ø è | c |ø

Therefore


AP = AQ + QP

æb cö

Þ r - a = lç + ÷
è | b| | c | ø

æ b cö
Þ r = a + lç + ÷
è | b | | c |ø

Similarly, the other angle bisector can be seen as



æ -b c ö
r = lç + ÷
è | b | | c |ø
340 Chapter 5 Addition and Scalar Multiplication of Vectors

A Q L

FIGURE 5.36 Theorem 5.33. ■

WORKED-OUT PROBLEMS
Single Correct Choice Type Questions

1. Let a and b be non-collinear
vectors. If the vectors Solution: We have

(l - 1)a + 2b and 3a + lb are collinear vectors, then


the value of l is AB + AE + BC + DC + ED + AC


(A) 2 or 3 (B) -2 or 3 = ( AB + BC ) + ( AE + ED + DC ) + AC
(C) -2 or -3 (D) 2 or -3


= AC + AC + AC

Solution: Since the vectors (l - 1)a + 2b and 3a + lb


are collinear vectors, there exists scalar x such that = 3AC

3a + lb = x[(l - 1)a + 2b] D

= x(l - 1)a + 2 xb

Since a and b are non-collinear vectors, the corresponding
scalar coefficients on both sides of Eq. (5.8) must be C
E
equal. Therefore
x(l - 1) = 3 and 2 x = l

On solving the equations for x we get


l B
(l - 1) = 3
2
FIGURE 5.37 2.
and hence l = -2 or 3. Answer: (D)
Answer: (B)
3. In DABC, let O and H denote the

circumcentre

and


orthocentre, respectively. Then OA + OB + OC =
2. If ABCDE

E is

a pentagon

Figure 5.37), then AB +



(see
AE + BC + DC + ED + AC is equal to

(A) OH (B) 2OH (C) OH (D) 3OH


(A) 6AC (B) 5AC (C) 4AC (D) 3AC 3
Worked-Out Problems 341

Solution: In Chapter 4 we have proved that

1
AD = b
AO = 2 R cos A 2

1
and OD = R cos B AE = c
2

where D is the mid-point of the side BCC (Figure 5.38).

b + c
Now and AF =

2
OA + OB + OC = OA + 2OD

Now
= OA + AH

BE + AF = ( BA + AE) + AF
= AH
æ cö b + c
= ç -b + ÷ +
A è 2ø 2

b
=c-
H 2
O


= DC
C
D A

E
FIGURE 5.38 Single correct choice type question 3.
D
Answer: (A)
C
Note: With circumcentre as origin of reference, the
F
position vector of the orthocentre is sum of the position
vectors of the vertices of the triangle. Equivalently, in the
Argand’s plane (complex numbers), if the circumcentre FIGURE 5.39 Single correct choice type question 5.
of a triangle is the origin of the coordinate axes, then the Answer: (D)
complex number representing the orthocentre is the sum

of the complex numbers representing the vertices. 6. If a = (m, - 2, 5) and b = (1, n, - 3) are collinear vectors,
then

c are
4. a, b and non-coplanar vectors. If a + b + c = xd 5 -6 -5 -6
(A) m = , n = (B) m = ,n=
and b + c + d = ya, where x, y are scalars, then 3 5 3 5
(A) x = y = 1 -5 6 5 6
m= ,n= (D) m = , n =
(B) x = y = -1 3 5 3 5
(C) x = 1, y = -1
Solution: Since the vectors a and b are collinear

(D) a + b + c + d = ( x + y)(a + d ) vectors, there exists a scalar x such that b = xa. Therefore
Solution: From the given equations we get (1, n, - 3) = x(m, - 2, 5)

a + b + c = xd = x( ya - b - c ) Hence
Equating the corresponding coefficients we get mx = 1, - 2 x = n and 5 x = - 3
xy = 1, x = - 1, y = - 1 Solving these we get
Answer: (B) -3 -5 6
x= ,m= ,n=
5 3 5
5. D, E and F are the mid-points of the

sides AB,

AC
C and
BC, respectively. Then the vector BE + AF equals Answer: (C)
3


(A) BF (B) BF (C) 2BF (D) DC 7. Points D, E and F divide the sides BC, CA and
2 2 AB internally in the ratio 1: 4, 3 : 2 and 3 : 7. If the

Take A as origin (Figure 5.39) and let AB = b


Solution:

P
point

the side AB in the ratio 1: 3, then


divides
and AC = c . Therefore AD + BE + CF is equal to
342 Chapter 5 Addition and Scalar Multiplication of Vectors


8. Let a = i + j - k , b = 5i - 3 j - 3k and c = 3i - j + 2 k .
(A) CP (B) CP (C) CP (D) CP
3 5 5 5 If r is a vector in the direction of c with magnitude

| a + b |, then r is equal to
A 3 2
3 (A) 3c (B) 2c (C) c (D) c
2 2 3
F
E
Solution: We have

7 a + b = 6 i - 2 j - 4k
3

Þ | a + b | = 62 + 22 + 42 = 56 = 2 14

FIGURE 5.40 Single correct choice type question 7.


c c

Unit vector in the direction of c = =


Solution: Take A as origin and let AB = b, AC = c . |c | 14
Therefore
Therefore

c + 4b
AD = æ cö æ c ö
5 r =| a + b | ç ÷ = 2 14 ç ÷ = 2c
è | c |ø è 14 ø

2
AE = c Answer: (B)
5

3
9. The position
vectors of three points
are 2a - b + 3c ,
AF = b
10 a - 2b + mc and na - 5b where a, b, c are non-coplanar
vectors and m, n are scalars. The three points are

b
AP = collinear if and only if
4 9 9
(A) m = - 2, n = (B) m = , n = - 2
Now 4 4

c + 4b æ 2 ö æ 3 ö (C) m = 2, n =
9 9
(D) m = - , n = 2
AD + BE + CF = + ç c - b÷ + ç b - c ÷ 4 4
5 è5 ø è 10 ø
Solution: Let the points be P, Q and R,

respectively.


1 Then P, Q, R are collinear if and only if PQ = xQR for
= (- 4c + b)
10 some scalar x. This implies and is implied by
1
= (b - 4 c ) - a - b + (m - 3)c = x[(n - 1)a - 3b - mc ]
10
Û x(n - 1) = - 1, - 3 x = - 1 and m - 3 = - xm
But
1 9

b 1 Ûx= ,m= and n = - 2


3 4
CP = - c = (b - 4c )
4 4 Answer: (B)

10 æ b - 4c ö
= ç 10. a, b and c be non-coplanar vectors. Suppose
4 è 10 ÷ø A, B, C and D are four
points

with position vectors

- a + 4b -
3

c , 3a +

2b
- 5c ,

- 3a + 8b - 5c and - 3a +
5


= ( AD + BE + CF ) 2b + c . If AB = xAC + yAD, then
2
(A) x = 1, y = 1 (B) x = 1, y = -1
Therefore (C) x = -1, y = 1 (D) x = -1, y = -1

Solution: We have
AD + BE + CF = CP
5


AB = xAC + yAD
Answer: (D)

Þ 4a - 2b - 2c = x(- 2a + 4b - 2c ) + y(- 2a - 2b + 4c )
Worked-Out Problems 343

Equating the corresponding coefficients of both sides whose values also satisfy Eq. (5.14). Therefore
we get

4 a + 2b + 5c
-2x - 2y = + 4 (5.9) OP =
11
4x - 2y = -2 (5.10) l = 11
-2x + 4y = -2 (5.11) Answer: (A)

Solving Eqs. (5.9) and (5.10), we get that x = -1 and 12. DABC, E is the mid-point of AB and D is a point
y = -1 which also satisfy Eq. (5.11). on the side BCC such that BD : DC = 2 :1. The lines
Answer: (D) AD and CE E intersect in Q. Then the ratio AQ : QD is
(A) 2 :1 (B) 2 : 3 (C) 3 :1 (D) 3 : 2
11. Let a, b, c be non-coplanar vectors which are the
position vectors of three non-collinear points A, B A
and C, respectively. Points D and E divide the sides
BCC and BA in the ratios 5 : 2 and 2 :1. Let the lines
AD and CE E meet in P.
If

O is the origin of reference,


x
then 4 a + 2b + 5c = l(OP ), where l equals E
(A) 11 (B) 7 (C) 3 (D) 4 i

Solution: By hypothesis, Q y
j

2b + 5c

2a + b
OD = and OE =
7 3 FIGURE 5.42 Single correct choice type question 12.

See Figure 5.42. Taking A as origin, let AB = b


Solution:
and AC = c . Therefore
l
x

1
E 1 AE = b
2
y

b + 2c
P
1
2
2 C and AD =
D 3
5
AQ : QD = x :1 and CQ : QE = y :1. Therefore
FIGURE 5.41 Single correct choice type question 11.
x[(b + 2c )/ 3] + 1(0) y(b / 2) + c
=
Suppose AP : PD = x :1 and CP : PE = y :1. Therefore x+1 y+1

Since b and c are non-collinear vectors, we have
x[(2b + 5c )/ 7] + a

y[(2a + b)/ 3] + c
= OP = x y
x+1 y+1 = (5.15)
3( x + 1) 2( y + 1)
Since a, b, c are non-coplanar, in the above equation the
corresponding coefficients on both sides must be equal. So 2x 1
and = (5.16)
3( x + 1) y + 1
1 2y
= (5.12)
x + 1 3( y + 1) Therefore from Eqs. (5.15) and (5.16) we have

2x y æ y ö 1
= (5.13) 2ç =
7( x + 1) 3( y + 1) ÷
è 2( y + 1) ø y + 1
5x 1 Hence y = 1 and x = 3.
= (5.14)
7( x + 1) y + 1 Answer: (C)
From Eqs. (5.12) and (5.13) we get 13. P, Q and R are the mid-points of the sides BC, CA
7 6 and AB of DABC. If O is any point in the space (or
x= and y= in the plane of DABC), then
4 5


OA + OB + OC = l(OP + OQ + OR)
344 Chapter 5 Addition and Scalar Multiplication of Vectors

where the value of l is 15. The perimeter of the triangle with vertices (3, 1, 5),
2 (-1, -1, 9) and (0, -5, 1) is
(A) 2 (B) 3 (C) (D) 1
3 (A) 27 (B) 3 31 (C) 15 (D) 15 + 61

Solution: Since P, Q and R are mid-points of BC, CA Solution: Let the given points be A, B and C respec-
and AB, we have tively. Then

OB + OC AB = (- 4, - 2, 4)
OP =


2 BC = (1, - 4, - 8)

OC + OA


OQ = CA = (3, 6, 4)
2

Therefore

OA + OB
and OR =


2 | AB| = 42 + 22 + 42 = 6
Therefore


| BC | = 12 + 42 + 82 = 9


2(OP + OQ + OR) = 2(OA + OB + OC )


|CA| = 32 + 62 + 42 = 61
Hence l = 1. Answer: (D)
Answer: (D)
16. The position vectors of the vertices A, B and C of a
14. If the points triangle are respectively (a, b, c), (b, c, a) and (c, a, b).
Then DABC C is
( 2 - a )i + 2 j + 2 k (A) right angled

2 i + (2 - b ) j + 2k (B) right-angled isosceles
(C) equilateral
2 i + 2 j + (2 - g )k
(D) obtuse angled
and i + j +k
Solution: The sides are represented by the vectors
are coplanar, and abg ¹ 0, then


(A) a + b + g = 1 AB = (b - a)i + (c - b) j + (a - c)k


1 1 1 BC = (c - b)i + (a - c) j + (b - a)k
(B) + + =1
a b g


and CA = (a - c)i + (b - a) j + (c - b)k
1 1 1
(C) + + =1 Therefore
1-a 1- b 1-g


(D) a + b + g = 0 | AB| = | BC | = |CA| = (a - b)2 + (b - c)2 + (c - a)2
Solution: Let the given points be respectively A, B,
C and D. Therefore, the four points A,

B, Hence DABC
C is equilateral.
C D
are

and


coplanar if and only if the three vectors AB , AC

and AD Answer: (C)


are coplanar. Hence the vectors AB, AC , AD are linearly
dependent (see Theorem 5.19). Therefore 17. ABCD is a quadrilateral. M and N are mid-points of
the sides AB and CD, respectively, and the diagonals
a -b 0 ACC and BD intersect in ‘O’. If the points M, N and O
a 0 -g = 0 (See Theorem 5.26) are collinear, then ABCD is a
a - 1 -1 -1 (A) trapezium (not necessarily parallelogram,rectangle,
square or rhombus)
Þ a (0 - g ) + b[-a + g (a - 1)] = 0
(B) parallelogram (not necessarily rectangle, square
Þ -ag - ab + abg - bg = 0 or rhombus)
Þ bg + ga + ab = abg (C) rectangle (not necessarily square or rhombus)
1 1 1 (D) rhombus (not necessarily square)
Þ + + =1
a b g
Answer: (B)
Worked-Out Problems 345


Solution:

See Figure
5.43. ‘O’ as origin. Let OA = a,

Take = 2( xa + 2 yb) - (- 2 ya + 3 xb)



OB = b, OC = l a and OD = mb where l and m are scalars.
= (2 x + 2 y)a + (4 y - 3 x)b
Clearly a and b are non-collinear vectors. Therefore,

a + b Therefore
OM =
2 2x + 2y = 4 (5.17)

la + mb 4 y - 3x = - 2 (5.18)
ON =
2
Solving Eqs. (5.17) and (5.18), we get that
D N C 10 4
x= ,y=
7 7
O
a Answer: (A)
b

19. ABCD is a parallelogram and point P divides AD


in the ration 3 :1 internally. The line BP
P meets the
FIGURE 5.43 Single correct choice type question 17. diagonal AC C in Q. Then the ratio AQ : QC
C is equal to
(A) 2 : 3 (B) 3 : 2 (C) 4 : 3 (D) 3 : 4
Since M, O and N are collinear, there exists scalar K such
that Solution: Take A as origin (Figure 5.44) and let


ON = KOM AB = b and AD = d

That is so that


la + mb (a + b ) AC = b + d
=K
2 2 Therefore

3
AP = d
l=K=m 4

and hence Suppose AQ : QC = m :1 and BQ : QP = l :1. then



DC = la - mb = K (a - b) = K BA l[(3 / 4)d ] + b m (b + d )
=
l+1 m+1
So, the sides AB and CD are parallel and ABCD is a
trapezium. 1 m 3l
= =
Answer: (A) l + 1 m + 1 4(l + 1)

This gives
18. a and b are non-collinear vectors and
4 3 m 3
a = xa + 2 yb l= and = Þm=
3 7 m+1 4

b = - 2 ya + 3 xb Hence AQ : QC = 3 : 4.

and g = 4a - 2b
D C

If g = 2a - b , then
1
10 4 4 6
(A) x = , y = (B) x = , y = P 1
7 7 7 7 Q
1
8 2
(C) x = , y = (D) x = 2, y = 3 3
7 7
Solution: By hypothesis,
FIGURE 5.44 Single correct choice type question 19.
4a - 2b = g

= 2a - b
346 Chapter 5 Addition and Scalar Multiplication of Vectors

x, y, z is equal to 1 and
20. If none of the scalars the Hence
vectors xi + j + k , i + yj + k and i + j + zk are
coplanar, then æ 3ö æ 1ö
r = 4ç ÷ i + 4ç ÷ j
è 2 ø è 2ø
1 1 1
+ + =
1- x 1- y 1- z = 2 3 i + 2j
(A) 0 (B) 1 (C) -1 (D) xyz Answer: (D)
Solution: According to Theorems 5.19 and 5.26, we
22. If the vectors ai + aj + ck , i + k and ci + cj + bk are
have coplanar, then
x 1 1 1 1 2
(A) a + c = 2b (B) + =
1 y 1 =0 a c b
1 1 z (C) c2 = ab (D) b2 = ca
Solution: Since the vectors are coplanar, we have
The row operations R1 - R2 and R2 - R3 give
a a c
x -1 1- y 0
1 0 1 =0
0 y-1 1- z = 0
c c b
1 1 z
Þ a(0 - c) - a(b - c) + c(c - 0) = 0
Þ ( x - 1)[z( y - 1) - (1 - z)] - (1 - y)[0 - (1 - z)] = 0
Þ z( x - 1)( y - 1) - ( x - 1)(1 - z) + (1 - y)(1 - z) = 0 Þ - ac - ab + ac + c2 = 0

Þ z(1 - x)(1 - y) + (1 - x)(1 - z) + (1 - y)(1 - z) = 0 Þ c2 = ab


Answer: (C)
z 1 1
Þ + + =0
1- z 1- y 1- x 23. In DABC, D is the mid-point of BC
C and E is the mid-
æ z ö point of AD. BE E is produced to meet the side AC
1 1
Þç + 1÷ + + =1 in FF. Then BFF is equal to
è1- z ø 1- y 1- x
(A) 4 EF (B) 3 EF (C) 2 EF (D) 6 EF
1 1 1


Þ + + =1

See Figure 5.46. Take A as origin, let AB = b


Solution:
1- z 1- y 1- x and AC = c . Then
Answer: (B)

b + c
AD =
21. In the two-dimensional plane, if a vector r of magni- 2
tude 4 makes an angle 30° with the positive direction

b + c
of x-axis, then r is equal to and AE =
4
(A) 4 3 i + j (B) 2 i + 3 j
According to Theorem 5.29, the equations of the lines
(C) 2 i + 2 3 j (D) 2 3 i + 2 j BE
E and ACC are, respectively,

y A

P
F

30°
E
O M x

C
FIGURE 5.45 Single correct choice type question 21.

D
Solution: Let OP = r where O is the origin (Figure 5.45).
Therefore

its components
along

x-axis
and y-axis
are (|OP | cos 30°)i and (|OP | sin 30°) j , respectively.
FIGURE 5.46 Single correct choice type question 23.
Worked-Out Problems 347


(b + c ) (i.e., O ) and the two points 4 j , 2 i + k at the point
r = (1 - x)b + x
4 whose position vector is
1 1
and r = yc (A) (6 i - 10 j + 3k ) (B) (6 i + 10 j + 3k )
5 5
x, y are scalars. The two lines BE
E and AC
C intersect
1 1
in F
F. Therefore, the position vector F must satisfy both (C) (6 i + 10 j - 3k ) (D) (6 i - 10 j - 3k )
equations. Hence equating the corresponding coeff- 5 5
icients we get Solution: By Theorem 5.31,
the equation of the plane
x x passing through O, 4 j and 2 i + k is
1 - x + = 0 and =y
4 4
r = x(4 j ) + y(2 i + k ) (5.19)

Again, by Theorem 5.29, the equation of the line passing


x=
4
and y=
1 through the points i - 2 j + k and - 2 j + 3k is
3 3
r = (1 - z)(i - 2 j + k ) + z(- 2 j + 3k ) (5.20)
Therefore
The common point of the plane and the line satisfies both

1
AF = c Eqs. (5.19) and (5.20). Therefore, by equating the corre-
3 sponding coefficient of Eqs. (5.19) and (5.20), we have
Now 2 y = (1 - z) (5.21)

1 b + c 1 4 x = - 2(1 - z) - 2z (5.22)
EF = c - = (c - 3b)
3 4 12 y = 1 - z + 3z (5.23)

1
BF = c - b Solving Eqs. (5.21)–(5.23), we get
3
-1 3 -1
c - 3b x= ,y= and z =
= 2 5 5
3
Substituting these values in Eq. (5.19), we have
4(c - 3b)
=
12 -1 3

r= (4 j ) + (2 i + k )
= 4 EF 2 5
1
Answer: (A) = (6 i - 10 j + 3k )
5

24. If the vectors a + b - lc , 3a - 2b + 4c , 3a - 7b + 14c Answer: (A)

are linearly dependent ( a, b, c are non-coplanar)
then the value of l is equal to 26. If the vectors
(A) 14 (B) 2 (C) 3 (D) 4
a=i + j +k
Solution: Since the vectors are linearly dependent, by
Theorem 5.26, b = 4 i + 3 j + 4k

1 1 -l and c = i + aj + bk
3 -2 4 = 0
| c | = 3 , then
3 - 7 14 (A) a = ±1, b = 1 (B) a = 1, b = -1
Þ 1(-28 + 28) - 1(42 - 12) - l (-21 + 6) = 0 (C) a = 1, b = ±1 (D) a = -1, b = ±1
Þ - 30 + 15l = 0
Solution: Since the vectors a, b, c are linearly depen-
Þl=2 dent, by Theorem 5.26, we have
Answer: (B) 1 1 1
4 3 4 =0
25. The line joining the points i - 2 j + k and - 2 j + 3k
meets the plane passing through the points origin 1 a b
348 Chapter 5 Addition and Scalar Multiplication of Vectors

Þ (3b - 4a ) - (4 b - 4) + (4a - 3) = 0
29. Let a, b and c be non-coplanar vectors. Let L and P
Þ -b + 1 = 0 be a line and a plane whose equations are

Þb =1 r = a + 2 b + t (a - c )

Now | c | = 3 implies and r = x(a + 2b) + y(2b + c )

12 + a 2 + b 2 = 3 respectively, where t, x, y are scalars. Then



(A) the line L meets the plane P in the point a - b + c
Þ a2 = 1
L
(B) the line meets the plane P in the point
Þ a = ±1
2a + 2b - c
Answer: (A) (C) the line L and the plane P have infinitely many
common points
3i +
27. If 2 j - 5k = x(2 i - j + k ) + y(i + 3 j - 2k ) + z(- 2 i + (D) the line L does not intersect the plane P
j - 3k ) then
(A) x = 1, y = 2, z = 3 (B) x = 2, y = 3, z = 1 Solution: Equating the corresponding coefficient of
the line and the plane, we have
(C) x = 3, y = 1, z = 2 (D) x = 1, y = 3, z = 2
x =1+ t
Solution: Equating the coefficients of i , j and k on
both sides we get x+ y=1
and y = -t
2 x + y - 2z = 3 (5.24)
- x + 3y + z = 2 (5.25) Therefore, the equation of the line L is

x - 2 y - 3z = - 5 (5.26) r = a + 2b - y(a - c )

Solving these equations, we have x = 3, y = 1 and z = 2. = ( x + y)(a + 2b) - y(a - c )
Answer: (C)
= x(a + 2b) + y(2b + c)

28. The position vectors


A, B and C are,
of the vertices All points of the line L lie in the plane P.
respectively, i - j - 3k , 2 i + j - 2k and - 5i + 2 j - 6k. Answer: (C)
The internal bisector of angle A meets the side BC
in D. Then the length AD is
30. If
1 11 15 3 10
(A) (B) (C) (D)
4 2 2 4 a a2 a3 + 1
b b2 b3 + 1 = 0
Solution: It is known that D divides the segment BC
internally in the ratio of AB : AC. Now c c2 c3 + 1


AB = i + 2 j + k and
the vectors A = (1, a, a2 ), B = (1, b, b2 ) and

C = (1, c, c2 ) are non-coplanar, then abc is equal to


AC = - 6 i + 3 j - 3k
(A) 1 (B) -1 (C) 2 (D) -2
and hence AB = 6 and AC = 54 = 3 6. Therefore
Solution: Let
AB : AC = 1:3
a a2 a3 + 1
Now D = b b2 b3 + 1
BD : DC = AB : AC = 1: 3 c c2 c3 + 1

3
Þ AD = (- i + 3 j ) a a2 a3 a a2 1
4
= b b2 b3 + b b2 1
3
Þ AD = 10 c c2 c3 c c2 1
4
Answer: (D)
Worked-Out Problems 349


1 a a2 1 a a2 Since A, B, C are non-coplanar, they are linearly inde-
= abc 1 b b2 + 1 b b2 pendent and hence
1 c c2 1 c c2 1 a a2
1 b b2 ¹ 0
1 a a2
1 c c2
= (abc + 1) 1 b b2
1 c c2 (by Theorem 5.26). Therefore D = 0 Þ abc + 1 = 0.
Answer: (B)

Multiple Correct Choice Type Questions


1. Consider the cube OAFBDCEP
P (see Figure 5.47) Then (D) We have


(A) OA + OB + OC = OP OA + OC + OE = i + k + (i + k )


(B) OE + OF + OD = 2OP

= 2(i + k )
(C) OP + CF + BE + DA = 4OA

Hence (D) is not true.


(D) OA + OC + OE = 3OP
Answers: (A), (B), (C)


C D
2. Let a, b, c are non-coplanar vectors. Then

E P (A) the vectors a - 2b + 3c , - 2a + 3b - 4c , a - 3b + 5c
are coplanar.
B (B) the vectors given in (A) are linearly independent.
O
(C) if a - 2b + 3c = x(- 2a + 3b - 4c ) + y(a - 3b + 5c )
then x = -1/3 and y = 1/3.

(D) a - 2b + 3c cannot be written as a linear combina-

FIGURE 5.47 Multiple correct choice type question 1.
tion of the vectors - 2a + 3b - 4c and a - 3b + 5c .
Solution:

We can

suppose,
the cube is unit cube and Solution:
OA = i , OB = j , OC = k . Therefore
(A) We have
(A) We have

1 -2 3
OA + OB + OC = i + j + k = OP
- 2 3 - 4 = 1(15 - 12) + 2(- 10 + 4) + 3(6 - 3)
Therefore (A) is true. 1 -3 5
(B) We have

= 3 - 12 + 9 = 0
OE + OF + OD = (i + k ) + (i + j ) + ( j + k )
Therefore the vectors a - 2b + 3c , - 2a + 3b - 4c

= 2(i + j + k ) and a - 3b + 5c are linearly dependent and hence

they are coplanar. This implies (A) is true and (B)


= 2OP is not true.
(C) It is true because by equating the corresponding
(C) We have coefficients on both sides we have

-2x + y = 1
OP + CF + BE + DA = (i + j + k ) + (- k + i + j )
3x - 3 y = - 2
+ (- j + i + k ) + (- j - k + i )

and - 4 x + 5y = 3
= 4 i = 4OA
(C) is true.
350 Chapter 5 Addition and Scalar Multiplication of Vectors

On solving these equations, we get that A

-1 1
x= ,y=
3 3
4
Answers: (A), (C)
1 E
3. Let a, b, c be non-coplanar vectors. Consider the P
four points A, B, C and D whose position vectors
are, 1
1
respectively,
- a + 4 b - 3c , 3a + 2 b - 5c , 3a + 8 b - 5c and

- 3a + 2b + c. Then
(A) A, B, C and D are non-coplanar FIGURE 5.48 Multiple correct choice type question 4.

(B) A, B, C and D are coplanar

Solution: Take A as origin. Let AB = b and AC = c .


(C) AB, AC , AD are linearly dependent Therefore by hypothesis,



(D) AB, AC , AD are linearly independent

b + 3c

4c
AD = and AE =
Solution: Points A,
B,

D are
and

coplanar
if and 4 5
only if the

vectors

AB , AC

and AD are coplanar


and hence AB, AC and AD are linearly dependent Suppose BP : PE = l :1 and AP : PD = m :1. Then
(Theorem 5.19). We have

b + l (4c / 5) m (b + 3c )
=
AB = 4a - 2b - 2c l+1 4( m + 1)


AC = 4a + 4b - 2c Equating the coefficients of b and c on both sides we get


AD = - 2a - 2b + 4c 1 m
= (5.27)
l + 1 4( m + 1)
Now

4 -2 -2 2 -1 -1 4l 3m
and = (5.28)
5(l + 1) 4( m + 1)
4 4 - 2 = 8 2 2 -1
-2 -2 4 -1 -1 2 Therefore
= 8[2(4 - 1) + 1(4 - 1) - 1(- 2 + 2)] 4l 3
=
5(l + 1) l + 1
= 8(99) ¹ 0

15
4 l = 15 or l =
Hence AB, AC , AD are not linearly dependent. Hence 4
they are independent and non-coplanar.
BP : PE = 15 : 4 and so (D) is true.
Answers: (A), (D)
Substituting the value of l = 15/4 in Eq. (5.27) we get
4. In DABC, D and E are points on BC
C and AC, respec- 4 m
tively, such that BD = 3 DCC and AE = 4 EC. The lines =
19 4( m + 1)
AD and BE E intersect in P. Then
16 m + 16 = 19 m
(A) AP : PD = 16 : 3
(B) AP : PD = 15 : 3 m=
16
(C) BP : PE = 15 : 3 3
(D) BP : PE = 15 : 4 AP : PD = 16 : 3 and so (A) is true.
Answers: (A), (D)
Worked-Out Problems 351

Matrix-Match Type Questions


1. The position vectors
points A,
of the B, C and D are, Solution:


respectively, a, b, 2a + 3b and a - 2b. Match the items
(A) and (B) DE = AF = BC = c - b
of Column I with those of Column II.

AC + AD c + d
(C) AY = =
Column I Column II 2 2

c + d


(A) AC is (p) 3b - a (D) AX = b and AY = Þ XY

2 2
(B) DB is (q) 2a + 2b

c + d - b
= AY - AX =
(C) BC is (r) a + 3b 2


(D) AC + DB + BC equals (s) 2a + 8b Answer: (A) Æ (p), (B) Æ (p), (C) Æ (q), (D) Æ (r)

Solution: 3. Let i , j , k be unit vectors along positive x-, y- and

z-axes, respectively, with ‘O’ being the origin. Match


AC = (2a + 3b) - a = a + 3b the items of Column I with those of Column II.


DB = b - (a - 2b) = 3b - a

Column I Column II
BC = (2a + 3b) - b = 2a + 2b



AC + DB + BC = 2a + 8b i +
(A) The points j + k , i + 2 j + 3k (p) right
and 2 i - j + k form angled
Answer: (A) Æ (r), (B) Æ (p), (C) Æ (q), (D) Æ (s)
(B) The vectors
i + 2 j + 3k , 2 i + j + 3k (q) scalene
and - 3i - 3 j - 6k form
2. ABCD is a quadrilateral
and

ADEF

F is
a

parallelogram

5.49) with AF = BC . If AB = b, AC = c
(see
Figure (C) The vectors
-i - j - 4 k , - 3i + 3 j (r) isosceles
and AD = d, match the items of Column I with those and 4 i - 2 j + 4k form

of Column II.
7 j + 10
(D) The points k , - i + 6 j + 6k (s) equilateral
and - 4 i + 9 j + 6k constitute the
E vertices of a triangle which is

Solution:
D
(A) Let the given points be A, B and C, respectively.
F Therefore
d C


AB = j + 2k
c


BC = i - 3 j - 2k


A B CA = - i + 2 j
FIGURE 5.49 Matrix-match type question 2.


AB + BC + CA = 0 Þ the points A, B, C form a
triangle
Column I Column II


Also | AB | = |CA | = 12 + 22 Þ D ABC is isosceles.


(A) DE is equal to (p) c - b Answer: (A) Æ (r)


c+d (B) Let
a =
i + 2 j + 3k , b = 2 i + j + 3k and c = - 3i -
(B) BC equals (q)
2 3 j - 6k . Now

c+d-b
(C) If Y is the mid-point of DC

C then (r) a+b+c=0


AY is equal to 2

(D) If X is the mid-point of AB and

Y (s) c - d
is the mid-point of DCC then XY is
352 Chapter 5 Addition and Scalar Multiplication of Vectors


implies that a, b, c form a triangle. Also Column I Column II

| a | = | b | = 14 3
(A) The position vector of P is (p) (i + j )
2
Therefore a, b, c form an isosceles triangle.
1
Answer: (B) Æ (r) (B) The position vector of R is (q) (i + 5 j )
2
(C) Let
(C) The position vector of M is (r) 2 2(- i + j )

a = - i - j - 4k
2
(D) If the line OMM meets the (s) (i + 5 j )
b = - 3i + 3 j 3

PR in the point T,
diagonal T
then OT equals
and c = 4 i - 2 j + 4k


Solution:

Let i and j be the unit vectors along Ox


and Oy, respectively (Figure 5.50).
a+b+c=0 Now OP = 3 and xOP = 45° implies


implies that a, b, c form a triangle. Also OP = (3 cos 45°)i + (3 sin 45°) j

| a |2 + | b |2 = 18 + 18 = 36 = | c |2 3
= (i + j )
2
Therefore a, b and c form a right-angled isosceles
triangle. Answer: (A) Æ (p)
Answer: (C) Æ (p), (r) Again, xOR = 135° and OR = 4 implies that
(D) Let the given points be A, B and C, respectively,

4
OR = (- i + j )
so that 2


AB = - i - j - 4k = 2 2 (- i + j )

Answer: (B) Æ (r)


BC = - 3i + 3 j

The position vector of Q is given by


and CA = 4 i - 2 j + 4k

OP + PQ = OP + OR = (- i + 7 j )
AB + BC + CA = 0 Þ A, B, C form a triangle. 2
Also

y
| AB |2 = 18 = | BC |2 = 18

Q
and |CA | = 36

implies that DABC


C is right-angled isosceles. 3 M
Answer: (D) Æ (p), (r)
R 4

4. In the Cartesian plane, a man starts at origin and T P


4
walks a distance of 3 units in the North-East direction
3
and reaches a point P. From P, he walks a distance
45° 45°
of 4 units in the North-West direction to reach a
point Q. Construct the parallelogram OPQR with O x
PO and PQ as adjacent sides. Let M be the mid-point
of PQ. Match the items of Column I with those of
Column II.
FIGURE 5.50 Matrix-match type question 4.
Worked-Out Problems 353

Therefore Therefore

(3 / 2 )(i + j ) + (1/ 2 )(- i + 7 j )

1(OR) + 2(OP )
OM = OT =
2 3

2 i + 10 j i + 5 j (4 / 2 )(- i + j ) + 2[(3 / 2 )(i + j )]
= = =
2 2 2 3
Answer: (C) Æ (q) 2
Now, by Example 5.4, = (i + 5 j )
3
PT : TR = 1: 2 Answer: (D) Æ (s)

Comprehension-Type Questions

1. Passage: If a and b are the position vectors of two

2b + c
and AE =
points A and B respectively, then the position

vector 3
of the point P which divides the segment AB in the
ratio m : n(m + n ¹ 0) is Suppose AP : PE = l :1 and CP : PD = m :1. Therefore


na + mb l[(2b + c )/ 3]

m[(1/ 2)b] + c
= AP =
m+n l+1 m+1

Answer the following three questions. Since b and c are non-collinear
vectors, the corre-

(i) In DABC, D is the mid-point of AB and E is a point sponding coefficients of b and c on both sides are
C such that BE:EC = 1:2. The lines
on the side BC equal. Therefore
AEE and CD meet in P. Then, the ratio AP:PE = 2l m
= (5.29)
(A) 2 : 3 (B) 3 : 2 (C) 3 : 4 (D) 4 : 3 3(l + 1) 2( m + 1)
(ii) In the Question (i), the ratio CP : PD is l 1
(A) 4 :1 (B) 1: 4 (C) 3 :1 (D) 3 :1 and = (5.30)
3(l + 1) ( m + 1)
(iii) In DABC, a line is drawn parallel to the side BC
meeting the sides AB and AC C in M and N respec- Solving Eqs. (5.29) and (5.30) for l and m we get
tively. The lines BN
N and CM M intersect

in E. Then 3
the line AE
E divides the segment BC in the ratio m = 4 and l =
2
(A) 1: 2 (B) 3 :1
(C) 1:1 (D) 2 : 3 externally Hence AP : PE = 3 : 2 and CP : PD = 4 :1.
Answer: (i) Æ (B); (ii) Æ (A)
C (c )


(iii) Take A as origin and let AB = b, AC = c (see
Figure 5.52). Suppose the line AE E meets the side
BCC in D.
m 2
A
E
1
P 1
l 1
B (b ) M N

FIGURE 5.51 Comprehension-type question 1 part (i). E


(i) and (ii): Take A as origin and let AB = b


Solution:
and AC = c . Therefore

1
AD = b D
2
FIGURE 5.52 Comprehension-type question 1 part (iii).
354 Chapter 5 Addition and Scalar Multiplication of Vectors

Since MN
N is parallel to BC, it follows that (C) x = -1, y = -1, z = -1
AM AN (D) no real values of x, y, z exist
= = l (suppose)
AB AC (iii) If x(2 i - j + k ) + y(i + 2 j - 3k ) + z(3i + aj + 5k ) = 0

where x, y, z are scalars such that (x, y, z) ¹ (0, 0, 0)


so that AM = lb and AN = lc (l > 0). Therefore then the value of a is
from Theorem 5.29, the equation of the line BN
N is (A) 3 (B) -3 (C) -4 (D) 4

r = (1 - t )b + t(lc ) (5.31) Solution:
(i) Equating corresponding coefficients of both sides
and the equation of the line CM
M is
we get

r = (1 - s)c + s(lb) (5.32) x + 3 y - 4z = lx
From Eqs. (5.31) and (5.32) x - 3 y + 5z = ly

1 - t = sl and 1 - s = t l 3 x + y = lz

so that Therefore

1 (1 - l ) x + 3 y - 4z = 0
t= =s
1+ l x - (3 + l ) y + 5z = 0
There the position vector of E is 3x + y - lz = 0
l
(b + c )
l+1 and hence it has non-zero solution, if

Now, the equation of the line AE


E is 1- l 3 -4
1 -3 - l 5 = 0 (See Volume 1, Chapter 8)
l
r=x (b + c ) (5.33) 3 1 -l
l+1
and the equation of the line BC
C is In expanding the determinant, we have

r = (1 - y)b + yc (5.34) l (l + 1)2 = 0

Since D is a common point of the lines AE


E and BC, Therefore l = 0, -1. If l = 0, such scalars x, y, z with
from Eqs. (5.33) and (5.34), we have (x, y, z) ¹ (0, 0, 0) do not exist. Hence l = -1.
Answer: (A)
lx
1- y= =y (ii) From the given equation
l+1

- i + ( x - y) j + ( x + 2 y + z)k = 0
that y = 1/2. Hence the position vector
which implies
of D is (b + c )/ 2. Therefore the ratio BD:DC = 1:1.
Therefore
Answer: (iii) Æ (C)
-1 = 0

2. Passage: If i , j and k are unit vectors along positive x=y
directions of the coordinate axes, then every
vector
and x + 2y + z = 0
r can be represented as r = xi + yj + zk uniquely.
Answer the following three questions. which is impossible. Therefore no real values of

(i) If x( i + j + 3k x, y, z exist.
) + y(3i - 3 j + k ) + z(- 4 i + 5 j ) =
l( xi + yj + zk ) where at least one of x, y, z is not Answer: (D)
zero, then the number of distinct values of l is (iii) The given relation implies
(A) 1 (B) 2 (C) 3 (D) 0
2 x + y + 3z = 0
(ii) If x( j + k ) + y(- j + 2k ) + zk = i , then
- x + 2 y + az = 0
(A) x = 0, y = 1, z = -1
and x - 3 y + 5z = 0
(B) x = -1, y = 1, z = -1
Worked-Out Problems 355

Therefore a parallelogram bisects the angle between two


adjacent sides if either it is a rhombus or a square.
2 1 3 In any case the two adjacent sides must be of equal

- 1 2 a = 0 (Volume 1, Chapter 8) lengths. Hence a + b bisects angle between a and b

1 -3 5 if | a | = | b | .
Þ 2(10 + 3a) - 1(- 5 - a) + 3(3 - 2) = 0 Answer: (A)
(ii) Let
Þ 7a + 28 = 0

Þ a = -4 a
= xi + yj + zk
Answer: (C) |a |

3. Passage: Let r = a + sb and r = a + tc be lines passing
so that x2 + y2 + z2 = 1. Let b = 3i + 4 j . Therefore by

through the point a and parallel to the vectors b and Theorem 5.33, equation
of the bisector of the angle

c, respectively. Then the equations between a and b is

æ b c ö æ a b ö
r = a + tç + ÷ r = tç + ÷
è | b | | c |ø è | a | | b |ø

æ -b c ö æ 3i + 4 j ö
and r = a + tç + ÷ = t ç xi + yj + zk +
è | b | | c |ø è 5 ÷ø

are the angle bisectors of the two lines. Answer the


following three questions.
æ 3i + 4 j ö
(i) If a and b are non-collinear vectors, then the - i + j - k = t ç xi + yj + zk +

vector a + b bisects the angle between a and b, if è 5 ÷ø

(A) | a | = | b | for some scalar t. Hence

(B) angle between a and b is 0 or p
æ 3ö 1 3
tç x + ÷ø = - 1 Þ x = - -
(C) a, b form adjacent sides of a parallelogram è 5 t 5

(D) a, b form adjacent sides of a rectangle æ 4ö 1 4
tç y + ÷ = 1Þ y = -
(ii) The vector - i + j - k bisects angle between the è 5ø t 5

vectors a and 3i + 4 j . Then, unit vector along a is 1
tz = - 1 Þ z = -
i + j +k t
(A)
3

2 i - j + 2k
(B) x2 + y2 + z2 = 1
3
Þ (3t + 5)2 + (5 - 4t )2 + 25 = 25t 2
1
(C) (- 11i + 10 j - 2k ) 15
15 Þt=
2
1
(D) (11i + 10 j + 2k )
15
(iii) The position vectors A and B are,
of the points - 11 10 -2
respectively, i + 3 j - 2k and 3i + j - 2k . If O x= ,y= and z =
15 15 15
is the origin and the internal

bisector
of AOB
meets the line AB in P, then OP is equal to Answer: (C)


(A) i + j - k (B) 2(i + j - k ) (iii) |OA | = |OB | = 14 Þ D AOB is an isosceles triangle.
Hence P is the mid-point of AB. Therefore
i + j +k 2
(C) (D) (i + j + k )


3 3 OP = (OA + OB)
Solution: 2

= 2(i + j -k )
(i) a + b is a diagonal of the parallelogram for which

a and b are two adjacent sides. A diagonal of Answer: (B)
356 Chapter 5 Addition and Scalar Multiplication of Vectors

Assertion–Reasoning Type Questions



In the following set of questions, a Statement I is given 2. Statement I: If the vectors a, b and c are non-
and a corresponding Statement II is given just below it. coplanar,
then the points with
position vectors

Mark the correct answer as: a - 2b + 3c , 2a + 3b - 4c and - 7b + 10c are collinear.
(A) Both Statements I and II are true and Statement II Statement II:

points P, Q, R are collinear if


Three
is a correct explanation for Statement I and only if PQ = lQR for some scalar l.
(B) Both Statements I and II are true but Statement II
is not a correct explanation for Statement I Solution: Statement II is true according to
(C) Statement I is true and Statement II is false Definition 5.17. Let A, B and C be the three points in
(D) Statement I is false and Statement II is true Statement I so that


AB = a + 5b - 7c
1. Statement I: Let ABCD be a trapezium in which AB


and DC C are parallel sides. The diagonals AC C and BD and BC = - 2a - 10b + 14c
intersect in O. If M and N are the mid-points of AB


and CD, then M, O, N are collinear. = - 2 AB

Statement II: If a and b are non-collinear vectors Therefore A, B, C are collinear points.

(i.e., not
parallel), then every vector r in
the plane of Answer: (A)

a and b can be expressed as r = xa + yb where x and
y are scalars uniquely.
3. Statement I: Let a, b, c be non-coplanar
vectors.

Then the vectors 3a - 7b - 4c , 3a - 2b + c and
Solution: Statement

II is Theorem

5.15.
Take the

point

O as
origin and let OA = a and OB = b so that OC = xa a + b + 2c are linearly dependent.


and OD = yb, where x and y are scalars (Figure 5.53). Statement II: Three vectors are linearly dependent if
and only if one of them is a linear combination of the
D
N
C
other two vectors.

Solution: Proof of Statement II: Suppose a, b and c
O are linearly dependent. Therefore there exist scalars x, y,
z not all zero such that
a b
xa + yb + zc = 0
If z ¹ 0, then
FIGURE 5.53 Assertion–reasoning type question 1.
æ - x ö æ - yö
c =ç ÷a+ç ÷b
Since M and N are mid-points of AB and CD respectively è z ø è zø

a + b
so that c is a linear

combination of a and b. Conversely
OM =
2 suppose if xa + yb = c , then

xa + yb xa + yb + (- 1)c = 0
and ON =
2

and -1 ¹ 0. Hence a, b, c are linearly dependent. Hence


Since DC is parallel to AB we have Statement II is true.

Let the vectors in Statement
I be a , b and g respec-
xa - yb = l(b - a )
tively. Let g = xa + yb . That is,

Again, since a and b are non-collinear, by Statement II a + b + 2c = x(3a - 7b - 4c ) + y(3a - 2b + c )
x = -l = y On equating the corresponding coefficients both sides
Therefore we get

3x + 3 y = 1 (5.35)
(a + b )
ON = - l = - lOM
2 -7x - 2y = 1 (5.36)

M, O, N are collinear. -4x + y = 2 (5.37)


Answer: (A)
Summary 357

Solving Eqs. (5.35) and (5.37), we have


Hence by Statement II, a , b and g are linearly
1 2 dependent.
x=- and y=
3 3 Answer: (A)

which also satisfy Eq. (5.36). Therefore


1 2
g =- a+ b
3 3

Integer Answer Type Questions



1. If the vectors - 2 i + 3 j + yk and xi - 6 j + 2k are Therefore
collinear vectors, then the value of x – y is .
3(0 - 3 ) - 3(l - 3 ) + 3 ( 3 - 0) = 0
Solution: Since the vectors are collinear, there exists
scalar l such that That is

- 2 i + 3 j + yk = l( xi - 6 j + 2k ) - 3 3 - 3l + 3 3 + 3 = 0
Therefore lxl = -2, -6l = 3 and 2l = y. Now l = -1/2 Therefore l = 1.
implies that x = 4, y = -1. Therefore x - y = 5. Answer: 1
Answer: 5

4. Let a = i + lj + k and b = i + j + k . If | a + b | = | a | + | b |
2. A, B, C are three points
whose position vectors
are, then the value of l is
.
respectively, a - 2b + 3c , 2a + 3b - 4c and

- 7b +
10

c
Solution: Since
where a, b are non-coplanar vectors. If AC = mAB,
then the value of –m is .
|a + b| = |a | + |b|
Solution: We have

by Theorem 5.7 the vectors a and b are like vectors.


AC = mAB Therefore there exists positive scalar x such that

Þ - a - 5b + 7c = m(a + 5b - 7c )
a = xb

= - m(- a - 5b + 7c ) On equating the corresponding coefficients of both sides
Þ -m = 1 we get x = 1, x = l. Therefore l = 1.
Answer: 1 Answer: 1
5. Let A, B, C be the vertices of a triangle whose posi-
3. If the vectors 3i + 3 j +3k , i + k and 3i + 3 j + xk
are coplanar, then the value of x is . tion vectors are, respectively, a, b and c. M is the mid-
point of the side AB. If T is a point on the segment
Solution: Since the given vectors are coplanar, they MC C whose position vector is
are linearly dependent (Theorem 5.19). Hence

a+b+c
3 3 3 3
1 0 1 =0 (Theorem 5.26)
then the ratio CT : TM = l :1 where l is equal to .
3 3 l
Solution: Clearly T is the centroid of DABC. Hence
CT : TM = 2 :1.
Answer: 2

SUMMARY
5.1 Directed line segment: If A and B are distinct points, 5.2 Support of a

directed
line segment:

For the directed


then the ordered pair (A, B)

is called directed line line segment AB, the straight line AB is called support.
segment and is denoted by AB.
358 Chapter 5 Addition and Scalar Multiplication of Vectors

QUICK LOOK 5.7 Vector: Each equivalence class discussed in the


above theorem is called a vector. Usually we denote
Many directed line segments may have the same support.
vectors by a, b, c , r , p, etc.

QUICK LOOK
5.3 Same direction or opposite direction: Let AB and CD


be two directed line

Consider the line AC .


segments. (i) Vector a means it is an equivalence class of all


If the supports of AB and CD are parallel and both B equivalent directed line segments.

D lie in

and the
same

half
plane determined by the line
AC , then AB and CD said to have the same (ii) Vector a
can

be represented by any directed line


direction.

segment AB belonging
If B and

D lie on opposite half planes of AC , then AB

to the equivalence

class
a. For example, if AB belongs to a, then we
and CD said to have opposite directions.


write a

= AB . The magnitude of a is defined to


be | AB| .
B B


A A (iii) Two directed line segments AB and CD repre-
D sent the same vector if and only if they have the
C D
C
Same direction Same direction
same direction and same magnitude.

5.8 Zero vector or null vector:


The collection of all


B directed line segments PP, where P is any point from
A B
D Opposite A a vector is called zero vector and is denoted by 0.
C direction D

Opposite direction QUICK LOOK

Zero vector has no


direction and has zero magnitude.
QUICK LOOK We can consider 0 in any direction.


AB and BA are of opposite direction.

a: The support
5.9 Support
of

of any directed line
segment AB belonging to a is considered to be the
5.4 Magnitude or length: The distances AB

is called support of a.
magnitude of

the
directed line

segment
AB and

it
is
denoted by | AB| . Note that | AB| is equal to | BA| . 5.10 Unit vector: The equivalence class of all equivalent
directed line segments of magnitude 1 is called unit
5.5 Equivalent directed

line

segments: Two directed vector.


line segments AB and CD are said to be equivalent
if they have same direction and same magnitude.
5.11 Parallel or collinear vectors: Two vectors a and b
are said to be collinear vectors (parallel vectors)
if
5.6 Theorem: Let S be the set of all directed

line

segments.


any support of a is parallel to any support of b.
Define relation R on S as R = {( AB, CD)| AB and CD
are equivalent}
5.12 Like and unlike vectors: Two collinear vectors are
Then R is an equivalence relation in S, which divides said to be like or unlike vectors according as they
S into equivalence classes having the following three have the same direction or opposite directions.
properties.
The vectors a and b in the figure below are like
(1) Every directed line segment must belong to vectors.
exactly one equivalence classes.
(2) Any two directed line segments belonging to the
a
same equivalence class are equivalent (i.e., same
direction and same magnitude).
b
(3) Directed line segments belonging to different
equivalence classes are nott equivalent.
Note: For the above theorem see Chapter 1 of Volume 1.
Summary 359


Also a and b in the following figure are unlike 5.17 Theorem: Let a and b be two vectors. Then
vectors.
(i) | a + b | £ | a | + | b | and equality holds if and only

if a and b are like vectors.
a
(ii) || a | - | b || £ | a - b | and equality holds if and only

if a and b are like vectors and | a | ³ | b | .

5.18 Multiplication of a vector with a scalar: Let a be
any vector and l any scalar (i.e., real number). Then

5.13 Unit vector in the direction of a: If a non-zero (i) la is equal to 0 if either l = 0 or a = 0.

vector a and a unit vector e are like vectors, then

e is said to be unit vector in the direction of a. (ii) If l > 0 and a ¹ 0, then la is defined to be the

vector in the direction of a with magnitude

5.14 Coplanar and non-coplanar vectors: A set of l |a | .
vectors (or collection of vectors) are said to be
(iii) If l < 0 and a ¹ 0, then la is defined to be
coplanar vectors if their supports are in the same
the vector in the opposite direction of a with
plane or parallel to the same plane, Vectors which
magnitude (-l ) | a | .
are not coplanarr are called non-coplanar.

QUICK LOOK
QUICK LOOK

(i) | la | = | l || a |
(i) Any two
vectors

are coplanar

because
if a = OA


and b = OB, then the lines OA and OB deter- (ii) a and la are collinear vectors.
mine a plane.


(ii) If a = OA, b = OB and c = OC , then a, b, c are
coplanar if and only if the four points O, A, B and 5.19 Theorem: Let a, b be vectors and m, n be scalars.
C are coplanar. Hence coplanarity or non-copla- Then the following hold.
narity arises only when there are more than two
(i) m(na ) = n(ma ) = (mn)a
vectors.
(ii) m(a + b) = ma + mb

(iii) (m + n)a = ma + na

5.15 Vector addition: If a = OA

and b = AB, then a + b (iv) m(- a ) = (- m)a = -(ma )



is defined as the vector OB. a + (- b) is denoted by

(v) 1a = a
a - b.

B QUICK LOOK

If a ¹ 0, then a /| a | is a unit vector in the direction of
b
a and -( a /| a |) is a unit vector in the opposite direc-

tion of a.

O A
5.20 Position vector of a point: Let ‘O’ be a fixed point in
the space, called origin of reference.

If A is any point
in the space, then the vector OA is called the position
5.16 Theorem: Let a, b and c be three vectors. Then vector of the point A with reference to the origin.

(i) a + b = b + a (Commutative law)
QUICK LOOK
(ii) (a + b) + c = a + (b + c ) (Associative law)

(iii) a + 0 = 0 + a = a for all vectors a If O is the origin and a is any vector,

then there exists


point A in the space such that OA = a.
(iv) a + (- a ) = - a + a = 0 (- a is called additive inverse

of a)

(v) a + x = b Û x = b - a
360 Chapter 5 Addition and Scalar Multiplication of Vectors


5.21 Theorem: If a and b are the position

vectors
of the r = xa + yb
in one and only one way. In particular,

points A and B, respectively, then AB = b - a. if i and j are unit vectors along positive directions

of x and y axes, respectively,
and r is any vector in

5.22 Division formula (Section formula): Let a and b the plane, then r = xi + yj , where
(x,

y) are the
be the position vectors of two points A and B, coordinates of the point P such that OP = r .
respectively. Then the position vector of the point
which divides AB in the ratio m : n is QUICK LOOK

na + mb In the Cartesian

plane,
point P (x, y) is identified with
the vector OP = xi + yj where ‘O’ is the origin of the
m+n
coordinate axes.
The same formula is valid for both internal and
external division. The position vector of the mid-
point is
5.26 Space representation: Let a, b and c be three non-

coplanar vectors and r be any vector. Then there
a+b
exists unique
triad (x, y, z) of scalars
such that
2
r = xa + yb + zc . In particular, let i , j and k be
unit vectors along the positive directions of the x, y
m n
and z axes respectively. Let r be any vector. Then
triad (x, y, z) of scalars such that
there exist unique

r = xi + yj + zk and (x,

y, z) are coordinates of the


5.23 Results:
point P such that r = OP. Hence in the space, we
(i) the position vector of the centroid G of DABC
C is identify vector xi + yj + zk as the point (x, y, z).











a+b+c 5.27 Linear combination: If a1 , a2 , a3 , … , an are vectors




3 and
x1 , x2 , …,



xn are scalars, then the vector x1 a1 +
x

a

2

2
+  +

x a

n n is called a linear combination of
where a, b and c are, respectively, the posi-
a1 , a2 , …, an .
tion vectors of A, B and C.
(ii) The segments joining the vertices A, B, C and
QUICK LOOK
D of a tetrahedron ABCD to the centroids of
the opposite faces are concurrent. This point is
(i) If a and b are two non-collinear
vectors then every
denoted by G and is called the centroid or centroid
vector in the plane of a and b can be written as a
of the tetrahedron. G divides the segment joining
linear combination of a and b uniquely.
the vertex to the centoid of opposite face in the
ratio 3:1 and its position vector is (ii) Every vector can be uniquely represented as a
linear combination of three non-coplanar vectors.
1
(a + b + c + d )
4

5.28 Linear





independence and dependence: Vectors a1 ,
where a, b, c and d are position vectors of A, a2 , …, an are said to be linearly dependent, if there
B, C and D, respectively. exist scalars x1 , x2 , … , xn , not
all zero


(that is at


least


one xi ¹ 0 ) such that x1 a1 + x2 a2 +  + xn an = 0.
(iii) If a, b and c are the position vectors of A, B
and C, respectively, of DABC C with respect to Vectors which are not linearly dependent are called
circumcentre as origin, then the position vector linearly independent.

of the orthocentre of DABC C is a + b + c .
QUICK LOOK

5.24 Theorem: Three points with position vectors a, b






Vectors a1 , a2 , … , an are linearly independent if and


and c are collinear if and only if there exist
scalars
only if
x, y, and z not all zero such that xa + yb + zc = 0
and x + y + z = 0.









x1 a1 + x2 a2 +  + xn an = 0

5.25 Plane representation: If a and b are two non- Þ x1 = x2 =  = xn = 0

collinear vectors, then every
vector r in the plane

determined by a and b can be represented as
Summary 361

5.29 Theorem: Vectors a1 , a2 , …, an are linearly depen- 5.33 Plane equations:


dent if and only if one of them is a linear combina- (i) The vector equation of a plane passing through

tion of the remaining. a point whose position
vector is a and parallel

to two vectors b and c is
5.30 (i) Two vectors are non-collinear if and only if
they are linearly independent. r = a + tb + sc , t and s Î 
(ii) Three vectors are non-coplanar if and only if (ii) Equation of the plane passing through two

they are linearly independent. points a and b and parallel to a vector c is

QUICK LOOK r = (1 - t )a + tb + sc , t and s Î 

(i) Two vectors are collinear if and only if they are (iii) Equation of the plane passing through three

linearly dependent. points a, b, c is
(ii) Three vectors are coplanar if and only if they are
r = (1 - t - s)a + tb + sc , t and s Î 
dependent.
(iii) In the three-dimensional space the maximum 5.34 Theorem: Three points A, B and C with position
number of linearly independent vectors (the
vectors a, b and c, respectively, are collinear if and
vectors may change) is three. That is, more than only if there exist scalars x, y, z not all zero such that
three vectors are linearly dependent.
x+ y+z=0
(iv) If a finite set of vectors contains zero vector then

they are linearly dependent. and xa + yb + zc = 0


5.35 Four points
A, B, C and D with position
vectors a, b, c and d are coplanar if and only if
5.31 Theorem
(Important):
a = a1i + a2 j + a3k
Let ,b=

b1i + b2 j + b3k and c = c1i + c2 j + c3k . Then a, b, c there exist scalars x, y, z and u not all zero such that
are linear dependent (hence coplanar) if and only if the x+ y+z+u=0

a1 a2 a3 and xa + yb + zc + ud = 0
det b1 b2 b3 = 0


c1 c2 c3 5.36

Let a = OA and
b = OB. Then the measure of AOB which lies

Note: The above condition also holds good even if a, b, c between
0 and p is defined to be
the angle between
are represented as linear combination of any


three non- a and b and is denoted by (a, b).

coplanar vectors,
that is by taking some l , m and n in
place of i , j and k, respectively. QUICK LOOK


5.32 Parametric vector equation of a line: 0 £ (a, b) £ p . The left equality holds if and only if a, b
are like vectors and right equality holds if and only if
(i) The equation of the line passing through point

whose position vector is a and parallel to a a, b are unlike vectors.

vector b is r = a + tb, t Î .
(ii) Vector equation of the line passing through
5.37 Note: Let a and b be two vectors. Then
two points a and b is r = (1 - t )a + tb, t Î .

(1) (a, b) = (b, a ) = (- a, - b) = (- b, - a )
b
(2) (a, - b) = p - (a, b) = (- a, b)

A P (3) (a, b) = ( xa, yb) for all positive x and y

(4) (- xa, yb) = p -( xa, yb) = ( xc , - yb) for all posi-
tive x and y
a r

5.38 If (a, b) = 90°, then a and b are called perpendic-
ular or orthogonal vectors.
362 Chapter 5 Addition and Scalar Multiplication of Vectors

and left-handed systems: Let a = OA, b = OB,


5.39 Right- QUICK LOOK
c = OC . When observed

point C, if the angle


from
the

of rotation of OA towards OB in anticlock sense
(i) If r = xi + yj + zk , then | r | = x2 + y2 + z2 .
does not exceed p p, then (a, b, c ) is called right-
handed system. System which is not right handed is (ii) Let P = (x1, y1, z1) and Q = (x2, y2, z2) so that
called left-handed system.


OP = x1i + y1 j + z1k


B and OQ = x2 i + y2 j + z2 k
Hence
b


PQ = ( x2 - x1 )i + ( y2 - y1 ) j + (z2 - z1 )k
<


and | PQ| = ( x2 - x1 )2 + ( y2 - y1 )2 + (z2 - z1 )2
O

a
c A
C 5.41 Direction cosines (DCS) and direction

ratios


(DRS) of a vector : Let X ¢OX , Y ¢OY and Z ¢OZ
be mutually perpendicular
lines called coordinate
Note:
axes. Let

i , j , k be

unit vectors
in the directions of
(1) In a system (a, b, c ) if two vectors are inter- OX , OY and OZ such that (i , j , k ) is a right-handed

changed then the system will change. system.Let a be any vector,and leta = (a, i ), b = (a , j )

(2) In a system if a vector is replaced by its additive and g = (a, k ) (i.e., the angles made by a with i , j
inverse, the system will change. and k respectively). Then the ordered triad

(cosa , cos b , cos g ) is called direction cosines of a.
(3) If the vectors are cyclically permuted, the
system does not change.
5.42 If l, m, n are DCS of a w.r.t (i , j , k ), then l2 + m2 +
n2 = 1.
5.40 Components of a vector: Let (a, b, c ) be a right-

handed system of non-coplanar vectors. If r is any

vector then r can be expressed as r = xa + yb + zc , QUICK LOOK

and x, y, z are called components of r w.r.t. the
(i) If (l, m, n) are DCS of a, then li + mj + nk is a
system (a, b, c ). In particular, if r = xi + yj + zk
unit vector parallel to a. In a sense unit vector
then x, y, z are x-component, y-component and

z-component of r, respectively. parallel to a given the DCS of a.

(ii) If (l, m, n) are DCS of a vector a and l ¹ 0, then

(ll, lm
l , ln
l ) are called DRS of a.

EXERCISES
Single Correct Choice Type Questions


1. In

a regular


hexagon

F vector AB + AC +
ABCDEF, 3. Let A and B be points
whose position
vectors are,
AD + AE + AF is equal to respectively, i + 3 j - 2k and 3i + j - 2k . The bisector

of the angle AOB (‘O’ is the origin) meets


(A) 2AD (B) 3AD (C) 4AD (D) 6AD

the segment
AB internally in C. Then the vector OC is equal to

2. ABCD is a quadrilateral and not a parallelogram, (A) i + j + k (B) 2(i + j - k )
described in this order. Let P, Q, R and S be the mid-
(C) i + j - k (D) 4(i + j + k )
points of the sides AB, BC, CD and AD, respectively.
Then PQRS is a
4. Let a, b, c be non-coplanar vectors. If
(A) parallelogram (B) rectangle
a + 3b + 4c = x(a + 5b - 2c ) + y(a - 2b + 3c )
(C) square (D) rhombus

+ z(6a + 14b + 4c )
Exercises 363

then (C) form an isosceles triangle


(A) x = -3, y = 2, z = 1 (B) x = 3, y = -2, z = 1 (D) non-collinear
(C) x = -3, y = -2, z = 1 (D) x = 3, y = 2, z = 1
11. OXYZ
Z is a rectangular coordinate system. Keeping


5. a is a vector in the Cartesian plane (in which OX and Z-axis fixed, the X
X- and Y
Y-axes are rotates in their

OY are coordinate axes) has components 2p 2 and 1. plane through angle 45° in the anti-clockwise sense.
The axes are rotated in the same plane about origin, If the components of a vector with respect new
through the angle 45° in the counterclock sense. The system are 2 2 , 3 2 and 4, then the components of
the same vector with respect to the original systems
components a with respect to the new system of axes
2 + 1 and 1. Then the value of p is
are 2p are
1 -1 (A) 5, -1, 4 (B) -1, 5, 4
(A) 0 (B) -1 or (C) or 1 (D) 1 or -1
3 3 (C) - 1, - 5, 4 2 (D) -1, 5, 4 2


A,
6. Let the points B and C be
represented
by the
12. In DABC, AB = 2 i + 4 j + 4k and AC = 2 i + 2 j + k .
vectors i + j , - 4 i + 2 j and 2 i + 3 j , respectively, in
Then the length of the median through A is
the Cartesian plane. Then the length of the median
through B of DABC C is 77
(A) (B) 5 (C) 5 2 (D) 10 2
5 2 11 11 2
(A) (B) (C) (D)
2 5 3 2
13. a, b, c are non-zero vectors such that they
are pair-

7. ABCD is a tetrahedron and O is a point in side the wise non-collinear.
It is given that a + 2b is collinear

tetrahedron. The lines AO, BO, CO and DO meet with c and b + 3c is collinear with a. Then the vector

the opposite faces in P, Q, R and S, respectively. a + 2b + 3c is
Then
(A) equal to 0 (B) parallel to b
(A)
AP BQ CR DS
+ + + =1
(C) parallel to a (D) parallel to c
OP OQ OR OS
OP OQ OR OS
+ + + =1 a , b , c be non-coplanar
14. Let vectors and x = 2a +
AP BQ CR DS
3b - c , y = a - 2b + 2c z = - 2a + b - 2c . Then the

AO BO CO DO vector 3a - b + 2c in terms of x, y and z is
+ + + =1
OP OQ OR OS (A) 5 x + 2 y + 3z (B) 3 x + 5 y + 2z
OP OQ OR OS
(D) + + + =1 (C) 2 x + 5 y + 3z (D) 5 x + 3 y + 2z
OA OB OC OD

15. Let a, b and c be three non-coplanar
vectors and

8. The line passing through the point 2a + 3b and
p = 5a + 6b + 7c , q = 7a - 8b + 9c and r = 3a + 20b +
parallel to the vector c cuts the plane
5c. If r = xp + yq where x and y are scalars, then
(A) x = 1 = y (B) x + y = 1
r = a - b + t (a + b - c ) s (a + c - b )
3
where t and s are scalars in the point whose position (C) x = , y = 1 (D) x = 2, y = -1
2
vector is
16. A, B, C and D are four points
(A) 2a + 3b + 4c (B) 2a - 3b + 4c whose
position
vectors

are,
respectively,
+ 5 j +

4
i

k , - i - k , 5i + 9 j + 4k and
(C) 2a - 3b - 4c (D) 2a + 3b - 4c - i + k. If AB = xAC + yAD, then
-2 2
9. P is the mid-point of the side AD of the parallelo- (A) x = ,y=
gram ABCD. The line BP P meets the diagonal AC C in 3 3
Q and the line CD in R. Then RQ : QB is equal to 2 -2
(B) x = , y =
(A) 2 :1 (B) 1: 2 (C) 3 :1 (D) 1: 3 3 3
3 2
10. The three points i - 2 j + 3k , 2 i + 3 j - 4k and - 7 j + 10k (C) x = , y =
2 3
(A) are collinear
(D) real values of x and y do not exist
(B) form the vertices of a right-angled triangle
364 Chapter 5 Addition and Scalar Multiplication of Vectors

17. Let A and B be two points whose position vectors 19. ABCD is a quadrilateral. P, Q, R and S are the mid-
are 5i and 5 j , respectively. A point P divides the points of the sides AB, BC, CD and AD, respectively.
line joining A and B in the ratio AP : PB = l :1. If the The lines PR and QS intersect in E. If ‘O’ is any point
position vector of P has magnitude less than or equal other than

any

point

mentioned

above, then the


to 37, then vector OA + OB + OC + OD is equal to


-1 1 -1 (A) 8OE (B) 6OE
(A) £l£ (B) l £ - 6 or l ³


6 6 6
(C) 4OE (D) 3OE
(C) l ³ 0 (D) 0 < l < 1
20. ABCDE
E is pentagon in which AB, CD are parallel
18. In DABC, E is the mid-point of the median AD. The and AE, BC C are parallel. The lines BE
E and AD inter-
line BE
E meets the side AC
C in F
F. Then AC
C equals sect in X
X. Then the ratio AX : XD is equal to
(A) 3 AF (B) 2 AF (A) 2 : 5 (B) 5 : 2 (C) 2 : 3 (D) 3:2
3
(C) 3 CF (D) CF
2

ANSWERS
Single Correct Choice Type Questions
1. (B) 11. (B)
2. (A) 12. (A)
3. (B) 13. (A)
4. (C) 14. (C)
5. (C) 15. (D)
6. (D) 16. (D)
7. (B) 17. (B)
8. (D) 18. (A)
9. (A) 19. (C)
10. (A) 20. (A)
Multiplication
of Vectors 6
Contents
6.1 Scalar or Dot Product
6.2 Vector Equations of a
Plane and a Sphere
6.3 Vector or Cross
Product
6.4 Vector Areas
6.5 Scalar Triple Product
6.6 Vector Triple Product
a b and Multiple Products
Multiplication of Vectors

Worked-Out Problems
Summary
b Exercises
Answers
n

q
Vector multiplication may
b a a concern any of the following
articles:
= (a b) r Dot product: Also known
as the “scalar product,”
an operation which takes
two vectors and returns a
scalar quantity.
r Cross product: Also
known as the “vector
product,” a binary opera-
tion on two vectors that
results in another vector.
r Triple products: Products
involving three vectors.
r Multiple cross products:
Products involving more
than three vectors.
366 Chapter 6 Multiplication of Vectors

In the previous chapter, we have discussed the operations of addition of vectors and multiplication of vectors by scalars.
In this chapter, we consider two more important operations on vectors, namely, scalar product (or dot product) and
vector product (or cross product) and discuss certain applications and properties of these. These will be used to derive
vector equations of a plane in several forms.

6.1 | Scalar or Dot Product


In this section, we introduce the concept of scalar product or dot product of two vectors and present a geometrical
interpretation which will be used in deriving the vector equation of a plane in the normal form and the orthogonal
projection of a vector on another vector. Let us begin with the following.

DEFINITION 6.1 Let a and b be vectors. Then we define

a ×b = 0 if a = 0 or b = 0

and a ×b = | a || b | cosq if a 0 and b 0

where q is the angle between the vectors a and b . a ×b is called the dot product of a and b, just

because a dot ××is used here. Since a ×b is defined as a scalar, it is also called the scalar product

of a and b.

For any two vectors a, b let us recall that the angle between a and b is denoted by (a, b) (see Definition 5.22,
Chapter 5) and we note that

( a, b) = p (a, b) = (a, b)

and (a, b) = (b, a )

In this notation, we have



a ×b = | a || b |cos(a, b)

= | b || a |cos(b, a )

= b ×a

for any non-zero vectors a and b. If either of a or b is a zero vector, then clearly by Definition 6.1 we have a ×b = 0 = b ×a

and therefore, for any two vectors a and b,

a ×b = b ×a
That is, the scalar product operation is commutative. However, we cannot talk on the associativity of this operation,

since a ×b is scalar and hence (a ×b)×c is not defined whereas (a ×b) c has meaning according to the concept of multi-
plication of a vector with a scalar.

The following are easy verifications and listed for the sake of ready reference on the angle (a, b).

QUICK LOOK 1


The following hold for any non-zero vectors a and b. 3. (a, b) = 90° a ×b = 0

1. 0 (a, b) 180° 4. 90° < (a, b) 180° cos(a, b) < 0 a ×b < 0

2. 0 (a, b) < 90° cos(a, b) > 0 a ×b > 0

Before going for the various properties of the scalar product of two vectors, we will first give a geometrical
interpretation.
6.1 Scalar or Dot Product 367


DEFINITION 6.2
a

and
Let

such that a = OA and


b be non-zero vectors and O, A and B be points in the space
b = OB. Let Q be the foot of the perpendicular from B to the line OA. (Figure 6.1). Then

B
B

®
® b
b

q
q

FIGURE 6.1 Definition 6.2.



1. The scalar component t (or, simply, the componentt or the projection) of b along a is defined as OQ if (a, b) £ 90° and

as -OQ if (a, b) > 90°.


2. The vector componentt (or the orthogonal projection) of b on a is defined as the vector OQ.

QUICK LOOK 2


1. The component of b on a is a scalar and the orthog-

B, C and D be coplanar points, a = AB and



A,
3. Let
onal projection of b on a is a vector. b = CD

. If the perpendiculars

from C and D to the


line AB meet AB at P and Q, respectively, then
2. If e is a unit vector and b is any vector, then (b × e ) e


PQ is the orthogonal projection of b on a. See
is the orthogonal projection of b on e and, for any
Figure 6.3.
non-zero vector a,

æ aö a ® D
b ×
çè | a | ÷ø | a | b

C
is the orthogonal projection of b on a. See Figure 6.2.

®
b

FIGURE 6.3
q

4. If b is perpendicular
to a, then the orthogonal

projection of b on a is the zero vector.
B
5. If b is parallel to a, then the orthogonal projection

of b on a is b.
®
b

FIGURE 6.2
368 Chapter 6 Multiplication of Vectors


T H E O R E M 6.1 Let a and b be non-zero non-parallel vectors. Then

a ×b
1. the projection (or the scalar component) of b on a is .
|a |

b ×a
2. the orthogonal projection (or the vector component) of b on a is 2 a.
|a |


PROOF

O, A and B such that a = OA and b = OB. Let AOB = q. Draw perpendicular BP


Choose points
to meet OA at P. Then OP P or OP P is the projection of b on a, according as q 90° or q > 90° as
shown in Figure 6.4. Then, if q 90°,


a ×b = | a || b | cos q = | a ||OB| cos q = | a | OP

and hence

a ×b
OP =
|a |

If q > 90°, then



a ×b = | a || b | cos q

= | a || b | ( cos(180° q ))

= | a | OB cos(180° q )

= | a | OP

and hence

a ×b
OP =
|a |

Thus (a×b)/| a | is the projection of b on a.

B
B

b b

q
q
O a P A P O A
a

FIGURE 6.4 Theorem 6.1. ■


In the above, if e is the unit vector, in the direction of a, that is,

a
e=
|a |

then

a a b ×a
OP = (b ×e )e = b × ÷ = a
|a| |a| | a |2
6.1 Scalar or Dot Product 369

QUICK LOOK 3

From Figure 6.4(b), we have b ×a
b a

(b ×a )a | a |2
PB = OB OP = b
| a |2
is the orthogonal projection of b in the direction

perpendicular to a in the AOB plane.
and therefore


T H E O R E M 6.2 The following hold for any vectors a and b and scalars a and b.

1. (aa )×b = a (a ×b) = a ×(ab)

2. (aa )×(bb) = ab (a ×b)

3. ( a )×b = (a ×b) = a ×( b)

4. ( a )×( b) = a ×b

PROOF These are trivial if a = 0 or b = 0 or a = 0 or b = 0. Therefore, we can assume that a and b are

vectors and a and b are non-zero scalars. Recall that (a, b) denotes the angle between
non-zero

a and b and that

( a , b ) = p ( a , b ) = (a , b )

If a > 0, then

(aa )×b = |aa || b | cos(aa, b)

= a | a || b | cos(a, b)

= a (a ×b)

If a < 0, then

(aa ) ×b = |aa || b | cos(a a, b)

= |a || a || b | cos[p ( a a, b)]

= | a || a || b | cos[p (a, b)] (since a > 0)

= ( a ) | a || b | [ cos(a, b)]

= a | a || b | cos(a, b)

= a (a ×b)


(aa ) ×( bb) = a [a ×(bb)]

= a [(bb)×a ]

= a b (b ×a )

= a b (a ×b)

(3) and (4) follow from (1) by taking a = 1 and/or b = 1. ■


370 Chapter 6 Multiplication of Vectors

QUICK LOOK 4


1. For any vectors a and b, 2. If a and b are like vectors, then (a, b) = 0° and

hence cos(a, b) = 1, so that a ×b = | a || b |.
cos(a, b) = 0 (a, b) = 90°
3. If a and b are unlike vectors, then (a, b ) = 180° and

Therefore hence cos(a, b) = 1, so that a ×b = | a || b |.

4. a ×a = | a || a | = | a |2. Here afterwards, a2 means a ×a.
a ×b = 0 a = 0 or b = 0 or a b


T H E O R E M 6.3 The following hold for any vectors a, b and c.

1. a ×(b + c ) = a ×b + a ×c

2. (a + b)×c = a ×c + b ×c

PROOF We prove this in the case when a, b and c are coplanar vectors and skip the general case. Let

us suppose that a, b and c are non-zero coplanar vectors and O, A, B and C are coplanar points
such that


a = OA, b = OB and c = BC
Then


b + c = OB + BC = OC


Draw perpendiculars BP P and CQ from B and C to the line OA to meet at P and Q respectively.

Let e be the unit vector in the direction of a; that is,

C
c

b +c
b

O Q a P A

C
b
b +c

O P a Q A

FIGURE 6.5 Theorem 6.3.


6.1 Scalar or Dot Product 371

1
e= a
|a|


OP = the orthogonal projection of b on a


PQ = the orthogonal projection of c on a


and OQ = the orthogonal projection of b + c on a
and therefore, by Quick Look 2 [part (2)], we have


OP = (b ×e )e


PQ = (c ×e )e


and OQ = [(b + c ) ×e ]e


[(b + c )×e ]e = OQ


= OP + PQ

= (b ×e )e + (c ×e )e

= [(b ×e ) + (c ×e )]e

Since e is a unit vector, it follows that

(b + c ) ×e = b ×e + c ×e

Now,

a ×(b + c ) = (b + c ) ×a

= (b + c )×(| a | e )

= | a | [(b + c ) ×e ]

= |aa | (b ×e + c ×e )

= | a | (b ×e ) + | a | (c ×e )

= b ×(| a | e ) + c ×(| a | e )

= b ×a + c ×a

= a ×b + a ×c
Also,

(a + b) ×c = c ×(a + b)

= c ×a + c ×b

= a ×c + b ×c ■


C O R O L L A R Y 6.1 For any vectors a1 , a2 , …, an , b1 , b2 , …, bm,
n
m
ai ÷× bj ÷ = (ai ×bj )
i =1 j =1 i, j
372 Chapter 6 Multiplication of Vectors


C O R O L L A R Y 6.2 For any vectors a, b and c,

a ×(b c ) = a ×b a ×c

and (a b) ×c = (a ×c ) (b ×c )


C O R O L L A R Y 6.3 For any non-zero vectors a, b and c,

a ×c = b ×c a = b

or a b is perpendicular to c
PROOF We have

a ×c = b ×c

(a b)×c = a ×c b ×c = 0

a b = 0 or a b is perpendicular to c ■

The following are direct consequences of the above and of the facts that | a |2 = a ×a and a ×b = b ×a for any vectors

a and b.


C O R O L L A R Y 6.4 The following hold for any vectors a, b and c.

1. | a + b |2 = | a |2 + | b |2 + 2a ×b

2. | a b |2 = | a |2 + | b |2 2a ×b

3. (a + b)×(a b) = | a |2 | b |2

4. | a + b + c |2 = | a |2 + | b |2 + | c |2 + 2(a ×b + b ×c + c ×a )


T H E O R E M 6.4 The following hold for any vectors a and b.

1. a ×a 0

2. a ×a = 0 a = 0

3. | a ×b | | a || b |

4. | a ×b | = | a || b | a = 0 or b = 0 or a and b are parallel

5. | a + b | | a | + | b |

6. | a + b | = | a | + | b | a = 0 or b = 0 or a and b are like vectors

7. || a | | b || | a b | | a | + | b |
PROOF (1) and (2) follow from the fact that

a ×a = | a |2

3. We have

| a ×b | = || a || b | cosq |

= | a || b ||cosq |

| a || b |

since |cosq | 1.
6.1 Scalar or Dot Product 373


4. If a = 0 or b = 0, then

| a ×b | = 0 = | a || b |

Now, for any non-zero vectors a and b,

| a ×b | = | a || b | |cosq | = 1 q = 0 or p

where q is the angle between a and b.
5. We have

| a + b |2 = | a |2 + | b |2 + 2a ×b

| a |2 + | b |2 + 2 | a || b |

= (| a | + | b |)2

and hence | a + b | | a | + | b |.

6. If q is the angle between a and b, then

|a + b| = |a | + |b|

a ×b = | a || b |

| a || b | (cosq 1) = 0

a = 0 orr b = 0 or cosq = 1

a = 0 or b = 0 or q = 0°

a = 0 or b = 0 or a and b are like vectors
7. Consider

|a | = |a b + b| |a b| + |b|

and hence

|a | |b| |a b|

Similarly,

|b| |a | |b a | = |a b|

Therefore

|| a | | b || | a b| |a | + | b| = |a | + |b| ■

Next, we express the dot product of given two vectors in terms of the components on the unit vectors along the
coordinate

axes

with respect to a given rectangular coordinate system OXYZ. If i , j and k be unit vectors along
OX , OY and OZ, respectively, then, since cos 0° = 1 and cos 90° = 0, we have

i ×i = 1 = j ×j = k ×k

and i ×j = 0 = j ×k = k ×i

Recall that (i , j , k ) is called a triad of orthogonal unit vectors.
374 Chapter 6 Multiplication of Vectors


T H E O R E M 6.5 Let a be a vector and (i , j , k ) be a triad of orthogonal unit vectors. Then

a = (a ×i )i + (a ×j ) j + (a ×k )k
PROOF We have learnt earlier that there exists a triad (a1, a2, a3) of real numbers such that

a = a1i + a2 j + a3k

Now,

a ×i = (a1i + a2 j + a3k )×i

= a1 (i ×i ) + a2 ( j ×i ) + a3 (k ×i )
= a1

and, similarly a ×j = a2 and a ×k = a3 . Thus

a = (a ×i )i + (a ×j ) j + (a ×k )k ■


T H E O R E M 6.6 Let (i , j , k ) be a triad of orthogonal unit vectors and

a = a1i + a2 j + a3k

and b = b1i + b2 j + b3k

Then the dot product of a and b is given by

a ×b = a1b1 + a2 b2 + a3b3

and in particular,

| a | = a12 + a22 + a32

PROOF From Theorem 6.5, we get



a ×b = a ×(b1i + b2 j + b3k )

= b1 (a ×i ) + b2 (a ×j ) + b3 (a ×k )
= b1a1 + b2 a2 + b3a3
= a1b1 + a2 b2 + a3b3

Also, we have

| a |2 = a ×a = a12 + a22 + a32

and hence

| a | = a12 + a22 + a32 ■

C O R O L L A R Y 6.5 For any triads of real numbers (a1, a2, a3) and (b1, b2, b3),
(C A U C H Y –
SCHWARTZ (a1b1 + a2 b2 + a3b3 )2 (a12 + a22 + a32 ) (b12 + b22 + b32 )
INEQUALITY)
a1, a2, a3 are proportional to b1, b2, b3.

PROOF This follows
from Theorem
6.6 and Theorem 6.4 [parts (3) and (4)] by taking a = a1i + a2 j + a3k
and b = b1i + b2 j + b3k . ■
6.1 Scalar or Dot Product 375


T H E O R E M 6.7 Let (i , j , k ) be a triad of orthogonal unit vectors and

a = a1i + a2 j + a3k

and b = b1i + b2 j + b3k

be two non-zero vectors. Then the angle between a and b is given by

a ×b a1b1 + a2 b2 + a3b3
÷ = Cos
1 1
Cos ÷
| a || b | a1 + a22 + a32 b12 + b22 + b32
2


PROOF If q is angle between a and b, then

a ×b = | a || b | cosq

and hence

a ×b a1b1 + a2 b2 + a3b3
cos q = =
| a || b | a1 + a22 + a32 b12 + b22 + b32
2

so that

a1b1 + a2 b2 + a3b3
q = Cos 1 ÷
a + a22 + a32 b12 + b22 + b32
2
1 ■


C O R O L L A R Y 6.6 Let a = a1i + a2 j + a3k and b = b1i + b2 j + b3k be two non-zero vectors. Then

1. a and b are perpendicular to each other if and only if a1b1 + a2b2 + a3b3 = 0.

2. a and b are parallel if and only if there exists a scalar t such that a1 = tb1, a2 = tb2 and a3 = tb3.

Example 6.1

Find
the
dot product of the vectors 6 i + 2 j + 3k and where q = (a, b), the angle between a and b. Now,
2 i 9 j + 6k and the angle between them.
a ×b = 6 2 + 2 ( 9) + 3 6 = 12
Solution: Let
Therefore

a = 6 i + 2 j + 3k
a ×b
q = Cos
1

and b = 2 i 9 j + 6k | a || b | ÷
Then
12
= Cos 1
| a | = 62 + 22 + 32 = 36 + 4 + 9 = 7 7 11÷

and | b | = 22 + ( 9)2 + 62 = 4 + 81 + 36 = 11 12
= Cos 1 ÷
77

a ×b = | a || b | cosq

Example 6.2

Let a = i + 2 j 3k and b = 3i j + 2k . Prove that a + b = [12 + 22 + ( 3)2 ] [32 + ( 1)2 + 22 ]

and a b are perpendicular to each other. =0
Solution: We have, by Corollary 6.4 [part (3)], that
a + b and a b are perpendicular to each other.

(a + b)×(a b) = | a |2 | b |2
376 Chapter 6 Multiplication of Vectors

Example 6.3

Find the angles of


the triangle
whose
position
vectors
of This implies
the vertices are i + 2 j 5k , 2 i + 2 j + k and 2 i + j k .
7
A = Cos 1 ÷
Solution: Let ABCC be the given triangle and 3 10


a = OA = i + 2 j 5k

we can find that B, the angle between BA


Similarly,

and BC is given by
b = OB = 2 i + 2 j + k

BA ×BC
and c = OC = 2 i + j k B = Cos 1


| BA|| BC | ÷
Then

8
= Cos 1 ÷
AB = OB OA = 3i + 6k 105


and AC = OC OA = i j + 4k and


Now A is the angle between AB and AC . Therefore CA ×CB
C = Cos 1

|CA |×|CB | ÷
AB ×AC
A = Cos 1


| AB|×| AC | ÷ = Cos 1
1
÷
42
( 3) 1 + 0 ( 1) + 6 4
= Cos 1 ÷
( 3) + 0 + 6
2 2 2
1 + ( 1) + 42
2 2

In the following we prove certain important geometrical results using the dot product of vectors and its properties.

T H E O R E M 6.8 Let A, B and C represent the angles of a triangle ABC C and a, b and c represent the lengths of the
sides opposite to them, respectively. Then
1. a2 = b2 + c2 2bc cos A (Law of cosines)
2. a = b cos C + c cos B (Projection formula)


PROOF See Figure 6.6. Let BC = a, AC = b and BA = c . Then

| a | = a, | b | = b and | c | = c

A 180° A

b
c

180° C

B C
a
180° B

FIGURE 6.6 6.8.


6.1 Scalar or Dot Product 377

1. We have

(c , b) = 180° A


where (c , b) is the angle between c and b. Since BC = BA + AC , we have a = c + b. Therefore

a2 = | a |2 = | c + b |2

= | c |2 + | b |2 + 2c ×b

= c2 + b2 + 2 | c || b | cos(c , b)
= c2 + b2 + 2cb cos(p A)
= b2 + c2 2bc cos A
Consider


BC = BA + AC = (CA + AB)

We have

a2 = | a |2


= | BC |2


= BC ×BC


= (CA + AB) ×BC


= CA ×BC AB ×BC


= |CA || BC | cos(CA, BC ) | AB || BC | cos( AB, BC )
= ba cos(p C) ca cos(p B)
= ab cos C + ac cos B
Therefore

a2 = a(b cos C + c cos B)


or a = b cos C + c cos B ■

Try it out We can prove that other laws of cosines and projection formulae, namely,
1. b = c + a 2ca cos B
2 2 2

2. c2 = a2 + b2 2ab cos C
3. b = c cos A + a cos C
4. c = a cos B + b cos A

T H E O R E M 6.9 The altitudes of a triangle are concurrent.


PROOF Consider a triangle ABCC and draw perpendiculars from A and B to BC C and CA to meet them at
D and E, respectively. Suppose that O is the point of intersection of AD and BE. Join C and O
and produce it to meet AB at F
F. With reference to O, let the position vectors of A, B and C be

a, b and c respectively. From Figure 6.7 we have the following:


BC = BO + OC = b + c


CA = CO + OA = c + a


AB = AO + OB = a + b
378 Chapter 6 Multiplication of Vectors

F O

B C
D

FIGURE 6.7 Theorem 6.9.



Since AD BC , OA ×BC = 0 and hence a ×(c b) = 0 so that

a ×c = a ×b
Similarly, from BE AC, we get that

b ×a = b ×c
From these two, we have

a ×c = b ×c

and hence (b a ) ×c = 0. Therefore CF AB. Thus the altitudes of the triangle ABC
C are
concurrent. ■

T H E O R E M 6.10 The perpendicular bisectors of the sides of a triangle are concurrent.


PROOF Let ABC C be a triangle and D, E and F be the mid-points of the sides BC, CA and AB, respec-
tively. Let the perpendicular bisectors drawn to the BCC and CA at D and E meet at O. Join

O and F F. Let a, b, c be the position vectors of A, B, C, respectively, with respect to O. Now
(see Figure 6.8), we have


a = OA, b = OB, c = OC
Hence


BC = BO + OC = b+c


CA = CO + OA = c+a


AB = AO + OB = a+b

a
F E

O
b
c

FIGURE 6.8 Theorem 6.10.


6.1 Scalar or Dot Product 379

Also

1
OD = (b + c )
2

1
OE = (c + a)
2

1
OF = (a + b)
2
Since OD BC,
1
(b + c ) ×( b + c ) = 0
2

and hence | c |2 = | b |2. Similarly, from OE CA, we get | a |2 = | c |2. Therefore | a |2 = | b |2 and hence

1
OF ×AB = (a + b) ×( a + b)
2
1
= (| b |2 | a |2 ) = 0
2
Thus OF
F is perpendicular to AB, so that OF F becomes the perpendicular bisector of AB. Thus, the
perpendicular bisectors of the sides of a triangle are concurrent. ■

T H E O R E M 6.11 Let a and b be any non-negative real numbers such that a + b p p. Then
1. cos(a + b)
b = cos a cos b sin a sin b
2. cos(a b)b = cos a cos b + sin a sin b

PROOF Let i and j be the unit vectors along the rectangular

coordinate axes OX X and OY, Y respectively.


Choose a point A in the XY Y plane such that OA = a is a unit vector

making an angle a with the


positive X Y plane such that OB = b is a unit vector making an
X-axis. Also choose a point B in the XY
angle b with the positive X
X-axis. Draw perpendiculars AP
P and BQ to OX (Figure 6.9). Then we have

A (cos a, sin a)

j
a

a
O X
b i P Q

FIGURE 6.9 Theorem 6.11.


380 Chapter 6 Multiplication of Vectors


a = OA


= OP + PA

= (OP )i + ( PA) j

= cos a i + sin a j


b = OB


= OQ + QB

= (OQ)i + (QB) j

= cos bi sin bj

Therefore, we get that



cos(a + b ) = | a || b | cos(a, b) (since | a | = 1 = | b |)

= a ×b

= (cos a i + sin a j )×(cos bi sin bj )
= cos a cos b sin a sin b
Similar technique yields

cos(a b ) = cos a cos b + sin a sin b ■

T H E O R E M 6.12 The angle in a semicircle is a right angle.


PROOF Consider a circle with AOB as a diameter and O as the centre. Let P be an arbitrary point on the
semicircle, as shown in Figure 6.10. With reference to O as the origin, let the position vectors of

A and P be a and p, respectively. Then


AP = OP OA = p a


and BP = OP OB = p ( a ) = p + a
Now,


AP ×BP = ( p a )×( p + a )

= | p|2 | a |2
= r2 r2 = 0

where r is the radius of the circle. Thus AP P and hence APB = 90°.
P is perpendicular to BP

–a

FIGURE 6.10 Theorem 6.12. ■


6.1 Scalar or Dot Product 381

Example 6.4

In a triangle ABC, let D be the midpoint of BC. Then A


prove that
AB2 + AC 2 = 2( AD2 + BD2 )

Solution: See Figure 6.11. Consider A as the origin. Let c



b
b and c be the position vectors of B and C respectively.
Then


BC = BA + AC = b + c

1
and BD = (c b) FIGURE 6.11 6.4.
2
Now consider
Also,

2( AD2 + BD2 ) = 2(| AD|2 + | BD|2 )


AD = AB + BD
1 1 2 1
=2 |b + c | + |c b |2 ÷
= b + (c b ) 4 4
2
1 By Corollary 6.4, we have
= (b + c )
2 1
2( AD2 + BD2 ) = (| b |2 + | c |2 + 2b ×c + | c |2 + | b |2 2b ×c )
2

= | b |2 + | c |2 = AB2 + AC 2

Example 6.5


Prove that the smaller angle between any two diagonals OE = OB + BE
of a cube is


= OA + AB + BE
1


Cos 1 ÷ = OA + OC + OG (since AB = OC , BE = OG)
3
=i + j +k
G and
D


GB = GO + OB


k = k + OA + AB
F

E =i + j k

j Let q be the smaller angle between the diagonals OE


C and GB. Then
O


OE ×GB
i cosq =


|OE ||GB|
(1 1) + (1 1) + 1 ( 1)
=
FIGURE 6.12 Example 6.5.
12 + 12 + 12 12 + 12 + ( 1)2
1
=
Solution: Let OABCDEFG be a unit
cube

as

shown
3
in Figure

6.12. O as origin, let OA = i , OC = j


Keeping Thus
and OG = k . Then i , j , k form an orthogonal unit
vector triad. Consider the diagonals OE E and BG. From 1
Figure 6.12, we have q = Cos 1 ÷
3
382 Chapter 6 Multiplication of Vectors

6.2 | Vector Equations of a Plane and a Sphere


Using vector addition and multiplication of a vector by a scalar we have derived vector equations of a straight line and
a plane in the previous chapter. In this chapter, using dot product of vectors, we derive vector equation of a plane at
a given distance from the origin and perpendicular to a given vector.


T H E O R E M 6.13 The equation of the plane, whose unit normal drawn away from the origin is n and whose distance
from the origin is a, is given by

r .n = a
This is called the equation of a plane in normal form.
PROOF See Figure 6.13. Let R be an arbitrary point in the required plane and the position vector of R

with respect to the origin O

r. Let a perpendicular
be

be drawn from the origin to the plane to


meet the plane at N. Then RN is perpendicular to ON . Since the distance from the origin to the
required plane is given to be a, we have

a = ON = OR cos(r , n) = OR | n | cos(r , n)

since n is a unit normal. Therefore

a = r ×n

On the other hand, if P is any point such that



OP ×n = a
then


NP ×n = (OP ON ) ×n


= OP ×n ON ×n

= a a n ×n
=a a=0
and hence PN N is perpendicular to ON. Since the required plane is perpendicular to ON N and

passes through N, it follows that P lies in the required plane. Thus r ×n = a is the equation of the
required plane.

R N

r n

FIGURE 6.13 Theorem 6.13. ■


C O R O L L A R Y 6.7 Equation of the plane passing through the origin and perpendicular to the unit vector n is r ×n = 0.
PROOF The proof follows from Theorem 6.13 and the requirement that the plane passes through the
origin and hence the distance from the origin to the plane is zero. ■
6.2 Vector Equations of a Plane and a Sphere 383

QUICK LOOK 5

If R = (x, y, z) with respect to an orthogonal system of Therefore the equation of the plane in Theorem 6.13
coordinate axes passing through the origin O and the reduces to
direction cosines of the normal to the plane are l, m, n,
xl + ym + zn = a
respectively, then, with the usual notation,
This is the Cartesian equation of the plane in normal
r = xi + yj + zk form. Also, the Cartesian equation of the plane passing

and n = li + mj + nk through the origin and having l, m, n as the direction
cosines of its normal is
and hence
xl + ym + zn = 0

r ×n = xl + ym + zn

Next we derive the vector equation of a sphere. First recall that if C is a fixed point in the space and a is a fixed
non-negative real number, then the set of all points P in the space such that CP = a is called the sphere with centre at
C and radius a.


T H E O R E M 6.14 Let c be the position vector of the centre C of a sphere S with radius a. Then the vector equation
of S is given by

| r |2 2r ×c + | c |2 = a2

PROOF See Figure 6.14. Let P be an arbitrary point in the space and r be the position vector of P with

respect to the origin O. The position vector of the centre C is given as c. Then we have


OP = r


OC = c


and CP = OP OC = r c

Now
P is a point on the sphere S CP = a
CP 2 = a2

| r c |2 = a2

| r |2 + | c |2 2r ×c = a2

Thus the vector equation of the sphere with radius a and centre at C with a position vector c is

| r |2 2r ×c + | c |2 = a2

P r c
C

r
c

FIGURE 6.14 Theorem 6.14. ■


384 Chapter 6 Multiplication of Vectors


C O R O L L A R Y 6.8 If the origin O lies on the sphere S whose radius is a and centre at C with position vector c, then
the vector equation of the sphere S is given by

| r |2 = 2r ×c

PROOF The general equation of a sphere with radius a and centre at C (with position vector c ) is given by

| r |2 2r ×c + | c |2 = a2 (6.1)

If the origin lies on the sphere, then



| 0 |2 2 0 ×c + | c |2 = a2

and hence | c |2 = a2. Substituting this in Eq. (6.1), we get the equation of the sphere as

| r |2 = 2r ×c ■

C O R O L L A R Y 6.9 Equation of the sphere with radius a and centre at the origin is given by

| r |2 = a2

PROOF Substituting c = 0 in Eq. (6.1), we get the equation as | r |2 = a2. ■

QUICK LOOK 6

Let us consider the Cartesian form of the sphere with If the origin lies on the sphere, then
radius a and centre at C = (c1, c2, c3). Then
c12 + c22 + c32 = a2


c = OC = c1i + c2 j + c3k
and the equation of the sphere reduces to
Let P = (x, y, z) be an arbitrary point in the space and
x2 + y2 + z2 = 2(c1 x + c2 y + c3z)


r = OP = xi + yj + zk
If origin is the centre of the sphere, then equation of
Then P lies on the sphere S if and only if CP = a ; that is,
2 sphere reduces to

(x c1 )2 + ( y c2 )2 + (z c3 )2 = a2 x2 + y2 + z2 = a2

This is the Cartesian form of the equation of the sphere


with radius a and centre at C = (c1, c2, c3).

Try it out The following is a generalization of Theorem 6.12:


Any diameter of a sphere S subtends a right angle at any point on S other than extremities of the diameter.

T H E O R E M 6.15 Let AB be a diameter of a sphere S and P be a point on S. Then



AP ×BP = 0
PROOF Let a be the radius and C the centre of the sphere S. Then


AC = CB = BC


AP = AC + CP


BP = BC + CP
6.2 Vector Equations of a Plane and a Sphere 385

Hence


AP ×BP = ( AC + CP ) ×( BC + CP )


= (CP + AC ) ×(CP AC )


= |CP |2 | AC |2

= a2 a2 = 0 ■

T H E O R E M 6.16 Let A and B be any distinct points in the space and a = (1/2)AB. Let C be the mid-point of AB.
Then the sphere S with radius a and centre at C is given by


S = {P : AP ×BP = 0}

PROOF By Theorem 6.15,



S {P : AP ×BP = 0}


On the other hand, let P be any point in the space such that AP ×BP = 0. Then


CP = CA + AP


and CP = CB + BP = CA + BP
Therefore


CP 2 = CP ×CP


= (CA + AP ) ×(CB + BP )


= ( AP + CA) ×( BP CA)


= AP ×BP AP ×CA + CA×BP CA×CA


= 0 + CA×(BP AP ) a2


= CA ×BA a2


= CA ×(2CA) a2

= 2a2 a2 = a2
and hence P lies on the sphere. Thus


S = {P : AP ×BP = 0} ■


C O R O L L A R Y 6.10 Let A and B be two given points with position vectors a and b, respectively. Then the equation
of the sphere with AB as a diameter is

(r a )×(r b) = 0


PROOF If P is a point with position vector r, then AP = r a and BP = r b. Therefore, by

Theorem 6.16, P lies on the sphere with AB as a diameter if and only if (r a ) ×(r b) = 0. ■

DEFINITION 6.3 Let 1 and 2 be two planes and n1 and n2 be unit normals to and , respectively. Then
1 2
the angle between n1 and n2 is called the angle between 1 and 2.
386 Chapter 6 Multiplication of Vectors

QUICK LOOK 7

Since a given plane has two different unit normals (one p q is the other. If n1 and n2 are unit normals to the

is opposite to the other), angle between a given pair of planes 1 and 2, respectively, then Cos 1 (n1 ×n2 ) is an
planes is not unique. In general, there are two angles angle between the planes 1 and 2.
between a given pair of planes. If q is one of them, then


T H E O R E M 6.17 Let A be a point in the space with position vector a and n be any non-zero vector. Then the

vector equation of the plane passing through A and perpendicular to n is

(r a )×n = 0

PROOF If P is a point in the space with position vector r, then P lies on the required plane if and only if


AP is perpendicular to n; that is, (r a ) ×n = 0. ■

Example 6.6

2, 1, 3) Then P lies on the required plane if and only if


and perpendicular to 3i + j + 5k .


AP is perpendicular to 3i + j + 5k
Solution: Let O be the origin and A = ( 2, 1, 3) be the


given point. Then the position vector of A with respect (OP OA)×(3i + j + 5k ) = 0
to the origin is
[( x + 2)i + ( y 1) j + (z 3)k ]×(3i + j + 5k ) = 0


OA = 2 i + j + 3k
( x + 2)3 + ( y 1) + (z 3)5 = 0
If P = (x, y, z) is any point in the space, then the position
vector of P is 3 x + y + 5z = 10

This is the equation of the required plane.


OP = xi + yj + zk = r , say

Example 6.7

Find the angles between the planes 2x 3y 6z = 5 and 1 1 1


cosq = a × b = (a ×b)
6x + 2y
2 9z = 4. |a | |b| | a || b |

Solution: Let 2 6 + ( 3) 2 + ( 6) ( 9)
=
2 + ( 3) + ( 6)2 62 + 22 + ( 9)2
2 2

r = xi + yj + zk
60
a = 2i 3j 6k =
77

and b = 6i + 2 j 9k Therefore

60
q = Cos 1 ÷
77
r ×a = 5 and r ×b = 4
The other angle is
Here a /| a | and b /| b | are unit vectors perpendicular to
the given planes. If q is the angle between these planes, 60
then p Cos 1 ÷
77
6.3 Vector or Cross Product 387

Example 6.8

If | r |2 2r ×(3i + 4 j + 6k ) + 4 = 0 represents the equation The given equation is
of a sphere, then find the centre and radius of the sphere.
| r |2 2r ×(3i + 4 j + 6k ) + 4 = 0 (6.3)
Solution: The equation of a sphere whose centre is the
Therefore comparing Eqs. (6.2) and (6.3) we get
point with position vector c and radius a is

c = 3i + 4 j + 6 k
(r c ) ×(r c ) = a2

and a2 = | c |2 4
Simplifying we get
Thus the centre is (3, 4, 6) and the radius is
r ×r 2r ×c + c ×c = a2

| r |2 2r ×c = a2 | c |2 (6.2) | c |2 4 = 32 + 42 + 62 4 = 2 15

6.3 | Vector or Cross Product


We introduce the concept of vector product or cross product of two vectors and study its various properties and appli-
cations in finding vector areas and volumes of certain geometrical figures.

DEFINITION 6.4 Let a and b be any vectors. We define

0 if a and b are collinear or a = 0 or b = 0
a b=
| a || b | sin q n

where q = (a, b) is the
angle between a and b and n is the unit
vector orthogonal to both a
and b such that (a, b, n) form a triad of right-handed vectors. a b is called
the vector product

(since it is a vector)
or cross productt (since is used in between a and b in contrast with the

dot product a, b ) of a and b. Figure 6.15 shows the vector product.

b
n

FIGURE 6.15 Definition 6.4.


Note that if a = 0 or b = 0, then a and b are obviously collinear and hence a b = 0. Conversely, if a b = 0, then

a = 0 or b = 0 or a and b are non-zero collinear (parallel) vectors.


T H E O R E M 6.18 For any vectors a and b

|a b | | a || b |

PROOF We can assume that a and b are non-zero non-collinear vectors (otherwise a b = 0 and the

result is trivial). Then 0 < (a, b) < p and hence 0 < sin(a, b) 1 so that

|a b | = | a || b || n | sin(a, b) | a || b |

since n is a unit vector. ■
388 Chapter 6 Multiplication of Vectors


that a ×b = b ×a for all vectors a and b.
We have proved earlier that the dot product is commutative in the sense

The vector product is not commutative, that is, a b may not be equal to b a. However, we have the following.


T H E O R E M 6.19 For any vectors a and b,

a b = (b a)

That is, a b and b a are opposite to each other.

PROOF If one of a and b is the zero vector or if a and b are collinear, then

a b=0= 0= b a

Therefore, we can assume
that a and b are non-zero and non-collinear vectors. q be the
Let
angle between a and b and n be the unit vector perpendicular to both a and b such that

(a, b, n) is a right-handed system. Then, by Definition 6.4, we have

a b = (| a || b | sin q )n

Since (b, a, n) is a right-handed system, we have

b a = (| b || a | sin q ) ( n)

= (| a || b | sin q )n

= (a b ) ■

C O R O L L A R Y 6.11 We have


b| = |b
|a a | = | a || b | sin q

where q is the angle between a and b.


T H E O R E M 6.20 The following hold for any vectors a and b and scalars l and m:

1. ( a ) b = b a = a ( b) = (a b)

2. ( a ) ( b) = a b

3. (la ) b = l(a b) = a (lb)

4. (la ) (mb) = lm(a b)

PROOF All these equalities hold good trivially if a = 0 or b = 0 or a and b are collinear or l = 0 or m = 0.

Therefore we can assume that a and b are non-zero non-collinear vectors and l and m are non-

zero scalars.
Let q be the angle between a and b and n be the unit vector perpendicular to both

a and b such that (a, b, n) is a right-handed system.

1. The vector triad ( a, b, n) becomes a left-handed system while ( a, b, n) becomes a right-

handed system. Also, the angle between a and b is p q. Therefore

( a ) b = [| a || b | sin(p q )]( n)

= (| a || b | sin q )n

= (a b )

=b a
6.3 Vector or Cross Product 389

and

a ´ (- b) = - [(- b) ´ a ]

= -(a ´ b) (by the above equality)

2. We have

(- a ) ´ (- b) = - [a ´ (- b)] = -[-(a ´ b)] = a ´ b

3. Let us first assume that l > 0. Then the angle between la and b is q and | la | = l | a |. Therefore

since (ln, b, n) is a right-handed system

(la ) ´ b = (| la || b | sin q )n,

= (l | a || b | sin q )n

= l (a ´ b )

If l < 0, then -l > 0 and, by (1) above, we have



(la ) ´ b = -[(- l )a ´ b]

= - ( - l ) (a ´ b )

= l (a ´ b )


a ´ (lb) = -(lb) ´ a = - l(b ´ a ) = l(- b ´ a ) = l(a ´ b)

4. We have

(la ) ´ (mb) = l[a ´ (mb)] = lm(a ´ b) ■


T H E O R E M 6.21 For any vectors a and b,

a ´ (a + b ) = a ´ b

PROOF This equality trivially holds if a = 0 or b = 0. Therefore, we can assume that

a and b are

non-


zero vectors. Let O be the origin and choose points A and B such that OA = a and OB = b.
Complete the parallelogram OABC. Let D be the foot of the perpendicular from C to the line
OA (Figure 6.16). Now we have


OC = a + b

B C

® ®
a+b
®
b ®
b

FIGURE 6.16 Theorem 6.21.


390 Chapter 6 Multiplication of Vectors


Clearly, a (a + b) and a b have the same direction. Also, regarding the magnitudes, we have

| a (a + b)| = | a || a + b | sin COD

= | a | OC sin COD = | a | CD

= | a | AC sin DAC

= | a || b | sin AOB = | a b |

Thus a (a + b ) = a b. ■


C O R O L L A R Y 6.12 For any vectors a and b and any scalar l,

a (l a + b) = a b

PROOF If l = 0, this is trivial. Suppose that l 0. Then

1
a (l a + b) = a l a + b÷
l

1
=l a a + b÷
l

1
=l a b÷ (by Theorem 6.21)
l

=a b ■

Note: The proof of the following theorem is bit lengthy. The reader may skip the proof in first reading and assume
its validity.


T H E O R E M 6.22 For any vectors a, b and c,
(D I S T R I B U T I V E
1. a (b + c ) = (a b) + (a c )
LAW)
2. (a + b) c = (a c ) + (b c )

PROOF (2) is a simple consequence of (1), because



(a + b) c = [c (a + b)]

= [(c a ) + (c b)], by (1)

= c a c b

=a c+b c

1. This is trivial if a = 0 or b = 0 or c = 0. Therefore we can assume that a, b and c are non-zero
vectors. We shall distinguish
the following cases.

Case I: Suppose that b and c are linearly dependent. In this case there exists a scalar, a such

that b = ac and hence

a (b + c ) = a (ac + c )

= a [(a + 1)c ]
6.3 Vector or Cross Product 391


= (a + 1) (a ´ c )

= a (a ´ c ) + ( a ´ c )

= [a ´ (ac )] + (a ´ c )

= (a ´ b) + (a ´ c )

II: Suppose that b and c are linearly independent
Case and a ´ (b + c ) = 0. Then a and

b + c are non-zero parallel vectors and hence b + c = l a for some scalar l. Now, we have

a ´ c = a ´ (l a - b)

1
= [l a ´ (l a - b)], by Theorem 6.20 [part (3)]
l
1
= [l a ´ (- b)], by Theorem 6.21
l

= a ´ (- b), by Theorem 6.20 [part (3))]

= -a ´ b


(a ´ b ) + (a ´ c ) = 0 = a ´ (b + c )

Case III: Suppose that b and c are linearly independent and

a is perpendicular

to both b

and c. Let O be the origin and B and C be points such that OB = b and OC = c. Then O, B,

C are not collinear (since b and c are linearly

independent,
Figure 6.17). Choose a point D
such that OBDC C is a parallelogram. Then OD = b + c . Now, choose points B¢ and C¢ such


that OB ¢ = a ´ b and OC ¢ = a ´ c . Then B¢ and C¢ lie in the OBC C plane, since a is perpen-

dicular to both b and c. Also,

OB¢ = | a ´ b | = | a || b | = | a | (OB)

and OC ¢ = | a ´ c | = | a || c | = | a | (OC )


® ®+a´c
® ®


C¢ ®
a´b

® ® b
a´b C D

® ® ®
a´c c ®
c

O ® B
b

FIGURE 6.17 Theorem 6.22 [part (1), Case III].


392 Chapter 6 Multiplication of Vectors

Next, choose a point D such that OB D C is a parallelogram, as shown in Figure 6.17. Then
we have, by the parallelogram law,


OD = (a b) + (a c ) (6.4)

Let L be a straight line passing through O and perpendicular to the OBC C plane. Since

(a, b, a b) and (a, c , a c ) are right-handed orthogonal

systems,
a rotation about the line
L through

the angle

BOC, so that the new direction


of OB consider with the old direction


of OC , brings OB along the old direction of OC . Therefore
B OC = BOC
and OB D = p B OC = p BOC = OBD

Since
OB BD
= |a | =
OB BD

it follows that the triangles OB D and OBD are similar. Hence


OD
= |a |
OD
and B OD = BOD

Also, since (a, b, a b) and (a, c , a c ) are right-handed

orthogonal
systems, a rotation
about the L through

line

90°, so that the new direction

of OB coincides with the old direc-


of OB , brings OC along the


tion

old direction of OC . Therefore the same rotation brings



OD along the old direction of OD

and hence it follows that (a, OD, OD ) is a right-handed



orthogonal system. Therefore OD and a (b + c ) have the same direction. Now

OD = | a | (OD) = | a || b + c | = | a (b + c )|


Thus OD and a (b + c ) have the same direction and magnitude, so that


a (b + c ) = OD = (a b) + (a c ) [by Eq. (6.4)]

Case IV: Suppose
b and
that c are linearly independent, a (b + c ) 0 and a is not perpen-

dicular to b or c. If b = l a for some non-zero scalar l, then

a (b + c ) = a ( l a + c )

= a c (by Corollary 6.12)

= l 0 + (a c )

= l (a a ) + (a c )

= (a l a ) + (a c )

= (a b ) + (a c )

Similarly,
we are through if c = l a for some scalar l 0. Therefore, we can assume that a

and b are linearly independent and that a and c are linearly independent. Let O denote the
origin. Choose points A, B and C such that


OA = a, OB = b and OC = c
6.3 Vector or Cross Product 393

Then the points O, A and B as well as the points O, A and C are non-collinear. Choose a


point D in the OAB plane such that OD is perpendicular to OA and (a, OD, a b) is a right-
handed system (see Figure 6.18).

q
a
b
b

A
B
D

–9
a


b

b
B
A

FIGURE 6.18 6.22 [part (1). case (iv)].



Let b be the orthogonal projection of b on OD. Then a b = a b and b b is parallel to

a. Similarly we can choose a vector c such that a is perpendicular to c , a c = a c and

c c is parallel to a. If b and c are linearly dependent, then as in case (i),

a (b + c ) = a (b + c )

= (a b ) + (a c )

= (a b ) + (a c )

Now, suppose that b and c are linearly independent. Since a is perpendicular to both b

and c , it follows from case (iii) that

a (b + c ) = (a b ) + (a c )

Since both b b and c c are parallel to a, it follows that b + c (b + c ) is parallel to a
and hence

a [(b + c ) (b + c )] = 0

and therefore, from case (ii),



a (b + c ) a (b c ) = a [(b + c ) (b + c )] = 0
394 Chapter 6 Multiplication of Vectors

and hence

a (b + c ) = a (b + c )
But we have

a (b + c ) = (a b ) + (a c )

= (a b ) + (a c )

Thus, in any case, a (b + c ) = (a b) + (a c ). ■

Note: The above proof is apparently involved and complicated. We offer an alternate proof later (see Theorem 6.45)
which is elegant and simple. The strength of the above proof is that it is direct from the definition of the vector product.
Next, we derive a simple formula for the vector product of two vectors in terms of their components in the direc-
tions of an orthogonal right-handed system of unit vectors. First, let us have the following.


T H E O R E M 6.23 Let (i , j , k ) be a right-handed orthogonal unit vector triad. Then

1. i i = 0 = j j = k k

2. i j = k , j k = i and k i = j

3. j i = k , k j = i and i k = j

PROOF 1. Follows from the fact that a a = 0 for any vector a.

2. Since (i , j , k ) is a right-handed system and k is the unit vector orthogonal to both i and j ,
we have
p
i j = | i || j | sin k = k
2

Similarly j k = i and k i = j .

3. Follows from (2) and the fact that a b = b a for any vectors a and b. ■


T H E O R E M 6.24 (i , j , k ) be a right-handed orthogonal system of unit vectors and a = a1i + a2 j + a3k and
Let
b = b1i + b2 j + b3k . Then

a b = (a2 b3 a3b2 )i + (a3b1 a1b3 ) j + (a1b2 a2 b1 )k

PROOF We have

a b = (a1i + a2 j + a3k ) (b1i + b2 j + b3k )

= a1b1 (i i ) + a1b2 (i j ) + a1b3 (i k )

+ a2 b1 ( j i ) + a2 b2 ( j j ) + a2 b3 ( j k ) (by Thheorem 6.22)

+ a3b1 (k i ) + a3b2 (k j ) + a3b3 (k k)

= a1b2 k a1b3 j a2 b1k + a2 b3 i + a3b1 j a3b2 i (by Theorem 6.23)

= (a2 b3 a3b2 )i + (a3b1 a1b3 ) j + (a1b2 a2 b1 )k ■

Note: The above expression for a b can be given in the form of a determinant as

i j k

a b = a1 a2 a3
b1 b2 b3
6.3 Vector or Cross Product 395


T H E O R E M 6.25 Let a = a1i + a2 j + a3k and b = b1i + b2 j + b3k be non-zero vectors where
(i , j , k ) is a right-handed

system of orthogonal unit vectors. Let q be the angle between a and b. Then

(a2 b3 a3b2 )2
sin q =
( a12 )( b12 )
a1b1 + a2 b2 + a3b3
and cos q =
a12 b12

PROOF Since a b = | a || b | sin q n, we have
(a2 b3 a3b2 )2
|a b|
sin q = =
| a || b | a12 b12

and hence

cosq = 1 sin2 q

=
( a12 )( b12 ) (a2 b3 a3b2 )2
a12 × b12
a1b1 + a2 b2 + a3b3
=
a + a22 + a32 b12 + b22 + b32
2
1 ■

Note: In the above, the value of sin q will not give us the value of q, since the sine function is not an injection on the
interval [0, p
p]. However the cosine function is an injection on the interval [0, p
p] and hence q is determined uniquely by
the value of cos q. For these reasons, the formula for cos q is more useful in determining the angle between two vectors.


T H E O R E M 6.26 For any vectors a and b,

2 2 2 2 a ×a a ×b
|a b | = | a | | b | (a ×b) =
a ×b b ×b

PROOF These equalities
if a = 0 or b = 0 or a and b are collinear. Therefore we can assume
hold trivially

that a 0, b 0 and a and b are not collinear (parallel). Then

a b = (| a || b | sin q ) n

where q is the angle between a and b and n is a unit vector perpendicular to both a and b such

that (a, b, n) is a right-handed system. Now, we have

| a b |2 = | a |2 | b |2 sin2 q (since | n | = 1)

= | a |2 | b |2 (1 cos2 q )

= | a |2 | b |2 | a |2 | b |2 cos2 q

= | a |2 | b |2 (a ×b)2

= (a ×a ) (b ×b) (a ×b) (a ×b)

a ×a a ×b
= the determinant
a ×b b ×b ■
396 Chapter 6 Multiplication of Vectors


T H E O R E M 6.27 Let a and b be two non-zero and non-parallel vectors. Then a unit vector n perpendicular to

both a and b is given by

a b (a2 b3 a3b2 )i
n=± =±
|a b| (a2 b3 a3b2 )
2


where a = a1i + a2 j + a3k and b = b1i + b2 j + b3k .
PROOF This is a direct consequence of Definition 6.4 and Theorem 6.24. ■

Note: The signs + or depend on whether (a, b, n) is a right-handed system or a left-handed system.
In the following we derive a formula for the cross product of two vectors when they are expressed as linear combi-
nations of three given non-coplanar vectors. This generalizes Theorem 6.24.


T H E O R E M 6.28 Let (a, b, c ) be a non-coplanar vector triad and

a = r1a + r2 b + r3c

and b = s1a + s2 b + s3c


b c c a a b

a b= r1 r2 r3
s1 s2 s3

PROOF 6.22), we have



a b = (r1a + r2 b + r3 c ) ( s1a + s2 b + s3 c )

= r1 s2 (a b) + r1 s3 (a c ) + r2 s1 (b a ) + r2 s3 (b c )

+ r3 s1 (c a ) + r3 s2 (c b)

= (r2 s3 r3 s2 ) (b c ) (r1 s3 r3 s1 ) (c a ) + (r1 s2 r2 s1 ) (a b)

b c c a a b
= r1 r2 r3
s1 s2 s3


Try it out If (a, b, c ) is an orthogonal right-handed system of unit vectors, then Theorem 6.28 implies

Theorem 6.24, since a b = c , b c = a and c a = b.
Prove it.

Example 6.9

Let a = 2 i 3 j + 5k and b = i + 4 j + 2k . Then find the i j k

a b and unit vectors perpendicular to
cross product a b= (a2 b3 a3b2 )i = a1 a2 a3
both a and b,
b1 b2 b3
Solution: Recall the formula
6.3 Vector or Cross Product 397


Substituting the values we get The unit vectors perpendicular to both a and b are

i j k a b 1
± =± ( 26 i 9 j + 5k )
a b= 2 3 5 |a b| 782
1 4 2

= ( 6 20)i (4 + 5) j + (8 3)k

= 26 i 9 j + 5k

Example 6.10


Let ABCC be a triangle, BC = a, CA = b and AB = c . and hence a + b = c . Now, by Theorem 6.21, we have
Then prove that
a b = a (a + b ) = a ( c ) = a c = c a

a b=b c=c a
and
Solution: First observe that
b c = b (b + c ) = b ( a ) = b a=a b


a + b + c = BC + CA + AB = BA + AB = 0
Therefore

a b=b c=c a

Example 6.11

If a = 2 i j + k and b = 3i + 4 j k, find angle between Therefore

a and b.
155 155
sinq = =
Solution: If q is the required angle, then 6 26 156

|a b| Also,
sinq =
| a || b |
a ×b = 2 ×3 + ( 1)4 + 1( 1) = 1
Now
Therefore

i j k
a ×b 1
a b= 2 1 1 = 3i + 5 j + 11k cosq = =
| a || b | 156
3 4 1

|a b | = ( 3)2 + 52 + 112 = 155
1
q = Cos 1 ÷
| a | = 22 + ( 1)2 + 12 = 6 156

| b | = 32 + 42 + ( 1)2 = 26


Note that for finding the angle between a and b, use of the dot product is more suitable.

Example 6.12

Let a = 2 i + j 2k and b = i + j . Let c be a vector such Solution: We have

that a ×c = | c |, | c a | = 2 2 and the angle between a b

and c is 30°. Then find the value of |(a b) c |. | a | = 22 + 12 + ( 2)2 = 3

and | b | = 12 + 12 + 02 = 2
398 Chapter 6 Multiplication of Vectors


Now, 2 2 = | c a | implies that | c a |2 = 8. Therefore and |a b| = 3

| c |2 + | a |2 2(c ×a ) = 8 Therefore

| c |2 + 9 2 | c | = 8 |(a b) c | = | a b || c | sin 30°

(| c | 1)2 = 0 1 3
= 3(1) =
|c | = 1 2 2

Also

i j k

a b= 2 1 2 = 2i 2j + k
1 1 0

6.4 | Vector Areas


The orientation of a closed curve is usually denoted by an ordered trial of non-collinear points on the curve as indi-
cated in Figure 6.19. Consider the plane region D bounded by a closed curve described in the anticlockwise sense.
It is indicated by the ordered trial (P1, P2, P3) of points on it (see figure (a) below). In the figure (b), the region D is
bounded by a curve described in the clockwise sense and it is indicated by the ordered trial (P1, P3, P2). While the area
of the region D is independent of the orientation of the bounding curve, the vector area depends on the orientation
of the bounding curve, as we see from the definition given below.

n P3 P3

D
D
P1 P1

P2 P2

–n

FIGURE 6.19 Orientation of closed curve.

DEFINITION 6.5 Let D be the plane region bounded by an oriented closed curve C whose orientation is indi-
cated by an ordered triad (Q1, Q2, Q3) of non-collinear points on C. The vector area of D is

defined as A n,
where A is the area of the region D and n is the unit vector

perpendicular



to D such that (Q1Q2 , Q1Q3 , n) is a right-handed system. In Figure 6.19(a), ( P1 P2 , P1 P3 , n) is a

system and in this case A n is the vector area of the region D. In Figure 6.19(b),
right-handed
( P1 P3 , P1 P2 , n) in a right-handed system and hence, in this case, the vector area of the region

D is A( n).

QUICK LOOK 8

1. For any plane region, there will be two vector areas 2. The magnitude of the vector area of the plane region
and they are opposite to each other. is the usual area of the region and its direction is
perpendicular to the plane of the region.
6.4 Vector Areas 399

T H E O R E M 6.29 The vector area of a triangle ABC


C is
1


( AB AC )
2

b
c

FIGURE 6.20 Theorem 6.29.



PROOF Consider a triangle ABC C (Figure 6.20) and let its area

be

. Let n be the unit vector perpen-



dicular to the plane of the triangle ABCC such that ( AB, AC , n) forms a right-handed system.

Then the vector area of the triangle is n. But, we have
= Area of the triangle ABC
1
= bc sin A
2
1


= | AC || AB| sin A
2
ABC
C is
1


( AB AC )
2 ■


C O R O L L A R Y 6.13 The vector area of a parallelogram (Figure 6.21) with adjacent sides a and b is a b.

FIGURE 6.21 Corollary 6.13.


C O R O L L A R Y 6.14 Let ABC C be a triangle and a, b and c be the position vectors of A, B and C (with respect to the
origin O), respectively. Then the vector area of the triangle ABCC is
1
[(b c ) + (c a ) + (a b)]
2
PROOF The vector area of the triangle ABC
C is given by
1

1
( AB AC ) = [(b a ) (c a )]
2 2
1
= [(b c ) + {b ( a )} + {( a ) c } + {( a ) ( a )}]
2
400 Chapter 6 Multiplication of Vectors

1
= [ b c b a a c + 0]
2
1
= [(b c ) + (c a ) + (a b)]
2

three points A, B, C are collinear if and only if the vector area of the triangle
Note that
ABC
C is 0. ■

T H E O R E M 6.30 The vector area of a parallelogram ABCD is


1


( AC BD)
2
PROOF

ABCD
Consider a parallelogram

6.22). Let O be the point of intersection of the diago-


(Figure

C and BD. Let AC = a and BD = b. Then, we have
nals AC

a
AO = OC =
2

b
and BO = OD =
2
Now
Vector area of the parallelogram ABCD = Vector area of OAB
+ Vector area of OBC
+ Vector area of OCD
+ Vector area of ODA
Hence vector area of the parallelogram ABCD is given by
1


[(OA OB) + (OB OC ) + (OC OD) + (OD OA)]
2

1 a b b a a b b a
= ÷ + ÷ + + ÷
2 2 2 ÷ 2 ÷ 2 2 2 2 2

1
= [(a b ) (b a ) + (a b ) (b a )]
8
1
= [(a b ) + (a b ) + (a b ) + (a b)]
8
1
= (a b )
2
1


= ( AC BD)
2

D C

b
2
a
2
O
b
a 2
2
A B

FIGURE 6.22 Theorem 6.30. ■


6.4 Vector Areas 401

T H E O R E M 6.31 Let A, B and C denote the internal angles of a triangle ABC C (Figure 6.23) and a, b, c denote the
lengths of the sides opposite to these angles respectively. Let
1
s = (a + b + c )
2

Then the following hold:


a b c
1. = =
sin A sin B sin C
(Note that this is called the sine rulee for triangles.)
2. The area of the triangle is given by

= s( s a) ( s b) ( s c)

PROOF 1. From Theorem 6.29, the vector area of the triangle is


1


( AB AC ) = ( BC BA)
2 2
1


= (CA CB)
2


| AB AC | = | BC BA| = |CA CB|

That is,
cb sin A = ac sin B = ba sin C
Hence
a b c
= =
sin A sin B sin C


2. Let BC = a, CA = b and AB = c . Then | a | = a, | b | = b and | c | = c. We have


BC + CA = BA = AB
Therefore

a+b+c=0

b a

FIGURE 6.23 Theorem 6.31.


402 Chapter 6 Multiplication of Vectors

Hence

(a + b ) × (a + b ) = ( - c ) × ( - c ) = c 2

a2 + b2 + 2(a × b) = c2
1
a × b = [c2 - (a2 + b2 )] (6.5)
2
Now
1


D= |CA ´ CB|
2
Therefore

4 D2 = | b ´ a |2

= | a |2 | b |2 - (a × b)2

= (| a || b | + a × b) (| a || b | - a × b)
æ 1 öæ 1 ö
= ç ab + (c2 - a2 - b2 )÷ ç ab - (c2 - a2 - b2 )÷ , [by Eq. (6.5)]
è 2 ø è 2 ø
1
= [c2 - (a - b)2 ][(a + b)2 - c2 ]
4
1
= [c - (a - b)](c + a - b) (a + b - c) (a + b + c)
4
1
= (a + b + c) (a + b - c) (b + c - a) (c + a - b)
4
1
= 2 s × 2( s - c) × 2( s - a) × 2( s - b)
4
= 4 s( s - a) ( s - b) ( s - c)

So

D = s( s - a) ( s - b) ( s - c) ■

T H E O R E M 6.32 For any q and f Î[0, p],


p
sin(q - f ) = sin q cos f - cosq sin f

PROOF Fix a right-handed

rectangular

Cartesian
coordinate system OXYZ and i , j and k be the unit
vectors along OX , OY and OZ, respectively (Figure 6.24). The required equation is trivial
if q = f or one of them is zero and the other is p p. Therefore, we can assume that q ¹ f and
{q, f} ¹ {0, p
p}. Choose points P and Q in the XY-plane
Y such that OP = OQ = 1, OP makes an
angle f and OQ makes an angle q with the positive direction of the X X-axis. Then the points O,
P and Q are non-collinear, since q ¹ f Î [0, p p] and {q, f} ¹ {0, p
p}. Now, we have


OP = (cos f )i + (sin f ) j


and OQ = (cos q )i + (sin q ) j
Therefore


OP ´ OQ = (sin q cos f - cosq sin f )k
6.4 Vector Areas 403

But


OP OQ = (OP )(OQ) sin(OP, OQ) n


= sin(OP, OQ) n


where n is the unit vector orthogonal to the OPQ plane (= OXY
Y plane) such that (OP, OQ, n) is
a right-handed system. Also,


(OP, OQ) = q f and n = k if q > f


and (OP, OQ) = f q and n = k if f > q
In either case, we have


OP OQ = sin(q f )k
and hence
sin(q f ) = sin q cos f cos q sin f


k

q
f
→ X
i

j

FIGURE 6.24 Theorem 6.32. ■

QUICK LOOK 9

Though we have proved the above theorem only for q f, as we have learnt it from trigonometry.
and f [0, p
p], this holds good for all real numbers q and

We conclude this section with few examples in which we make use of the results proved in this section and earlier
sections.

Example 6.13

the area of the parallelogram having 2 i
Find 3 j and = ( 3 0) i ( 2 0) j + (0 + 9)k
3i k as adjacent sides.
= 3i + 2 j + 9 k
Solution: The vector area of the given parallelogram
Therefore the area of the parallelogram is
is given by

i j k | 3i + 2 j + 9k | = 32 + 22 + 92 = 94 sq. units

( 2 i 3 j ) ( 3i k ) = 2 3 0
3 0 1
404 Chapter 6 Multiplication of Vectors

Example 6.14

Find the vector area of the triangle having vertices


(1, 2, 3), (2, 5, 1) and ( 1, 1, 2). Also find its magnitude.


AC = ( i + j + 2k ) (i + 2 j + 3k )
Solution: Let A = (1, 2, 3), B = (2, 5, 1) and C = ( 1,
= 2i j k
1, 2). Then the position vectors of A, B and C with
respect to the origin are Therefore


OA = i + 2 j + 3k 1


Vector area of ABC = ( AB AC )

2
OB = 2 i + 5 j k

i j k
and OC = i + j + 2k 1
= 1 3 4
2
2 1 1
Then

1
AB = OB OA = ( 7 i + 9 j + 5k )
2

= (2 i + 5 j k ) (i + 2 j + 3k ) The magnitude of the vector area of ABC
C is

= i + 3 j 4k 1 1
49 + 81 + 25 = 155 sq. units
2 2

Example 6.15

For any vector a, prove that Similarly

| a i |2 + | a j |2 + | a k |2 = 2 | a |2 | a j |2 = a12 + a32

and | a k |2 = a12 + a22
Solution: a = a1i + a2 j + a3k . Then we have
Hence
i j k
2
a i = a1 a2 a3 = a3 j a2 k | a i |2 + | a j | + |a k |2 = (a22 + a32 ) + (a12 + a32 ) + (a12 + a22 )
1 0 0 = 2((a12 + a22 + a32 )

Therefore = 2 | a |2

|a i |2 = a22 + a32

6.5 | Scalar Triple Product


We shall introduce the concept of scalar triple product and discuss certain properties and geometrical interpretation
of this.

DEFINITION 6.6 The scalar triple product of three vectors a, b and c is defined as (a b) ×c and is denoted by

[a b c ]. This is also called the box product or simply box [a b c ].

QUICK LOOK 10


1. [a b c ] is a scalar, since it is the dot (scalar) product 3. If a, b and c are non-zero and non-coplanar vectors,

of the vectors a b and c. then [a b c ] 0.

2. [a b c ] = 0 a = 0 or b = 0 or c = 0, or a, b or b , c or

c , a are collinear, or c is perpendicular to a b.
6.5 Scalar Triple Product 405


For non-zero and non-coplanar vectors a, b and c, the scalar triple product [a b c ] can be interpreted geometri-

cally as the volume of the parallelopiped with a, b and c as coterminus edges. This is proved in the following.


T H E O R E M 6.33

a, b and c

Let be non-zero non-coplanar vectors. Let O, A, B and C be points

= a,

that OA

such

OB = b and OC = c . Let V be the volume of the parallelopiped with OA, OB and OC as
coterminus edges. Then

1. (a b) ×c = V if (a, b, c ) is a right-handed system.

2. (a b)×c = V if (a, b, c ) is a left-handed system.

PROOF Since a, b and c are non-coplanar, the

O,
A,

points

B and C are non-coplanar. Complete the


parallelopiped OADBFCGE E having OA, OB and OC as coterminus edges (Figure 6.25).

C
F
n

c
G q E

b
O B

a M

FIGURE 6.25 Theorem 6.33.

(a, b,
c

1. Suppose that ) is a right-handed system. Draw the perpendicular CM M to the plane



deter-
mined by OA and OB and N be the foot of the perpendicular to the support of a b from C.

Let n be the unit vector in the direction of a b. Then ( a , b , n) is a right-handed system. Let

q be the angle between a b and c; that is, CON = q. Then
V = (Area of the base parallelogram OADB) (Height of the vertex C from the base)

= | a b | (CM )

= | a b | (ON )

= | a b | (OC ) cos q

= | a b || c | cos q

= (a b) ×c

= [a b c ]

2. Suppose that (a, b, c ) is a left-handed system. Then (a, b, c ) is a right-handed system and
hence

V = (a b) ×( c ) = [(a b)×c ]

Therefore

(a b) ×c = V ■
406 Chapter 6 Multiplication of Vectors


T H E O R E M 6.34 Let a, b and c be three non-zero vectors such that no two of them are collinear. Then

[a b c ] = 0 a, b and c are coplanar

PROOF Suppose that a, b and c are coplanar. Since a b is perpendicular to the plane determined by

a and b, it follows that a b is perpendicular to c and hence

(a b) ×c = 0

Conversely,
suppose that ( a b) ×c = 0. Then a b is perpendicular
to c. But, by the definition

of a b, we have that a b is perpendicular to both a and b. Therefore a b is perpendicular

to a, b and c. Thus a, b and c are coplanar. ■


C O R O L L A R Y 6.15 Four distinct points A, B, C and D are coplanar if and only if [ AB AC AD] = 0.
PROOF This


follows

Theorem 6.34 and the fact that A, B, C and D are coplanar if and only if
from
AB, AC and AD are coplanar. ■


T H E O R E M 6.35 Let OXYZ Z be a rectangular

Cartesian

coordinate system and i , j and k be unit vectors along


the positive axes OX , OY and OZ, respectively. Let

a = a1i + a2 j + a3k

b = b1i + b2 j + b3k

c = c1i + c2 j + c3k

be any three vectors. Then the scalar triple product [a b c ] is precisely given by

a1 a2 a3

[a b c ] = b1 b2 b3
c1 c2 c3

PROOF From Theorem 6.24, we have



a b = (a2 b3 a3b2 )i + (a3b1 a1b3 ) j + (a1b2 a2 b1 )k

= (a2 b3 a3b2 )i (a1b3 a3b1 ) j + (a1b2 a2 b1 )k

Therefore

[ a b c ] = (a b)×c
= (a2 b3 a3b2 )c1 (a1b3 a3b1 )c2 + (a1b2 a2 b1 )c3
a1 a2 a3
= b1 b2 b3
c1 c2 c3 ■

C O R O L L A R Y 6.16 Any vectors



a = a1i + a2 j + a3k

b = b1i + b2 j + b3k

c = c1i + c2 j + c3k
6.5 Scalar Triple Product 407

are coplanar if and only if the following determinant is zero:

a1 a2 a3
b1 b2 b3 = 0
c1 c2 c3


T H E O R E M 6.36 For any vectors a, b and c,

[a b c ] = [b c a ] = [c a b ]

that is

(a b)×c = (b c )×a = (c a )×b

PROOF If one of a, b and c is 0 or if any two of them are collinear, then the equality trivially holds.
Therefore, we can assume that all of them are non-zero and no-two of them are collinear. Also,

we can assume
that a , b and c are non-coplanar. If ( a , b , c ) is a right-handed system, then so

are (b, c , a ) and (c , a, b) and hence, by the Theorem 6.35

(a b) ×c = (b c )×a = (c a )×b = Volume of the parallelopiped = V

If (a, b, c ) is a left-handed system, then so are (b, c , a ) and (c , a, b) and hence

(a b) ×c = V = (b c )×a = (c a )×b ■


T H E O R E M 6.37 For any three vectors a, b and c,

(a b) ×c = a ×(b c)

(That is, in a scalar triple product, the operations dot and cross can be interchanged.)
PROOF From Theorem 6.36,

(a b) ×c = (b c ) ×a = a ×(b c)

since dot product is commutative. ■


C O R O L L A R Y 6.17 Let a , b and g be three non-coplanar vectors and

a = a1a + a2 b + a3g

b = b1a + b2 b + b3g

c = c1a + c2 b + c3g

be any vectors. Then a, b and c are coplanar if and only if

a1 a2 a3
b1 b2 b3 = 0
c1 c2 c3
408 Chapter 6 Multiplication of Vectors

PROOF By Theorem 6.28, we have



b g g a a b

a b = a1 a2 a3
b1 b2 b3

= (a2 b3 a3b2 )(b g ) (a1b3 a3b1 )(g a ) + (a1b2 a2 b1 )(a b)

Therefore

[ a b c ] = (a b)×c

= (a2 b3 a3b2 )(b g )×(c1a ) (a1b3 a3b1 )(g a )×(c2 b )

+ (a1b2 a2 b1 )(a b )×(c3g ) [since (b g )×b = 0 = (b g )×g , etc.]

= [(a2 b3 a3b2 )c1 (a1b3 a3b1 )c2 + (a1b2 a2 b1 )c3 ][a b g ]

since, (b g )×a = (g a )×b = (a b )×g = [a b g ] (see Theorems 6.36 and 6.37). Therefore
a1 a2 a3

[a b c ] = b1 b2 b3 [a b g ]
c1 c2 c3

Since a , b and g are coplanar, [a b g ] 0. Therefore

a, b and c are coplanar [a b c ] = 0

a1 a2 a3
b1 b2 b3 = 0
c1 c2 c3 ■

QUICK LOOK 11

1. During the course of the proof of Corollary 6.17, we 2. Using the property that determinant of product of
have proved the following result. two square matrices is equal to the product of the

If a , b , g are non-coplanar vectors and determinants of the matrices, the following result
can be easily proved:

a = a1a + a2 b + a3g If l , m, n, a, b, c are two sets
of vectors
each of
which is expressed in terms of i , j and k then
b = b1a + b2 b + b3g
l l ×a l ×b
c = c1a + c2 b + c3g
(i) [l m n] (a b) = m m ×a m ×b
Then
n n ×a n ×c
a1 a2 a3
l ×a l ×b l ×c
[a b c ] = b1 b2 b3 [a b g ]
c1 c2 c3 (ii) [l m n] [a b c ] = m ×a m ×b m ×c

n ×a n ×b n ×c


T H E O R E M 6.38 The volume of a tetrahedron with a, b and c as coterminus edges is
1
|[a b c ]|
6
6.5 Scalar Triple Product 409


PROOF Let OABCC be a tetrahedron and OA = a, OB = b and OC = c . Let V be the volume of the tetra-
hedron OABCC (Figure 6.26). Then the volume is given by
1
V = (Area of the base DOAB)(Length of the perpendicular from C to the base DOAB)
3

Draw perpendicular CN N from C to the triangle OAB and perpendicular CM


M from C to the

supporting line of a ´ b.

M C
a b 90°

O
b
a N

FIGURE 6.26 Theorem 6.38.

Then
CN = OM

= Length of the projection of c on a ´ b

|(a ´ b) × c |
=
|a ´ b|

|[a b c ]|
=
|a ´ b|

Also,
1
Area of DOAB = |a ´ b|
2
Therefore

1 æ 1 ö æ |[a b c ]| ö |[a b c ]|
V = ç | a ´ b |÷ ç =
3è 2 ø è | a ´ b | ÷ø 6 ■

C O R O L L A R Y 6.18 The volume of a tetrahedron whose vertices are A, B, C and D is


1


(|[ AB AC AD]|)
6


PROOF Since AB, AC and AD are coterminus edges of the tetrahedron ABCD, it follows from
Theorem 6.38 that the volume of the tetrahedron is
1


(|[ AB AC AD]|)
6 ■
410 Chapter 6 Multiplication of Vectors

Example 6.16

Consider
the vectors
a = 2 i - j + k , b = i - 3 j - 5k and = 2(12 - 20) + 1(- 4 + 15) + 1(- 4 + 9)

c = 3i - 4 j - 4k . Prove that they are coplanar.
=0
Solution: For the given vectors we have
Hence a, b, c are coplanar.

[ a b c ] = (a ´ b ) × c
2 -1 1
= 1 -3 -5
3 -4 -4

Example 6.17


a, b and
Let c be non-coplanar
vectors and let a = 3a + AC = g - a = - 6a + 6c

2b - 5c , b = - 3a + 8b - 5c , g = - 3a + 2b + c and d = - a +


AD = d - a = - 4a + 2b + 2c
4b - 3c be four points. Prove that a , b , g and d are
coplanar. Therefore

Solution: Let A, B, C and

D be four points and O -6 6 0



be the

origin
such that OA = a , OB = b , OC = g [ AB AC AD] = - 6 0 6 [a b c ] (see Quick Look 11)
and OD = d . Then
-4 2 2


AB = b - a
= [- 6(- 12) - 6(- 12 + 24)][a b c ] = 0

= (- 3a + 8b - 5c ) - (3a + 2b - 5c )


Therefore AB, AC and AD are coplanar. Hence the points

= - 6a + 6b A, B, C and D are coplanar. So a , b , g and d are coplanar.

Example 6.18

= 2(- 1) + 3[- 1 - (- 3)] + 0(3)
Find
the of the parallelopiped with 2 i - 3 j ,
volume
i + j - k and 3i - k as coterminus edges. =4

Solution: Let a = 2 i - 3 j , b = i + j - k and c = 3i - k . Therefore the volume of the parallelopiped is
Then
|[a b c ]| = 4
2 -3 0

[a b c ] = 1 1 - 1
3 0 -1

We have discussed in the previous chapter about the vector equation of a plane in various forms. In the following,
we obtain vector equation of a plane using dot, cross and scalar triple products. Also, we introduce the concept of
skew lines and derive a formula for the shortest distance between two skew lines. In the following, we fix a point O as

the origin of reference and adopt the notation that A(a ) is a point, that is, a is the position vector of a point A with
respect to the origin O. Let us first prove the following.


T H E O R E M 6.39 Let A(a ) be a point and b and c be two non-collinear vectors. Then the vector equation of the

plane passing through A and parallel to the vectors b and c is

[r b c ] = [a b c ]
6.5 Scalar Triple Product 411


PROOF Let P(r ) be any point in the required plane. We may assume that A P. Then


P lies in the plane Vectors AP, b and c are coplanar


[ AP b c ] = 0


AP ×(b c ) = 0

(r a )×(b c ) = 0

r ×(b c ) = a ×(b c )

[r b c ] = [a b c ]

Thus the equation of the plane passing through A and parallel to b and c is

[r b c ] = [a b c ] ■


T H E O R E M 6.40 Let A(a ) and B(b) be two given distinct points and c a vector. Then the vector equation of the

plane passing through A and B and parallel to c is

[r b c ] + [r c a ] = [a b c ]

PROOF Consider an arbitrary point P(r ) A. Then


P(r ) lies in the required plane Vector AP AB is perpendiicular to the plane


AP AB is perpendicular to c


( AP AB)×c = 0


AP ×( AB c ) = 0

(r a )×[(b a ) c ] = 0

(r a )×(b c + c a ) = 0

r ×(b c ) + r ×(c a ) = a ×(b c )

[r b c ] + [r c a ] = [a b c ]

Thus the vector equation of the required plane is



[r b c ] + [r c a ] = [a b c ] ■


T H E O R E M 6.41 A(a ), B(b) and C (c ) be three given non-collinear points. Then the vector equation of the
plane passing through A, B and C is

[r b c ] + [r c a ] + [r a b ] = [a b c ]

PROOF Let be the plane passing through A, B and C and P(r ) be an arbitrary point. Then

P(r ) lies on A, B, C and P are coplanar


PA, PB and PC aree coplanar

a r , b r and c r are coplanar
412 Chapter 6 Multiplication of Vectors


[a r b r c r ] = 0

(a r )×[(b r ) (c r )] = 0

r ×[(b c ) + (c a) + (a b)] = a ×(b c)

[r b c ] + [r c a ] + [r a b ] = [a b c ]

Thus the vector equation of the plane is



[r b c ] + [r c a ] + [r a b ] = [a b c ] ■


T H E O R E M 6.42 The vector equation of the plane containing the line r = a + tb, t  and perpendicular to the

plane r ×c = q is [r b c ] = [a b c ].

PROOF Recall that the vector c is normal to the plane r ×c = q. Since the required plane contains the line

r = a + tb, t , it should pass through the point a and is parallel to the vectors b and c. Thus, by

Theorem 6.39, the equation of the required plane is [r b c ] = [a b c ]. ■

DEFINITION 6.7 Skew Lines Two lines in the space are said to be skew lines if there is no plane containing
both the lines.

Note that if two lines intersect, then there is a plane containing both of them and hence they are not skew lines.
Therefore, two skew lines have no common point. However, they will have a common perpendicular. In the following
we prove that any two skew lines have a common perpendicular and derive a formula to find its length.

T H E O R E M 6.43 Let L1 and L2 be two skew lines. Then there exists unique pair (P, Q) of points with P on L1 and
Q on L2 such that PQ is perpendicular to both L1 and L2.

PROOF Let A(a ) and C (c ) be points on L1 and L2, respectively, and let b and d be parallel to L1 and L2,
respectively (Figure 6.27). Then the equations of L1 and L2 are

r = a + tb

and r = c + sd

where r and s are scalars. Observe that L1 and L2 are not parallel if and only if | b ×d | | b || d |.
Consider the following simultaneous equations:

(b ×d )s | b |2 t = (a c )×b (6.7)
2
| d | s | b ×d | t = (a c )×d (6.8)

A
b
P

90°
L1

90°
L2
C Q

FIGURE 6.27 Theorem 6.43.


6.5 Scalar Triple Product 413


Since | b || d | | b ×d |, Eqs. (6.7) and (6.8) have unique common solution, say (s1, t1) Let P and Q be
the points on L1 and L2 corresponding to t1 and s1, respectively. Now, we have


b ×PQ = b ×[(c + s1d ) (a + t1b)]

= b ×[(c a ) + ( s1d t1b)]

= b ×(c a ) + s1b ×d t1b ×b
= 0 [from Eq. (6.7)]


Therefore, PQ is perpendicular to b and hence to L1 (since b is parallel to L1). Similarly, using
Eq. (6.8), we can prove that PQ is perpendicular to L2.
Next,
we prove that P and Q are unique. If P and Q are points on L1 and L2, respectively,

will
such that P Q is perpendicular to both L1 and L2, then the corresponding scalars s and t satisfy
it follows that P = P and Q = Q. Thus P and Q
Eqs. (6.7) and (6.8). By the uniqueness of (s1, t1)

are unique on L1 and L2, respectively, such that PQ is perpendicular to both L1 and L2. ■
Note that, if P and Q are points on L1 and L2 respectively, then PQ P Q . Therefore PQ is the shortest distance
between L1 and L2.


T H E O R E M 6.44 The shortest distance between two skew lines whose equations are r = a + tb and r = c + sd, s,
t , is

|(a c )×(b d )|

|b d |


PROOF

P and Q on L 1 and
By Theorem 6.43,, there are unique points L2, respectively, such that PQ is
perpendicular

to
both

L1 and L2. Then PQ is parallel to b d. Choose points A and C such that



OA = a and OC = c , where O is the origin. Now, we have


PQ = Magnitude of the projection vector of AC or PQ


= Magnitude of the projection vector of AC on b d

|(c a )×(b d )|
= (see Theorem 6.1)
|b d |

|(a c ) ×(b d )|
=
|b d |

Note that PQ is the shortest distance between L1 and L2. ■

Example 6.19

Let A = (2, 3, 1), B = (4, 5, 2) and C = (3, 6, 5). Find the Then, by Theorem 6.41, the equation of the plane passing
equation of the plane passing through A, B and C. through A, B and C is

Solution: Let O be the origin. Then, we are given that [r b c ] + [r c a ] + [r a b ] = [a b c ]


a = OA = 2 i + 3 j k x y z x y z x y z 2 3 1

4 5 2+ 3 6 5 + 2 3 1= 4 5 2
b = OB = 4 i + 5 j + 2k
3 6 5 2 3 1 4 5 2 3 6 5


c = OC = 3i + 6 j + 5k
Alternate Solution: Problems of this type can be solved

Let r = xi + yj + zk be the position vector (with respect directly as follows without referring to Theorem 6.41.
to the origin O) of an arbitrary point P in the space.
414 Chapter 6 Multiplication of Vectors


P(r ) lies on the plane passing through A, B and C and


AP, AB and AC are coplanar x 2 y 3 z+1

[ AP AB AC ] = 2 2 3
[ AP AB AC ] = 0
1 3 6
We have = (x 2) (12 9) ( y 3) (12 3)


AP = r a = ( x 2)i + ( y 3) j + (z + 1)k + (z + 1) (6 2)


AB = b a = 2 i + 2 j + 3k = 3x 9 y + 4z + 25

Thus, the equation of the plane passing through A, B and


AC = c a = i + 3 j + 6k
C is
3x 9 y + 4z + 25 = 0

Example 6.20

Find the shortest distance between the following skew Then


lines:
a c = 4j

L1 : r = (i 2 j k ) + t(4 i 3k )

i j k

L2 : r = (i + 2 j k ) + s(2 i 4 j 5k ) b d= 4 0 3 = 12 i + 14 j 16k
2 4 5
Solution: Let

a = i 2j k From Theorem 6.44, the shortest distance between L1
and L2 is
b = 4 i 3k
|(a c )×(b d )| |( 4 j )×( 12 i + 14 j 16k )|
=
c = i + 2j k |b d | | 12 i + 14 j 16k |

d = 2i 4 j 5k | ( 4)14 | 56 28
= = =
(12) + (14) + (16)
2 2 2
596 149

As per note under Theorem 6.22, the following is an alternate proof of Theorem 6.22.


T H E O R E M 6.45 For any three vectors a, b, c

a (b + c ) = a b+a c

PROOF r be any vector. Since the dot product is distributive

r ×[a (b + c ) (a b) (a c )]

= r ×[a (b + c )] r ×(a b) r ×(a c )

= (r a ) ×(b + c ) r ×(a b) r ×(a c ) ( ∵ dot and cross can be interchanged)

= (r a)×(b + c ) (r a )×b (r a)) ×c ( ∵ dot and cross can be interchanged)

= (r a )×[b + c b c ] ( ∵ dot product is distributive)
=0

This is true for any vector r. Therefore

a (b + c ) (a b ) (a c ) = 0

a (b + c ) = a b + a c ■
6.6 Vector Triple Product 415

6.6 | Vector Triple Product


The concept of scalar triple product of three vectors is introduced in the previous section and using this we have
derived equations of a plane in different forms. In this section we introduce the notion of the vector triple product of
three vectors and discuss certain properties.

DEFINITION 6.8 For any three vectors a, b and c , (a b) c is called the vector triple productt of a, b and c.

In respect of scalar triple products, we have proved that



(a b) ×c = a ×(b c)
Such type of associativity does not hold for vector triple products. For, consider the following example.

Example 6.21

Let a = i + j + 2k, b = i + 2 j + k and c = 2 i + j + k . Prove
i j k


that associatively does not hold. That is prove that
b c= 1 2 1 =i + j 3k
(a b ) c a (b c ) 2 1 1
Solution: For the given vectors we have i j k

i j k a (b c) = 1 1 2 = 5i + 5 j

a b = 1 1 2 = 3i + j + k 1 1 3
1 2 1
Therefore (a b) c a (b c ).
i j k

(a b) c = 3 1 1 = 5j 5k
2 1 1


T H E O R E M 6.46 The following hold for any vectors a, b and c.

1. (a b) c = (a ×c )b (b ×c )a

2. a (b c ) = (a ×c )b (a ×b)c

PROOF 1. If a and b are collinear, say a = tb, t , then

(a b ) c = 0 c = 0

and (a ×c )b (b ×c )a = (tb ×c )b (b ×c ) (tb) = 0

Also, if c is parallel to a b, then c is perpendicular to both a and b
and hence (1) holds.

so that a ×c = 0 = b ×c and therefore

(a b) c = 0 = (a ×c )b (b ×c )a

Therefore, we can assume that a and b are non-collinear
and

c is not parallel

to a b.
Fix the origin O and select
points

A and B such that OA = a and OB = b. Let i be the


unit vector in the direction of OA and let j be the unit vector perpendicular to i in the
OAB plane. Let k be the unit vector perpendicular to the OAB plane such that (i , j , k ) is a
right-handed system of orthogonal unit vectors. Note that the plane OAB is same as the XY
plane. Now, we have

a = a1i

b = b1i + b2 j

c = c1i + c2 j + c3k
416 Chapter 6 Multiplication of Vectors

Therefore

(a ´ b) ´ c = (a1b2 k ) ´ (c1 i + c2 j + c3 k )

= a1b2 c1 j - a1b2 c2 i

= a1c1 (b1 i + b2 j ) - (b1c1 + b2 c2 )a1 i

= (a × c )b - (b × c )a
2. We have

a ´ (b ´ c ) = -[(b ´ c ) ´ a ]

= -[(b × a )c - (c × a )b]

= (a × c )b - (a × b)c ■


C O R O L L A R Y 6.19 Let a, b and c be vectors such that a and b are non-collinear and b is perpendicular to neither

a nor c. Then

(a ´ b ) ´ c = a ´ (b ´ c )

if and only if the vectors a and c are collinear.
PROOF Suppose that

(a ´ b ) ´ c = a ´ (b ´ c )

Then

(a × c )b - (b × c )a = (a × c )b - (a × b)c

Therefore

(b × c )a = (a × b)c

Since b is not perpendicular to c and to a, we get that b × c and a × b are non-zero scalars.

Therefore a and c are collinear.

Conversely, suppose that a and c are collinear. Then c = la for some scalar l. Now, consider

(a ´ b ) ´ c - a ´ (b ´ c ) = (a ´ b ) ´ ( l a ) - a ´ ( b ´ l a )

= l[{(a ´ b) ´ a} - {a ´ (b ´ a )}]

= l[(a × a )b - (b × a )a - (a × a )b + (b × a )a ]

=0
Therefore

(a ´ b ) ´ c = a ´ (b ´ c ) ■


T H E O R E M 6.47 Let a, b and c be any vectors such that b is perpendicular to both a and c. Then

(a ´ b ) ´ c = a ´ (b ´ c )

PROOF We have a × b = 0 = c × b. Now,

(a ´ b) ´ c = (a × c )b - (b × c )a = (a × c )b

a ´ (b ´ c ) = (a × c )b - (a × b)c = (a × c )b

Therefore

(a ´ b ) ´ c = a ´ (b ´ c ) ■
6.6 Vector Triple Product 417

In the following we derive formulae for scalar and vector products of four vectors and apply there to get a repre-
sentation of vectors in terms of three non-coplanar vectors.


T H E O R E M 6.48 The following hold for any four vectors a, b, c and d.

a ×c a ×d
1. (a b) ×(c d ) = = (a ×c )(b ×d ) (a ×d )(b ×c )
b ×c b ×d
and, in particular,

| a b |2 = | a |2 | b |2 (a ×b)2

2. (a b ) (c d ) = [a c d ]b [b c d ]a = [a b d ]c [a b c ]d
PROOF 1. We have

(a b) ×(c d ) = a ×[b (c d )] (by Theorem 6.37)

= a ×[(b ×d )c (b ×c )d ] [by Theorem 6.46, part (2)]

= (b ×d ) (a ×c ) (b ×c ) (a ×d )

a ×c a ×d
=
b ×c b ×d

and

|a b |2 = (a b)×(a b)

= (b ×b) (a ×a ) (b ×a ) (a ×b)

= | a |2 | b |2 (a ×b)2

2. We have

(a b ) (c d ) = [a ×(c d )]b [b ×(c d )]a [by Theorem 6.466, part (1)]

= [a c d ]b [b c d ]a

(a b ) (c d ) = [(a b) ×d ]c [(a b) ×c ]d [by Theorem 6.46, part (2)]

= [a ×(b d )]c [a ×(b c )]d

= [a b d ]c [a b c ]d ■

If a, b and c are any non-coplanar vectors, then we have proved earlier in Chapter 5 (Theorem 5.20) that any

vector r can be uniquely expressed as xa + yb + zc for some triad (x, y, z) of real numbers. We express these compo-
nents x, y and z in terms of scalar triple products in the following.


T H E O R E M 6.49 Let a, b and c be non-coplanar vectors. Then any vector r can be expressed as

[b c r ] [c a r ] [a b r ]
r = a+ b+ c
[a b c ] [a b c ] [a b c ]

PROOF By replacing d with r in part (2) of Theorem 6.48, we get that

[a c r ]b [b c r ]a = [a b r ]c [a b c ]r
Therefore

[a b c ]r = [b c r ]a + [c a r ]b + [a b r ]c
418 Chapter 6 Multiplication of Vectors


Since a, b and c are non-coplanar, [a b c ] 0 and therefore we get that

[b c r ] [c a r ] [a b r ]
r = a+ b+ c

[a b c ] [a b c ] [a b c ] ■

Note: In the above, if r = xi + yj + zk , then

[b c r ] [c a r ] [a b r ]
x = , y = and z =
[a b c ] [a b c ] [a b c ]


T H E O R E M 6.50 Let a, b and c be non-coplanar vectors. Then b c , c a and a b are also non-coplanar and

any vector d can expressed in terms of b c , c a and a b as

a ×d b ×d c ×d
d = (b c ) + (c a ) + (a b)
[a b c ] [a b c ] [a b c ]

PROOF We have

[b c c a a b] = [a b c ]2 0

since a, b and c are non-coplanar. Therefore, b c , c a and a b are also non-coplanar. Let

d = x(b c ) + y(c a ) + z(a b)

Since a ×(c a ) = 0 = a ×(a b), we have

a ×d = x[a ×(b c )]

= x[a b c ]

Similarly,

b ×d = y[b c a ] = y[a b c ]

and c ×d = z[c a b] = z[a b c ]


a ×d b ×d c ×d
d = (b c ) + (c a ) + (a b)
[a b c ] [a b c ] [a b c ] ■

Example 6.22

a = i + j + k , b = 2 i j + 3k , c = i j and d = 6 i +
a, b and
Therefore

c are coplanar and hence so are
2 j + 3k . Express d in terms of b c , c a and a b. b c , c a and a b. We have

Solution: First note that [a b c ] = 5

[b c c a a b] = [a b c ]2 a ×d = (1 6) + (1 2) + (1 3) = 11

1 1 1
2 b ×d = (2 6) + ( 1)2 + (3 3)) = 19

= 2 1 3 c ×d = (1 6) + ( 1)2 = 4
1 1 0
From Theorem 6.50, we have
= 52 = 25 11
19 4
d = (b c ) + (c a ) + (a b)
5 5 5
Worked-Out Problems 419

WORKED-OUT PROBLEMS
Single Correct Choice Type Questions

1. Let a = 5i j + 8k and b = i + j + lk . If the vectors 4. The vectors lxi 6 j + 3k and xi + 2 j + 2l xk make

a + b and a b are orthogonal to each other then |l | an obtuse angle with each other for all real x. Then l
is equal to belongs to the interval
(A) 80 (B) 90 (C) 88 (D) 99 4 4
(A) , 0÷ (B) ,0
3 3
Solution: By hypothesis

(C)
4
,0 (D) 0,
4
(a + b) ×(a b) = 0 3 3
Therefore Solution: By hypothesis,

| a |2 | b |2 = 0 90 = 2 + l 2
(l xi 6 j + 3k )×( xi + 2 j + 2 l xk ) < 0
This implies for all real x. Therefore
| l | = 88 l x2 + 6 l x 12 < 0
Answer: (C)
for all real x. This implies

2. If a is a unit vector and x is any vector such that 36 l 2 + 48 l < 0

( x + a ) ×( x a ) = 48 3l 2 + 4 l < 0
4
then | x | is equal to <l<0
(A) 4 (B) 5 (C) 6 (D) 7 3

Solution: We have When l = 0, then clearly



48 = ( x + a )×( x a) ( 6 j + 3k ) ×( xi + 2 j ) = 12 < 0

= | x |2 | a |2 Therefore

= | x |2 1 4
<l 0
3
Therefore
Answer: (B)
| x| = 7

Answer: (D) 5. Let a and b two vectors such that | a | = 2 2 and | b | = 3
and contain angle 45°. A parallelogram is constructed

with a 3b and 5a + 2b as adjacent sides. Then the
q is the angle between the vectors i + 3 j + 7k and
3. If
length of the larger diagonal is
i 3 j + 7k , then cos q is equal to
41 40 42 39 (A) 393 (B) 493 (C) 593 (D) 693
(A) (B) (C) (D)
59 59 59 59 Solution: The diagonal vectors are
Solution: Let
(5a + 2b) + (a 3b) = 6a b

a = i + 3 j + 7k
and (5a + 2b) (a 3b) = 4a + 5b

and b = i 3 j + 7k
Now
By Theorem 6.7,
| 6a b |2 = 36 | a |2 12a ×b + | b |2 (see Corollary 6.4)
1 9 + 49 41
cosq = = 1
59 59 59 = 36(8) 12(2 2 )(3) ÷+ 9
2
Answer: (A)
= 288 72 + 9 = 225
420 Chapter 6 Multiplication of Vectors

Therefore 1 1
(A) ( i + j + 4k ) (B) (4 i + j + k )
3 3
| 6a b | = 15
1 1
(C) (i 4 j + k ) (D) (i + 2 j + k )
Also 3 3

| 4a + 5b |2 = 16 | a |2 + 40(a ×b) + 25 | b |2 Solution: We have | c | = 2 which implies
1 x2 + y2 + z2 = 1 (6.9)
= 16(8) + 40(2 2 )(3) + 25(9)
2
a ×b 1
= 128 + 240 + 225 cos q = = (6.10)
| a || b | 2
= 593
a ×c x+y
cos q = = (6.11)
Therefore | a || b | 2

| 4a + 5b | = 593 Also
Hence this is the larger diagonal. y+z
cosq = (6.12)
Answer: (C) 2
From Eqs. (6.10), (6.11) and (6.12),
6. In the two-dimensional analytical plane having rect-
angular coordinate system, A B are two
points

and

on x+ y=1
the curve
y = 2 x+2
such that OA ×i = 1 and OB ×i = 2, and y+z=1
where i is the unit
vector in

the positive direction of


the X
X-axis. Then | 4OA OB| is equal to Substituting these values in Eq. (6.9) we get
(A) 10 (B) 12 (C) 8 (D) 6 x2 + (1 x)2 + x2 = 2
Solution: Let A = (x1, y1) and B = (x2, y2). Therefore
Therefore
y1 = 2x1 + 2
1
x = 1,
and y2 = 2x2 + 2 3
Now x = 1 y = 0. But by hypothesis y 0. Therefore


OA ×i = 1 x1 = 1 1 4 1
x= , y= ,z=
3 3 3
and OB ×i = 2 x2 = 2
and so
Therefore y1 = 2 and y2 = 16. Hence
1

c= (i 4 j + k)
OA = ( 1, 2) = i + 2 j 3


and OB = (2, 16) = 2 i + 16 j Answer: (C)

Therefore 8. If a, b and c are three vectors such that | a | = 3, | b | = 4

and | c | = 24 and sum of any two vectors is orthog-


4OA OB = ( 4 i + 8 j ) (2 i + 16 j )
onal to the third vector, then | a + b + c | is equal to

= 6i 8 j (A) 7 (B) 5 2 (C) 7 2 (D) 6 2
Hence Solution: By hypothesis


| 4OA OB| = ( 6)2 + ( 8)2 = 10 (a + b) ×c = 0

Answer: (A) (b + c ) ×a = 0

(c + a) ×b = 0
7. Let a = i + j and b = j + k and
the angle between a

and b be q. If c = xi + yj + zk such
that y 0, | c | = 2
Therefore
and makes angle q with a and b, then c is equal to

a ×b + b ×c + c ×a = 0
Worked-Out Problems 421


Now 11. Let a = i + j + 2k and b = i + 2 j + k and c be a unit

by a and b. If c is
vector in the plane determined
| a + b + c |2 = | a |2 + | b |2 + | c |2 + 2(a ×b + b ×c + c ×a ) perpendicular to the vector i + j + k and makes an

= 9 + 16 + 24 + 2(0) obtuse angle with a, then c is

= 49 j k j +k
(A) (B)
2 2
Hence | a + b + c | = 7.
Answer: (A) i k i +k
(C) (D)
2 2

9. If | a | = | b | = | a + b | = 1, then | a b | equals
Solution:
Since c is coplanar with a and b (notice
that

(A) 1 (B) 2 (C) 2 (D) 3 a and b are not collinear vectors), let c = xa + yb where
Solution: By hypothesis x and y are scalars. Now

|c | = 1 ( x + y)2 + ( x + 2 y)2 + (2 x + y)2 = 1 (6.13)
1 = | a + b |2 = | a |2 + | b |2 + 2(a ×b) = 1 + 1 + 2 cos q

where q is the angle between a and b. Therefore Also c is perpendicular to i + j + k , we have

1 c ×(i + j + k ) = 0
cosq =
2
( x + y) + ( x + 2 y) + (2 x + y) = 0
Hence 4x + 4y = 0

|a b |2 = | a |2 + | b |2 2(a ×b) x= y
1 y = x in Eq. (6.13), we have
= 1 + 1 2(1)(1) ÷
2
2x = ±1
=3
Answer: (D) so that

x=±
1
10. If a, b and c are mutually perpendicular vectors
2
having same magnitude, then the vector a + b + c is

equally inclined to each of a, b and c at an angle Therefore
1 1
(A) Cos 1 ÷ (B) Cos 1 ÷ 1 j
k
3 3 x= + c=
2 2 2
2 1
(C) Cos 1 ÷ (D) Cos 1 ÷ 1 j k
3 2 and x= c=
2 2 2
Solution: We have
But c makes obtuse angle with a. This means

| a + b + c |2 = | a |2 + | b |2 + | c |2 (∵ a ×b = b ×c = c ×a = 0)
j k
= 3K 2 c=
2

where K = | a | = | b | = | c |. Therefore because

|a + b + c | = K 3 1 2 1
a ×c = 0 + = <0
2 2 2
let a,
Now, a b and g be the angles of inclination of a +

b + c with a, b, c , respectively. Therefore Answer: (A)

a ×(a + b + c ) K2 1 12. Consider the
following two statements:

cos a = = = S1: Let a, b and c be unit vectors such that a is
| a || a + b + c | K (K 3 ) 3
perpendicular
to both b and c and further
the angle
Answer: (B) p
between b and c is . Then a = ±2(b c )
6
422 Chapter 6 Multiplication of Vectors


S2: The
points with position vectors a + b, a - b and That is

a + l b are collinear for all real values of l where a
l 6 - 3l 2 - 2 = 0
and b are non-collinear vectors.
Then, (l 2 + 1)2 (l 2 - 2) = 0 (∵ ± 1 and ± i are repeated roots)
(A) Both S1 and S2 are true
This gives
(B) S1 is true, but S2 is false
(C) S1 is false and S2 is true l=± 2
(D) Both S1 and S2 are false Answer: (C)

Solution: S1 : a is perpendicular to both b and c.
14. Let a = xi - j + k (x is a scalar) and b = 2 i - j + 5k .
This implies
If the scalar projection of a on b is 1/ 30 , then x is
equal to
a = l(b ´ c )
2 3 -5 -5
for some scalar l. Therefore (A) (B) (C) (D)
5 5 3 2

1 = | a |2 = l 2 | b ´ c |2 Solution: By hypothesis

= l 2 [| b |2 | c |2 - (b × c )2 ] (see Theorem 6.26) 1 a ×b
=
æ pö 30 | b |
= l 2 ç 1 - cos2 ÷
è 6ø 2x + 6
=
æ 3ö 30
= l2 ç 1 - ÷
è 4ø Therefore
l
2
-5
= x=
4 2
Answer: (D)
Therefore l = ±2 and so

15. If | a | = 3, | b | = 4, | c | = 1 and a + b + c = 0, then the
a = ± 2(b ´ c )
value of a × b + b × c + c × a is
Hence S1 is true. Let A, B and

C be the points whose
(A) 12 (B) -12 (C) -13 (D) 13
position vectors are a + b, a - b and a + lb, respectively.

Solution: We have
Now, A, B and C are collinear if and only if AB = xBC
for some scalar x. That is,
a + b + c = 0 Þ | a + b + c |2 = 0


(a - b) - (a + b) = x[a + lb - (a - b)] Þ | a |2 + | b |2 + | c | + 2å a × b = 0

That is, Þ 9 + 16 + 1 + 2å a × b = 0

- 2b = x(l + 1)b
Þ a × b + b × c + c × a = - 13
Therefore x(l + 1) = -2 for all real l and for some real Answer: (C)
x. This is not possible when l = -1, in which case, C = B.
Hence S2 is also true. a = 2 i + 3 j + k , b = 4 i + j and c = i + 3 j + 2k . If
16. Let
Answer: (A) d is such that d × a = 9, d × b = 7 and d × c = 6, then d is
equal to

13. The number of distinct of l for which
real values the (A) i + 3 j - 2k (B) i - 3 j + 2k
vectors - l 2 i + j + k , i - l 2 j + k and i + j - l 2 k are
coplanar is (C) - i + 3 j - 2k (D) i + 3 j + 2k

(A) 0 (B) 1 (C) 2 (D) 3 Solution: Let d = xi + yj + zk .
Solution: The given vectors are coplanar if and only if
d ×a = 9 Þ 2 x + 3y + z = 9 (6.14)

-l2 1 1 d ×b = 7 Þ 4x + y = 7 (6.15)
1 -l2 1 = 0 (see Corollary 6.16)
d × c = 6 Þ x + 3 y + 2z = 6 (6.16)
1 1 -l2
Worked-Out Problems 423

Solving Eqs. (6.14), (6.15) and (6.16) we have x = 1, y = 3 Now


and z = 2. Therefore
|a b |2 + | b c |2 + | c a |2 = 2(| a |2 + | b |2 + | c |2 ) 2 (a ×b)
d = i + 3 j 2k
3
Answer: (A) =6 2 (a ×b) 6 + 2 ÷ [by Eq. (6.17)]
2
17. The points A(2, 1, 1), B(1, 3, 5) and C(3, 4, 4) Answer: (C)
form the vertices of

(A) an equilateral triangle 20. The angle between
the two vectors a = 2 i + 2 j k
and b = 6 i 3 j + 2k is
(B) isosceles but not equilateral
1 1
(C) right angled (A) Cos 1 (B) Cos 1
3 2
(D) right angled isosceles
5 7 4
(C) Cos 1 (D) Cos 1 ÷
21 ÷
Solution: We have
21


AB = i 2 j 6 j
Solution: We have


CA = i + 3 j + 5 j

 a ×b = 12 6 2 = 4
BC = 2 i j + k

| a || b | = 36 + 9 + 4 4 + 4 + 1 = 21
AB + BC + CA = 0
Therefore the angle between the vectors, by Theorem
T 6.7, is
Therefore A, B, C form a triangle and

a ×b 4
BC ×CA = 2 3 + 5 = 0 Cos 1 ÷ = Cos 1 ÷
| a || b | 21
Therefore C = 90°. Answer: (D)
Answer: (C)
21. If the scalar product of the vector i + j + k with the
18. If the vectors 2 i + aj + 4k and 3i + 5 j 3k are at unit vector
in the direction
of the resultant
of the
right angles, then the value of a is vectors 2 i + 4 j 5k and li + 2 j + 3k is unity, then
18 1 l equals
(A) (B) 1 (C) (D) 1
5 5 (A) 5 (B) 2 (C) 1 (D) 1
Solution: We have Solution: Let

(2 i + aj + 4k )×( 3i + 5 j 3k ) = 0 r = (2 i + 4 j 5k ) + (li + 2 j + 3k )

6 + 5a 12 = 0 = ( 2 + l )i + 6 j 2 k
18
a= e = unit vectorr in the direction of r
5
( 2 + l )i + 6 j 2 k
Answer: (A) =
(2 + l )2 + 36 + 4

19. If a, b and c are unit vectors, then the value of
Therefore

|a b |2 + | b c |2 + | c a |2
e ×(i + j + k ) = 1
is less than or equal to
(2 + l ) + 6 2 = (2 + l )2 + 40
(A) 8 (B) 12 (C) 9 (D) 6
Solution: We have (l + 6)2 = (2 + l )2 + 40
12l + 36 = 4 l + 44
|a + b + c | 0
8l = 8
| a |2 + | b |2 + | c |2 2 a ×b 0
l =1
3 Answer: (C)
a ×b + b ×c + c ×a (6.17)
2
424 Chapter 6 Multiplication of Vectors


22. Let a = 2 i + 3 j + 2k and b = i + 2 j + k . The vector Assuming q is the angle between a and b we get

component
of a perpendicular to the direction of
b is 5 8 + 6 cosq = 0
1 1 1
(A) (i + j + k ) (B) (i j + k ) cosq =
3 3 2
1 1 q = 60°
(C) (i + j k ) (D) ( i + j + k )
3 3
Answer: (A)

Solution: Vector
component of a perpendicular to the

direction of b is given by (see Quick Look 3) a and
25. Angle between b is 2p / 3. If | b | = 2 | a | and the

vectors a + xb and a b are at right angles, then x is

a ×b 5 equal to
a 2 ÷ b = ( 2 i + 3 j + 2k ) (i + 2 j + k )
|b| 3 2 2 1 1
(A) (B) (C) (D)
5 3 5 3
1
= (i j + k)
3 Solution: We have
Answer: (B)
(a + xb)×(a b) = 0

23. If a is collinear with b = 3i + 6 j + 6k and a ×b = 27, | a |2 x | b |2 + ( x 1) (a ×b) = 0

then a is
2p
(A) 3(i + j + k ) (B) i + 2 j + 2k | a |2 4 x | a |2 + ( x 1)2 | a |2 cos =0
3
(C) 2 i + 2 j + 2k (D) i + 3 j + 3k
1 4x (x 1) = 0

Solution: a is collinear with b. This implies
2
x=
a = lb 5
Answer: (A)
for some scalar l. Now
26. A plane is at a distance of 5 units
a ×b = 27 from
the origin
and perpendicular to the vector 2 i + j + 2k . Then its

l (b ×b) = 27 equation is

l (9 + 36 + 36) = 27 (A) r ×(2 i + j + 2k ) = 15

1 (B) r ×(2 i + j + k ) = 5
l=
3 (C) r ×(i + 2 j + 2k ) = 15

Therefore (D) r ×(2 i + j + 2k ) = 5

1 Solution: Unit normal to the given plane is


a = b = i + 2 j + 2k
3 2 i + j + 2k
Answer: (B) n=
3

24. Let a and b be unit vectors. If the vectors x = a + 2b By Theorem 6.13, the plane equation

and y = 5a 4b are orthogonal
vectors, then the
(2 i + j + 2k )
angle between a and b is r× =5
3
(A) 60° (B) 30° (C) 45° (D) 75°
Solution: By hypothesis That is

r ×(2 i + j + 2k ) = 15
x ×y = 0
Answer: (A)
5 | a |2 8 | b |2 + 6 a ×b = 0
Worked-Out Problems 425


27. M is the foot of the perpendicular drawn from the Let q be the angle between a and b. Then
point A(2, 4, 3) onto the line joining the points P(1,
2, 4) and Q(3, 4, 5). Then the vector representing the 36 p2 = (a × b)2
point M is
= | a |2 | b |2 cos2 q £ | a |2 | b |2
1
(A) (19 i + 28 j + 40k ) = ( p2 - 4 + p2 + p2 + 4)(tan2 A + tan2 B + tan2 C )
9
1 = 3 p2 (tan2 A + tan2 B + tan2 C )
(B) (28 i + 9 j + 41k )
9
Hence
1
(C) (21i + 28 j + 19k )
9 tan2 A + tan2 B + tan2 C ³ 12
1
(D) (19 i + 28 j + 41k ) and is equal to 12 when q is either 0 or p.
p
9
Answer: (B)
Solution: By Theorem 5.29 (Chapter 5), the equation
of the line PQ is 29. In DABC
C (Figure 6.28), AB = BC = 8 and AC = 12.
P is a point on the side AB such that

AP = 1: 3.
: PB

r = (1 - t ) (i + 2 j + 4k ) + t(3i + 4 j + 5k ) Then the angle between the vectors CP and CA is


That is æ 3 5ö æ 3 3ö
(A) Cos-1 ç ÷ (B) Cos-1 ç ÷
r = (1 + 2t )i + (2 + 2t ) j + (4 + t )k (6.18) è 8 ø è 8 ø

Therefore, the position vector of M is given by Eq. (6.18). So æ 3 7ö æ 3 2ö


(C) Cos-1 ç ÷ (D) Cos-1 ç ÷

è 8 ø è 8 ø
AM = (- 1 + 2t )i + (- 2 + 2t ) j + (1 + t )k

Solution: Take A as origin and let AB = a and AC = b


Since AM is perpendicular to PQ = 2 i + 2 j + k , we have
so that


AM × PQ = 0
|a | = 8 = | b - a |
Therefore
and | b | = 12
2(- 1 + 2t ) + 2(- 2 + 2t ) + (1 + t ) = 0
- 5 + 9t = 0 A
I
5
t= P
9
3
Substituting the value t = 5/9 in Eq. (6.18), we have
12
1 B q
M = (19 i + 28 j + 41k )
9
8
Answer: (D)

28. Let p be real and | p| ³ 2. If A, B and C are variable FIGURE 6.28 Single correct choice type question 29.
angles such that
Now AB = BC = 8 implies
p2 - 4 tan A + p tan B + p2 + 4 tan C = 6 p

|a | = | b - a |
then the minimum value of tan2 A + tan2 B + tan2 C is

(A) 8 (B) 12 (C) 18 (D) 6 Þ |a |2 = | b |2 + |a |2 - 2a × b

Solution: Consider the vectors Þ 2a × b = | b |2

a = p2 - 4 i + pj + p2 + 4k Þ 2 |a || b | cos A = | b |2

and b = (tan A)i + (tan B) j + (tan C )k Þ 2(8)(12) cos A = 144
3
Þ cos A = (6.19)
4

a ×b = p2 - 4 tan A + p tan B + p2 + 4 tan C
426 Chapter 6 Multiplication of Vectors


Now the new position of OA makes an acute angle

with

a
the positive X X-axis. Then the new position of OA is
AP = 1
4 (A) (4 i 2 j 2k )

a
2
1
and CP = b = (a 4b ) 1
4 4 (B) ( 4 i + 2 j + 2k )
2
Again
(C) 2 j 2k


1 (D) 6 i 3k
| CP |2 = |a 4 b |2
16


Solution: Let

the new position of OA be r. Since r is


1 2 coplanar with OA and i , let
= [8 + 16 122 8(a ×b )]
16


r = x(OA) + yi
1 3 3
= 64 + 16 144 8(8) (12) ∵ cos A = ÷
16 4 4 = ( x + y)i + 2 xj + 2 xk
= 4 + 144 36


Since r is perpendicular to OA, we have
= 112


r ×OA = 0
Therefore

( x + y) + 4 x + 4 x = 0
|CP | = 112 = 4 7
y = 9x (6.20)


Let q be the angle between CP and CA. Then Also


CP ×CA |r | = 3 ( x + y)2 + 4 x2 + 4 x2 = 9
cosq =


|CP ||CA|
64 x2 + 8 x2 = 9 [by Eq. (6.20)]

(1/ 4) (a 4 b )×( b ) 1
= x=±
4 7 122 2 2
2
4 | b | a ×b so that
=
16 7 12
9
4 144 8(12) (3 / 4) y=∓
= 2 2
16 7 12
Now
144 18
=
4 7 12 1
r= (4 i j k)
2
126
= 1
4 7 12 or r= ( 4i + j + k)
2
21
=
8 7 But r makes acute angle with the positive X
X-axis. Hence

3 7 1
= r= (4 i j k)
8 2
Therefore Answer: (A)

31. If a, b, c are vectors such that a + b + 2c = 0 and
q = Cos 1 3 7
8 ÷ | a | = 1, | b | = 4 and | c | = 2, then the value of a ×b +

b ×c + c ×a is
Answer: (C) 13 13 17 17

(A) (B) (C) (D)


2 2 2 2
30. OA

= i + 2 j + 2k . In the plane of OA and i ,


rotate OA through 90° about the origin O such that
Worked-Out Problems 427


Solution: By hypothesis (d b)×c = 0


a+b+c= c BD×AC = 0

Therefore Therefore angle between AC


C and BD is 90°.
Answer: (C)
4 = | c |2 = | a + b + c |2
34. In ABC (Figure 6.29), if BC = a, CA = b and AB = c
= | a |2 + | b |2 + | c |2 + 2 (a ×b)

of the angle A meets the


and the internal bisector
C in L, then | AL|2 is equal to
side BC
= 1 + 16 + 4 + 2 (a ×b)
bca2 abc
Therefore (A) bc (B) bc
(b + c)2 (b + c)2
17 ab bc
a ×b + b ×c + c ×a = (C) a2 bc + (D) a2 bc
2 (b + c)2 (b + c)2
Answer: (D)

Solution:

BL : LC = c : b. Take
It is known

that A as
origin, let AB = a and AC = b so that |a | = c, | b | = b and
32. If a and b are unit vectors
such that | a + b | = 3 ,
the angle between a and b is A.
then (3a 4b)×(2a + 5b) is equal to
15 15 21 21 A
(A) (B) (C) (D)
2 2 2 2
Solution: We have A
2

A b
c
| a + b |2 = 3 2


1 + 1 + 2(a ×b) = 3 C

1 b
a ×b = L
2 c
b
B
Now
FIGURE 6.29 Single correct choice type question 34.
(3a 4b)×(2a + 5b) = 6 | a |2 20 | b |2 + 7(a ×b)
7

BL : LC = c : b. This means that the position vector


Now
=6 20 + AL of L is given by
2
21

ba + cb
= AL =
2 b+c
Answer: (C) Therefore

1
33. In quadrilateral ABCD, if | AL |2 = [b2 |a |2 + c2 | b |2 + 2bc(a ×b )]

(b + c )2

| AB|2 + |CD|2 = | BC |2 + | AD|2


1
= [b2 c2 + b2 c2 + 2b2 c2 cos A]
then the angle between the diagonals AC C and BD is (b + c)2
(A) 60° (B) 75° (C) 90° (D) 120° (b2 + c2 a2 )
1

= 2b2 c2 + 2b2 c2
Solution:

Take A as origin and let AB = b, AC = c and (b + c )2


2bc
AD = d. Then

2b2 c2 2bc + b2 + c2 a2
=
| AB|2 + |CD|2 = | BC |2 + | AD|2 (b + c)2 2bc

| b |2 + | d c |2 = (c b)2 + | d |2 bc
= [(b + c)2 a2 ]
(b + c)2
| b |2 + | d |2 + | c |2 2(d ×c ) = | c |2 + | b | 2(b ×c ) + | d |2
bc a2
d ×c = b ×c = bc
(b + c)2
Answer: (A)
428 Chapter 6 Multiplication of Vectors


35. a, b, c are unit vectors. If r is a vector such that It is known that

| r | = 2, a + b + c = r , a ×r = 1, b ×r = 3 / 2, then the angle

between c and r is

a + b + g
OG = (see Corollary 5.4)
1 3 3
1 1
(A) Cos ÷ (B) Cos ÷
4 4 Therefore
1 1


(C) Cos 1 ÷ (D) Cos 1 ÷ 9|OG |2 = |a + b + g |2
2 2 3

= |a |2 + | b |2 + | g |2 + 2 (b ×g )
Solution: We have
= 3R2 + 2R2 (cos 2A + cos 2B + cos 2C )
(a + b + c ) ×r = r ×r = 4
= 3R2 + 2R2[1 2 sin2 A + 1 2 sin2 B + 1 2 sin2C ]
a ×r + b ×r + c ×r = 4
3 = 9R2 4R2 sin2 A 4R2 sin2 B 4R2 sin2 C
1+
+ 2 cosq = 4
2
a b c
= 9R2 a2 b2 c2 ∵ = = = 2R÷
where q is the angle between c and r. Therefore sin A sin B sinC
3
2 cosq = Hence
2

1 2
3 |OG |2 = R2 (a + b2 + c2 )
q = Cos 1
÷ 9
4
Answer: (B)
Answer: (B)
37. Quadrilateral ABCD is inscribed in a circle of radius
36. In ABC C (Figure 6.30), if “O” is the circumcentre
R (Figure 6.31). If (AB)2 + (CD)2 = 4R2 then the
and

R is the circumradius and G is the centroid, then


angle between the diagonals ACC and BD is
|OG |2 equals
(A) 120° (B) 90° (C) 75° (D) 60°
1 2
(A) R (a + b2 + c2 )
9 Solution:

Let
“O”

be the

centre
of the circle. Let

OA = a, OB = b, OC = c and OD = d so that
1 2
(B) R2 (a + b2 + c2 )
9 |a | = |b| = |c | = |d | = R
1 2
(C) R2 (a + b2 + c2 )
6 A

1 2
(D) R2 (a + b2 + c2 )
3
a


Solution:

Take “O” as the origin. Let OA = a , OB = b and



OC = g . Therefore
T (b , g ) = 2A,(g , a ) = 2B and (a , b ) = 2C .

A
O
D
d
b
B c
O
2A C C

FIGURE 6.30 Single correct choice type question 36.


FIGURE 6.31 37.
Worked-Out Problems 429

By hypothesis, æ 2 2ö

(C) Tan-1 ( 2 ) (D) Tan-1 ç ÷


| AB|2 + |CD|2 = 4 R2 è 3 ø


Þ | b - a |2 + | d - c |2 = 4 R2 Solution:

With respect to origin B, let BA = a and



BC = c so that c × a = 0.
Þ - 2(a × b) - 2(c × d ) = 0 (6.21)
Þ - 2 R [cos AOB + cos COD] = 0
2 A

Þ cos AOB + cos COD = 0


Þ AOB + COD = p (6.22)
E
Therefore a
90°
BOC + AOD = p

Now


AC × BD = (c - a )×(d - b) FIGURE 6.32 Single correct choice type question 39.

= (c × d ) - (b × c ) - (a × d ) + (aa × b) Now,

= - (b × c + a × d )

c
BD =
(because from Eq. (6.21) a × b + c × d = 0) 2
= - R2 [cos BOC + cos AOD] [by Eq. (6.22)]

c + a
and BE =
= - R2 (0) 2

=0 E and AD are perpendicular to each other. This implies



Therefore AC
C and BD are at right angles. æ c + a ö æ c ö
çè ÷ ç - a ÷ø = 0
×
2 ø è2
38. “O” is the circumcentre and H is the orthocentre of
Þ (c + a ) × (c - 2 a ) = 0
a

D ABC.

If
P

is any

arbitrary point, then the vector


PA + PB + PC - PH is equal to Þ | c |2 - 2 | a |2 - c × a = 0


(A) PO (B) 2PO (C) OP (D) 2OP
Þ | c |2 - 2 | a |2 = 0 (∵ c × a = 0)
Solution:

With
respect

to the

circumcentre ‘O’ as
| a |2 1
origin, let OA = a, OB = b and OC = c . Þ 2 =
Therefore by Example 5.3 (Chapter 5) |c | 2

( AB)2 1
OH = a + b + c Þ =

( BC )2 2
Let OP = p. Therefore
AB 1

Þ =
PA + PB + PC - PH = (a - p) + (b - p) + (c - p) BC 2
1
- (a + b + c - p) Þ tan C =
2
= -2 p

æ 1 ö
Þ C = Tan-1 ç
= 2 PO è 2 ÷ø
Answer: (B)
Answer: (A)
39. In DABC
C (Figure 6.32), angle B is a right angle.
40. In a DABC, if a2 + b2 = 2c2 and ma, mb and mc are the
If the medians AD and BE E are perpendicular to
lengths of the medians through the vertices A, B and
each other, then angle C is
C, respectively then amb + bma is equal to
æ 1 ö æ 1 ö
(A) Tan-1 ç (B) Tan-1 ç (A) c mc (B) 2c mc (C) 3c mc 3c mc
è 2 ÷ø
(D)
è 3 ÷ø
430 Chapter 6 Multiplication of Vectors

Solution: In Chapter 4, we have proved (see

1
AM = a
Theorem 4.25) that 2

1

ba + cb
ma = 2b2 + 2c2 - a2 AL = (∵ BL : LC = c : b)
2 b+c
1

a
mb = 2c2 + 2a2 - b2 CM = - b
2 2
1
mc = 2a2 + 2b2 - c2 By hypothesis,
2


AL ×CM = 0
Therefore

æ ba + cb ö æ a - 2b ö
a b Þç × =0
a mb + b ma = 2c2 + 2a2 - b2 + 2b2 + 2c2 - a2 è 2 ÷ø çè 2 ÷ø
2 2

a b Þ b |a |2 - 2c | b |2 + (c - 2b) (a × b ) = 0
= 3a2 + 3b2 (∵ a2 + b2 = 2c2 )
2 2 Þ bc2 - 2cb2 + bc(c - 2b)cos A = 0
3 2 Þ c - 2b + (c - 2b) cos A = 0
= (a + b2 )
2
Þ (c - 2b) (1 + cos A) = 0
3
= ( 2 c2 ) Þ c = 2b (6.23)
2
Again
= 3c2


|CM |:| AL| = m : 1
æ 2a + 2b - c ö
2 2 2


= 2c ç ÷ Þ |CM |2 = m2 | AL|2
è 2 ø
2
|a - 2 b |2 2 | ba + cb |
= 2c mc Þ =m
4 (b + c)2
Answer: (B)
Therefore
41. In DABC
C (Figure 6.33), the median CM
M is perpen

-
|a - 2 b |2 m2

to the angle bisector AL of angle A. If |CM |:


dicular = |a + 2 b |2 [By Eq. (6.23)]
| AL| = m : 1, then 4 9
2 2
9 æ 1 + cos A ö 9 æ 1 - cos A ö 9[|a | + 4 | b | - 4(a × b )] = 4 m2 [|a |2 + 4 b 2 + 4(a × b )]
(A) m2 = ç (B) m2 = ç
4 è 1 - cos A ÷ø 4 è 1 + cos A ÷ø
9[c2 + 4b2 - 4bc cos A] = 4 m2 [c2 + 4b2 + 4bc cos A]
4 æ 1 - cos A ö 4 æ 1 + cos A ö 9[4b2 + 4b2 - 8b2 cos A] = 4 m2 [4b2 + 4b2 + 8b2 cos A]
(C) m2 = ç (D) m2 = ç
9 è 1 + cos A ÷ø 9 è 1 - cos A ÷ø

[by Eq. (6.23))]


Solution: Taking “A” as origin, let AB = a , AC = b
9(1 - cos A) = 4 m2 (1 + cos A)
and so that |a | = c, | b | = b and (a , b ) = A. Therefore
9 æ 1 - cos A ö
m2 = ç
4 è 1 + cos A ÷ø
C

Answer: (B)
b

42. Let a and b be two non-zero
vectors of same magni-

L tude. If the vectors a + 3b and 5a + 3b are at right


90° angles to each other, then the angle between a and b is
c
æ 7ö æ 7ö
(A) Cos- 1 ç ÷ (B) p - Cos-1 ç ÷
è 9ø è 9ø
æ 1ö æ 1ö
FIGURE 6.33 Single correct choice type question 41. (C) Cos- 1 ç ÷ (D) p - Cos-1 ç ÷
è 3ø è 3ø
Worked-Out Problems 431


Solution: It is given that | a | = | b |. Let q = (a, b). Now Solution:

Take A
as
origin and let AB = b and AD = d
so that BC = nd, b ×d = 0. Now AC C and BD are at right
(a + 3b)×(5a + 3b) = 0 angles. This implies

5| a |2 + 9 | b |2 + 18(a ×b) = 0


AC ×BD = 0

5| a |2 + 9 | b |2 + 18 | a || b | cosq = 0
(b + nd ) ×(d b) = 0

5 + 9 + 18 cosq = 0 (∵ | a | = | b |)
b ×d | b |2 + n | d |2 n(b ×d ) = 0
7
cosq =
9 n | d |2 | b |2 = 0 (∵ b ×d = 0)
7
q =p Cos 1 ÷ n | d |2 = | b |2 (6.24)
9
Answer: (B) Now,


| AC |2 | b + nd |2
43. a, b, c are positive reals and are the lth, mth and nth

= 2
terms of a geometric progression, respectively. If | BD|2 |d b|

A = (log a)i + (log b) j + (log c)k | b |2 + n2 | d |2 + 2 n(b ×d )
=
| d |2 + | b |2 b ×d
and B = (m n)i + (n l ) j + (l m)k
| b |2 + n2 | d |2
then the angle between A and B is = 2 2 (∵ b ×d = 0)
|d | + |b|
(A) 60° (B) 45° (C) 30° (D) 90°
n | d |2 + n2 | d |2
Solution: Let a be the first term and x be the common = 2 2 [by Eq. (6.24)]
ratio of the GP. Then |d | + n |d |

a = a xl 1 (| d |2 + n | d |2 )
=n
| d |2 + n | d |2
b = a xm 1

=n
c = a xn 1

Therefore

AC : BD = n : 1
log a = (l 1)log x + log a
log b = (m 1)log x + log a D
log c = (n 1)log x + log a

d 90° C

A ×B = (m n)log a + (n l)log b + (l m)log c nd
= (log x)[(m n)(l 1) + (n l )(m 1) + (l m)(n 1)]
+ (log a )[(m n) + (n l ) + (l m)]
= (log x)(0) + (log a )(0) = 0 FIGURE 6.34 Single correct choice type question 44.

Answer: (D) Answer: (C)



44. ABCD is a trapezium (Figure 6.34) in which the 45. If | a | = 13, | b | = 5 and a ×b = 30, then | a b | is equal to
sides BC
C and AD are parallel and the adjacent sides 65
AB and AD are at right angles and AC, BD are at (A) 30 (B) 493
23
right angles. If BC : AD = n :1, then the ratio of the
diagonals AC : BD is equal to 30 65
(C) 493 (D) 133
(A) (n + 1) :1 (B) n :1 (C) n :1 (D) n2 :1 23 13
432 Chapter 6 Multiplication of Vectors


Solution: We have Solution: We have AB = b a, BC = c b, AC = c a.
The vector perpendicular to the plane of the triangle
30 = a ×b = | a || b |cos q ABCC is


= (65)cos q AB AC = (b a) (c a )

where q = (a, b). Therefore =b c b a+c a

30 6 =b c+c a+a b
cosq = =
65 13 Answer: (C)
36 133
sin q = 1 cos2 q = 1 =
169 13 |a | = 2 2
48. a and b are non-collinear vectors such that
and | b | = 3 and the
angle between (a, b) is p/ 4. If

So 5a + 2b and a 3b are adjacent sides of a parallelo-
gram, then its area in square units is
65× 133
|a b | = | a || b |sin q = (A) 72 (B) 92 (C) 102 (D) 112
13
Solution:
The area of a the parallelogram with a and
Answer: (D) b as adjacent sides is (see Corollary 6.13)

46. a, |a b |
2
|a i |2 + | a j | + |a k |2 = Therefore area of the parallelogram
whose adjacent

sides are 5a + 2b and a 3b is
1 2
| a |2 (B) 2 | a |2 (C) |a | (D) 3| a |2
2 |(5a + 2b) (a 3b)| = | 17(a b)|

Solution: Let a = a1i + a2 j + a3k . Then
= 17 | a b |

a i = a2 k + a3 j p
= 17 | a || b |sin ÷
4
a j = a1k a3 i

= 17(2 2 )(3)
1
a k = a1 j + a2 i ÷
2
Therefore = 102 sq. units
2 Answer: (C)
|a i |2 + | a j | + |a k |2

= (a22 + a32 ) + (a12 + a32 ) + (a12 + a22 ) r is perpendicular
49. The vector to the vectors
a = 3i + 2 j + 2k and b = 18 i 22 j 5k and makes

= 2((a12 + a22 + a32 ) an obtuse angle with i . If | r | = 14, then r equals

(A) 4 i 6 j + 12k (B) 4 i + 6 j + 6k
= 2 | a |2

Answer: (B) (C) 4 i + 6 j 6k (D) 6 i 4 j 12k

Solution: r is parallel to a b. This implies
47. a, b, c are the position vectors of the vertices A,

B and C of ABC, respectively. Then the vector r = l (a b )
perpendicular to the plane of ABC
C is
i j k
(A) b c
=l 3 2 2
(B) a + b + c
18 22 5
(C) b c + c a + a b
= (l )17(2 i + 3 j 6k )
(D) (a b) (a + b + c )
Worked-Out Problems 433


Now r makes obtuse angle with i . This means l < 0. Now
Hence
i j k
| r | = 14 142 = 172 (l 2 )(49)
a b= 3 1 2
4 1 3 4
l2 =
172
= 2 i 14k 10k
l=
2
(∵ l < 0)
17 = 2(i + 7k + 5k )
Therefore
r = 2(2 i + 3 j 6k)
Answer: (A) 1
Area = |a b|
2
50. (1, y1) and B(x2, 11) are two points on a plane
curve y = x2 2x + 3
with = 12 + 72 + 52

x2 >
0.

Then the area of the


parallelogram with OA and OB (“O” is the origin)
as adjacent sides is = 75 = 5 3
(A) 3 (B) 9 (C) 6 (D) 4 Answer: (B)

Solution: We have
52. Let a = 3i + 2 j + 2k , b = i + 2 j 2k . Then a unit

y1 = 1 2 + 3 = 2 vector perpendicular to both a b and a + b is
1 1
and (A) ( 2i + 2 j + k ) (B) ( 2 i + 2 j k )
3 3
11 = x22 2 x2 + 3 1 1
(C) (2 i 2 j + k ) (D) (i + j + k )
3 3
x22 2 x2 8=0
Solution: We have
( x2 4)( x2 + 2) = 0
a + b = 4i + 4 j
x2 = 4, 2
a b = 2 i + 4k
Therefore B = (4, 11) and A = (1, 2). Hence

Then
OA OB = (i + 2 j ) (4 i + 11 j )
i j k

= 11k 8k = 3k (a + b ) (a b) = 4 4 0
So 2 0 4


Area = |OA OB| = 3 = 16 i 16 j 8k

Answer: (A) = 8(2 i 2 j k )

51. The area of the parallelogram whose
diagonals
Unit vector perpendicular to both a + b and a b is
are by the vectors a = 3i + j
represented
2k and

b = i 3 j + 4k is 1
± (2 i 2j k)
3
(A) 4 3 (B) 5 3 (C) 6 3 (D) 3 3
One of them is
Solution: The area of the parallelogram ABCD in
terms of its diagonal vectors is (see Theorem 6.30) 1
( 2i + 2 j + k )
3
1


| AC BD| Answer: (A)
2
434 Chapter 6 Multiplication of Vectors


53. The perpendicular distance of a vertex of a unit cube Solution:
Let
A, B and C be represented by i + j + k,
from a diagonal not passing through it is i - j + k and k, respectively. Then


2 2 1 2 AB = - 2 j
(A) (B) (C) (D)
3 3 3 3

AC = - i - k
Solution: Let OA = (1, 0, 0), OB = (0, 1, 0) and OC =
(0, 0, 1) be three coterminus edges Now, the area of the DABC
C (see Theorem 6.29) is

of the
unit cube (see
Figure 6.35). Clearly the vector OR = i + j + k . OR is a 1

1
diagonal not passing through A. | AB ´ AC | = | 2 j ´ (i + k )|
2 2

B P = | -k + i | = 2
Answer: (B)
Q R
M
j - k , b = - i + 2 j - 4k and c = i + j + k .
55. Let a = 2 i +

90° Then (a ´ b)×(a ´ c ) is equal to
(A) 26 (B) 24 (C) -24 (D) -26
O A Solution: We have (see Theorem 6.48)

a ×a a ×c
(a ´ b ) × (a ´ c ) =
b×a b×c
FIGURE 6.35 Single correct choice type question 53.
6 2
It is known that the area of the triangle whose vertices
=
have the position vectors a, b and c is (see Corollary 6.14) 4 -3

1 = - 26
|b ´ c + c ´ a + a ´ b|
2 Answer: (D)

Therefore the area of DOAR is


a , b and g be non-coplanar
56. Let

vectors. If xa +

1 1 1 yb + zg is equal to a(a + g ) + b(-a + g ) + c(a + b + g ),
| i ´ (i + j + k ) + (i + j + k ) ´ 0 + 0 ´ i | = | k - j | = then a is equal to
2 2 2
x z x z
But, (A) - + y - (B) - +
2 2 2 2
1

x
- y+
z
(D) x - +
y z
Area of DOAR = |OR |× AM (C)
2 2 2 2 2

where AMM is the perpendicular distance of the vertex A Solution: We have


from the side OR given by
xa + yb + zg = (a - b + c)a + cb + (a + b + c)g
1
3 ×( AM ) Therefore
2
a-b+c= x
Therefore
y=c
1 3
= AM a+b+c=z
2 2
Now
2 2
AM = = a - b = -y + x
3 3
Answer: (B) and a+b=z- y
Hence
54. The
area of the triangle whose vertices are i + j + k ,
i - j + k and k is x - 2y + z x z
1 a= = -y +
(A) 1 (B) 2 (C) 2 (D) 2 2 2
2 Answer: (C)
Worked-Out Problems 435


57. Let OA = a, OB = 10a + 2b and OC = b where O, Therefore
A and C are non-collinear points. Let p denote the
area of the quadrilateral OABC a b=b c=c a 0

C

and q

denote
the
area of the parallelogram with OA and OC as adja- Answer: (B)
cent sides. If p = lq, then l equals
(A) 6 (B) 8 (C) 10 (D) 9 60. A non-zero vector a is parallel to the line of intersec-

tion of the planes determined by the vectors i , i + j
Solution: q = area of the parallelogram = | a b |. and the plane determined by the vectors i j , i + k .

1

The angle between a and the vector i 2 j + 2k is


p = Area of the Quadrilateral OABC = |OB AC | p p p p
2 (A) (B) (C) (D)
4 3 6 2
1
= |(10a + 2b) (b a )|
2 n1 = i (i + j ) = k is
Solution: normal to the first plane
n2 = (i j ) (i + k ) = j + k i is normal to second
1
= | 10(a + b) 2(b a )| plane. Since a is parallel to the line of intersection of the
2
planes, it is perpendicular to both n1 and n2 and hence a
12 is parallel to the vector
= | a b | = 6q
2
Answer: (A) n1 n2 = k ( j + k i ) = i j

58. Let a = i k , b = xi + j + (1 x)k and c = yi + xj + The angle between a and i 2 j + 2k is same
as the

(1 + x y)k . Then [a b c ] depends on angle between n1 n2 = i j and i 2 j + 2k . If q is the
angle between these two vectors, then
(A) only x (B) only y

(C) neither x nor y (D) both x and y (i j ) ×(i 2 j + 2k ) 1 + 2 1
cos q = = =
Solution: Using Theorem 6.35 we know that | i j || i 2 j + 2k | 2 (3) 2

1 0 1 Therefore

[a b c ] = x 1 1 x p
y x 1+ x y q=
4
= (1 + x y) x(1 x) ( x2 y) Answer: (A)
=1+ x y x+x 2
x +y
2
61. The perpendicular distance of the point
i + j + k
=1 the line joining the points 3i + 4 j k and
from
Answer: (C) 2 i + j k is
3 10 7
59. Let a, b, c be unit vectors such that a + b + c = 0. (A) (B) (C) (D) 2 10
10 2 10
Which one of the following is correct?

(A) a b = b c = c a = 0 Let A and B be the points 3 i + 4 j k and
Solution:
2 i + j k , respectively, and P the point i + j + k . Draw
(B) a b = b c = c a 0 AM M perpendicular to the line AB (Figure 6.36). Then


(C) a b = b c = a c 0
PA = 2 i + 3 j 2k
(D) a b, b c , c a are mutually perpendicular


PB = i 2k
Solution: We have a + b = c . Therefore
P (1,1,1)
(a + b ) c = 0

b c= a c=c a

Again

(b + c ) a = 0

c a= b a=a b FIGURE 6.36 Single correct choice type question 61.
436 Chapter 6 Multiplication of Vectors


Now r a = tb, t 

i j k r = a + tb, t 


PA PB = 2 3 2 = 6i + 2 j 3k Answer: (A)
1 0 2
64. Let p be the area of a quadrilateral ABCD and q the


and AB = i 3j area of a parallelogram inscribed in the quadrilat-
eral with its sides drawn parallel to the diagonals of
Now (see Theorem 6.29) the quadrilateral. Then
1 1

(A) p 2q (B) p > 3q


( AB) ( PM ) = Area of PAB = | PA PB|
2 2 (C) p = 3q (D) p < 2q


1 Solution: Take A as origin and let AB = b, AC = c and
= ( 6)2 + 22 + ( 3)2


2 AD = d. Let PQRS be the parallelogram inscribed in the
7 quadrilateral with P, Q, R and S on the segments AB,
= BC, CD and DA, respectively (Figure 6.37). Let
2
Hence AP : PB = CQ :QB = l : 1

7 7 7 Therefore
PM = = =
AB ( 1) + ( 3)
2 2
10 AS : SD = CR : RD = l : 1
Answer: (C) Hence

62. If a, b, c are the position vectors of the vertices A, B

lb
AP =
and C of a triangle, then, the
angle

between the l+1



vectors b c + c a + a b and BC is

lb + c
p p AQ =
(A) (B) l+1
2 3

ld
1 1 AS =
(C) Cos 1 ÷ (D) Cos 1 ÷ l+1
3 3

c
Solution: It is known that AB AC is perpendicular PQ =
l+1
to the plane of ABC. But

l l


AB AC = (b a ) (c a ) PS = (d b) = ( BD)
l+1 l+1

=b c b a a c Now q = Area of the parallelogram PQRS is given by

=b c+c a+a b


| PQ PS | = | AC BD|
Answer: (A) (l + 1)2

2l
63. Let a be the position vector of a point and b be a = (Area of ABCD)
non-zero vector. Then, the locus of the point P whose (l + 1)2

position vector r satisfies the relation r b = a b is 2l
= ( p)
(A) a straight line (l + 1)2

(B) a line passing through a and perpendicular to

the line r = a + tb, t  D
1
(C) a line making an angle of 60° with the line
1 R
r = a + tb l
S C
(D) is a circle with centre at the point a.
l
Solution: We have l Q
1
r b=a b

(r a ) b = 0 FIGURE 6.37 Single correct choice type question 64.
Worked-Out Problems 437

Therefore l + t = 1 and l = s

(l + 1)2 éæ l 1ö ù Therefore the position


p of M, L and K are, respec-
vectors
p= (q) = êç + ÷ø + 1ú (q) ³ 2q tively, b + l d, lb and l(b + d ). Hence
2l è
ë 2 2l û


Equality holds when l = 1, that is, P, Q, R and S are the | LM |2 = | b + l d - l b |2

mid-points of the sides. = |(1 - l )b + l d |2
Answer: (A)
= (1 - l )2 | b |2 + l 2 | d |2 + 2 l (1 - l ) (b × d )
65. In the rhombus ABCD, A = 60°. K is any point on æ a2 ö
the segment AC. L and M are points on the segments = a2 [(1 - l )2 + l 2 + l (1 - l )] ç∵ b × d = a2 cos 60° = ÷
è 2ø
AB and BC, respectively, such that KLBMM is a paral-
lelogram. Then the angles of DLMD are = a2 (l 2 - l + 1)
(A) 90°, 45°, 45° (B) 60°, 60°, 60°
l2 - l + 1 > 0. Now
(C) 90°, 60°, 30° (D) 75°, 30°, 75°

| MD|2 = | b + ld - d |2
Solution:

Take A as origin. Let AB = b and AD = d so that


AC = b + d, | b | = | d | = a (suppose) and (b, d ) = 60°. Let = | b + (l - 1)d |2 = | b |2 + (l - 1)2 | d |2 + 2(l - 1) (b × d )


AK = l AC = l(b + d ) = a2 [1 + (l - 1)2 + l - 1]

= a2 (l 2 - l + 1)
and 0 < l < 1. LK is parallel to BC C implies that


| LD|2 = | lb - d |2
AL AK
= = l (suppose) = l 2 | b |2 - 2 l (b × d ) + | d |2
AB AC
= a2 (l 2 - l + 1)
Therefore

Therefore
AL = lb
LM = DL = DM
Equation of the line KMM is
and so DLMD is equilateral.
r = l(b + d ) + tb (6.25)
Answer: (B)
where t Î (see Theorem 5.27, Chapter 5). Equation of
the line BC
C is 66. P is a point on the circumcircle of DABC
C other than
the vertices A, B and C. H is the orthocentre, M is
r = b + sd (6.26) the mid-point of PHH and D is the mid-point

BC.
of
Since M is the point of intersection of the lines KM
M and Then the angle between the vectors DM and AP is
BC, on equating the coefficients of Eqs. (6.25) and (6.26) (A) 75° (B) 60° (C) 120° (D) 90°
we have Solution: Take the

circumcentre

“O” of

the
triangle

A
as origin and let OA = a, OB = b and OC = c . See
Figure 6.39.
30°
30° A

L
P
K D H
M C

D
M

FIGURE 6.39 Single correct choice type question 66.


FIGURE 6.38 65.
438 Chapter 6 Multiplication of Vectors


Therefore by Example 5.3 (Chapter 5) (l d + b)

r =t (6.27)
OH = a + b + c l+1


Let OP = p so that Equation of the line BC
C is (see Theorem 5.27)

p + a + b + c r = b + sd, s Î  (6.28)
OM =
2
Equation of the line DCC is (see Theorem 5.27)
and
r = d + ub, u Î  (6.29)

p + a + b + c b + c
DM = - From Eqs. (6.27) and (6.28), we have
2 2
t tl
p+a = 1 and =s
= l+1 l+1
2
s = l, t = l + 1. Therefore
Now

æ p + a ö AQ = b + l d (6.30)
DM × AP = ç ×( p - a)
è 2 ÷ø Also from Eqs. (6.27) and (6.29), we have
1 tl t
= (| p|2 - | a | 2) = 1 and =u
2 l+1 l+1
1
= (0) (∵ | p| = | a | = R) Therefore
2
=0 1
lu = 1 or u =

l
DM and AP are at right angles.
Hence
Answer: (D)

1
AR = d + b (6.31)
67. ABCD is a parallelogram. Through the vertex A, an l
arbitrary line is drawn to meet the diagonal BD in
P, the side BCC in Q and the side DC
C produced in R. Therefore
Then AP P is


PQ = AQ - AP
(A) AM between PR and PQ
æ b + ld ö
(B) GM between PR and PQ = (b + ld ) - ç ÷
(C) HM between PR and PQ è l+1 ø
(D) AR = PQ × PR l

= (b + ld )
Solution: Take A as origin and let AB = b and AD = d. l+1


Suppose BP : PD = l :1 so that = l ( AP )

b + ld
AP = This gives
l+1
PQ
Therefore the equation of the line AP
P is (see =l (6.32)
AP
Theorem 5.29)
Also
D C R


PR = AR - AP
1 Q
æ 1 ö b + ld
= ç d + b÷ - [see Eq. (6.31))]
P è l ø l+1
l
æ1 1 ö
= (b + ld ) ç -
è l l + 1÷ø
FIGURE 6.40 Single correct choice type question 67.
Worked-Out Problems 439

b + ld 6.29)
=
l (l + 1) 1


Area of DEF = | DE DF |
1

2
= ( AP )
l 1 b + (1 l )c (l 1)b lcc
=
Hence 2 l+1 l+1

AP 1
=l (6.33) = | l (b c ) + (l 1)2 (b c )|
PR 2(l + 1)2

Equations (6.32) and (6.33) give l2 l + 1


= |b c|
2(l + 1)2
PQ AP
=
AP PR l2 l + 1
= (Area of ABC )
( AP )2 = PQ ×PR (l + 1)2
Answer: (B) 1
∵ |b c | = Area ABC ÷
2
68. On the sides BC, CA and AB of ABC, points D, E
and F are taken, respectively, such that BD : DC = Answer: (A)
CE : EA = AF : FB = l :1. Then the ratio of the area
of DEF F to the area of ABC C is 69. 2 and coplanar with the
vectors 3i j k and i +
j 2 k which is perpendic-
(A) (l 2 l + 1):(l + 1)2 ular to the vector 2 i + 2 j + k is one of the following
(B) (l 2 l + 1):(l + 1) vectors. Which one is it?
(C) (l 2 + l + 1):(l + 1)2 2 1
(D) (l 2 + l + 1):(l + 1) (A) (i + j + k ) (B) (3i 5 j + 4k )
3 5


Solution: Take A as origin and let AB = b and AC = c . 1 2
(C) (3i + 5 y + 4k ) (D) (i j + k )
Therefore 5 6

Solution: Let r be the required vector. Since r is
A
coplanar with the
vectors b = 3i j k and c = i + j 2k
1
l (observe that b and c are non-collinear), let
E
F r = xb + yc
l
1 = (3 x + y)i + ( y x) j ( x + 2 y)k (6.34)
B l
D 1 |r | = 2 (3 x + y)2 + ( y
x)2 + ( x + 2 y)2 = 2 (6.35)
C

FIGURE 6.41 Single correct choice type question 68. where
x and
y are scalars. Now r is perpendicular to
2 i + 2 j + k implies

b + l c 2(3x + y) + 2( y x) x 2y = 0
AD =
l+1 3x + 2 y = 0


c 3x
AE = y=
l+1 2

lb
AF = Putting this values of y = 3x/2 in Eq. (6.35) we get
l+1 2 2
3x 3x

b + (1 l )c 3x ÷ + x÷ + ( x 3 x)2 = 2
DE = AE AD = 2 2
l+1
9 x2 25 x2

(l 1)b lc + + 4 x2 = 2
DF = 4 4
l+1
50 x2 = 8
440 Chapter 6 Multiplication of Vectors

2 2 2 (see Theorem 6.33 and Theorem 6.35) where A is the matrix


x=± =±
5 2 5 é 1 a - 1ù
ê0 1 a ú
Now ê ú
êë 1 1 1 úû
2 3x 3
x= Þy=- =-
5 2 5
-2 3
x= Þy= det A = 1 - a + a2 + 1
5 5
= a2 - a + 2
In tabular form we have
2
3x æ 1ö 7 7
x y=- = ça - ÷ + ³
2 è 2ø 4 4
2 3 Therefore
-
5 5
2
2 3 æ 1ö 7 7
- V = det A = ç a - ÷ + ³
5 5 è 2ø 4 4
Answer: (B)
V is equal to 7/4 when a = 1/2. Hence V is minimum when
a = 1/2 and the minimum value of V is 7/4.
70. The position vectors
A,
of the points B, C and
D are,
Answer: (B)
respectively,
3i - 2 j - k , 2 i + 3 j - 4k , - i + j + 2k and
4 i + 5 j + lk . If all the four points lie in a plane, then
72. If the volume of the parallelopiped whose set of
the value of l is
coterminus
edges
are represented
by the vectors
146 146 98 98 - 12 i + lk , 3 j - k and 2 i + j - 15k is 546 cubic units,
(A) - (B) (C) (D) -
17 7 17 17 then a value of l is
Solution: We have (A) -3 (B) 3 (C) -179 (D) 178

Solution: We have
AB = - i + 5 j - 3k

V = Volume
AC = - 4 i + 3 j + 3k

- 12 0 l
AD = i + 7 j + (l + 1)k = Absolute value of 0 3 -1
Now 2 1 - 15

A, B, C and D are coplanar = | 12 ´ 44 - 6 l |



Û AB, AC and AD are coplanar Therefore

-1 5 -3 88 - l = ± 91
Û -4 3 3 = 0 (see Corollary 6.16) Now
1 7 (l + 1)
88 - l = 91 Þ l = - 3
Û 17 l + 146 = 0
and 88 - l = - 91 Þ 179
Answer: (A)
Answer: (A)
71. The value of a so that the volume
of the parallelo-

piped formed by the vectors i + aj - k , j + ak, i + j + k to a = 3i + 2 j + 6 k and
73. Unit vector orthogonal

becomes minimum is coplanar with b = 2 i + j + k and c = i - j + k is

(A) 3 (B)
1
(C)
1
(D)
7 6 i - 5k 3j - k
(A) (B)
2 3 4 61 10

Solution: We have 2i - 5 j 2 i + j - 2k
(C) (D)
V = Volume of the parallelopiped = |det A| 29 3
Worked-Out Problems 441

Solution:
By the definition of cross product, the vector Hence

a ´ (b ´ c ) is coplanar with b and c and is orthogonal to
|a ´ b| = 3
a. Now (using Theorem 6.46) we have

a ´ (b ´ c ) = (a × c )b - (a × b)c So

= 7b - 14c = 7(3 j - k ) |(a ´ b) ´ c | = | a ´ b || c | sin 30°
æ 1ö 3
Therefore the required unit vector is = 3(1) ç ÷ =
è 2ø 2

7(3 j - k ) 3 j - k Answer: (B)
=
7 9+1 10

76. The vectors
a = (sec2 a )i + j + k , b = i + (sec2 b ) j + k
Answer: (B)
and c = i + j + (sec2 g )k are coplanar only if

74. If a = i + j + k and b is
a vector such that a × b = 1 (A) a, b, g are distinct such that a + b + g = p

and a ´ b = j - k , then b is (B) a = b = g = np,
p n Î

(A) i - j + k (B) 2 j - k (C) i (D) 2 i p
(C) a = b = g =
4
Solution: We have
(D) no real values exist

(a ´ b ) ´ a = ( j - k ) ´ ( i + j + k ) Solution: The vector are coplanar only if
Therefore (using Theorem 6.46)
sec2 a 1 1

i j k 1 sec2 b 1 =0

sec2 g
(a × a)b - (b × a)a = 0 1 - 1 1 1
1 1 1
Replacing sec2 aa, sec2 b and sec2 g with 1 + tan2 a
a, 1 +

3b - a = 2 i - j - k (∵ a × b = 1)) tan b and 1 + tan2 g and simplifying we have
2


3b = (i + j + k ) + (2 i - j - k ) å (tan 2
b tan2 g ) + tan2 a tan2 b tan2 g = 0
1
b = ( 3i ) = i which is possible only when
3
Answer: (C) tan2 a = tan2 b = tan2 g = 0
Answer: (B)
75. Let a = 2 i + j - 2k and b = i + j . If c is a vector such

that a × c = | c |, | c - a | = 2 2 and the angle between
a, b be unit vectors. If c is a vector such that
77. Let
a ´ b and c is 30°, then |(a ´ b) ´ c | is equal to b = c + c ´ a and if
2 3
(A) (B) (C) 2 (D) 3 1
3 2 |(a ´ b) × c | =
2
Solution: We have
then the angle between a and b is
8 = | c - a |2 (A) 60° (B) 30° (C) 45° (D) 90°

= | c |2 - 2(c × a ) + | a |2 Solution: Let q be the angle between a and c. Now

= | c |2 - 2 | c | + 9
|(a ´ b) × c | = | b ×(c ´ a )| = |(c + c ´ a ) ×(c ´ a )|
Therefore
= | c ´ a |2 = | c |2 sin2 q

(| c | - 1)2 = 0 or | c | = 1
Also
Now
| b |2 = | c + c ´ a |2 = | c |2 + | c ´ a |2 = | c |2 + | c |2 sin2 q

i j k
Therefore
a ´ b = 2 1 - 2 = 2i - 2 j + k
1 1 0 1 = | b |2 = (1 + sin2 q )| c |2 (∵ | b | = 1)
442 Chapter 6 Multiplication of Vectors


So For the plane CDE, CD CE is a normal. So

1 i j k
| c |2 = (6.37)


1 + sin2 q CD CE = 2 2 2
Therefore from Eqs. (6.36) and (6.37) we get 4 5 2

sin2 q 1 = 6 i 4 j + 2k
|(a b) ×c | = =
1 + sin q 2
2

Therefore 3i 2 j + k is also normal to the plane CDE.
if and only if q = 90°. Now The plane passes through the point C. By Theorem 6.17,
the equation of the plane is
b ×a = c ×a + (c a )×a

[r ( 4 j + 4k )]×(3i 2 j + k ) = 0
= 0 + 0 (∵ c ×a = 0)

where r = xi + yj + zk is any point in the plane CDE.
Therefore a and b are perpendicular to each other. That is
Answer: (D)
[ xi + ( y + 4) j + (z 4)k ]×(3i 2 j + k ) = 0

78. a, b and c are vectors of magnitudes
1, 1 and 2,
3x 2( y + 4) + z 4=0
respectively. If a (a c ) + b = 0, then acute angle

between a and c is 3x 2 y + z 12 = 0
(A) 90° (B) 60° (C) 45° (D) 30° From Eqs. (6.38) and (6.39), we have
Solution: We have (using Theorem 6.46) 3(2 t) 2(1 + t ) + 2 t 12 = 0

b = a (a c ) = (a ×c )a (a ×a )c 6 3t 2 2t + 2 t 12 = 0
Therefore 6t 6=0
t= 1
1 = | b |2 = (a ×c )2 | a |2 + | c |2 2(a ×c )2 (∵ | a | = 1 = | b |)
t = 1 in Eq. (6.38), we have
= (a ×c))2 + 4 2(a ×c )2

r = 3i + 3k
= 4 (a ×c )2
Therefore (3, 0, 3) is the point where the line AB meets
= 4 4 cos2 q the plane CDE.
Answer: (B)
q = (a, c ). Therefore

3 80. Let a = i j , b = j k , c = k i . d is a unit vector
cosq = ±
2 such that a ×d = 0 = [b c d ], then d equals

Hence q is acute q = 30°. (i + j 2k ) (i j 2k )
(A) ± (B) ±
Answer: (D) 6 6

(2 i + j k ) (i + 2 j k )
79. Let A = (1, 2, 1), B = (2, 1, 2), C = (0, 4, 4), D = (2, (C) ± (D) ±
6 6
2, 2) and E = (4, 1, 2) be five points. The line AB

meets the plane CDE E at the points Solution: Let d = xi + yj + zk and | d | = 1.
(A) (3, 1, 3) (B) (3, 0, 3)
a ×d = 0 x y = 0 (6.40)
(C) (1, 1, 1) (D) (2, 0, 2)
Solution: Equation of the line AB is (using Theorem
T 5.29) 0 1 1

[b c d ] = 0 1 0 1 =0
r = (1 t )(2 i + j + 2k ) + t(i + 2 j + k ), t 
x y z

r = (2 t )i + (1 + t ) j + (2 t )k (6.38) x+ y+z=0 (6.41)

|d | = 1 x2 + y2 + z2 = 1 (6.42)
Worked-Out Problems 443

From Eqs. (6.40) and (6.41) we get A

y=x and z = 2x
2i
+3
Substituting these values in Eq. (6.42), we have j

1 D
6 x2 = 1 or x=±

j
6

3i
Therefore
1 E
d=± (i + j 2k )
6
B C
Answer: (A)
FIGURE 6.42 Single correct choice type question 82.

81. Let a , b and c be non-coplanar vectors and

2a + 3b c , a 2b + 3c , 3a + 4b 2c and a 6b + 6c Vector area of the shaded region is given by
be position vectors of the points A, B, C

and D, respec-


tively. Then the scalar triple product [ AB AC AD] is ED EB + EC ED = [ED EB + ED CE]
equal to 2 2 2
1


(A) 9[a b c ] (B) 6[a b c ] = [ED (CE + EB)]
2
(C) 4[a b c ] (D) 0
1


Solution: We have = (ED CB)
2


AB = a 5b + 4c 1

= [( 4 i + 2 j ) (i 4 j )]
2
AC = a + b c

1
= (16k 2k )
AD = a 9b + 7c 2

Therefore (using Quick Look 11) = 7k
1 5 4 Therefore


[ AB AC AD] = 1 1 1 [a b c ]
Area = |Vector area| = 7
1 9 7
Answer: (C)
= (2 + 30 32)[a b c ]

=0 83. The angle between two unit vectors a and b is

Answer: (D) Cos 1 (1/ 4). If c is a vector such that | c | = 4 and

c 2b = l a, then value of l is
QUICK LOOK (A) 3, 4 (B) 3, 4 (C) 3, 4 (D) 3, 4
The points A, B, C and D are coplanar. Solution: We have
1 1

a ×b = | a || b | cos cos 1 ÷ = (6.43)


4 4
6.42, AB = 3i j , AC = 2 i + 3 j and DE =
82. In Figure

4 i 2 j . Then the area of the shaded region in square
c = l a + 2b
units is

(A) 5 (B) 6 (C) 7 (D) 8 16 = | c |2

Solution: We have = | l a + 2b |2


BC = BA + AC = l 2 | a |2 + 4 l (a ×b) + 4 | b |2

= ( 3i + j ) + ( 2 i + 3 j ) 1
= l 2 + 4l ÷ + 4 [by Eq. (6.43))]
= i + 4j 4
444 Chapter 6 Multiplication of Vectors

Therefore 85. In ABC, AB = BC C and the median AD is perpen-


dicular to the bisector of the angle C (Figure 6.44).
l2 + l 12 = 0 Then ACB is
(l + 4)(l 3) = 0 (A) 60° (B) 45°
1 1
l = 3 or 4 (C) Cos 1 ÷ (D) Cos 1 ÷
3 4
Answer: (A)


Solution: Take A as origin. Let AB = a and AC = b so
84. In ABC (Figure 6.43), if(BC)2 + (AC)2 = 5(AB)2,
that |a | = c, | b | = b and (a , b ) = A. Let CE
E be the bisector
then the angle between the medians AD and BE E is of ACB so that AE : EB = b : a. Therefore
(A) 60° (B) 90° (C) 120° (D) 45°

a + b
Solution: Take A as origin. Let AB = a and AC = b AD =
2
so that |a | = c, | b | = b and (a , b ) = A. Therefore

ba

a + b and AE =
a+b
AD =
2

1
and AE = b AB = BC
2


| AB| = | BC |
A

|a |2 = |a b |2

| b |2 2(a ×b ) = 0
E
| b |2 2 |a || b | cos A = 0
b = 2c cos A (6.46)
Q
C Now


a+b ba
D AD×CE = 0 ÷× b÷ = 0
2 a+b

(a + b ) ×(ba (a + b)b ) = 0
FIGURE 6.43 Single correct choice type question 84.
b|a |2 (a + b)(a ×b ) + b(a ×b ) (a + b)| b |2 = 0

Now b|a |2 a(a ×b ) (a + b)| b |2 = 0


a + b [(b / 2) a ] bc2 abc cos A (a + b)b2 = 0
AD×BE = ×
2 ÷ 2
B
1
= (| b |2 2 |a |2 a ×b ) (6.44)
8
a
But by hypothesis

D
|b a |2 + | b |2 = 5 |a |2
E
90°
2 | b |2 2(a ×b ) = 4 |a |2
b
C
| b |2 (a ×b ) 2 |a |2 = 0 (6.45)


From Eqs. (6.44) and (6.45), it follows that AD×BE = 0.
Therefore the medians AD and BE E are at right angles.
FIGURE 6.44 Single correct choice type question 85.
Answer: (B)
Worked-Out Problems 445


ba2 ba2 cos A (a + b)b2 = 0 (∵ AB = BC c = a) Solution: Since a, b and c are mutually perpendicular
vectors, the parallelopiped formed by them is a rectan-
a2 a2 cos A ab b2 = 0
gular box. Therefore
a2 a2 cos A a(2a cos A) 4a2 cos2 A = 0
|[a b c ]| = | a || b || c | = 4
[from Eq. (6.46) and c = a]
Answer: (A)
1 3 cos A 4 cos2 A = 0

88. If a, b, c are non-coplanar unit vectors such that
4 cos2 A + 3 cos A 1 = 0
(4 cos A 1)(cos A + 1) = 0 1
a (b c) = (b + c )
1 2
cos A =
4 and b, c are non-collinear vectors, then the angle

But between a and b is
p 3p p 2p
A = ACB (∵ AB = BC ) (A) (B) (C) (D)
4 4 3 3
Therefore Solution: By Theorem 6.46
1 1
cosC = or ACB = Cos 1 ÷ a (b c ) = (a ×c )b (a ×b)c
4 4
Therefore
Answer: (D)
1
86. Let a, b, c be three non-coplanar vectors and r be a (a ×c )b (a ×b)c = (b + c )
2
non-zero vector such that
5.15)
r = x(b c ) + y(c a ) + z(a b)
1 1
a ×c = and a ×b =
where x, y, z are scalars and [a b c ]( x + y + z) = 1. 2 2

Then r ×(a + b + c ) is equal to
Therefore
1
(A) 3 (B) 2 (C) 1 (D)
3 1
= a ×b = | a || b |cosq
Solution: We have 2

r = x(b c ) + y(c a ) + z(a b) where q = (a, b). Hence
1 3p
Therefore = cosq q=
2 4

r ×a = x[a b c ] Answer: (B)

r ×b = y[a b c ]
89. a (b c ) = (a b) c and a, c are not collinear,

r ×c = z[a b c ] then

(A) (a c ) b = 0 (B) (a c )×b = 0
So

(C) a b = a c (D) b c = c a = a b
r ×a + r ×b + r ×c = ( x + y + z)[a b c ] = 1
Solution: We have
This implies
a (b c ) = (a b) c

r ×(a + b + c ) = 1
This implies
Answer: (C)

(a ×c )b (a ×b)c = (a ×c )b (b ×c )a (see Theorem 6.46)
87. If a, b and c are mutually perpendicular vectors of
magnitudes 1,, 2 and 2,, respectively, then the value which further gives

of |[a b c ]| is
(a ×b)c = (b ×c )a
(A) 4 (B) 5 (C) 5 (D) 2
446 Chapter 6 Multiplication of Vectors


But a and c are non-collinear vectors. Therefore 1 11
= (3)(4)
2 6
a ×b = 0 = b ×c
= 11
Hence b is perpendicular to both a and c and so b is
Answer: (D)
parallel to a c. Therefore

(a c ) b = 0 91. If the vectors a, b and c are coplanar vectors, then
the value of the determinant
Answer: (A)


a ×a a ×b a ×c
90. In ABC (Figure 6.45), if | AC | = 3 and | BC | = 4 and
the medians AD and BEE are at right angles, then the b ×a b ×b b ×c =
area of ABC C is
c ×a c ×b c ×c
(A) 12 (B) 7 (C) 7 (D) 11
(A) 0 (B) 2 (C)
(D) 1 2
A
Solution: In Quick Look 11, part (2), take l = a, m = b

and n = c . Then
3
a ×a a ×b a ×c

[a b c ]2 = b ×a b ×c b ×a
E
90° c ×a c ×b c ×c


But a, b, c are coplanar. This implies

[a b c ] = 0

Direct Solution:
FIGURE 6.45 Single correct choice type question 90.
Let


Solution: Take and let CA = a and CB = b
C as origin a = a1i + a2 j + a3k
so that | a | = 3, | b | = 4 and (a, b) = C . Now

b = b1i + b2 j + b3k
AD×BE = 0
c = c1i + c2 j + c3k
b a
a ÷× b÷ = 0 Therefore
2 2
a1 a2 a3 a1 a2 a3
(b 2a )×(a 2b) = 0
[a b c ]2 = b1 b2 b3 b1 b2 b3
2 | a |2 2 | b |2 + 5(a ×b) = 0
c1 c2 c3 c1 c2 c3
2(9) 2(16) + 5(3) (4)cos C = 0
a1 a2 a3 a1 b1 c1
60 cos C = 50 = b1 b2 b3 a2 b2 c2 [∵ det A = det(AT )]
5 c1 c2 c3 a3 b3 c3
cos C =
6
a12 a1b1 a1c1
Therefore
= b1a1 b 2
1 b1c1
25 11 c1a1 c1b1 c12
sinC = 1 =
36 6
a ×a a ×b a ×c
Now area of ABC
= b ×a b ×b c ×c
1

1
= |CA CB| = | a || b |sin C c ×a c ×b c ×c
2 2
Answer: (A)
Worked-Out Problems 447


92. If a, b, c are coplanar
vectors, then the value of Equation of the line AL is

[b ´ c c ´ a a ´ b] is
(2a + b )
(A) 2 (B) 1 (C) 0 (D) | a × b + b × c + c × a | r =t , t Î
3
Solution: We have
Equation of the line CK
K is

[b ´ c c ´ a a ´ b] = ((b ´ c ) ´ (c ´ a )) ×(a ´ b) s
r = a + (1 - s)b
= ([b c a]c - [c c a ]b) ×(a ´ b) 3
[For Eqs. (6.47) and (6.48) see Theorem 5.30.] Equating
= [b c a ] [c a b ] - 0
the corresponding coefficients of a and b in Eqs. (6.47)

= [a b c ]2 (∵ [a b c ] = [b c a ] = [c a b]) and (6.48) we have

= 0 (∵ a, b, c are coplanar) 2t s
= and
t
=1- s
3 3 3
For deriving this we have used Theorem 6.48 part (2)
and Theorem 6.36. Therefore
Answer: (C) s t
= =1- s
6 3
93. In the DABC (Figure 6.46), points K and L are taken
6 3
respectively on the segments AB and BC C such that Þs= and t =
AK :KB = 1:2 and BL :LC = 1:2. Let P be the point 7 7
of intersection of the lines AL and CK. If the area


Therefore position vector AP of P is
of the DBCP P is 2 sq. units, then the area of the D ABC
in sq. units is 1
(2a + b )
7 7 7
(A) (B) (C) 7 (D) 7
4 2
Hence
A

1 1
PB = a - (2a + b ) = (5a - b )
7 7
and
1

1 1 2
PC = b - (2a + b ) = (6 b - 2a ) = (3b - a )
7 7 7
K
P Now
2 = Area of D ABC
1


2 = | PB ´ PC |
2

C
1 2
2 = ´ |(5a - b ) ´ (3b - a )|
L 2 49
2
1
1
= | 15a ´ b - a ´ b | = |a ´ b |
FIGURE 6.46 Single correct choice type question 93.
49 7

Therefore |a ´ b | = 7. Now
let AB = a , AC = b , so
Solution: Take A as origin and

that the angle between a and b is A. Note 1

1 7
Area of D ABC = | AB ´ AC | = |a ´ b | =

1 2 2 2
AK : KB = 1: 2 Þ AK = a Answer: (B)
3

2a + b
94. Let a = 3i - 6 j - k , b = i + 4 j - 3k and c = 3i - 4 j -
BL : LC = 1: 2 Þ AL =
3 12k then the projection of a ´ b on c in vector
form is
448 Chapter 6 Multiplication of Vectors


14c 14c 12c 15c 97. If (a b) c = 5a + 4b and a ×b = 3, where a, b, c
(A) (B) (C) (D)
13 13 13 13 are non-coplanar, then a (b c ) is equal to

Solution: Projection vector of b on a is [see Theorem
T 6.1 (A) 4b 3c (B) 3b 5c
part (2)]
(C) 3c 4b (D) 5b 3c

b ×a Solution: We have
a
|a |2 ÷
5a + 4b = (a b) c = (a ×c )b (b ×c )a

a b on c is Therefore

3 6 1 a ×c = 4 and b ×c = 5
(a b) ×c 1
(c ) = 1 4 3 c
| c |2 169 Now
3 4 12

1 a (b c ) = (a ×c )b (a ×b)c
= [3( 48 12) + 6(( 12 + 9) 1( 4 12)]c
169 = 4b 3c
1 Answer: (A)
= ( 180 18 + 16)c
169

98. Let a = i + j + 2k and b = 2 i k . The point of
14c
= intersection of

the lines represented

by the equa-
13 tions r a = b a and r b = a b is

Answer: (B) (A) 3i j + k (B) 3i + j + k

(C) i j k (D) i + j + k
95. If | a | = 6 and | b | = 5, then [(a b) b] b is equal to
Solution: We have
(A) 5(b a ) (B) 5(a b)

(C) 6(a b) (D) 6(b a) r a=b a

Solution: We have (r b ) a = 0

[(a b) b] b = [(a b) ×b]b (b ×b)(a b) r b is collinear with a

r b = ta, t 
= 0(b) 5(a b)

r = b + ta (6.49)
= 5(b a)
Similarly
Answer: (A)

r b=a b r = a + sb (6.50)
the angle between a and b be 120° and | a | = 1,
96. Let
| b | = 2. Then where s . The two lines given by Eqs. (6.49) and (6.50)
intersect if t = s = 1. Therefore the point of intersection is
|(a + 3b) (3a b)|2 =
a + b = 3i + j + k
(A) 300 (B) 225 (C) 275 (D) 325 Answer: (B)
Solution: We have
99. a ×[(b + c ) (a + b + c )] is equal to

|(a + 3b) (3a b)|2 = | (a b) + 9(b a )|2 (A) 0 (B) 2[a b c ]

= 102 | a b |2 (C) [a b c ] (D) [a b c ]
Solution: We have
= 100 | a |2 | b |2 sin2 120°

3 a ×[(b + c ) (a + b + c )] = a ×[(b + c ) a ]
= 100 (1)(4) ÷
4 = a ×[b a + c a ]
= 300
= [a b a ] + [a c a ]
Answer: (A) =0+0=0
Answer: (A)
Worked-Out Problems 449


100. a, b, c are three non-coplanar vectors. Then 102. If a, b, c are mutually perpendicular vectors of

equal magnitudes and x is a vector satisfying the
(a + b + c )×[(a + b) (a + c )] = equation

(A) 0 (B) [a b c ] a [( x b)a ] + b [( x c ) b] + c [( x a ) c ] = 0

(C) 2[a b c ] (D) [a b c ]
then x equals
Solution: Given scalar is
1 1
(A) (a + b 2c ) (B) (a + b + c )
(a + b + c ) ×[ c a a b + b c ] = [a b c ] [b c a] [c a b] 2 2
1 1
= [a b c ] (C) (a + b + c ) (D) (2a + b c )
3 3
Answer: (D)
Solution: We have

101. Let a = xi + yj + zk be a vector of magnitude 2 3

a [( x b) a ] = (a ×a )( x b) a ×( x b)a
and makes an obtuse angle with y-axis.
If a makes

equal angles with the vectors b = yi 2zj + 3 xk and
= | a |2 ( x b) (a ×x a ×b)a (6.55a)
c = 2zi
+ 3 xj yk and is perpendicular to d = i

j + 2k, then a is equal to Similarly

(A) 2 i 2 j 2k (B) 2 i + 2 j 2k
b [( x c ) b] = | b |2 ( x c ) (b ×x b ×c )b (6.54b)
(C) 2 i 2 j + 2k (D) 2 i + 2 j + 2k
c [( x a ) c ] = | c |2 ( x a) (c ×x c ×a)c (6.55c)
Solution: We have | a | = 2 3 . Therefore

Let | a | = | b | = | c | = l . Since a ×b = b ×c = c ×a = 0, by adding
x2 + y2 + z2 = 12 (6.51) Eqs. (6.55a), (6.55b) and (6.55c), we have

Since | b | = | c | and a makes equal angles with b and c,
we have 3l 2 x l 2(a + b + c ) [(a ×x)a + (b ×x)b + (c ×x)c ] = 0 (6.56)
Suppose
a ×b = a ×c

Therefore x = a1a + b1b + c1c
xy 2 yz + 3zx = 2zx + 3 xy yz
(This is possible, since a, b, c are non-coplanar.) Therefore
2 xy + yz zx = 0 (6.52)
a ×x = a1 | a |2 = l 2 a1
Also
b ×x = l 2 b1
a ×d = 0
x y + 2z = 0 (6.53)
c ×x = l 2 c1

Also a makes an obtuse angle with y-axis. This implies
Substituting these values in Eq. (6.56) we have
y<0 (6.54)

3l 2 x l 2 (a + b + c ) l 2 x = 0
From Eqs. (6.52) and (6.53) we have

2x = a + b + c
2 x( x + 2z) + ( x + 2z)z zx = 0
1
2 x2 + 4zx + 2z2 = 0 x = (a + b + c )
2
(z + x)2 = 0
Note: Since a, b, c are mutually perpendicular
vectors

z= x
of same
magnitude, one can take a = pi , b = pj and

From Eq. (6.53) we have y = x. Therefore, from Eq. (6.51) c = pk and proceed.
Answer: (B)
x2 + x2 + x2 = 12 x2 = 4

x = ±2 103. Let the
vectors
a, b, c and d be such that (a b)

(c d ) = 0. Let P1 and P2 be the planes determined
Since from Eq. (6.54) y < 0, we have x = 2, y = 2 and
by the pairs of vectors a, b and c , d respectively.
z = 2. Therefore
Then the angle between P1 and P2 is

a = 2 i 2 j 2k p p p
(A) 0 (B) (C) (D)
Answer: (A) 4 3 2
450 Chapter 6 Multiplication of Vectors


Solution: (a b ) (c d ) = 0 a b and c d are (A) 2 :1 (B) 2 : 3
(C) 3 : 2 (D) 1: 2


parallel. But a b and c d are normals to the planes
Solution: Take A as origin, let AB = b and AC = c .
P1 and P 2, respectively. Also, since (a, b, a b) and
Therefore
(c , d, c d ) are right-handed systems, it follows that a b

1
and c d are like vectors. Since the angle between P1 and AD = c
P2 is the angle between their normals (see Definition 6.3) 2

it follows that the angle between P1 and P2 is 0.

b + 2c
Answer: (A) and AE =
3

104. A plane
P1 is parallel to the vectors a = 2 j + 3k and Suppose the ratio AP :PE = l :1 and the ratio BP :PD = m :1.
b = 4 j 3k and a plane
P is parallel
to the vectors Then
2
c = j k and d = 3i + 3k . Vector A is parallel to the
b + m (c / 2) l (b + 2c )
line of intersection
of P1 and the angle q
P 2. Then =
between A and the vector 2 i + j 2k , 0 q p / 2 is m+1 3(l + 1)
p p p p
(A) (B) (C) (D) b and c are non-collinear vectors. This implies
2 4 6 3
1 l
Solution: Normal to P1 = a b = 18 i and normal to =
m + 1 3(l + 1)
P2 = c d = 3i 3 j 3k . By hypothesis, A is perpen-

dicular to both a b and c d and hence we can take m 2l
and =
2( m + 1) 3(l + 1)
A = (a b) (c d ) = 54( j + k )
Therefore Solving these we get that
3
54( j k )×(2 i + j 2k ) 54 3 1 m = 4 and l =
cosq = = = 2
54 2 3 54 2 3 2
Therefore
and so
AP : PE = 3 : 2
p
q=
4 C
3p 1
Note: If A = (c d ) (a b), then q = . E
4 1
D
l P m 2

FIGURE 6.48 Single correct choice type question 105.


P2
Answer: (C)

106. If a, b, c are unit coplanar
vectors, then the scalar

triple product [2a b 2b c 2c a ] equals
P1
(A) 2 (B) 1 (C) 3 (D) 0
Solution: The given scalar triple product is

= (2a b) ×[(2b c ) (2c a )]
FIGURE 6.47 Single correct choice type question 104.
= (2a b)×[4(b c ) + 2(a b) + (c a )]
Answer: (B)
= 8[a b c ] [b c a ]
105. In ABC (Figure 6.48), D is the mid-point of
= 7[a b c ]
ACC and E is a point on the side BC C such that
BE : EC = 2 :1. If AEE and BD intersect in P, then = 7(0) = 0
the ratio AP : PE is Answer: (D)
Worked-Out Problems 451


107. Let a = 2 i + 3 j + 4k , b = i + 5 j + 2k and c = 3i + Solution: As per Theorem 5.33
15 j + 6k . Then
u+v
a=
a ×a a ×b a ×c |u + v |

b ×a b ×b b ×c = v + w
b=
c ×a c ×b c ×c | v + w|

w+u
(A) 0 (B) 8 (C) 4 (D) 6 c=
|w + u |

Solution: Under Quick Look 11, part (2), put l = a, m = b
Now
and n = c . Then the given
determinant value is [a b c ]2.

But by hypothesis c = 3b, so that [a b c ] is zero. [a bb cc a ] = (a b)×[(b c ) (c a )]
Answer: (A)
= (a b)×([b c a ]c [b c c ]c )
108. Let A, B, C and D be any four points in the space. (by Theorem 6.48)
( )2 + (BD)2 + (AD
Then the minimum value of (AC ( )2 +
2
(BC) is = [(a b)×c ][b c a ] = [a b c]2
(A) (AD)2 + (BC)2
a, b and c we have
(B) (AB)2 + (CD)2

(C) (AC)2 + (BD)2 [a b b c c a]
(D) no minimum value 1

= 2 2 2 [u + v v + w w + u]
|u + v | | v + w| |w + u|
Solution:

Take A as origin and let AB = b, AC = c and


AD = d. Then 1
= (2[u v w])2
a 2 b 2 g
( AC )2 + ( BD)2 + ( AD)2 + ( BC )2 4 cos2 ÷ 4 cos ÷ 4 cos ÷
2 2 2

= | c |2 + | d b |2 + | d |2 + | c b |2 1
= [u v w]2
a b g
= | c |2 + | b |2 + | d |2 2(b ×d ) + | d |2 + | b |2 + | c |2 2(b ×c ) 16 cos2 cos2 cos2
2 2 2

= | b c d |2 + | b |2 + | c |2 + | d |2 2(c ×d ) Answer: (D)

= | b c d |2 + | b |2 + | c d |2 | b |2 + | c d |2 ( ) + (CD) = (BC) +
110. In a Quadrilateral ABCD, if (AB
2 2 2

2
(AD) then the angle between the diagonals AC
where the equality occurs

if b = c + d which is

if

and
only and BD is
equivalent to b c = d or CB = AD. Hence the minimum (A) 120° (B) 90°
(C) 60° (D) 45°
value is


Solution:

Take A as origin and let AB = b, AC = c and


AD = d. Now
| b |2 + | c d |2 = ( AB)2 + (CD)2
Answer: (B) ( AB)2 + (CD)2 = ( BC )2 + ( AD)2

| b |2 + | d c |2 = | c b |2 + | d |2
109. Let u, v and w be non-coplanar unit vectors and the

angles (u, v) = a , (v, w) = b and (w, u) = g . Suppose
c ×d = c ×b (6.57)
a, b and c be the unit vectors along the bisectors of
a b and g respectively. Then the value of the scalar
a, Again
triple product


AC ×BD = c ×(d b)
1 a b g
[a b b c c a ] = [u v w]2 sec2 sec2 sec2
K 2 2 2 = c ×d c ×b
= 0 [by Eq. (6.57)]
where the value of K is
(A) 2 (B) 4 (C) 8 (D) 16 Therefore AC
C and BD are at right angles.
Answer: (B)
452 Chapter 6 Multiplication of Vectors

Multiple Correct Choice Type Questions



1. Let a = 2 i + j k, b = i + 2 j + 3k and c = 3i j + k. (D) We have
Then
[a b b c c a] = [a b c ]2 = ( 5)2 = 25
(A) the volume of the parallelopiped whose coter-

minus edges are a, b, c is 5 cubic units. So (D) is correct.

(B) | a (b c )| = 5 5 Answers: (A), (C), (D)

(C) area of the face with a, b as adjacent sides is 83
2. Let l be a real scalar and
(D) [a b b c c a ] = 25
a = li + 2 lj 3lk
Solution:
b = (2 l + 1)i + (2 l + 3) j + (l + 1)k
(A) Volume of the parallelopiped = |[a b c ]| where

2 1 1 c = (3l + 5)i + (l + 5) j + (l + 2)k

[a b c ] = 1 2 3
Then a, b, c are coplanar
3 1 1
(A) for exactly one real values of l
= 2(2 + 3) 1(1 + 9) 1( 1 + 6) (B) for exactly two real values of l
= 5 (C) for exactly three real values of l
(D) for no non-zero real value of l
Therefore
Solution: The vectors are coplanar if and only if
|[a b c ]| = 5
l 2l 3l
Hence (A) is correct. = 2l + 1 2l + 3 l + 1 = 0
(B) We have 3l + 5 l + 5 l + 2

a (b c ) = (a ×c )b (a ×b)c The operations C2 2C1 and C3 + 3C1 imply

= ( 6 1 1)(i + 2 j + 3k ) (2 + 2 3))( 3i j + k)
l 0 0

= 8(i + 2 j + 3k ) ( 3i j + k ) = 2l + 1 2l + 1 7l + 4
3l + 5 5l 5 10 l + 17
= 5i 15 j 25k
= l[( 2 l + 1)(10 l + 17) + 5(l + 1)((7 l + 4)]
= 5(i + 3 j + 5k )
= l[ 20 l 2 24 l + 17 + 35l 2 + 55l + 20]
Therefore
= l (15l 2 + 31l + 37)
|a (b c )| = 5 1 + 9 + 25
Clearly l = 0 = 0 and the vectors are coplanar. In

= 5 35 this case a = 0 which is coplanar with any two vectors.
Now
Hence (B) is not correct.

(C) Area of the parallelogram with a and b as adjacent 15l 2 + 31l + 37 0

sides = | a b |. Now for all l 0 because its discriminant is negative.
Therefore (A) and (D) are correct.
i j k
Answers: (A), (D)
a b= 2 1 1 = 5i 7 j + 3k
1 2 3
3. Let a = i + j + k and b is a vector such that a ×b = 1

and a b = j k . Then
Therefore
(A) b = 2 i (B) b = i

|a b | = 25 + 49 + 9 = 83 (C) (a b) b = ( j + k ) (D) (a b) b = k j
Hence (C) is correct.
Worked-Out Problems 453


Solution: We have (C) | x ´ y | = 2 sin q

j - k = a ´ b Þ a ´ ( j - k ) = a ´ (a ´ b ) (D) | x ´ y |, two times the area of the parallelogram

with a and b as adjacent sides

Þ (i + j + k ) ´ ( j - k ) = (a × b)a - (a × a )b
Solution: We have

Þ - 2 i + j + k = a - 3b
x × y = (a + b ) × (a - b )
Þb=i
= | a |2 - | b |2
=1-1= 0
Also
So (B) is correct. Also
(a ´ b ) ´ b = (a ´ i ) ´ i

= (a × i )i - (i × i )a x ´ y = (a + b ) ´ (a - b )

= i - a = -j - k = - 2(a ´ b)

So (C) is correct. Therefore


Answers: (B), (C)
| x ´ y| = 2| a ´ b |

4. Let a, b and c be the position vectors of three non-
= 2 | a || b | sin q
collinear points A, B and C, respectively, and p = b ´
= 2 sin q
c + c ´ a + a ´ b. Then

(A) p is perpendicular to the plane of DABC

(B) | p| is twice the area of DABC | x ´ y| = 2 | a ´ b |

(C) [b ´ c c ´ a a ´ b] = 2[a b c ] This implies (D) is correct.

(D) a × p + b × p + c × p = 3[a b c ] Answers: (B), (C), (D)


Solution: We have AB = b - a, AC = c - a. Then
6. ABCD is a regular tetrahedron. P and Q are the mid-


AB ´ AC = (b - a ) ´ (c - a ) points of the edges AC C and AB, respectively, G is the
centroid of the

face

and q is the angle between


BCD
=b´c-b´a-a´c the vectors PG and DG. Then


=b´c+c´a+a´b (A) the angle between AB and CD is 90°

æ 5 ö
But AB ´ AC is perpendicular to the plane of DABC. (B) the angle q is p - Cos-1 ç
è 6 3 ÷ø
Therefore p is perpendicular the plane of DABC. Hence
(A) is correct.

æ 5 ö
(C) q is Cos-1 ç
Also (1/ 2)| AB ´ AC | is the area of DABC. So (B) is è 6 3 ÷ø
correct. Again


(D) angle between AB and CD is 120°
[b ´ c c ´ a a ´ b] = [a b c ]2

Take D as the origin and let DA = a, DB = b


Solution:
This implies (C) is not correct. Finally and DC = c . See Figure 6.49. Then

a × p + b × p + c × p = [a b c ] + [b c a ] + [c a b] = 3[a b c ]

(a + c )
DP =
2
This means (D) is correct.

a + b
Answers: (A), (B), (D) DQ =
2

5. Let a and b be
unit vectors and q be the angle

b + c

between a and b. Let x = a + b and y = a - b. Then DG =
3
(A) x × y = 1
Since ABCD is regular all the faces are equilateral
(B) x × y = 0 triangles with equal edges and hence the angles of each
454 Chapter 6 Multiplication of Vectors

triangular face are 60°. In a tetrahedron ABCD, the pairs 1 é 2 3 2 K2 K2 ù


=- ê 3K + K - K 2 - 2 K 2 + +
of edges (AD, BC), (CD, AB) and (BD, AC) are called 12 ë 2 2 2 úû
opposite pairs of edges. Now

=-
5K 2
(6.61)
AD × BC = (- a) × (c - b) = a × b - a × c
24

= | a || b | cos 60° Therefore, using Eqs. (6.60) and (6.61) we have

= - | a || c | cos 60° = 0


PG × DQ 5K 2 4
cos q =

= -

´
because | a | = | b | = | c |. | PG || DQ| 24 3K 2
5
A =-
6 3
Hence

æ 5 ö
q = cos ç -
Q
P è 6 3 ÷ø
D
æ 5 ö
= p - Cos-1 ç
è 6 3 ÷ø
G
(B) is correct.
Answers: (A), (B)
FIGURE 6.49 Multiple correct choice type question 6.
7. A, B, C and D are fixed points in space or in plane and
M is any arbitrary point in the space. Which of the
Therefore AD and BC C are at right angles. Hence oppo-
following true?
site pairs of edges are at right angles. (A) is correct. Now

1 (A) MA × MC = MB × MD if ABCD is a rectangle.


1


PG = (b + c ) - (a + c ) (B) | MA|2 + | MC |2 = | MB|2 + | MD|2 if ABCD is rectangle.
3 2


1 1 (C) If AM ×CM ¹ BM × DM for all positions of M,
= (2b - c - 3a ) = - (3a - 2b + c ) then ABCD is a parallelogram without being a
6 6

1 rectangle.
DQ = (a + b)


2 (D) If AM ×CM ¹ BM × DM for all positions of M,

2 then ABCD is a trapezium which is not a


36 | PG | = 9 | a |2 + 4 | b |2 + | c |2 - 12(a × b) - 4(b × c ) + 6(c × a) parallelogram.

= 9 | a |2 + 4 | b |2 + | c |2 - 12 | a || b | cos 60°

Take A as origin and let AB = b, AD = d


Solution:
- 4 | b || c | cos 60° + 6 | c || a | cos 60° and AM = m. See Figure 6.50. ABCD is a rect-

Suppose
angle so that b × d = 0 and AC = b + d. Now

It is known that | a | = | b | = | c | = K (suppose). Therefore

M
36 | PG |2 = K 2 (14 - 6 - 2 + 3) = 9 K 2 (6.58)
m
Again

D C
4 | DQ|2 = | a + b |2 = | a |2 | b |2 + 2 | a || b | cos 60°
= 3K 2 (6.59)
d
From Eqs. (6.58) and (6.59) we have

3K
| PG | = and | DQ| = (6.60) A B
2 2
Now FIGURE 6.50 Multiple correct choice type question 7.

1 1
PG × DQ = - (3a - 2b + c ) × (a + b)
6 2
Worked-Out Problems 455


MA ×MC = ( m) ×(b + d m) a b
g =
[a b c ]
= m ×b m ×d + | m|2

and x > 0. Then


MB ×MD = (b m) ×(d m)
[a b g ]
= b ×d b ×m m ×d + | m|2 (A) x[a b c ] + 2
x
(B) [a b g ] is a positive scalar
= m ×b m ×d + | m|2 (∵ b ×d = 0) (6.63)
(C) a ×a + b ×b + c ×g = 3
Therefore Eqs. (6.62) and (6.63) imply
(D) b ×a + c ×b + a ×g = 3


MA ×MC = MB ×MD Solution: We have
Hence (A) is correct. 1
Again [a b g ] = 3 [b c c a a b]
[a b c ]


| MA|2 + | MC |2 = | m|2 + | b + d m|2
[a b c ]2 1
= 3 = 0
= 2 | m|2 + | b |2 + | d |2 2 m ×(b + d ) (∵ b ×d = 0) (6.64) [a b c ] [a b c]

Also
Also (a, b, c ) is a right-handed system (a , b , g ) is also

right-handed system.
| MB|2 + | MD|2 = | b m|2 + | d m|2

[a b g ][a b c ] = 1
= 2 | m|2 + | b |2 + | d |2 2 m ×(b + d ) (6.65)

and [a b c ] > 0 [a b g ] > 0
Equations (6.64) and (6.65) imply

Hence (B) is correct.


| MA|2 + | MC |2 = | MB|2 + | MD|2
Now AM GM implies
Hence (B) is correct.


Suppose AM ×CM BM ×DM for all points M. Then [a b g ] [a b g ]
x[a b c ] + 2 x[a b c ] =2
x x
m ×(m c ) (m b)×(m d )
and hence least value is 2. So (A) is correct.
m ×(d + b c ) (b ×d ) (6.66) Now

Let a = b + d

c. We

prove
that a = 0 so that b c = d [a b c ] [b c a ] [c a b ]
a ×a + b ×b + c ×g = + + = 1 + 1 + 1 = 3
in which case BC = AD and hence ABCD is a parallelo- [a b c ] [a b c ] [a b c ]

gram. Suppose a 0. Then the vector
So (C) is correct.
b ×d
m = 2 ÷a Also, b ×a = 0, c ×b = 0 and a ×g = 0, therefore (D) is not
|a | correct.
Answers: (A), (B), (C)
satisfies the equation m ×a = b ×d which

is
a

contradiction



to Eq. (6.66). Therefore a = 0. Now AM ×CM BM ×DM
9. b and c are non-collinear vectors. If a is a vector such
assures that ABCD is not a rectangle because of (A). 2
that | c | a = c and
Answers: (A), (B), (C)
a (b c ) + (a ×b)b = (4 2 x sin y)b + ( x2 1)c

8. Let a, b and c be non-coplanar vectors such that
then
(a, b, c ) is a right-handed system. Let
(A) x = 1
b c p
a= (B) y = (4 n + 1) , n 
[a b c ] 2
c a p
b= (C) y = (2 n + 1) , n 
[a b c ] 2
(D) x = 1
456 Chapter 6 Multiplication of Vectors

Solution: By hypothesis E D

(a ×c )b (a ×b)c + (a ×b)b = (4 2x sin y)b + ( x2 1)c

and | c |2 a = c

Now
O
F C
| c |2 a = c

| c |2 (a ×c ) = | c |2

a ×c = 1
Therefore

(1 + a ×b)b (a ×b)c = (4 2x sin y)b + ( x2 1)c FIGURE 6.51 Multiple correct choice type question 10.

This gives (B) is correct.



1 + a ×b = 4 2 x sin y Answers: (A), (B)

and (a ×b) = x2 1 11. Let a and b be two
non-collinear
unit vectors. If
Solving we get a = a (a ×b)b and b = a b, then | b | is

(A) |a | (B) |a | + |a ×a |
1+ 1 x2 = 4 2x sin y
(C) |a | + |a ×b | (D) |a | + a ×(a + b)
x2 2 x + 2 = sin y

Solution: Let q be the angle between a and b. Then
(x 1)2 + 1 = sin y

a ×b = | a || b | cosq = cosq
So 1 sin y 1 x 1 = 0 and sin y = 1. Therefore
x=1 Now

p |a |2 = | a (a ×b)b |2
and y = (4 n + 1)
2
= | a |2 + (a ×b)2 | b |2 2(a ×b)2
Answers: (B), (D)
= 1 + cos2 q 2 cos2 q
10. Let ABCDEF

F be

a regular

hexagon

(Figure 6.51)
such that AD = xBC and CF = yAB. Then = sin2 q

2
(A) xy = 4 (B) AF ×AB + BC = 0 = (| a || b | sin q )2 (∵ | a | = | b | = 1)
2

2
(C) xy = 4 (D) AB ×AF + BC = 0 = | a b |2
3
= | b |2
Solution: Let “O” be the centre of the hexagon. Then


xBC = AD = 2 AO = 2 BC

x=2 | b | = |a |


yAB = CF = 2CO = 2 AB Hence (A) is correct. Now
y= 2
a ×b = [a (a ×b)b]×b
Hence
= a ×b (a ×b)(b ×b)
xy = 4
= a ×b a ×b (∵ | b | = 1)
(A) is correct. Now

2 =0
1
AB ×AF = | AB| cos 120° = ( BC )2
2
Worked-Out Problems 457


Since (C) not equal to (a ´ b)× c

|a | + |a × b | = |a | + 0 = |a | = | b | (D) not equal to c ×(b ´ a )
Solution: It is known that
Therefore (C) is correct.

Answers: (A), (C) a × (b ´ c ) = b × (c ´ a ) = c × (a ´ b )
Therefore
12. The vector (1/ 3)(2 i - 2 j + k ) is
(A) a unit vector b × (a ´ c ) = c × (b ´ a )

(B) makes an angle p /3 with the vector 2 i - 4 j + 3k Therefore (D) is not correct.
1 Answers: (A), (B), (C)
(C) parallel to the vector - i + j - k
2
14. For any two vectors a and b which of the following
(D) perpendicular to the vector 3i + 2 j - 2k are true?

Solution: Let (A) | a ´ b |2 + (a × b)2 = | a |2 | b |2

1 (B) | 1 - a × b |2 + | a + b + a ´ b |2 = (1 + | a |2)(1 + | b |2)
a = (2 i - 2 j + k )
3 (C) (a ´ b) ´ a = | a |2 b + (a × b)a

so that (D) a ´ (b ´ a ) = | a |2 b - (a × b)a
1 Solution:
| a |2 = (22 + 22 + 1) = 1
9 (A) We have
Therefore
2 a ×a a ×b
| a ´ b | = (Theorem 6.48)
|a | = 1 b×a b×b
(A) is correct.
= | a |2 | b |2 - (a × b)2
Let b = 2 i - 4 j + 3k . Then
Therefore
1
a × b = (4 + 8 + 3) = 5 | a ´ b |2 + (a × b)2 = | a |2 | b |2
3
So (A) is correct.
Let q = (a, b). Then
(B) We have
a ×b 5 5
cosq = = = | 1 - a × b |2 + | a + b + a ´ b |2
| a || b | 1× 4 + 16 + 9 29

= 1 - 2(a × b) + (a × b)2 + | a |2 + | b |2 + 2(a × b) + | a ´ b |2
Hence (B) is not correct.

Now [∵ a × (a ´ b) = b × (a ´ b) = 0]
1
1 3 = 1 + | a |2 | b |2 cos2 q + | a |2 + | b |2 + | a |2 | b |2 sin2 q
- i + j - k = - (2 i - 2 j + k ) = - a
2 2 2
= 1 + | a |2 + | b |2 + | a |2 | b |2 (cos2 q + sin2 q )

Therefore a is parallel b. Hence (C) is correct.
Also = (1 + | a |2 )(1 + | b |2 )

1 This implies (B) is correct.


a ×(3i + 2 j - 2k ) = (6 - 4 - 2) = 0 (C) From Theorem 6.46 we have
3

Therefore (D) is correct. (a ´ b) ´ a = (a × a )b - (a × b)a
Answers: (A), (C), (D) Hence (C) is not correct.
(D) Now
13. For three vectors a, b, c , b × (a ´ c ) is

(A) not equal to a ×(b ´ c ) a ´ (b ´ a ) = (a × a )b - (a × b)a

(B) not equal to (b ´ c )× a Therefore (D) is correct.
Answers: (A), (B), (D)
458 Chapter 6 Multiplication of Vectors


15. Let AB = 3i + 4k and AC = 5i - 2 j + 4k . Then Therefore
(A) length of the median through A of DABC
C is 33


| BC ´ AB | = 64 + 64 + 36 = 164
(B) the length of the altitude from A to the base BC
41 Also
of DABC C is


2 | BC | = 8
(C) the area of DABC
C is 41 sq. units
Now by Eq. (6.67)
(D) the area of DABC
C is 10 sq. units
Solution: 164
AM =
(A) Suppose AD is the median. Then 8

41

AB + AC =
AD = 2
2

8 i - 2 j + 8k Hence (B) is correct.
=
2 (C) We have

= 4 i - j + 4k i j k


So AB ´ AC = 3 0 4

5 -2 4
| AD| = 42 + 12 + 42 = 33
= 8 i + 8 j + 6k
Hence (A) is correct.
Therefore by Theorem 6.29,
(B) We have


AB + BC = AC Þ BC = AC - AB = 2 i - 2 j Area of D ABC = | BC ´ AB |
2
Suppose AM M is the altitude from A to BC 1
= 164 = 41
(Figure 6.52). Then 2
AM This means (C) is correct.
sin B =
AB Answers: (A), (B), (C)
Þ AM = ( AB)sin B
16. If the lines
( BC )( AB)sin B
=
BC x-2 y-3 z-4

= =
| BC ´ AB| 1 1 -K
=

(6.67)
| BC | x-1 y-4 z-5
and = =
K 2 1
Now
are coplanar, then K may be
i j k (A) 0 (B) 3 (C) -2 (D) -3


BC ´ AB = 2 - 2 0
Solution: Let the given lines be L1 and L2, respec-
3 0 4 tively. L1 is passing through the point (2, 3, 4) and
having direction ratios 1, 1 and -K. L2 is passing through
= - 8 i - 8 j + 6k the point (1, 4, 5) and having direction ratios K, 2, 1
A
(see Theorem 5.28, Chapter 5). Let A
= (2, 3, 4) and
B = (1, 4, 5). Also a = (1, 1, - K ) and b

= (K , 2, 1). The

lines are coplanar if and only if vectors AB, a and b are
coplanar. Therefore by Corollary 6.16

1 -1 -1
1 1 -K = 0
K 2 1
FIGURE 6.52 Multiple correct choice type question 15.
Þ (1 + 2 K ) + (1 + K 2 ) - (2 - K ) = 0
Worked-Out Problems 459

K 2 + 3K = 0 Hence by Eq. (6.68)


K = 0 or 3 270
Shortest distance =
Answers: (A), (D) 3 30

17. Consider the lines = 3 30

x 3 y 8 z 3 So (D) is correct.
L1 : = =
3 1 1 Answers: (C), (D)
x+3 y+7 z 6
L2 : = = . Then 18. Let a be a vector parallel to the line of intersection
3 2 4 P
of the planes
1 and P 2. Plane
P1 is parallel to the
(A) L1 and L2 are coplanar vectors 2 j + 3k and 4 j 3 k. Plane P2 is parallel to
the vectors j k and 3i + 3 j . Then the angle
between
(B) L1 and L2 are not coplanar and shortest distance
the vector a and a given vector 2 i + j 2k is
between them is 2 30
p p 3p 2p
(C) L1 and L2 are skew lines (A) (B) (C) (D)
3 4 4 3
(D) the shortest distance between L1 and L2 is 3 30
Solution: We have
Solution: The line L1 is passing through the point A (3, 8,

3) and having direction ratios 3, 1, 1. The line L2 is passing n1 = normal to P1 = (2 j + 3k ) (4 j 3k ) = 18 i
through the point B ( 3, 7, 6) and having direction

ratios –3,
2, 4. L and L are coplanar if and only if AB, a = (3, 1, 1) n2 = normal to P2 = ( j k ) (3i + 3 j ) = 3(i j k )
1 2
and b = ( 3, 2, 4) are coplanar. Therefore by Corollary 6.16,
this would happen if and only if
Now
a is parallel n1 n2 angle between

a and 2 i +
j 2k is same as angle between n1 n2 and 2 i + j 2k .
6 15 3 Then
3 1 1 =0
i j k
3 2 4
n1 n2 = 54 1 0 * = 54( j + k )
Now 1 1 1

6 15 3 q be the angle between n1 n2 and 2 i + j 2k .
3 1 1 = 3[ 2( 4 2) + 5(12 + 3) + (6 3)] Therefore
3 2 4 54( 1 2) 1
cosq = =
= 3[12 + 75 + 3] 0 54 2 (3) 2
Therefore L1 and L2 are not coplanar and hence they are This implies
skew lines. Therefore (C) is correct.
The shortest distance between L1 and L2 is (using p 3p
q= ,
Theorem 6.44) 4 4


| AB ×(a b)| Angle depends on the direction of a which is parallel to

(6.68) n1 n2 . If we take a = n2 n1, then
|a b|
p
q=
Now 4


AB = ( 6, 15, 3) = 3( 2, 5, 1) Answers: (B), (C)

i j k 19. x, y and z are vectors of equal magnitudes 2 and

a b= 3 1 1 = 6 i 15jj + 3k each is inclined to the others at an angle of 60°. If

a = x ( y z), b = y (z x) and c = x y, then
3 2 4
(A) x ×y = y ×z = z ×x = 1
Therefore
(B) x = a c


AB ×(a b) = 3(12 + 75 + 3) = 270 (C) y = b c

| a b | = 3 4 + 25 + 1 = 3 30 (D) z = b + (a c )
460 Chapter 6 Multiplication of Vectors


Solution: We have Now PQ is perpendicular to i 4 j + 3k . This implies
1 ( 2 3l )(1) + (l 3)( 4) + (5l 4)3 = 0
x ×y = | x || y | cos60° = 2 2 =1
2 1
8l = 2 l=
Therefore (A) is correct. 4
Hence (A) is correct. Now
a = x ( y z) = ( x ×z) y ( x ×y)z = y z (6.69)


b = y (z x) = ( y ×x)z ( y ×z) x = z x (6.70) 29 = | PQ|2 = (2 + 3l )2 + (l 3)2 + (5l 4)2
35l 2 34 l = 0
Now by Eq. (6.69)
l = 0 or
34
a c = ( y z) ( x y)
35

= y (x y) z ( x y)
Hence (B) is correct. Again

= ( y ×y) x ( y ×x) y [(z ×y) x (z ×x) y]


PQ = i 4 j 9k
= 2x y x+ y=x
2 3l = 1, l
3 = 4 and 5l 4 = 9
Hence (B) is correct. Again


Hence l = 1 and (C) is correct. PQ is perpendicular to

b c = (z x) ( x y) X-axis means
X


= z ( x y) x ( x y) PQ ×i = 0

= (z ×y) x (z ×x) y [( x ×y) x ( x ×x) y] 2 3l = 0

= x y x + 2y = y 2
l=
Hence (C) is correct. From Eq. (6.70), 3
So (D) is not correct.
z=b+x=b+a c
Answers: (A), (B), (C)
So (D) is correct.

Answers: (A), (B), (C), (D) 21. a, b, c and d are unit vectors such that

(a b) ×(c d ) = 1
20. Let P (3, 2, 6) be a point
in
the space Q is a point
and

on the line r = (i j + 2k ) + l ( 3i + j + 5k ). Then 1

and a ×c =
the value of l for 2
(A) PQ is perpendicular to
which
the vector i 4 j + 3k is 1/4


(B) the value of l for which PQ has magnitude
(A) a, b, c are coplanar
29 is 0 or 34/35

(B) b, c , d are coplanar


(C) the
value
of l for which PQ is equal to
i 4 j 9k is 1 (C) b, d are non-parallel


(D) the value of l for which PQ makes angle 90° (D) a, d are parallel and b, c are parallel
with X
X-axis is 3/2
Solution: (a b) ×(c d ) = 1 the angle between
a b

Solution: We have and c d is 0. Therefore a b and c d are parallel
vectors. So
Q = (1 3l )i + (l 1) j + (2 + 5l )k

(a b ) ( c d ) = 0
P = 3i + 2 j + 6 k
which implies
Therefore


[a b d ]c [a b c ]d = 0 (6.71)
PQ = ( 2 3l )i + (l 3) j + (5l 4)k

and [a c d ]b [b c d ]a = 0 (6.72)
Worked-Out Problems 461


If a, b, c are not coplanar, then from Eq. (6.71) it follows Therefore

that c and d are parallel so that c d = 0 which is not
x = a ×c = cosq
possible because (a b) ×(c d ) = 1. Therefore a, b, c are
coplanar. Similarly from Eq. (6.72) b, c , d are coplanar. Similarly
Hence (A) and (B) are correct.
Again y = b ×c = cosq

(a b) ×(c d ) = 1 | a b || c d | cos a = 1 Again

where a is the angle between a b and c d. So a = 0. 1 = | c |2
This means
= x2 | a |2 + y2 | b |2 + z2 | a b |2 + 2 xy(a ×b)

| a b || c d | = 1
+ 2zx[a a b] + 2 yz[b a b]

(| a || b | sin b )(| c || d | sin g ) = 1
= x2 + y2 + z2 | a |2 | b |2 sin2 90°

b = (a, b) and g = (c , d ). Hence
= cos2 q + cos2 q + z2
sin b sin g = 1
Therefore
p
b =g = z2 = cos 2q
2
Answers: (A), (C)

So a and b are at right angles and c and d are at right

angles. Now suppose b and d are parallel. Since a is unit vectors such that a is perpen-
23. Let a, b and c be
perpendicular to b, a is perpendicular to d. Now dicular to both b and c. If the angle between b and

c is p / 6, then a = l(b c ) where l may be
1 = (a b) ×(c d ) = a ×[b (c d )]
(A) 4 (B) 4 (C) 2 (D) 2

= a ×[(i ×d )c (b ×c )d ] Solution: We have

= (a ×c )(b ×d ) (a ×d )(b ×c ) 1 = | l || b c|

= (a ×c )(b ×d ) (∵ a ×d = 0) p
= | l || b || c | sin
6
1
= (b ×d )
2 1
= |l | ÷
2
Hence b ×d = 2 which is not possible because b and d
are unit vectors. So b and d cannot be parallel. Hence Therefore
(C) is correct.
l = ±2
Answers: (A), (B), (C)
Answers: (C), (D)

22. Let a and b be perpendicular unit vectors and c be

a unit vector equally inclined to both a and b at an 24. The
position
of two points A and C are 9 i
vectors
 j + 2k and 7 i 2 j + 7k,
angle q. If c = xa + yb + z(a b), then

6.53).

The
point
of

intersection
of
the vectors

AB = 4 i
(A) x = y = cos q
j + 3k and CD

= 2 i j
+

2k is P.

PQ
is perpendic-
(B) x = cos q, y = sin q ular to both AB and CD and | PQ| = 15. Then the
(C) x = cos q, z = cos 2q position vectors of the point Q is
(D) x = y = cos q, z = cos 2q (A) (4, 11, 11) (B) (6, 9, 9)
(C) ( 4, 11, 11) (D) ( 6, 9, 9)
Solution: By hypothesis a ×b = 0, a ×c = cosq = b ×c . Now
Solution: Equation of the lines AB and CD are,
a ×c = x(a ×a ) + y(b ×a ) + z[a a b] respectively (see Theorem 5.27, Chapter 5)
=x+0+0
462 Chapter 6 Multiplication of Vectors

C (7, –2, 7) Case 1: l = 5. This implies



PQ = 5i - 10 j - 10k

Hence


OQ = PQ + OP

2i –

j+
2k
= (5i - 10 j - 10k ) + (i + j + k )

= 6 i - 9 j - 9k

P
So (B) is correct.
A 4i – j + 3k B Case 2: l = -5. This implies
(9, –1, 7)


PQ = - 5i + 10 j + 10k
FIGURE 6.53 Multiple correct choice type question 24. Therefore


r = (9 i - j + 7) + t(4 i - j + 3k ), t Î  OQ = PQ + OP = (- 5i + 10 j + 10k ) + (i + j + k )

and r = (7 i - 2 j + 7k ) + s(2 i - j + 2k ), s Î  = - 4 i + 11jj + 11k

Equating the corresponding coefficients in the above So (C) is correct.


equations we get Answers: (B), (C)
9 + 4t = 7 + 2 s
25. In DABC, let I and O be the incentre and circum-
or 4t - 2 s = - 2 (6.73) centre, respectively. Then


-1 - t = - 2 - s (A) aAI + bBI + cCI = 0 where a, b and c are the
or t-s=1 (6.74) sides lengths of BC, CA and AB, respectively


7 + 3t = 7 + 2 s (B) OI = (aOA + bOB + cOC ) where 2s 2 =a+b+c
2s
or 3t - 2 s = 0 (6.75)


(C) if H is the orthocentre, then OH = OA + OB + OC
Now t = -2 and s = -3 satisfy Eqs. (6.73), (6.74) and


(6.75). Therefore (D) OG = (OA + OB + OC )
3

P = (9 i - j + 7k ) - 2(4 i - j + 3k ) = i + j + k Solution: Let AD be the internal bisector of A meeting
Now the side BC
C in D. Then
BD : DC = c : b


i j k


AB ´ CD = 4 - 1 3 Take

the
circumcentre “O” as origin and let OA = a , OB = b

and OC = g . Then
2 -1 2

bb + cg
= i - 2 j - 2k OD =
b+c


Since PQ is perpendicular to both AB and CD, let Now I divides AD in the ratio (b + c) : a. So


PQ = l ( AB ´ CD) = l (i - 2 j - 2k ), l Î  æ bb + cg ö
aa + (b + c) ç ÷

è b+c ø
Then OI =

a + (b + c)
| PQ | = 15
1
Þ 3 | l | = 15 = (aa + bb + cg )
2s
Þ l = ±5
Worked-Out Problems 463

Hence (B) is correct. Also Therefore


1
AI = OI OA = (aa + bb + cg ) a aAI + bb + cg = [ab(b a ) ab(b a ) + ]
2s 2s
1 1
= (bb + cg ba ca) = (0) = 0
2s 2s
Similarly So (A) is correct. For (C) see Example 5.3, Chapter 5,
and (D) follows from the fact the centroid G divides the

1
BI = (aa + cg ab cb ) median through A in the ratio 2 :1 reckoning from A.
2s Answers: (A), (B), (C), (D)

1
CI = (aa + bb ag bg )
2s

Matrix-Match Type Questions



1. e1 , e2 and e3 are unit vectors such that Now

e1 + e2 + e3 = a e1 + e2 + e3 = a 1 + 1 + 1 + 2(e1 ×e2 ) + 2(e2 ×e3 )

e1 (e2 e3 ) = b + 2(e3 ×e1 ) = | a |2 = 4

(e1 e2 ) e3 = c Therefore

3 1
a ×e1 = e1 ×e2 + e2 ×e3 + e3 ×e1 = (6.78)
2 2
7 Again
a ×e2 =
4 a = e1 + e2 + e3

|a | = 2 3
a ×e1 =
2
7
a ×e2 =
4
Column I Column II
These imply
2
(A) e1 ×e2 is (p) 3
3 = a ×e1 = 1 + e1 ×e2 + e1 ×e3
2
1
(B) e2 ×e3 is
(q) 7
4 and = a ×e2 = e1 ×e2 + 1 + e2 ×e3
1 4
(r)
(C) e3 ×e1 is 2 Therefore
(s) 0 1
3 e1 ×e2 + e1 ×e3 =
(D) c = le2 where l is (t) 2
4 3
and e1 ×e2 + e2 ×e3 =
4
Solution: We have

e1 (e2 e3 ) = b (e1 ×e3 )e2 (e1 ×e2 )e2 = b (6.76)
e2 ×e3 = 0
and 1
e3 ×e1 =
(e1 e2 ) e3 = c (e1 ×e3 )e2 (e2 ×e3 )e1 = c (6.77) 4
3
e1 ×e2 =
4
464 Chapter 6 Multiplication of Vectors


Using the values of e1 ×e2 = 3 / 4, e2 ×e3 = 0 and e3 ×e1 = (1/ 4) (C) We have from Quick Look 11 that
in Eqs. (6.76) and (6.77) we have
1 1 0

1 3 [a b b + c c + a] = 0 1 1 [a b c ]
e2 e3 = b
4 4 1 0 1
1
and c= e2
4 = 0[a b c ] = 0
Answer: (A) Æ (t), (B) Æ (s), (C) Æ (q), (D) Æ q Answer: (C) Æ (p)
(D) We have
2. Match the items of Column I to those of Column II.
p 1
|a b | = sin =
6 2
Column I Column II
1
(A) If a = i + j + k , a ×b = 1 and (p) 0 [ a b c ] = (a b) ×c = | a b || c | sin 90° =
2
a b = j k then | b | is
Therefore
(B) If a = i 3 j + k , b = 2 i + lj + k (q) 1

and c = 3i + j 2k are coplanar, 1
then l equals [a b c ]2 =
4
1
(C) If a, b, c are non-coplanar

(r) Answer: (D) Æ (r)
vectors, then [a b b + c c + a ] is 4
equal to 3.

(D) a, b are unit vector which (s) 4

include angle p / 6. If c is a unit
Column I Column II
vector perpendicular to both a
2
and b, then [a b c ] equals (A) Let a = 2 i j + k , b = i + (p) 2 i j + 5k
2 j k and c = i + j 2k .
Solution: A vector in the plane of

(A) We have b and c whose projection

on a has magnitude
(a b ) a = ( j k ) (i + j + k ) (q) 5 (i + j 2k )
2 / 3 is


(a ×a )b (b ×a )a = 2 i j k a = 2 i + j + k , b = i +
(B) Let
2 j k. Then a vector

3b a = 2 i j k coplanar with a and b
and having magnitude (r) 2 i + 3 j 3k
Therefore 2 5 and perpendicular

to a is
3b = 3i or b = i
a = i j , b = j k , c =
(C) Let
so that | b | = 1. k i . If d (s) 10( j + k )
Answer: (A) Æ (q) magnitude 30
such that

(B) a, b, c are coplanar. This implies a ×d = 0 = [b c d ], then d
may be
1
1 3 1 (D) a = i + j + k , c = j k . (t) ( 5i + 2 i + 2 j )
3
2 l 1 =0 If b is a vector such that

3 1 2 a ×b = 3 and a b = c is

2l 1 + 3( 4 3) + (2 3l ) = 0 Solution:

5l 20 = 0 (A) Neither b nor c is a required vector. Let d be the
required vector and
l= 4

Answer: (B) Æ (s) d = b + xc = (1 + x)i + (2 + x) j (1 + 2 x)k
Worked-Out Problems 465

Therefore We have

| d ×a | 2 a=i + j +k
=
|a | 3 c=j k
2
(d ×a ) = 4
a ×b = 3
[2(1 + x) (2 + x) (1 + 2 x)]2 = 4
a b=c
x + 1= ± 2
Now
x = 1, 3

Answer: (A) Æ (p), (r) a b=c= j k

(B) Let c be required vector and (a b ) a = ( j k ) (i + j + k )

c = xa + yb = (2 x + y)i + ( x + 2 y) j + ( x y)k (a ×a )b (b ×a )a = 2 i j k

Then 3b 3a = 2 i j k

c ×a = 0 2(2 x + y) + 1( x + 2 y) + ( x y) = 0 3b 3(i + j + k ) = 2 i j k
6 x + 3y = 0 1
b = ( 5i + 2 j + 2 k )
3
2x + y = 0 (6.79)
Answer: (D) Æ (t)
Also

4. Let a = 2 i + j k , b = i + 2 j + 3k , c = 3i j + k and
|c | = 2 5 (2 x + y) + ( x + 2 y) + ( x
2 2
y) = 20
2
suppose
9 x + 9 x = 20 [by Eq. (6.79)]
2 2

b c
10 l =
x=± [a b c ]
3
c a
m=
Now x = 10 / 3 and y = 2 10 / 3 imply [a b c ]

c = 10( j + k ) a b
n=
Answer: (B) Æ (s) [a b c ]

(C) Let d = xi + yj + zk . Therefore
Let d be i + j + k . If d = xl + ym + zn, then match
d ×a = 0 x y = 0 (6.80) the items of Column I to those of Column II.
x y z
Column I Column II
[b c d ] = 0 0 1 1 =0

1 0 1 (A) The value of [a b c ] equals (p) 5
x+ y+z=0 (6.81) (B) Value of x is (q) 2
(r) 1
From Eqs. (6.80) and (6.81) we have (C) Value of y equals (s) 1
y=x and z = 2x (6.82) (D) z is equal to (t) 2
Now
Solution: We have
| d | = 30 x2 + y2 + z2 = 30
(b c ) (c a ) (a b )
6 x2 = 30 [by Eq. (6.82)] d = xl + ym + zn = x + y + z
[a b c ] [a b c ] [a b c]
x=± 5
Now

2 1 1

x= 5 d = 5i + 5 j 2 5k = 5 (i + j 2k ) [a b c ] = 1 2 3
Answer: (C) Æ (q) 3 1 1
466 Chapter 6 Multiplication of Vectors

= 2(2 + 3) 1(1 + 9) 1( 1 + 6) Solution:



= 5 (A) Let q be the angle between a and b. Then

Also | a + b |2 = (| a | + | b |)2 2 | a || b | cos q = 2 | a || b |
q =0
[a b c ]
2 = d ×a = x = x
[a b c ] Answer: (A) Æ (p)
(B) We have
x=2

[a b c ] | a + b |2 = | a 2(a ×b) = 2(a ×b)
b |2
1 = d ×b = y = y
[a b c ] a ×b = 0
y=1 Answer: (B) Æ (r)
(C) Let ABCD be a regular tetrahedron. Therefore, all
1 = d ×c = z the faces are equal A

triangles

of equal

edges.

Take

z= 1 as origin and let AB = b, AC = c and AD = d. (AB,
Answer: (A) Æ (p), (B) Æ (q), (C) Æ (r), (D) Æ (s) CD) is one pair of opposite edges and


AB ×CD = b ×(d c )
5.

= b ×d b ×c
Column I Column II
= | b || d | cos 60° | c || d | cos 60°

(A) If a and b are non-zero (p) 0
= 0 (∵ | b | = | c | = | d |)
vectors and | a + b | = | a | + | b |,

then
the angle between a and
(q) Cos 1
1 Therefore angle between AB or CD is p/ 2.
b is ÷
3 Answer: (C) Æ (r)


(B) If a and b non-zero vectors p
(r) (D) Let OA = a, OB = b and OC = c such that | a | = | b | = | c |.

such that | a + b | = | a b |, then 2 Consider the cube with a, b, c as coterminous edges
angle q between them is (student is advised to draw the diagram). We can see
(C) In a regular tetrahedron, the p
(s) that a + b + c and a + b c are a pair of diagonals.
angle between the opposite 3 If q is the angle between them, then
pair of edges is
1
(D) In a cube, the angle between (t) Cos 1 ÷ a ×a + b ×b c ×c 1 1
3 cos q = = =
any two diagonals is | a + b + c || a + b c | 3 3 3
Answer: (D) Æ (q)

Comprehension-Type Questions

non-zero vectors and q is the
1. Passage: If a and b are (ii) In the above

problem, the angle between a b
angle between a and b, then and a + b + c is
3 p
a ×b (A) Cos 1 ÷ (B)
q = Cos
1

| a || b | ÷
7 2
p 6
Answer the following questions. (C) (D) Cos 1 ÷
3 7

(i) If a, b and c be three vectors such that each is
(iii) If a, b, c are mutually perpendicular unit vectors,
perpendicular to sum of the other two and
then the angle between the vectors b c + c a +
| a | = 2, | b | = 3 and | c | = 6. Then the angle between
a b and a + b + c is
a and a + b + c is
1
2 p (A) Cos 1 ÷ (B) Cos 1 (1)
(A) Cos 1 ÷ (B) 7
7 4
p p
1 p (C) (D)
(C) Cos 1
÷ (D) 2 6
7 2
Worked-Out Problems 467

Solution: (iii) We have




(i) By hypothesis
a ×(b + c ) = 0, b ×(c + a ) = 0 and
(b ´ c + c ´ a + a ´ b)×(a + b + c ) = 3[a b c ] = 3
c ×(a + b) = 0. Therefore
Now
a ×b + a ×c = 0

| b ´ c + c ´ a + a ´ b |2 = | b ´ c |2 + | c ´ a |2
b×c + b×a = 0

+ | a ´ b |2 + 2å (b ´ c ) × (c ´ a )
c ×a + c ×b = 0
= 1 + 1 + 1 + 2 å (b ´ c ) × (c ´ a ) (6.83)
Adding all the three equations, we have
By Theorem 6.48
a ×b + b×c + c ×a = 0

and hence b×c b×a
(b ´ c ) × (c ´ a ) =
c ×c c ×a
a ×b = b×c = c ×a = 0
0 0
Therefore = =0
1 0

[a b c ] = | a || b || c | = 2 ´ 3 ´ 6 = 36 From Eq. (6.83)
Now
|b ´ c c ´ a a ´ b| = 3

| a + b + c |2 = | a |2 + | b |2 + | c |2 + 2å (b × c ) Also
= 4 + 9 + 36 + 0 = 49
|a + b + c | = 3

Therefore
angle between b ´ c + c ´ a + a ´ b and

|a + b + c | = 7 a + b + c is

Now, angle between a + b + c and a is æ 3 ö
Cos-1 ç ÷ = Cos-1 (1) = 0
è 3 ´ 3ø
æ
-1 (a + b + c ) × a )
ö æ a ×a ö
Cos ç ÷ = Cos-1 ç
è | a + b + c || a | ø è 7 ´ 2 ø÷
Note: You can take a, b and c as i , j , k .
æ 4 ö Answer: (B)
= Cos-1 ç
è 7 ´ 2 ÷ø
2. Passage: If a, b, c are non-coplanar vectors and r is
æ 2ö any vector, then there exist
unique

set of scalars {x,
= Cos-1 ç ÷
è 7ø y, z} such that r = xa + yb + zc . Also three vectors

a, b, c are non-coplanar if and only if [a b c ] ¹ 0. Let
Answer: (A)
(ii) We have a=i + j -k

b = - i + 2 j + 2k
|a ´ b| = 2 ´ 3 = 6
c = -i + 2 j - k

Now since a, b, c are at right angles,

(a ´ b) ×(a + b + c ) = [a b c ] = | a || b || c | (i) If d = 2 i + 3 j + k and d = xa + yb + zc , then
= 2 ´ 3 ´ 6 = 36 7 11 8
(A) x = - , y = , z = -
9 9 9
Therefore angle between a ´ b and a + b + c is
7 11 8
(B) x = , y = , z = -
æ
-1 (a ´ b) × (a + b + c )
ö æ 36 ö æ 6ö 9 9 9
Cos ç ÷ = Cos-1 ç = Cos-1 ç ÷
è | a ´ b || a + b + c | ø è 6 ´ 7 ÷ø è 7ø 7 11 8
(C) x = , y = , z = -
3 9 9
Answer: (D)
11 7 8
(D) x = , y=- ,z=-
3 3 9
468 Chapter 6 Multiplication of Vectors


(ii) If the same d as in (i) is equal to x(b ´ c ) + y(c ´ a ) + (iii) We have

z(a ´ b), then
æ b´c ö æ c´a ö æ a´b ö
4 6
(A) x = - , y = - , z = -
1 2i + 3 j + k = d = x ç ÷ + y ç ÷ + z ç ÷
9 9 3 è [a b c ] ø è [a b c ] ø è [a b c ] ø
2 6 1 Therefore
(B) x = , y = - , z = -
9 9 9

[a b c ]
2 6 1 4 = d ×a = x = x
(C) x = - , y = , z = - [a b c ]
9 9 9

2 6 1 6 = d ×b = y
(D) x = , y = , z = -
9 9 9
3 = d ×c = z
(iii) If the same
Answer: (C)

æ b´c ö æ c´a ö æ a´b ö
d = x ç ÷ + yç ÷ + zç ÷ 3. Passage: The equation of a straight line passing through
è [a b c ] ø è [a b c ] ø è [a b c ] ø
points with position vectors a and b is r = (1 - t )a +
the
then tb, t Î . It is given that in D ABC , A = 90° and the
vertices B and C lie on the line joining the points
(A) x = -2, y = -6, z = -1
(–1, 3, 2) and (1, 1, 3) and ABC = 30°. If A = (-1, 2, 3)
(B) x = -2, y = 6, z = 1 (see Figure 6.54), then answer the following three
(C) x = 4, y = 6, z = 3 questions.
(D) x = 2, y = 3, z = 1 (i) The vertex B may be
Solution: æ 2 3 - 1 7 - 2 3 7 + 3ö
(A) ç , ,
(i) We have è 3 3 3 ÷ø

2 i + 3 j + k = xa + yb + zc æ 2 - 3 7 - 2 3 7 + 3ö
(B) ç , ,

è 3 3 3 ÷ø
= ( x - y - z)i + ( x + 2 y + 2z) j + (- x + 2 y - z)k
æ 2 3 + 1 7 - 2 3 7 + 3ö
Therefore (C) ç , ,
è 3 3 3 ÷ø
x- y-z=2 (6.84)
x + 2 y + 2z = 3 (6.85) æ 2 3 - 1 7 + 3 7 - 2 3ö
(D) ç , ,
è 3 3 3 ÷ø
-x + 2y - z = 1 (6.86)
(ii) The vertex C may be
Solving Eqs. (6.84), (6.85) and (6.86) we get that
æ - 3 - 2 3 2 3 - 3 21 - 3 ö
(A) ç , ,
9 ÷ø
7 11 8
x= , y= ,z=- è 9 9
3 9 9
Answer: (C) æ 2 3 - 3 21 - 2 3 21 + 3 ö
(B) ç , ,
(ii) We have è 9 9 9 ÷ø

1 1 -1 æ 2 3 - 3 - 3 - 2 3 21 + 3 ö
(C) ç , ,
[a b c ] = - 1 2 2 = - 9 è 9 9 9 ÷ø
-1 2 -1 æ - 3 - 2 3 21 + 3 21 - 3 ö
(D) ç , ,
d = 2 i + 3 j + k = x(b ´ c ) + y(c ´ a ) + z(a ´ b) è 9 9 9 ÷ø



4 = d × a = x[a b c ] = x(- 9) Þ x = -
4

G
(iii) If centroid of the triangle, then GA +
is

the
9 GB + GC equals


2 (A) AB + BC + CA (B) AB ´ AC
6 = d × b = y[b c a] = y(- 9) Þ y = -


3 (C) BC ´ BA (D) CA ´ CB
1
3 = d × c = z[a b c ]z = (- 9) Þ z = -
3
Answer: (A)
Worked-Out Problems 469

Solution: Equation of the line BC C is æ - 2 - 3 2 + 7 3 7 3 - 1ö


and C =ç , ,
r = (1 - t )(- 1, 3, 2) + t(1, 1, 3) è 3 3 3 3 3 3 ÷ø
= (2t - 1, 3 - 2t, 2 + t ) OR

æ -1 - 2 3 7 + 2 3 7 - 3 ö
A (–1,2,3) B=ç , ,
è 3 3 3 ÷ø
90°
æ 2 - 3 - 2 + 7 3 7 3 + 1ö
and C =ç , , ÷
30°
è 3 3 3 3 3 3 ø

Let a, b and c be the position vectors of the vertices A,
FIGURE 6.54 Comprehension-type question 3.
B and C respectively. Then

Therefore a+b+c

G=
3
BA = (- 2t, 2t - 1, 1 - t )

a + b + c 2a - b - c
MN = (2, - 2, 1) GA = a - =
3 3
Now Similarly

3 | - 4t - 4t + 2 + 1 - t |

2b - c - a
= cos 30° = GB =
2 3 4t + 4t 2 + 1 - 4t + 1 + t 2 - 2t
2
3

2c - a - b
| 9t - 3 | and GC =
=
3 9t 2 - 6t + 2 3
Hence
Therefore


GA + GB + GC = 0 = AB + BC + CA
3(9t - 6t + 2) = 4(3t - 1)
2 2
Answer: (i) Æ (A), (ii) Æ (B), (iii) Æ (A)
9t 2 - 6t - 2 = 0
4. Passage: Two non-zero vectors a and b are collinear
6 ± 6 3 1± 3
if and only if b = la, l Î . Non-zero vectors a and b
t= =
18 3 are at right angles if and only if a × b = 0. If a and b are

non-collinear vectors, then every vector r in the plane


Let CA = (- 2t1 , 2t1 - 1, 1 - t1 ) and BA = (- 2t, 2t - 1, 1 - t ).
determined by a and b can be written as r = xa + yb in
Then one and only one way. Answer the following questions.


BA ×CA = 0 Þ 9tt1 - 3t - 3t1 + 2 = 0 (i) If b = xi + yj + zk such that | b | = 10, xyz < 0 and is

collinear with the vector a = 2 2 i - j + 4k then b
Þ 3(3t - 1)t1 = 3t - 2
is
3t - 2 1é 1 ù (A) (-4 2 , 2, 8) (B) (4 2 , - 2, 8)
Þ t1 = = ê1 -
3(3t - 1) 3 ë 3t - 1 úû
(C) (- 4 2 , - 2, - 8) (D) (4 2 , 2, - 8)

(ii) Let a = (2, 3, - 1) and b = (1, - 2, 3 ). c is a vector

1+ 3 3-1
perpendicular
a and
to both b and
satisfies
t= Þ t1 = c × d = -6 where d = 2 i - j + k . Then c is equal to
3 3 3 (A) (-3, 3, 3) (B) (2, 2, -2)
1- 3 3+1 (C) (2, 1, -1) (D) (0, 1, 1)
t= Þ t1 =
3 3 3 (iii) Let a = (- 1, 1, 1) and b = (2, 0, 1). If c is vector

So
coplanar with the vectors a and b, is perpendic-

ular to b and satisfies the condition a × c = 7, then

æ 2 3 - 1 7 - 2 3 7 + 3ö c equals
B=ç , ,
è 3 3 3 ÷ø æ 3 5 ö
(A) ç - , , 3÷ (B) (1, 0, -2)
è 2 2 ø
470 Chapter 6 Multiplication of Vectors

3 Now
(C) ( 3, 0, 6) (D) , 0, 3÷
2
c is perpendicular to b c ×b = 0
Solution:
2x + 4y + x + y = 0
(i) Let
5y x=0 (6.88)
xi + yj + zk = b = la = l(2 2 , 1, 4)
Again
Therefore
a ×c = 7 x 2y + x + x + y = 7
x = 2l 2 , y = l , z = 4l
3x y=7 (6.89)
Now
Solving Eqs. (6.88) and (6.89) we have
| b | = 10 8 l 2 + l 2 + 16 l 2 = 100
5 1
x= ,y=
l = ±2 2 2
So Substituting these values in Eq. (9.87) we get
l=2 x = 4 2 , y = 2, z = 8 5 2 5 5 1 3 5
c= + , , + ÷= , , 3÷
l= 2 x = 4 2 , y = 2, z = 8 2 2 2 2 2 2 2
Answer: (A)
We have

xyz < 0 x = 4 2 , y = 2, z = 8 5. Passage: For any three vectors a, b, c , the following
hold good:
Answer: (B)

(ii) a = (2, 3, 1) and b = (1, 2, 3). Then (a b) c = (a ×c )b (a ×b)c

i j k a (b c ) = (a ×c )b (a ×b)c

a b= 2 3 1 = 7 i 7 j 7k = 7(i j k)
Let a and b be mutually perpendicular unit vectors.
1 2 3
Answer the following three questions.


Since c is perpendicular to both a and b, let vector such that d b = a d, then
(i) If d is any
(d b) b equal


c = l (a b ) = 7 l ( i j k)
(A) a (B) d (C) d (D) 2b

then (ii) If d b = a d, then d equals

c ×d = 6 7 l (i j k ) ×(2 i j + k) = 6 (a b ) a a + (a b )
(A) (B)
2 2
7 l (2 + 1 1) = 6
a (a b )
3 (C) a b (D)
l= 2
7
(iii) If d b = a d which of the following statements
Therefore is false?

c = 7 l(i j k ) = 3(i j k ) = ( 3, 3, 3) (A) Vectors
V a, d and d b are linearly dependent

Answer: (A) (B) [d a d b] 0

(iii) a = ( 1, 1, 1) and b = (2, 0, 1). Let 1
(C) d ×b = 0 and | d | =
2
c = xa + yb
(D) [d a d b] = 0
= ( x + 2 y, x, x + y) (6.87)
Worked-Out Problems 471


Solution: By hypothesis d b = a d and a ×b = 0. Also

Therefore a
(a b)
d=
0 = (d b) ×b = a ×b d ×b = 0 d ×b 2
Again
d ×b = 0

Now [d a d b] = (d a ) ×(d b)

d ×d d ×b
d b=a d (d b ) b=a b d b =
a ×d a ×b
(d ×b)b(b ×b)d = a b d b (6.90)
d ×d 0
d=a b d b = a b (a d ) = =0
a ×d 0
(∵ d b = a d )
Therefore (B) is false.
a (a b)
d= Answer: (i) Æ (b), (ii) Æ (D), (iii) Æ (B)
2

(d b ) b = a b d b

= a b d (a b) [by Eq. (6.90)]

= d

Assertion–Reasoning Type Questions


In the following set of questions, a Statement I is given
a ×a a ×b a
and a corresponding Statement II is given just below it.
Mark the correct answer as: Statement II: [a b c ](a b) = b ×a b ×b b

(A) Both Statements I and II are true and Statement II c ×a c ×b c
is a correct explanation for Statement I
(B) Both Statements I and II are true but Statement II Solution: Let
is not a correct explanation for Statement I
(C) Statement I is true and Statement II is false a = a1i + a2 j + a3k
(D) Statement I is false and Statement II is true
b = b1i + b2 j + b3k

1. Statement I: If a, b, c are non-coplanar, then b c,
c = c1i + c2 j + c3k
c a and a b are also non-coplanar.
Then
Statement II: Vectors a, b, c are coplanar if and only

if [a b c ] = 0. a1 a2 a3 i j k

Solution: According to Theorem 6.34 Statement II is [a b c ](a b) = b1 b2 b3 a1 a2 a3
correct. Now c1 c2 c3 b1 b2 b3

[b c c a a b] = [a b c ]2 0
If A and B are two matrices of 3 3 order, then we know
that det (AB) = (det A) (det B). Therefore
because by Statement I a, b, c are non-coplanar.

Answer: (A) a ×a a ×b a

[a b c ](a b) = b ×a b ×b b
2. Statement I: If a, b, c are coplanar vectors, then

c ×a c ×b c
a ×a a ×b a

b ×a b ×b b = 0 a, b, c are coplanar [a b c ] = 0. So

c ×a c ×b c [a b c ](a b) = 0
Answer: (A)
472 Chapter 6 Multiplication of Vectors

3. Statement I: The shortest distance between the lines Hence


x-3 y-5 z-7 3
= = - £ cos 2 A + cos 2 B + cos 2C
1 -2 1 2
x+1 y+1 z+1 Answer: (A)
and = =
7 -6 1


5. PQ, QR, RS, ST , TU and UP represent the sides of a
is 2 19. regular hexagon (Figure 6.55).


Statement II: The shortest distance
between two Statement I: PQ ´ (RS + ST ) ¹ 0
skew lines r = a + tb and r = c + sd is


Statement II: PQ ´ RS = 0 PQ ´ ST ¹ 0.
|(a - c )×(b ´ d )|

|b ´ d | T S

Solution: According to Theorem 6.44, Statement II is


correct.
U R
In the given lines a = (3, 5, 7), b = (1, - 2, 1), c = (- 1, - 1, - 1)
and d = (7, - 6, 1). Now

a - c = (4, 6, 8)

i j k FIGURE 6.55 Assertion–reasoning type question 5.

and b ´ d = 1 - 2 1 = 4 i + 6 j + 8k
Solution: We have
7 -6 1


PQ ´ (RS + ST ) = PQ ´ RT ¹ 0
Therefore the shortest distance is


because PQ and RT are not parallel.
|(a - c ) × (4 i + 6 j + 8k )| | 16 + 36 + 64 |
= Statement I is true. Now
2 4 + 9 + 16 2 29


PQ ´ RS ¹ 0
58
= = 2 29


29 because they

parallel vectors. Also PQ ´ ST = 0


are not
because PQ and ST are parallel. Hence Statement II is
Hence Statement I is false. false.
Answer: (D) Answer: (C)
4. Statement I: In any D ABC , cos 2A + cos 2B + cos 2C ³
6. Consider the planes 3x - 6y - 2z = 15 and 2x + y - 2z = 5.
3
- .
2 Statement I: Let n1 = (3, - 6, - 2) and n2 = (2, 1, - 2)

bee normals to the planes. Then n1 ´ n2 is parallel to
Statement

II:

is the circum centre of DABC,


If
O the line of intersection of the planes.
then |OA + OB + OC | ³ 0.
Statement II: The vector 14 i + 2 j + 15k is parallel to
Solution: Statement


II

is clearly

true.

It
is known the line of intersection of the planes.
that the

angles (OB , OC ) = 2 A , (OC , OA) = 2 B and


(OA, OB) = 2C . Now Solution: We have


0 £ |OA + OB + OC |2 = |OA|2 + |OB|2 + |OC |2 + 2å (OB ×OC) i j k


n1 ´ n2 = 3 - 6 - 2 = 14 i + 2 j + 15k
= 3R2 + 2R2 (cos 2A + cos 2B + cos 2C)
2 1 -2
where OA = OB = OC = R. Therefore
Answer: (B)
0 £ 3 + 2(cos 2 A + cos 2 B + cos 2C )
Worked-Out Problems 473

Integer Answer Type Questions



1. Let a = (8, - 10, 13). If b = ( x, y, z) is a vector of magni- 3. OA = a, OB = 10a + 2b and OC = b where O, A and

tude 37 and collinear with a and making an acute C are non-collinear points. Let p be the area of the
angle with Z-axis, then the integral part of x + y + z quadrilateral OABCC and q be the area of the parallel-
is . ogram with OA and OC C as adjacent sides. The p = Kq
where K is .
Solution: We have
Solution: See single correct choice type question 57.
b = la Þ x = 8 l , y = - 10 l , z = 13l
Answer: 6
Now
4. P (1, -1, 2), Q

(2,
0,

-1)

and

R

(0,

2,

1) are three
37 = | b |2 = x2 + y2 + z2 = (333)l 2 points, then [QR ´ RP RP ´ PQ PQ ´ QR] is equal
to .
Therefore
Solution: We have
1 1
l =
2
or l = ±


9 3 PQ = i + j - 3k


But it makes acute angle with Z-axis. This implies QR = - 2 i + 2 j + 2k
1


l= RP = i - 3 j + k
3
Now
Therefore
1 1 -3
8 10 13


x= ,y=- and z = [ PQ QR RP ] = - 2 2 2 =0
3 3 3
1 -3 1
So


11 This is also evident from PQ + QR + RP = 0. Now
x+ y+z= = 3.66 …


3
[QR ´ RP RP ´ PQ PQ ´ QR] = [ PQ QR RP ]2 = 0
[ x + y + z] = 3
Answer: 0
Answer: 3


5. In a quadrilateral
ABCD, AB = a, AD = d and AC =
2. Let a = i - j + 3k , b = 3i - 5 j + 6k . Then integer part
ma + nd m and n are positive integers. If the

of the magnitude of the projection of 2 a - b on to the
area of ABCD is 5| a ´ d |, then the possible number
vector a + b is . of pairs (m, n) is .
Solution:
Solution:

a + b = 4 i - 6 j + 9k 1


Area of ABCD = | AC ´ BD|
2
2a - b = - i + 3 j
1
(a + b) ×(2a - b) = - 4 - 18 = - 22 = |(ma + nd ) ´ (d - a )|
2
| a + b | = 133
1
= |(m + n)(a ´ d )|
According to Quick Look 2, the magnitude of the projec-
2
tion of 2a - b on (a + b) is
m+n
= (a ´ d )
|(a + b) ×(2a - b)| 22 2
=
|a + b| 133
= 5 |a ´ d |
whose integer part is 1.
Answer: 1
474 Chapter 6 Multiplication of Vectors

This implies 8. If A1 , A2 ,… Ag are vertices of a regular octagon, then

m+n 7


=5 (OAj OAj +1 ) = K (OA1 OA2 )
2 j =1

Therefore m + n = 10. Hence m takes values from 1 to 9 where the value of K is .


so that n takes values from 9 to 1. Hence number of
pairs = 9. Solution: Each side subtends an angle of 2p / 8 = p / 4
Answer: 9 at the centre of the octagon. Let “O” be the centre of
the octagon and r the radius of the circumcircle of the
octagon. Therefore
6. If (b c ) (c a ) = 3c , then the value of [b cc a


a b] is . p
OA1 OA2 = r2 sin ÷ n
Solution: We have 4


3c = (b c ) (c a ) = [b c a ]c where n is the vector perpendicular to the plane of
0
the polygon such that from the side of n, the points
Therefore A1 , A2 , A3 , …, An are in counterclock sense. Hence

p
[a b c ] = [b c a ] = 3 OA2 OA3 = r 2 sin ÷ n
4

[b cc aa b] = [a b c ]2 = 32

p
Answer: 9 OA3 OA4 = r2 sin ÷ n , … etc.
4
7. If the point of intersection of the lines Therefore
x y 2 z+ 3 7

p
= = (OAj OAj + 1 ) = 7 r 2 sin ÷ n
1 2 3 4
j =1

x 2 y 6 z 3


and = = = 7(OA1 OA2 )
2 2 4
Answer: 7
is (x, y, z), then y + x is .


Solution: Let L1 and L2 be the given lines, respectively. 9. OA = 2 i 2 j , OB = i + j k and OC = 3i k are
P is a point on L1 and Q is a point on L2 . Therefore edges of a parallelopiped. Then the volume of the
parallelopiped in cubic units is .
P = (t, 2 + 2t, 3 + 3t )
Solution:
and Q = (2 + 2 s, 6 + 2 s, 3 + 4 s)
2 2 0


where t, s . Now P = Q implies [OA OB OC ] = 1 1 1
3 0 1
t = 2 + 2s or t 2s = 2 (6.91)
= 2( 1 + 0) + 2( 1 + 3)
2 + 2t = 6 + 2s or 2t 2s = 4 (6.92)
= 2+4
and 3 + 3t = 3 + 4s or 3t 4s = 6 (6.93)
=2
From Eqs. (6.91) and (6.92) t = +2, s = 0 which also satisfy


Eq. (6.93). Therefore the point of intersection is olume of OABC = |[OA OB OC ]| = 2
Vo
(2, 6, 3) = (x, y, z) Answer: 2


So 10. Let OA = a, OB = b and OC = c . Let V1 be
the

y+x=8 volume of the parallelopiped with edges a, b and

c and V2 be the volume of the parallelopiped with
Answer: 8
coterminous edges a + b, b + c and c + a. Then
V2 = KV
V1 where K equals .
Summary 475

Solution: We have D C

V1 = |[a b c ]| d

V2 = |[a + b b + c c + a]| = | 2[a b c ]| = 2V1 L
K
Answer: 2

a , b, c be non-coplanar
11. Let vectors
and A = 2a +
A B
3b c , B = a 2b + 3c , C = 3a +

4 b

D=a
2 c

and b

6b + 6c be four points. Then [ AB AC AD] is equal FIGURE 6.56 Integer type question 12.
to .

1
Solution: We have AK = d
5


AB = a 5b + 4c

AL = (b + d )
6
AC = a + b c

1 1
KL = (b + d ) d
AD = a 9b + 7c 6 5
Therefore using Quick Look 11 [part (1)] we have 1
= (5b d )
30
1 5 4


[ AB AC AD] = 1 1 1 [a b c ] BL = (b + d ) b
6
1 9 7
1
= (d 5b)
= (2 + 30 32)[a b c ] = 0 6
Answer: 0 1
= (5b d )
6
12. Let ABCD be a parallelogram (Figure 6.56) in which 1
AB is parallel to CD and AD is parallel to BC. K is a = ( 5) ÷(5b d )
30
point on the side AD such that AK = (1/ 5)( AD). L is a


point on the diagonal AC C such that AL = (1/ 6)( AC = ( 5)KL

).
Then

the
area of the parallelogram for which LB


and LK are adjacent sides is . Therefore the vectors LK and LB are collinear
vectors.

Hence

the area of the parallelogram for which LK and


Solution: A

Take as origin and let AB = b and AD = d LB are adjacent sides is zero.


so that AC = b + d. By hypothesis
Answer: 0

SUMMARY


6.1 Scalar product (or dot product):
Let a and b two such that a = OA and b = OB. Let Q be the

foot
of

vectors. Then we define a ×b = 0 if either of them is the perpendicular from B onto the support OA of a.

zero vector. If a 0, b 0 and q = (a, b) is the angle Then OQ is called the scalar component (or simply

between a and b, then a ×b = | a || b | cos q . the component or the projection) of

b on a if

(a, b) 90°, and –OQ if (a, b) > 90°. OQ is called
QUICK LOOK the vector componentt (or the orthogonal projection)

of b on a.

(i) a ×b = b ×a. But (a ×b)×c has no meaning.
6.3 Orthogonal
projection: The orthogonal projection
(ii) 0 (a, b) < 90° cos(a, b) > 0 a ×b > 0.
of b on a is
(iii) (a, b) = 90° a ×b = 0 , when a 0 b.
b ×a a a
(iv) (a, b) > 90° a ×b < 0. 2 ÷ a = b × ÷ = (b ×e )e
|a | |a | |a|

6.2 Scalar and vector components: Let a and b be non- where e = a /| a | is the unit vector in the direction of a.
zero vectors and O, A and B be points in the space
476 Chapter 6 Multiplication of Vectors

6.9 Cauchy–Schwartz inequality: For any triads of real


QUICK LOOK
numbers (a1 , a2 , a3 ) and (b1 , b2 , b3 ),

(i) The orthogonal projection of b on l a (l 0) is (a1b1 + a2 b2 +a3b3 )2 (a12 + a22 + a32 )(b12 + b22 + b32 )

same as the orthogonal projection of b on a.
(ii) The magnitude
of the orthogonal projection of and the equality holds if and only if a1 , a2 , a3 are
| b ×a | proportional to b1 , b2 , b3 . In vector language,
b on a is .
|a|
| a ×b |2 | a |2 | b |2

2 2 where the equality holds if and only if a and b are
6.4 Notation: a ×a is denoted by a 2 and note that a = | a | . collinear vectors.

6.5 For any two vectors a, b : 6.10 Vector equation of a plane (Normal form): The equa-
tion of the plane, whose unit normal drawn from the
(i) (a + b)2 = a2 + 2(a ×b) + b2
origin is n and whose distance from the origin is a, is
(ii) (a b)2 = a2 2(a ×b) + b2 given by

For three vectors a, b, c , r ×n = a
In particular, if the plane passes through origin, its
(a + b + c )2 = a2 + b2 + c 2 + 2(a ×b + b ×c + c ×a )
equation is r ×n = 0.

6.6 Magnitude of sum and difference: If a and b are
6.11 Equation
of the plane
in
Cartesian
form: If r = xi +
two vectors, then
yj + zk and n = li + mj + nk , then the equation of
(i) | a + b | | a | + | b | . Equality holds either of a, b the plane is lx + my + n = a.

is a zero vector or a, b are like vectors.

(ii) || a | | b || | a b | | a | + | b | . Equality holds if and 6.12 Cross-product: Let a and b be any vectors. We
define
only if either a, b is 0 or a, b are like vectors.

0 if either of a, b is 0 or a, b are collinear
QUICK LOOK a b=
| a || b | sinq n otherwise

| a ×b | | a || b |. Equality holds either of a, b is 0 or

they are collinear vectors. where q = (a, b) and n is the unit vector perpen-

dicular to both a and b such that (a, b, n) is a

triad of right-handed system. a b is called vector

6.7 Formulae: product or cross-product of a and b.

(i) a = (a ×i )i + (a ×j ) j + (a ×k )k
QUICK LOOK
(ii) If a = a1i + a2 j + a3k and b = b1i + b2 j + b3k then
(i) a b = 0
a ×b = a1b1 + a2 b2 + a3b3 and in particular | a |2 = either of a, b is 0 or a, b are collinear
vectors.
a ×a = a12 + a22 + a32

(ii) | a b | | a || b | equality holds if and only if a, b
QUICK LOOK are orthogonal to each other.

(iii) (a, b, n) is a right-handed system (b, a, n) is a
If a = a1i + a2 j + a3k , then | a | = a12 + a22 + a32 . right-handed system so that

b a = | b || a | sin q ( n) = (a b)
6.8 Expression
for the angle between two vectors:
Let


a and b be non-zero
vectors and a = a1i + a2 j +
a3k , b = b1i + b2 j + b3k and q = (a, b). Then 6.13 About (i, j, k): Let (i , j , k ) be a triad of mutually
perpendicular unit vectors forming right-handed

a ×b a1i + a2 j + a3k system. Then
cosq = =
| a || b | a12 + a22 + a32 b12 + b22 + b32 (i) i i = j j = k k = 0.

(ii) i j = k , j k = i , k i = j .
Summary 477


6.14 Formulae: P1, P2 and P3 are clockwise, then ( n) is the vector

(i) If a = a1i + a2 j + a3k and b = b1i + b2 j + b3k , area of the region D.
then
QUICK LOOK
a b = (a2 b3 a3b2 )i (a1b3 a3b1 ) j + (a1b2 a2 b1 )k

(i) Vector area of the region D = (± n) where n is
which is denoted by unit normal to the plane region D and is its
area.
i j k
(ii) |Vector area| = Area.
a1 a2 a3
b1 b2 b3
6.18 Vector area of a quadrilateral and a parallelogram:
and can be expanded as in the case of usual
(i) a b is the vector area of the parallelogram
determinant expansion.
with a and b as adjacent sides and (1/ 2)(a b )

(ii) If a = a1a + a2 b + a3g and b = b1a + b2 b + b3g is the area of the triangle for which a and b

where a , b and g are non-coplanar vectors, are two of the sides.
then (ii) The vector area of a quadrilateral
ABCD

in
terms of its diagonal vectors is (1/ 2)( AC BD)
b a g a a b which is also valid, if ABCD is a parallelogram.

a b = a1 a2 a3
b1 b2 b3 6.19 (i) The vector area of ABC
C is


6.15 If q is AB AC = CB BA = CA CB
the angle between two non-zero vectors a 2 2 2
and b then

(ii) If a, b and c are the vertices of a triangle, then
|a b| (a1b2 a2 b1 )2 its vector area is
sin q = =
| a || b | a12 b12 1
(b c+c a+a b)
2
where a = a1i + a2 j + a3k and b = b1i + b2 j + b3k
QUICK LOOK
QUICK LOOK
In both 6.18 and 6.19 the corresponding modulus of

(i) | a b |2 = | a |2 | b |2 (a ×b)2 . vector area will give area.

(ii) Unit vector perpendicular to both a and b and
hence perpendicular
to the plane determined by
a and b is 6.20 Formula: If a is any vector, then

2
|a i |2 + | a j | + |a
k |2 = 2 | a |2
a b
±
|a b|
6.21 If a, b, c are three vectors,

then (a b) ×c is called the scalar triple product of

a, b and c and is denoted by [a b c ].
6.16 Important advise: To find the angle between two
vectors, always use dot product because the cross- 6.22 Note:
product gives sin q and sinq > 0 for 0 < q < 180°.
(i) a, b, c are coplanar if and only if [a b c ] = 0.
6.17 Vector area: Let D be a plane region bounded by (ii) Four points
A,

B,

C and D

are
coplanar
closed curve zz. On z2 mark three points P1, P2 and the
vectors AB

, AC and AD are coplanar


[ AB AC AD] = 0.
P3. Let n be unit vector perpendicular to the plane

of the region D such that from the side of n, the
6.23 Geometrical interpretation of [a b c]: Let a, b
points P1, P2 and P3 be in counterclock sense. If

and c be three vectors and V be the volume of the
is the area of the plane region D, then n is called

the vector area of the region D. From the side of n if parallelopiped with a, b, c as coterminus edges.
478 Chapter 6 Multiplication of Vectors


[a, b, c ] = V or –V V according as (a, b, c ) is a right- a ×a a ×b a
handed system or left-handed system. In fact
(b) [a b c ](a b) = b ×a b ×b b
|[a, b, c ]| = V . The volume of the tetrahedron whose

coterminus edges are a, b and c c ×a c ×b c

1  l ×a l ×b l ×c
[a b c ]
6
(ii) [l m n][a b c ] = m ×a m ×b m ×c
6.24 Since the triads
n ×a n ×b n ×c
(a, b, c ), (b, c , a ) and (a, b, c ) are all either right-
handed systems or all left-handed systems, it follows
that QUICK LOOK

[a b c ] = [b c a ] = [c a b] = + V or –V In 6.27 (i) (b), if a, b, c are coplanar, then
Hence
a ×a a ×b a

(a b)×c = [a b c ] = [b c a ] = (b c )×a = a ×(b c) b ×a b ×b b = 0

since dot is commutative. c ×a c ×b c


6.25 Formula for [a, b, c]: In (ii) if either of the triad (l , m, n) or (a, b, c ) is a set

(i) Let a = a1i + a2 j + a3k , b = b1i + b2 j + b3k and of coplanar vectors, then

c = c1i + c2 j + c3k then
l ×a l ×b l ×c
a1 a2 a3
m ×a m ×b m ×c = 0

[a, b, c ] = b1 b2 b3 n ×a n ×b n ×c
c1 c2 c3

(ii) If a = a1l + a2 m + a3 n, b = b1l + b2 m + b3 n and
6.28 Volume
of parallelopiped
and
tetrahedron:
Let

c = c1l + c2 m + c3 n where l , m and n are non-
a = a1 i + a2 j + a3 k , b = b1 i + b2 j + b3 k , c = c1 i + c2 j +
coplanar vectors, then
c3 k . Then

a1 a2 a3 (i) Volume of the parallelopiped with a, b, c as

[a, b, c ] = b1 b2 b3 [l m n] coterminus edges is the absolute value of the
c1 c2 c3 determinant

a1 a2 a3
6.26 If a, b, c are expressed in i , j and
b1 b2 b3
k or any three non-coplanar vectors l , m and n,
c1 c2 c3
the necessary and sufficient condition for a, b, c to
be coplanar is
(ii) The volume of the tetrahedron with a, b and c
a1 a2 a3 as coterminus edges is the absolute value of
b1 b2 b3 = 0
a1 a2 a3
c1 c2 c3 1
b1 b2 b3
6
c1 c2 c3
6.27 Two
important
formulae:
a = a1 i + a2 j + a 3 k ,
Let
b = b1 i + b2 j + b3 k , c = c1 i + c2 j + c3 k, l = l1 i + l2 j +
(iii) Volume of the prism is half of the volume of
l3 k , m = m1 i + m2 j + m3 k and n = n1i + n2 j + n3k the parallelopiped.
then

l ×a l ×b l 6.29 Vector product

of three vectors: For any three

vectors a, b and c
(i) (a) [l m n](a b) = m ×a m ×b m
(i) (a b) c = (a ×c )b (b ×c )a
n ×a n ×b n
(ii) a (b c ) = (a ×c )b (a ×b)c
Summary 479


are called corresponding vectors and the other
6.30 Let a, b and c be vectors such that a and b are
pairs are called non-corresponding vectors.
non-collinear and b is perpendicular to neither a

nor c. Then
6.34 Properties of reciprocal systems:







(a ´ b ) ´ c = a ´ (b ´ c )



(i) a × a

¢
= b × b
¢ = c × c
¢
= 1 and a × b ¢ = a × c ¢ = b × a ¢=


if and only if the vectors a and c are collinear vectors. b × a ¢ = b × c ¢ = c × a ¢ = c × b¢ = 0









(ii) [a b c ][a ¢ b¢ c ¢] = 1
6.31 Vector product and dot product of four vectors:


(iii) If
a
¢¢,
b¢¢, c ¢¢)
is
Let a, b, c and d be four vectors. Then
(

the reciprocal


system of
(a ¢, b¢, c ¢) then a ¢¢ = a, b¢¢ = b and c ¢¢ = c.
(i) (a ´ b) ´ (c ´ d ) = [a c d ]b - [b c d ]a
and also Remark: Without using the name reciprocal systems,
questions have been asked in the past JEE examinations.








(a ´ b) ´ (c ´ d ) = [a b d ]c - [a b c ]d Note: If a, b, c are non-coplanar, then a ¢, b¢, c ¢ (defined

From this we have above) are also non-coplanar




and


hence if r is any
vector, such that r = xa ¢ + yb¢ + zc ¢, then x = r × a, y = r × b

[a b c ]d = [b c d ]a + [c a d ]b + [a b d ]c and z = r × c .

and hence if [a b c ] ¹ 0 (i.e. a, b, c are non 6.35 Vector equation of a plane using a scalar triple
coplanar) then product:

[b c d ] [c a d ] [a b d ] (i) The equation of the plane passing through a

d= a+ b+ c point A( a ) and parallel to two non-collinear
[a b c ] [a b c ] [a b c ]
vectors b and c is
Therefore as per the space representation

[r b c ] = [a b c ]

theorem (Theorem 5.20, Chapter 5), if d = xa +
yb + zc , then (ii) The equation of the plane passing through two

points A(a ), B(b) and parallel to a vector c is
[b c d ] [c a d ] [a b d ]
x= , y= and z =
[a b c ] [a b c ] [a b c ] [r b c ] + [r c a ] = [a b c ]

a ×c a ×d (iii) The equation of the plane
passing three non-
(ii) (a ´ b) ×(c ´ d ) = collinear points A(a ), B(b) and C (c ) is
b×c b×d

[r b c ] + [r c a ] + [r a b ] = [a b c ]
6.32 For any three vectors a, b, c :
(iv) The equation of the plane containing the line
(i) [a + b b + c c + a ] = 2[a b c ]
r = a + tb and perpendicular plane r × c = q is

a ×a a ×b a ×c
[r b c ] = [a b c ]
(ii) [b ´ c c ´ a a ´ b] = [a b c ]2 = b × a b × b b × c
Finally we conclude the summary with the concept
c ×a c ×b c ×c
of skew lines and shortest distance between two
skew lines.
6.33 Reciprocal systems: Let a, b, c be three non-
coplanar vectors. Define
6.36 Skew lines: Any two non-coplanar lines are called

b ´ c

skew lines. That is two lines are said to be skew
a¢ = lines, if there is no plane containing both.
[a b c ]




c´a 6.37 Shortest distance
between two skew lines: Two
b¢ =
[a b c ] lines r = a + tb and r = c + sd where s, t Î are

skew lines if and only if [a - c b d ] ¹ 0. If these lines

a ´ b

c¢ = are skew, then the shortest between them is
[a b c ]






|(a - c )×(b ´ d )|

Then (a ¢, b¢, c ¢) is called reciprocal
system

of |b ´ d |




(a, b, c ). In these systems the pairs a, a¢; b, b¢; c , c ¢
480 Chapter 6 Multiplication of Vectors

EXERCISES
Single Correct Choice Type Questions

1. If a = i + 2 j + k and b = 2 i - j , then the angle between a2 + 2b2 + c2 c2 + b2 - a2
(C) (D)
the vectors a - b and a + b is 2 2
æ 1ö æ 1ö


(A) Cos-1 ç ÷ (B) Cos-1 ç ÷ 9. ABCD is a parallelogram in which AB = p and
è 13 ø è 11ø


p p

= q. DM

AD M is drawn perpendicular to AB. Then
(C) (D) DM is equal to
3 4
( p × q) p ( p × q) p
(A) q - 2 -q
| p| | p|2
°, | a | = 3 and
2. If the angle between a and b is 120
| b | = 4,
then the scalar product of 3a - 2b and a + 2b is (q × p)q (q × p) p
(C) -p (D) q -
(A) 61 (B) 41 (C) -41 (D) -61 | p| | p|


3. Let a = i + j , b = i - j . Suppose x and y are two vectors
10. In D ABC

,
C

= 90

° and

| AB| = c. Then the sum AB


satisfying the two equations 2 x + y = a and x + 2 y = b. × AC + BC × BA + CA ×CB equals

If q is the angle between the two vectors x and y, then
cosqq is equal to (A) 0 (B) 2c2 (C) 3c2 (D) c2
1 11. A(1, y1) and B(x2, 1) are points on the curve y = x -
2
(A) -1 (B) 0 (C) (D) 1
2 44x + 5 in the rectangular Cartesian coordinate system.
If “O”

is the
origin

of the axes, then the scalar product


4. Let a = 6 i - 8 j - (15 / 2)k . If b is collinear
with a, of OA and OB is
making obtuse angle with z-axis and | b | = 50, then b (A) 4 (B) 2 (C) 6 (D) 8
is equal to

(A) - 24 i + 32 j + 30k (B) 24 i + 32 j - 30k 12. In a quadrilateral ABCD, angle A is equal to 120°

and


(C) 24 i - 32 j - 30k (D) 24 i - 32 j - 15k

AC
the diagonal C is the

bisector of the angle A.If | AC | =


(1/ 5) | AB |

= (1 / 3) |

| and q is the angle between the


AD
vectors BA and CD, then cosq q is equal to
5. a, b, c are three non-zero vectors such that no two
1 2 3 4
are collinear. If a + b is collinear with c and b + c is
(A) (B) (C) (D)
collinear with a, then a + b + c is equal to 7 7 17 17

(A) 0 (B) 2(a + c ) (C) 2b (D) 3b
13. If a and b are non-collinear vectors and
6. If A (3, 2, -3), B (5, 1, -1) and C (1, -2, 1) are the vertices
[( x - 2)a + b] ´ [(2 x + 1)a - b] = 0
of a triangle, then the external angle at the vertex A is
æ 2ö æ 4ö then the scalar x is equal to
(A) p - Cos-1 ç ÷ (B) p - Cos-1 ç ÷
è 9ø è 9ø 1 1 1 1
(A) (B) - (C) - (D)
2 2 3 3
æ 4ö æ 2ö
(C) Cos-1 ç ÷ (D) Cos-1 ç ÷
è 9ø è 9ø
14. ABCD is a rhombus having sides

of one unit length.


If the angle BAD is 30°, then | AC | is equal to


7. In a

trapezium

ABCD, BC = l
AD

. If the vector



x = AC + BD is collinear with AD and x = m AD, (A) 2- 3 (B) 2+ 3
m equals
(C) 1+ 3 (D) 3-1
(A) l - 1 (B) 1 - l (C) 2 + l (D) l + 1


8. In

ABCD, it is given that | AB|


=

a parallelogram a , 15. The vectors a and b are non-zero


vectors of equal

= b and | BD| = c, then the scalar product of AC
| AD
|

magnitudes. If the vectors a + 2b and 5a - 4b are



and AD is equal to orthogonal
to each other, then the angle between a
and b is
a2 + 3b2 - c2 3a2 + b2 - c2
(A) (B) p p p 2p
2 2 (A) (B) (C) (D)
6 4 3 3
Exercises 481

16. In a triangle, the vertices are at the points A (3, 2, 24. OAB is a triangle right angled at the vertex O.
3), B (5,, 1, 1) and C (1, 2, 1). Then the angle at the Squares OALM M and OBPQ on the sides OA and
vertex A is OB, respectively, are constructed externally. The
4 2

AP
lines P and

BL meet in H. Then the angle between


(A) Cos 1 ÷ (B) Cos 1 ÷ OH and AB is
9 9
(A) 90° (B) 60° (C) 45° (D) 75°
p 2p
(C)
3
(D)
3
25. a, b and c are unit vectors such that a is perpen-

dicular to both b and c. If the angle

between b and
17. The vector
a + 3b is perpendicular
to the vector c is p /3, then the value of | a + b + c | is

7a 5b and the
vector a 4b is perpendicular to the
5 7
vector 7a 2b. Then the angle between a and b is (A) 2 (B) 3 (C) (D)
2 2
2p p 3p p
(A) (B) (C) (D)
3 3 4 4 26. Let i + 2 j + 3k and 2 i j k be the position vectors
of the points

A and B, respectively.
Then the projec-

18. a and b are two vectors which include angle 120°. tion of AB on the vector i + j + k is

If a + lb is orthogonal to the vectors a b and 7
(A) 2(i + j + k ) (B) (i + j + k )
| a | = (1/ 2) | b |, then the value of l is 3
(A)
4
(B)
3
(C)
2
(D)
1 7
5 5 5 5 (C) (i + j + k ) (D) 2(i + j + k )
3

19. If a + b + c = 0 , | a | = 3, | b | = 1 and | c | = 4, then the


27. AB = 3i 2 j + 2k and BC = i 2k represent the
value of a ×b + b ×c + c ×a is equal to adjacent sides of a parallelogram. Then the acute
(A) 12 (B) 12 (C) 13 (D) 13 angle between the diagonals is
p 3
20. The angle between the vectors (a ×c )b (a ×b)c and (A) (B) Cos 1
6 10
a is
p p p 5p 3 2
(A) (B) (C) (D) (C) Cos 1 (D) Cos 1
4 2 3 12 5 5
21. ABCD is a quadrilateral and R is the radius

of 28. Let be a plane and P a point not in . Q is the foot


the

circle circumscribing the quadrilateral. If | AB|2 +


of the perpendicular drawn from P onto . QM M is
|CD| = 4 R2 , then the angle between the diagonals AC
2
drawn perpendicular to a line L in . Then the angle
and BD is
between the lines PM
M and L is
p 5p 2p p p p
(A) (B) (C) (D) (A) (B)
2 12 3 3 2 3
22. In ABC, a line is drawn parallel to the side AB 3 2
(C) Cos 1 ÷ (D) Cos 1
meeting ACC in E and BE F. If AF = BE, then
E in F 5 3 ÷
ABCC is
(This result is called “Theorem of three perpendicu-
(A) right angled lars” in solid geometry.)
(B) right-angled isosceles

29. Let a = i + j + k , b = 2 i + 4 j 5k , c = xi + 2 j + 3k .
(C) isosceles
scalar
If the product of a with a unit vector parallel
(D) equilateral
to b + c is unity, then the value of x is equal to
23. The area of ABC C is 6 square units. K is a point on (A) 5 1 (B)
(C) 3 (D) 1
the side AB is such that AK : KB = 2 : 3. L is a point
on the side AC C is such that AL : LC = 5 : 3. P is the 30. Let a = 3i + j 2k and b = i 3 j + 4 k . Then the

point of intersection of the lines BL and CK K inside area of the parallelogram with a and b as diagonal
the triangle. If the distance of P from the side of AB vectors is
is 3/2 units, then the length of the side AB is equal to (A) 3 5 (B) 3 (C) 5 3 (D) 5
(A) 6 (B) 5 (C) 4 (D) 3
482 Chapter 6 Multiplication of Vectors

31. Let A (2, 3, 5), B ( 1, 3, 2) and C (


x,

5, y) be the (A) XY-plane


Y
vertices of ABC. If the median AD is equally (B) ZX-plane
X
inclined to the coordinate axes, then
(C) YZ-plane
(A) x = 10, y = 7 (B) x = 7, y = 10
(D) along the bisector of the angle between k
(C) x = 7, y = 10 (D) x = 10, y = 7
and a.

32. Let a = 2 i 3 j + 4k , b = i + 2 j 2k , c = 3i j + k and
38. a, b, c are non-coplanar vectors and
x = a + b + c , y = a b + c and z = a + b c . Then

the volume of the parallelopiped with x, y and z as p = x(b c ) + y(c a ) + z(a b)
coterminus edges is
(A) 24 (B) 6 (C) 28 (D) 7 y, z are
where x, scalars. If p is perpendicular to the
vector a + b + c , then
(Hint: Use part (2) of Quick Look 11.)
(A) xy + yz + zx > 0 (B) x3 + y3 + z3 = 3xyz

33. Let a = l ( i + 2 j 3k ), b = ( 2 l + 1) i + ( 2 l + 3) j +
(C) x = y = z (D) x2 + y2 + z2 = xyz

(l + 1)k , c = (3l + 5)i b + c + (l + 5) j + (l + 2)k .

Then a, b, c are 39. If a and b are orthogonal vectors, then [{(a b)

(A) coplanar for all real values of l a} a ] a is equal to

(B) non-coplanar for all real values of l (A) | a |4 b (B) | a |4 b

(C) non-coplanar for all non-zero real values of l (C) | a |2 | b |2 a (D) | b |4 a
(D) coplanar for all non-zero real values of l
40. If a and b are non-collinear unit vectors, then

34. b and c are non-collinear vectors. a is a vector such |(a b) (a + b)| is equal to

that a ×b + a ×c = 4 and
(A) 2 1 (a ×b)2 (B) 2 (a ×b)2

a (b c ) = ( p2 2 p + 6)b + (sin q)c
(C) 4 (a ×b)2 (D) 4 2(a ×b)2
where 0 < q < p
p. Then the point (p, q) is
p p 41. Let b, c , d be three non-coplanar vectors. Then for
f
(A) (1, p)
p (B) , 1÷ (C) (1, 1) (D) 1, ÷
any
vector
a 0,, the vector (a b) (c d ) + (a c)
2 2
(d b) + (a d ) (b c ) is equal to

35. Let L be
the line of intersection
of the planes (A) 2[b c d ]a (B) 2[c b d ]a

r ×(3i + 3 j + 2k ) = 0 and r ×(i + 2 j + 3k ) = 0, then (C) 0 (D) [a b d ]c
the angle between the line L and the unit vector
along the positive X X-axis is 42. If the volume
of tetrahedron
having
coterminus
1 5 1 8 edges 12 i + lk , 3 j k and 2 i + j 15k is 91, then
(A) Cos ÷ (B) Cos ÷
83 83 one of the values of l is
(A) 6 (B) 5 (C) 4 (D) 3
7 7
(C) p Cos 1 ÷ (D) Cos 1 ÷
83 83 43. Let a, b and c be non-coplanar vectors and


36. Let a = i + j k and b = i + 3k . If c is a unit vector
b c




c a
a b
a = ,b = ,c =

such that [a b c ] is maximum, then c can be [a b c ] [a b c ] [a b c ]

(A)
1
( 3i + 4 j + k ) (B)
1
( 3i 4 j k )









26 26 Then [a b c ]{a b + b c +c a } equals



1 1 (A) 0 (B) a + b + c
(C) (4 i + 3 j k ) (D) ( i 3 j + 4k )
26 26 (C) [a b c ](a + b + c ) (D) 3(a + b + c )

37. Let p = p1i + p2 j + p3k where p1, p2 and p3 are rational 44. On the sides AB and BC
C of ABC, points P and Q

numbers and | p| is also rational.
If p makes
an angle are taken, respectively, such that AP : PB = 2 : 3 and

p/4 with the vector a = 2 i + 3 2 j + 4k , then p lies in CQ : QB = 3 : 2. The lines AQ and CPP intersect in R.
Exercises 483


If the area of BRC
C is one square unit, then the area 45. Let a = i + j and b = j + k . r is a vector such that

of ABC C is r a = b a and r b=a b. Then r is equal to

9 7 17 19 (A) i + j 2k (B) i j + 2k
(A) (B) (C) (D)
19 4 4 9
(C) i j + k (D) i + 2 j + k

Multiple Correct Choice Type Questions



1. Let A ( 1, 3, 7), B (2, 1, 0) and C (0, 1, 5) be three 6. Let a, b, c and d be four vectors. Then
points. Then,

(A) [(a b) (a c )]×d = [a b c ](d ×a )


(A) | AB| = 74

(B) [(a b) (b c )]×d = [a b c ](b ×d )


(B) (2 AB CB)×(2 BC + BA) = 19

(C) [(a c ) (b d )]×d = [a c d ](b ×d ) + [a b c ](c ×d )


(C) | BC | = 33

(D) [(a d ) (c d )]×d = [a c d ](d ×d )


(D) ( AB CB)×( BC + AB) = 19
7. Consider the four points A (0, 1, 2), B (3, 0, 1),

2. If the vectors a = (3, x, 1) and b = (2, 1, y) are orthog- C (4, 3, 6) and D (2, 3, 2). Then,

onal vectors and | a | = | b |, then (A) area of ABC C is 3 10
31 41 (B) the altitude drawn from A on to the side BC
C has
(A) x = (B) y =
12 12 magnitude 6 / 7 14
31 41 (C) volume of the tetrahedron ABCD is 6
(C) x = (D) y =
12 12 (D) the shortest distance between the lines AB and
CD is 6

|a | = 2 2, |b| = 3
3. Let a, b be two vectors such that

and the angle between a and b is p /4. Then, the 8. Consider the line L whose equation is

lengths of the diagonals constructed with p = 5a + 2b
x 1 y + 1 z + 10
and q = a 3b as adjacent sides of a parallelogram and = =
its area are 2 3 3
(A) 15 (B) 693 Which of the following are correct?
(C) 593 (D) 102 (A) The direction cosines of L are (2, 3, 8).
(B) The foot of the perpendicular drawn from the


4. Consider the parallelopiped with OA = i + j + k, point P (1, 0, 0) onto the line is (3, 4, 2).


OB = 2 i + 4 j k and OC = i + j + 3k as edges. Then (C) The image of the point P (1, 0, 0) in the line L

is (5, 8, 4).
(A) [OA OB OC ] = 4

(D) The distance of the point P (1, 0, 0) from the


(B) [OA OB OC ] = 4 line L is 2 6.
(C) the length of the altitude CM M drawn from the
corner

C
onto

the base of parallelogram having 9. Let a, b, c be non-zero real numbers such that a +
OA and OB as adjacent sides is 4 / 38 b = 0 and c = 2b. Then,

2 (A) the vectors p = ai + bj + ck and r = i + j + 2k
(D) CM =
3 are collinear vectors

(B) the number of vectors p = ai + bj + ck such

a = i + j + k , b = i
5. Let j + k , c = i + j k and d = i + that | p| is equal to 6 is infinite
j + k . Then
(C) the magnitude of p = ai + bj + ck is 2 6 if
(A) (a b) (c d) = 8 j 2a + 3b + 6c = 26

(B) (a c ) (b d ) = 8k
(D) the vectors p = ai + bj + ck and q = 4 i + 2 j + k

are perpendicular to each other
(C) (a b) ×(c d) = 0

(D) (a c )×(b d) = 0
484 Chapter 6 Multiplication of Vectors

10. Let A, B and C be the points (1, 1, 1), (1, 1, 1) and (C) foot of the perpendicular drawn from the point
( 1, 3, 5), respectively. Then, 3 19
(1, 1, 2) is , 1, ÷
(A) equation of the plane ABC C is 3x z 2 = 0 10 10

(B) the vector 3i k is normal to the plane ABC (D) image of (1, 1, 2) in the plane ABC
C is (1, 1, 3)

Matrix-Match Type Questions


In each of the following questions, statements are given 2. Match the items of Column I with those of Column II.
in two columns, which have to be matched. The state-
ments in Column I are labeled as (A), (B), (C) and
Column I Column II
(D), while those in Column III are labeled as (p), (q),
(r), (s) and (t). Any given statement in Column I can p
(A) If a + b + c = 0, | a | = 3, | b | = 5 and (p)
have correct matching with one or more statements in 2
| c | = 7 then the angle between a
Column II. The appropriate bubbles corresponding to and b is
the answers to these questions have to be darkened as p
illustrated in the following example. (B) If a and b are unit vectors such (q)
4
that | a b | is also unit vector,
Example: If the correct matches are (A) (p), (s);
then
the angle between a and
(B) (q), (s),(t); (C) (r); (D) (r), (t); that is if the b is
matches are (A) (p) and (s); (B) (q), (s) and (t); p
(C) In a regular tetrahedron (i.e., (r)
(C) (r); and (D) (r), (t), then the correct darkening 6
all edges are equal) the angle
of bubbles will look as follows:
between opposite pair of edges is
p
(D) If a and b are two non-collinear (s)
p q r s t 3
vectors such that | a + b | = | a b |,
A
then
the angle between a and
B
b is
C
D
3. Match the items of Column I with those of Column II.

1. Match the items of Column I with those of Column II.


Column I Column II

Column I Column II (A) The line (p) 2


x 1 y 2 z+ 3
(A) In the rectangular Cartesian (p) 1 = =
coordinate plane A and B 2 3 4
are points
on curve xy = 6

the x + 4y z 1 = 0
such

that OA ×i = 2 and in the point (a, b, c), then the value


OB ×i = 3 where O is the origin (q) 1 of a + b + c is
of
the

coordinate

axes. Then
(B) The square of the reciprocal of the (q) 3
| 2OA + 3OB| is equal to
distance of the point (1, 2, 3) from
(B) a = (2, 3, 5), b = (3, 0, 1) and the plane x y + z 5 = 0 is
(r) 5
c = (4, 3, 2). If d = 3a + b c , (C) The distance of the line (r) 7
1
then | d | is x 1 y+2 z 1
17 = =
(C) A ( 3, 2, 0), B (3, 3, 1) and C 3 2 2
(s) 3
(5, 0, 2) are three consecutive from the plane 2x + 2y
2 z = 0 is
vertices of a parallelogram. Then (D) Let V be the volume of a (s) 1
the sum of the coordinates of its tetrahedron s and V be the
fourth vertex is volume of the tetrahedron formed
(t) 3

(D) If a = ( x, 2, 1) and b = ( x, x, 3) by the centroids of the faces of s,
s
are orthogonal vectors, then then V/9
V V is equal to
possible values of x are
Exercises 485


4. Let a, b, c be three vectors such that [a b c ] = 2. Match
the items of Column I with those of Column II.

Column I Column II

(A) The value of [a + b b + c c + a ] is (p) 2

(B) The value of [b ´ c c ´ a a ´ b] (q) 6
equals (r) 4

(C) [(a - b) ´ (a + b)]× c is equal to (s) 8

(D) a × a a × b a × c (t) 12

b×a b×b b×c

c ×a c ×b c ×c

Comprehension-Type Questions

1. Passage: Let a = i - j , b = j - k and c = k - i . Answer 3. Passage:
If a, b, c are three non-coplanar vectors and

the following three questions. d = xa + yb + zc , then

(i) If a × d = 0 = [b c d ] and | d | = 1, then d equals [b c d ] [c a d ] [a b d ]
1 1 x= , y= , z=
(A) ± (i + j - 2k ) (B) ± (i + j - k ) [a b c ] [a b c ] [a b c ]
6 3

1 let a = 2 i - j + k , b = i + 2 j - k and c = i + j -
Now
(C) ± (i + j + k ) (D) ± k 2k. Answer the following three questions.
3
(i) If i + j + k = xa + yb + zc , then
(ii) (a ´ b) ´ (a ´ c ) is equal to
3 9 7
(A) i + j + k (B) - 2 i + 2 j (A) x = , y = , z = -
8 8 8
(C) 2 i + k (D) 0 3 9 7
(B) x = - , y = , z = -
(iii) [b ´ c c ´ a a ´ b] is equal to 8 8 8
(A) 0 (B) 1 (C) 2 (D) 4 3 9 7
(C) x = , y = , z =
8 8 8
2. Passage: Let a = i + 2 j + k , b = i - j + k , c = i + j - k . 3 9 7
Answer the following questions. (D) x = - , y = - , z =
8 8 8
(i) A vector coplanar with a and b whose projec-
(ii) If i - j + k = x(b ´ c ) + y(c ´ a ) + z(a ´ b), then
tion on c is 1/ 3 is
1 1 1
(A) 4 i - j + 4k (B) 3i + j - 3k (A) x = - , y = - , z =
2 4 4
(C) 2 i + j - 2k (D) 2 i + j + 2k 1 1 2
(B) x = , y = , z =
(ii) a × a a × b a 2 4 8

b×a b×b b 1 1
(C) x = - , y = , z =
1
2 4 4
c ×a c ×b c
1 1 2
(D) x = - , y = - , z =
(A) 9(i - k ) (B) 9(i - j ) 8 8 8

(C) 18(i - k ) (D) 18(2 j - k ) (b ´ c ) (c ´ a ) (a ´ b )
(iii) i + j - k = x + y + z , then
(Hint: Use part (2) Quick Look 11.) [a b c ] [a b c ] [a b c ]

(iii) The area of the parallelogram for which a + b (A) x = 0, y = 2, z = -2 (B) x = 0, y = 4, z = 4

and b + c are adjacent sides is
(C) x = 0, y = -4, z = -2 (D) x = 0, y = -4, z = 4
(A) 2 2 (B) 2 (C) 3 2 (D) 4 2
486 Chapter 6 Multiplication of Vectors


4. Passage: Let a and b be two non-zero non-collinear
Then which of the following triads are orthogonal?

vectors. Then the projection vector of b on a is
(A) (a, b1 , c3 )
b ×a
2 ÷a vector component of b perpendic-
|a| (B) (a, b1 , c2 )

b ×a (C) (a, b1 , c1 )
ular to the direction of a is b a. Answer the
| a |2 ÷
(D) (a, b2 , c2 )
following three questions.
(ii) Let a = i + 2 j + k , b = i j + k and c = i + j k .

(i) Let a, b, c be non-zero and non-coplanar vectors Then the projection vector of c on a b is
and
(A) 2(i k )

b ×a (B) 3 2 (i + 2k )
b1 = b a
| a |2 ÷
2 2
(C) (i k)
b ×a 3
b2 = b + 2 ÷ a
|a | (D) i k

of a perpendicular
(iii) The component

to the direc-
c ×a b ×c tion of (b c ) where a, b and c are same as in
c1 = c a + 2 ÷ b1
| a |2 ÷ |c | (ii) is

(A) 2 i + j k
c ×a b ×c
c2 = a b1 + c
| a |2 ÷ | b1 |2 ÷ 1
(B) (2 i + j k )
2

c ×a b ×c
c3 = c a + 2 ÷ b1 (C) j k
| c |2 ÷ |c |
1
(D) ( j k )
c ×a b ×c 2
c4 = c a = 2 ÷ b1
| c |2 ÷ |b|

Assertion–Reasoning Type Questions


In the following set of questions, a Statement I is given 2. Statement I: A, B, C and D are

four

points

such


and a corresponding Statement II is given just below it. that for any point P in the space AP ×CP BP ×DP.
Mark the correct answer as: Then ABCD is a parallelogram which cannot be a
(A) Both Statements I and II are true and Statement II rectangle.
is a correct explanation for Statement I Statement II: If the points A, B, C and D

are

the

(B) Both Statements I and II are true but Statement II vertices of a rectangle, then for any point Q, QA ×QC =

2
is not a correct explanation for Statement I QB ×QD and |QA| + |QC | = |QB| + |QD| .
(C) Statement I is true and Statement II is false
(Hint: See multiple correct choice type question 7 in
(D) Statement I is false and Statement II is true
Worked-Out Problems.)

1. Statement
a = a1i + a2 j + a3k , b = b1i + b2 j + b3k ,
I: Let 3. Statement I: The angle at the vertex A of ABCC is
c = c1i + c2 j + c3k where ar , br , cr (r = 1, 2, 3) are non- p /6, H is the orthocentre and M is the mid-point of
3
negative real numbers and (ar + br + cr ) = L. If V is BC. If T is a point

on

HM
the line M produced such that
r =1
HM = MT, T then | AT | = 2 | BC |.
the volume of the parallelopiped with a, b and c as
coterminus edges, then V L .3 Statement II: In ABC, if

O is the

circumcentre

and
H is the orthocentre, then OH = OA + OB + OC .
Statement II: The Arithmetic mean of non-nega-
tive real numbers is greater than or equal to their 4. Statement I: T
The sum of the squares of the sides
Geometric mean. of any quadrilateral equals the sum of the squares
Exercises 487

of its diagonals together with four times the square coplanar and the angle of incidence is equal to the
of the distance between the middle points of the angle of reflection. See Figure 6.57.
diagonals.

Statement II: If a and b are two vectors, then | a b |2 =

| a |2 2(a ×b) + | b |2.
a
5. Statement I: If the incident ray on a surface is along
v w
unit vector v, the reflected ray along the unit vector w

and the normal is along the unit ray a outward, then

w = v 2(a ×v)a.
Statement II: T
The incident vector, the reflected vector FIGURE 6.57 Assertion–reasoning type question 5.
and the normal vector at the point of incidence are

Integer Answer Type Questions



The answer to each of the questions in this section is a 3. If e1 , e2 , e3 are mutually perpendicular unit vectors
non-negative integer. The appropriate bubbles below the forming a right-handed system, then the value of

respective question numbers have to be darkened. For [e1 + e2 + e3 e1 + e2 e2 + e3 ] is .
example, as shown in the figure, if the correct answer to
the question number Y is 246, then the bubbles under Y
4. If | a | = 2 and a ×b = 0, then a [a {a (a b)}] is
labeled as 2, 4, 6 are to be darkened. nb where n is .

X Y Z W
a, b, c be three non-coplanar
5. Let vectors and p = a

0 0 0 0 2b + 3c , q = 2a + 3b 4c and r = a 3b + 5c . Then

1 1 1 1 [ p q r ] equals .
2 2 2
6. A (1, 1, 1), B (1, 1, 1) and C (0, 0, 1) are the vertices
3 3 3 3
of a triangle. Then the area of ABC
C in square units
4 4 4 is .
5 5 5 5
6 6 6 7. M and N are the mid-points of the sides AC
C and AB,
respectively, of ABC. Through the vertex C, a line
7 7 7 7
is drawn parallel to the side AB meeting the line BM
8 8 8 8
in P. Then the area of ABC C is equal to l times the
9 9 9 9 area of PMN N where l equals .

8. D, E and F are points on the sides BC, CA and AB,


1. The volume of a parallelopiped with a, b, c as
coterminus edges is 3. Then the volume of the paral- respectively, such that BD : DC = CE : EA = AF : FB =
2 :1. Then m times the area of DEF F is equal to
lelopiped with a + b, b + c and c + a as coterminus
edges is . n times the area of ABC C where m/n is equal
to .
2. The volume
of a parallelopiped with coterminus
9. If V is the volume of the tetrahedron whose vertices
edges a, b, c is 3. Then the value of the determinant
are (2, 1, 3), (4, 1, 3), (3, 2, 1) and (1, 4, 2), then

a ×a a ×b a ×c the integer part of V is .

b ×a b ×b b ×c
10. If the vectors a = i + j + k , b = 4 i + 3 j + 4k , c = i +
c ×a c ×b c ×c
aj + bk are coplanar and | c | = 3 , then a 2 + b is
.
.
488 Chapter 6 Multiplication of Vectors

ANSWERS
Single Correct Choice Type Questions
1. (B) 24. (A)
2. (D) 25. (A)
3. (A) 26. (D)
4. (C) 27. (B)
5. (A) 28. (A)
6. (B) 29. (D)
7. (D) 30. (C)
8. (A) 31. (C)
9. (B) 32. (C)
10. (D) 33. (C)
11. (A) 34. (D)
12. (B) 35. (A)
13. (D) 36. (B)
14. (B) 37. (A)
15. (C) 38. (B)
16. (A) 39. (A)
17. (B) 40. (A)
18. (C) 41. (B)
19. (D) 42. (D)
20. (B) 43. (B)
21. (A) 44. (D)
22. (C) 45. (D)
23. (C)

Multiple Correct Choice Type Questions


1. (A), (B), (C) 6. (A), (B), (D)
2. (C), (D) 7. (A), (C), (D)
3. (A), (C), (D) 8. (B), (C), (D)
4. (B), (C) 9. (A), (D)
5. (A), (B), (C), (D) 10. (A), (B), (C)

Matrix-Match Type Questions


1. (A) (r), (B) (r), (C) (p), (D) (q), (t) 3. (A) (q), (B) (q), (C) (s), (D) (q)
2. (A) (s), (B) (s), (C) (p), (D) (p) 4. (A) (r), (B) (r), (C) (r), (D) (r)

Comprehension-Type Questions
1. (i) (A); (ii) (D); (iii) (A) 3. (i) (A); (ii) (C); (iii) (B)
2. (i) (D); (ii) (C); (iii) (A) 4. (i) (B); (ii) (D); (iii) (B)

Assertion–Reasoning Type Questions


1. (A) 4. (A)
2. (A) 5. (A)
3. (A)

Integer Answer Type Questions


1. 6 6. 1
2. 9 7. 4
3. 1 8. 3
4. 4 9. 7
5. 0 10. 2
Probability
7
Contents
7.1 Random Experiments
and Events
7.2 Classical Definition of
Probability
7.3 Axiomatic Approach
to Probability
7.4 Independent and
Dependent Events
7.5 Random Variables
and Probability
Distributions
7.6 Theoretical Discrete
Distributions

Impossible Unlikely Even Chance Likely Certain Worked-Out Problems


Summary
Exercises
Answers

1-in- 6 Chance 4-in- 5 Chance


Sometimes ‘Human being’
vices lead to great inven-
Probability

tions. One such, gambling


disease of some 17th century
French people, is the main
root for the origin of
Probability. In modern day
science, statistics and prob-
ability are treated as almost
exact sciences. Probability is
vastly applicable in pharma-
ceutics, veterinary science
and defence fields.
490 Chapter 7 Probability

Probability is a measure of uncertainty and deals with the phenomenon of chance or randomness. We quite often
make unknowingly some decisions about probability. For example, when one leaves his house in the morning on a
cloudy day he may decide to take an umbrella with him, even if it is not raining, because he thinks that it may rain
later in the day.
The theory of probability is developed to explain such kind of decisions mathematically. Infact, the theory of
probability has its origins in gambling and games of chance. It is known that a French gambler Chevalier de Mere
approached a French mathematician Blaise Pascal for a solution of a problem concerning gambling. Pascal gave a
solution and subsequently he corresponded with another French mathematician Pierre de Farnat and established
the foundations of the theory of probability. Later Laplace, a French scientist, made first attempt towards giving
some mathematical rigour to the subject of probability. The present day theory of probability is credited to Andrei
Nikolaevich Kolmogorov, a 20th century Russian probabilist, who laid the set theoretic foundations of probability.

7.1 | Random Experiments and Events


In this section we define certain important elementary technical terms related to probability and discuss some exam-
ples concerning these. Let us begin with the following.

DEFINITION 7.1 Random Experiment A random experiment is an experiment in which


1. The experiment can be repeated any number of times under identical conditions.
2. All possible outcomes of the experiment are known in advance.
3. The actual outcome in a particular experiment is not known in advance.

Examples

(1) Rolling of an Unbiased Die This experiment can be performed any number of times
Rolling of an unbiased die is a random experiment in under identical conditions.
which all the possible outcomes are 1, 2, 3, 4, 5 and 6, if (2) Tossing of an Unbiased Coin
we denote the six faces of the die with the numbers 1, 2, Tossing of an unbiased coin is a random experiment in
T
3, 4, 5 and 6. The faces of the die may also contain dots which there are only two possible outcomes, namely, Head
in numbers 1, 2, 3, 4, 5 and 6. In any case, we identify (H) and Tail (T). In a particular experiment (tossing of the
the faces of a die with the numbers 1, 2, 3, 4, 5 and 6. coin), the outcome is not known in advance.This experiment
The actual outcome in a particular experiment (rolling can also be performed any number of times under iden-
of the die) is the number that appears on the upper- tical conditions and therefore this is a random experiment.
most face of the die and this is not known in advance.

“Tossing of an unbiased coin till tail appears” is also a random experiment. However, experiments such as
“measuring the acceleration due to gravity using a compound pendulum” is not a random experiment, since the
experiment cannot be repeated under identical conditions and the possible outcomes are not known in advance.
Throughout this chapter, by a coin or die we always mean an unbiased coin (or a fair coin) or unbiased die (or a
fair die) unless otherwise mentioned.

DEFINITION 7.2 Some of the important definitions are as follows:


1. A set of possible outcomes of an experiment is called an event.
2. Two or more events are said to be exhaustive if the performance of the experiment always
results in the occurance of atleast one of them.
3. Two or more events are said to be mutually exclusive if the occurance of one of the events
prevents the occurance of any of the remaining events.
4. Two or more events are said to be equally likely (or equiprobable) if there is no reason to
expect one of them in preference to others.
7.2 Classical Definition of Probability 491

Example

In rolling of a die, consider the events: and E2: occurrence of an odd number.
E1: occurrence of an even number (that is, an T
Then E1 and E2 are exhaustive, mutually exclusive and
even number appears on the upper most equiprobable.
face)

Example

If a coin is tossed, “the occurrence of Head” and “the occurrence of T


Tail” are exhaustive, mutually exclusive and
equiprobable.

Example 7.1

Consider the experiment of “throwing a pair of dice and


finding the sum of the two numbers that show up (on the Die-1
upper most faces).” The possible outcomes (events) are
+ 1 2 3 4 5 6
numbers 2 to 12 (i.e., a + b where 1 £ a £ 6 and 1 £ b £ 6).
These events are exhaustive and mutually exclusive, but 1 2 3 4 5 6 7
not equiprobable. Why? 2 3 4 5 6 7 8
Die-2

3 4 5 6 7 8 9
Solution: These sums are not equally likely, as we can 4 5 6 7 8 9 10
observe from the following table. 5 6 7 8 9 10 11
The sum 12 occurs only once, the sum 10 appears 6 7 8 9 10 11 12
thrice and hence “the sum 12 occurs” and “the sum 10
occurs” are not equiprobable (or equally likely).

Example 7.2

In rolling of a die, consider the events Then E1, E2 and E3 are exhaustive, but not mutually
E1: occurrence of an even number exclusive. Why?
E2: occurrence of a prime number
Solution: This is because 2 occurs in both E1 and E2 as
and E3: occurrence of 1 it is both even and prime.

7.2 | Classical Definition of Probability


In the following we give a classical definition of probability and discuss certain examples.

DEFINITION 7.3 Suppose that in a random experiment there are n exhaustive, mutually exclusive and equi-
probable possible outcomes. If m of them are favourable to the happening of an event E,
then the probability of occurance of E (or simply the probability of E) is defined as m/n and is
denoted by P(E). That is,
m
P(E ) =
n
Clearly 0 £ P(E) £ 1.
492 Chapter 7 Probability

Note that since the number of outcomes not favourable to an event E is n - m, the probability of non-occurance of
the event E is (n - m)/n and is denoted by P(Ec). That is,
n-m m
P (Ec ) = = 1 - = P(E )
n n

P (E ) + P (Ec ) = 1

Example 7.3

Suppose that an integer is picked from among 1 to 20 2, 3, 5, 7, 11, 13, 17 and 19 and these are 8 in number.
(both inclusive). What is the probability of picking a Therefore, 8 are favourable to the event of picking a
prime? prime and hence the probability of picking a prime is
8 2
Solution: There are 20 outcomes of the experiment =
of picking an integer. The primes between 1 and 20 are 20 5

Example 7.4

Suppose that 8 dice are thrown. Find the probability that An outcome that none of the dice shows 3 can be
none of the dice shows 3 (on the upper most face). expressed as 8-tuple of integers from the 5-element set
{1, 2, 4, 5, 6} and there are 58 such outcomes. Thus, the
Solution: Any outcome of “throwing 8 dice” can be probability that none of the dice shows 3 is
expressed as an 8-tuple of integers from 1 to 6 and hence 8
the total number of possible outcomes is 68. 58 æ 5 ö
=ç ÷
68 è 6 ø

Example 7.5

Suppose that a bag contains 6 red, 5 black and 4 blue Drawing one blue, one black and one red ball simulta-
balls. Find the probability that three balls drawn simul- neously can be expressed as a triple (a1, a2, a3), where
taneously are one blue, one black and one red. 1 £ a1 £ 4, 1 £ a2 £ 5 and 1 £ a3 £ 6. The number of such
tuples is
Solution: The total number of balls is
4 ´ 5 ´ 6 = 120
6 + 5 + 4 = 15
Thus, the probability that 3 balls drawn simultaneously
Out of these 15 balls, 3 balls can be drawn in 15C3 ways. are one blue, one black and one red is
Therefore, 3 balls can be drawn simultaneously in
120 24
=
15× 14 × 13 455 91
= 455 ways
1× 2 × 3

Example 7.6

A number x is drawn arbitrarily from the set of integers 100. Let A be the event that the number x has the
from 1 to 100, both inclusive. Find the probability that property
100 100
x > 29 - x > 29 -
x x
Now
Solution: When we draw an arbitrary number x from
the set {1, 2, …, 100} the total number of outcomes is 100
x > 29 -
x
7.2 Classical Definition of Probability 493

100 Therefore the number of cases favourable to A is 78.


Ûx+ > 29
x Thus the required probability is
Û x2 - 29 x + 100 > 0 78
P( A) = = 0.78
Û ( x - 25)( x - 4) > 0 100

Û x > 25 or x<4
Û x Î{1, 2, 3} È {26, 27, 28, …, 100}

Example 7.7

of the chosen two integers to be odd, one must be odd


Find the probability that their sum is odd. and the other must be even. Therefore, the number of
cases favourable is
Solution: The total number of ways of choosing 2 out
15 ´ 15 = 225
of 30 integers is
Thus, the required probability is equal to
30
C2 = 435
225 15
Out of these 30 integers, 15 are odd and 15 are even =
435 29
(since the given integers are consecutive). For the sum

7.2.1 Limitations of the Classical Definition


1. If the outcomes of a random experiment are not equally likely, the probability of an event in such an experiment
cannot be found. For example, the probability of a student passing an examination is not 1/2 as the outcomes of
success and failure in the examination are not equally likely.
2. If the random experiment contains infinitely many outcomes, then the classical definition cannot be applied for
the probability of an event in such an experiment. For example, tossing a coin until Head appears is a random
experiment in which there are infinite number of outcomes, namely all finite sequences of H’s and T’s (Heads and
Tails). The classical definition gives no answer for finding the probability that the Head never appears before 100th
tossing.
We have noticed certain limitations and deficiencies in the classical defination of probability. In order to over-
come one of these limitations, we now give the following definition which is known as the statisticall (or empiricall or
relative frequency) definition of probability.

DEFINITION 7.4 Let E be an event of a random experiment. Let the experiment be repeated n times out of
which E occurs mn times. Then the ratio
mn
rn =
n
is called the nth relative frequency of the event E. Clearly 0 £ mn £ n and hence 0 £ rn £ 1. If
{rrn} is a convergent sequence, then its limit is defined to be the probability of E and is denoted
by P(E). That is,
mn
P(E) = lim rn = lim
n®¥ n®¥ n
Z+.
It can be easily seen that 0 £ P(E) £ 1, since 0 £ rn £ 1 for all n ÎZ

One can easily observe the following deficiencies in the above statistical definition of probability
1. Repeating a random experiment infinitely many times is impossible.
2. The sequence of relative frequencies must be assumed to be convergent, which may not be true all the time.
494 Chapter 7 Probability

7.3 | Axiomatic Approach to Probability


We have noticed certain shortcomings and deficiencies both in the classical definition (Definition 7.3) and the statis-
tical definition (Definition 7.4). In order to overcome these shortcomings and deficiencies, a Russian probabilist
A.N. Kolmogorov has proposed an axiomatic approach to probability. Before going to this, first let us have certain
basic definitions.

DEFINITION 7.5 1. An outcome of a random experiment is called an elementary eventt or simple event.
2. The set of all outcomes of a random experiment is called the sample space associated with
the experiment. In other words, when x is a random experiment and S is the sample space
associated with x, then every element of S is an outcome of x and any performance of x
results in an outcome that corresponds to exactly one element of S.

DEFINITION 7.6 Some important terms are:


Let S be the sample space associated with a random experiment x.
1. The elements of S are called the sample points or elementary events of x.
2. A subset E of S is called an event. That is, a set of elementary events is called an event.
3. An event E is said to happen or occurs if an outcome of the experiment x belongs to E.
4. Otherwise, we say that E does not happen or E does not occur.
5. The empty set f and the whole set S, being subsets of S, are called impossible eventt and
certain (or sure) event, respectively.
6. The complement of the event E is the event given by S-E E and is denoted by Ec or E. Such
c
event called the complementary eventt of E. Usually E is denoted by E.

Example

Let x be the experiment of rolling a die. Then the sample event, “occurrence of a positive even integer £ 6 on the
space of x is the set S = {1, 2, 3, 4, 5, 6}, where the integers uppermost face of the die” is an event and “occurrence
denote the faces of the die. “Occurrence of a positive of a prime ³ 6 on the uppermost face of the die” is an
integer £ 6 on the uppermost face of the die” is a sure impossible event.

DEFINITION 7.7 1. A set S is set to be countably infinite if it is bijective with the set + of positive integers and,
in this case we can write S = {s1, s2, …, sn, …}.
2. A set S is said to be finite if S is empty or S is bijective with the set {1, 2, …, n} for some
n Î+ and in this case we write S = {s1, s2, …, sn} and say that S is an n-element sett or the
number of elements in S is n.
3. A set S is said to be atmost countable is S is either finite or countably infinite. For example
+,  are countably infinite sets.

Note that the sample space S of a random experiment x may be finite or infinite. Throughout our discussions in this
chapter, we take the sample space S of an experiment to be atmost countable; that is, either finite or countably infinite.
For example, the sample space in Examples (1) and (2) below Definition 7.1 are finite and that of the experiment
“Tossing a coin till tail appears” is countably infinite.

DEFINITION 7.8 Let E1, E2, …, En be some events of a random experiment x. That is, E1, E2, …, En are subsets
of the sample space S of x.
1. The events E1, E2, …, En are said to be mutually exclusive if Ei Ç Ej = f for all i ¹ j; that is,
if the happening of an event Ei prevents the happening of any other event Ej, j ¹ i.
2. The events E1, E2, …, En are said to be equiprobable or equally likely if there is no reason
to expect one of them to happen in preference to others.
7.3 Axiomatic Approach to Probability 495

3. The events E1, E2, …, En are said to be exhaustive if E1 È E2 È  È En = S; that is, if the
performance of the experiment always results in the occurrence of at least one of Ei ’s.

Note that for any random experiment x whose sample space is S, the set of all possible events of x is Ã(S), the power
set of S (the set of all subjects of S). In the following we introduce Kolmogorov’s axiomatic approach to the theory of
probability.

DEFINITION 7.9 Let x be a random experiment and S its sample space and suppose that S is finite. Then a func-
tion P : Ã(S) ®  is called a probability function if it satisfies the following conditions.
1. Positive axiom: P(E) ³ 0 for all E ÎÃ(S)
2. Completeness axiom: Ã(S) = 1
3. Union axiom: If E1 and E2 ÎÃ(S) and E1 Ç E2 = f, then
P(E1 È E2 ) = P(E1 ) + P(E2 )
In this case, for any E ÎÃ(S), P(E) is called the probability of the event E. If E = {s}, then we
write P(s) for P({s}) for simplicity. In the case when S is countably infinite, we have to replace
(3) above by (3¢) given below.
3¢. If {En} is a sequence of mutually exclusive events, then

æ ¥ ö ¥
P ç ∪ En ÷ = å P(En )
è n=1 ø n=1

provided this infinite sum exist.

Example 7.8

Let S = {1, 2, 3, 4, 5, 6} be the sample space of a random (2) Completeness axiom:


experiment. Define P(E) = å P( s) for any E £ S, where
sÎE P(S) = å P( s) = P(1) + P(2) + P(3) + P(4) + P(5) + P(6)
sÎS
P(1) = 0.1
= 0.1 + 0.2 + 0.2 + 0.4 + 0.05 + 0.05
P(2) = 0.2 = P(3)
=1
P(4) = 0.4
(3) Union axiom: For any E1 and E2 ÎÃ(S) with
P(5) = 0.05 = P(6) E1 Ç E2 = f, we have

Show that P is a probability function. P(E1 È E2) = å


sÎE1 È E2
P( s) = å P( s) + å P( s) = P(E1) + P(E2)
sÎE1 sÎE2

Solution: We have
s ÎE1 È E2 if and only if s ÎE1 or s ÎE2 but not
(1) Positive axiom: both.
Thus P is a probability function.
P ( E ) = å P ( s) > 0
s ÎE

since P(s) > 0 for all s ÎS.

Try it out Let S = {H, T


T} be the sample space of a random experiment x and define
1
P( H ) = = P(T )
2
P(f ) = 0
and P (S ) = 1
Then verify that P is a probability function.
496 Chapter 7 Probability

In the following we prove certain important elementary properties of probability functions.

T H E O R E M 7.1 The following hold good for any probability function P on Ã(S), where S is the sample space of
a random experiment x.
1. P(f) = 0
2. If Ec is the complementary event of E, then

P (Ec ) = 1 - P (E )

3. 0 £ P(E) £ 1 for all E ÎÃ(S)


4. If E1 Í E2 Í S, then P(E2 - E1) = P(E2) - P(E1)
5. If E1 Í E2 Í S, then P(E1) £ P(E2)
6. If E1, E2, …, En are mutually exclusive events, then

P(E1 È E2 È  È En ) = P(E1 ) + P(E2 ) +  + P(En )

PROOF All these are derived from the axioms (1), (2) and (3) of Definition 7.9.
1. Since S Ç f = f, we have, from Axiom (3) of Definition 7.9 that

P(S) = P(S È f ) = P(S) + P(f )

and hence P(f) = 0.


2. Since E Ç Ec = f and E È Ec = S, we have
1 = P (S ) = P ( E È E c ) = P ( E ) + P ( E c )

and hence P(Ec) = 1 - P(E)


3. Since 1 - P(E) = P(Ec) ³ 0 [by Axiom (1), Definition 7.9], we get that

0 £ P(E ) £ 1

4. Let E1 Í E2 Í S, Then
E2 = E1 È (E2 - E1 )
and E1 Ç (E2 - E1 ) = f

Therefore, we have

P(E2 ) = P(E1 ) + P(E2 - E1 )

and hence

P(E2 - E1 ) = P(E2 ) - P(E1 )

5. If E1 Í E2 Í S, then
P(E2 ) - P(E1 ) = P(E2 - E1 ) ³ 0

[by Axiom (1), Definition 7.9] and hence

P(E1 ) £ P(E2 )

6. This follows by induction on n and by using Axiom (3), Definition 7.9. ■

Note: For any event E, P(E): P(E) (ratio) is called odds in favour of E and the ratio P(E): P(E) is odds against E.
7.3 Axiomatic Approach to Probability 497

T H E O R E M 7.2 Let A, B and C be three events of a random experiment x and P is a probability function on Ã(S),
(A D D I T I O N where S is the sample space of x. Then the following hold.
THEOREM ON 1. P( A È B) + P( A Ç B) = P( A) + P( B)
PROBABILITY)
2. P( A - B) = P( A) - P( A Ç B)
3. P( A È B È C ) = P( A) + P( B) + P(C ) - P( A Ç B) - P( B Ç C ) - P(C Ç A) + P( A Ç B Ç C )

PROOF 1. From the Venn diagram in Figure 7.1, we have


A È B = [ A - ( A Ç B)] È ( A Ç B) È [ B - ( A Ç B)]

Now A - (A Ç B), A Ç B and B - (A Ç B) are mutually exclusive and hence

P( A È B) = P[ A - ( A Ç B)] + P( A Ç B) + P[ B - ( A Ç B)]

= P( A) - P( A Ç B) + P( A Ç B) + P( B) - P( A Ç B) [by (4), Theorem 7.1]


= P( A) + P( B) - P( A Ç B)

P( A È B) + P( A Ç B) = P( A) + P( B)

2. Since A - B = A - (A Ç B) and A Ç B Í A, we have

P( A - B) = P[ A - ( A Ç B)] = P( A) - P( A Ç B)

3. We make use of (1) repeatedly. Consider


P( A È B È C ) = P( A È B) + P(C ) - P[( A È B) Ç C ]

= P( A) + P( B) - P( A Ç B) + P(C ) - P[( A Ç C ) È ( B Ç C )]

= P( A) + P( B) + P(C ) - P( A Ç B) - [ P( A Ç C ) + P( B Ç C ) - P( A Ç C Ç B Ç C )]

= P( A) + P( B) + P(C ) - P( A Ç B) - P( B Ç C ) - P(C Ç A) + P( A Ç B Ç C )

A – (AÇB) A ÇB B – (AÇB)

A B

FIGURE 7.1 7.2. ■

Note that a probability function P on Ã(S), where S is given to be finite, is completely determined by P(s), s ÎS. If
P(s) is a non-negative real number for each s in a finite set S such that

å P ( s) = 1
sÎS

then the function P : Ã(S) ® , defined by

P( A) = å P( s)
sÎA

for any A Í S, is a probability function.


498 Chapter 7 Probability

In Table 7.1 we give set theoretic descriptions of various events, which will be useful in solving problems on prob-
ability later.

Table 7.1 Set theoretic descriptions of various events


S. No. Event Set theoretic description
1. Events A or B to occur AÈB
2. Events A and B to occur AÇB
3. A occurs and B does not occur A - B or A Ç Bc or A Ç B
4. Neither A nor B occurs Ac Ç Bc or A Ç B
5. Exactly one of the events A and B ( A Ç B) È ( A Ç B) or ( A Ç Bc ) È ( Ac Ç B)
occurs or ( A È B ) - ( A Ç B)
6. Not more than one of the events A or (A Ç Bc) È(Ac Ç B ) È (Ac Ç Bc) or
B occurs ( A Ç B) È ( A Ç B) È ( A È B)
7. The event B occurs whenever A occurs AÍB

Example 7.9

A pack of cards means a pack containing 52 cards, out Therefore


of which 26 are with red figures and another 26 are
4 13 1
with black figures. These 52 cards are divided into 4 P(E1 ) = , P(E2 ) = and P(E1 Ç E2 ) =
sets, namely Hearts (in red colour), Diamonds (in red 52 52 52
colour), Spades (in black) and Clubs (in black). Each set
consists of 13 cards labelled as
A, 2, 3, 4, 5, 6, 7, 8, 9, 10, J, Q, K P(E1 È E2 ) = P(E1 ) + P(E2 ) - P(E1 Ç E2 )
where A is Ace, J is Jack, Q is Queen, K is King. Find 4 13 1
= + -
the probability of drawing an ace or a spade from a well- 52 52 52
shuffled pack of 52 cards. 16 4
= =
52 13
Solution: Let E1 and E2 be the events of drawing an
ace and drawing a spade, respectively. Then we have to Thus 4/13 is the probability of drawing an ace or a spade
find P(E1 È E2). Note that E1 and E2 are not mutually from the pack.
exclusive, since there is a spade in aces. In fact, E1 Ç E2
has exactly one member. Note that
n(E1 ) = 4, n(E2 ) = 13 or n(E1 Ç E1 ) = 1

Example 7.10

A, B and C are three newspapers from a city. 25% of 4


and P( A Ç B Ç C ) =
the population reads A, 20% reads B, 15% reads C, 100
16% reads both A and B, 10% reads both B and C, 8%
reads both A and C and 4% reads all the three. Find the We have to find P(A È B È C). We can use the formula
percentage of the population who read atleast one of A, P(A È B È C ) = P( A) + P( B) + P(C ) - P( A Ç B)
B and C.
- P( B Ç C ) - P(C Ç A) + P( A Ç B Ç C )
Solution: We are given that 1
= (25 + 20 + 15 - 16 - 10 - 8 + 4)
25 20 15 100
P( A) = , P( B) = , P(C ) =
100 100 100 30
=
16 10 8 100
P( A Ç B) = , P( B Ç C ) = , P(C Ç A) =
100 100 100 Thus 30% of the people read atleast one of the newspapers.
7.4 Independent and Dependent Events 499

Example 7.11

If two numbers are selected randomly from 30 consecu- are 15C2 + 15C2 such ways. Therefore the probability
tive natural numbers, find the probability that the sum of that the sum of the two numbers selected is even is
the two numbers is 15
C2 + 15C2 15 ´ 14 14
(a) even. 30
= =
C2 435 29
(b) odd.
The event “a + b is odd” is complementary to the
Solution: Two numbers can be selected from 30 in event “a + b is even” and therefore, the probability
30
C2 = 435 ways. Since the given 30 numbers are consecu- that the sum of the two numbers selected odd is
tive, there are 15 even and 15 odd numbers among them.
14 15
(a) For a + b to be even, it is necessary and sufficient 1- =
that either both a and b are even or both odd. There 29 29

7.4 | Independent and Dependent Events


An icecream vendor sells more icecreams during a sunny day than other days. The probability of getting more profit
depends on the weather conditions that the day is sunny and hot. Situations like these lead to the following, in which
we assume that there is a probability function P on the set of events of a random experiment.

DEFINITION 7.10 Let x be a random experiment and A and B be two events of x. Then the event “occurrence
of B after the occurrence of A” is called a conditional eventt and is denoted by B/A
/ . Similarly,
the event “occurrence of A after the occurrence of B” is denoted by A/B.

DEFINITION 7.11 Let A and B be two events of a random experiment x. The conditional probability of occur-
rence of A after the occurrence of B is defined by
P( A Ç B)
P( B)
where P(B) > 0 and is denoted by P(A/B). That is,
P( A Ç B)
P( A / B) = , when P( B) > 0
P( B)

P( B Ç A)
P( B / A) = , when P( A) > 0
P( A)

QUICK LOOK 1

Number of favourable cases to A among B


P ( A / B) =
Number of favourable caases to B

Example 7.12

A pair of fair dice is thrown. Find the probability that from 1 to 6. Let A be the event “2 appears on either of
either of the dice shows 2 if the sum is 6. the dice” and B be the event “sum is 6”. We want to find
P(A/B). Note that
Solution: The sample space of the experiment
“throwing a pair of fair dice” consists of 36(= 6 ´ 6) A = {(2, b)| 1 £ b £ 6} È {(a, 2)| 1 £ a £ 6}
ordered pairs (a, b), where a and b can be any integers and B = {(1, 5), (2, 4), (3, 3), (4, 2), (5, 1)}
500 Chapter 7 Probability

Also, So
A Ç B = {(2, 4), (4, 2)} P( A Ç B) 2 / 36 2
P( A / B) = = =
Therefore P( B) 5 / 36 5

5 2
P( B) = and P( A Ç B) =
36 36

Example 7.13

In a class, 30% of the students failed in Physics, 25% Also,


failed in Mathematics and 15% failed in both Physics
15
and Mathematics. If a student is selected at random P( A Ç B) =
failed in Mathematics, find the probability that he failed 100
in Physics also. Therefore
Solution: Let A be the event “failed in Physics” and B P( A Ç B) 15 / 100 15 3
P( A / B) = = = =
be the event “failed in Mathematics”. We want to find P( B) 25 / 100 25 5
P(A/B). It is given that
30 25
P( A) = and P( B) =
100 100

T H E O R E M 7.3 Let A and B be two events of a random experiment such that P(A) > 0 and P(B) > 0. Then
(M U L T I P L I C A T I O N
P( A Ç B) = P( A)P( B / A) = P( B)P( A / B)
THEOREM ON
CONDITIONAL
PROBABILITY)
PROOF This is an immediate consequence of the definition of the conditional probabilities P(A/B) and
/ ). This can be extended to any finite number of events E1, E2, …, En, by using induction
P(B/A
on n. ■

C O R O L L A R Y 7.1 Let E1, E2, …, En be n events of a random experiment such that


n-1
P æ Ç Ei ö > 0
è i =1 ø

Then

æ n-1
ö
P ( E1 Ç E2 Ç  Ç En ) = P(E1 )P ( E2 / E1 ) P ( E3 /(E1 Ç E2 ))  P ç En / æ Ç Ei ö ÷
è è i =1 ø ø

Example 7.14

A bag contains 20 identical balls of which 8 are black and then the probability that the third ball drawn is blue is 10/18.
12 are blue. Three balls are taken out at random from Therefore, by Corollary 7.1, the required probability is
the bag one after the other without replacement. Find
12 11 10 11
the probability that all the three balls drawn are blue. × × =
20 19 18 57
Solution: The probability that the first ball drawn is blue Note: If the drawn ball is replaced every time, then the
is 12/20, since there are 12 blue balls among 20 balls in the probability is
bag. If the first ball is blue, then the probability that the
3 3
second ball drawn is blue is 11/19, since 11 of the remaining æ 12 ö æ 3 ö
19 are blue. Similarly, if the first two balls drawn are blue, çè ÷ø = çè ÷ø
20 5
7.4 Independent and Dependent Events 501

Example 7.15

A jar contains 10 white balls and 6 blue balls, all are of since 10 out of 10 + 6 balls are white. But, after one ball
equal size. Two balls are drawn without replacement. is chosen, there remain 9 white balls and 6 blue balls.
Find the probability that the second ball is white if it is Therefore the required probability is
known that the first is white.
P(E1 Ç E2 )
P(E2 / E1 ) =
Solution: Let E1 be the event “the first ball drawn is P(E1 )
white” and E2 be the event “the second ball drawn is 10 9
white again. Then, .
9 3
= 16 15 = =
10 10 / 16 15 5
P(E1 ) =
16

T H E O R E M 7.4 Let S be the sample space of a random experiment x and P be a probability function on Ã(S), the
set of all events of x. Let A ÎÃ(S) such that P(A) > 0. Define PA : Ã(S) ®  by
PA (E) = P(E / A)
for any E ÎÃ(S). Then PA is also a probability function.
PROOF Recall that
P(E Ç A)
PA (E) = P(E / A) =
P( A)
Since P(A) > 0 and P(E Ç A) ³ 0, PA(E) ³ 0 for all E ÎÃ(S). Also,
P(S Ç A) P( A)
PA (S) = = =1
P( A) P( A)
since A Í S. Further, let E1 and E2 be two mutually exclusive events (i.e., E1 and E2 Í S and
E1 Ç E2 = f). Then,
P[(E1 È E2 ) Ç A]
PA (E1 È E2 ) =
P( A)
P[(E1 Ç A) È (E2 Ç A)]
=
P( A)
P(E1 Ç A) + P(E2 Ç A)
= (since E1 Ç A Ç E2 Ç A = f )
P( A)
= PA (E1 ) + PA (E2 )

Thus, PA is a probability function on Ã(S). ■

DEFINITION 7.12 Let x be a random experiment and S its sample space. Let P be a probability function on
Ã(S). Two events A and B are said to be independent of each otherr if the occurrence of one
of them does not influence the occurrence of the other. That is, B is independent of A if
P( B) = P( B / A)
If A and B are not independent, then we say that they are dependent.

T H E O R E M 7.5 The following are equivalent to each other for any two events A and B of a random experiment.
1. A and B are independent of each other.
2. P(B) = P(B/A
/ )
502 Chapter 7 Probability

3. P(A) = P(A/B)
4. P(A Ç B) = P(A) P(B)
PROOF (1) Û (2) follows from Definition 7.12.
(2) Û (3) Û (4) follow from the multiplication theorem (Theorem 7.3), where we have
P( A Ç B) = P( A)P( B / A) = P( B)P( A / B) ■

Example 7.16

Let A and B be independent events with P(A) = 0.6 and (2) P(A/B) is given by
P(B) = 0.2. Find the following.
P( A Ç B) 0.12 3
(1) P(A Ç B) P( A / B) = = =
P( B) 0.2 5
(2) P(A/B)
P(B/A
/ ) is given by
(3) P(B/A
/ )
(4) P(A È B) P( A Ç B) 0.12 1
P( B / A) = = =
P( A) 0.6 5
Solution: From Theorem 7.5, we have
P(A È B) is given by
(1) P(A Ç B) is given by
P( A È B) = P( A) + P( B) - P( A Ç B)
3
P( A Ç B) = P( A)P( B) = (0.6)(0.2) = 0.12 = 17
25 = 0.6 + 0.2 - 0.12 = 0.68 =
25

Example 7.17

A bag contains 20 balls out of which 10 balls are white Similarly


and others are black. Two balls are drawn from the
1
bag at random with replacement. Let A be the event P( B) =
“first ball drawn is white” and B be the event “second 2
ball drawn is white”. Verify whether A and B are Further, there are 10 ´ 10 ways to draw the balls so that
independent. both the first and the second balls are white. Hence

Solution: It is given that the first ball drawn is 10 ´ 10 1


P( A Ç B) = =
replaced before drawing the second ball. Therefore, 20 ´ 20 4
there are 20 ´ 20 ways to draw the two balls, of which Now,
10 ´ 20 have the property that the first ball is white.
Therefore, 1 1 1
P( A Ç B) = = × = P( A)P( B)
4 2 2
10 ´ 20 1
P( A) = =
20 ´ 20 2 Therefore, by Theorem 7.5, A and B are independent of
each other.

Example 7.18

A bag B1 contains 3 white balls and 4 black balls and Let A be the event that the ball chosen is white. Then,
another bag B2 contains 2 black balls and 4 white balls.
3 4 2
A bag is drawn at random and a ball is drawn at random P( A / E1 ) = and P( A / E2 ) = =
from it. Find the probability that the ball drawn is white. 7 6 3
Since E1 and E2 are exhaustive and mutually exclusive,
Solution: Let E1 and E2 be the events of choosing B1 it follows that
and B2, respectively. Then,
A = ( A Ç E1 ) È ( A Ç E2 )
1
P(E1 ) = = P(E2 ) and ( A Ç E1 ) Ç ( A Ç E2 ) = f
2
7.4 Independent and Dependent Events 503

Therefore 1 3 1 2
= × + ×
2 7 2 3
P( A) = P( A Ç E1 ) + P( A Ç E2 )
3 1 23
= P(E1 )P( A / E1 ) + P(E2 )P( A / E2 ) = + =
14 3 42

T H E O R E M 7.6 Let E1, E2, E3, …, En be mutually exclusive and exhaustive events with non-zero probabilities of
(T O T A L a random experiment x. Then for any event A connected with x,
PROBABILITY) n
P( A) = å P(Ei )P( A / Ei )
i =1

PROOF We have
A= AÇS
= A Ç (E1 È E2 È  È En )
= ( A Ç E1 ) È ( A Ç E2 ) È  È ( A Ç En )

n n
P( A) = å P( A Ç Ei ) = å P(Ei )P( A / Ei )
i =1 i =1 ■

T H E O R E M 7.7 Let E1, E2, …, En be exhaustive and mutually exclusive events of a random experiment x and S be
(B A Y E S ’ the sample space of x with a probability function P on Ã(S) such that P(Ei) > 0 for all 1 £ i £ n.
THEOREM) Then for any event A of the random experiment x,
P ( Ej ) P ( A / Ej )
P(Ej / A) = n

å P (E )P ( A / E )
i =1
i i

£ j £ n.
PROOF By hypothesis E1, E2, …, En are exhaustive and mutually exclusive and hence
E1 È E2 È  En = S
and Ei Ç Ej = f for i¹j
Also, it is given that P(Ei) > 0 for all 1 £ i £ n. For any event A (i.e., A Í S), we have

A = A Ç S = A Ç æ È Ei ö = È( A Ç Ei )
n n

è i =1 ø i =1

A Ç Ei Ç A Ç Ej = A Ç Ei Ç Ej = f for i ¹ j we have
n
P( A) = å P( A Ç Ei )
i =1
n
= å P(Ei )P( A / Ei ) (by Theorem 7.3)
i =1

Thus, by Theorem 7.3 again, for any 1 £ i £ n, we have


P(Ej Ç A)
P(Ej / A) =
P( A)
P(Ej Ç A)
= n

å P(E )P( A / E )
i =1
i i

504 Chapter 7 Probability

Example 7.19

Three bags, B1, B2 and B3 contain balls as given in Table 7.2. Having choosen the bag Bi, the probability of choosing a
red ball is P(R/Ei) and is given by
Table 7.2 Example 7.19
2 4 2
Red White Black P(R / E1 ) = , P(R / E2 ) = and P(R / E3 ) =
5 9 9
B1 2 2 1 We want to find the probability P(E2/R). By Bayes’
B2 4 3 2 theorem, we get that
B3 2 4 3
P(E2 )P(R /E2 )
P(E2 /R) =
P(E1)P(R / E1) + P(E2 )P(R /E2 ) + P(E3 )P(R /E3)
A die is thrown. B1 is chosen if either 1 or 2 turns up,
B2 is chosen if either 3 or 4 turns up and B3 is chosen if 1 4
´
either 4 or 5 turns up. Having chosen a bag in this way, a = 3 9
ball is chosen at random from this bag. If the ball chosen æ 1 2ö æ 1 4ö æ 1 2ö
çè ´ ÷ø +ç ´ ÷ +ç ´
è 3 9ø è 3 ÷
is of red colour, find the probability that it comes from 3 5 9ø
bag B2?
4
Solution: Let Ei be the event of choosing the bag Bi, for = 27
i = 1, 2, 3. Let R be the event of choosing a red ball. Then æ 18 + 20 + 10 ö
çè 3 ´ 5 ´ 9 ÷ø
1
P(Ei ) = for i = 1, 2, 3
3 4 3´5´9 5
= ´ =
27 48 12

7.5 | Random Variables and Probability Distributions


In this section we define the concepts of random variables and probability distributions and discuss certain important
properties of these. First we define a random variable which quantifies the events of a sample space of a random
experiment.

DEFINITION 7.13 Let S be the sample space of a random experiment. Then any real-valued function defined on
S is called a random variable on S. That is, a random variable of S is simply a function J :S ® R.

Examples

(1) Let S be the sample space of the experiment (3) Let S be the sample space of the experiment “tossing
That is, S = {1, 2, 3, 4, 5, 6}. Define
“rolling a fair die”. T three fair coins simultamously”.
J :S ®  by Then

ïì1 if s is even S = {HHH, HHT, HTH, HTT, THH, THT, TTH, TTT}
J( s) = í
îï0 if s is odd Define J : S ® R by
That is, J (1) = J (3) = J (5) = 0 and J (2) = J (4) =
T J (s) = The number of heads in s
J (6) = 1. Then J is a random variable on S.
For example, J (HHH) =3, J (HHT) = 2, J (THT) = 1,
(2) Let S be the sample space of the experiment “tossing J (TTT) = 0. Then J is a random variable on S.
a fair coin”. That is, S = {H, T
T}, where H = Head and
T = Tail. Define J : S ® R by
J ( H ) = 1 and J (T ) = 0
Then J is a random variable on S.

For any random experiment x and its sample space S, we prove in the following that any probability function on
Ã(S) and a random variable on S give rise to a probability function on P().
7.5 Random Variables and Probability Distributions 505

T H E O R E M 7.8 Let S be the sample space of a random experiment. Let P : Ã(S) ®  be a probability function
and J : S ®  be a random variable. Define P¢ : Ã() ®  by
P¢(T) J-1(T))
T = P(J T for any T Í ,
where J-1(T)
T = {s ÎS | J (s) ÎT}.
T Then P¢ is a probability function on Ã() and is called the prob-
ability function induced by P and J.
PROOF Since P(E) ³ 0 for all E Í S, we have

P ¢(T ) = P(J -1 (T )) ³ 0 for all T Í R


since J-1(T)
T Í S. Also,

P ¢(R) = P(J -1 ()) = P(S) = 1


If A and B Í R such that A Ç B = f, then

J -1 ( A) Ç J -1 ( B) = J -1 ( A Ç B) = J -1 (f ) = f
and J -1 ( A È B) = J -1 ( A) È J -1 ( B)
and therefore

P ¢( A È B) = P[J -1 ( A È B)]
= P[J -1 ( A) È J -1 ( B)]
= P[J -1 ( A)] + P[J -1 ( B)]
= P ¢( A) + P ¢( B)
Thus P¢ is a probability function on Ã(). ■

Example

Let S be the sample space of the experiment “rolling a Also, since


fair die” and P be the usual probability function on Ã(S)
1 1 1 1
determined by P({1, 3, 5}) = + + =
6 6 6 2
1
P ( s) = for any s Î S 1 1 1 1
6 and P({2, 4, 6}) = + + =
6 6 6 2
Note that S is the 6-element set {1, 2, 3, 4, 5, 6}.
we have
Let J :S ®  be the random variable given by J (1) =
J (3) = J (5) = 0 and J (2) = J (4) = J (6) = 1. Then, for ì0 if 0, 1 ÏT
any T Í R, ï
ï1
P ¢(T ) = P(J -1 (T )) = í if 0, 1 ÎT but not both
J -1 (0) = {1, 3, 5}
ï2
J -1 (1) = {2, 4, 6} ï1 if 0, 1 ÎT
î
ìf if 0 ÏT and 1 ÏT
ï
-1 ï{1, 3, 5} if 0 ÎT and 1 ÏT
J (T ) = í
ï{2, 4, 6} if 0 ÏT and 1 ÎT
ï
îS if 0 ÎT and 1 ÎT

It is a convention to use the letter X to denote a random variable and as such a random variable is simply a func-
tion X from the sample space S into the real number system. Here afterwards, we use X to denote a random variable.
506 Chapter 7 Probability

DEFINITION 7.14 Let S be the sample space of a random experiment and X : S ®  be a random variable.
Then, to each real number r, the event

X -1 ((-¥, r ]) = {s Î S | X ( s) £ r}

is denoted by(X £ r). That is, for each r Î,


( X £ r ) = { s Î S | X ( s) £ r }

DEFINITION 7.15 X, the function F :  ® , defined by


For any random variable X
F r) = P((X £ r))
F( for any r Î,
is called the probability distribution function of X.
X

T H E O R E M 7.9 Let X be a random variable and F the probability distribution of X


X. Then the following hold.
1. 0 £ F(
F r) £ 1 for all r Î
2. r £ s Þ F(
F r) £ F(
F s) for any r and s Î
3. lim F (r ) = 1 and lim F (r ) = 0
r ®¥ r ®-¥

F (r + 0) = lim F ( s) = F (r )
s® r
s >r

PROOF For any real number, we have


F (r ) = P((X £ r ))
where
( X £ r ) = { s Î S | X ( s) £ r }
Since P is a probability function, we have 0 £ P(E) £ 1 for all E Í S and therefore
0 £ F (r ) £ 1
Also, if r and s Î, then
r £ s Þ (-¥, r ] Í (-¥, s]
Þ X -1 ((-¥, r ]) Í X -1 ((-¥, s])
Þ ( X £ r ) Í ( X £ s)
Þ P(( X £ r )) £ P(( X £ s))
Þ F (r ) £ F ( s)
The remaining results follow from the fact that F is an increasing function and
lub F (r ) = 1, glb F (r ) = 0
r ÎR r ÎR

and, for any r ÎR,


(-¥, r ] = Ç (-¥, s] ■
r < sÎR

DEFINITION 7.16 A random variable whose range is atmost countable (i.e., finite or countably infinite) is
called a discrete random variable. A random variable which is not discrete is called a contin-
uous random variable.

The random variables given in all the above examples are discrete. Though there are certain continuous random
variables, we confine our discussion to the discrete random variables only. In the following we introduce the notions
of mean, variance, standard deviation and probability distribution of discrete random variables.
7.5 Random Variables and Probability Distributions 507

DEFINITION 7.17 Let X be a discrete random variable on the sample space S of a random experiment and let
Z+,
the range of X be {x1, x2, …, xn, …}. Let P be a probability function on Ã(S). For each n ÎZ
let P(X = xn) be defined by
P( X = xn ) = P( X -1 {xn })

1. If å x P( X = x ) is finite, then it is called the mean of X and is denoted by m


n
n n X, or simply
m when there is no ambiguity about X
X. That is,

m = å xn P( X = xn )
n

2. If å ( xn - m )2 P( X = xn ) is finite, then it is called the variance of X and is denoted by uX or


n
simply u.
3. If u is the variance of X
X, then u is called the standard deviation of X and is denoted by
sX, or simply ss. Therefore u = s or s 2 = u.
4. The system of numbers
X : x1 x2 x3  xn 
P( X = xi ): p1 p2 p3  pn 
probability distribution of X.
X
X-1{xi}).)
(Note that pi = P(X = xi) = P(X

QUICK LOOK 2

From the points given in Definition 7.17, we have = å xn2 pn - 2 m 2 + m 2 (since å xn pn = m and å pn = 1)
n n n
s 2 = å ( xn - m )2 pn
n = å x pn - m
2
n
2

n
= å xn2 pn - 2 m å xn pn + m 2 å pn
n n n Therefore

s 2 + m 2 = å xn2 pn
n

Example 7.20

The probability distribution of a random variable X is = 0.25 + 1.00 + 2.25 + 4 + 6.25 + 9 + 12.25
given in Table 7.3.
= 35
Table 7.3 Example 7.20 The variance is given by
X=x 1 2 3 4 5 6 7 7

P(X = x) 0.25 0.50 0.75 1.0 1.25 1.50 1.75 u = å ( xr - m )2 P( X = xr )


r =1

Find the mean, variance and standard deviation of X.


X = (-34)2(0.25) + (-33)2(0.5) + (-32)2 (0.75)) + (-31)2(1.00)
+ (-30)2(1.25) + (-29)2(1.5) + (-28)2(1.75)
Solution: The mean is given by
= 289 + 544.5 + 768 + 961 + 1125 + 1261.5 + 1372
7
m = å xr P( X = xr ) = 6321
r =1
The standard deviation is
= 1 ´ (0.25) + 2(0.50) + 3(0.75) + 4(1.0)
+ 5(1.25) + 6(1.50) + 7(1.75) s = u = 6321 = 7 129
508 Chapter 7 Probability

Example 7.21

Let X be a discrete random variable whose probability Therefore


distribution is given by 5
K+1
(K + 1)a aå K
=1
P( x = K ) = K =0 2
2K
æ 2 3 4 5 6ö
for K = 0, 1, 2, 3, 4 and 5. Find the value of a. aç1 + + 2 + 3 + 4 + 5 ÷ = 1
è 2 2 2 2 2 ø
Solution: Since æ 32 + 32 + 24 + 16 + 10 + 6 ö

è ÷ø = 1
5 32
å P( x = K ) = 1
K =0 4
a=
15
we have
5
(K + 1)a
å
K =0 2K
=1

7.6 | Theoretical Discrete Distribution


In this section, we discuss certain probability distributions in which the variables are distributed according to some
definite probability law that can be expressed mathematically. In particular, binomial distributions and Poisson distri-
butions are of special importance.
A random variable X which takes just two values 0 and 1 with the probabilities p and q, respectively, is of partic-
ular interest. In this case, note that
P( X = 0) = p, P( X = 1) = q and p+q=1
H =p
Quite often we come across such random variables. For example, in coin tossing experiment, we can define P(H)
T = 1 - p, for some 0 < p < 1 and the random variable X can be defined by X(
and P(T) H = 1 and X(
X H) T = 0, so that
X T)
P( X = 1) = p and P( X = 0) = 1 - p
Each repetion of such an experiment is called a trial.

7.6.1 Binomial Distribution


Let E and Ec be two complementary events in a random experiment with probabilities p and q, respectively. Let us call
the occurrence of the event E as a success and the occurrence of the event Ec as a failure in a trial. If the experiment is
repeated n times and Ek is the event having k successes in these trials, then one of these cases is “E E occurs in the first
k trials, Ec occures in the remaining n - k trials”. The required probability for such an event is pk qn-k.
However the number of such cases is
n!
n
Ck =
(n - k )! k !
Thus
P(Ek ) = n Ck pk qn - k
Let X denote the number of successes in these n trials. Then X is a random variable with range {0, 1, 2, …, n}. Further
P( X = k ) = n Ck pk qn - k = n Ck pk (1 - p)n - k (since p + q = 1)
X is summarized in Table 7.4. This distribution shown in Table 7.4 is called a binomial distribution.
Here n and p are called the parameters of X.
X
7.6 Theoretical Discrete Distribution 509

Table 7.4 Binomial distribution


X=k 0 1 2 … r … n
P(X = k) n
C0 p0qn-0 n
C1 p1qn-1 n
C2 p2qn-2 … n
Cr prqn-r … n
Cn pnq0

DEFINITION 7.18 A random variable X is said to follow binomial distribution, or simply, it is a binomial variate
with parameters n and p if
P(X = k) = nCk pkqn-k for k = 0, 1, 2, …, n
and this is described by writing X ~ B(n, p).

Note that P(X = k) are nothing but the terms of the binomial expansion of (p( + q)n and this justifies the name ‘bino-
mial distribution”. Also, recall that q = 1 - p.
The following can be easily proved by using the results in Chapter 7, Binomial Theorem, Vol. 1.

T H E O R E M 7.10 If X ~ B (n, p), then the mean m and the variance s 2 of X are equal to np and npq, respectively.

Notice that a binomial distribution takes place under the following experimental conditions.
1. Each trial results in two mutually exclusive outcomes, termed as success and failure.
2. The number n of trials is finite.
3. The trials are independent of each other.
4. The probability p of success is constant for each trial.

7.6.2 Poisson Distribution


Another important theoretical discrete distribution is the Poisson distribution which is indeed a limiting case of the
binomial distribution. A random variable X is said to follow Poisson distribution with parameter l and X is called a
Poisson variate if

l k -l
P( X = k ) = e for k = 0, 1, 2, …
k!
l > 0 is a constant. This is called the Poisson law. Table 7.5 shows the Poisson distribution.

Table 7.5 Poisson distribution


X=k 0 1 2 3 … k …
l -l l -l 2
l -l
3
… l -l
k

P(X = k) e-l e e e e
1! 2! 3! k!

The following is an important result and we skip the proof of this.

T H E O R E M 7.11 Let X be a Poisson random variate with parameter l. Then the mean and variance of X are both
equal to l.

Note that the Poisson distribution can be used under the following experimental conditions.
1. Each trial results in two mutually exclusive outcomes, termed as success and failure.
2. The number n of trials is sufficiently large.
3. The probability p of success is very small.
4. The trials are independent of each other.
510 Chapter 7 Probability

QUICK LOOK 3

The Poisson distribution can be used, for instance, in 3. The number of vehicles passing a given spot per
problems like the following: minute during the peak hours of a day in a given
1. The number of defective items in a packing city.
2. The number of deaths from non-epidemic diseases
such as heart attack or cancer or snake bites

Example 7.22

Eight coins are tossed simultaneously. Find the probability r = 0, 1, 2, …, 8. Therefore, the probability of getting
of getting atleast six Heads, using the binomial distribution. atleast 6 heads is

Solution: Let P( X ³ 6) = P( X = 6) + P( X = 7) + P( X = 8)
8 8 8
1 æ 1ö æ 1ö æ 1ö
p = the probability of getting a head = = 8C6 ç ÷ + 8C7 ç ÷ + 8C8 ç ÷
2 è 2ø è 2ø è 2ø
1 7 1 1 37
and q = the probability of getting a tail = = + + =
2 64 32 256 256
The probability of getting r heads in a random toss is
r 8-r 8
æ 1ö æ 1ö æ 1ö
P ( X = r ) = 8 Cr ç ÷ ç ÷ = 8 Cr ç ÷
è 2ø è 2ø è 2ø

Example 7.23

In a book of 750 pages, there are 500 typographical The probability of r errors per page is
errors. Assuming Poisson law for the number of errors r
per page, find the probability that a random sample of l r - l æ 2 ö 1 -2 / 3
P( X = r ) = e =ç ÷ e
5 pages will contain no error. r! è 3ø r!

Therefore
Solution: The average member of errors per page in
the book is P( X = 0) = e-2 / 3
500 2 The required probability that a random sample of
l= =
750 3 5 pages will contain no error is
( P( X = 0))5 = (e-2 / 3 )5 = e-10 / 3

Example 7.24

A Poisson variate X satisfies P(X = 1) = P(X = 2). Find (2 l - l 2 )e- l = 0


P(X = 6).
l (2 - l ) = 0
Solution: Recall that Since l > 0, we should have l = 2. Therefore
l -lr
l 6 - l 64 -2 æ 4 ö -2
P( X = r ) = e ,l >0 P( X = 6) = e = e =ç ÷e
r! 6! 720 è 45 ø
r = 0, 1, 2, … . It is given that P(X = 1) = P(X = 2).
Therefore

l -l l2 -l
e = e
1! 2!
Worked-Out Problems 511

WORKED-OUT PROBLEMS
Single Correct Choice Type Questions
1. A, B and C are three athletes running in a race. If Solution:
the probability of A winning is twice as likely to win real number and sum of all the probabilities is equal to 1,
as probability of B and that of B is as likely to win as the correct answer is (A).
of C, then the probability of A’s win is Answer: (A)
1 2 3 4
(A) (B) (C) (D)
7 7 7 7 4. A die is so weighed such that the probability of a
number appearing when tossed is proportional to the
( ) = 2P(B) and P(B) = 2P(C).
Solution: By hypothesis P(A number on the face. Then, the probability of a prime
Since numbered face to appear is
P( A) + P( B) + P(C ) = 1 4 5 10 11
(A) (B) (C) (D)
7 21 21 21
we have
Solution: Let p be the constant of proportionality so
4 P(C ) + 2 P(C ) + P(C ) = 1
that
Therefore P(K ) = Kp for K = 1, 2, 3, 4, 5, 6
1
7 P(C ) = 1 or P(C ) = Sum of the probabilities = 1. This implies
7
1
So 21 p = 1 Þ p =
21
4
P( A) = 4 P(C ) = Let E be the event of a prime number face. That is
7 E = {2, 3, 5}. Then
Answer: (D)
10
P(E ) = 2 p + 3 p + 5 p =
2. In Problem (1), the probability that one of B or C will 21
win is Answer: (C)
2 3 4 5
(A) (B) (C) (D) 5. A and B are two events of a random experiment. If
7 7 7 7
P( A È B) = 7 / 8, P( A Ç B) = 1/ 4 and P( A) = 5 / 8, then
Solution: The events that either of B or C will win are P( A Ç B) is equal to
mutually exclusive events so that P(B Ç C) = 0. Hence 1 1 1 3
(A) (B) (C) (D)
P( B È C ) = P( B) + P(C ) 8 4 3 8
2 1 3 Solution: We have
= + =
7 7 7
P( A Ç B) = P( A - B) = P( A) - P( A Ç B)
Answer: (B)
æ 5ö 1
= ç1 - ÷ -
3. Let S = {x1, x2, x3, x4} be a four-element sample space. è 8ø 4
Which of the following functions defines a probability
3 1 1
function on S? = - =
8 4 8
1 1 1 1
(A) P( x1 ) = , P( x2 ) = , P( x3 ) = , P( x4 ) =
4 8 2 8 Answer: (A)
1 1 1 1
(B) P( x1 ) = , P( x2 ) = , P( x3 ) = , P( x4 ) = 6. Two dice of different colours are thrown at a time.
3 5 4 2
The probability that the sum of the faces appeared is
1 1 1 1 either 7 or 11 is
P( x1 ) = , P( x2 ) = - , P( x3 ) = , P( x4 ) =
2 3 4 2 7 4 2 5
1 1 2 (A) (B) (C) (D)
P( x1 ) = 0, P( x2 ) = , P( x3 ) = , P( x4 ) = 36 9 3 9
2 3 3
512 Chapter 7 Probability

Solution: The total number of possible outcomes = Therefore


6 ´ 6 = 36. Let A = Event of the sum is 7 and B = event of
240 16
the sum is 11. Therefore P(E ) = =
495 33
A = {(1, 6), (2, 5), (3, 4), (6, 1), (5, 2), (4, 3)} Answer: (C)
B = {(5, 6), (6, 5)}
9. There are three boxes B1, B2 and B3. B1 contains
It is observed that A Ç B = f. Hence 1 white and 2 black balls. B2 contains 3 white, 1 black
ball and B3 contain 2 white and 3 black balls. One ball
6 2 4
P( A È B) = P( A) + P( B) = + = is taken from each box. Among the balls drawn, the
36 36 9 probability that there are two black and 1 white ball is
Answer: (B) 7 23 37 5
(A) (B) (C) (D)
12 60 60 12
7. From a well-shuffled pack of 52 cards, four cards
are selected at random. The probability of drawing Solution: Let E be the event that the draw contains
exactly 2 spades and exactly 2 aces is 2 black and one white ball. Let wi drawing white ball
1494 1594 from Bi and bj be the event of drawing a black ball from
(A) (B) the box Bj (i = 1, 2, 3, and j = 1, 2, 3). Therefore
270725 270725

(C)
1296
(D)
1396 E = (b1 Ç b2 Ç w3 ) È (b1 Ç w2 Ç b3 ) È (w1 Ç b2 Ç b3 )
270725 270725
Each of the events in the union is mutually exclusive and
Solution: Total number of outcomes = 52C4 = 270725 independent. Hence
E = Event of exactly two spade cards and exists two aces.
A = Event of 1 spade ace, 1 non-spade ace and 1 spade P(E) = P(b1 Ç b2 Ç w3 ) + P(b1 Ç w2 Ç b3 )
card and 1 non-spade card + P(w1 Ç b2 Ç b3 )
B = Event of 2 non-spade aces and 2 spade cards.
= P(b1 )P(b2 )P(w3 ) + P(b1 )P(w2 )P(b3 )
1(3C1 )(12C1 )(36C1 ) 3 ´ 12 ´ 36 1296
P( A) = 52
= = + P(w1 )P(b2 )P(b3 )
C4 270725 270725
2 1 2 2 3 3 1 1 3
3
C2 ´ 12C2 3 ´ 66 198 = × × + × × + × ×
P( B) = 52
= = 3 4 5 3 4 5 3 4 5
C4 270725 270725
25 5
= =
Now E = A È B and A Ç B = f. This implies 60 12
P(E) = P( A È B) Answer: (D)

1494 10. A class contains 20 boys and 20 girls of which half the
= P( A) + P( B) =
270725 boys and half the girls have cat eyes. If one student
Answer: (A) is selected from the class, the probability that either
the student is a boy or has cat eyes is
8. 1 3 3 2
(A) (B) (C) (D)
husband) are standing in a room. Four people are 2 4 8 3
chosen at random. The probability that the selection
contains exactly one couple is Solution: Let A be the event of a boy and B the event
of having cat eyes. So
1 1 16 17
(A) (B) (C) (D)
11 33 33 33 20 1
P( A) = =
40 2
Solution: From 12 people, 4 can be selected in
C4 = 495 ways.
12
and P( B) =
20 1
=
E = Event of the selection contains exactly one couple. 40 2
Therefore the selection must contain one couple and the
rest 2 must be non-couple. This can be done in
10 1
6
C1 ´ 2(5C1 ´ 4 C1 ) = 6 ´ 40 = 240 ways P( A Ç B) = =
40 4
Worked-Out Problems 513

Therefore 13. A positive integer is selected at random from the


first 200 natural numbers. The probability that it is
P( A È B) = P( A) + P( B) divisible by 4 or 5 is
- P( A Ç B) (see Theorem 7.2) 1 1 3 3
(A) (B) (C) (D)
1 1 1 3 3 5 5 10
= + - =
2 2 4 4 Solution: Any integer divisible by 4 or 5 must be divis-
Answer: (B) ible by 20.
Number of multiples of 4 lying between 1 and 200
11. If a positive divisor of 1049 is selected at random, (including 200) is 50.
then the probability that it is an integer multiple of Number of multiples of 5 is 20.
1038 is Number of multiples of 20 is 10.
9 18 36 9 Therefore required probability is
(A) (B) (C) (D)
625 625 625 2500 50 + 20 - 10 60 3
= =
Solution: 10 = 2 × 5 Þ there are 50 ´ 50 = 2500 posi-
49 49 49 200 200 10
tive divisions of 1049. Now 1038 = 238 × 538 Þ any integer Answer: (D)
multiple of 1038 must be of the form 2a × 5b where 38 £ a,
b £ 49. That is, a and b will have 12 choices each. Hence 14. Out of 5 men and 6 women a committee of 5 is to be
required probability is formed. If the selection is at random, the probability
that the committee consists of at least 3 women is
12 ´ 12 36
= 141 143 131 1
2500 625 (A) (B) (C) (D)
462 462 462 4
Answer: (C)
Solution: Let
12. Let A and B be two events such that P(A) = 0.3, E: Event that committee consists at least 3 women.
P(B) = 0.6 and P(B/A
/ ) = 0.5. Then P( A / B) equals The favourable cases to E are:

3 5 9 1
(A) (B) (C) (D) Women Men No. of selections
4 8 40 4
3 2 6
C3 ´ 5C2
Solution: We have by Theorem 7.3
4 1 6
C4 ´ 5C1
P( A Ç B) = P( A)P ( B / A) = (0.3)(0.5) = 0.15
5 0 6
C5 ´ 5C0
Now
Therefore
P( A È B) = P( A) + P( B) - P( A Ç B)
= 0.3 + 0.6 - 0.15
6
C5 ´ 5C2 + 6C4 ´ 5C1 + 6C5 ´ 5C0
P(E ) = 11
C5
= 0.75
6 ´ 10 + 15 ´ 5 + 6 ´ 1
Also =
462
P( A Ç B) 141
P( A / B) = =
P( B) 462
P( A È B) Answer: (A)
=
P( B)
15. Each of two bags A and B contain n cards numbered
1 - P( A È B) 1 to n. One card from each of A and B is drawn.
=
1 - P( B) The probability that the card drawn from A bears
number smaller than the number on the card drawn
1 - 0.75
= from B is
1 - 0.6
n+1 n-1 n+1 n-1
(A) (B) (C) (D)
0.25 250 5 2n 2n n n
= = =
0.4 400 8
Answer: (B)
514 Chapter 7 Probability

Solution: Let 17. Let E, F and G be pairwise independent events with


P(G) > 0 and P(E Ç F Ç G) = 0. Then
T P((E Ç F )/G))
E1 = Number drawn from A is smaller than that drawn
is equal to
from B.
E2 = Number drawn from A is greater than that drawn (A) P(E) + P(F ) (B) P(E) - P(F )
from B.
(C) P(E) - P(F ) (D) P(E) - P(F )
E3 = Both numbers are equal.
Solution: Note that
Note that
P( A - B) = P( A - A Ç B) (∵ A - B = A - A Ç B)
P(E1 ) = P(E2 )
and also
n 1
and P(E3 ) = 2 =
n n P( A - B) = P( A Ç B) (∵ A - B = A Ç B)
Therefore

1 P(E Ç F Ç G) = P(G - (E Ç F ))
2 P(E1 ) = 1 - P(E3 ) = 1 -
n = P(G - (E È F ))
n-1 = P(G - G Ç (E È F ))
Þ P(E1 ) =
2n
= P(G) - P((G Ç E) È (G Ç F ))
Answer: (B) (See Theorem 7.2)
= P(G) - P(G Ç E) - P(G Ç F )
16. A, B, C are three events such that P(A) = 0.3,
P(B) = 0.4, P(C) = 0.8, P(A Ç B) = 0.08, P(C Ç A) = + P(G Ç E Ç F ) (By Theorem
m 7.2)
0.28, P(A Ç B Ç C) = 0.09, and P(A È B È C) ³ 0.75. = P(G) - P(G)P(E) - P(G)P(F ) + 0
Then P(B Ç C) belongs to the interval:
(∵ P(E Ç F Ç G) = 0 and E, F
(A) [0.11, 0.23] (B) [0.23, 0.48]
and G are pairwise independent)
(C) [0.13, 0.25] (D) [0.15, 0.20]
= P(G)(1 - P(E) - P(F ))
Solution: We have
= P(G)( P(E) - P(F ))
P( A È B È C ) = (å P( A)) - P( A Ç B) - P( B Ç C )
Hence
- P(C Ç A) + P( A Ç B Ç C )
P(E Ç F Ç G)
= (0.3 + 0.4 + 0.8) - 0.08 - P( B Ç C ) P((E Ç F )/G)) =
P(G)
- 0.28 + 0.09 (7.1)
P(G)[ P(E) - P(F )]
= = P (E ) - P (F )
Therefore P(G)
P( B Ç C ) = 1.23 - P( A È B È C ) ³ 1.23 - 1 Answer: (C)

= 0.23 [∵ P( A È B È C ) £ 1] 18. Six students are to be selected for a quiz competition


from 10 aspirants. The probability that two partic-
0.23 £ P( B Ç C ) ular students are excluded is
Also 2 1 1 2
(A) (B) (C) (D)
15 3 5 3
P( B Ç C ) £ 1.23 - (0.75) = 0.48
Solution: Total number of selections of 6 from 10 = 10C6.
[∵ 0.75 £ P( A È B È C )]
Since two particular students are to be excluded, we have
So to select 6 from 8 only. This can be done in 8C6 ways. The
required probability is
P( B Ç C ) £ 0.48 (7.3) 8
C6 8 6´ 4
= ´
Equations (7.2) and (7.3) imply 10
C6 6 ´ 2 10

0.23 £ P( B Ç C ) £ 0.48 3× 4 2
= =
9 × 10 15
Answer: (B)
Answer: (A)
Worked-Out Problems 515

19. E and F are complementary events of the events (D) We have


F. If 0 < P(F)
E and F F < 1, then
P (E Ç F ) P (E Ç F )
(A) P(E / F ) + P(E / F ) < 1 P (E / F ) + P (E / F ) = +
P(F ) P(F )
(B) P(E / F ) + P(E / F ) = 1 P(E Ç F ) + P(E Ç F )
=
(C) P(E / F ) + P(E / F ) = 1 P(F )
(D) P(E / F ) + P(E / F ) = 1 P(F Ç (E È E )
=
P(F )
Solution:
(A) We have P(F )
= =1
P(F )
P(E Ç F ) P(E Ç F )
P(E / F ) + P(E / F ) = +
P(F ) P(F ) Therefore (D) is correct.
Answer: (D)
P(F Ç (E È E))
=
P(F ) 20. If P( B) = 3 / 4, P( A Ç B Ç C ) = 1/ 3 and P( A Ç B Ç C ) =
1/3, then P(B Ç C
C) is
P(F )
= =1
P(F ) 1 1 1 1
(A) (B) (C) (D)
12 6 15 9
Thus, (A) is not correct.
(B) We have Solution: From Figure 7.2 we have
B = (B Ç C ) È ( A Ç B Ç C ) È ( A Ç B Ç C )
P(E / F ) + P(E / F )
Therefore
P(E Ç F ) P(E Ç F )
= +
P(F ) P(F ) P( B Ç C ) = P( B) - P( A Ç B Ç C ) - P( A Ç B Ç C )

P(E Ç F )P(F ) + P(E Ç F )P(F ) 3 æ 1 1ö


= = -ç + ÷
P(F )P (F ) 4 è 3 3ø
3 2 1
P(E Ç F )(1 - P(F )) + P(E Ç F )P(F ) = - =
= 4 3 12
P (F )P (F )

[ P(E Ç F ) - P(E Ç F )]P(F ) + P(E Ç F ) A B


= ¹1
P(F )P(F )
Hence (B) is not correct.
(C) We have

P(E / F ) + P(E / F ) C

P(E Ç F ) P(E Ç F )
= + FIGURE 7.2 Single correct choice type question 20.
P(F ) P(F )
Answer: (A)
P(E Ç F )P(F ) + P(E Ç F )P(F )
=
P(F )P(F ) 21. If three distinct natural numbers are chosen randomly
from the first 100 natural numbers, then the probability
P(E Ç F )(1 - P(F )) + P(E Ç F )P(F ) that all three of them are divisible by both 2 and 3 is
=
P(F )P(F ) 4 4 4 4
(A) (B) (C) (D)
P(F )[ P(E Ç F ) - P(E Ç F )] + P(E Ç F ) 25 35 33 1155
= ¹1
P(F )P(F ) Solution: A number is divisible by 6, only when it is
So (C) is not correct. divisible by both 2 and 3. Therefore the number of multi-
ples of 6 (<100) is 16 (they are 6, 12, 18, …, 96).
516 Chapter 7 Probability

From 100 numbers three are selected in 100C3 ways. 23. A natural number x is selected at random from the
From 16 numbers 3 are selected in 16C3 ways. first 100 natural numbers. The probability that
Therefore probability is x + (100 / x) > 50 is
16
C3 16 3 197 11 9 13 3
= ´ (A) (B) (C) (D)
100
C3 3 13 100 20 20 20 4
Solution: We have
16 ´ 15 ´ 14 4
= = 100
100 ´ 99 ´ 98 1155 x+ > 50
x
Alternate Solution
Since the three numbers are distinct, we can select the Û ( x - 25)2 > 525
three one after other without replacement. Therefore the Û | x - 25| > 22
probability is
Hence x < 3 or x > 47. So the number of favourable
16 ´ 15 ´ 14 4
= cases to x is 2 + 53 = 55. The required probability is
100 ´ 99 ´ 98 1155
55 11
Answer: (D) =
100 20
22. Let A, B and C be mutually independent events. Answer: (A)
Consider the following two statements.
Statement I: A and B È C are independent. 24. The first 12 letters of English alphabet are written
Statement II: A and B Ç C are independent. in a row at random. The probability that there are
Then, exactly four letters in between A and B is
(A) both I and II are true 5 1 7 1
(A) (B) (C) (D)
(B) only I is true 66 22 66 11
(C) only II is true Solution: A and B can be arranged in 12P2 = 11 ´ 12
(D) both I and II are false ways. Since we want 4 letters in between A and B, the
order of the four letters appearing has no importance.
Solution: We know that A and B can take the following places.
A Ç ( B È C ) = ( A Ç B) È ( A Ç C )
Place for A Place for B
Now
1 6
P( A Ç ( B È C )) 2 7
= P( A Ç B) + P( A Ç C ) - P( A Ç B Ç C ) 3 8
4 9
= P( A)P( B) + P( A)P(C ) - P( A)P( B)P(C )
5 10
= P( A)[ P( B) + P(C ) - P( B)P(C )] 6 11
= P( A)[ P( B) + P(C ) - P( B Ç C )] 7 12
= P( A)P( B È C )
A and B can be interchanged. Therefore required prob-
Therefore Statement I is true. Again ability is
P( A Ç ( B Ç C )) = P( A)P( B)P(C ) 14 7
=
= P( A)P( B Ç C ) 11 ´ 12 66
Alternate Solution
Hence A and B Ç C are independent. So Statement II is
12 letters can be arranged in 12 ways. In between
also true.
A and B, four letters can be arranged in 10P4 ways. Now
Answer: (A) 6 letters for whom A and B are at extreme positions
together with 6 other letters can be arranged in 7 ways.
Worked-Out Problems 517

Further A and B can be interchanged. Therefore the 27. Each of the letters A, B, C, D, E and F are, respec-
required probability is tively, written on six cards (one letter on one card only)
and they are well-shuffled and then the top four cards
2 ´ 10 P4 ´ 7 2 ´ 10 ´ 7 are turned face up without changing their order. Then
=
12 6 ´ 12 the probability that they form the word “DEAF” is
2´7 7 1 1
= = (A) (B) (C) 0.03 (D) 0.13
11 ´ 12 66 120 360
Answer: (C)
Solution: Total number of arrangements by cards = 6P4 =
360. In only one way, the letters form the word DEAF.F
25. If the letters of the word UNIVERSITY are arranged
Therefore
in a random order, the probability that all the vowels
come together is 1
Probability =
1 2 1 1 360
(A) (B) (C) (D)
15 15 5 30 Answer: (B)
Solution: The word UNIVERSITY consists of 10
28. A boy remembers all but the last digit of his girlfriend’s
letters out of which the vowels are e, i, i, u. The 10 letters
mobile number. He randomly chooses a digit from
can be arranged in 10 / 2 ways (since there two identical
0 to 9 (including 0 and 9). If he attempts two times, the
letters i). Since all the vowels are to be together, treat all
probability that he reaches her at least once is
the four written as a single object. The 6 + 1 = 7 objects
can be permuted in (A) 0.2 (B) 0.3 (C) 0.02 (D) 0.03
Solution: Let
æ 4ö
7 ç ÷ ways E = Event that the boy makes correct dial at least once.
è 2ø
E = Event of failing in both attempts.
(since 4 vowels can be arranged among themselves in In the first attempt, the probability of failing is 9/10. In the
4 / 2 ways). Therefore required probability is second attempt, the probability of failing is 8/9. Therefore
7( 4 / 2) 24 1 1 9 8 4
= = = P(E ) = ´ =
10 / 2 8 ´ 9 ´ 10 3 ´ 10 30 10 9 5
Answer: (D) 4 1
P(E ) = 1 - P(E ) = 1 - = = 0.2
5 5
26. A cubical die has four blank faces, one face marked Answer: (A)
2 and another marked 3. If the die is thrown 5 times,
the probability that the sum is 12 is 29. The probability of the birth dates of all 6 persons to
5 5 5 5 fall in only two different months is
(A) (B) (C) (D)
1296 1294 2596 2592 341 341 341 541
(A) (B) (C) (D)
Solution: Total number of outcomes is 126 125 124 126
Solution: Since the birth date of any person can fall in
6 ´ 6 ´ 6 ´ 6 ´ 6 = 65
anyone of the 12 months, the number of total outcomes
Let is 126. Let
E = sum of the faces is 12. E: Event that the birth dates of all 6 fall in two different
Favourable cases to E : 2 three times, 3 two times or months.
3 four times. This is possible in Number of possibilities of 6 persons’ birth dates to fall
in two different months (say February and March) is
5 5 26 -2 (this 2 corresponds to either all their birth dates fall
+ ways
23 4 in February or all fall in March). The number of ways
that we can select 2 months out of 12 is 12C2. Therefore
Therefore
12
C2 (26 - 2) 66 ´ 62 11 ´ 31 341
10 + 5 15 5 5 P(E ) = = = = 5
P(E ) = = 4 = = 126 126 125 12
65
6 ´6 2´6 4
2592
Answer: (B)
Answer: (D)
518 Chapter 7 Probability

30. A is one of the six horses entered for a race and So


one of the two jockeys B and C ride it. If B rides A,
then all the six horses are equally likely to win. If P(W2 ) = P(W2 Ç W1 ) + P(W2 Ç B1 ) (∵ B1 Ç W1 = f )
C rides A, then chances of A’s win will be trebled.
Therefore by Theorem 7.3
Then, the odds in favour of A’s win is
(A) 2 :1 (B) 3 : 2 (C) 1: 2 (D) 2 : 3 P(W2 ) = P(W1 )P(W2 /W1 ) + P( B1 )P(W2 / B1 )
Solution: Let m m+k n m
= × + ×
A = Event of A’s win. m+n m+n+k m+n m+n+k
B = Event of B riding A.
C = Event of C riding A. m(m + k ) + mn
=
(m + n)(m + n + k )
Then
m(m + k + n) m
P( A / B) =
1 = =
6 (m + n)(m + n + k ) m + n

æ 1ö 1 Answer: (C)
and P ( A /C ) = 3 ç ÷ =
è 6ø 2
32. Let A, B and C be three events such that p = P(exactly
Now one of A or B) = P(exactly one of B or C) = P(exactly
A = A Ç ( B È C ) = ( A Ç B) È ( A Ç C ) one of C or A) and P(A, B, C simultaneously) = p2
where 0 < p < 1/ 2. Then P(at least one of A, B or C)
Since B and C are mutually exclusive we have is equal to
P( A) = P( A Ç B) + P( A Ç C ) 3 p + 2 p2 2 p + 3 p2
(A) (B)
2 2
= P( B)P( A / B) + P(C )P( A / c)
2 p + 3 p2 3 p + 2 p2
1 1 1 1 1+ 3 1 (C) (D)
= ´ + ´ = = 4 4
2 6 2 2 12 3
( - B) È (B - A).
Solution: Exactly one of Aor Bmeans (A
Therefore By hypothesis
1 2 P(( A - B) È ( B - A)) = p
P( A) = 1 - =
3 3
P(( B - C ) È (C - B)) = p
Hence odds in favour (see Note under Theorem 7.1) is
P(E): P(E) = 1 : 2. P((C - A) È ( A - C )) = p
Answer: (C)
Therefore
31. An urn contains m white balls and n black balls. p = P(( A - B) È ( B - A)) = P( A - B) + P( B - A)
A ball is drawn at random and put back into the urn
along with k additional balls of the same colour as = [ P( A) - P( A Ç B)] + [ P( B) - P( B Ç A)]
that of the drawn ball. A ball is again taken out at
So
random. The probability that it is white is
m n P( A) + P( B) - 2 P( A Ç B) = p (7.4)
(A) (B)
m+n m+n
Similarly
m+n+k mn
(C) (D)
mn + k mn + k P( B) + P( B Ç C ) - 2 P( B Ç C ) = p (7.5)
Solution: Let and P(C ) + P( A) - P(C Ç A) = p (7.6)
W1 = Event of drawing a white ball in the first draw. Adding Eqs. (7.4)–(7.6), we have
B1 = Event of drawing black ball in the first draw.
W2 = Drawing white ball in the second draw. 2[ P( A) + P( B) + P(C ) - P( A Ç B)
Now
- P( B Ç C ) - P(C Ç A)] = 3 p (7.7)
W2 = W2 Ç (W1 È B1 ) = (W2 Ç W1 ) È (W2 Ç B1 )
Worked-Out Problems 519

Now P(atleast one of A, B or C) is given by [see part (3), Solution: Let


Theorem 7.2] E1 = Event of the rupee coin transferred from A to B and
coming to A.
P( A È B È C ) = P( A) + P( B) + P(C ) - P( A Ç B) E2 = Event of rupee coin does not get transferred from A.
- P( B Ç C ) - P(C Ç A) + P( A Ç B Ç C ) E = Event of the rupee coin is still in A.
Clearly E = E1 È E2 and E1, E2 are mutually exclusive.
3p Therefore
= + p2 [Eq. (7.7) and P( A Ç B Ç C ) = p2 ]
2
P(E) = P(E1 ) + P(E2 )
3 p + 2 p2
=
2
9
C8 ´ 1C1 18C8 ´ 1C1 9C9
= 10
´ 19 + 10
Answer: (A) C9 C9 C9
9 18 9 ´ 10 1
33. A and B are two events with positive probabilities. = ´ ´ +
10 8 ´ 10 19 10
Consider the following two statements.
Statement I: P( A / B) = 1 - P( A / B). =
9
´
9
+
1
Statement II: P( A È B) = 1 - P( A)P( B / A). 10 19 10
Then 81 + 19 10
(A) both I and II are true (B) I and II are false = =
10 ´ 19 19
(C) only I is true (D) only II is true Answer: (B)
Solution:
35. A set S contains n elements. A subset P of S is selected
(1) We have
at random and after inspecting the elements, they
P( A Ç B) are put back into S and then a subset Q is selected
P( A / B) = at random. The probability that P and Q are disjoint
P( B) subsets is equal to
P( B) - P( A Ç B) æ 2ö
n
æ 1ö
n
æ 1ö
n
æ 3ö
n
= (A) ç ÷ (B) ç ÷ (C) ç ÷ (D) ç ÷
P( B) è 3ø è 4ø è 3ø è 4ø
[∵ B = ( B - A) È ( A Ç B) = ( B Ç A) È ( A Ç B)] Solution: Since a set containing n has 2n subsets,
P( A Ç B) the total number of outcomes = 2n ´ 2n = 4n. Suppose
=1- P contains “r” elements. Then, Q will have 2n-rr choices
P( B)
because P Ç Q = f. For each selection of an r-element
= 1 - P( A / B) set P, there correspond 2n-rr choices for Q. Therefore the
total number of choices for P and Q with P Ç Q being
Hence Statement I is true.
empty set is (since P can be selected in nCr ways)
(2) By De’Morgans law we have
n

P( A È B) = P( A Ç B) å
r =0
n
Cr 2n-r = nC0 2n + nC1 2n-1 + nC2 2n- 2 +  + nCn 20

= 1 - P( A Ç B) = (2 + 1)n = 3n
= 1 - P( A)P( B / A) (Theorem 7.3) Therefore the required probability is
Therefore Statement II is also true. Hence both I n
3n æ 3 ö
and II are true. =ç ÷
4n è 4 ø
Answer: (A)
Answer: (D)
34. Purse A contains 9 coins of 50 paise denomination and
a one rupee coin. Another purse B contains 10 coins 36. n + 1 consecutive natural numbers, if three
of 50 paise denomination. 9 coins are selected at numbers are selected at random, the probability that
random from A and transferred to B. Again 9 coins they are in Arithmetic progression is
are selected at random from B and transferred to A. 3n 3n
(A) (B)
The probability that the rupee coin is still in purse A is 4 n2 - 1 4 n2 + 1
11 10 9 7 2n 2n
(A) (B) (C) (D) (C) (D)
19 19 19 19 4 n2 - 1 4 n2 + 1
520 Chapter 7 Probability

Solution: Three numbers out of 2n + 1 numbers can be Hence


selected in
1 4
P(E ) = 1 - P(E ) = 1 - =
( 2 n + 1) n(4 n2 - 1) 5 5
C3 = ways
3 Answer: (D)
Let a, b, c be three numbers from the given 2n + 1
numbers that are in AP. We need not distinguish Try it out Find the fault in the following argu-
between a, b, c and c, b, a. ment. One number can be selected from {1, 2, 3, 4}
a, b, c are in AP Û a + c = 2b and the other from S = {1, 2, 3, 4, 5, 6}. Therefore
total number of favourable cases to E is 4 ´ 6 = 24
Therefore either both a and c are even or both are odd, and hence
because b is fixed as per the relation
24 4
P(E ) = =
a + c = 2b 30 5
Now 2n + 1 consecutive numbers contain n even and
n + 1 odd or n + 1 even and n odd numbers. Therefore 38. Two numbers are selected at random from the natu-
the total number of choices for a and c is rals 1, 2, 3, …, 100 and are multiplied together. The
probability that the product thus obtained is divis-
n
C2 + ( n+1)C2 = n2 ible by 3 (upto two places of decimals) is
Hence required probability is (A) 0.65 (B) 0.55 (C) 0.45 (D) 0.35

n2 3n Solution: Two numbers can be selected from 100 in


= 100
C2 = 99 ´ 50 ways. A product of two positive integers is
[n(4 n2 - 1)]/ 3 4 n2 - 1
divisible if and only if at least one of them is divisible by
Answer: (A) 3 (since 3 is a prime).
Let E be the event that the product is divisible by 3.
37. Let a and b be the selected numbers. Therefore 3 divides
S = {1, 2, 3, 4, 5, 6} one by one without replacement. ab if and only if either 3 divides a or 3 divides b. Among
The probability that minimum of the numbers is less the numbers 1, 2, 3, …, 100, there are 33 multiples of 3.
than 4 is Now E = Event of none of a and b is divisible by 3 so
1 14 1 4 that there 67C2 favourable cases to E. Therefore
(A) (B) (C) (D)
15 15 5 5
67
C2
Solution: Two numbers can be selected one after other P(E ) = 100
C2
without replacement in 6 × 5 = 30 ways. Let
E = The minimum of the two is less than 4. 67 ´ 33
=
The favourable cases to E are 99 ´ 50
(1, 2) (1, 3), (1, 4), (1, 5), (1, 6)
(2, 1), (2, 3), (2, 4), (2, 5), (2, 6) 67
=
(3, 1), (3, 2), (3, 4), (3, 5), (3, 6) 150
(4, 1), (4, 2), (4, 3)
Hence
(5, 1), (5, 2), (5, 3)
(6, 1), (6, 2), (6, 3) 83
P(E ) = 1 - P(E ) = = 0.55
Therefore number of favourable cases to E is 24. The 150
required probability is
Answer: (B)
24 4
P(E ) = = 39. There are four machines and it is known that exactly
30 5
two of them are faulty. They are tested one by one,
Alternate Solution in a random order till both the faulty machines are
E is the event that both numbers must be different and identified. The probability that only two tests are
among 4, 5, 6. The number of favourable cases to E is needed is
3 × 2 = 6. Therefore 1 1 1 1
(A) (B) (C) (D)
6 1 3 6 2 4
P(E ) = =
30 5
Worked-Out Problems 521

Solution: The procedure ends in first two tests if either and hence
both are faulty or both are good. Therefore the prob-
5 13
ability is P( A Ç B) = 1 - =
18 18
2 1 2 1 1
´ + ´ = Now
4 3 4 3 3
Answer: (A) P( A) = P( A) + P( A Ç B Ç A)

40. Let A = {1, 2, 3, 4} and B = {x1, x2} be two sets. If + P( A Ç B Ç A Ç B Ç A) +  + ¥


a function from A to B is selected at random, the = P( A) + P(( A È B) Ç A)
probability that it is a surjection is
1 2 3 7 + P(( A È B) Ç ( A È B) Ç A) +  + ¥
(A) (B) (C) (D)
2 3 5 8 = P( A) + P( A È B) × P( A) + ( P( A È B))2 +  + ¥
Solution: Total number of functions from A to B 2
1 13 1 æ 13 ö 1
is 24 = 16, out which 2 are constant functions and the = + × + ç ÷ × ++ ¥
remaining 14 are surjection. Hence the required prob- 9 18 9 è 18 ø 9
ability is 1 é 13 æ 13 ö
2
ù
= ´ ê1 + + ç ÷ +  + ¥ú
14 7 9 ëê 18 è 18 ø úû
=
16 8
1 1
Answer: (D) = ´
9 1 - (13 / 18)
41. A pair of dice is rolled together till the sum of the 18 2
= =
faces is either 5 or 7. The probability that 5 comes 9´5 5
before 7 is Answer: (B)
1 2 3 4
(A) (B) (C) (D)
5 5 5 5 42. Three faces of a fair die are yellow, two faces are
green and one face is blue. If the die is tossed three
Solution: Let A be the event that the sum is 5 and times, the probability that the colours yellow, green
B the event that the sum is 7. A and B are, respectively, and blue appear in the first, second and third toss,
their complements. Therefore respectively, is
A = A È ( A Ç B Ç A) È ( A Ç B Ç A Ç B Ç A) È  1 1 11 35
(A) (B) (C) (D)
36 12 12 36
For A, the favourable cases are
Solution: The probability of yellow in the first toss is
{(1, 4), (4, 1), (2, 3), (3, 2)} 3/6, the probability of green in the second toss is 2/6 and
the probability of blue in the third toss is 1/6. As these
For B, the favourable cases are events are independent, the required probability is
{(1, 6), (6, 1), (2, 5), (5, 2), (3, 4), (4, 3)} 3 2 1 1
´ ´ =
Therefore 6 6 6 36
Answer: (A)
4 1 6 1
P( A) = = and P( B) = =
36 9 36 6 43. A boy whose hobby is tossing a fair coin is to score
one point for every tail and 2 points for every head.
The boy goes on tossing the coin, till his score reaches
1 8 n or exceeds n where n ³ 2. If pn is the probability
P( A) = 1 - =
9 9 that his score attains exactly n, then pn is equal to
1 5 1
P( B) = 1 - = (A) pn-1 + pn- 2 (B) pn-1 + pn- 2
6 6 2
1 1 5 2 (-1)n 2n+1 + (-1)n
P( A È B) = P( A) + P( B) = + = (C) + n (D)
9 6 18 3 2 3× 2n
522 Chapter 7 Probability

Solution: As usual H denotes head and T denotes tail æ 1ö æ


2

so that = ç - ÷ ç pn - 2 - ÷
è 2ø è 3ø
1 3
P( H ) = = P(T ) æ 1ö æ 2ö
2 = ç - ÷ ç pn - 3 - ÷
è 2ø è 3ø
Let En denote the event that the score is n. One can 
easily see that n-1
æ -1ö æ 2ö
En = (En- 2 Ç H ) È (En-1 Ç T )
=ç ÷ çè p1 - ÷
è 2ø 3ø
n-1
Therefore æ 1ö æ 1 2ö
= ç- ÷ çè - ÷ø
è 2ø 2 3
pn = P(En )
= P(En- 2 Ç H ) + P(En-1 Ç T ) Hence
n-1
= P(En- 2 )P( H ) + P(En-1 )P(T ) 2 æ 1ö æ 1ö
pn = + ç- ÷ çè - ÷ø
1 3 è 2ø 6
= ( pn- 2 + pn-1 ) n
2 2 1 æ 1ö
= + ç- ÷
Now 3 3 è 2ø

1 1 2n+1 + (-1)n
pn + pn-1 = pn-1 + pn- 2 =
2 2 3× 2n
1 Answer: (D)
= pn- 2 + pn- 3
2
44. In Problem 43, the probability that his score is
1 exactly 5 is
= pn- 3 + pn- 4
2 21 21 65 31
(A) (B) (C) (D)
…………… 32 64 96 96
…………… Solution: In the formula,
1 2n+1 + (-1)n
= p2 + p1 pn =
2 3× 2n
Since put n = 5 so that
1
p1 = P(T ) = p5 =
63
=
21
2 3× 32 32
and p2 = P((T Ç T ) È H ) Answer: (A)
= P(T )P(T ) + P( H )
45. A pair of fair dice is rolled. The probability that the
1 1 1 3 sum is 9 given that 5 appeared on the first die is
= ´ + =
2 2 2 4 1 8 1 5
(A) (B) (C) (D)
we have 9 9 6 6

1 1 3 1 Solution: Let
pn + pn-1 = p2 + p1 = + = 1 E1 = Event of 5 occurs on the first die.
2 2 4 4
E2 = Sum of the faces is 9.
Therefore
Favourable cases to E1 are
2 1 1
pn - = - pn - 1 (5, 1), (5, 2), (5, 3), (5, 4), (5, 5) and (5, 6)
3 3 2
1æ 2ö Favourable cases to E2 are
= - ç pn - 1 - ÷
2è 3ø (5, 4), (4, 5), (3, 6) and (6, 3)
Worked-Out Problems 523

Therefore Table 7.6 Single correct choice type question 47


E1 Ç E2 = {(5, 4)} No. of values
“c” value 4c value b value(b2 ³ 4c) of b
Hence
1 4 2, 3, 4, 5, 6 5
P(E1 Ç E2 ) 2 8 3, 4, 5, 6 4
P(E2 / E1 ) =
P(E1 ) 3 12 4, 5, 6 3
æ 1ö æ 6ö 1 4 16 4, 5, 6 3
=ç ÷ ¸ç ÷ =
è 36 ø è 36 ø 6 5 20 5, 6 2
6 24 5, 6 2
Answer: (C)
Total 19
46. Two numbers x and y are selected at random from
the set of the first 3n natural numbers (n ³ 1) one Therefore required probability is 19/36.
after other without replacing the first drawn number. Answer: (C)
The probability that x2 - y2 is divisible by 3 is
4n - 1 2n - 3 48. In the game of cricket, an over means 6 balls.
(A) (B)
3(3n - 1) 3(3n - 1) A batsman can score 0 or 1 or 2 or 3 or 4 or 6 runs
per ball. In the last over of the match, a team needs
2n + 3 5n - 3
(C) (D) 30 runs to win. The probability to win the match is
3(3n - 1) 3(3n - 1) 70 51 91 71
(A) 6 (B) 6 (C) 6 (D) 6
Solution: Fermat’s theorem states that, if p is a prime 6 6 6 6
and x is any natural number, then either p divides x or
Solution: There are four possibilities to win.
xp-1 -1 is divisible by p. Hence, if x is a positive integer,
then either 3 divides x or x2 - 1 is divisible by 3. Similarly, (i) five sixes and a duck (means zero)
either 3 divides y or y2 - 1 is divisible 3. Observe that (ii) four sixes, a 2 and a 4
x2 - y2 = (x2 - 1) - (y2 - 1) is divisible by 3 if both x and y (iii) four sixes and two 3’s
are not multiples of 3.
(iv) three sixes and three fours
Among 1, 2, 3 …, 3n, there are n multiples of 3.
Among the multiples of 3, we can select two one after Table 7.7 gives the arrangements.
other without replacement in n(n - 1) ways. Out of
the remaining 2n numbers, we can select two one after Table 7.7 Single correct choice type question 48
other without replacement 2n (2n - 1) ways. Hence the
Case No. of arrangements
required probability is
(i) Five sixes 6
n(n - 1) + 2 n(2 n - 1) 5n - 3 and a 0 - 1 = 5 (∵ 666660 cannot occur)
= 5
3n(3n - 1) 3(3n - 1)
(ii) Four sixes, 6
Answer: (D) a 2 and a 4 = 30
4
47. The coefficients b and c of the quadratic expression (iii) Four sixes 6
and two 3’s = 15
x + bx + c will be determined by throwing a fair die. If
2
42
one such quadratic expression is selected at random, (iv) Three sixes 6
the probability that the equation x2 + bx + c = 0 has and three = 20
33
real roots is equal to fours
13 17 19 7 Total 70
(A) (B) (C) (D)
36 36 36 12
Solution: Total number of quadratic expressions Therefore the required probability is
formed is 6 ´ 6. The equation x2 + bx + c = 0 has real
70
roots if b2 ³ 4c. Table 7.6 gives you the number of values
taken by b and c such that b2 ³ 4c. 66
(Since for each ball the batsman can score 0, 1, 2, 3, 4 or
6, the number of total cases is 66.)
Answer: (A)
524 Chapter 7 Probability

Now
Try it out Six-digit numbers are formed
f randomly
using the digits 0, 1, 2, 3, 4 and 6 with repetitions. If a 1
number is selected at random from these, what is the = P(E )
2
probability that the sum of the digits is 30 with 0 not in
the units place? = P( A)P( B)P(C ) + P( A)P( B)P(C ) + P( A)P( B)P(C )
(This question is another form of Question 48.) æ 1ö æ 1ö æ 1ö
= pq ç ÷ + p(1 - q) ç ÷ + pq ç ÷
è 2ø è 2ø è 2ø
49. The number of throws with a single fair die required
Therefore
for a person such that his chance of getting the face
6 is 1/2 is pq + p(1 - q) + pq = 1
(A) 6 (B) 5 (C) 4 (D) 3 p + pq = 1
Solution: Let x be the number of chances required. Answer: (B)
1
E = Event of throwing 6 so that P(E) =
6 51. Four friends put their car keys on a table. When
Therefore they leave the place, they picked up their keys at
random. The probability that no person picks his
1 5 own key is
P(E ) = 1 - =
6 6 3 1 5 3
(A) (B) (C) (D)
x 8 4 8 4
æ 5ö
Probability of not throwing 6 in all the x chances = ç ÷
è 6ø Solution: This problem is nothing but derangement
Therefore x
problem (See Theorem
T 6.20 and Try
T it out, page 295,Vol.
V 1).
æ 5ö 1 The number of derangements of 4 distinct elements is
Probability of throwing 6 at least once = 1 - ç ÷ =
è 6ø 2
This implies æ 1 1 1 1ö
4 ç 1 - + - + ÷ = 12 - 4 + 1 = 9
è 1 2 3 4ø
log(1/ 2)
x= = 3.8 approximately
log(5 / 6)

Therefore the number of chances required is 4. 9 9 3


= =
Answer: (C) 4 24 8
Answer: (A)
50. A person is assigned three works A, B and C. The
probabilities of the person finishing the works A, B 52. An unbiased coin is tossed. If the result is head, then
and C are, respectively, p, q and 1/2. The person will a pair of fair dice is rolled and the number obtained
get full payment only if he either completes atleast by adding the numbers on the faces is noted. If the
the works A and B or A and C. If the probability of result is a tail, then a card is picked from a well-shuf-
his getting full payment is 1/2, then fled pack of 11 cards numbered 2, 3, 4, …, 12 and
(A) p + q = pq (B) p + pq = 1 the number is noted. The probability that the noted
2 1 number is either 7 or 8 is
(C) p + pq = (D) p - pq = 183 193 173 153
3 2 (A) (B) (C) (D)
792 792 792 792
Solution: Let A, B and C denote the events of the
person finishing the works A, B and C, respectively. By Solution: Let H and T be the events of head and tail,
hypothesis respectively. E is the event of the noted number is 7 or 8.
Then
1
P( A) = p, P( B) = q and P(C ) = E = E Ç ( H È T ) = (E Ç H ) È (E Ç T )
2
Let E be the event of the person getting full payment. and E Ç H, E Ç T are mutually exclusive. Therefore
Therefore P(E ) = P(E Ç H ) + P(E Ç T )
E = (A Ç B Ç C) È (A Ç B Ç C) È (A Ç B Ç C) = P(H )P(E /H ) + P(T )P(E /T ) (Theorem 7.3) (7.8)
Worked-Out Problems 525

When a pair of dice is rolled, the favourable cases to the 54. The probability that India winning a hockey match
event of the sum is 7 or 8 are against Pakistan is 1/2. In a 5-match series, India
surely wins the third match is
(1, 6), (6, 1), (2, 5), (5, 2), (3, 4),
1 1 2 1
(4, 3), (4, 4) (5, 3), (3, 5), (2, 6), (6, 2) (A) (B) (C) (D)
8 4 3 2
Therefore Solution: India winning the third match is independent
11 2 of the results of the first two matches. Hence, the prob-
P(E / H ) = and P(E /T ) = ability that India surely winning the third match is 1/2.
36 11
Answer: (D)
Therefore from Eq. (7.8),
55. The probability of India winning a test match against
1 11 1 2
P(E ) = ´ + ´ England is 1/2. In a five match series, the probability
2 36 2 11 that India registers its second win in the third test is
121 + 72 193 (you can assume the independence from match to
= =
72 ´ 11 792 match)
Answer: (B) 1 1 1 2
(A) (B) (C) (D)
8 2 4 3
53. The odds in favour of a book reviewed by three
independent critics are, respectively, 5 : 2, 4 : 3 and Solution: India has to win one of the first two tests and
3 : 4. The probability that majority of the critics give a win in the third test. Therefore required probability is
favourable remark is P(W Ç L Ç W ) + P(L Ç W Ç W ) = P(W )P(L)P(W )
210 209 211 205 + P(L)P(W )P(W )
(A) (B) (C) (D)
343 343 343 343
1 1 1 1 1 1
= ´ ´ + ´ ´
Solution: Let E1, E2 and E3 be the events of the critics 2 2 2 2 2 2
giving favourable remarks. Then 1
=
5 4 3 4
P(E1 ) = , P(E2 ) = and P(E3 ) =
7 7 7 where W denotes win and L denotes loss.
E be the event that majority reviewed favourably. Answer: (C)
Therefore
56. A person has to go through three successive tests.
E = (E1 Ç E2 Ç E3 ) È (E1 Ç E2 Ç E3 ) The probability of his passing first test is p. If he fails
È (E1 Ç E2 Ç E3 ) È (E1 Ç E2 Ç E3 ) in one of the tests, then the probability his passing
next test is p/2, otherwise it remains the same. For
Hence selection, the person must pass atleast two tests. The
probability that the person to be selected is
P(E) = P(E1 )P(E2 )P(E3 ) + P(E1 )P(E2 )P(E3 )
(A) p2 + p3 (B) 22p2 - p3
+ P(E1 )P(E2 )P(E3 ) + P(E1 )P(E2 )P(E3 ) 1 3
(C) p2 - 2p
2 3 (D) p2 + p
5 4 æ 3ö æ 5ö 4 3 5 2
= ´ ´ ç1 - ÷ + ç1 - ÷ ´ ´ +
7 7 è 7ø è 7ø 7 7 7 Solution: Let Ei (i = 1, 2, 3) be the event of the person
æ 4ö 3 5 4 3 passing the ith test and E is the event that he is selected.
´ ç1 - ÷ ´ + ´ ´ Then
è 7ø 7 7 7 7
5 4 4 2 4 3 5 3 3 5 4 3 E = (E1 Ç E2 ) È (E1 Ç E2 Ç E3 ) È (E1 Ç E2 Ç E3 )
= ´ ´ + ´ ´ + ´ ´ + ´ +
7 7 7 7 7 7 7 7 7 7 7 7 Therefore
80 + 24 + 455 + 60 P(E) = P(E1 )P(E2 / E1 ) + P(E1 )P(E2 / E1 )P(E3 / E2 )
=
7´7´7
+ P(E1 )P(E2 / E1 )P(E3 / E2 )
209
= p p
343 = p × p + p(1 - p) + (1 - p) × p
2 2
Answer: (B)
= 2 p2 - p3
Answer: (B)
526 Chapter 7 Probability

57. A and B are independent events and C is the event It is given that
in which exactly one of A or B occurs. Then
1
(A) P(C ) £ P( A È B)P( A Ç B) P(defective screw of A) =
10
(B) P(C ) ³ P( A È B)P( A Ç B)
1
(C) P(C ) = P( A È B)P( A Ç B) and P(defective screw of B) =
5
(D) P(C) = P(A È B) P(A Ç B)
Therefore
Solution: We have
P(E) = P(E1 ) + P(E2 ) + P(E3 ) + P(E4 ) + P(E5 ) (7.10)
C = ( A - B) È ( B - A) = ( A Ç B) È ( A Ç B)
Now
Since A and B are independent, the pairs ( A, B), ( A, B)
and ( A, B) are also pairs of independent events. Further 4
C1 ´ 5C1 1 4 2
A Ç B and A Ç B are mutually exclusive. Now, let P(E1 ) = 9
´ ´ =
C2 10 5 45
P(A) = x and P(B) = y. Therefore
5
C1 ´ 4 C1 1 9 1
P(C ) = P( A Ç B) + P( A Ç B) P(E2 ) = 9
´ ´ =
C2 5 10 10
= P( A)P( B) + P( A)P( B) 5
C1 ´ 4 C1 1 1 1
P(E3 ) = 9
´ ´ =
= x(1 - y) + (1 - x) y (7.9) C2 10 5 90
4 2
Also C2 æ 1 ö 1
P(E4 ) = ´ç ÷ =
9
C2 è 10 ø 600
P(A È B)P(A Ç B) = [ P(A) + P(B) - P(A Ç B)]P(A)P(B)
5 2
C2 æ 1 ö 1
= ( x + y - xy)(1 - x)(1 - y) P(E5 ) = ´ç ÷ =
9
C2 è 5 ø 90
= ( x + y)(1 - x)(1 - y) - xy(1 - x)(1 - y)
Substituting these values in Eq. (7.10), we have
£ ( x + y)(1 - x)(1 - y) [∵ xy(1 - x)(1 - y) ³ 0]
303
= x(1 - x)(1 - y) + y(1 - x)(1 - y) P(E ) = = 0.168 (approximately)
1800
= x(1 - y) - x2 (1 - y) + y(1 - x) - y2 (1 - x) Answer: (A)

= x(1 - y) + y(1 - x) - [ x2 (1 - y) + y2 (1 - x)] 59. If a circle is selected at random touching all the sides
of a triangle, then the probability that it touches two
£ x(1 - y) + y(1 - x) = P(C ) [by Eq
q. (7.9)]
sides externally and one side internally is
Answer: (B)
1 3 1 2
(A) (B) (C) (D)
4 4 2 3
58. A factory A produces 10% of defective screws and
another factory B produces 20% of defective screws. Solution: It is known that there are four circles touching
A bag contains 4 screws of factory A and 5 screws of all the three sides of a triangle out of which one is the
factory B. If two screws are drawn at random from incircle and the other three are ex-circles. Therefore the
the bag, then the probability that atleast one screw is required probability is 3/4.
defective (correct to three places of decimals) is Answer: (B)
(A) 0.168 (B) 0.158 (C) 0.165 (D) 0.155
Solution: Let E be the event that atleast one screw is 60. Three groups A, B and C are competing for the posi-
defective. Then E can be regarded as the union of the tions on the Board of Directors of a company. The
following events. probabilities of their winning are 0.5, 0.3 and 0.2,
respectively. If the group A wins, the probability of
E1 = one defective from A and one non-defective from B.
introducing a new product is 0.7 and other corre-
E2 = one defective from B and one non-defective from A. sponding probabilities for groups B and C are, respec-
E3 = one defective from A and one defective from B. tively, 0.6 and 0.5. The probability that new product
E4 = both defective from A. will be introduced is
E5 = both defective from B. (A) 0.43 (B) 0.53 (C) 0.63 (D) 0.73
Worked-Out Problems 527

Solution: Let A, B and C be the events winning of A,B and 62. Bag A contains 5 red and 7 white balls and bag B
C, respectively. E is the introduction of new product. Then contains 3 red and 12 white balls. One of the bags
is selected at random and one ball is drawn from it.
E = E Ç ( A È B È C ) = (E Ç A) È (E Ç B) È (E Ç C ) The probability that the drawn ball is red is
Therefore 37 83 63 17
(A) (B) (C) (D)
P(E) = P(E Ç A) + P(E Ç B) + P(E Ç C ) 120 120 120 120

= P( A)P(E / A) + P( B)P(E / B) + P(C )P(E /C ) Solution: Let A and B denote the events of selecting
bags A and B, respectively, and R denote drawing a red
5 7 3 6 2 5 ball. Then
= ´ + ´ + ´
10 10 10 10 10 10
R = ( A È B) Ç R = ( A Ç R) È ( B Ç R)
63
= = 0.63 Therefore
100
Answer: (C) P(R) = P( A Ç R) + P( B Ç R)
= P( A)P(R / A) + P( B)P(R / B)
61. A box contains m green balls and n yellow balls. It is
given that the probability of drawing 2 yellow balls 1 5 1 3
= ´ + ´
from the bag is 5 times the probability of drawing 2 2 12 2 15
green balls. Also, the probability of drawing 1 ball
1æ 5 3ö
of each colour is equal to 6 times the probability of = ç + ÷
drawing 2 green balls. Then the pair (m, n) is equal to 2 è 12 15 ø
(A) (6, 3) (B) (3, 5) (C) (4, 6) (D) (3, 6) 25 + 12 37
= =
2 ´ 60 120
Solution: Let GK and YK be the events of drawing K
green balls and K yellow balls. Therefore Answer: (A)

n(n - 1) 63. On each evening a boy either watches


P(Y2 ) = nC2 ¸ ( m+ n)C2 =
(m + n)(m + n - 1) DOORDARSHAN channel or TEN SPORTS. The
probability that he watches TEN SPORTS is 4/5. If
m(m - 1)
P(G2 ) = mC2 ¸ ( m+ n)C2 = he watches DOORDARSHAN, there is a chance of
(m + n)(m + n - 1) 3/4 that he will fall asleep, while it is 1/4 when he
2 mn watches TEN SPORTS. On one day, the boy is found
P(G1 Ç Y1 ) = (mC1 ´ nC1 ) ¸ ( m+ n)C2 = to be asleep. The probability that the boy watched
(m + n)(m + n - 1)
DOORDARSHAN is
Now by hypothesis 5 2 3 4
(A) (B) (C) (D)
P(Y2 ) = 5P(G2 ) Þ n(n - 1) = 5m(m - 1) (7.11) 7 7 7 7

and P(G1 Ç Y1 ) = 6 P(G2 ) Þ 2 mn = 6 m(m - 1) (7.12) Solution: Let E1 and E2 be the events of the boy
watching DOORDARSHAN and TEN SPORTS, respec-
From Eq. (7.12), we have n = 3(m -1). Put this value of n tively. It is given that
in Eq. (7.11) so that
1 4
3(m - 1)(3m - 4) = 5m(m - 1) P(E1 ) = and P(E2 ) =
5 5
4 m2 - 16 m + 12 = 0 E be the event of the boy falls asleep. Again by
(m - 1)(m - 3) = 0 hypothesis
m = 1, 3 3 1
P(E / E1 ) = and P(E / E2 ) =
4 4
Now
Now
m n = 3m - 3
1 0 E = E Ç (E1 È E2 ) = (E1 Ç E) È (E2 Ç E)
3 6 so that
Since n ¹ 0, the pair (m, n) = (3, 6). P(E) = P(E1 )P(E / E1 ) + P(E2 )P(E / E2 )
Answer: (D)
528 Chapter 7 Probability

By Bayes’ theorem (Theorem 7.7) H2: Second toss head.


P(E1 )P(E / E1 ) E1: Selecting fair coin.
P(E1 / E) = E2: Selecting two headed coin.
P(E1 )P(E / E1 ) + P(E2 )P(E / E2 )
Then
(1/ 5) ´ (3 / 4) 3
= =
(1/ 5) ´ (3 / 4) + (4 / 5) ´ (1/ 4) 7 H2 = (E1 Ç H1 Ç H2 ) È (E1 Ç T1 Ç H2 ) È (E2 Ç H1 Ç H2 )
Answer: (C) Therefore

64. The chance that doctor A will diagnose disease X P( H2 ) = P(E1 Ç H1 )P( H2 /(E1 Ç H1 )) + P(E1 Ç T1 )
correctly is 60%. The chance that a patient of doctor P( H2 /(E1 Ç H1 )) + P(E2 Ç H1 )P( H2 /(E2 Ç H1 ))
A dies after correct treatment is 75% while it is 80%
after wrong diagnosis. A patient of doctor A having = P(E1 )P( H1 /E1 )P( H2 /(E1 Ç H1 )) + P(E1 )P(T1 /E1)
disease X dies. The probability that his disease is P( H2 /(E1 Ç T1 )) + P(E2 )P( H1 /E2 )
correctly diagnosed is
P( H2 /(E2 Ç H1 )) (Corollary 7.1)
8 9 11 6
(A) (B) (C) (D)
17 17 17 17 1 1 1 1 1 1 1
= ´ ´1+ ´ ´ + ´1´
2 2 2 2 2 2 2
Solution: Let A denote the event of correct diagnosis
and E the event of patient’s death. It is given that 1 1 1 5
= + + =
4 8 4 8
60 40
P( A) = , P( A) = , Answer: (A)
100 100
75 80 66. In a test, an examinee either guesses or copies or
P(E / A) = and P(E / A) =
100 100 knows the answer for a multiple choice question
having FOUR choices of which exactly one is correct.
By Bayes’ theorem (Theorem 7.7)
The probability that he makes a guess is 1/3 and the
P( A)P(E / A) probability for copying is 1/6. The probability that
P( A / E ) = his answer is correct, given that he copied it is 1/8.
P( A)P(E / A) + P( A)P(E / A)
The probability that he knew the answer, given that
60 75 his answer is correct is
´
= 100 100 5 9 24 20
60 75 40 80 (A) (B) (C) (D)
´ + ´ 29 29 29 29
100 100 100 100
3 3 Solution: Let the events be defined as
´ E1: Guessing
= 5 4
3 3 2 4 E2: Copying
´ + ´
5 4 5 5 E3: Knowing
9 E: Correct answer
=
17 By hypothesis,
Answer: (B)
1 1 1 1 1
P(E1 ) = , P(E2 ) = , P(E3 ) = 1 - - =
65. A person has two coins in his pocket of which one is 3 6 3 6 2
a fair coin and the other has heads on both the sides 1
P(E / E1 ) = (out of four choicces only one is correct)
(i.e., two headed coin). He selects one of the coins at 4
random and tosses. If head appears, then he will toss
1
the other coin otherwise he tosses the same coin. The P(E / E2 ) =
probability that head appears in the second toss is 8
5 3 7 1 P(E / E3 ) = 1
(A) (B) (C) (D)
8 8 8 8
Solution: Let P(E3)P(E /E3)
P(E3 /E) =
H1: First toss head. P(E1)P(E /E1) + P(E2)P(E /E2) + P(E3)P(E /E3)
T1: First toss tail.
Worked-Out Problems 529

1 Solution: Let the events be


´1
= 2 E1: Event of Mr. Iyyengar’s selection
1 1 1 1 1
´ + ´ + ´1 E2: Event of Dr. Singh’s selection
3 4 6 8 2
E3: Event of Prof. Mukherjee’s selection.
24
= E: Event of introduction of co-education.
29
By hypothesis,
Answer: (C)
P(E1 ) = 0.5, P(E2 ) = 0.3, P(E3 ) = 0.2
67. A letter is to come from either LONDONorCLIFTON.
and P(E / E1 ) = 0.5, P(E / E2 ) = 0.7, P(E / E3 ) = 0.6
The postal mark on the letter legibly shows consecutive
letters “ON”. The probability that the letter has come By Bayes’ theorem,
from LONDON is
P(E2)P(E /E2)
12 13 5 4 P(E2 /E) =
(A) (B) (C) (D) P(E1)P(E /E1) + P(E2)P(E /E2) + P(E3)P(E /E3)
17 17 17 17
(0.3)(0.7)
Solution: Let the events be defined as =
(0.5)(0.5) + (0.3)(0.7) + (0.2)(0.6)
E1: Letter coming from LONDON.
0.21 0.21 21
E2: Letter coming from CLIFTON. = = =
0.25 + 0.21 + 0.12 0.58 58
E: Two consecutive letters ON.
Answer: (B)
The word LONDON contains 5 types of consecutive
letters (LO, ON, ND, DO, ON) of which there are two
69. Bag A contains 3 white and 2 black balls. Bag B
ON’s. The word CLIFTON contains 6 types of consecu-
contains 2 white and 2 black balls. One ball is drawn
tive letters (CL, LI, IF, FT, TO, ON) of which there is
at random from A and transferred to B. One ball is
one “ON”. Now
selected at random from B and is found to be white.
1 The probability that the transferred ball is white is
P(E1 ) = = P(E2 )
2 8 5 4 9
(A) (B) (C) (D)
2 13 13 13 13
P(E / E1 ) =
5 Solution: Let E1 and E2 denote the events of the
1 transferred ball being white and black, respectively. W
and P(E / E2 ) = denotes the drawn ball from B is white. By hypothesis,
6
3
By Bayes’ theorem, C1 3
P(E1 ) = 5
=
C1 5
1 2
´ 2
2 5 12 C1 2
P(E1 / E) = = P(E2 ) = =
1 2 1 1 17 5
C1 5
´ + ´
2 5 2 6 3
C1 3
Answer: (A) P(W / E1 ) = 5
=
C1 5
68. Three persons Mr. Iyyengar, Dr. Singh and 2
C1 2
Prof. Mukherjee are competing for the post of the P(W / E2 ) = 5
=
C1 5
principal of a degree college exclusively meant
for boys. Their chances are, respectively, 0.5, 0.3 By Bayes’ theorem,
and 0.2. If Mr. Iyyengar is selected, he will introduce
co-education with probability 0.5 while the prob- P(E1 )P(W / E1 )
P(E1 /W ) =
abilities are 0.7 and 0.6 with regard to Dr. Singh P(E1 )P(W / E1 ) + P(E2 )P(W / E2 )
and Prof. Mukherjee, respectively. Co-education 3 3
is introduced in the college. The probability that ´
5 5 9
Dr. Singh is selected as principal is = =
3 3 2 2 13
31 21 27 37 ´ + ´
(A) (B) (C) (D) 5 5 5 5
58 58 58 58 Answer: (D)
530 Chapter 7 Probability

70. A bag contains 4 black, 2 white and 6 red balls. Solution: Let Bj be the number of black balls trans-
Another bag contains 3 black and 5 white balls. An ferred (j = 0, 1, 2, 3). B is the event of drawing a black
unbiased die is thrown. If either 1 or 2 appears, a ball ball. Therefore
is chosen from the first bag, otherwise a ball from 5
the second bag. If the drawn ball is black then the C4 5
P( B0 ) = 8
=
probability that 2 appeared on the die is C4 70
2 11 6 7
(A) (B) (C) (D) 3
C1 ´ 5C3 30
13 13 13 13 P( B1 ) = 8
=
C4 70
Solution: Let the events be
3
C2 ´ 5C2 30
E1: Event of 1 appearing. P( B2 ) = 8
=
C4 70
E2: Event of 2 appearing.
E3: E1 È E2 = E1 Ç E2.
3
C3 ´ 5C1 5
P( B3 ) = 8
=
B: Event of drawing a black ball. C4 70
Now Also
1 1 4 P( B / B0 ) = 0 (∵ no black ball is transferred)
P(E1 ) = , P(E2 ) = , P(E3 ) =
6 6 6
1
4 P( B / B1 ) =
P( B / E1 ) = 4
12
2
4 P( B / B2 ) =
P( B / E2 ) = 4
12
3
3 P( B / B3 ) =
P( B / E3 ) = 4
8
Therefore by Bayes’ theorem,
By Bayes’ theorem
P( B3 )P( B / B3 )
P(E2)P(B /E2) P( B3 / B) =
P(E2 /B) = 3
P(E1)P(B /E1) + P(E2)P(B /E2) + P(E3)P(B /E3) å P(B )P(B / B )
i =0
i i

1 4
´ 5 3
= 6 12 ´
1 4 1 4 4 3 = 70 4
´ + ´ + ´ 5 30 1 30 2 5 3
6 12 6 12 6 8 ´0+ ´ + ´ + ´
70 70 4 70 4 70 4
1
3 1 6 2 15 15 1
= = ´ = = = =
1 1 3 3 13 13 30 + 60 + 15 105 7
+ +
3 3 2 Answer: (D)
Answer: (A)
72. A person goes to office either by car, scooter, bus
71. Bag A contains 5 white and 3 black balls. Bag B is or train whose probabilities are, respectively, 1/7, 3/7,
empty. Four balls are taken at random from A and 2/7 and 1/7. The probability that he reaches office on
transferred to empty bag B. From B, a ball is drawn time, if he takes car, scooter bus or train is 7/9, 8/9, 5/9
at random and is found to be black. Then, the prob- and 8/9, respectively. Given that he reached office in
ability that among the transferred balls three are time, the probability that he travelled by car is
black and one is white is 1 2 6 5
(A) (B) (C) (D)
1 7 6 1 7 7 7 7
(A) (B) (C) (D)
8 8 7 7
Solution: Let E1, E2, E3 and E4 denote the events
of the person travelling by car, scooter, bus or train,
Worked-Out Problems 531

respectively. Let E denote the event of his reaching 74. If a natural number n is selected from the set {4, 5,
office in time. By hypothesis 6, …, 23}, then the probability that
1 3 2 1 x1 x2 x3 x4 + x2 x3 x4 x5 + x3 x4 x5 x6 +  + xn x1 x2 x3 = 0
P(E1 ) = , P(E2 ) = , P(E3 ) = , P(E4 ) =
7 7 7 7 where each of xj is either 1 or -1 is
7 8 1 2 1 3
P(E / E1 ) = , P(E / E2 ) = (A) (B) (C) (D)
9 9 5 5 4 4
5 8 Solution: Out of 20 numbers, one is selected in 20 ways.
P(E / E3 ) = , P(E / E4 ) =
9 9 Let E be the event that
By Bayes’ theorem x1 x2 x3 x4 + x2 x3 x4 x5 +  + xn x1 x2 x3 = 0
P(E1 )P(E / E1 ) where each xj = 1 or -1 (j = 1, 2, …, n). Let
P(E1 / E) = 4

å P(E )P(E / E )
j =1
j j y1 = x1 x2 x3 x4 , y2 = x2 x3 x4 x5 , yn = xn x1 x2 x3
so that
1 7
´ y1 + y2 + y3 +  + yn = 0
= 7 9
1 7 3 8 2 5 1 8
´ + ´ + ´ + ´ and each of yj = 1 or -1 (∵ xj = ±1). Therefore n must
7 9 7 9 7 9 7 9 be even say n = 2m. Among yj, m are +1 and m are -1.
7 1 Therefore
= =
49 7 y1 y2 y3  yn = (-1)m (1)m = (-1)m (7.13)
Answer: (A)
But
73. a is selected from the first 100 y1 y2 y3  yn = x14 x24 x34  xn4 = 1 (∵ xj = ± 1)
natural numbers. The probability that
From Eq. (7.13), (-1)m = 1 and m = 2K. So
éaù éaù éaù
a=ê ú+ê ú+ê ú n = 2m = 4K
ë2û ë3û ë5û
Therefore n is a multiple of 4. Among the numbers 4, 5,
where for any real x, [x] denotes the integral part of 6, …, 23 there are 5 multiples of 4. Hence
x, is
(A) 0.4 (B) 0.3 (C) 0.2 (D) 0.1 5 1
P(4) = =
20 4
Proof: The natural number “a” can be selected from
the 100 numbers in 100 ways. Let Answer: (C)

a = 30 n + r 75. “A” is an officer of a company and B is his subordi-


nate. B noticed that his officer A is happy on 60%
where 0 £ r < 30. Therefore
of his calls. B also noticed that if A is happy then A
ér ù ér ù ér ù accepts his request with probability 0.4, and when
15n + 10 n + 6 n + ê ú + ê ú + ê ú = 30 n + r A is not happy he accepts B’s request with proba-
ë2 û ë3û ë5û
bility 0.1. On one day B calls on A for a request and
ér ù ér ù ér ù A accepts it. The probability that A is happy is
n=r-ê ú-ê ú-ê ú
ë2û ë3û ë5û 1 6 3 4
(A) (B) (C) (D)
7 7 7 7
Now, for each value of r, there is exactly one value of
n and hence one value of a. Therefore the number of Solution: Let H be the event that A is happy when B
values of “a” satisfying the given conditions is 30. The calls on him and H the event that A is not happy. Let E be
required probability is the event that A accepts the request of B. By hypothesis,

30 60 3
= 0.3 P( H ) = =
100 100 5
Answer: (B) 2
P( H ) =
5
532 Chapter 7 Probability

4 77. Let S be the set of all matrices of order 3 ´ 3 whose


P(E / H ) =
10 elements are 1, 2, 3, 0, -1, -2, -3. If a matrix is selected
at random from S, the probability that it is skew-
1
P(E / H ) = symmetric is
10
1 2 1 2
(A) (B) (C) (D)
By Bayes’ theorem, 76 76 73 73
P( H )P(E / H ) Solution: Let A ÎS. Each element of A can be among
P( H / E ) =
P ( H )P (E / H ) + P ( H )P (E / H ) the given 7 elements. Hence the total number of matrices
in S is 79.
3 2 Let E be the event that A is skew-symmetric. For a
´
= 5 5 skew-symmetric matrix, the principal diagonal elements
3 2 2 1 must be zeros. Therefore, place 0 in the principal diag-
´ + ´
5 5 5 10 onal elements positions. When once the elements above
6 50 6 the principal diagonal are fixed, then automatically, the
= ´ = positions of the elements below the principal diagonal
25 14 7
will be fixed (∵ aji = -aij). Hence number of favourable
Answer: (B) cases to E is 73. Therefore,

76. A bag contains 4 balls of unknown colours. A ball is 73 1


P(E ) = =
drawn at random from it and is found to be white. 79 76
The probability that all the balls in the bag are
Answer: (A)
white is
4 1 3 2 78. Let a and b be two consecutive integers selected
(A) (B) (C) (D)
5 5 5 5 from the first 20 natural numbers. The probability
that a2 + b2 + a2 b2 is an odd positive integer is
Solution: Let Wj ( j = 1, 2, 3, 4) denote 1, 2, 3 and 4
white balls are in the bag. Let W be the ball drawn is 9 10 13
(A) (B) (C) (D) 1
white. Then 19 19 19

1 Solution: With the first 20 natural numbers, there are


P(W1 ) = P(W2 ) = P(W3 ) = P(W4 ) = 19 pairs of consecutive numbers. Let a = n, b = n + 1
4
so that n = 1, 2, 3, …, 19. Now
1
P(W /W1 ) = a2 + b2 + a2 b2 = n2 + (n + 1)2 + n2 (n + 1)2
4
2 = n4 + 2 n3 + 3n2 + 2 n + 1
P(W /W2 ) =
4 = (n2 + n + 1)2
3
P(W /W3 ) = Therefore
4
P(W /W4 ) = 1 a2 + b2 + a2 b2 = n2 + n + 1 = n(n + 1) + 1
Therefore by Bayes’ theorem which is always odd because n(n + 1) is always even. So

P(W4 /W ) =
P(W4 )P(W /W4 ) Probability = 1
4

å P(W )P(W /W )
j =1
j j
Answer: (D)

79. x, y, z are three numbers selected at random


from

1
the first 15 natural numbers and r = xi + yj + zk.
´1
= 4 The probability that r × a £ 12 where a = i + j + k is
1 æ 1 2 3 4ö
ç + + + ÷ 44 37 46 47
4 è 4 4 4 4ø (A) (B) (C) (D)
91 91 91 91
4 2
= = Solution: The number of ways of selecting x, y, z is
10 5 C3 = 455. Now
15

Answer: (D)
r × a £ 12 Û x + y + z £ 12 (7.14)
Worked-Out Problems 533

The number of possible integral solutions satisfying the 81. An urn contains 5 red balls and 3 blue balls. A ball is
inequality Eq. (7.14) is (Theorem 6.15, Vol. 1) selected at random and discarded, but 2 more balls
12
of the other colour are added into the bag. A second
å ( n - 1)
C2 = 2C2 + 3C2 + 4C2 +  + 11C2 ball is selected at random. Then the probability that
n= 3 this ball is red is
= (3C3 + 3C2 ) + (4C2 + 5C2 +  + 11C2 ) 31 13 41 23
(A) (B) (C) (D)
(∵ C3 = 1 = C2 )
3 2 72 36 72 36

= ( C3 + 4C2 ) + (5C2 + 5C2 +  + 11C2 )


4 Solution: Let R1 and B1 denote the events that first
drawn ball is red and blue, respectively. R2 is the event
= (5C3 + 5C2 ) +  + 11C2 of the second drawn ball being red. Then

R2 = R2 Ç (R1 È B1 ) = (R2 Ç R1 ) È (R2 Ç B1 )

= (11C3 + 11C2 ) Therefore

= 12C3 = 12C9 = 220 P(R) = P(R1 )P(R2 / R1 ) + P( B1 )P(R2 / B1 )

The required probability is 5 4 3 7


= ´ + ´
8 9 8 9
220 44
=
455 91 20 + 21 41
= =
Answer: (A) 72 72
Answer: (C)
80. Using the vertices of a cube, triangles are formed. If
a triangle is selected at random from these, the prob- 82. Let S = {2, 3, 4, 5, 6, 7, 8, 9, 10}. A subset of S is said
ability that it is an equilateral triangle is to be nice if it is a four-element subset such that the
2 3 1 5 four elements have no common divisor except 1. If
(A) (B) (C) (D)
7 8 7 7 a four-element subset of S is chosen at random, the
probability that it is nice is
Solution: A cube has 8 vertices out of which 3 can be
elected in 8C3 = 56 ways. Consider Figure 7.3 in which the 121 41 1 5
(A) (B) (C) (D)
vertices of the cube A1, B1, C1, D1; A2, B2, C2 and D2. If 126 42 42 126
we consider the vertex C1, then C1D2B2 is an equilateral
Solution: The number of four-element subsets of
triangle because the sides are the diagonals of 3 faces.
S = 9C4 = 126. Let E be the event of nice subset. Any four
Again C2A2B1 is equilateral. For each vertex, there
numbers selected from the numbers 2, 4, 6, 8, 10 have
corresponds an equilateral triangle so that there are 8
common divisor greater than 1. Four elements from 2, 4,
equilateral triangles. Hence the probability is
6, 8 and 10 can be selected in 5C4 = 5 ways. Therefore
8 1
= 5 121
56 7 P(E ) = Þ P(E ) = 1 - P(E ) =
126 126
C1 B1 Answer: (A)

83. An urn contains 6 white balls and 4 black balls.


Another urn contains 4 white and 6 black balls. A ball
A1 is drawn from first urn and transferred to second urn
D1
and again ball is transferred from second to first.
Now, a ball is drawn from the first urn. The prob-
ability that it is white is
C2 B2
181 81 221 123
(A) (B) (C) (D)
275 275 275 275
Solution: Let
D2 W1: Event of transfer of white ball from the
A2
first urn.
FIGURE 7.3 B1: Event of transfer of black ball from first urn.
Answer: (C) W2: Event of white ball from second to first.
534 Chapter 7 Probability

B2: Event of black ball from second to first. 3 50


= ´
W: Event of drawing white ball after the trans- 10 15 + 6
actions.
15 5
Therefore = =
21 7
W = W Ç [(W1 È B1 ) Ç W2 È (W1 È B1 ) Ç B2 ] Answer: (D)
= W Ç (W1 Ç W2 È B1 Ç W2 È W1 Ç B2 È B1 Ç B2 )
85. Box A contains 2 white, 1 black and 3 red balls. Box
= (W1 Ç W2 Ç W ) È [( B1 Ç W2 ) Ç W ] B contains 3 white, 2 black and 4 red balls. Box C
contains 4 white, 3 black and 2 red balls. A die is
È (W1 Ç B2 Ç W ) È ( B1 Ç B2 Ç W )
rolled. If 1, 2 or 3 appears, then a ball from box A
Hence will be drawn. If 4 or 5 appear, then a ball from box
B will be drawn, otherwise a ball from box C will be
6 5 6 4 4 7 6 6 5 drawn. A ball is chosen from one of the boxes and
P(W ) = ´ ´ + ´ ´ + ´ ´
10 11 10 10 11 10 10 11 10 it is found to be red. The probability that is from
4 7 6 box B is
+ ´ ´
10 11 10 12 16 11 13
(A) (B) (C) (D)
47 47 47 47
(since all the events are independent).
Answer: (A) Solution: Let A, B and C denote the selections of the
box A, B and C, respectively, and R denote drawing red
84. In a city 60% are males and 40% are females. ball. By hypothesis
Suppose 50% of males and 30% of females have
3 2 1
colour blindness. One is selected at random and is P( A) = , P( B) = and P(C ) =
found to be colour blind. The probability that the 6 6 6
selected person is male is Also
10 12 9 5
(A) (B) (C) (D) 3 4 2
19 87 19 7 P(R / A) = , P(R / B) = and P(R /C ) =
6 9 9
Solution: Let
Therefore by Bayes’ theorem
E1 = Event of selecting male.
E2 = Event of selecting female. P( B / R) =
P( B)P(R / B)
B = Event of colour blindness. P( A)P(R / A) + P( B)P(R / B) + P(C )P(R /C )
Then 2 4
´
60 3 = 6 9
P(E1 ) = = 3 3 2 4 1 2
100 5 ´ + ´ + ´
6 6 6 9 6 9
40 2
P(E2 ) = = 4
100 5
= 27
30 1 1 4 1
P( B / E1 ) = = + +
60 2 4 27 27
12 3 4 108 16
P( B / E2 ) = = = ´ =
40 10 27 27 + 20 47
By Bayes’ theorem Answer: (B)

P(E1 )P( B / E1 ) 86. Let X be a random variable whose range is {-1, 0, 1}.
P(E1 / B) =
P(E1 )P( B / E1 ) + P(E2 )P( B / E2 ) If mean m of X is 0.1, P(X = 0) = 0.3, then P(X = 1) is
3 1 (A) 0.2 (B) 0.4 (C) 0.32 (D) 0.3
´
= 5 2 Solution: By Definition 7.17
3 1 2 3
´ + ´ m = å xk P( X = xk )
5 2 5 10
Worked-Out Problems 535

Therefore It is known that


0.1 = (-1)P( X = - 1) + (0)P( X = 0) + (1)P( X = 1) x x2 x3
ex = 1 + + + ++ ¥
This implies 1 2 3

0.1 = - u + v (7.15) where x is real or complex. Therefore


where u = P(X = -1) and v = P(X = 1). Again sum of the el - 1 = 1
probabilities of X = 1. This implies
Þ el = 2
u + 0.3 + v = 1
Þ l = loge 2
Þ u + v = 0.7 (7.16) Answer: (A)
From Eqs. (7.15) and (7.16), u = 0.3, v = 0.4. Therefore
89. The distribution of a random variable X whose
P( X = 1) = 0.4 range is {1, 2, 3, 4} is given in Table 7.8.
Answer: (B)
Table 7.8 Single correct choice type question 89
87. The range of random variable X is {1, 2, 3} and X 1 2 3 4
P(X = 1) = 3l
l3,P(X = 2) = 4l - 10l
l2, P(X = 3) = 5l -1 P(X = x) K 2K 3K 4K
where l is constant. Then P(2 £ X £ 3) is equal to
8 2 4 1 Then, the mean and variance of X are, respectively
(A) (B) (C) (D)
9 3 9 3
(A) 2, 3 (B) 3, 1 (C) 3, 2 (D) 2, 4
Solution: We have Solution: We have
3

å P( X = K ) = 1
4

K =1 å P ( X = x) = 1
x =1

Þ 3l 3 - 10 l 2 + 9 l - 1 = 1 Þ 10 K = 1
Þ 3l 3 - 10 l 2 + 9 l - 2 = 0 1
ÞK=
Þ (l - 1)(3l - 1)(l - 2) = 0 10

l cannot be 1 and 2. Therefore l = 1/3. Now 7.17, mean is

P(2 £ X £ 3) = P( X = 2) + P( X = 3) m = å xP( X = x)
æ 1ö æ 1ö 5 = 1(K ) + 2(2 K ) + 3(3K ) + 4(4 K )
= 4 ç ÷ - 10 ç ÷ + - 1
è 3ø è 9ø 3
30
= 30 K = =3
10 8 10
=- +2=
9 9
s 2 is the variance, then by Quick Look 2
Answer: (A)
s 2 + m 2 = å x2 P ( X = x)
88. The range of a random variable is the set of all natural
numbers and l is a positive constant. If = 12 (K ) + 22 (2 K ) + 32 (3K ) + 42 (4 K )
100
lk = 100 K = = 10
P( X = k ) = (k = 1, 2, 3, …) 10
k
Therefore
then l equals
s 2 = 10 - 32 = 1
1 1
(A) loge 2 (B) loge 3 (C) (D) loge 3 Answer: (B)
2 2
Solution: We have 90. Let X be a binomial variate with parameters n and p.
¥
If the mean is 20 and variance is 15, then p is equal to
l l2 l3
å
K =1
P ( X = K ) = 1 Þ
1
+
2
+
3
++ ¥ = 1
(A)
1
(B)
1
(C)
1
(D)
3
3 4 2 4
536 Chapter 7 Probability

Solution: Mean m = np and variance s 2 = npq 93. X is a Poisson variate such that
(see Theorem 7.10). Therefore
2
P( X = 2) = P( X = 1)
np = m = 20 3
and 15 = npq then P(X = 3) is
Solving we get 34 -3/ 4 36 -4 / 3
(A) e (B) e
81 81
15 = 20 q
32 -4 / 3 36 -3/ 4
3 (C) e (D) e
Þq= 81 81
4
Solution: Let m be the parameter of the Poisson distri-
3 1
Þ p=1- q =1- = bution. Then
4 4
e- m mk
Answer: (B) P( X = k ) =
k
91. X is a binomial variate with mean 10 and variance 5.
Then the number of trials is equal to Hence
(A) 10 (B) 15 (C) 20 (D) 16 2
P ( X = 2) = P( X = 1)
3
Solution: Let n be the number of trial and p the prob-
ability of a success. Then by Theorem 7.10 e- m × m2 2 - m
Þ = e ×m
2 3
10 = np, 5 = npq
4
Hence Þm=
3
1
5 = 10q Þ q = and so
2
1 e-4 / 3 (4 / 3)3 64 -4 / 3 32 -4 / 3
and p=1- q = P( X = 3) = = e = e
2 3 27 ´ 6 81

So Answer: (C)

æ 1ö 94. The probability that a candidate securing admis-


10 = np = n ç ÷ sion in IIT through entrance test is 1/10. Seven
è 2ø
candidates are selected at random from a centre.
Þ n = 20 The probability that two will get admission in IIT
Answer: (C) through entrance test is
(A) 20(0.1)2 (0.9)5 (B) 15(0.1)2 (0.9)5
92. X is a Poisson variate such that P(X = 1) = p(X = 2),
(C) 21(0.1)2 (0.9)5 (D) 2(0.1)2 (0.9)5
then the parameter m is equal to
1 1 Solution: Let
(A) 2 (B) 3 (C) (D)
2 3 1
p = Probability of success = = 0.1
Solution: By hypothesis (Section 7.6.2) 10
q = Probability of failure = 1 - 0.1 = 0.9
-m mk
P( X = k ) = e
k
P( X = 2) = Probability of 2 success and 5 failures
By hypothesis
-m -m 2
= 7C2 (0.1)2 (0.9)5 (Deefinition 7.18)
e m e m
= = 21(0.1)2 (0.9)5
1 2
Answer: (C)
Therefore
95. A person has three coins A, B and C in his pocket
m=2 out of which A is a fair coin. The probability of B
Answer: (A) showing head is 2/3 and that of C is 1/3. He selected
Worked-Out Problems 537

one of the coins at random and tossed it three times 97. The probability of a coin showing head is p. 100 coins
and observed 2 heads and 1 tail. The probability that are tossed at a time. If the probability of 50 coins
the selected coin is A is showing heads is same as 51 coins showing heads,
7 18 9 16 then the value of p is
(A) (B) (C) (D)
25 25 25 25 49 51 1 49
(A) (B) (C) (D)
100 101 2 101
Solution: Let A, B and C denote the events of selecting
the coins A, B and C, respectively, so that Solution: By hypothesis,

P( A) = P( B) = P(C ) =
1 100
C50 p50 q50 = 100C51 p51 q49
3
where q = 1 - p. Therefore
Let E be the event of getting 2 heads and 1 tail. Hence by
the binomial distribution we have 100 100
(q) = ( p)
50 50 51 49
2
æ 1ö æ 1ö 3
P(E / A) = 3C2 ç ÷ ç ÷ = q p
è 2ø è 2ø 8 =
50 51
2
æ 2 ö æ 1ö 4 51q = 50 p
P(E / B) = C2 ç ÷ ç ÷ =
3
è 3 ø è 3ø 9
51(1 - p) = 50 p
2
æ 1ö æ 2 ö 2 51
P ( E /C ) = 3 C2 ç ÷ ç ÷ = p=
è 3ø è 3ø 9 101
Therefore by Bayes’ theorem Answer: (B)

P( A)P(E / A) 98. In an experiment, the odds in favour of success


P( A / E ) =
P( A)P(E / A) + P( B)P(E / B) + P(C )P(E /C ) are 2 :1. In six trials, the probability of at least five
success is
1 3
´ 240 256 496 396
= 3 8 (A) (B) (C) (D)
1 3 1 4 1 2 729 729 729 729
´ + ´ + ´
3 8 3 9 3 9 Solution: Let X denote the number of successes. Then
3 X is a binomial variate with
= 8
2
3 4 2
+ + p(probability of success) =
8 9 9 3
27 9 1
= = q =1- p=
75 25 3
Answer: (C) Then

96. A man takes a step forward with probability 0.4 and P(at least five successes) = P( X = 5) + P( X = 6)
a step backward with probability 0.6. After 11 steps, 5 6 0
æ 2 ö æ 1ö æ 2 ö æ 1ö
the probability that he is one step away from the = 6C5 ç ÷ ç ÷ + 6C6 ç ÷ ç ÷
è 3 ø è 3ø è 3 ø è 3ø
starting point is
(A) 462 ´ (0.24)5 (B) 462 ´ (0.25)5 6 ´ 32 64 256
= + =
(C) 362 ´ (0.24)5 (D) 362 ´ (0.25)5 729 729 729
Answer: (B)
Solution: One step away means, six forward and five
backward steps or six backward and five forward steps. 99. A book writer writes a good book with probability
Therefore 1/2. If it is a good book, the probability that it will
P(one step away) = C6 (0.4) (0.6) + C6 (0.6) (0.4)
11 6 5 11 6 5 be published is 2/3, otherwise it is 1/4. If he writes
2 books, the probability that at least one book will
= 11C6 (0.4)5 (0.6)5 (0.4 + 0.6) be published is
= 462 ´ (0.24)5 407 411 405 307
(A) (B) (C) (D)
Answer: (A) 576 576 576 576
538 Chapter 7 Probability

Solution: Let 15 6 1
= + +
G = Event of good book 64 64 64
G ¢ = Event of not a good book 22 11
= =
E = Event of publication 64 32
Then Answer: (D)

E = (G È G¢) Ç E = (G Ç E) È (G¢ Ç E) 101. If on average 1 ship in every 10 is wrecked, then out


of 5 ships expected to reach the shore, the prob-
Now
ability that 4 at least will arrive safely is
2 (A) 0.9 (B) 0.905 (C) 0.891 (D) 0.99
P(E /G) =
3 Solution: Let p be the probability of safe arrival and q
1 be the probability of wreck. By hypothesis
P ( E /G ¢ ) =
4 1 1 9
q= and p=1- =
1 10 10 10
P(G) = = P(G¢)
2
Let X denote the number of ships arriving safely. Now,
Therefore
P(at least four safe arrivals) = P( X = 4) + P( X = 5)
1 2 1 1 11
P(E ) = ´ + ´ = 4
æ 9ö æ 1ö
2 3 2 4 24 = 5 C4 ç ÷ ç ÷
è 10 ø è 10 ø
Further, X denotes the number of books published. Then 5 0
æ 9ö æ 1ö
P(at least one book will be published) = P(X = 1) + P(X = 2) + 5 C5 ç ÷ ç ÷
è 10 ø è 10 ø
2 0
æ 11 ö æ 13 ö æ 11 ö æ 13 ö 94 95
= 2C1 ç ÷ ç ÷ + 2C2 ç ÷ ç ÷ =5´ +
è 24 ø è 24 ø è 24 ø è 24 ø 105 105
11 13 æ 11 ö
2 5 ´ 6561 59049
=2´ ´ +ç ÷ = +
24 24 è 24 ø 105 105
91854
407 =
= 10 ´ 104
576
Answer: (A) 45927
=
50000
100. A fair coin is tossed 6 times. The probability of = 0.905
getting at least four heads is Answer: (B)
21 19 13 11
(A) (B) (C) (D)
32 32 32 32 102. An electrical bulb manufacturing company supplies
bulbs to a firm. It is known that 1 bulb out of every
Solution: With a fair coin, 10 is defective. If 8 bulbs of the company are
Probability of getting head = Probability of getting tail selected at random, the probability that the selec-
tion contains 5 good and 3 defective bulbs is
1
= 56 ´ 95 44 ´ 95
2 (A) (B)
108 108
Let X be the number of heads. Now 46 ´ 95 54 ´ 95
(C) (D)
P(getting at least four heads) = P( X = 4) + P( X = 5) 108 108

+ P( X = 6) Solution: We have
9
p = Probability for a good bulb =
4 2 5 6 0
æ 1ö æ 1ö æ 1ö æ 1ö æ 1ö æ 1ö
= 6 C 4 ç ÷ ç ÷ + 6 C 5 ç ÷ ç ÷ + 6 C6 ç ÷ ç ÷ 10
è 2ø è 2ø è 2ø è 2ø è 2ø è 2ø
1
q = Probability for a defective bulb =
10
Worked-Out Problems 539

Let X denote the number of good bulbs out of 8. Then 3 3


(C) 1 + (D) 1 -
5 3 e2 e2
æ 9ö æ 1ö
P( X = 5) = 8C5 ç ÷ ç ÷ Solution: We know that
è 10 ø è 10 ø
¥

=
56 ´ 95 å P( X = k ) = 1
k =0
108
Answer: (A)
¥
e-2 2k
Þå =1
k =0 k
103. A random variable X has Poisson distribution such
that P(X = 1) = P(X = 2). If m and s 2 are the mean Therefore
and variance of the distribution, then
æ 2 22 ö
m å P( X ³ 3) = 1 - e -2
çè 1 + +
1 2 ÷ø
=
s2
5
1 =1-
(A) 2 (B) (C) 1 (D) 2 e2
2 Answer: (B)
Solution: Let l be the parameter of X
X. By hypothesis
105. If the mean of a Poisson distribution is 1/2 then the
P( X = 1) = P( X = 2) ratio of P(X = 3) is to P(X = 2) is
-l
e l e l
1 -l 2 (A) 1: 6 (B) 1: 8 (C) 1: 4 (D) 1: 2
Þ =
1 2 Solution: For a Poisson distribution, the mean is equal
to the parameter. Hence the parameter is 1/2. Now,
Þl=2
e-1/ 2 (1/ 2)3
For a Poisson distribution, mean and variance are equal P( X = 3) =
(see Theorem 7.11) 3

l 2 e-1/ 2 (1/ 2)2


= =1 P( X = 2) =
s2 2 2
Answer: (C) Therefore

104. A random variable X has Poisson distribution with 1 1


P( X = 3): P( X = 2) = :
parameter 2. Then P(X ³ 3) is 48 8
5 5 = 1: 6
(A) 1 + (B) 1 -
e2 e2 Answer: (A)

Multiple Correct Choice Type Questions


1. A and B are two events of a random experiment such 1 - P( A È B)
that P(A) > 0 and P(B) ¹ 1. Then P( A / B) equals =
P( B)
(A) 1 - P( A / B) (B) 1 - P( A / B)
Therefore (D) is correct. Also,
1 - P( A È B)
(C) 1 - P( A / B) (D) P( A Ç B) P( A Ç B)
P( B) P( A / B) + P( A / B) = +
P( B) P( B)
Solution: We have
P(( A È A) Ç B)
=
P( A Ç B) P( B)
P( A / B) =
P( B)
P( B)
= =1
P( A È B) P( B)
=
P( B)
540 Chapter 7 Probability

This implies Solution: Mutually exclusive events means one event


prevents the other. Hence they are dependent events.
P( A / B) = 1 - P( A / B) Now E and F are independent events implies that
So (B) is correct. P(E / F ) = P(E )
Answers: (B), (D)
Þ P(E Ç F ) = P(E )P(F ) (7.17)
2. If M and N are two events, then the probability of Now
exactly one of them occurs is
(A) P(M) + P(N) N - 2P(M Ç N) N P (E Ç F )
P (E / F ) =
(B) P(M) + P(N)
N - P(M Ç N) N P (F )
(C) P(M ) + P( N ) - 2 P(M Ç N ) P (E - F )
=
(D) P(M Ç N ) + P(M Ç N ) P(F )
Solution: We have P(E ) - P(E Ç F )
=
P(exactly one of M and N occurs) P(F )

= P ( M DN ) P ( E ) - P ( E )P (F )
= [ From Eq. (7.17)]
P(F )
= P((M - N ) È ( N - M ))
= P(M - N ) + P(N
N - M) P(E)(1 - P(F ))
=
P(F )
= P(M ) - P(M Ç N ) + P( N ) - P(M Ç N )
P ( E )P (F )
[part (2), Theorem 7.2] = = P(E )
P(F )
= P(M ) + P( N ) - 2P(M Ç N )
Hence (B) is correct.
Hence (A) is correct. Also, (C) follows from (B) and so (C) is correct. Now,
P(M Ç N ) + P(M Ç N ) = P(M - N ) + P( N - M ) P (E Ç F ) P (E Ç F )
P (E / F ) + P (E / F ) = +
= P(M DN ) P(F ) P(F )

So (D) is correct. Again P((E È E) Ç F )


=
P(F )
P (M ) + P ( N ) - 2 P (M Ç N )
P(F )
= 1 - P(M ) + 1 - P( N ) - 2 P(M È N ) = =1
P(F )
= 2 - P(M ) - P( N ) - 2[1 - P(M È N )]
So (D) is correct.
= - P(M ) - P(N ) + 2[ P(M ) + P( N ) - P(M Ç N )] Answers: (B), (C), (D)
= P (M ) + P ( N ) - 2 P (M Ç N )
4. A bag contains 9 coins of which 3 are fair, 2 are
= P(M - N ) + P( N - M ) two headed and 4 are two tailed coins. Define the
following events,
Hence (C) is correct.
A: Drawing a fair coin
Answers: (A), (C), (D) B: Drawing two headed coin
C: Drawing coin having tails on both sides
3. If E and F are independent events such that 0 < P(E), E: Drawn coin showing tail
F < 1, then
P(F)
Then,
(A) E and F are mutually exclusive
1 4
(B) E and F are mutually independent (A) P( A Ç E) = (B) P(C Ç E) =
6 9
(C) E and F are mutually independent 1 8
(C) P(E) = (D) P(C/ E) =
(D) P(E / F ) + P(E / F ) = 1 3 11
Worked-Out Problems 541

Solution: We have 1 - P( A È B)
=
3 1 1 - P( A)
P( A) = =
9 3 1 - P( A È B)
=
1 1 - (0.4)
P(E / A) =
2 1 - P( A È B)
=
Therefore 0.6

1 1 1 Therefore
P(E Ç A) = ´ =
3 2 6 P( A È B) = 1 - (0.6)(0.6)
So (A) is correct. Now = 1 - 0.36 = 0.64
4 So (A) is correct. Now
P(C ) = and P(E /C ) = 1
9
P( A Ç B) = P( A)P( B / A)
Therefore
= (0.4)(0.9) = 0.36
4 4
P(C Ç E) = ´ 1 = Hence (B) is correct. Again
9 9
So (B) is correct. Now the event E is given by 0.64 = P( A È B)
= P( A) + P( B) - P( A Ç B)
E = ( A È C ) Ç E = ( A Ç E) È (C Ç E)
= 0.4 + P( B) - (0.36)
Therefore
Therefore
P(E) = P( A Ç E) + P(C Ç E)
P( B) = 1 - (0.4) = 0.6
1 4 11
= + =
6 9 18 Given P(B) we have
P( A Ç B)
P( A / B) =
P( B)
P(C Ç E)
P(C/ E) =
P(E ) 0.36
= = 0.6
0.6
4 11 8
= ¸ =
9 18 11 So (C) is correct. Again

So (D) is correct. P( A Ç B)
P( A / B) =
Answers: (A), (B), (D) P( B)
P( A) - P( A Ç B)
5. A, B are two events with positive probabilities. If =
P(A) = 0.4, P( B / A) = 0.9, P( B / A) = 0.6, then 1 - P( B)

(A) P(A È B) = 0.64 (B) P(A Ç B) = 0.36 0.4 - (0.36)


=
1 - (0.61)
(C) P( A / B) = 0.6 (D) P( A / B) = 0.2
0.044
Solution: We have = = 0.1
0.4
0.6 = P( B / A) Here we have used A = ( A Ç B) È ( A Ç B). This implies
P( B Ç A) (D) is not correct.
= Answers: (A), (B), (C)
P( A)
P( A È B) 6. Two fair dice are thrown. Let A be the event of both
=
P( A) faces are even numbers and B the event of sum of the
faces is 10. Then
542 Chapter 7 Probability

1 1
(A) P( A) = (C) P(C Ç A) =
4 10
1 37
(B) P( B) = (D) P( A È B È C ) =
18 50
1 Solution: We have
(C) P( A Ç B) =
18 Number of even numbers £ 100 is equal to 50.
2 Number of multiples of 3 £ 100 is 33.
(D) P(exactly one of A and B) =
9
Number of multiples of 5 £ 100 is 20.
Solution: For the event A, the favourable cases are Number of common multiples of 2 and 3 is 16.
(2, 2), (2, 4), (2, 6), Number of common multiples of 3 and 5 is 6.
(4, 2), (4, 4), (4, 6), Number of common multiples of 2 and 5 is 10.
(6, 2), (6, 4), (6, 6) Number of common multiples of 2, 3 and 5 is 3.
Therefore Now,
9 1 50 33 20
P( A) = = P( A) = , P( B) = , P(C ) =
36 4 100 100 100
Hence (A) is correct. For the event B, the favourable 16 6 10
cases are (4, 6), (5, 5) and (6, 4). Therefore P( A Ç B) = , P( B Ç C ) = , P(C Ç A) =
100 100 100
3 1 Also
P( B) = =
36 12
P( A È B È C )
So (B) is not correct. Now
= P( A) + P( B) + P(C ) - P( A Ç B) - P( B Ç C )
A Ç B = {(4, 6), (6, 4)}
- P(C Ç A) + P( A Ç B Ç C )
This implies
50 33 20 16 6 10 3
= + + - - - +
2 1 1000 100 100 100 100 100 100
P( A Ç B) = =
36 18 106 - 32 74 37
= = =
So (C) is correct. Finally 100 100 50

P(exactly one of A and B is correct) Hence all (A), (B), (C) and (D) are correct.
Answers: (A), (B), (C), (D)
= P(( A - B) È ( B - A))

= P( A - B) + P( B - A) 8. A and B are two among 10 students from which


3 students are to be selected at random to form a
7 1 8 2
= + = = committee. Then,
36 36 36 9
3
(A) Probability that A belongs to the committee is .
So (D) is correct. 10
Answers: (A), (C), (D) (B) Probability that A and B belong to the committee
1
is .
7. A natural number is selected at random from the 15
first 100 natural numbers. Let A, B and C denote the (C) Probability that either A or B belong to the
events of selection of even number, a multiple of 3 8
and a multiple of 5, respectively. Then committee is .
15
4
(A) P( A Ç B) = (D) Probability that exactly one of A and B belongs
25 7
to the committee is .
3 15
(B) P( B Ç C ) =
50
Worked-Out Problems 543

Solution: Let A and B denote the events “A belongs Solution: Since there are 4 even numbers among
to the committee” and “B belongs to the committee,” 1 to 9, it follows that
respectively. Then
4
9
P(E / H ) =
C 36 3 9
P( A) = 10 2 = =
C3 120 10
Again, since there are 2 even numbers among 1 to 5, it
3 follows that
P( B) =
10 2
P(E /T ) =
8
C1 8 1 5
P( A Ç B) = 10
= =
C3 120 15 Therefore (A) and (B) are correct. Now,
Now
E = ( H È T ) Ç E = ( H Ç E) È (T Ç E)
P( A È B) = P( A) + P( B) - P( A Ç B)
This implies
3 3 1 16 8
= + - = = P(E) = P( H Ç E ) + P(T Ç E )
10 10 15 30 10
= P( H )(E / H ) + P(T )P(E /T ) (Theorem 7.3)
So
2 4 1 2 58
P(exactly one of A and B) = P(( A - B) È ( B - A)) = ´ + ´ =
3 9 3 5 135
= P( A - B) + P( B - A)
So (C) is correct. By Bayes’ theorem,
8 8
C C
= 10 2 + 10 2 P ( H )P (E / H )
C3 C3 P( H / E ) =
P( H )P(E / H ) + P(T )P(E /T )
28 28
= + 2 4
´
120 120 3 9
=
56 2 4 1 2
= ´ + ´
120 3 9 3 5
8 135 20
=
7 = ´ =
15 277 58 29

Hence all (A), (B), (C) and (D) are correct. Hence (D) is correct.
Answers: (A), (B), (C), (D) Answers: (A), (B), (C), (D)

10. G1 and G2 are two groups of students. G1 consists of


9. A coin is so weighted such that the probability of it
showing H (Head) is 2/3 and that of T (Tail) is 1/3 4 boys and 3 girls. G2 consists of 3 boys and 3 girls. Let
when it is tossed. If head appears, then a number from p1 be the probability of arranging the boys and girls
the first 9 naturals is selected at random, otherwise a of G1 alternately. p1¢ is the probability of arranging
number from 1, 2, 3, 4, 5 will be selected. Let E be the the boys and girls of G1 such that all the 4 boys are
event of getting an even number. Then, together and all the 3 girls together. p2 is the prob-
ability of arranging boys and girls of G2 alternately.
4 p2¢ is the probability of arranging boys and girls of G2
(A) P(E / H ) =
9 such that no two boys and no of two girls are sepa-
2 rated. Then,
(B) P(E /T ) =
5 1 2
(A) p1 = (B) p1¢ =
58 35 35
(C) P(E) =
135 1 1
(C) p2 = (D) p2¢ =
20 10 20
(D) P( H / E) =
29 Solution: Since number of boys is greater than number
of girls, they can be arranged alternately only when the
544 Chapter 7 Probability

first place is occupied by a boy (example BG BG BG B P( A) + P( B) - 1


where B stands for boy and G for girl). Therefore ³ [∵ P( A È B) £ 1]
P( B)
4 3 3 2 2 1 1 1
p1 = × × × × × × = Hence (A) is correct. Again
7 6 5 4 3 2 1 35
A = ( A - B) È ( A Ç B)
Therefore (A) is correct. Since boys are together and
girls are together (example b1, b2, b3, b4, g1, g2, g3), we Þ P( A) = P( A - B) + P( A Ç B)
consider all the 4 boys as a single object and all the 3 girls
as a single object. Two objects can be arranged in 2 Þ P( A - B) = P( A) - P( A Ç B)
ways. In each of these arrangements boys can be inter-
Þ P( A Ç B) = P( A) - P( A Ç B)
changed among themselves in 4 ways and girls can be
interchanged among themselves in 3 ways. Therefore A and B are independent,
the number of arrangements of boys and girls satisfying then so are A and B and hence
the condition is 2 ´ 4 ´ 3. So
P( A Ç B) = P( A)P( B)
2´ 4´ 3 2´6 2
p1¢ = = =
7 5 ´ 6 ´ 7 35 Therefore

Hence (B) is correct. Number of arrangements of boys P( A È B) = 1 - P( A È B)


and girls alternately (example: bgbgbg or gbgbgb)
= 2 ´ 3 ´ 3. Therefore = 1 - P( A Ç B)

2 ´ 3 ´ 3 72 1 = 1 - P( A)P( B)
p2 = = =
6 720 10 Consider the following example which shows P(AÈB) ¹
Hence (C) is correct. Finally 1 - P( A)P( B) when A Ç B = f.
Let S be the sample space obtained when a fair die
2´ 3´ 3 1 is rolled so that S = {1, 2, 3, 4, 5, 6}. Let A = {1, 3, 5} and
p2¢ = =
6 10 B = {2, 4, 6} so that A Ç B = f and P(A È B) = P(S) = 1.
But
(Example: b1b2b3g1g2g3 or g1g2g3b1b2b3.) Therefore (D) is
1
not correct. P( A) = P( B) = = P( A) = P( B)
2
Answers: (A), (B), (C)
Therefore
11. Let A and B be any two events in a sample space.
Then 1 1
1 - P( A)P( B) = 1 - ´
(A) When P(B) ¹ 0, 2 2
3
P( A) + P( B) - 1 =
P( A / B) ³ 4
P( B)
¹ P( A È B)
is always true
(B) P( A Ç B) = P( A) - P( A Ç B) is always true Hence (D) is not correct.
Answers: (A), (B), (C)
(C) P( A È B) = 1 - P( A)P( B) if A and B are
independent 12. A and B are two random events such that 0 < P(A) < 1
(D) P( A È B) = 1 - P( A)P( B) if A and B are and 0 < P(B) < 1. Then which of the following are
disjoint. true?
Solution: We have (A) P(A/B) > P(A) Þ P(B/A/ ) > P(B)

P( A Ç B) (B) P( B / A) + P( B / A) = 1
P( A / B) =
P( B) (C) P( A / B) = P( A / B) only when A Ç B = f
P( A) + P( B) - P( A È B) (D) P( B / A) = P( B / A) only when A and B are
=
P( B) independent
Worked-Out Problems 545

Solution: Suppose (C) is also false for which consider the same sample space:
S = {1, 2, 3, 4, 5, 6} and let A = {2, 4, 6} and B = {2, 3, 5}.
P( A / B) > P( A) Then
P( A Ç B)
Þ > P( A) 2
P( B) P( A / B) =
3
P( A Ç B)
Þ > P( B) [∵ 0 < P( A), P( B)] P( A / B) =
2
P( A) 3
Þ P( B / A) > P( B)
but A Ç B ¹ f. Finally
So (A) is correct. We now show that (B) is false. P( B / A) = P( B / A)
Consider the following example: Let S = {1, 2, 3, 4, 5, 6}
which is obtained by rolling a fair die. Let A = {2, 6} and P( A Ç B) P( B Ç A)
Û =
B = {2, 3, 5}. Then P( A) P( A)
1 Û [1 - P( A)]P( A Ç B) = P( A)P( B Ç A)
P( A / B) =
3 Û [1 - P( A)]P( A Ç B) = P( A)[ P( B) - P( A Ç B)]
1
P( A / B) = Û P( A Ç B) = P( A)P( B)
3
Û A and B are independent events
so that
Hence (D) is correct.
2
P( A / B) + P( A / B) = ¹ 1 Answers: (A), (D)
3

Matrix-Match Type Questions


1. A man will live 10 more years for which the prob- (B) We have
ability is 1/3 and the probability that his wife will live
10 more years is 1/4. Match the items of Column I with P( H È W ) = P( H ) + P(W ) - P( H Ç W )
those of Column II. 1 1 1
= + -
3 4 12
Column I Column II
6 1
1 = =
(A) Probability that both wife and (p) 12 2
husband will live 10 more years is 2
Answer: (B) Æ (p)
3
(q) (C) We have
(B) Probability that at least one of them 4
will live 10 more years is 1 P( H Ç W ) = P( H È W )
(r)
4
(C) Probability that neither will be alive 1 = 1 - P( H È W )
in 10 more years is (s)
12 1 1
2 =1- =
(D) Probability that only husband will (t) 2 2
live 10 more years is 3 Answer: (C) Æ (p)
We have that since H , W are independent
Solution: Let H be the event that husband will live 10 more
years and W be the event that wife will live 10 more years. P( H Ç W ) = P( H )P(W )
(A) We have that since H and W are independent 1 æ 1ö 1
= ´ ç1 - ÷=
P( H Ç W ) = P( H )P(W ) 3 è 4ø 4
1 1 1 Answer: (D) Æ (r)
= ´ =
3 4 12
Answer: (A) Æ (s)
546 Chapter 7 Probability

2. In a class, there are 10 boys and 5 girls. Three students 3. A and B are two events with P(A) = 1/3, P(B) = 1/4
are selected one after other without allowing the and P(A È B) = 1/2. Match the items of Column I
earlier selected student to join the class. Match the with those of Column II.
items of Column I with the items of Column II.
Column I Column II
Column I Column II
1
9 (A) P(A/B) is equal to (p)
(A) The probability that the first two (p) 3
are boys and the third a girl is 13 1
3 (B) P(B/A
/ ) equals (q)
(B) The probability that first and third
T (q) 4
are boys and the second a girl is 7 1
5 (C) The value of P( A Ç B) is (r)
(C) The probability that first and third (r) 2
are of same sex and the second is of 21 2
(D) P( A / B) is equal to (s)
opposite sex is 3
15
(D) If the first and third are of same sex (s)
and the second is of opposite sex, 91 Solution: We have
then probability that the second
1
student is girl is = P( A È B)
2
Solution: Let b and g denote boy and girl, respectively. = P( A) + P( B) - P( A Ç B)
(A) The probability that the first two are boys and the 1 1
third a girl is = + - P( A Ç B)
3 4
10 9 5 15 Therefore
P(bbg ) = ´ ´ =
15 14 13 91
1 1 1
Answer: (A) Æ (s) P( A Ç B) = + -
3 4 2
The probability that first and third are boys and the
second a girl is 4+3-6 1
= =
12 12
10 5 9 15
P(bgb) = ´ ´ =
15 14 13 91 (A) We have
Answer: (B) Æ (s) P( A Ç B) 1 1 1
P( A / B) = = ¸ =
(C) The probability that first and third are of same sex P( B) 12 4 3
and the second is of opposite sex is
Answer: (A) Æ (p)
P ((bgb) È ( gbg )) = P(bgb) + P( gbg ) (B) We have
15 5 10 4 P( B Ç A) 1 1 1
= + ´ ´ P( B / A) = = ¸ =
91 15 14 13 P( A) 12 3 4
15 20 Answer: (B) Æ (q)
= +
91 3 ´ 91
(C) We have
65 5
= =
3 ´ 91 21 P( A Ç B) = P( A - B)
Answer: (C) Æ (r) = P( A) - P( A Ç B)
Let E1 = (bgb) È (gbg) and E2 = bgb. Then
1 1
= -
P(E2 ) 3 12
P(E2 / E1 ) =
P(E1 )
3 1
= =
15 21 9 12 4
= ´ =
91 5 13 Answer: (C) Æ (q)
Answer: (D) Æ (p)
Worked-Out Problems 547

(D) We have 2 5
´
= 3 16
P( A Ç B)
P( A / B) = 2 5 1 3
´ + ´
P( B) 3 16 3 8
1/ 4 1 10 5
= = = =
1 - (1/ 4) 3 10 + 6 8
Answer: (D) Æ (p)
Answer: (D) Æ (p)
4. Bag X contains 5 red, 3 white and 8 black balls. Bag Y
contains 3 red and 5 white balls. A fair die is tossed. If 5. , B and C are three students of Mathematics. Each
2 or 5 appears a ball from bag Y is chosen, otherwise a is given two problems in probability. Their chances
ball from bag X is chosen. Match the items of Column I of solving the problems are, respectively, 1/2, 1/3
with those of Column II. Here R, W and B denote and 1/6. They try independently. Let A1, A2, B1, B2,
drawing red ball, white ball and black ball, respectively. C1, C2 denote the events of their solving the first and
second problems. Match the items of Column I with
those of Column II.
Column I Column II
5 Column I Column II
/X) =
(A) P(R/X (p)
8
1
3 (A) P(A1 Ç B2) (p)
Y =
(B) P(R/Y) (q) 4
16
1
3 (B) P(C1 Ç A2) (q)
W/X) =
(C) P(W/ (r) 6
8
1
5 (r)
X R) =
(D) P(X/ (s) 12
16 (C) P(B1 Ç C2)
1
(s)
Solution: We have 18
1
4 2 (D) P(A1 Ç A2) (t)
P( X ) = = 36
6 3
2 1 Solution:
P(Y ) = =
6 3 (A) The required probability is
(A) The required probability is P( A1 Ç B2 ) = P( A1 )P( B2 )
5 1 1 1
P(R / X ) = = ´ =
16 2 3 6
Answer: (A) Æ (s) Answer: (A) Æ (q)
(B) The required probability is (B) The required probability is
3 P(C1 Ç A2 ) = P(C1 )P( A2 )
P ( R /Y ) =
18
1 1 1
Answer: (B) Æ (r) = ´ =
6 2 12
(C) The required probability is
Answer: (B) Æ (r)
3 (C) The required probability is
P(W / X ) =
16
Answer: (C) Æ (q) P( B1 Ç C2 ) = P( B1 )P(C2 )
(D) By Bayes’ theorem, 1 1 1
= ´ =
3 6 18
P ( X )P (R / X )
P( X / R) = Answer: (C) Æ (s)
P( X )P(R / X ) + P(Y )P(R /Y )
548 Chapter 7 Probability

(E) The required probability is æ 3ö æ 1ö


0 5
æ 3ö æ 1ö
1
æ 3ö æ 1ö
4 2 3

= 5C0 ç ÷ ç ÷ + 5C1 ç ÷ ç ÷ + 5C2 ç ÷ ç ÷


P( A1 Ç A2 ) = P( A1 )P( A2 ) è 4ø è 4ø è 4ø è 4ø è 4ø è 4ø
5
1 1 1 æ 1ö 5 ´ 3 10 ´ 9
= ´ = =ç ÷ + 5 +
2 2 4 è 4ø 4 45
Answer: (D) Æ (p) 106
=
45
6. The probability that a person can hit a target is 3/4.
53
He tries 5 times. Let X denote binomial variate with =
parameters n = 5 and p = 3 / 4, q = 1/ 4. Match the items 512
of Column I with those of Column II.
Answer: (A) Æ (r)
(B) The required probability is
Column I Column II
P( X ³ 3) = 1 - P( X £ 2)
459
(A) P(X £ 2) (p) 106
512 =1-
15 45
(B) P(X ³ 3) (q)
16 1024 - 106
=
53 1024
(C) Mean of X (r)
512 918
15 =
(D) Variance of X (s) 1024
4 459
=
512
Solution: By hypothesis
Answer: (B) Æ (p)
P( X = k ) = 5Ck pk q5 - k (C) Mean is given by
k 5- k
æ 3ö æ 1ö 3 15
= 5 Ck ç ÷ ç ÷ np = 5 ´ =
è 4ø è 4ø 4 4
for k = 0, 1, 2, 3, 4, 5. Answer: (C) Æ (s)
(A) The required probability is (D) Variance is given by

P( X £ 2) = P( X = 0) + P( X = 1) + P( X = 2) 3 1 15
npq = 5 ´ ´ =
4 4 16
Answer: (D) Æ (q)

Comprehension-Type Questions
1. Passage: We consider a two-player game in which 2 7 2 1
there is always a winner. A, B and C are three players. (A) (B) (C) (D)
9 9 3 3
2
If A plays B, then the probability of A beating B is . (ii) When B plays C, the winner plays A. Then, the
3 probability that A is the final winner is
2 2 5 7 4
If B plays C, then the probability of B beating C is . (A) (B) (C) (D)
3 9 9 9 9
2
If C plays A, then the probability of C beating A is . (iii) When C plays A, the winner plays B. Then, the
3
probability that A is the final winner is
Answer the following questions:
5 4 2 7
(i) When A plays B, the winner plays C. Then, the (A) (B) (C) (D)
9 9 9 9
probability that A is the final winner is
Worked-Out Problems 549

Solution: Therefore
(i) First, A must beat B and then A must beat C. The 4 1
probability that A is the final winner is = 1 + P( B) Þ P( B) =
3 3
2 1 2 Answer: (B)
´ =
3 3 9 (ii) The required probability is
Answer: (A)
P( A Ç B)
(ii) Whether B beats C or C beats B, the winner must P( A / B) =
P( B)
lose to A. Therefore
Probability (A is final winner) P( A)P( B)
=
= P(B beats C and loses to A) P( B)
+ P(C
C beats B and loses to A) 1
= P( A) =
2 2 1 1 5 2
= ´ + ´ =
3 3 3 3 9 Answer: (D)
Answer: (B) (iii) A and B are independent. This implies A and B are
(iii) First, A must beat C and then A must beat B. The also independent. So
probability that A is the final winner is
P( B Ç A)
P( B / A) =
1 2 2 P( A)
´ =
3 3 9
P( B)P( A)
Answer: (C) =
P( A)

2. Passage: A and B are independent events with = P( B)

1 2 = 1 - P( B)
P( A) = and P( A È B) =
2 3 1 2
= 1- =
Answer the following questions. 3 3
(i) P(B) is equal to Answer: (B)
1 1 2 1
(A) (B) (C) (D) 3. Passage: A box contains 3 coins out of which two are
2 3 3 4
fair coins and the third is a two headed coin. A coin
(ii) P(A/B) equals is selected at random and tossed. If head appears, the
3 2 1 1 same coin is tossed again. If tail appears, another is
(A) (B) (C) (D) selected from the remaining two coins and tossed.
4 3 3 2
Answer the following questions.
(iii) P( B / A) is equal to (i) The probability that head appears twice is
1 2 1 2 1 2 1 1
(A) (B) (C) (D) (A) (B) (C) (D)
2 3 3 5 2 3 3 4

Solution: (ii) If the same is tossed twice, then the probability


that it is a two headed coin is
(i) We have
1 1 2 1
2 (A) (B) (C) (D)
= P( A È B) 3 2 3 3
3
(iii) The probability that tail appears twice is
= P( A) + P( B) - P( A Ç B)
2 1 1 1
(A) (B) (C) (D)
1 1 3 3 12 4
= + P( B) - P( B)
2 2
Solution: Let E1 be the event of selecting fair coin and
1 1 E2 be the event of selecting two headed coin. Hj denotes
= + P( B)
2 2 head and Tj denotes tail where j = 1, 2.
550 Chapter 7 Probability

(i) We have (ii) The probability of A winning first set 6-1 is


H1 Ç H2 = (E1 Ç H1 Ç H2 ) È (E2 Ç H1 Ç H2 ) 40 40 20 32
(A) (B) (C) (D)
37 36 37 37
Then
(iii) The probability of B winning the first set 6-1 is
P( H1 Ç H2 ) = P(E1 Ç H1 Ç H2 ) + P(E2 Ç H1 Ç H2 ) 20 20 40 4
(A) 6 (B) 7 (C) 6 (D) 6
2 1 1 1 1 3 3 3 3
= ´ ´ + ´1´1=
3 2 2 3 2
Solution:
Answer: (A)
(i) “A” wins the set at 6-0 means A has to win all his
(ii) Let E be the event of the same coin being tossed
3 service games and B has to lose all his 3 service
twice. Then
games. Therefore the probability is
E = (E1 Ç H ) È E2 3 3 3
æ 2 ö æ 1ö 2
çè ÷ø çè ÷ø =
Therefore 3 3 36
P(E) = P(E1 Ç H ) + P(E2 ) Answer: (C)
2 1 1 2 (ii) Probability of A winning by 6-1 is
= ´ + =
3 2 3 3 4 2 3 3
æ 2 ö æ 2 ö æ 1ö æ 1 ö æ 2 ö æ 1 ö 40
3
C1 ç ÷ ç ÷ ç ÷ + 3C1 ç ÷ ç ÷ ç ÷ = 6
Now è 3 ø è 3 ø è 3ø è 3ø è 3 ø è 3ø 3
P(E2 )P( H / E2 ) Answer: (B)
P(E2 / E) =
P(E1 )P( H / E1 ) + P(E2 ) (iii) Probability of B winning the first set by 6-1 is
1 3 3
æ 2 ö æ 2 ö æ 1ö æ 1 ö æ 2 ö æ 1 ö 20
2 4
´1 3
C1 ç ÷ ç ÷ ç ÷ + 3C1 ç ÷ ç ÷ ç ÷ = 6
1 è 3 ø è 3ø è 3ø è 3ø è 3 ø è 3ø
= 3 = 3
2 1 1 2
´ + Answer: (A)
3 2 3
Answer: (B) 5. Passage: A fair die is tossed repeatedly until face six
T = Tail appearing twice. This implies selection of appears. Let X denote the number of tosses required.
fair coin first, getting tail and then selection of fair Answer the following questions:
coin and again getting tail. Hence (i) The probability that X is equal to 3 is
P(T ) = P(E1 Ç T1 ) ´ P(E1 Ç T2 ) 25 25 5 125
(A) (B) (C) (D)
216 36 36 216
æ 2 1ö æ 1 1ö
=ç ´ ÷ ´ç ´ ÷
è 3 2ø è 2 2ø (ii) The probability that X ³ 3 is
1 125 25 5 25
= (A) (B) (C) (D)
12 216 36 36 216
Answer: (C) (iii) The conditional probability that X ³ 6 given
X > 3 equals
4. Passage: and B are two tennis players participating 125 25 5 25
in a singles match as opponents. As per the rules of (A) (B) (C) (D)
216 216 36 36
the lawn tennis, the one who wins atleast six games
with at least two games more than his opponent wins Solution: Let p be the probability of getting six
the set. They serve alternately from game to game (success) in a single throw of a die so that
(i.e., for each game). A starts to serve after winning
the toss. It is given that the probability of A winning 1
p=
his service game is 2/3 and that of losing it is 1/3. 6
Answer the following questions:
and the probability of failure
(i) The probability of A winning the first set at six-
LOVE (6-0) is 5
6 6 3
q =1- p=
æ 2ö æ 1ö 2 1 6
(A) ç ÷ (B) ç ÷ (C) (D)
è 3ø è 3ø 36 2
Worked-Out Problems 551

(i) The required probability is (iii) If the drawn ball is white, then the probability
2
that it is from urn C is
æ 5 ö æ 1 ö 25
P( X = 3) = q2 p = ç ÷ ç ÷ = 2 3 12 13
è 6 ø è 6 ø 216 (A) (B) (C) (D)
5 5 25 25
Answer: (A)
(ii) The required probability is Solution:
(i) Let W denote that drawn ball is white. Then
P( X ³ 3) = q2 p + q3 p + q4 p +  + ¥
W = (A È B È C) Ç W
= q2 p(1 + q + q2 +  + ¥)
= ( A Ç W ) È ( B Ç C ) È (C Ç A)
æ 5ö æ 1ö æ ö 25
2
1
=ç ÷ ç ÷ç = Therefore
è 6 ø è 6 ø è 1 - (5 / 6) ÷ø 36
P(W ) = P( A Ç W ) + P( B Ç W ) + P(C Ç W )
Answer: (B)
= P( A)P(W / A) + P( B)P(W / B) + P(C )P(W /C )
(iii) The required probability is (∵ X > 3)
2 2 2 1 1 4
P(( X ³ 6)/( X > 3)) = 1 - ( P( X = 4) + P( X = 5)) = ´ + ´ + ´
5 5 5 5 5 5
= 1 - ( p + pq) 10 2
= =
= 1 - p(1 + q) 25 5
Answer: (C)
1æ 5ö
= 1 - ç1 + ÷ (ii) Let E denote that drawn ball is black. Then
6è 6ø
P(E) = P( A)P(E / A) + P( B)P(E / B) + P(C )P(E /C )
11 25
=1- = 2 3 2 4 1 1
36 36 = ´ + ´ + ´
Answer: (D) 5 5 5 5 5 5
15 3
= =
6. Passage: Urns A, B, C, respectively, contain 2 white 25 5
and 3 black balls, 1 white and 4 black balls, 4 white and Answer: (D)
1 black balls. The probabilities of choosing the urns
(iii) By Bayes’ theorem we have
are, respectively, 2/5, 2/5, and 1/5. One of the urns is
chosen at random and a ball is drawn from it. Answer P(C )P(W /C )
the following questions: P(C /W ) =
P( A)P(W / A) + P( B)P(W / B) + P(C )P(W /C )
(i) Probability that the drawn ball is white is
1 4
12 13 2 3 ´
(A) (B) (C) (D) = 5 5
25 25 5 5 2 2 2 1 1 4
´ + ´ + ´
(ii) Probability that the drawn ball is black is 5 5 5 5 5 5
13 12 2 3 4 2
(A) (B) (C) (D) = =
25 25 5 5 10 5
Answer: (A)

Assertion–Reasoning Type Questions


In the following set of questions, a Statement I is given (C) Statement I is true and Statement II is false
and a corresponding Statement II is given just below it. (D) Statement I is false and Statement II is true
Mark the correct answer as:
(A) Both Statements I and II are true and Statement II 1. Statement I: Let A and B be two events and
is a correct explanation for Statement I A D B = ( A - B) È ( B - A)
(B) Both Statements I and II are true but Statement II
is not a correct explanation for Statement I Then
|P( A) - P( B)| £ P( A D B)
552 Chapter 7 Probability

( ) - P(B) is not greater than P(A


Statement II: P(A ( - B). are non-singular matrices. Therefore the probability is
Solution: We have 6 3
=
P( A) - P( B) £ P( A È B) - P( B) = P( A - B) 16 8
Answer: (B)
Statement II is true. Similarly
4. Statement I: If P(A È B) = 0.65 and P(A Ç B) = 0.15,
P( B) - P( A) £ P( A È B) - P( A) = P( B - A)
then
Therefore
P( A) + P( B) = 1.2
| P( A) - P( B)| £ P( A - B) + P( B - A) = P( A D B)
Statement II: For any two events E and F,
F
Statement I is correct.
P(E È F ) = P(E ) + P(F ) - P(E Ç F )
Answer: (A)
Solution: Statement II is the addition theorem for
2. Statement I: A is an event with non-zero probability. probabilities, and hence statement II is correct. Now
If A is independent of itself, then P(A) = 1.
P( A) + P( B) = 1 - P( A) + 1 - P( B)
Statement II: Two independent events cannot be
= 2 - ( P( A) + P( B))
mutually exclusive.
= 2 - ( P( A È B) + P( A Ç B))
Solution: A is independent of itself. This implies
= 2 - (0.65 + 0.15)
P( A) = P( A Ç A) = P( A)P( A)
= 1.2
Þ P( A) = 1 (∵ P( A) ¹ 0) Answer: (A)
Statement I is true. If E and F are independent, then 5. Statement I: A and B are two events such that
P(E Ç F ) = P(E )P(F ) ¹ 0
1 1 1
P( A) = , P( B) = and P( A Ç B) =
but if E and F are mutually exclusive, then P(E Ç F)
F = 0. 2 3 6
Answer: (B) Then

3. Consider the system of equations ax + by = 0 and cx + 1


P(exactly one of A and B) =
dy = 0, where a, b, c, d belong to the set {0, 1}. 2
Statement I: The probability that the system has Statement II: For any two events E and F
unique solution is 3/8.
P(exactly one of E and F ) = P(E) + P(F ) - 2 P(E Ç F )
Statement II: The probability the system has a solu-
tion is 1. Solution: We have
Solution: Statement II is correct, because x = 0, y = 0 is P(exactly one of E and F )
always a solution. The system has unique solution if and
= P((E - F ) È (F - E))
only if the matrix
= P (E - F ) + P (F - E )
éa b ù
êc d ú = P(E ) - P(E Ç F ) + P(F ) - P(F Ç E )
ë û
(∵ E - F = E - E Ç F )
is non-singular. Since each of a, b, c and d is either 0 or 1 = P(E ) + P(F ) - 2 P(E Ç F )
it follows that the number of matrices is 24 out of which
the matrices Therefore Statement II is correct. Now

é 1 0 ù é 1 1ù é1 0 ù P(exactly one of A and B) = P( A) + P( B) - 2 P( A Ç B)


ê0 1ú , ê0 1ú , ê1 1ú , 1 1 æ 1ö
ë û ë û ë û = + - 2ç ÷
2 3 è 6ø
é1 1ù é0 1ù é0 1ù
ê1 0 ú , ê 1 0 ú , ê 1 1ú =
1
ë û ë û ë û
2
Answer: (A)
Worked-Out Problems 553

6. A and B are two equally strong players in tennis. 4 1


Consider the two events = =
16 4
E1: A beats B exactly in 3 games out of 4. 5 3
E2: A beats B exactly 5 games out of 8. æ 1ö æ 1ö
P(E2 ) = 8C5 ç ÷ ç ÷
è 2ø è 2ø
Statement I: E1 is more possible than E2.
56 7
Statement II: In a binomial distribution B(n, p) = =
where n is the number of trials and p is the probability 28 32
of success in a single trial and q = 1 - p, the probability Therefore
k success = nCk p kqn-k.
P(E1 ) > P(E2 )
Solution: Statement II correct (see Section 7.6.1). Now
Answer: (A)
P(E1 ) = C3 × p q
4 3

3
æ 1ö æ 1ö æ 1ö
=4´ç ÷ ç ÷ çè∵ p = q = ÷ø
è 2ø è 2ø 2

Integer Answer Type Questions


1. Let n be a positive integer and A = {1, 2, 3, …, n}. Let S
be the set of all mappings f :A ® A. The probability that
n-m=7-4=3
a randomly chosen element of S to be injective mapping
Answer: 3
is given to be 3/32. Then the value of n is .
Solution: Number of mappings from A to A is nn, that 3. A bag contains n balls (n > 3) of different colours.
is S contains nn elements. It is known that the number of If the bag has 3 white balls and the probability of
injections from A to A is n (see Theorem 6.18, page 293, drawing 2 white balls from the bag is 3/10, the value
Vol. 1). Therefore by hypothesis of n is .
n 3 4 Solution: By hypothesis
= =
nn 32 44 3
C2 3
n
=
So C2 10
n=4 3´2 3
=
Answer: 4 n(n - 1) 10
n2 - n - 20 = 0
2. 5 pairs of socks are in a cupboard of which 4 socks
are drawn randomly. The probability that the drawn (n - 5)(n + 4) = 0
sample contains exactly one pair is m/n, where m
n=5
and n are natural numbers having no common factor
except unity. Then n - m is . Answer: 5
Solution: 4 socks out of 10 can be selected in 10C4 = 210
4. A sportsman’s chance of shooting an animal at a
ways. Let E be the event that the draw contains only one
distance r > a (“a” is constant) is given to be a2 / r2.
pair. Out of 5 pairs 1 pair can be selected in 5C1 = 5 ways.
He fires at r = 2a and if he misses, then again fires at
From the remaining 8 socks 2 can be selected in 8C2 = 28
r = 3a. He repeats the same process at r = 4a, 5a and
ways. But these 28 ways contain 4 pairs also. Therefore
6a. When he misses at r = 6a, the animal escapes into
number of ways of selecting exactly one pair is
the jungle. If the odds against the sportsman are p : q,
5 ´ (28 - 4) = 5 ´ 24 then q - p is .
Solution: The sportsman’s chance of missing when r = ja is
Hence
5 ´ 24 4 m a2 1
P(E ) = = = 1- 2 2
= 1 - 2 ( j = 2, 3, 4, 5, 6)
210 7 n ja j
554 Chapter 7 Probability

The animal escapes when the sportsman misses in all the By hypothesis
five shots. Therefore the probability of animal escaping
23
to jungle is P( B) =
90
6 æ 1ö æ 1 öæ 1öæ 1 öæ 1öæ 1ö
Õ ç1 - 2 ÷ = ç1 - 2 ÷ ç1 - 3 ÷ ç1 - 4 ÷ ç1 - 5 ÷ ç1 - 6 ÷ Therefore
j=2 è j ø è 2 øè 2 øè 2 øè 2 øè 2 ø
3n + 4(n + 4) 23
éæ 1ö æ 1ö æ 1ö æ 1ö æ 1ö ù =
= êç 1 - ÷ ç1 - ÷ ç1 - ÷ ç1 - ÷ ç1 - ÷ 18(n + 4) 90
ëè 2ø è 3ø è 4ø è 5ø è 6 ø úû
35n + 80 = 23(n + 4)
éæ 1ö æ 1ö æ 1ö æ 1ö æ 1ö ù
´ êç 1 + ÷ ç1 + ÷ ç1 + ÷ ç1 + ÷ ç1 + ÷ 12 n = 12
ëè 2ø è 3ø è 4ø è 5ø è 6 ø úû
n=1
æ 1 2 3 4 5ö æ 3 4 5 6 7 ö
=ç ´ ´ ´ ´ ÷ç ´ ´ ´ ´ ÷ Answer: 1
è 2 3 4 5 6ø è 2 3 4 5 6ø
1 7 7 p 6. A and B are two independent events whose probabili-
= ´ = =
6 2 12 q ties are, respectively, 1/n and 1/(n + 1). If the proba-
bility of A Ç B is 1/12, then n equals .
Therefore
Solution: A and B are independent events. This implies
q - p = 12 - 7 = 5
P( A Ç B) = P( A)P( B)
Answer: 5
Therefore
5. Boxes B1, B2, B3 contain different coloured balls as
1
given in Table 7.9. The probabilities of selecting boxes = P( A)P( B)
are, respectively, 1/6, 1/2 and 1/3. One of the boxes is 12
chosen at random and a ball is drawn from it. If the 1
=
probability of the drawn ball is black is 23/90 then the n(n + 1)
value of n is equal to .
which gives
Table 7.9 Integer answer type question 5 (n is
a positive integer) n=3
Answer: 3
White Black Red
B1 2 n 2 7. A number x is selected from the set of first 9 natural
numbers (i.e., x = 1, 2, 3, …, 9). If the probability that
B2 3 2 4
f x)) = x where
ff( f(
B3 4 3 2
f ( x) = x2 - 3 x + 3

Solution: Let B be denote the event of drawing a black is m/9, then m is equal to .
ball. Then
Solution: Clearly all the solutions of f( f x) = x are also
B = ( B1 È B2 È B3 ) Ç B solutions of f( f x)) = x. First, we solve f(
f f( f x) = x.
= ( B1 Ç B) È ( B2 Ç B) È ( B3 Ç B) f ( x) = x Þ x2 - 3 x + 3 = x

Therefore Þ x2 - 4 x + 3 = 0
Þ ( x - 1)( x - 3) = 0
P( B) = P( B1 )P( B / B1 ) + P( B2 )P( B / B2 ) + P( B3 )P( B / B3 )
Þ x = 1, 3
1 n 1 2 1 3
= ´ + ´ + ´
6 n+4 2 9 3 9 Therefore x = 1, 3 are also solutions of f( f x)) = x. We
f f(
n want to seek if there are any more solutions of f( f x)) = x
f f(
2
= + other than 1 and 3.
6(n + 4) 9
f ( f ( x)) = x Þ f ( x2 - 3 x + 3) = x
3n + 4(n + 4)
=
18(n + 4) Þ ( x2 - 3 x + 3)2 - 3( x2 - 3 x + 3) + 3 = 0
Worked-Out Problems 555

Þ x4 - 6 x3 + 12 x2 - 9 x + 3 = 0 since

Þ ( x2 - 4 x + 3)( x2 - 2 x + 1) = 0 1
P( P1 pairing with X ) =
4
Þ ( x - 1)( x - 3)( x - 1)2 = 0
as there are four pairs.
Þ x = 1, 3
P(P1 to lose in II) = P(P1 wins in I)P(P1 pairing with X in II)
the probability that x satisfies equation f( f x)) = x is 2/9.
f f( P( P1 losing)
Therefore m = 2.
1 1 1
Answer: 2 = ´ ´
2 2 2
8. A fair coin is tossed n times. Let X denote the number 1
=
of heads appeared. If P(X = 4), P(X = 5) and P(X = 6) 8
are in AP, then the smallest values of n is .
P( P1 to lose in III) = P( P1 wining I and II) P( P1 loosing)
Solution: Since the coin is fair,
æ 1 1ö 1
1 =ç ´ ÷ ´
P( H ) = P(T ) = è 2 2ø 2
2
1
By binomial distribution, =
8
n- K K
æ 1ö æ 1ö
P ( X = K ) = n CK ç ÷ çè ÷ø Therefore probability of P1 losing to X is
è 2ø 2
1 1 1 3
æ 1ö
n + + =
= CK ç ÷
n 8 8 8 8
è 2ø
Hence
By hypothesis
n-m=8-3=5
2 P( X = 5) = P( X = 4) + P( X = 6) Answer: 5
Therefore
10. The odds against an event A is 2 :3 and odds in
2(nC5 ) = nC4 + nC6 favour of another event B is 1:2. If A and B are
independent and P( A È B) = m / n, then | m - n| is
n2 - 21n + 98 = 0
. Here m and n do not have proper common
n = 7, 14 divisor.

Therefore n = 7 (smaller value). Solution: We have


Answer: 7 P( A È B) = P( A) + P( B) - P( A Ç B)

9. P1, P2, …, P8 are equally strong players (i.e., for each 3 1 3 1


= + - ´
of them the probability of win or lose is 1/2) are partic- 5 3 5 3
ipating in tennis singles tournament. If the probability
9+5-3
of P1 losing to eventual winner of the tournament is =
m/n then n - m is equal to . 15
11
Solution: Let X be the eventual winner. P1 may lose to =
X in I, II and III rounds. 15

P( P1 to lose in I) = P( P1 pairing with X and losing) Therefore m = 11 and n = 15. So |m - n| = 4.


Answer: 4
1 1
= ´
4 2
556 Chapter 7 Probability

SUMMARY
7.1 Random experiment: An experiment is called 7.8 Usual probability: The classical probability is also
random experiment if the following conditions are called usual probability.
satisfied.
(1) The experiment can be repeated any number of 7.9 Sample points and sample space: Any possible
times under similar conditions. outcome of a random experiment is called a sample
point and the set of all sample points is called
(2) All possible outcomes of the experiment are
the sample space of the random experiment. An
known in advance
elementary event means a sample point. Generally
(3) The actual outcome in a particular experiment sample space is denoted by S.
cannot be exactly predicted.
7.10 Finite sample space: A set A is called finite if either
7.2 Sample space and event: The set of all possible A is an empty set or it is bijective with the set {1,
outcomes of a random experiment is called sample 2, 3, …, n} for some positive integer n. If a sample
space of the experiment and any subset of the sample space is finite, then it is called a finite sample space.
space is called an event.
7.11 Countably infinite sample space: A set A is called
7.3 Exhaustive events: Two or more events are called countably infinite set if it is bijective with the set
exhaustive events if the performance of the experi- + of all positive integers. If the sample space of a
ment results in the occurrence of at least one of random experiment is countably infinite set, then
these events. the sample space is called countably infinite. For
example, tossing a fair coin till head appears has a
7.4 Mutually exclusive events: Two or more events are countably infinite sample space.
said to be mutually exclusive if the occurrence of one
of the events prevents the occurrence of any one of 7.12 Definition: Here afterwards events mean subsets
the other events. of the sample space. If A and B are two events, then
A È B means at least one of A or B and A Ç B
7.5 Equally likely events: Two or more events are said means both A and B. Impossible event is denoted
to be equally likely (or equiprobable) if there is no by empty set f and a certain event means the entire
reason to expect one of them in preference to the sample space.
others.
7.13 Various events in set theoretical form:
7.6 Probability (classical definition): Suppose in a
(1) Events E1, E2, …, En are said to be mutually
random experiment there are n exhaustive, mutually
exclusive if Ei Ç Ej = f for i ¹ j.
exclusive, equally likely outcomes. If m of them are
favourable to an event E, then the probability P(E) (2) E1, E2, …, En are called exhaustive events if E1 È
of E is defined as E2 È  È En = S (sample space).
(3) Mutually exclusive and exhaustive events means
m
P(E ) = E1 È E2 È  È En = S and Ei Ç Ej = f for i ¹ j.
n
(4) E1, E2, …, En are called equally likely (or equi-
probable) if there is no reason to expect one of
QUICK LOOK
them in preference to the others.
0 £ P(E) £ 1 (∵ 0 £ m £ n)
7.14 Probability function (Axiomatic): Let S be the
sample space connected with a random experi-
ment and Ã(S) is the power set of S (i.e., the set
7.7 Complementary event: If E is an event, then non-
of all subsets of S). Then a function P : Ã(S) ® 
occurrence of E is called complementary eventt of E
is called a probability function on the sample S, if it
and is denoted by E (or Ec).
satisfies the following conditions:
(1) P(E) ³ 0 for all E ÎÃ(S)
QUICK LOOK
(2) P(S) = 1
n-m m (3) If E1 and E2 belong to Ã(S) and E1 Ç E2 = f, then
P(E ) =
c
= 1 - = 1 - P(E )
n n
P(E1 È E2 ) = P(E1 ) + P(E2 )
Summary 557

If P is a probability function on S, then for any 7.18 Conditional probability: Let A and B be two events
E ÎÃ(S), P(E) is called the probability of the event E. and P be a probability function with P(B) > 0, then
If E = {s}, then we write P(s) instead of P({s}). we define P( A Ç B)/ P( B) as the conditional prob-
If S is a countably infinite space then (3) will be replaced ability of the occurrence of event A after the occur-
by the following: rence of the event B and it is denoted by P( A / B).
(3¢) If { En } is a sequence of mutually exclusive events, That is
then
P( A Ç B)
¥ ¥ P( A / B) =
P æ È En ö = å P(En )
P( B)
è n=1 ø n=1
QUICK LOOK
provided the infinite sum exists.
If the sample space S is finite, then
QUICK LOOK Number of favourable cases to both A and B
P( A /B) =
One can verify that the usual probability (classical Number of favourablee cases to B
definition) is also a probability function according to
modern probability definition given above in 7.14.
7.19 Multiplication theorem: Let A and B be two events
of a random experiment with positive probabilities,
7.15 Theorem: The following hold good for any prob- then
ability function P defined on a sample space S. P( A)P( B / A) = P( A Ç B)
(1) P(f) = 0
= P( B)P( A / B)
(2) P(E ) = 1 - P(E )
(3) 0 £ P(E) £ 1 for all E ÎÃ(S) 7.20 Corollary: Let E1, E2, …, En be n events with posi-
n-1
(4) P(E1 - E2) = P(E1) - P(E1 Ç E2) and in particular tive probabilities and P æ Ç Ei ö > 0, then
P(E1 - E2) = P(E1) - P(E
E2) whenever E2 Ì E1 and è i =1 ø
hence E2 Ì E1 Þ P(E1) ³ P(E E2).
P(E1 Ç E2 Ç  Ç En ) = P(E1 )P(E2 / E1 )P(E3 /(E1 Ç E2 ))
(5) If E1, E2, …, En are mutually exclusive events,
n-1
then  P(En /( Ç Ei ))
i =1
P(E1 È E2 È  È En ) = P(E1 ) + P(E2 ) +  + P(En )
7.21 Definition (independent events): Two events A
7.16 Definition (Odds in favour, odds against): If E is and B are said to be independent of each other if
an event, E is its complementary event and P is a the occurrence of one does not effect the occur-
probability function, then the ratio P(E): P(E) is rence of the other. That is, B is independent of A,
called odds in favour of E and the ratio P(E): P(E) if P( B / A) = P( B).
is called odds against E.
7.22 Theorem: Two events A and B are independent if
7.17 Addition theorem: If A and B are two events of a and only if P(A Ç B) = P(A)P(B).
random experiment, then
7.23 Theorem (total probability): Let E1, E2, …, En be
P( A È B) = P( A) + P( B) - P( A Ç B)
mutually exclusive and exhaustive events with posi-
and in particular, if A Ç B = f, then tive probabilities. Then for any event E,
P( A È B) = P( A) + P( B)
n
P(E) = å P(Ej )P(E / Ej )
j =1

QUICK LOOK
7.24 Bayes’ theorem: Let E1, E2, …, En be mutually
If A, B and C are any three events, then
exclusive and exhaustive events with positive prob-
P( A È B È C ) = P( A) + P( B) + P(C ) abilities. If E is an event then
- P( A Ç B) - P( B Ç C ) P(EK )P(E / EK )
P(EK / E) = n
- P(C Ç A) + P( A Ç B Ç C ) å P(E )P(E / E )
j j
j =1
558 Chapter 7 Probability

7.25 Definition (random variable): If S is a sample Table 7.10


space and  is the real number set then any func-
X x1 x2 x3… xn…
tion from S to  is called a random variable on
S. Generally, random variable is denoted by the (X = xn) p1 p2 p3… pn…
capital letters X,
X Y,
Y Z, etc.
7.31 Formula: If m and s 2 are, respectively, the mean
7.26 Probability function induced by a random vari- and variance of a random variable X whose range
able: Let S be a sample space, P a probability func- is {x1, x2, x3 …, xn, …} and P(X = xi) = pi, then
tion on S and X a random variable on S. Define
PX : Ã() ®  by PX(T)T = P(XX-1(T))
T for T ÎÃ(). s 2 + m 2 = å xn2 pn
Then PX is a probability function on  and is called n

the probability function induced by X on .


7.32 Table 7.11 enables for quick calculation of mean
7.27 Probability distribution: Let S be a sample space and variance.
with probability function P and X a random vari-
able. Then the function F :  ®  defined by Table 7.11

F (r ) = P( X £ r ), r Î X=x P(X = x) xP(X = x) x2P(X = x)

is called the probability distribution function of the x1 p1 = P(X = x1) x1 p1 x21 p1


random variable X.X x2 p2 = P(X = x2) x2 p2 x22 p2
x3 p3 = P(X = x3) x3 p3 x23 p3
7.28 Theorem: Let X be a random variable on a sample    
space S and F the probability distribution function xn pn = P(X = xn) xn pn 2
xn pn
of X
X. Then the following hold.    
(1) 0 £ F(
F r) £ 1 for all r Î. Total = m Total = s 2 + m2
(2) r £ s Þ F(
F r) £ F(
F s) for any r and s of .
(3) lim F (r ) = 1 and lim F (r ) = 0.
r ®¥ r ®-¥ 7.33 Bernoulli trial: A random experiment in which we
(4) F (r + 0) = lim F ( s) = F (r ). consider an event called success and its comple-
s® r
s >r mentary called failure is called Bernoulli trial.

7.29 Definition (discrete random variable): A random 7.34 Binomial variate: Let n be the number of indepen-
variable X is called discrete if its range is finite or dent Bernoulli trials in which the probability for
countably infinite. a success is p and that of failure is q = 1 - p. If X
denotes the number of successes, then
7.30 Definition (mean and variance): Let X be a P( X = k ) = n Ck pk qn - k
random variable on a sample space S and suppose
the range of X = {x1, x2, …, xn, …}. Let P be a prob- P(X = k) is called the probability of k successes and
ability function on S. Let P(X = xn) = P(X X-1{xn}). n – k failures. n and p are called parameters of the
Then binomial variate X and we write X ~ B(n, p).
(1) If å x P( X = x ) is finite, then it is called the
n
n n
7.35 Theorem (mean and variance of X ~ B (n, p)):
mean of X and is denoted by mX or simply m If X ∼ B(n, p), then the mean of X is np and the
when there is no confusion about X
X. That is variance is npq.
m = å xn P( X = xn )
n QUICK LOOK

The mean of a binomial variate is greater than or


(2) If å (x- m )2 P( X = xn ) is finite, then it is called
n
equal to its variance.
variance, denoted by s 2 and | s | is called stan-
dard deviation of X.X Note: If a set of n Bernoulli trials are repeated N
times, then the expected number of sets which contain
(3) Table 7.10 is called the probability distribution
r success and n – r failures is N(nCr prqn-r).
table of X where pi = P(X = xi) = P(X X-1({xi})).
Exercises 559

7.36 Poisson distribution: Let X be a random variable 7.37 Theorem: The mean and variance of a Poisson
whose range is {0, 1, 2, 3, …, n, …} and l a positive variate X with parameter l are equal and equal
number. We define to l.
Note: In Vol. 3 (Calculus) we will prove that the Poisson
e- l l k
P( X = k ) = distribution is a limiting case of binomial distribution.
k

then X is called a Poisson variate and l is called the


parameter of X.X

EXERCISES
Single Correct Choice Type Questions
1. A bag contains 8 white, 6 black, 2 red and 4 green 8. An ellipse of eccentricity 2 2 / 3 is inscribed in a
balls. Two balls are picked at random at a time. The circle. A point inside the circle is selected at random.
probability that they are of different colours is The probability that the point lies outside the
7 9 14 12 ellipse is
(A) (B) (C) (D)
19 19 19 19 2 1 1 3
(A) (B) (C) (D)
3 3 2 4
2. An urn contains 6 white, 4 red and 10 green marbles.
Three are drawn one after other without replacement. 9. Two subsets A and B are chosen at random from
The probability that they are of different colours is a set containing 6 elements one after other with
2 3 6 2 replacement. The probability that A and B contain
(A) (B) (C) (D) equal number of elements is
9 19 19 57
131 231 331 101
(A) (B) (C) (D)
3. A bag contains 4 black and 7 white balls. Two balls 1021 1021 1021 1021
are drawn randomly. The probability that at least one
of them is black is 10. A lot contains 12 cell phones out of which 4 are
(A) 0.71 (B) 0.618 (C) 0.728 (D) 0.628 defective. If two cell phones are drawn from the lot
at random, the probability that one is defective and
4. Three fair coins are tossed at a time. The probability the other is non-defective is
of getting at least one head is 16 15 13 17
(A) (B) (C) (D)
7 1 3 4 33 33 33 33
(A) (B) (C) (D)
8 8 8 8
11. On the real line (i.e., x-axis) points p and q are
5. There are 9 books of which one is very popular. The selected at random such that -2 £ q £ 3 and 0 £ p £ 3.
books are arranged in a row at random. The probability The distance between p and q is greater than 3 with
that the popular book is always at the middle place is probability
2 1 (4!)2 2(4!)2 2 1 1 3
(A) (B) (C) (D) (A) (B) (C) (D)
9 9 9! 9! 3 3 4 4

6. The letters of the word PECULIAR are arranged 12. A and B are two events such that
at random. The probability that all the vowels are 3 1 1
together is P( A) = , P( B) = and P( A Ç B) =
8 2 4
3 1 1 3
(A) (B) (C) (D) Then P( A Ç B) is equal to
7 7 14 14
1 1 1 1
(A) (B) (C) (D)
7. A and B are two students among six students. If all 12 3 16 4
the six students are arranged in a row, the probability
that B immediately succeeds A is 13. A bag contains 4 red, 3 white, 2 blue and 1 green
1 1 2 2 marbles. Four marbles are drawn from it at random.
(A) (B) (C) (D)
5 6 5 3
560 Chapter 7 Probability

The probability that the draw contains one each 1/2, 3/4 and 1/4. They try the problem independently.
from the given colours is The probability that majarity of them solving the
1 6 4 31 problem is
(A) (B) (C) (D)
7 7 35 35 1 1 2 3
(A) (B) (C) (D)
3 2 3 4
14. A gardner plants 3 apple trees, 4 neem trees and
5 mango trees in a row at random. The probability 22. A lot contains 15 cameras out of which 4 are defec-
that no two mango trees are next to one another is tive. Cameras are drawn one after other to identity
7 1 8 1 the defective pieces. The process continues till all
(A) (B) (C) (D) the defective pieces are identified. The probability
99 9 99 11
that the proceedure ends at the ninth testing is
15. The first 11 letters of the English alphabet are arranged 11 13 8 9
(A) (B) (C) (D)
in all possible ways at random. The probability that 195 195 195 195
there are exactly 4 letters in between A and B is
12 14 7 6 23. The probability that man aged x years to die in five
(A) B) (C) (D) years is 1/3. Out of persons P1, P2, P3, P4 and P5, each
55 55 55 55
aged x years, P1 will die in 5 years and he is the first
person to die in 5 years is
16. Two positive real numbers x and y such that x + y =
12 are selected at random. The probability that xy 211 311 111 1
(A) (B) (C) (D)
is greater than or equal to 3/4 times their greatest 1215 1315 1115 5
product is
1 1 2 3 24. The probabilities of three mutually exclusive events are
(A) (B) (C) (D)
3 2 3 4 1 + 3p 1 - p 1 - 2 p
, ,
3 4 2
17. From 15 consecutive positive integers, 3 are selected
at random. The probability that they are in AP is Then
3 7 9 11 1 1 1
(A) (B) (C) (D) (A) 0 < p £ (B) £ p£
65 65 65 65 3 3 2
1 1 1
18. Five numbers are selected at random from the first 50 (C) < p<1 (D) £ p £
2 5 4
natural numbers and arranged in the increasing order
x1 < x2 < x3 < x4 < x5. The probability that x3 is 30 is 25. Two events A and B are such that P(A) = 0.7,
29
C2 ´ 20C2 39
C2 ´ 19C2 P(B) = 0.4 and P( A Ç B) = 0.5. Then P( B /( A È B))
(A) (B)
50
C5 50
C5 is equal to
30
C2 ´ 20C2 55 1 1 1 2
(C) (D) (A) (B) (C) (D)
50
C5 1029 2 3 4 3

19. Two different coloured dice are rolled. A is the event 26. A fair die is rolled four times. Out of the four face
that the sum of the faces appeared is odd and B is values obtained, the probability that the minimum
the event that one of the faces turned up shows face value is greater than or equal to 2 and the
face 1. Then P( A Ç B) is maximum value is less than or equal to 5 is
1 2 1 3 16 1 8 65
(A) (B) (C) (D) (A) (B) (C) (D)
3 3 4 4 81 81 81 81

20. Two dice are thrown at a time. The probability that 27. A purse contains four 50 paise coins and three
the sum is greater than 8 is Rs. 1 coins. Another purse contains six 50 paise coins
and two Rs. 1 coins. One of the purses is selected
1 1 1 5
(A) (B) (C) (D) random and a coin is drawn from it. The probability
6 9 3 18 that it is a 50 paise coin is
35 25 37 27
21. A problem in combinatorics is given to three students (A) (B) (C) (D)
whose probabilities of solving it are, respectively, 56 56 56 56
Exercises 561

28. If each of a, b and c takes values from the set {1, 2, is in a police court. The probability that they have
3, 4, 5, 6}, then the probability that the equation given identical statements is
ax2 + bx + c = 0 has real roots is equal to 5 1 7 2
(A) (B) (C) (D)
53 43 23 63 12 3 12 3
(A) (B) (C) (D)
216 216 216 216
36. There are five pairs of socks in a cupboard. If 4 socks
29. In a regular hexagon ABCDEF,
F if three vertices are are drawn at random, the probability that the draw
selected at random, then the probability that they contains exactly one matching is
form the vertices of an equilateral triangle is 8 2 5 13
(A) (B) (C) (D)
1 1 2 1 21 7 7 21
(A) (B) (C) (D)
5 3 3 10
37. In a bolt-producing factory, machines A, B and C
30. From the set {1, 2, 3, …, 15}, seven numbers are give 25%, 35% and 40% of the total output, respec-
selected at random one after other with replace- tively. It is also known that 5%, 4% and 2% of them
ment. The probability that the largest of these is 9 is are defective. If a bolt of the factory is selected at
7 7 7 7 random then the probability that it is defective is
æ 8ö æ 9ö æ 7ö æ 3ö
(A) ç ÷ (B) ç ÷ (C) ç ÷ (D) ç ÷ 69 63 73 83
è 15 ø è 10 ø è 10 ø è 5ø (A) (B) (C) (D)
2000 2000 2000 2000
31. Two cards are drawn at random from a deck of
38. A and B are two politicians who settle disputes
52 playing cards one after other, replacing the earlier
drawn card. The probability that the first card is between parties independently. The probabilities of
diamond and the second card is Queen is their settling disputes amicably are, respectively, 4/7
and 7/15. A firm has engaged them for a settlement
3 4 5 1 of dispute between the firm and the employees. The
(A) (B) (C) (D)
13 13 52 52 probability that neither of them settle the dispute is
73 32 8 16
32. Let S be a four-element set. If a two-element subset (A) (B) (C) (D)
105 105 35 35
of S is selected at random, the probability that it
belongs to a partition class of S is
39. S is a five-element set. A subset P of A is selected
2 1 3 4 at random. After inspecting the elements of P,
(A) (B) (C) (D)
9 5 5 5 the elements are restored to S. Again a subset Q
(Hint: See Definition 1.28 and Single Correct Choice of S is selected at random. The probability that P
Type Question 13, Chapter 1, Vol. 1, page 60.) and Q form a partition of S (i.e., P È Q = S and
P Ç Q = f) is
33. Let w ¹ 1 be a cube root of unity. A fair die is 1 1 1 1
(A) (B) (C) (D)
thrown three times. If r1, r2 and r3 are the numbers 2 16 8 32
that appeared on the faces, the probability that
wr1 + wr2 + wr3 = 0 is 40. A set X has 10 elements. Subset A of X is selected at
1 1 2 1 random and then the elements of A are returned to
(A) (B) (C) (D) X. Again a subset B of X is chosen at random. The
X
18 9 9 36
probability that B is disjoint with A is
10 5
34. “A” is one of the six horses entered for a race and it 1 1 æ 3ö æ 3ö
(A) (B) (C) ç ÷ (D) ç ÷
is to ridden by one of the two jockeys B and C. If B 210 25 è 4ø è 4ø
rides A, then all the horses are equally likely to win.
If C rides A, the chances of A’s win will be trebled, 41. Three groups of workers contain 3 women, 1 male;
then, the odds in favour of A is 2 women and 2 male and 3 males and 1 woman. One
(A) 1: 2 (B) 2 :1 (C) 1: 3 (D) 3 :1 worker from each group is selected for a work at
random. The probability that the selection consti-
35. A and B are two persons. A speakes truth 2 out of tutes 2 males and 1 lady is
3 times, while B speaks truth 3 out of 4 times. They 9 3 1 13
(A) (B) (C) (D)
are independent witnesses in an accident case which 32 32 32 32
562 Chapter 7 Probability

42. Two persons A and B each tosses a fair coin thrice. n+2 n+1 m+2 m+1
(A) (B) (C) (D)
The probability that they get same number of heads is 2m+1 2m+1 2n+1 2n+1
5 11 7 9
(A) (B) (C) (D) 47. Three identical bags contain the following composi-
16 16 16 16
tion of balls.
43. If A and B are two independent events in a random Bag I: 2 white, 1 black
experiment then P( A È B) + P( A Ç B) is equal to Bag II: 1 white, 2 black
Bag III: 2 white, 2 black
1 2
(A) 0 (B) (C) 1 (D) The probabilities of selecting the bags are equal.
2 3 One of the bags is selected and a ball is drawn from
it. The drawn ball is found to be black and it is kept
44. Three bags have the following combination of balls. outside. Then the probability of a black ball again is
Bag I: 7 black, 3 white
1 2 2 3
Bag II: 4 black, 6 white (A) (B) (C) (D)
Bag III: 2 black, 8 white 3 3 5 5
The probabilities of selecting bags are, respectively,
1/5, 3/5 and 1/5. One of the bags is chosen at random 48. A fair die is rolled. The probability that first 1 appears
and two balls are drawn from it, one after other at even number of trials is
without replacement. The probability that both balls 6 1 5 5
(A) (B) (C) (D)
are black is 11 6 36 11
4 1 41 8
(A) (B) (C) (D) 49. An experiment has 10 equally likely outcomes. Let
45 9 45 45
A and B be two non-empty events of the experi-
(Hint: Total probability.) ment. If A consists of 4 outcomes, the number of
outcomes B should have such that A and B are inde-
45. Five men out of 100 and 25 women out of 1000 have pendent is
colour blindness. If a colour blind person is selected
at random, the probability that the person is a male is (A) 2, 4 or 3 (B) 3, 6 or 9
1 2 1 1 (C) 4 or 8 (D) 5 or 10
(A) (B) (C) (D)
3 3 2 4
50. If the mean and variance of a Binomial variate X
(Hint: Use Bayes’ theorem.) are, respectively, 2 and 1, then P(X > 1) is
11 5 7 9
46. A fair coin is tossed (m + n) times where m > n. The (A) (B) (C) (D)
16 16 16 16
probability for m consecutive heads is

Multiple Correct Choice Type Questions


1. If A and B are independent events, then 3. Let P be a probability function on a sample space S
(A) A and B are independent and B a fixed non-empty subset of S. For any subset
A of S, define
(B) A and B are independent
(C) A and B are independent PB ( A) = P( A / B)
(D) A and B are exclusive events Then
(A) PB(S) = 1
2. Two different coloured fair dice are thrown at a time.Then
P( A Ç B Ç C )
1 (B) PB ( A Ç C ) =
(A) (Sum of the faces is odd) = P( B)
2
(C) PB(A È C) = PB(A) + PB(C) - PB(A Ç C)
11
(B) P(At least one face is odd) = (D) PB(A Ç C) = PB(A) PB(C), whenever A and C are
36
independent.
1
(C) P(Sum of the faces is 5) =
9 4. Let A, B, C be mutually exclusive and exhaustive
25 events of a random experiment with sample space S.
(D) P(Neither of the faces is 1) =
36
Exercises 563

Suppose P is a probability function on S. Which of the (A) mutually exclusive


following are not correct? (B) independent
(A) P(A) = 0.24, P(B) = 0.4, P(C) = 0.2 (C) exhaustive and dependent
1 1 1 (D) such that one is twice as likely as the other
(B) P( A) = , P( B) = , P(C ) =
4 4 2
(C) P(A) = 0.1, P(B) = 0.2, P(C) = 0.6 8. The letters of the word PROBABILITY are arranged
(D) P( A) = 0.6, P( A Ç B) = 0.5 in a row at random. Let E1 be the event that the two
I’s are together and E2 is the event that two B’s are
together. Then
5. Let A and B be two events of a random experiment.
Suppose P(A) = p1, P(B) = p2 and P(A È B) = p3. Then (A) P(E1 Ç E2 ) =
2
(B) P(E1 È E2 ) =
18
55 55
(A) ( A È B) = 1 - p1 + p3
1 2
(B) P( A Ç B) = 1 - p1 - p2 + p3 (C) P(E1 / E2 ) = (D) P(E2 / E1 ) =
5 5
(C) P( A È ( A Ç B)) = p1 + p2 - p3
(D) P( A È B) = p1 + p2 - p3 9. The probabilities of a 12th standard student passing
the subjects Maths, Physics and Chemistry are,
respectively, m, p and c, the probability of passing
6. Two fair dice are thrown at a time. Let A, B, C be the
at least one subject is 3/4, the probability of passing
following events:
at least two subjects is 1/2 and the probability of
A = {(x, y) | x is odd} passing exactly two subjects is 2/5. Which of the
B = {(x, y) | y is odd} following relations are true?
C = {(x, y) | x + y is odd} 19 27
(A) p + m + c = (B) p + m + c =
Then 20 20
(A) P(A Ç B) = P(A) P(B) 1 1
(C) pmc = (D) pmc =
(B) P(B Ç C) = P(B) P(C) 10 4
(C) P(C Ç A) = P(C) P(A)
10. A and B are independent events of a random experi-
(D) P(A Ç B Ç C) = P(A) P(B) P(C)
ment. If P( A Ç B) = 1/ 6 and P( A Ç B) = 1/ 3. Then
7. The probabilities of the events A Ç B, A, B and A È B
1 1
(A) P( A) = (B) P( A) =
of a random experiment are in AP in the given order, 2 3
with the common different as P(A). Assume that 1 1
(C) P( B) = (D) P( B) =
P(A) > 0. Then A and B are 3 2

Matrix-Match Type Questions


In each of the following questions, statements are given p q r s t
in two columns, which have to be matched. The state- A
ments in Column I are labeled as (A), (B), (C) and B
(D), while those in Column III are labeled as (p), (q),
C
(r), (s) and (t). Any given statement in Column I can
have correct matching with one or more statements in D
Column II. The appropriate bubbles corresponding to
the answers to these questions have to be darkened as 1. A, B and C are three events of a random experiment
illustrated in the following example. and P is a probability function. If P(A) = 0.6, P(B) =
Example: If the correct matches are (A) ® (p), (s); 0.4, P(C) = 0.5, P(A È B) = 0.8, P(A Ç C) = 0.3,
(B) ® (q), (s),(t); (C) ® (r); (D) ® (r), (t); that is if the P(A Ç B Ç C) = 0.2 and P(A È B È C) = 0.85, then
matches are (A) ® (p) and (s); (B) ® (q), (s) and (t); match the items of Column I with those of Column II.
(C) ® (r); and (D) ® (r), (t), then the correct darkening
of bubbles will look as follows:
564 Chapter 7 Probability

4. Match the items of Column I with those of Column II.


Column I Column II
(A) P(A Ç B) is (p) 0.35 Column I Column II
(q) 0.8
(B) P(B Ç C) is 1
(r) 0.2 (A) If two fair dice are rolled, then (p)
the probability of sum of the 4
(C) P( A / B) is equal to (s) 0.3 faces is 7 is
2 1
(D) P( A È B) equals (t) (B) If a card is picked from a deck (q)
3 of 52 playing cards, then the 2
probability of getting a red
card is
2. A, B, C are three events such that 3
(C) P(x) is a polynomial (r)
3 1 satisfying the relation 8
P( B) = , P( A Ç B Ç C ) =
4 3 P(x) + P(2x2 ) = 5x2 -18 for all
real x. Now, each coefficient
1
and P( A Ç B Ç C ) = a, b and c of the quadratic
3 expression ax2 + bx + c is one
Match items of Column I with those of Column II. of the roots of the equation
P(x) = 0. The probability that
ax2 + bx + c = 0 has real roots is
Column I Column II 1
(D) In class X of a school 75% (s)
are boys and 25% are girls. 6
1
(A) P( B Ç C ) is (p) Probability of boy getting
12
first class is 1/3 while girl
4
(B) P(B Ç C) is (q) getting first class is 1/2. If
9 one candidate is selected at
1 random, the probability of the
(C) P(( A Ç C )/ B) is (r)
3 candidate getting first class is
2
(D) P(( A Ç C )/ B) is (s)
3 5. The letters of the word MATHEMATICS are written
in a row at random. Match the items of Column I with
3. A and B are two independent events. It is given that those of Column II.
P( A Ç B) = 1/ 12 and P( A Ç B) = 1/ 2. Match the
items of Column I with those of Column II. Column I Column II
2
Column I Column II (A) Probability of the two (p)
M’s together is 11
1 4
(A) P(A) is equal to (p) (B) The probability that (q)
3 two M’s and two A’s 55
1 are together is
(B) P(B) is equal to (q)
6 6
(C) The probability that the (r)
1 55
(C) P( A Ç B) is (r) vowels are together is
2 1
(D) The probability of (s)
1 55
(D) P( A Ç B) is (s) beginning with M and
4 end with M is

Comprehension-Type Questions
1. Passage: A and B are two bags. A contains 4 fair coins sides. Two coins are transfered from bag A to bag B
and 3 counterfeit coins while B has 5 fair and 7 coun- and then a coin is taken from the bag B and tossed.
terfeit coins. The counterfeit coins have tail on both Answer the following three questions.
Exercises 565

(i) If both coins transfered from A to B are counter- (i) The proportionality constant is equal to
feit coins, then probability of getting a tail is 1 1 1 1
(A) (B) (C) (D)
3 5 1 23 74 148 112 504
(A) (B) (C) (D)
4 28 2 28
(ii) If E is the event of selecting a biased coin out of
(ii) If tail appears, then the probability of both coins 6 coins, then P(E1 / E) is
transferred from bag A are counterfeit is
1 1 1 2
1 23 23 23 (A) (B) (C) (D)
(A) (B) (C) (D) 266 132 133 133
7 28 153 66
(iii) If the coin selected is found to be biased, then
(iii) If head appears, then the probability of both P(E6 / E) is equal to
coins transferred from bag A are fair is
2 3 5 3
14 21 25 14 (A) (B) (C) (D)
(A) (B) (C) (D) 19 38 38 19
43 43 43 19
4. Passage: There are n urns numbered from 1 to n. The
2. Passage: In a tennis tournament, there are 12 players Kth urn contains K white balls and (n + 1 - K) black
K
S1, S2, …, S12 and they are divided into 6 pairs at random. balls. Let Ek denote the event of selecting the K
Kth urn
From each pair, the winner will be decided on the basis at random and let W denote the event that the ball
of the game played between the two players of the pair. drawn from the selected urn is white. Let P(A) denote
Assume that all players are of equal strength so that the probability of an event A. Answer the following
the probability of any player beating any other player questions.
is 1/2. Answer the following questions.
(i) If P(EK) is proportional to K for K = 1, 2, 3, …, n,
(i) The probability that S1 and S2 are not playing then lim P(W ) is
against other is n®¥

1 10 1 5 1 2 1 3
(A) (B) (C) (D) (A) (B) (C) (D)
2 11 11 22 3 3 4 4

(ii) The probability that exactly one of S1 and S2 is (ii) If P(EK) = C, a constant for all K = 1, 2, 3, …, n,
among the losers is then P(En /W ) is equal to
1 5 6 5 2 1 n 1
(A) (B) (C) (D) (A) (B) (C) (D)
2 22 11 11 n+1 n+1 n+1 2
(iii) The probability that both S1 and S2 are among (iii) If P(EK ) = 1/ n for all K = 1, 2, 3, …, n and E
the winners is denote the event of choosing an even numbered
1 5 6 5 urn, then P(W / E) is equal to
(A) (B) (C) (D)
4 11 11 12 1 n+4
(A) (B)
3. Passage: A box contains 6 coins out of which at least 2n 2(n + 1)
one is biased. Let EK(1 £ K £ 6) denote the event that n+2 n+1
(C) (D)
exactly K out of 6 coins are biased. Also let P(EK) 2(n + 1) 2n
be directly proportional to K(K + 1). Answer the
following questions.

Assertion–Reasoning Type Questions


In the following set of questions, a Statement I is given 1. A fair coin is tossed 3 times. Let A be the event that
and a corresponding Statement II is given just below it. the first toss gives head; B is the event that the second
Mark the correct answer as: toss gives head and C be the event that there are
(A) Both Statements I and II are true and Statement II exactly two consecutive heads or exactly two consecu-
is a correct explanation for Statement I tive tails.
(B) Both Statements I and II are true but Statement II Statement I: A, B, C are independent events
is not a correct explanation for Statement I
(C) Statement I is true and Statement II is false Statement II: A, B, C are pairwise independent.
(D) Statement I is false and Statement II is true
566 Chapter 7 Probability

2. Suppose two persons A and B have n + 1 and n coins. Statement I: P(Ei / E) > P(Ei )P(E / Ei )
All the coins are fair. They toss their coins.
n
Statement I: The probability that A gets more heads Statement II: å P(E ) = 1
i =1
i
than B is 1/2.
Statement II: The probability that A gets more heads 5. A random number selector can only select one of the
than B is the same as A gets more tails than B. numbers from the set {1, 2, 3, …, 9} and he will make
these selections with equal probabilities.
3. In throwing a pair of fair dice, let A1 be the event that
the first die turns up odd number, A2 be the event that Statement I: After n trials (n > 1), the probability
the second die turns up odd number and A3 be the that the product of these selected numbers is divisible
event that the sum of the faces turn up is odd. by 5 is
n n n
Statement I: The events A1, A2, A3 are independent. æ 8 ö æ 5ö æ 4 ö
1-ç ÷ -ç ÷ + ç ÷
è 9ø è 9ø è 9ø
Statement II: A1, A2, A3 are pairwise independent.
Statement II: For the product to be divisible by 10,
4. Let E1, E2, …, En be mutually exclusive and exhaus- the selections must contain at least one 5 and one
tive events of a random experiment with P(Ei) > 0 for even number.
i = 1, 2, 3, …, n. Let E be any event connected to the
experiment with 0 < P(E) < 1.

Integer Answer Type Questions


The answer to each of the questions in this section is a 3. Let A and B be two sets. A contains 4 elements while
non-negative integer. The appropriate bubbles below B contains 3. The probability of choosing a surjective
the respective question numbers have to be darkened. map from A to B is m/n (m/n is in lowest terms). Then
For example, as shown in the figure, if the correct answer the value of n – m is .
to the question number Y is 246, then the bubbles under (Hint: See Theorem 6.19, Vol. 1.)
Y labeled as 2, 4, 6 are to be darkened.
4. A number from the set {1, 2, 3, …, 15} is selected at
X Y Z W random. The probability that it is a proper divisor of 30
0 0 0 0 is a/b (in lowest terms). Then a + b is equal to .
1 1 1 1
2 2 2
5. Let m/n, in lowest terms, be the probability that a
randomly selected divisor of 1019 is an integer multiple
3 3 3 3
of 1010. Then m + n is .
4 4 4
5 5 5 5 6. A box contains 3 gold coins and 4 silver coins. Coins
6 6 6 are drawn one by one without replacement until all
the gold coins are drawn. If a/b (in lowest terms) is the
7 7 7 7
probability that the number of draws required is more
8 8 8 8 than 4, then b – a value is .
9 9 9 9
7. If the integers m and n are chosen from the set {1, 2,
1. Each of two bags contain both black and white balls. 3, …, 100}, then the probability that a number of the
The total number of balls in both bags is 25. One ball form 7m + 7n is divisible by 5 is 1/K
K where K is equal to
is taken from each bag. The probability of both balls .
are black is 27/50. Then the probability of both balls
are white is p/50 where p is . 8. The probability of a bomb hitting a bridge is 1/3 and
two direct hits are enough to destroy the bridge. The
2. 10 girls and 4 boys are to sit at a round table. If the least number of bombs required so that the prob-
arrangement is random, the probability of no two ability of the bridge getting destroyed is greater than
boys sit next to each other is a!/(b!c !) Then (b + c) - a 0.5 is .
is equal to .
Answers 567

ANSWERS
Single Correct Choice Type Questions
1. (C) 26. (A)
2. (D) 27. (C)
3. (B) 28. (B)
4. (A) 29. (D)
5. (B) 30. (D)
6. (C) 31. (D)
7. (B) 32. (A)
8. (A) 33. (C)
9. (B) 34. (A)
10. (A) 35. (C)
11. (B) 36. (D)
12. (A) 37. (A)
13. (C) 38. (C)
14. (A) 39. (D)
15. (D) 40. (C)
16. (B) 41. (D)
17. (B) 42. (A)
18. (A) 43. (C)
19. (A) 44. (D)
20. (D) 45. (B)
21. (B) 46. (A)
22. (C) 47. (A)
23. (A) 48. (D)
24. (B) 49. (D)
25. (C) 50. (A)

Multiple Correct Choice Type Questions


1. (A), (B), (C) 6. (A), (B), (C)
2. (A), (B), (C), (D) 7. (A), (D)
3. (A), (B), (C), (D) 8. (A), (B), (C)
4. (A), (C), (D) 9. (B), (C)
5. (A), (B), (C) 10. (A), (B), (C), (D)

Matrix-Match Type Questions


1. (A) ® (r); (B) ® (p); (C) ® (t); (D) ® (q) 4. (A) ® (s); (B) ® (q); (C) ® (p); (D) ® (r)
2. (A) ® (s); (B) ® (p); (C) ® (q); (D) ® (q) 5. (A) ® (p); (B) ® (q); (C) ® (r); (D) ® (s)
3. (A) ® (p), (s); (B) ® (p), (s); (C) ® (q), (s);
(D) ® (q), (s)

Comprehension Type Questions


1. (i) (D); (ii) (C); (iii) (A) 3. (i) (C); (ii) (A); (iii) (B)
2. (i) (B); (ii) (C); (iii) (D) 4. (i) (B); (ii) (A); (iii) (C)

Assertion–Reasoning Type Questions


1. (B) 4. (D)
2. (A) 5. (A)
3. (D)
568 Chapter 7 Probability

Integer Answer Type Questions


1. 2 5. 5
2. 9 6. 4
3. 5 7. 4
4. 3 8. 5
Inequalities
8
Contents
8.1 Introduction

Worked-Out Problems
W
Exercises

Euclid, also known as


Euclid of Alexandria, was a
Greek mathematician, often
Euclid referred to as the “Father of
Geometry.” He was active in
Alexandria during the reign
Y of Ptolemy I (323–283 BC).
His Elements is one of the
Im z most influential works in
the history of mathematics,
R(z1 + z2 ) serving as the main textbook
for teaching mathematics
Q(z2) | (especially geometry) from
+z
2

|z 1 the time of its publication


until the late 19th or early
|z |
2

20th century. The origin of


Inequalities

y2
inequalities stemmed from
P(z1) the famous theorem of
Euclid: The sum of two sides
| z 1| y1
of a triangle is greater than
X the third side.
O x2 x1 Re z
570 Chapter 8 Inequalities

8.1 | Introduction
The origin of inequalities stemmed from the famous theorem of EUCLID, namely, the sum of two sides of a triangle
is greater than the third side. Almost all geometric inequalities, in some way or other, are based on this result. In
this chapter, we deal with geometric inequalities as well as algebraic inequalities, especially more with Arithmetic–
Geometric Mean inequality. Let us begin with formal definitions of Arithmetic Mean (AM), Geometric Mean (GM)
and Harmonic Mean (HM) of real numbers.

DEFINITION 8.1 (i) Let a1, a2, …, an be n positive real numbers. Then the number
a1 + a2 +  + an
n
is called Arithmetic Mean or simply AM of a1, a2, …, an.
(ii) (a1 , a2 , a3 , …, an )1/ n is called Geometric Mean (GM).
n
(iii) is called Harmonic Mean (HM).
(1/ a1 ) + (1/ a2 ) +  + (1/ an )
Note that n ³ 2.

T H E O R E M 8.1 If a1, a2, …, an are n positive real numbers (n ³ 2), then


1
(a1 + a2 +  + an ) ³ (a1a2  an )1/ n
n
a1 = a2 =  = an.
PROOF We prove the theorem by the principle of mathematical induction. If a1, a2 are two positive real
numbers, then clearly
a1 + a2
³ a1a2
2

because ( a1 - a2 )2 ³ 0 and equality occurs if and only if a1 = a2. Assume the validity of the
theorem for (n - 1) positive real numbers and suppose a1, a2, ¼, an be n positive real numbers not
all equal.
Without loss of generality, we may suppose a1 is the greatest and an is the least. Let Gn be
(a1a2  an )1/ n so that using the assumption for the numbers a2 , a3 , …, an - 1 , a1an /Gn , we have
1 /( n - 1)
1 æ aa ö é aa ù
ç a2 + a3 +  + an - 1 + 1 n ÷ ³ ê(a2 a3  an - 1 ) 1 n ú
n -1è Gn ø ë Gn û

That is
a1an
a2 + a3 +  + an - 1 + ³ (n - 1)Gn
Gn

Therefore

æ aa ö
nGn £ ç a2 + a3 +  + an - 1 + 1 n ÷ + Gn
è Gn ø
a1an
= [nAn - (a1 + an )] + + Gn
Gn
[Gn2 - (a1 + an ) Gn + a1an ]
= nAn +
Gn
Worked-Out Problems 571

(Gn - a1 )(Gn - an )
= nAn +
Gn
< nAn (∵ a1 > Gn > an )

So
Gn < An

Hence, the theorem follows from induction. ■

C O R O L L A R Y 8.1 If a1, a2, …, an are positive and a1 + a2 +  + an = s (constant), then the product p = a1a2 … an is
greatest when
s
a1 = a2 =  = an =
n

C O R O L L A R Y 8.2 If a1, a2, …, an are positive such that their product a1a2 … an = p (constant), then their sum
a1 + a2 +  + an is least when

a1 = a2 =  = an = p1/ n

T H E O R E M 8.2 Let a1, a2, …, an and b1, b2, …, bn be two sets of real numbers. Then
(C A U C H Y ’ S
INEQUALITY) (a1b1 + a2 b2 +  + an bn )2 £ (a12 + a22 +  + an2 ) ´ (b12 + b22 +  + bn2 )

Equality occurs if and only if a1, a2, …, an are proportional to b1, b2, …, bn.
PROOF We proved this inequality when n = 3 in Chapter 6 (see Corollary 6.5). Let
n n n
A = å ai2 , B = å ai bi and C = å bi2
i =1 i =1 i =1

For any real x, we have


n
0 £ å (ai + xbi )2 = A + 2 Bx + Cx2
i =1

This implies

4 B2 - 4 AC £ 0
Þ B2 - AC £ 0
Þ B2 £ AC
and equality holds if and only if ai + xbi = 0 for i = 1, 2, …, n. ■

WORKED-OUT PROBLEMS
Caution: When AM–GM inequality is to be applied, be sure that the numbers are positive.
1. If a and b are positive real numbers such that a + b = 1, Proof: By Theorem 8.1
then prove that
a+b 1
2 2 ³ ab Þ ab £
æ 1ö æ 1ö 25 2 4
çè a + ÷ø + çè b + ÷ø ³
a b 2
572 Chapter 8 Inequalities

Now C A-B C 1
2 sin cos - 2 sin2 =
2 2 2 2 2 2
æ 1ö æ 1ö æ 1 1ö
çè a + ÷ø + çè b + ÷ø = (a + b ) + 4 + çè 2 + 2 ÷ø
2 2

a b a b A-B C 1
cos = sin +
2 2 4 sin C
æ 1 1ö
= (a + b)2 - 2ab + 4 + ç 2 + 2 ÷ 2
èa b ø
2
æ C 1 ö
2 = ç sin -
³ 1 - 2ab + 4 + ÷ +1
ab è 2 2 sin(C / 2) ø

æ 1ö ³1
³ 1 - 2 ç ÷ + 4 + 2(4) [By Eq. (8..1)]
è 4ø
Hence we get
1
= 12 + A-B
2 1 £ cos £1
2
Equality holds if
which implies that
1
a=b= A-B
2 cos =1
2
2. If x1, x2, …, xn are positive real numbers, then show so that A = B. Similarly B = C. Therefore DABC
C is
that equilateral.
æ 1 1 1ö
( x1 + x2 +  + xn ) ç + +  + ÷ ³ n2 4. Assuming that ex ³ 1 + x for all real x (this result can
è x1 x2 xn ø be proved using differential calculus), prove that AM
of positive real numbers is greater than or equal to
Equality occurs if and only if x1 = x2 =  = xn.
their GM.
Proof: By AM-GM inequality we have
Proof: In ex ³ 1 + x replace x on both sides with x - 1.
x1 + x2 +  + xn ³ n( x1 x2 … xn )1/ n (8.2) We get

and ex - 1 ³ x (8.4)
1/ n Let x1, x2, …, xn be n positive real numbers. Now, replace
1 1 1 æ 1 1 1ö x with x1 / x , x2 / x , …, xn / x in Eq. (8.4) where
+ ++ ³ nç ×  ÷ (8.3)
x1 x2 xn è x1 x2 xn ø
x1 + x2 +  + xn
x=
Multiplying Eqs. (8.2) and (8.3), we have n
æ 1 1 1ö Now multiply all of these so that
( x1 + x2 +  + xn ) ç + +  + ÷ ³ n2
è x1 x2 xn ø x1 x2 x
e( x1 / x ) - 1 × e( x2 / x ) - 1  e( xn / x ) - 1 ³ ×  n
x x x
3. In DABC, if
Therefore
3
cos A + cos B + cos C =
2 æ x + x2 +  + xn ö (x x  x )
exp ç 1 - n÷ ³ 1 2 n n
è x ø (x )
then prove that the triangle is equilateral.
æ nx ö x x x
Proof: We have exp ç - n÷ ³ 1 2 n n
è x ø (x)
3
cos A + cos B + cos C = x1 x2  xn
2 1 = e0 ³
( x )n
Therefore
x ³ ( x1 x2  xn )1/ n
A+ B A-B C 3
2 cos cos + 1 - 2 sin2 =
2 2 2 2 5. In DABC, if tan A tan B + tan B tan C + tan C tan A
= 9, then prove that DABC
C is equilateral.
Worked-Out Problems 573

Proof: First we show that the angles A, B, C are acute. which implies that tan A = tan B = tan C. Hence the
Suppose one of the angles say A is obtuse so that tan A is triangle is equilateral.
negative. Therefore B and C are acute, and B + C is also
acute. Also 6. In DABC, if

tan( B + C ) = - tan A > 0 a cos A + b cos B + c cos C a + b + c


=
a sin B + b sin C + c sin A 9R
Therefore
where R is the circumradius of DABC, prove that the
tan B + tan C
= - tan A > 0 triangle is equilateral.
1 - tan B tan C
Proof: In Chapter 4, we proved that
and tan B tan C ¹ 1
a b c
so that tan B tan C < 1. Now = = = 2R
sin A sin B sin C
9 = å tan A tan B Therefore a = 2R sin A, b = 2R sin B and c = 2R sin C.
= tan A(tan B + tan C ) + tan B tan C According to the hypothesis

which implies that sin 2 A + sin 2 B + sin 2C 2


= (sin A + sin B + sin C )
2å (sin A sin B) 9
0 > tan A(tan B + tan C )
= 9 - tan B tan C
4 sin A sin B sin C 2
2å (sin A sin B) 9
= (å sin A)
>0
which is a contradiction. Therefore A cannot be obtuse
9 sin A sin B sin C = (sin A + sin B + sin C ) (å sin A sin B)
and hence A, B, C are acute. This means tan A, tan B and 9 = (sin A + sin B + sin C )
tan C are positive. Again
æ 1 1 1 ö
9 = tan A tan B + tan B tan C + tan C tan A çè + + ÷
sin A sin B sin C ø
tan A tan B tan C tan A tan B tan C ³9 (by Problem 2)
= +
tan C tan A
Equality implies that sin A = sin B = sin C. Therefore
tan C tan B tan A DABCC is equilateral.
+
tan B
tan A + tan B + tan C tan A + tan B + tan C 7. Let a1, a2, …, an; b1, b2, …, bn be two sets of positive
= + real numbers such that
tan C tan A
tan A + tan B + tan C a1 + a2 +  + an = b1 + b2 +  + bn
+
tan B
Then prove that
because n
æ ai2 ö 1
tan A + tan B + tan C = tan A tan B tan C å ç ÷ ³ 2 (a1 + a2 +  + an )
i = 1 è ai + bi ø

Therefore n
æ bi2 ö 1
tan A tan B tan B tan C tan A tan C
and å çè a + b ÷ø ³ 2 (b + b 1 2 +  + bn )
6= + + + + + i =1 i i
tan C tan C tan A tan A tan B tan B
Proof: We have
1/ 6
é tan A tan B tan B tan C tan A tan C ù
³ 6ê × × × × × a2 + ai bi - ai bi
ë tan C tan C tan A tan A tan B tan B úû
ai2 ab
= i = ai - i i (8.5)
ai + bi ai + bi ai + bi
(Theorem 8.1)
=6 Again,

Because equality is holding, æ 1 1ö


(ai + bi ) ç + ÷ ³ 22 (Problem 2)
è ai bi ø
tan A tan B tan C
= =
tan B tan C tan A
574 Chapter 8 Inequalities

which implies that 9. If x, y, z are positive real numbers and x + y + z = 3,


then prove that
ai + bi 1
³
4 (1/ ai ) + (1/ bi ) x+ y + z ³ xy + yz + zx
(8.6)
ai bi Proof: We have
=
ai + bi
2 x - x( y + z) = 2 x - x(3 - x) (8.8)
From Eq. (8.5)
Now
ai2 ab
= ai - i i [2 x - x(3 - x)][2 x + x(3 - x)] = 4 x - x2 (3 - x)2
ai + bi ai + bi
= x[4 - x(9 - 6 x + x2 )]
a + bi
³ a1 - i [from Eq. (8.6)] = x(- x3 + 6 x2 - 9 x + 4)
4
Therefore = x( x - 1)(- x2 + 5 x - 4)
= ( x - 1)( x - 1)2 (4 - x)
ai2 3a - bi
³ i ³ 0 (∵ 0 < x < 3)
ai + bi 4

So
2 x - x( y + z) ³ 0
n
ai2 1
å a
i =1 i + b
³ [3(a1 + a2 +  + an ) - (b1 + b2 +  + bn )
4 Similarly
i

1 2 y - y(z + x) ³ 0
= [3(a1 + a2 +  + an ) - (a1 + a2 +  + an )]
4
and 2 z - z( x + y) ³ 0
1
= (a1 + a2 +  + an )
2 Therefore

8. If x1, x2, x3, …, xn are positive real numbers such that


2( x + y + z ) - 2( xy + yz + zx) ³ 0
x1 + x2 +  + xn = s (constant), then prove that x+ y + z ³ xy + yz + zx
s1 s2 s
(1 + x1 )(1 + x2 )  (1 + xn ) £ 1 + + ++ n
1 2 n 10. DABC, if

Proof: Note that (tan A + 2 tan B + 3 tan C )2


³ 14(tan2 A + tan2 B + tan2 C )
n(n - 1)(n - 2)  (n - r + 1)
n
Cr =
r then show that B + C is 3p / 4.

so that Proof: In Theorem 8.2 (Cauchy’s Inequality), take a1 = 1,


n a2 = 2, a3 = 3 and b1 = tanA, b2 = tanB, b3 = tanC. Then
Cr 1
£ (8.7)
nr r (tan A + 2 tan B + 3 tan C )2 £ (12 + 22 + 32 )
Now by Theorem 8.1, we have (tan2 A + tan2 B + tan2 C )

(1 + x1 )(1 + x2 )  (1 + xn ) (tan A + 2 tan B + 3 tan C )2 £ 14(tan2 A + tan2 B + tan2 C )


n £ (tan A + 2 tan B + 3 tan C )2
é (1 + x1 ) + (1 + x2 ) +  + (1 + xn ) ù
£ê ú (by hyypothesis)
ë n û
æ sö
n Therefore
= ç1 + ÷
è nø (tan A + 2 tan B + 3 tan C )2 = 14(tan2 A + tan2 B + tan2 C )
2 n
æ sö æ sö æ sö But it is known that equality occurs if and only if a1 : a2 :
= 1 + n C1 ç ÷ + n C2 ç ÷ +  + n Cn ç ÷
è nø è 2ø è nø a3 = b1 : b2 : b3. So
s s2 sn tan A tan B tan C
£1+ + ++ [By Eq. (8.7)] = =
1 2 n 1 2 3
Worked-Out Problems 575

Hence (ii) We have


p 3p æa b cö
A= and B+ C = (ad1 + bd2 + cd3 ) ç + + ÷ = (a2 + b2 + c2 )
4 4 è d1 d2 d3 ø

11. In Figure 8.1 P is a point inside DABC. Let d1, d2, d3 æd d ö


+ å ab ç 1 + 2 ÷
be the distances of P from the sides BC, CA and AB, è d2 d1 ø
respectively. R and r are, respectively, the circumra-
dius and inradius of DABC. Prove that and
(i) The greatest value of ( d1 + d2 + d3 ) 2
is
æa b cö
a2 + b2 + c2 (ad1 + bd2 + cd3 ) ´ ç + + ÷
. è d1 d2 d3 ø
2R
a b c a+b+c æ d d ö
(ii) The least value of + + is . ³ (a2 + b2 + c2 ) + å 2ab ç∵ 1 + 2 ³ 2÷
d1 d2 d3 r è d2 d1 ø
a b c = (a + b + c)2
(iii) + + is least when P is the incentre of
d1 d2 d3
DABC. Therefore

Proof: Let a1 = ad1 , a2 = bd2 and a3 = cd3 and b1 = a b c (a + b + c)2


+ + ³
1/ a , b2 = 1/ b , b3 = 1/ c. d1 d2 d3 2D
2s a+b+c
= (a + b + c ) =
A 2D r
(iii) is obvious.
Note: The above problem may be asked under
comprehension.
d3 d2

P 12. If x, y, z are positive real numbers and x + y + z = 1,


d1 then prove that the minimum value of
C
B
4 9 16
+ +
FIGURE 8.1 Worked-out problem 11. x y z

(i) Using Cauchy inequality (Theorem 8.2) we have is 81.


Proof: Since x + y + z = 1 we have
æ 1 1 1ö
( d1 + d2 + d3 )2 £ (ad1 + bd2 + cd3 ) ç + + ÷ (8.9)
è a b cø 4 9 16 æ 4 9 16 ö
+ + = ( x + y + z) ç + + ÷
But x y z èx y zø

ad1 + bd2 + cd3 = 2 DPBC + 2 DPCA + 2 DPAB = 2D. æ 4z 16 x ö æ 9z 16 y ö æ 4 y 9 x ö


= (4 + 9 + 16) + ç + ÷+ + + +
è x z ø çè y z ÷ø çè x y ÷ø
From Eq. (8.9),
³ 29 + 2 64 + 2 144 + 2 36 = 81
æ 1 1 1ö
( d1 + d2 + d3 ) £ 2 D ç + + ÷
2
Equality holds if and only if
è a b cø
2 3 4
2D x= ,y= ,z=
= (bc + ca + ab) 9 9 9
abc
2D 2 a2 + b2 + c2 13. a, b, c are positive real numbers such that
£ (a + b2 + c2 ) =
abc 2R a + b + c = 1, then prove that

Therefore the least value of ( d1 + d2 + d3 )2 is a2 + b2 + c2 ³ 4(ab + bc + ca) - 1

a2 + b2 + c2 where the equality holds if and only if a = b = c = 1/3.


2R
576 Chapter 8 Inequalities

Proof: We have then prove that

a2 + b2 + c2 = (a + b + c)2 - 2å ab x+ y+z³ x-1+ y-1+ z-1


= 1 - 2å ab ³ 4(ab + bc + ca) - 1
(Maths Olympiads IRAN, 1998–99)
Û 2 ³ 6å ab Proof: In Cauchy’s inequality (Theorem 8.2) for n = 3,
take
Û 1 ³ 3å ab
a1 = x , a2 = y , a3 = z
Û (a + b + c)2 ³ 3å ab
x-1 y-1 z-1
Û a2 + b2 + c2 - ab - bc - ca ³ 0 b1 = , b2 = , b3 =
x y z
1
Û é(a - b)2 + (b - c)2 + (c - a)2 ùû ³ 0 Therefore

(a1b1 + a2 b2 + a3 b3 )2 £ (a12 + a22 + a32 )(b12 + b22 + b32 )
which is true and equality occurs if and only if a = b =
c = 1/3.

14. If a, b, c are real numbers such that 0 < a, b, c < 1 ( x-1+ y - 1 + z - 1)2 £ ( x + y + z)
and a + b + c = 2, then prove that
æ x - 1 y - 1 z - 1ö
abc çè x + y + z ÷ø (8.10)
³8
(1 - a)(1 - b)(1 - c)
But
Equality occurs when a = b = c = 2 / 3.
x-1 y-1 z-1 æ 1 1 1ö
Proof: We have + + =3-ç + + ÷
x y z è x y zø
(a + b - c ) + (a - b + c )
a= =3-2=1
2
So by Eq. (8.10) we have
³ (a + b - c)(a - b + c)
x-1+ y-1+ z-1£ x+ y+z
(b + c - a) + (b - c + a)
b=
2
16. If x, y, z are positive real numbers such that x3 y2
³ (b + c - a)(b - c + a) z4 = 7, then show that

(c + a - b) + (c - a + b) æ 525 ö
1/ 9
c= 2 x + 5 y + 3z ³ 9 ç 7 ÷
2 è 2 ø
³ (c + a - b)(c - a + b) Proof: Using AM ³ GM for the nine numbers

Therefore 2 x 2 x 2 x 5 y 5 y 3z 3z 3z 3z
, , , , , , , ,
3 3 3 2 2 4 4 4 4
abc ³ (b + c - a)(c + a - b)(a + b - c)
= (2 - 2a)(2 - 2b)(2 - 2c)
1/ 9
2 x + 5 y + 3z æ 2 x 2 x 2 x 5 y 5 y 3z 3z 3z 3z ö
So ³ç × × × × × × × × ÷
9 è 3 3 3 2 2 4 4 4 4ø
abc
³8 Therefore
(1 - a)(1 - b)(1 - c)
1/ 9
éæ 2 ö 3 æ 5 ö 2 æ 3 ö 4 ù
15. If each of x, y, z is greater than unity and 2 x + 5 y + 3z ³ 9 êç ÷ ç ÷ ç ÷ x3 y2 z4 ú
êëè 3 ø è 2 ø è 4 ø úû
1 1 1
+ + =2 éæ 2 ö 3 æ 5 ö 2 æ 3 ö 4 ù
1/ 9 1/ 9
x y z æ 525 ö
= 9 êç ÷ ç ÷ ç ÷ (7)ú = 9ç 7 ÷
êëè 3 ø è 2 ø è 4 ø úû è 2 ø
Worked-Out Problems 577

17. If a, b, c are positive real numbers such that abc = 1, 19. If a, b, c are positive real numbers, then prove that
then prove that
b2 + c2 c2 + a2 a2 + b2
ab bc ca a+b+c£ + + < 2(a + b + c)
+ + £1 b+c c+a a+b
a5 + b5 + ab b5 + c5 + bc c5 + a5 + ca
where the preceding equality holds if a = b = c.
Equality holds when a = b = c = 1.
Proof: We have
Proof: We have
b2 + c2 ³ 2bc
a5 + b5 = (a + b)[a4 - a3 b + a2 b2 - ab3 + b4 ]
Þ 2(b2 + c2 ) ³ 2bc + b2 + c2 = (b + c)2
= (a + b)[a3 (a - b) - b3 (a - b) + a2 b2 ]
b2 + c2 b + c
= (a + b)[(a3 - b3 )(a - b) + a2 b2 ] Þ ³
b+c 2
= (a + b)[(a - b)2 (a2 + ab + b2 ) + a2 b2 ]
Therefore
³ (a + b)a2 b2
æ b2 + c2 ö b + c c + a a + b
Therefore
å çè b + c ÷ø ³ 2 + 2 + 2
a5 + b5 + ab ³ (a + b)a2 b2 + ab
=a+b+c
1 1
£ So
a5 + b5 + ab (a + b)a2 b2 + ab
ab ab b2 + c2
£ a+b+c£å
a + b + ab (aa + b)a2 b2 + ab
5 5
b+c
1
= Also
ab(a + b) + 1
abc b2 + c2 < (b + c)2
= (∵ abc = 1)
ab(a + b) + abc b2 + c2
Þ <b+c
c b+c
=
a+b+c Therefore
So
b2 + c2
ab c å b + c < (b + c) + (c + a) + (a + b)
£
a5 + b5 + ab a + b + c = 2(a + b + c)
Similarly
20. If a, b, c are positive real numbers such a + b + c = 1,
bc a
£ then prove that
b + c + bc a + b + c
5 5

(1 + a)(1 + b)(1 + c) ³ 8(1 - a)(1 - b)(1 - c)


ca b
£
c5 + a5 + ca a + b + c and equality occurs if a = b = c = 1/ 3.
Hence Proof: We have
ab bc ca a+b+c 1 + a = 1 + (1 - b - c)
+ 5 + 5 £ =1
a + b + ab b + c + bc c + a + ca a + b + c
55 5 5
= (1 - b) + (1 - c)

18. If 0 < a < 1 and 0 < b < 1, then prove that ³ 2 (1 - b)(1 - c)
ab + (1 - a)(1 - b) £ 1.
Similarly
Proof: Put a = sin2q and b = sin2f. Then
1 + b ³ 2 (1 - c)(1 - a)
ab + (1 - a)(1 - b) = sin q sin f + cos q cos f
1 + c ³ 2 (1 - a)(1 - b)
= cos(q - f )
£1 Therefore
Note that equality holds when a = b. (1 + a)(1 + b)(1 + c) ³ 8(1 - a)(1 - b)(1 - c)
578 Chapter 8 Inequalities

21. Prove that Proof: We have


1 1 3 5 2n - 1 1 (a + b)(ax2 + by2 ) = a2 x2 + b2 y2 + ab( x2 + y2 )
< × ×  <
2 n+1 2 4 6 2n 2n + 1 = (a2 x2 + b2 y2 ) + ab
Proof: Let ³ 2 (a2 x2 + b2 y2 )ab
1 3 5 2n - 1
an = × × 
2 4 6 2n
ax2 + by2 2 ab
Therefore ³
a x +b y
2 2 2 2 a+b
2 4 6 2n æ 1 2 3 4 ö
an < × ×  ç∵ < , < …÷
3 5 7 2n + 1 è 2 3 4 5 ø 23. If a1, a2, a3; b1, b2, b3 are positive real numbers, then
prove that
So
æa a a ö
æ 1 3 5 2 n - 1ö æ 2 4 6 2n ö (a1b1 + a2 b2 + a3 b3 ) ç 1 + 2 + 3 ÷ ³ (a1 + a2 + a3 )2
an2 < ç × ×  ÷ × ×  è b1 b2 b3 ø
è2 4 6 2 n ø çè 3 5 7 2 n + 1÷ø
and equality holds if and only if b1 = b2 = b3.
1
= Proof: We have
2n + 1
which gives æa a a ö
(a1b1 + a2 b2 + a3 b3 ) ç 1 + 2 + 3 ÷
1 è b1 b2 b3 ø
an <
2n + 1 æaba a ba ö æa a b aab ö
= a12 + a22 + a32 + ç 1 1 2 + 2 2 1 ÷ + ç 2 3 2 + 2 3 3 ÷
è b2 b1 ø è b3 b2 ø
Again,
æa ab a ab ö
3 5 7 æ 2 n + 1ö + ç 3 1 1 + 3 1 3÷
(2 n + 1)an = × × ç ÷ è b3 b1 ø
2 4 6 è 2n ø
³ a12 + a22 + a32 + 2a1a2 + 2a2 a3 + 2a3 a1 (∵ AM ³ GM)
4 6 8 æ 2n + 2 ö
> × × ç
3 5 7 è 2 n + 1 ÷ø and equality occurs if and only if b1 = b2 = b3.

So 24. If a, b, c are positive integers such that a + b + c = n,


then prove that
2n + 2
(2 n + 1) a >
2 2
n =n+1 (aa bb cc )1/ n + (ab bc ca )1/ n + (ac ba cb )1/ n £ n
2
n+1 1 Equality holds when a = b = c and n is a multiple of 3.
Þ an2 > >
(2 n + 1)2 4(n + 1) Proof:
Step 1: Consider a number of a’s, b number of b’s and
1 c number of c’s and use AM ³ GM. Then
Þ an >
2 n+1
a2 + b2 + c2
³ (aa bb cc )1/( a + b + c )
a+b+c
22. If a and b are two given positive real numbers and
x, y are variable real numbers satisfying the relation a2 + b2 + c2
³ (aa bb cc )1/ n
x2 + y2 = 1, then show that the minimum value of n
ax2 + by2 Step 2: Consider b number of a’s, c number of b’s and
a x +b y
2 2 2 2 a number of c’s and use AM ³ GM. Then
ab + bc + ca
is 2 ab /(a + b). ³ (ab bc ca )1/ n
n
Worked-Out Problems 579

Step 3: Consider c number of a’s, a number of b’s and Þ 4n + 1 < 2n + 1 + 2 n(n + 1)


b number of c’s and use AM ³ GM. Then
ac + ab + bc = ( n + n + 1)2
³ (ac ba cb )1/ n
n Therefore
Adding all the results in steps 1 to 3, we have
4n + 1 < n + n + 1 (8.12)
1 2
(a + b2 + c2 + 2ab + 2bc + 2ca) Again
n
³ (aa bb cc )1/ n + (ab bc ca )1/ n + (ac ba cb )1/ n 2 n(n + 1) < 2 n + 1
that is
Þ n + n + 1 + 2 n(n + 1) < 4 n + 2
(a + b + c)2
³ (aa bb cc )1/ n + (ab bc ca )1/ n + (ac ba cb )1/ n Þ ( n + n + 1)2 < 4 n + 2
n
a + b + c = n. Þ n + n + 1 < 4n + 2 (8.13)

25. If a, b, c are positive real numbers, then prove that From Eqs. (8.12) and (8.13) we have

[(1 + a)(1 + b)(1 + c)]7 > 77 a4 b4 c4 4n + 1 < n + n + 1 < 4n + 2

(IIT-JEE, 2004)
27. If a1, a2, …, an are positive real numbers whose
Proof: We have product is a fixed number c, then the minimum value
(1 + a)(1 + b)(1 + c) = 1 + a + b + c + ab + bc + ca + abc of a1 + a2 +  + an-1 + 2an is
(A) n(2c)1/ n (B) (n + 1)c1/ n
This implies (C) (2 n)c1/ n (D) (n + 1)(2c)1/ n
(1 + a)(1 + b)(1 + c) > a + b + c + ab + bc + ca + abc (IIT-JEE, 2002)
(8.11) Solution: Using AM ³ GM for the n numbers a1, a2, …,
Using AM ³ GM, we have an-1, 2an, we have

1 a1 + a2 +  + an-1 + 2an
(a + b + c + ab + bc + ca + abc) ³ (a1a2 a3 … an-1 (2an ))1/ n
7 n
³ [(abc)(ab × bc × ca)abc]1/ 7 = (2c)1/ n
= (a4 b4 c4 )1/ 7 Therefore

a1 + a2 +  + an - 1 + 2an ³ n(2c)1/ n
(1 + a)(1 + b)(1 + c) > 7(a4 b4 c4 )1/ 7
Answer: (A)
So
28. If a, b, c and d are positive real numbers such that
[(1 + a)(1 + b)(1 + c)]7 > 77 a4 b4 c4 a + b + c + d = 2, then M = (a + b) (c + d) satisfies the
relation
26. For any positive integer n, prove that (A) 0 < M £ 1 (B) 1 < M £ 2
4n + 1 < n + n + 1 < 4n + 2 (C) 2 < M £ 3 (D) 3 £ M £ 4

(IIT-JEE, 2000) (IIT-JEE, 2000)

Proof: We have Solution: Using AM ³ GM for the two numbers (a + b)


and (c + d) we have
n < n(n + 1)
(a + b) + (c + d)
³ [(a + b)(c + d)]1/ 2
Þ 2 n < 2 n(n + 1) 2
580 Chapter 8 Inequalities

Therefore æ a ö æ b ö æ c ö 9
2 çè b + c + 1÷ø + çè c + a + 1÷ø + çè a + b + 1÷ø ³ 2
³ M 1/ 2
2 a b c 9 3
+ + ³ -3=
and M cannot be zero because a, b, c, d, e are posi- b+c c+a a+b 2 2
tive. Hence 0 < M £ 1 and equality occurs if and only if
a + b = c + d = 1. 31. a, b, c are positive real numbers, then prove that
Answer: (A) 2 2 2 9
+ + ³
b+c c+a a+b a+b+c
29. The product of n positive numbers is unity, then
their sum is (Ireland Olympiads, 1998–99)
(A) a positive integer Proof: As in Problem 30 we have
(B) divisible by n
1 2 æ 1 1 1 ö
(C) equal to n + (a + b + c ) ç + + ³1
n 9 è b + c c + a a + b ÷ø
(D) never less than n
Therefore
(IIT-JEE, 1991)

Solution: Let a1, a2, …, an be positive such that the 2 2 2 9


+ + ³
product a1a2a3 … an = 1. Using AM ³ GM we have b+c c+a a+b a+b+c
a1 + a2 +  + an
³ (a1a2 … an )1/ n = 1 32. If a, b, c are the sides of a triangle, then show that
n
a2 + b2 + c2
This implies 1£ <2
ab + bc + ca
a1 + a2 +  + an ³ n
and left side equality holds when the triangle is
Answer: (D) equilateral.
Proof: We have
30. If a, b, c are positive real numbers, then prove that
a2 + b2 + c2 - ab - bc - ca
a b c 3
+ + ³ 1
b+c c+a a+b 2 = [(a - b)2 + (b - c)2 + (c - a)2 ] ³ 0
2
and equality occurs when a = b = c.
Therefore
Proof: We have
a2 + b2 + c2
(b + c) + (c + a) + (a + b) ³1
³ [(b + c)(c + a)(a + b)]1/ 3 ab + bc + ca
3
Also, since a, b, c are the sides of a triangle, we have
Therefore
|b - c| < a, |c - a| < b and |a - b| < c. Therefore
2
(a + b + c) ³ [(b + c)(c + a)(a + b)]1/ 3 (8.14) (a - b)2 + (b - c)2 + (c - a)2 < a2 + b2 + c2
3
2(a2 + b2 + c2 ) - 2(ab + bc + ca) < a2 + b2 + c2
Again
a2 + b2 + c2 < 2(ab + bc + ca)
1 1 1
+ + 1/ 3
b+c c+a a+b æ 1 1 1 ö a2 + b2 + c2
³ç × × (8.15) <2
3 è b + c c + a a + b ÷ø ab + bc + ca

Multiplying Eqs. (8.14) and (8.15), we have 33. If a, b, c are sides of a triangle ABC, then prove that

æ 1 1 1 ö 9 (i) (a + b + c)3 ³ 27(b + c - a)(c + a - b)(a + b - c)


(a + b + c ) ç + + ³
è b + c c + a a + b ÷ø 2 (ii) abc ³ (b + c - a)(c + a - b)(a + b - c)
A B C 1
(iii) sin sin sin £
2 2 2 8
Worked-Out Problems 581

In all the three relations equality occurs when DABC


C is (See Theorem 4.4 of Chapter 4.) Therefore
equilateral.
A B C ( s - a)( s - b)( s - c)
Proof: (i) In any triangle, since sum of any two sides is sin sin sin =
2 2 2 abc
greater than the third side, we have
(2 s - 2a)(2 s - 2b)(2 s - 2c)
b+c-a>0 =
8abc
c+a-b>0
(b + c - a)(c + a - b)(a + b - c)
a+b-c>0 =
8abc
Put
1
£ [from (ii)]
x=b+c-a 8
y=c+a-b
34. In DABC, prove that
and z=a+b-c
1
so that x, y and z are positive. Also x + y + z = a + b + c. Area D £ (a + b + c)abc
4
Now AM ³ GM implies
x+ y+z 3 (IIT-JEE, 2001)
³ xyz
3 Proof: In part (ii) of Problem 33, we proved that
Therefore (b + c - a)(c + a - b)(a + b - c) £ abc
( x + y + z) ³ 27 xyz
3
Therefore
that is
( s - a)( s - b)( s - c) £ abc
(a + b + c) ³ 27(b + c - a)(c + a - b)(a + b - c)
3

abc
( s - a)( s - b)( s - c) £
(ii) Again 8
y+z æ a + b + c ö æ abc ö
³ yz s( s - a)( s - b)( s - c) £ ç ÷ø çè ÷
2 è 2 8 ø
z+ x
³ zx 1
2 D2 £ (a + b + c)abc
16
x+y
³ xy
2 1
D£ (a + b + c)abc
4
This implies

æ y + zö æ z + x ö æ x + y ö 35. In DABC, prove that


çè ÷ç ÷ç ÷ ³ xyz
2 øè 2 øè 2 ø
3
1 < cos A + cos B + cos C £
that is 2
abc ³ (b + c - a)(c + a - b)(a + b - c) Proof: We have

(iii) We have A B C
cos A + cos B + cos C = 1 + 4 sin sin sin > 1
2 2 2
A ( s - b)( s - c)
sin =
2 bc Again

B ( s - c)( s - a) cos A + cos B + cos C


sin =
2 ca
A æ B + Cö æ B - Cö
= 1 - 2 sin2 + 2 cos ç cos ç
C ( s - a)( s - b) 2 è 2 ÷ø è 2 ÷ø
sin =
2 ab
582 Chapter 8 Inequalities

A A æ B - Cö We know that a = 2R sin A, b = 2R sin B, c = 2R sin C and


= 1 - 2 sin2 + 2 sin cos ç
2 2 è 2 ÷ø a, b, c are positive. Therefore

(a + b + c)2 £ 3(a2 + b2 + c2 )
A A æ æ B - Cö ö
£ 1 - 2 sin2 + 2 sin çè∵ cos çè 2 ÷ø £ 1÷ø
2 2 4 R2 (sin A + sin B + sin C )2 £ 12 R2 (sin2 A + sin2 B + sin2 C )

æ A 1ö 1 3
2
(sin A + sin B + sin C )2
= 1 - 2 ç sin - ÷ + £
è 2 2ø 2 2 æ 1 - cos 2 A 1 - cos 2 A 1 - cos 2C ö
£ 3ç + + ÷ø
Note that è 2 2 2

3 3
cos A + cos B + cos C = = [3 - (cos 2 A + cos 2 B + cos 2C )]
2 2

if and only if DABC


C is equilateral. 3
= [3 - (- 1 - 4 cos A cos B cos C )]
2
36. In DABC, prove that 3
= (4 + 4 cos A cos B cos C )
1 2
cos A cos B cos C £
8 = 6(1 + cos A cos B cos C )
Equality occurs if and only if it is an equilateral æ 1ö
triangles. £ 6ç1 + ÷ (By Problem 36)
è 8ø
Proof: If DABC C is right-angled or obtuse, the inequality
is clear. Suppose DABC 27
C is acute angled. By AM–GM =
inequality, 4

cos A + cos B + cos C Therefore


³ (cos A cos B cos C )1/ 3
3
3 3
sin A + sin B + sin C £
Therefore 2
3
æ cos A + cos B + cos C ö 38. In DABC, prove that
cos A cos B cos C £ ç ÷ø
è 3
9æ A B Cö
3 cosec A + cosec B + cosec C ³ çè sec sec sec ÷ø
æ 3/ 2 ö 1 4 2 2 2
£ç =
è 3 ÷ø 8
Proof: We know from Problem 2 that
37. In DABC, prove that æ 1 1 1 ö
(sin A + sin B + sin C ) ç + + ³ 32
è sin A sin B sin C ÷ø
3 3
sin A + sin B + sin C £
2 Therefore
Equality holds if and only if the triangle is equilateral.
æ A B Cö
Proof: First, we prove that çè 4 cos cos cos ÷ø (cosec A + cosec B + cosec C ) ³ 9
2 2 2

( x + y + z)2 £ 3( x2 + y2 + z2 ) 9æ A B Cö
cosec A + cosec B + cosec C ³ çè sec sec sec ÷ø
4 2 2 2
where x, y, z are positive real numbers. Consider

( x + y + z)2 = x2 + y2 + z2 + 2 xy + 2 yz + 2zx 39. In DABC, prove that

£ ( x2 + y2 + z2 ) + ( x2 + y2 ) + ( y2 + z2 ) cos A cos B cos C £


1
[cos2 ( A - B) + cos2 ( B - C )
24
+ (z2 + x2 )
+ cos2 (C - A)]
= 3( x + y + z )
2 2 2
Worked-Out Problems 583

Proof: If x and y are any two real numbers, then we Consider


have
b+c>a
( x + y)2 - 4 xy = ( x - y)2 ³ 0
Þ 2(b + c) > a + b + c
Þ ( x + y)2 ³ 4 xy
a+b+c
Þb+c>
Now take x = cos A and y = 2cos B cos C. Then 2

(cos A + 2 cos B cos C )2 ³ 8 cos A cos B cos C 1 2


Þ <
b+c a+b+c
[- cos( B + C ) + 2 cos B cos C ]2 ³ 8 cos A cos B cos C
a 2a
(cos B cos C + sin B sin C )2 ³ 8 cos A cos B cos C Þ <
b+c a+b+c
cos2 ( B - C ) ³ 8 cos A cos B cos C Therefore
Similarly, a b c 2a + 2b + 2c
+ + < =2
cos2 (C - A) ³ 8 cos A cos B cos C b+c c+a a+b a+b+c

cos2 ( A - B) ³ 8 cos A cos B cos C 42. In DABC, prove that

Therefore 3(bc + ca + ab) £ (a + b + c)2 < 4(bc + ca + ab)


cos2 ( B - C ) + cos2 (C - A) + cos2 ( A - B) Proof: We have
³ 24 cos A cos B cos C a2 + b2 ³ 2ab
and hence the result.
b2 + c2 ³ 2bc
40. In DABC, prove that and c2 + a2 ³ 2ca
a2 (b + c - a) + b2 (c + a - b) + c2 (a + b - c) £ 3abc Now
Proof: We have 2(a2 + b2 + c2 ) ³ 2(bc + ca + ab)
a2 (b + c - a) + b2 (c + a - b) + c2 (a + b - c) Therefore
= a(b + c - a ) + b(c + a - b ) + c(a + b - c )
2 2 2 2 2 2 2 2 2
a2 + b2 + c2 ³ ab + bc + ca
= 2abc cos A + 2abc cos B + 2abc cos C Adding 2(ab + bc + ca) to both sides we have
= 2abc(cos A + cos B + cos C )
(a + b + c)2 ³ 3(ab + bc + ca)
æ 3ö
£ 2abc ç ÷ Also
è 2ø

The last step has been derived using Problem 35. | a - b| < c
|b - c| < a
41. In DABC, prove that
|c - a| < b
3 a b c
£ + + <2
2 b+c c+a a+b This implies

Proof: (a - b)2 + (b - c)2 + (c - a)2 < a2 + b2 + c2


a b c 3 So
+ + ³
b+c c+a a+b 2
a2 + b2 + c2 < 2ab + 2bc + 2ca
for any positive real numbers a, b and c. Now we prove that
Adding to both sides 2ab + 2bc + 2ca, we have
a b c
+ + <2 (a + b + c)2 < 4(ab + bc + ca)
b+c c+a a+b
584 Chapter 8 Inequalities

43. In DABC, prove that Similarly


a+b+c 1 1 1 4 D £ c2 + a2
£ 2 + 2 + 2
abc a b c
4 D £ a2 + b2
Proof: We have
Therefore
b c + c a ³ 2 b c × c a = 2abc
2 2 2 2 2 2 2 2 2
4 D £ min{b2 + c2 , c2 + a2 , a2 + b2 }
c2 a2 + a2 b2 ³ 2bca2
46. In DABC, prove that
a2 b2 + b2 c2 ³ 2cab2
b2 c2 + c2 a2 + a2 b2 ³ 16 D2
Therefore
Proof: We have
2(b2 c2 + c2 a2 + a2 b2 ) ³ 2abc(a + b + c)
16 D2 = (2 s)2( s - a)2( s - b)2( s - c)
b2 c2 + c2 a2 + a2 b2 ³ abc(a + b + c)
= (a + b + c)(b + c - a)(c + a - b)(a + b - c)
Dividing both sides with a2b2c2, we have
= [(b + c)2 - a2 ][a2 - (b - c)2 ]
1 1 1 a+b+c
+ 2 + 2 ³
a2
b c abc = a2 [(b + c)2 + (b - c)2 ] - a4 - (b2 - c2 )2
which is the required one. = 2(a2 b2 + b2 c2 + c2 a2 ) - a4 - b4 - c4

44. In DABC, prove that 1


= a2 b2 + b2 c2 + c2 a2 - [(a2 - b2 )2 + (b2 - c2 )2
2
48( s - a)( s - b)( s - c) £ bc(b + c)
+ (c2 - a2 )2 ]
+ ca(c + a) + ab(a + b)
£ a2 b2 + b2 c2 + c2 a2
where 2s = a + b + c.
Proof: We have 47. In DABC, prove that

bc(b + c) + ca(c + a) + ab(a + b) bc + ca + ab ³ (4 3 )D

= b(c2 + a2 ) + c(a2 + b2 ) + a(b2 + c2 ) Proof: We have


³ b(2ca) + c(2ab) + a(2bc) 1 1 1
D= bc sin A = ca sin B = ab sin C
2 2 2

bc(b + c) + ca(c + a) + ab(a + b) ³ 6abc (8.16) This implies

But by part (ii) of Problem 33 æ 1 1 1 ö


bc + ca + ab = 2 D ç + +
è sin A sin B sin C ÷ø
abc ³ (b + c - a)(c + a - b)(a + b + c)
1/ 3
= 8( s - a)( s - b)( s - c) (8.17) é 1 1 1 ù
³ (2 D)(3) ê × × ú

ë sin A sin B sinC
Therefore from Eqs. (8.16) and (8.17)
1/ 3
æ 8 ö
48( s - a)( s - b)( s - c) £ 6abc £ bc(b + c) ³ (2 D)(3) ç
è 3 3 ÷ø
+ ca(c + a) + ab(a + b).
æ 3 3ö
ç∵ sin A sin B sin C £ 8 ÷
45. In DABC, prove that è ø

4 D £ min{b2 + c2 , c2 + a2 , a2 + b2 } æ 2 ö
= (2 D)(3) ç
è 3 ÷ø
Proof: We have
= 4 3D
4 D = 2bc sin A £ 2bc £ b2 + c2
Worked-Out Problems 585

48. In DABC, prove that Proof: Let x = s - a, y = s - b, z = s - c where 2s = a + b


+ c, so that
1 1 1
+ + y+z=a
b+c-a c+a-b a+b-c
z+ x=b
ì 9 1 1 1ü
³ max í , + + ý x+ y=c
îa + b + c a b c þ
Now
Proof: Let x = b + c - a, y = c + a - b and z = a + b - c
so that x, y, z are positive and R æ abc ö æ s ö
=ç ÷ç ÷
r è 4D ø è D ø
x+ y+z=a+b+c
abc
By Problem 2, =
4( s - a)( s - b)( s - c)
æ 1 1 1ö ( y + z)(z + x)( x + y)
( x + y + z) ç + + ÷ ³ 32 =
è x y zø 4 xyz
y(z + x)( x + y) + z(z + x)( x + y)
This implies =
4 xyz
1 1 1
+ + ³
9 (z + x)( x + y) (z + x)( x + y)
= + (8.20)
x y z x+ y+z 4zx 4 xy
Therefore Now
1 1 1 9 4zx £ (z + x)2
+ + ³ (8.18)
b+c-a c+a-b a+b-c a+b+c
and 4 xy £ ( x + y)2
Again,
This implies
1 1 2c
+ = 1 1
b + c - a c + a - b c2 - (a - b)2 ³
4zx (z + x)2
2c 2
³ = 1 1
c2 c and ³
4 xy ( x + y)2
Similarly
Therefore using Eq. (8.20) we have
1 1 2
+ ³ R (z + x)( x + y) (z + x)( x + y)
c+a-b a+b-c a ³ +
r (z + x)2 ( x + y)2
1
+
1
³
2 R x+ y z+ x c b
³ + = +
a+b-c b+c-a b r z+ x x+ y b c
Adding all the three in equalities we have 50. If a, b, c are positive real numbers, then prove that
1 1 1 1 1 1 1 1 1 1 æ 1 1 1ö
+ + ³ + + (8.19) + + £ ç + + ÷
b+c-a c+a-b a+b-c a b c a + b b + c c + a 2 è a b cø
From Eqs. (8.18) and (8.19), the result follows.
(Ireland Olympiads, 1998–99)
49. For DABC, prove that there exist positive real Proof: We have
numbers x, y, z such that y + z = a, z + x = b and
1 1 2
x + y = c and hence prove that + ³
a b ab
R b c
³ + 1 1 2
r c a + ³
b c bc
where r is the inradius and R is the circumradius. 1 1 2
+ ³
c a ca
586 Chapter 8 Inequalities

which imply that This means that Eq. (8.21) is true. Therefore

æ 1 1 1ö æ 1 1 1 ö a2 + b2 + c2 + 2abc < 2
2ç + + ÷ ³ 2ç + + ÷
è a b cø è bc ca ab ø
1 1 1 1 1 1 52. In DABC, prove that a2 + b2 + c2 ³ 4 3D.
+ + ³ + +
a b c bc ca ab Proof: We know that
2 2 2
³ + + a2 = b2 + c2 - 2bc cos A
b+c c+a a+b
Therefore
because
a2 + b2 + c2 = 2(b2 + c2 ) - 2bc cos A
a+b
³ ab
2
a2 + b2 + c2 - 4 3D
b+c
³ bc
2 = 2(b2 + c2 ) - 2bc cos A - 2 3 siin A
c+a
³ ca = 2(b2 + c2 ) - 2bc(cos A + 3 sin A)
2
æ pö
51. If a, b, c are the sides of a triangle and a + b + c = 2, = 2(b2 + c2 ) - 4bc cos ç A - ÷
è 3ø
then

a2 + b2 + c2 + 2abc < 2 æ æ pö ö
³ 2(b2 + c2 ) - 4bc ç∵ cos ç A - ÷ £ 1÷
è è 3ø ø
Proof: We have

4 = (a + b + c)2 = a2 + b2 + c2 + 2å ab = 2(b - c)2 ³ 0

Therefore So

a2 + b2 + c2 + 2abc = 4 - 2å ab + 2abc a2 + b2 + c2 ³ 4 3D
a2 + b2 + c2 + 2abc < 2 Equality holds, if the triangle is equilateral.
Û 4 - 2å ab + 2abc < 2 Note: In Problem 47, it is proved that bc + ca + ab ³ 4 3D
and hence Problem 52 follows from Problem 47, because
Û 2 - å ab + abc < 1 a2 + b2 + c2 ³ ab + bc + ca. In both cases equality holds if
the triangle is equilateral.
Û 1 - å ab + abc < 0 (8.21)
53. In DABC, if p1, p2 and p3 are the lengths of the alti-
Now
tudes from the vertices A, B and C on to the oppo-
2s = a + b + c = 2 Þ s = 1 site sides respectively, then prove that

Therefore 1 1 1 3
+ + ³
p1 - 2r p2 - 2r p3 - 2r r
D2 = s( s - a)( s - b)( s - c)
= (1 - a)(1 - b)(1 - c) r is the in radius of DABC.
= 1 + ab + bc + ca - (a + b + c) - abc Proof: It is known that
= 1 + ab + bc + ca - 2 - abc 1 ap
D= ( BC ) p1 = 1
= ab + bc + ca - 1 - abc 2 2

Therefore

ab + bc + ca - 1 - abc = D2 > 0 1 a
=
Þ 1 - ab - bc - ca + abc < 0 p1 2 D
Worked-Out Problems 587

From this we get r1, r2, r3 are the ex-radii opposite to the vertices A, B
and C, respectively, then prove that
1 1 1 a b c
+ + = + + r1 r r
p1 p2 p3 2 D 2 D 2 D + 2 + 3 ³3
p1 p2 p3
s 1
= =
D r Proof: We have
Now 1 2D
D= ap1 Þ p1 =
2r 2r 2r 2 a
- - - = -2
p1 p2 p3 Now
æ 2r ö æ 2r ö æ 2r ö 1 1 s-b s-c a 2
Þ ç1 - ÷ + ç1 - ÷ + ç1 - ÷ = 3 - 2 = 1 + = + = =
è p1 ø è p2 ø è p3 ø r2 r3 D D D p1

Therefore
p1 - 2r p2 - 2r p3 - 2r r1 r1 2r1
+ =
+ + =1 (8.22)
p1 p2 p3 r2 r3 p1

Also Similarly

2D 2D r2 r2 2r2
p1 - 2r = - + =
a s r3 r1 p2
æ 1 1ö r3 r3 2r3
= 2D ç - ÷ + =
è a sø r1 r2 p3
æ 2 2ö Adding all the three equalities, we have
= Dç - ÷
è a sø
ær r r ö r r r r r r
D (b + c - a ) 2ç 1 + 2 + 3 ÷ = 1 + 1 + 2 + 2 + 3 + 3
= >0 è p1 p2 p3 ø r2 r3 r3 r1 r1 r2
as
³6
By Problem 2, we have
(using AM ³ GM for the six numbers on the RHS.)
æ p1 - 2r p2 - 2r p3 - 2r ö Therefore
çè p + +
p2 p3 ÷ø
1
r1 r r
+ 2 + 3 ³3
æ p1 p2 p3 ö p1 p2 p3
çè p - 2r + p - 2r + p - 2r ÷ø ³ 3
2

1 2 3
55. In DABC, prove that
So from Eq. (8.22), we have
æ cos2 B + cos2 C ö r
p1
+
p2
+
p3
³9 å çè cos B + cos C ÷ø ³ 1 + R
p1 - 2r p2 - 2r p3 - 2r
æ p1 ö æ p2 ö æ p3 ö Proof: First, we prove that, if x, y and z are real
çè p - 2r - 1÷ø + çè p - 2r - 1÷ø + çè p - 2r - 1÷ø ³ 6 numbers such that the sum of any two is positive, then
1 2 3

2r 2r 2r x2 + y2 y2 + z2 z2 + x2
+ + ³6 + + ³x+ y+z
p1 - 2r p2 - 2r p3 - 2r x+y y+z z+ x

1 1 1 3
+ + ³
p1 - 2r p2 - 2r p3 - 2r r x2 + y2 ³ 2 xy Þ 2( x2 + y2 ) ³ ( x + y)2

54. In DABC, if p1, p2, p3 are the lengths of the altitudes x2 + y2 x + y


Þ ³
drawn from the vertices onto the opposite sides and x+ y 2
588 Chapter 8 Inequalities

Therefore 57. Consider Figure 8.2. P is a point interior to DABC.


The lines AP, BP
P and CP P meet the opposite sides in
x2 + y2 x + y y + z z + x
å x+y ³ 2 + 2 + 2 D, E and F respectively. Prove that
AP BP CP
+ + ³6
=x+ y+z PD PE PF
To prove the given problem, take x = cos A, y = cos B
A
and z = cos C so that
x + y = cos A + cos B

A+ B æ A - Bö
= 2 cos cos ç
2 è 2 ÷ø E

>0 y

F
Therefore x + y, y + z and z + x are positive. So P

æ x2 + y2 ö x
å çè x + y ÷ø
³x+ y+z
B
D

æ cos A + cos B ö
2 2
C
Þ åç ³ cos A + cos B + cos C
è cos A + cos B ÷ø FIGURE 8.2 Worked-out problem 57.

r Proof: Let D1 = area of DPBC, D2 = area of DPCA,


=1+
R D3 = area of DPAB and D = area of DABC. Let the
length of the altitude drawn from the vertex A on to BC
56. In DABC, prove that be y and the length of the altitude drawn from P onto
1/ 4
BC be x. Now since x and y are parallel we have
æ 3ö
sin A + sin B + sin C £ 3 ç ÷ D y AD
è 4ø = =
D1 x PD
Proof: Let x, y, z be positive real numbers. Then
Therefore
( x + y + z)2 = x2 + y2 + z2 + 2 xy + 2 yz + 2zx
D1 + D2 + D3 AD
=
£ ( x2 + y2 + z2 ) + ( x2 + y2 ) + ( y2 + z2 ) D1 PD

+ (z2 + x2 ) D2 D3 AD AP
+ = -1=
D1 D1 PD PD
= 3( x2 + y2 + z2 )
Similarly
This implies
D3 D1 BP
+ =
x + y + z £ 3( x + y + z )
2 2 2
D2 D2 PE
D1 D2 CP
Put x = sin A , y = sin B , z = sin C . Therefore + =
D3 D3 PF
sin A + sin B + sin C £ 3(sin A + sin B + sin C ) Adding all we have

æ 3 3ö AP BP CP æD D ö
£ 3ç ÷ (see Problem 37) + + = åç 2 + 3 ÷
è 2 ø PD PE PF è D1 D1 ø
1/ 6
1/ 4 éD D D D D D ù
æ 3ö ³ 6ê 2 × 3 × 2 × 1 × 3 × 1 ú
= 3ç ÷
è 4ø ë D1 D1 D 3 D 3 D 2 D 2 û
=6
Worked-Out Problems 589

58. In DABC, prove that Proof: By hypothesis,

æ sin A ö xy + 1 yz + 1 zx + 1
= =
å çè ÷ ³3
sin B + sin C - sin A ø
y z x

Proof: Since a = 2R sin A, b = 2R sin B and c = 2R sin C,


we have to prove that xyz + z = y2 z + y

æ a ö xyz + x = z2 x + z
å çè b + c - a ÷ø ³ 3 x2 y + x = xyz + y

Let Now

x= b+ c- a xyz + z = y2 z + y
Þ yz( x - y) = y - z
y= c + a - b
Þ xyz( x - y) = x( y - z) (8.23)
z= a + b - c
Similarly
Since
xyz( y - z) = y(z - x) (8.24)
( b + c )2 = b + c + 2 bc
xyz(z - x) = z( x - y) (8.25)
>b+c>a
On multiplying Eqs. (8.23), (8.24) and (8.25) and cancel-
This implies ling xyz(x - y) (y - z) (z - x) we have (xyz)2 = 1 which
b + c - a >0 implies that xyz = 1. Therefore

x + y + z > 3( xyz)1/ 3 = 3
So x, y, z are positive. Now y + z = 2 a , z + x = 2 b and
x + y = 2 c imply that 1 1 1
and + + >3
x y z
a y+z
= Here equality cannot occur, because x, y, z are distinct.
b+ c- a 2x

b z+ x 60. If a, b and c are positive rational numbers such that


=
c+ a- b 2y the sum of any two is greater than the third, then
prove that
c x+y
= æ
a
b - cö æ
b
c - aö æ a - bö
c
c+ a- b 2z
çè 1 + ÷ ç1 + ÷ ç1 + ÷ £1
a ø è b ø è c ø

Equality occurs if and only if a = b = c.


1/ 6
æ a ö æ z x y z x yö
2å ç ÷ ³ 6 çè y × y × x × x × z × z ÷ø Proof: We can suppose, a = x/m, b = y/m, c = z/m where
è b + c - aø x, y, z and m are positive integers. Also note that b + c - a,
c + a - b and a + b - c are positive and
æ a ö
å çè ÷ ³3
b + c - aø x + y + z = m(a + b + c)

Consider
59. If x, y, z are distinct positive real numbers and
b-c
1 1 1 1+ ( x times)
x+ = y+ =z+ a
y z x
c-a
1+ ( y times)
then prove that b
1 1 1 a-b
x + y + z > 3 and + + >3 1+ (z times)
x y z c
590 Chapter 8 Inequalities

and use AM ³ GM. Therefore By Eq. (8.26) we have

æ b - cö æ c - aö æ a - bö æ 1ö æ 1ö 1 1 1
xç1+
è ÷ + y çè 1 + ÷ + z çè 1 + ÷ çè 1 + ÷ø ç 1 + ÷ = 1 + + +
a ø b ø c ø x è yø x y xy
x+ y+z
2 1
z 1/( x + y + z) ³1+ +
éæ b - cö æ
x
c - aö æ
y
a - bö ù xy xy
³ êç 1 + ÷ø çè 1 + ÷ø çè 1 + ÷ ú
êë è a b c ø úû
³ 1 + 2(2) + 4
( x + y + z) + m[(b - c) + (c - a) + (a - b)]
x+ y+z Therefore

éæ
1 / m( a + b + c )
æ 1ö æ 1ö
a - bö ù
x y z
b - cö æ c - aö æ çè 1 + ÷ø ç 1 + ÷ ³ 9
³ êç 1 + ÷ ç 1 + ÷ ç 1 + ÷ ú x è yø
êëè a ø è b ø è c ø úû
1 /( a + b + c )
éæ x/m
b - cö æ
y/m
c - aö æ a - bö ù
z/ m
62. In DABC, prove that
1 ³ êç 1 + ÷ ç 1+ ÷ ç 1+ ÷ ú
êëè a ø è b ø è c ø úû a b c
+ + ³3
1 /( a + b + c ) c+a-b a+b-c b+c-a
éæ a
b - cö æ
b
c - aö æ a - bö ù
c

1 ³ êç 1 + ÷ ç 1 + ÷ ç 1 + ÷ ú
êëè a ø è b ø è c ø úû Equality holds when the triangle is equilateral.
Proof: Using AM ³ GM we get
Therefore
a b c
æ
a
b - cö æ c - aö æ
b
a - bö
c
+ +
çè 1 + ÷ø çè 1 + ÷ø çè 1 + ÷ £ (1)
a + b+ c
=1 c+a-b a+b-c b+c-a
a b c ø
1/ 3
é a b c ù
³ 3ê × × ú
61. If x and y are positive real numbers such that ëc + a - b a + b - c b + c - aû
x + y = 1, then show that
But, from Problem 33, part (ii), we have that
æ 1ö æ 1ö
çè 1 + ÷ø ç 1 + ÷ ³ 9 abc ³ (b + c - a)(c + a - b)(a + b - c)
x è yø
Therefore
Equality occurs when x = y = 1/ 2.
Proof: We have a b c
+ + ³3
c+a-b a+b-c b+c-a
x+y 1
³ xy Þ ³ xy (8.26)
2 2

EXERCISES
Subjective Questions Only
1. If a, b, c are positive real numbers and a + b + c = 1, 3. If n4 < 10n for a positive integer n ³ 2, then show that
then prove that (n + 1)4 < 10n+1.

æ1 öæ1 öæ1 ö 8 4. If x, y are positive such that 3x + 4y


4 = 5, then show that
8 £ ç - 1÷ ç - 1÷ ç - 1÷ £
èa øèb øèc ø 27abc
3
x2 y3 £
Equality holds when a = b = c = 1/ 3. 16

2. If a, b, c are positive real numbers, then prove that and equality holds if and only if x = 2 / 3, y = 3 / 4.

2(a3 + b3 + c3 ) ³ ab(a + b) + bc(b + c) + ca(c + a) 5. If x, y, z are positive such that x + y + z = 18, then
prove that the maximum value of x2y3z4 is 426384.
³ 6abc
Exercises 591

6. If a, b, c, e, f,
f g are positive, then show that 14. If A, B, C are independent events with positive
probabilities of a random experiment, then prove
æ a b cö æ e f gö that
çè e + f + g ÷ø çè a + b + c ÷ø ³ 9
P( A) + P( B) + P(C ) ³ 3( P( A Ç B Ç C ))1/ 3
7. If a, b, c are positive, then prove that When will the equality hold?
æ 1 1 1ö
(a + b + c ) ç + + ÷ ³ 9 15. Let A, B, C be mutually exclusive and exhaustive
è a b cø events with non-zero probabilities of a random
experiment, and E is any event of the experiment
When does the equality occur?
with P(E) > 0, then prove that
8. If a, b are positive, then prove that 2 lies between P(E) ³ 3( P( A)P( B)P(C ))1/ 3
a/b and (a + 2b)/(a + b) .
´ [ P(E / A)P(E / B)P(E /C )]1/ 3
9. If 0 < x £ p / 2, then show that the minimum value of
16. If a1, a2, …, an are positive real numbers such that
9 x2 sin2 x + 4
each ai < 1 and a1 + a2 +  + an = sn ¹ 1, then prove
x sin x that
is 12. 1
(1 + a1 )(1 + a2 )  (1 + an ) <
1 - sn
10. In DABC, prove that the inradius

a2 + b2 + c2 17. If n > 1 integer, then prove that n > nn / 2 .


r<
3(a + b + c)
18. If a, b, c are positive real numbers, then prove that
11. If a, b, c are the sides of DABC, then prove that (a + b + c)(bc + ca + ab) ³ 9abc
| a (b - c) + b (c - a) + c (a - b)| < abc
2 2 2

19. If n > 1 integer, then prove that n > 1·3·5(2n - 1).


n

(Hint: Factorize the expression and use that the


difference of two sides is less than the third side.) *20. In many books, one finds the following problem:
If a1, a2, …, an are positive and (n - 1) s = a1 + a2 +
12. If q1, q2, …, qn are real, then show that  + an, then show that the product

æ n
2
ö æ n
2
ö a1 × a2 × a3  an > (n - 1)n ( s - a1 )( s - a2 )  ( s - an )
çè å cos q i ÷ø + çè å sin qi ÷ø £ n
2

i =1 i =1
This result is wrong, if we consider a1 = 1, a2 = 2
and a3 = a4 = 6. Find the fallacy in the argument (or
13. If a1, a2, …, an are positive real numbers such that proof) given by the authors.
a1 + a2 +  + an = s then prove that

s s s s n2
+ + ++ ³
s - a1 s - a2 s - a3 s - an n - 1

where n ³ 2
Index
A Conditional probability 499 Events 490
Construction of least positive angle 142 Excentres 216
sine is equal to a 142 Excentric triangle 224
Addition of vectors 300
Construction of least positive angle 143 Excircles 216
Addition theorem on probability 497
cosine is equal to a 143 Exradius 216
Additive identity 304
Construction of least positive angle 143
Additive inverse 304
tangent is a given real number a 143
Altitudes of a triangle 377
Angle 2
Continuous random variable 506 F
Coplanar vectors 300, 320, 370
Angle in a semicircle 380
Cosecant 9,, 15 Failure 508
Angles and their measures 2
Cosine 15 Finite set 494
Arcsin x 132
Cosine formula 188 Fundamental identities 133
Area of a triangle 209
Cosine ratio 7
Arithmetic mean 570
Cosine ratios 7
Associative property for addition 302
Cotangent 9,, 15
Associativity of addition 302
Coterminal 18 G
Atmost countable set 494
Coterminal angles 18
Axiomatic approach to probability 494 General solution of a sin x +
Countably infinite set 494
Cross product 387 b cos x = c 148
General solution of equations 144
B cos x = 0 144
cos x = a 146
Basis vector triad 324
D sin x = a 144
Bayes’ theorem 503 sin x = 0 144
Bijection 100 Decreasing function 21 sin2 x = sin2 q 147
Binomial distribution 508 Decreasing on 21 tan x = 0 144
Binomial variate 509 Definition 132 tan x = a 146
Box product 404 Degrees 2 Geometric mean 570
Dependent events 499 Graph of 3 sin 2x 25
Directed line segment 298 Graph of cos(x - p/4)
p 25
Direction cosines 327 Graph of cosec x 24
C Direction ratios 329 Graph of cos x 22
Discrete random variable 506 Graph of cot x 23
Cauchy’s inequality 571 Domain of inverse trigonometric Graph of sec x 23
Cauchy–Schwartz inequality 374 functions 100 Graph of sin x 21
Centroid 225 Dot product 366 Graph of tan x 22
Certain 494
Graph of y = cos 2x 26
Circular measure system 2
Graph of y = cos x 26
Circumcentre 210
Circumcircle 210 E
Circumradius 210
Classical definition of probability 491 Elementary event 494 H
Collinear vectors 299 Elementary events 494
Commutative property for Elements 186 Harmonic mean 570
addition 302 Empirical definition of probability 493
Commutativity of addition 302 Equation of a plane in normal form 382
Equivalence 299
Complementary 14
Equivalent directed line segments 299
I
Complementary angles 14
Complementary event 494 Escentre 216
Impossible event 494
Completeness axiom 495 Escribed circle 216
Incentre 212
Component 367 Esradius 216
Incircle 212
Components of a vector 321 Event 490,, 494
Increasing function 21
Compound angle 30 equally likely 490,, 494
Increasing on 21
Concurrent 222 equiprobable 490,, 494
Independent events 499
Concyclic 223 exhaustive 490
Independent vectors 319
Conditional event 499 mutually exclusive 490,, 494
Inequalities 569
594 Index

Injection 100 Orthocentre 222 Resultant vector 301


Inradius 212 Orthogonal projection 367 Right-handed orthonormal vector
Inscribed circle 212 Orthogonal unit vectors 373 basis 327
Inverse circular function 101 Right-handed system 325
Inverse sine function 100 Root 142
Inverse trigonometric P
functions 100, 102
domain 100 Parallelogram law for addition 301 S
graphs of inverse trigonometric Parallel vector 372
functions 101 Parameters 508 Same direction 299
properties 102 Pedal triangle 222 Sample points 494
range 100 Period 19 Sample space 494
Periodic 19, 20 Scalar component 367
Periodicity 19 Scalar product 366
L Perpendicular bisectors 378 Scalar quantities 298
Perpendicular vectors 324 Scalar triple product 404
Law of cosines 376 Point of concurrence 222 Secant 9,, 15
Least positive angles 142 Poisson distribution 509 Seconds 2
Left-handed system 325 Poisson distributions 508 Sexagesimal system 2
Like vectors 299,, 372 Poisson variate 509 Shortest distance 413
Limitations of the classical Position vector 305 Simple event 494
definition 493 Positive axiom 495 Sine 15
Linear combination 319 Positive directions 325 Sine formula 186
Linearly dependent vectors 319 Principal solutions 142 Sine function 100
Principal value 100 Sine ratio 7
Probability 490 Sine ratios 7
M Probability distribution 507 Skew lines 412
Probability distribution function 506 Solution 142
Magnitude 298 Probability distributions 504 Solution of a triangle 203,, 204, 205, 207
Magnitude of the vector area 398 Probability function 495 all the angles given 207
Maximum value 41 Probability of occurance 491 one side and two angles given 207
Mean 507 Probability of the event 495 three sides given 203
Measure of the angle 2 Product transformation 47 two sides and an angle opposite to
Median 315 Projection 367 one of them given 205
Medians 225 Projection formula 376 two sides and the angle between
Minimum period 20 Projection formulae 191 them given 204
Minimum value 41 Properties of a triangle 209 Solution of the triangle 186
Minutes 2 Solution of triangles 201
Mollweide rules 199 Solutions of right-angled triangles 201
Multiple angle 36 Q Solutions of simple trigonometric
Multiplication of a vector by a scalar 309 equations 148
Multiplication of vectors 365 Quadrant angles 10 Solving a triangle 186
Multiplication theorem on conditional Standard deviation 507
probability 500 Statistical definition of probability 493
Submultiple angle 36
R Success 508
N Radian 2
Sum transformation 47
Supplementary 17
Radian system 2 Supplementary angles 17
Napier analogy 198 Random experiment 490,, 499 Support 299
Negative directions 325 Random experiments 490 Surjection 100
Nine-point circle 228 Randomness 490
Non-coplanar vectors 300, 321 Random variables 504
Range of inverse trigonometric
functions 100
T
O Relationship among trigonometric
ratios 13 Tangent 9,, 15
Opposite direction 299 Relative frequency 493 Tangent rules 198
Orientation of closed curve 398 Relative frequency definition of Theoretical discrete distribution 508
Oriented closed curve 398 probability 493 Total probability 503
Index 595

Triad 373 U Vector areas 398


Trial 508 Vector component 367
Triangle law for addition 301 Vector equation of a line 333
Union axiom 495
Triangular inequality 306 Vector equation of a plane 333
Unit vector 299,, 312, 371
Trignometric functions 18 Vector equations 382
Unlike vectors 299
Trigonometric equations 142 plane 382
Trigonometric equations 148 sphere 382
Trigonometric function–Domain 21 Vector multiplication 365
Trigonometric function–Range 21 V Vector product 387
Trigonometric ratio 142 Vector quantities 298
Trigonometric ratios 7 Variance 19, 507 Vector triple product 415
Trigonometric ratios of angles of a Variance of cosec x 24
triangle 186 Variance of cos x 22
Variance of cot x 23
Trigonometric ratios–compound
Variance of sec x 23
Z
angles 30
Trigonometric ratios–multiple angle 36 Variance of sin x 21
Zero vector 299,, 304
Trigonometric ratios–submultiple Variance of tan x 22
angle 36 Vector 299
Trigonometry 2 Vector addition 300
Trisecting points 314 commutative property 302

You might also like